Download as pdf or txt
Download as pdf or txt
You are on page 1of 459

Comprehensive Trigonometry

13 February 2017 02:12:47 PM


ABOUT THE AUTHOR
REJAUL MAKSHUD (R M) Post graduated from Calcutta University in PURE MATHEMATICS having teaching
experience of 15+ years in many prestigious institution of India. Presently, he trains IIT Aspirants at RACE IIT
ACADEMY, Jamshedpur, playing a role of DIRECTOR cum HOD OF MATHEMATICS.
Rejaul Makshud

McGraw Hill Education (India) Private Limited


CHENNAI

Chennai New York St Louis San Francisco Auckland Bogotá Caracas


Kuala Lumpur Lisbon London Madrid Mexico City Milan Montreal
San Juan Santiago Singapore Sydney Tokyo Toronto
Published by McGraw Hill Education (India) Private Limited,
444/1, Sri Ekambara Naicker Industrial Estate, Alapakkam, Porur, Chennai-600116
Comprehensive Trigonometry with Challenging Problems & Solutions for Jee Main and Advanced
Copyright © 2017, McGraw Hill Education (India) Private Limited.
No part of this publication may be reproduced or distributed in any form or by any means, electronic, mechanical, photocopying,
recording, or otherwise or stored in a database or retrieval system without the prior written permission of the publishers. The program
listings (if any) may be entered, stored and executed in a computer system, but they may not be reproduced for publication.
This edition can be exported from India only by the publishers,
McGraw Hill Education (India) Private Limited.
ISBN (13): 978-93-5260-510-1
ISBN (10): 93-5260-510-1

Information contained in this work has been obtained by McGraw Hill Education (India), from sources believed to be reliable.
However, neither McGraw Hill Education (India) nor its authors guarantee the accuracy or completeness of any information
published herein, and neither McGraw Hill Education (India) nor its authors shall be responsible for any errors, omissions, or
damages arising out of use of this information. This work is published with the understanding that McGraw Hill Education
(India) and its authors are supplying information but are not attempting to render engineering or other professional services. If
such services are required, the assistance of an appropriate professional should be sought.

Typeset at Script Makers, 19, A1-B, DDA Market, Paschim Vihar, New Delhi, India, and text and cover printed at

Cover Designer: Creative Designer

visit us at: www.mheducation.co.in


PREFACE

This text book on TRIGONOMETRY with Problems & Solutions for JEE Main and Advanced is meant for aspirants preparing
for the entrance examination of different technical institutions, especially NIT/IIT/BITSAT/IISc. In writing this book I
have drawn heavily from my long teaching experience at National Level Institutes. After many years of teaching I have
realised the need of designing a book that will help the readers to build their base, improve their level of mathematical
concepts and enjoy the subject.
This book is designed keeping in view the new pattern of questions asked in JEE Main and Advanced Exams. It has
eight chapters. Each chapter has a large number of worked out problems and exercise based problems as given below:
Level – I: Questions based on Fundamentals
Level – II: Mixed Problems (Objective Type Questions)
Level – III: Problems for JEE Advanced Exam
(0.......9): Integer type Questions
Passages: Comprehensive link passages
Matching: Match Matrix
Reasoning: Assertion and Reasoning
Previous years papers: Questions asked in past IIT-JEE Exams
become easy.
So please don’t jump to exercise problems before you go through the Concept Booster and the objectives. Once you are
arranged in a manner that they gradually
require advanced thinking.
tackle any type
of problem easily and skilfully.
My special thanks goes to Mr. M.P. Singh (IISc. Bangalore), Mr. Manoj Kumar (IIT, Delhi), Mr. Nazre Hussain (B.
Tech.), Dr. Syed Kashan Ali (MBBS) and Mr. Shahid Iqbal, who have helped, inspired and motivated me to accomplish
this task. As a matter of fact, teaching being the best learning process, I must thank all my students who inspired me most
for writing this book.
I would like to convey my affectionate thanks to my wife, who helped me immensely and my children who bore with
patience my neglect during the period I remained devoted to this book.
I also convey my sincere thanks to Mr Biswajit of McGraw Hill Education for publishing this book in such a beautiful
format.
and to
all my learned teachers— Mr. Swapan Halder, Mr. Jadunandan Mishra, Mr. Mahadev Roy and Mr. Dilip Bhattacharya,
who instilled the value of quality teaching in me.
I have tried my best to keep this book error-free. I shall be grateful to the readers for their constructive suggestions
toward the improvement of the book.

REJAUL MAKSHUD
M. Sc. (Calcutta University, Kolkata)
Dedicated to
My Beloved Mom and Dad
Contents

Preface v
1. The Ratios and Identities 1-99
1.1 Introduction 1
1.2 Application of Trigonometry 1
1.3 Trigonometrical Functions 1
1.4 Measurement of Angles 2
1.5 Some Solved Examples 2
Exercise 1 4
1.6 Trigonometrical Ratios 5
1.7 Limits of the Values of Trigonometrical Functions 5
1.8 Some Solved Examples 5
Exercise 2 10
1.9 Measurement of the Angles of Different T-ratios 11
1.10 Some Solved Examples 13
Exercise 3 15
Exercise 4 17
1.11 T-ratios of Compound Angles 17
1.12 Some Important Deductions 19
1.13 Some Solved Examples 20
Exercise 5 22
1.14 Transformation Formulae 23
Exercise 6 25
1.15 Multiple Angles 26
1.16 Some Important Deductions 27
Exercise 7 33
1.17 The Maximum and Minimum Values of
f (x) = a cos x + b sin x + c 34
Exercise 8 36
1.18 Sub–Multiple Angles 37
1.19 Some Solved Examples 40
Exercise 9 43
1.20 Conditional Trigonometrical Identities 43
1.21 Some Solved Examples 44
Exercise 10 47
1.22 Trigonometrical Series 47
1.23 Different Types of the Summation of a Trigonometrical Series 48
Exercise 11 48
Exercise 12 49
Exercise 13 50
Problems for JEE Advanced Exam 51
Level I (Problems Based on Fundamentals) 65
Level II (Mixed Problems) 67
Level III (Tougher Problems for JEE Advanced) 69
Integer Type Questions 70
Link Comprehension Type (For JEE Advanced Exam Only) 71
Match Matrix 72
Assertion & Reason 74
(Questions Asked in Past IIT-JEE Exams) 74
Answers 77
Hints and Solutions 79
viii Contents

Integer Type Questions 89


Questions asked in IIT-JEE Exams 91
2. Graphs of Trigonometric Functions 100-118
2.1 Introduction 100
2.2 Characteristics of co-sine Function 109
2.3 Characteristics of Tangent Function 111
2.4 Characteristics of co-tangent Function 112
2.5 Characteristics of co-secant Function 112
2.6 Characteristics of Secant Function: 113
Level I (Questions Based on Fundamentals) 115
Level II (For JEE Main Exam Only) 116
Level III (For JEE Advanced Exam Only ) 117
Answers 118
3. The Trigonometric Equation 119-183

3.2 Solution of a Trigonometric Equation 119


3.3 General solution of Trigonometric Equations 119
3.4 Ranges of Trigonometric Functions 119
3.5 Some Solved Examples 120
Exercise 1 121
3.6 A Trigonometric Equation is of the Form 122
Exercise 2 123
3.7 Principal Value 123
3.8 Method to Find Out the Principal Value 123
Exercise 3 124
3.9 Solutions in Case of Two Equations are Given: 124
Exercise 4 127
3.10 Some Important Remarks to Keep in
Mind While Solving a Trigonometric Equation 128
3.11 Types of Trigonometric Equations 128
Exercise 5 128
Exercise 6 129
Exercise 7 129
Exercise 8 130
Exercise 9 130
Exercise 10 131
Exercise 11 131
Exercise 12 132
Exercise 13 132
Exercise 14 133
Level I (Questions Based on Fundamentals) 140
Level II (Mixed Problesms) 141
Level III (Problems for JEE Advanced Exam) 144
Integer Type Questions 145
Linked Comprehension Type (For JEE Advanced Exams Only) 146
Match Matrix 147
Assertion and Reason 149
Questions Asked in Past IIT-JEE exams 150
Answers 151
Hints and Solutions 156
Level III 170
Integer Type Questions 175
Past IIT-JEE Questions 177
4. Trigonometric In-Equation 184-195
4.1 Trigonometric Inequalities 184
Type - I: An inequation is of the form sin x > k. 184
Contents ix

Type - II: An in-equation is of the form sin x < k. 185


Type - III: An in-equation is of the form cos x > k. 185
Type - IV: An in-equation is of the form cos x < k. 186
Type - V: An in-equation is of the form tan x > k. 186
Type -VI: An inequation is of the form tan x < k. 187
4.2 Some Solved Examples 187
Comprehensive Link Passage 192
Answers 194
5. Logarithm 196-231
5.1 Introduction 196
5.2 Some Solved Examples 198
5.3 Logarithmic Equation 201
5.4 Logarithmic Inequation 204
Problems for JEE Advanced Exam 207
Comprehensive Link Passages 218
Match Matrix 219
Integer Type Questions 219
Questions Asked in Past IIT-JEE Exams 220
Answers 220
Hints and Solutions 222
Integer Type Questions 226
Questions Asked In Past IIT-JEE Exams 228
6. Inverse Trigonometric Function 232-329
6.1 Introduction to Inverse Function 232
6.2 Some Solved Examples 233
Exercise 1 235
6.3 Inverse Trigonometric Functions 235
6.4 Graphs of Inverse Trigonometric Functions 236
Characteristics of arc sine function 236
Characteristics of arc cosine function: 236
Characteristics of arc tangent function 236
Characteristics of arc co-tangent function: 237
Characteristics of arc co-secant function: 237
Characteristics of arc secant function: 237
Exercise 2 240
6.5 Constant Property 241
Exercise 3 242
6.5 Conversion of Inverse Trigonometric Functions 243
Exercise 4 245
6.6 Composition of Trigonometric functions and its Inverse 245
6.7 Composition of Inverse Trigono Metric Functions and Trigonometric Functions 247
Exercise 5 251
6.9 Sum of Angles 252
Exercise 6 260
6.10 Multiple Angles 261
Exercise 7 263
6.11 More Multiple Angles 263
Exercise 8 265
Problems for JEE Main Exam 266
Questions with Solutions of Past JEE Main Exams 273
Problems for JEE Advanced Exam 275
Level II (Mixed Problems) 289
Level III (Problems for JEE Advanced) 292
Integer Type Questions 294
Comprehensive link passage 295
Match-Matrix 296
x Contents

Assertion and Reason 297


Questions Asked in Past IIT-JEE Exams 298
Answers 300
Hints and Solution 304
Integer Type Questions 316
Question Asked in Part IIT-JEE Exams 319
Questions with Solutions of Past IIT-JEE Exams from 1981 to 2015 323
7. Properties of Triangles 330-440
7.1 Introduction 330
Exercise 1 333
7.2 Cosine Rule 333
Exercise 2 337
7.3 Projection Formulae 337
Exercise 3 338
7.4 Napier’s Analogy (Law of Tangents) 338
Exercise 4 339
7.5 Half-Angled Formulae 340
Exercise 5 344
7.6 Area of triangle 344
Exercise 6 349
7.6 Radii of Circle Connected with a Triangle 349
Exercise 7 351
7.8 Inscribed Circle and its Radius 352
Exercise 8 355
7.9 Escribed Circle of a Triangle and their Radii 356
Exercise 9 360
7.10 Regular Polygon 361
Exercise 12 365
7.11 Orthocentre and Pedal Triangle of any Triangle 366
7.12 Distance between the Circumcentre and Orthocentre 367
7.13 Distance between the circumcentre and the Incentre 368
7.14 Distance between the Circumcentre and Centroid 368
7.15 Distance between the Incentre and Orthocentre 368
7.16 Excentral Triangle 369
7.17 Quadrilateral 372
Problems for JEE Advanced Exam 375
Level I (Questions Based on Fundamentals) 396
Level II Mixed Problems 398
Level III (For JEE Advanced Exam Only) 401
Integer Type Questions 403
(Questions Asked in IIT-JEE Exams with their Solutions) 404
Comprehensive Link Passage (For JEE Advanced Exam Only) 408
Match Matrix (For JEE Advanced Exam Only) 410
Assertion and Reason 412
Answers 413
Level III (Problems for JEE Advanced) 414
Integer Type Questions 424
Hints & Solutions of Past IIT-JEE Questions 426
8. The Heights and Distances 441-448
8.1 Introduction 441
8.2 Angle of Elevation, Angle of Depression and the Line of Sight 441
8.3 Bearing of a Line 441
8.4 Some Solved Examples 442
Level I (Problems Based on Fundamentals) 446
Level II (Mixed Problems) 447
Level III (Problems for JEE Main) 447
Answers 448
CHAPTER 1
The Ratios and Identities

1.1 INTRODUCTION cryptology), seismology, meteorology, oceanography, many


physical sciences, land surveying and geodesy, architecture,
Trigonometry (from Greek trigonon ‘triangle’ + metron phonetics, economics, electrical engineering, mechanical
“measure”) is a branch of mathematics that study triangles engineering, civil engineering, computer graphics,
and the relationships between the lengths of their sides and cartography, crystallography and game development.
the angles between those sides.

describe those relationships that have applicability to cyclical


1.3 TRIGONOMETRICAL FUNCTIONS
In mathematics, the trigonometric functions (also called as
third century BC as a branch of geometry used extensively for the circular functions) are functions of an angle. They are
astronomical studies. It is also the foundation of the practical used to relate the angles of a triangle to the lengths of the
art of surveying. sides of a triangle. Trigonometric functions are important
Trigonometry basics are often taught in school either in the study of triangles and modeling periodic phenomena,
as a separate course or as a part of a precalculus course. among many other applications. The most familiar
The trigonometric functions are pervasive in parts of pure trigonometric functions are sine, cosine, and tangent. In
mathematics and applied mathematics such as Fourier the context of the standard unit circle with radius 1, where
analysis and the wave equation, which are in turn essential a triangle is formed by a ray originating at the origin and
to many branches of science and technology. making some angle with the x-axis, the sine of the angle
gives the length of the y-component (rise) of the triangle,
1.2 APPLICATION OF TRIGONOMETRY the cosine gives the length of the x-component (run), and the
tangent function gives the slope (y-component divided by
There are an enormous number of uses of trigonometry the x
and trigonometric functions. For instance, the technique of
triangulation is used in astronomy to measure the distance ratios of two sides of a right triangle containing the angle,
nearby stars, in geography to measure distances between
landmarks, and in satellite navigation systems. The sine and
cosine functions are fundamental to the theory of periodic
functions that describe sound and light waves. equations, allowing their extension to arbitrary positive and
The fields that use trigonometry or trigonometric negative values and even to complex numbers.
functions include astronomy (especially for locating Trigonometric functions have a wide range of uses
apparent positions of celestial objects, in which spherical including computing unknown lengths and angles in
trigonometry is essential) and hence navigation (on the triangles (often right triangles). In this case, trigonometric
oceans, in aircraft, and in space), music theory, acoustics, functions are used, for instance, in navigation, engineering,
and physics. A common use in elementary physics is
theory, statistics, biology, medical imaging (CAT scans and resolving a vector into Cartesian coordinates. The sine and
ultrasound), pharmacy, chemistry, number theory (and hence cosine functions are also commonly used to model periodic
2 Comprehensive Trigonometry with Challenging Problems & Solutions for Jee Main and Advanced

function phenomena, such as sound and light waves, the Here, –AOB = 1 radian = 1e.
position and velocity of harmonic oscillators, sunlight B
intensity and day length, and average temperature variations
throughout the year.
1e
In modern usage, there are six basic trigonometric O A
functions tabulated here with equations that relate them to
one another. Especially, with the last four, these relations are

Notes:
and then derive these relations. (i) When an angle is expressed in radians, the word
radian is omitted.
1.4 MEASUREMENT OF ANGLES (ii) Since 180∞ = p radian = ⎛⎜
22 ⎞

⎝ 7 × 180 ⎠
1. Angle: The measurement of an angle is the amount of
radian = 0.01746 radian
rotation from the initial side to the terminal side.
¥ 7ˆ˜
2. Sense of an Angle: The sense of an angle is +ve or –ve 180∞ Ê 180
(iii) 1 radian = =Á
according to the initial side that rotates in anti-clock- p Ë 22 ¯
wise or clockwise direction to get the terminal side. = 57∞ 16' 22'
B (iv) The angle between two consecutive
p
digits is 30∞ ÊÁ radiansˆ˜
Ë6 ¯
(v) The hour hand rotates through an angle of 30∞ in
q
1 hour (i.e., ÊÁ ˆ˜ in 1 minute).
A 1
O
+ ve angle Ë 2¯
O - ve angle (vi) The minute hand rotates through an angle of 6∞ in
C 1 minute.
q
(vii) The relation amongst three systems of measurement
of an angle is
D G 2R
D
= =
90∞ 100 p
3. System of measuring angles: (viii) The number of radians in an angle subtended by
There are three systems of measuring angles such as Arc
an arc of a circle at the centre is
(i) Sexagesimal system Radius
s
(ii) Centisimal system i.e., q =
(iii) Circular system r
In sexagesimal system, we have 1.5 SOME SOLVED EXAMPLES
1 right angle = 90∞
Ex-1. If the radius of the earth is 4900 km, what is the
1∞ = 60' length of its circumference?
1' = 60" Soln. Given r = 4900 km
In centasimal system, we have Circumference = 2p r
1 right angle = 100g 22
=2¥ ¥ 4900
g
1 = 100' 7
= 44 ¥ 700
1' = 100"
= 30,800 km
In circular system, the unit of measurement is radian.
Ex-2. The angles of a triangle are in the ratio 3 : 4 : 5.
Radian: One radian is the measure of an angle sub-
Find the smallest angle in degrees and the greatest
tended at the centre of a circle by an arc of length equal
angle in radians.
to the radius of the circle.
Soln. Let the three angles be 3x, 4x and 5x, respectively
Thus, 3x + 4x + 5x = 180∞
The Ratios and Identities 3

fi 12x = 180∞ Ex-5. The angles of a quadrilateral are in A.P. and the
fi x = 15∞ greatest is double the least. Express the least angles
Therefore, the smallest angle in radians.
Soln. Let the angles of the quadrilateral be
= 3x = 3 ¥ 15∞ = 45∞
a – 3d, a – d, a + d, a + 3d
and the greatest angle
It is given that, a + 3d = 2(a – 3d)
= 5x = 5 ¥ 15∞ = 75∞
fi a + 3d = 2a – 6d
p ˆ
= ÊÁ 75 ¥ ˜ radians fi a = 9d
Ë 180 ¯
Also, a + 3d + a – d + a + d + a + 3d = 360
5p
= ÊÁ ˆ˜ radians fi 4a = 360
Ë 12 ¯
fi a = 90
Ex-3. The angles of a triangle are in AP and the number
and d = 10
of degrees in the least is to the number of radians in
the greatest as 60 to p Hence, the smallest angle = 90∞ – 30∞
Soln. Let the three angles be a + d, a, a – d = 60∞
c
Thus, a + d + a + a – d = 180∞ Êpˆ
fi 3a = 180∞ = ÁË ˜¯ .
3
180∞ Ex-6. Find the angle between the hour hand and the
fi a= = 60∞
3 minute hand in circular measure at half past 4.
It is given that, 1
p 60 Soln. Clearly, at half past 4, hour hand will be at 4
(a – d)∞ : (a + d) ¥ = and minute hand will be at 6. 2
180 p
( a − d ) 180 60 In 1 hour angle made by the hour hand will be 30∞
fi × 1
(a + d ) π = p In 4 hours angle made by the hour hand
2
(a - d ) 1 9
fi = = ¥ 30∞ = 135∞
(a + d ) 3 2
fi a + d = 3a – 3d In 1 minute angle made by the minute hand = 6∞
fi 4d = 2a In 30∞ minutes, angle made by the minute
a hand = 6 ¥ 30∞ = 180∞
fi d= = 30∞ Thus, the angle between the hour hand and the
2
Hence, the three angles are 90∞, 60∞, and 30∞. minute hand = 180∞ - 135∞
Ex-4. The number of sides in two regular polygons are = 45∞.
5 : 4 and the difference between their angles is 9. Ex-7. Find the length of an arc of a circle of radius 10 cm
Find the number of sides of the polygon. subtending an angle of 30∞ at the centre.
Soln. Let the number of sides of the given polygons be 5x Soln. Angle subtended at the centre
p ˆ p
and 4x, respectively. = 30∞ = ÊÁ 30 ¥ ˜=
It is given that, Ë 180 ¯ 6
Ê 2 ¥ 5 x - 4 - 2 ¥ 4 x - 4 ˆ ¥ 90 = 9 p 5p
ÁË ˜¯ Hence, l = 10 ¥ = .
5x 4x 6 3

fi Ê 10 x - 4 - 2 x - 1ˆ = 1 Ex-8. The minute hand of a watch is 35 cm long. How far


ÁË ˜
5x x ¯ 10 does its tip move in 18 minutes?
Soln. The angle traced by a minute hand in 60 minutes
fi Ê 10 x - 4 - 10 x + 5 ˆ = 1 = 360∞ = 2p radians
ÁË ˜¯
5x 10 Thus, the angle traced by minute hand in 18 minutes
fi Ê 1ˆ = 1 18 3p
ÁË ˜¯ = 2p ¥ = radians
x 2 60 5
fi x=2 Hence, the distance moved by the tip in 18 minutes
Hence, the number of sides of the polygons will be 3p 22
10 and 8, respectively. = l = 35 ¥ = 21 ¥ = 66 cm
5 7
4 Comprehensive Trigonometry with Challenging Problems & Solutions for Jee Main and Advanced

Ex-9. At what distance does a man, whose height is 2 m 4 ¥ 640 ¥ p


subtend an angle of 10'? fi x=
9
Soln. Let AB be the height of the man and the required
distance be x, where BC = x 4 ¥ 640 ¥ 22
fi x=
A 9¥7
fi x = 894
Hence, the radius of the moon be 894 km.

10¢
EXERCISE 1
B x C 1. Find the length of an arc of a circle of radius 5 cm.
subtending a central angle of measuring 15∞.
2 180 10 2. In a circle of diameter 40 cm. the length of a chord is
Therefore, ¥ =
x p 60 20 cm. Find the length of minor arc corresponding to
2 180 the chord.
fi x= ¥ ¥ 60
10 p 3. If the arcs of same length in the circles subtends angles
of 60∞ and 75∞ at their centres. Find the ratio of their
12 ¥ 180
fi x= radii.
p 4. A horse is tied to a post by a rope. If the horse moves
12 ¥ 180 12 ¥ 180 ¥ 7 along a circular path always keeping the rope tight and
fi x= = discribes 88 meters when it has traced out 72∞ at the
22 22
7
5. The Moon’s distance from the Earth is 36,000 kms.
42 ¥ 180 and its diameter subtends an angle of 31∞ at the eye
fi x= = 687.3
11 1 of the observer. Find the diameter of the Moon.
Ex-10. Find the distance at which a globe 5 cm in 6. The difference between the acute angles of a right
2
diameter, will subtend an angle of 6'. 2p
Soln. Let the required distance be x cm angled triangle is radians. Express the angles in
degrees. 3
According to the question,
7. The angles of a quadrilateral are in A.P. and the greatest
11 180 angle is 120∞. Find the angles in radians.
6' = ¥
2¥ x p 8. The angles of a triangle are in A.P. such that the greatest
6 11 180 is 5 times the least. Find the angles in radians.
fi = ¥
60 2¥ x p 9. A wheel makes 180 revolutions per minute through
how many radians does it turn in 1 second?
11 180 60
fi x= ¥ ¥ 10. Find the distance from the eye at which a coin of 2 cm.
2 p 6 diameter should be held so as to conceal the full moon
11 180 ¥ 7 whose angular diameter is 31'.
fi x= ¥ ¥ 10
2 22 11. The interrior angles of a triangle are in A.P. The small-
fi x = 45 ¥ 7 ¥ 10 = 3150 est angle is 120∞ and the common difference is 5∞. Find
Hence, the required distance will be 3150 cms. the number of sides of the polygon.
Ex-11. The radius of the earth being taken as 6400 km and 12. A wheel makes 30 revolutions per minute. Find the
the distance of the moon from the earth being 60 circular measure of the angle described by a spoke in
1/2 second.
moon which subtends an angle of 16' at the earth. 13. A man running along a circular track at the rate of 10 miles
Soln. Let the radius of the moon be x km per hour travels in 36 seconds, an arc which subtends 56∞
16 2x 180 at the centre. Find the diameter of the circle.
It is given that, = ¥ 1
60 60 ¥ 6400 p 14. At what distance does a man 5 ft in height, subtends
an angle of 15"? 2
16 ¥ 6400 ¥ p
fi x= 15. Find the angle between the hour hand and minute hand
180 ¥ 2 in circular measure at 4 O’ clock.
The Ratios and Identities 5

1.6 TRIGONOMETRICAL RATIOS (iii) cosec2 n +1q , sec2 n +1 q ≥ 1

1.6.1 Definitions of Trigonometric Ratios cosec2 n +1q , sec2 n +1 q £ -1


where n ŒW
C Step VI Ranges of even power t-ratios.
(i) 0 £ sin 2 n q , cos 2 n q £ 1
p
h (ii) 0 £ tan 2 n q , cot 2 n q < •
(iii) 1 £ cosec2 nq , sec2 n q < •
q
B
where n ΠN
A b

1. Sin q =
p
2. cosec q =
h 1.7 LIMITS OF THE VALUES OF
h p TRIGONOMETRICAL FUNCTIONS
b h
3. cos q = 4. sec q = -1 £ sinq £ 1
h b 1.
p b 2. -1 £ cosq £ 1
5. tan q = 6. cot q = -• < tanq < •
b p 3.
1.6.2 Signs of Trigonometrical Ratios 4. -• < cotq < •
5. cosecq ≥ 1 and cosecq £ -1
The signs of the trigonometrical ratios in different quadrants
are remembered by the following chart. 6. sec q ≥ 1 and sec q £ -1

sin and cosec are All t-ratios are 1.8 SOME SOLVED EXAMPLES
+ve and rest are +ve
-ve Ex-1. If sec q + tan q = 3, where q
q.
tan and cot are cos and sec are Soln. Given sec q + tan q = 3 (i)
+ve and rest are +ve and rest are
-ve -ve 1 1
fi (sec q - tan q ) = = (ii)
(sec q + tan q ) 3
Adding (i) and (ii), we get,
It is also known as all, sin, tan, cos formula.
1.6.3 Relation between the Trigonometrical 1 10
2 secq = 3 + =
Ratios of an Angle 3 3
5
Step I (i) sin q . cosec q = 1 fi secq =
(ii) cos q . sec q = 1 3
(iii) tan q . cot q = 1 fi cosq =
5
sin q 3
Step II (i) tan q =
cos q Ex-2. If cosecq - cot q =
1
cos q 5
(ii) cot q = value of sin q.
sin q 1
Soln. Given cosecq - cot q = (i)
Step III (i) sin q . cosec q = 1 5
(ii) cos q . sec q = 1 1
fi cosec q - cot q = =5 (ii)
(iii) tan q . cot q = 1 cosec q + cot q
Step IV (i) sin2 q + cos2 q = 1 Adding (i) and (ii), we get,
(ii) sec2 q = 1 + tan2 q
1 26
(iii) cosec2 q = 1 + cot2 q 2 cosec q = 5 + =
Step V Ranges of odd power t-ratios. 5 5
13
(i) -1 £ sin 2 n +1 q , cos 2 n +1 q £ 1 fi cosec q =
5
(ii) -• < tan 2 n +1 q , cot 2 n +1 q < • 5
fi sin q =
13
6 Comprehensive Trigonometry with Challenging Problems & Solutions for Jee Main and Advanced

Ex-3. If a = c cos q + d sin q and b = c sin q - d cos q such


that a m + b n = c p + d q , where m, n, p, q ΠN
(
= r 2 cos 2 q cos 2 j + sin 2 j + r 2 sin 2 q ) ( )
m + n + p + q + 42. = r 2 cos 2 q + r 2 sin 2 q
Soln. Given a = c cos q + d sin q (i)
and b = c sin q - d cos q (ii)
2
(
= r cos q + sin q = r
2 2 2
)
Squaring and adding (i) and (ii), we get,
fi x2 + y 2 + z 2 = r 2
a + b = (c cos q + d sin q )
2 2 2

fi m = 2, n = 2, p = 2
+ (c sin q - d cos q )2
Thus, the value of ( m + n + p - 4)(m + n + p + 4)
fi 2 2
(
a + b = c cos q + d sin q
2 2 2 2
)
2 2
(
+ c sin q + d cos q
2 2
) = 210 = 1024
2 sin a
fi a 2 + b 2 = c2 + d2 Ex-6. If x =
1 + cos a + 3 sin a
fi m = 2, n = 2, p = 2, q = 2 sin a - 3 cos a + 3
value of
Hence, the value of m + n + p + q + 42 = 50 . 2 - 2 cos a
Ex-4. If 3 sin q + 4 cos q = 5 2 sin a
value of 3 cos q - 4 sin q . Soln. Given x =
1 + cos a + 3 sin a
Soln. Let x = 3 cos q - 4 sin q (i)
sin a - 3 cos a + 3
and 5 = 3 sin q - 4 cos q (ii) We have,
2 - 2 cos a
Squaring and adding (i) and (ii), we get
sin a + 3 (1 - cos a )
x2 + 52 = (3 cos q + 4 sin q )2 =
2 (1 - cos a )
+ (3 sin q - 4 cos q )2
sin a 3
+
(
fi x2 + 52 = 9 cos 2 q + 16 sin 2 q + 24 sin q cos q ) =
2 (1 - cos a ) 2
(
+ 9 sin q + 16 cos q - 24 sin q cos q
2 2
) =
sin a (1 + cos a )
+
3

( ) (
= 9 cos q + 16 sin q + 9 sin q + 16 cos q
2 2 2 2
) (
2 1 - cos a 2
) 2

sin a (1 + cos a ) 3
= 9 (cos q + sin q ) + 16 (cos q + sin q ) +
2 2 2 2
=
2 sin 2 a 2
fi x2 + 25 = 25
(1 + cos a ) 3
= +
fi x2 = 0 2 sin a 2
fi x=0 (1 + cos a + 3 sin a )
=
fi 3 cos q – 4 sin q = 0. 2 sin a
Ex-5. If x = r cos q sin j, y = r cos q cos j and z = r sinq 1
such that x m + y n + z p = r 2 , where m, n, p ΠN , = .
x
( m + n + p - 4 )m + n + p + 4 . Ex-7. If P = sec6 q - tan 6 q - 3 sec 2 q tan 2 q,
Soln. We have, x + y + z
2 2 2
Q = cosec6q - cot 6 q - 3 cosec 2q cot 2 q and
= ( r cos q cos j )2 + ( r cos q sin j )2 R = sin 6 q + cos6 q + 3 sin 2 q cos 2 q
the value of ( P + Q + R )( P +Q + R )
fi x2 + y 2 + z 2
Soln. We have, P = sec6 q - tan 6 q - 3 sec 2 q tan 2 q
(2 2 2 2 2
) (
= r cos q cos j + r cos q sin j
2
) (
= sec q - tan q
2 2
)
3
=1
+ r 2 sin 2 q ( )
Q = cosec6q - cot 6 q - 3 cosec 2q cot 2 q
fi x +y +z
2 2 2
The Ratios and Identities 7

( ) Ê t 2 - 1ˆ 1
3
= cosec q - cot q =1
2 2
fi t +Á =
Ë 2 ˜¯ 4
and R = sin 6 q + cos6 q + 3 sin 2 q cos 2 q 1
fi t 2 + 2t - 1 =
( )
3
= sin q + cos q =1
2 2 2
fi 2t2 + 4t - 3 = 0
Hence, the value of ( P + Q + R )( P +Q + R )
= 33 = 27. -4 ± 16 + 24
fi t=
4
p
Ex-8. If 3 sin x + 4 cos x = 5, for all x in ÊÁ 0, ˆ˜ -4 ± 2 10
Ë 2¯ = = -1 ±
1
10
the value of 2 sin x + cos x + 4 tan x 4 2
1
Soln. We have 3 sin x + 4 cos x = 5 fi t = -1 + 10
2
Let y = 3 cos x – 4 sin x
1
Now, y2 + 52 = (3 cos x – 4 sin x)2 fi sin q + cos q = -1 + 10
2
+ (3 sin x + 4 cos x)2
Now, (1 - sin q ) (1 - cos q )
fi y2 + 25 = 9 cos2 x + 16 sin2 x – 24 sin x cos x
= 1 - sin q - cos q + sin q cos q
+ 9 sin2 x + 16 cos2 x + 24 sin x cos x
= 1 - (sin q + cos q ) + sin q cos q
fi y2 + 25 = 25 (cos2 x + sin2 x) = 25
Ê 10 ˆ 1 Ê 10
fi y2 = 0 = 1 - Á -1 + ˜ + Á - 10 ˆ˜
Ë 2 ¯ 2Ë 4 ¯
fi y=0
Ê 5ˆ
fi 3 cos x – 4 sin x = 0 = ÁË 2 + ˜¯ - 10
4
fi 3 cos x = 4 sin x
Ê 13 ˆ
fi tan x = 3/4 = ÁË - 10 ˜¯ .
4
Hence, the value of 2 sin x + cos x + 4 tan x
Ex-11. Find the minimum value of the expression
= 2 ÊÁ ˆ˜ + ÊÁ ˆ˜ + 4 ÊÁ ˆ˜ = 2 + 3 = 5.
3 4 3 9 x 2 sin 2 x + 4
Ë 5¯ Ë 5 ¯ Ë 4¯ f ( x) = , for all x in (0, p ) .
x sin x
Ex-9. If sin A + sin B + sin C
of cos A + cos B + cos C + 10. 9 x 2 sin 2 x + 4 4
Soln. Given f ( x ) = = 9 x sin x +
Soln. Given sin A + sin B + sin C = – 3 x sin x x sin x
fi sin A = -1, sin B = -1, sin C = -1 Applying, A.M ≥ G.M, we get,
p p p Ê 9 x sin x + 4 ˆ
fi A= - ,B=- ,C=- Á x sin x ˜ 4
2 2 2
Á ˜ ≥ 9 x sin x ¥
Hence, the value of cos A + cos B + cos C + 10 Á 2 ˜ x sin x
Ë ¯
= 0 + 0 + 0 + 10 = 10.
5 Ê 9 x sin x + 4 ˆ
Ex-10. If (1 + sin q ) (1 + cos q ) = Á x sin x ˜
4 fiÁ ˜ ≥6
(1 - sin q ) (1 - cos q ) . Á 2 ˜
5 Ë ¯
Soln. We have (1 + sin q ) (1 + cos q ) =
4 Ê 4 ˆ
fi Á 9 x sin x + ≥ 12
5 Ë x sin x ˜¯
fi 1 + sin q + cos q + sin q cos q =
4 Hence, the minimum value of f (x) is 12.
Ê t 2 - 1ˆ 5 Ex-12. If cos q + sin q = 2 cos q , than prove that
fi 1+ t + Á = (sin q + cos q = t , say )
Ë 2 ˜¯ 4 cos q - sin q = 2 sin q
8 Comprehensive Trigonometry with Challenging Problems & Solutions for Jee Main and Advanced

Soln. We have, cos q + sin q = 2 cos q 2 sin q


Ex-15. If x = , then prove that
1 + cos q + sin q
fi sin q = ( )
2 - 1 cos q
1 - cos q + sin q
sin q = x.
fi cos q = 1 + sin q
( 2 -1) 2 sin q
Soln. Given x =
fi cos q = ( 2 + 1) sin q 1 + cos q + sin q
2 sin q
fi cos q - sin q = 2 sin q =
(1 + sin q ) + cos q
Ex-13. If tan 2 q = 1 - e2 then prove that 2 sin q ((1 + sin q ) - cos q )
( ) =
3/ 2
sec q + tan 3 q .cosec q = 2 - e2 ((1 + sin q ) + cos q ) ((1 + sin q ) - cos q )
Soln. We have, tan 2 q = 1 - e2 2 sin q ((1 + sin q ) - cos q )
=
fi 1 + tan 2 q = 1 + 1 - e2 = 2 - e2 ((1 + sin q ) 2
- cos 2 q )
2 sin q ((1 + sin q ) - cos q )
fi sec2 q = 2 - e2 =
(1 + sin 2 q + 2 sin q - cos2 q )
fi secq = 2 - e2 2 sin q ((1 + sin q ) - cos q )
=
Now, sec q + tan 3 q . cosec q
(sin 2 q + 2 sin q + (1 - cos2 q ))
2 sin q ((1 + sin q ) - cos q )
sin 3 q 1 =
= sec q + . (2 sin q + 2 sin 2 q )
cos3 q sin q
2 sin q ((1 + sin q ) - cos q )
sin 2 q =
= sec q + 2 sin q (1 + sin q )
cos3 q
((1 + sin q ) - cos q )
sin 2 q 1 =
= sec q + . (1 + sin q )
cos 2 q cos q
(1 - cos q + sin q )
= sec q + tan 2 q . sec q =
(1 + sin q )
(
= sec q 1 + tan q
2
) Ex-16. If
sin 4 a
+
cos 4 a
=
1
, then prove that
a b a+b
= sec q
3

sin 8 a cos8 a 1
( )
3/ 2
= 2-e
2
+ =
a3 b3 ( a + b )3
Ex-14. If sin q + sin 2 q + sin 3 q = 1, then prove that,
sin 4 a cos 4 a 1
cos6 q - 4 cos 4 q + 8 cos 2 q = 4 . Soln. We have, + =
a b a+b
Soln. Given sin q + sin 2 q + sin 3 q = 1
a + bˆ a + bˆ
fi ÊÁ sin 4 a + ÊÁ cos 4 a = 1
fi (sin q + sin3 q ) = 1 - sin 2 q = cos2 q Ë a ˜¯ Ë b ˜¯

(sin q + sin3 q ) = (cos2 q ) b a


2 2
fi fi ÊÁ1 + ˆ˜ sin 4 a + ÊÁ1 + ˆ˜ cos 4 a = 1
Ë a¯ Ë b¯
(sin q + sin3 q ) = (cos2 q )
2 2
fi b a
fi ÊÁ sin 4 a + cos 4 a ˆ˜ + sin 4 a + cos 4 a = 1
( )
(1 - cos2 q ) (2 - cos2 q ) = cos4 q Ëa b ¯
2

b a
fi (1 - cos2 q ) (4 - 4 cos2 q + cos4 q ) = cos4 q fi ÊÁ sin 4 a + cos 4 a ˆ˜ + 1 - 2 sin 2 a .cos 2 a = 1
Ëa b ¯ ( )
fi 4 - 4 cos 2 q + cos 4 q - 4 cos 2 q + 4 cos 4 q b a
fi ÊÁ sin 4 a + cos 4 a - 2 sin 2 a .cos 2 a ˆ˜ = 0
fi - cos6 q = cos 4 q Ëa b ¯
fi cos6 q - 4 cos 4 q + 8 cos 2 q = 4
The Ratios and Identities 9

2 2 + 6(1 + 2 sin x cos x)


Ê b ˆ Ê a ˆ
fi Á sin 2 a ˜ + Á cos 2 a ˜ + 4(sin2 x + cos2 x)2
Ë a ¯ Ë b ¯
– 12 sin2 x cos2 x.
b a = 3 – 6 sin2 x cos2 x – 12 sin x cos x
-2 sin 2 a . cos 2 a = 0
a b + 18 sin2 x cos2 x + 6 + 12 sin x cos x
2
Ê b a ˆ + 4 – 12 sin2 x cos2 x
fi Á sin 2 a - cos 2 a ˜ = 0
Ë a b ¯ =3+6+4
Ê b a ˆ =13.
fi Á sin 2 a - cos 2 a ˜ = 0
Ë a b ¯ Ex-18. If sin x + sin2 x
cos8 x + 2 cos6 x + cos4 x.
b a
fi sin 2 a = cos 2 a Soln. We have, sin x + sin2 x = 1
a b
fi sin x = 1 – sin2 x = cos2 x
sin 2 a cos 2 a 1
fi = = Now, cos8 x + 2 cos6 x + cos4 x
a b a+b
= (cos4 x)2 + 2 . cos4 x . cos2 x + (cos2 x)2
a b
fi sin 2 a = , cos 2 a = = (cos4 x + cos2 x)2
a+b a+b
= (sin2 x + sin x)2
sin a cos a
8 8
Now, + = (1)2 = 1.
a3 b3 2
q 2
q
Ex-19. If 0 £ ¸ £ 180∞ and 81sin + 81cos = 30
( ) +( )
4 4
sin a
2
cos a2
the value of q.
=
a3 b3
2
q 2
q
Soln. We have, 81sin + 81cos = 30
4 4 2
q 2
q
Ê a ˆ Ê b ˆ fi 81sin + 811-sin = 30
Ë a + b¯ Ë a + b¯ 81
= + fi 81sin
2
q
+ = 30
a3 b3 2
q
81sin
a4 b4 81
= + fi a+ = 30, a = 81sin q
2

a 3 ( a + b )4 b 3 ( a + b )4 a
a b fi a 2 - 30 a + 81 = 0
= +
(a + b) 4
(a + b) 4
fi (a - 27 ) (a - 3) = 0
a+b
= fi a = 3, 27
( a + b )4
When a = 3
1 2
q
= fi 81sin =3
(a + b) 3
4sin 2 q
fi 3 =3
Ex-17. Prove that 3 (sin x - cos x )4 + 6 (sin x + cos x )2
fi 4 sin q = 1
2
(
+ 4 sin 6 x + cos6 x = 13 . )
1 2
Soln. We have, 3 (sin x – cos x)4 + 6 (sin x + cos x)2 + 4 fi sin 2 q = ÊÁ ˆ˜
Ë 2¯
(sin 6x + cos 6x)
p
= 3 (sin4 x – 4 sin3 x cos x + 6 sin2 x cos2 x fi sin 2 q = sin 2 ÊÁ ˆ˜
Ë 6¯
– 4 sin x cos3 x + cos4 x)
p
+ 6(sin2 x + cos2 x + 2 sin x cos x) fi q = ÊÁ np ± ˆ˜
+ 4{(sin2 x)3 + (cos2 x)3}
Ë 6¯
= 3(sin4 x + cos4 x – 4 sin x cos x p 5p
fi q= ,
(sin2 x + cos2 x) + 6 sin2 x cos2 x) 6 6
10 Comprehensive Trigonometry with Challenging Problems & Solutions for Jee Main and Advanced

When a = 27 tan a + sec a - 1 1 + sin a


3. Prove that = .
fi 81sin 2 q
= 27 tan a - sec a + 1 cos a
4. Prove that sin8A – cos8A
2
q
fi 34sin = 33 = (sin2A – cos2A) ¥ (1 – 2sin2 A cos2A).
fi 4 sin 2 q = 3 5. If U n = sin n q + cos n q, prove that
2 2U 6 - 3U 4 + 1 = 0 .
Ê 3ˆ
fi sin q = Á ˜
2
1 1
Ë 2 ¯ 6. Prove that -
cosecq - cot q sin q
p
fi sin 2 q = sin 2 ÊÁ ˆ˜ 1
-
1
Ë 3¯ =
sin q cosecq + cot q
p ( a + b )2
fi q = ÊÁ np ± ˆ˜ 7. Prove that the equation = sin 2 q is
Ë 3¯ 4ab
possible only when a = b.
p 2p
fi q= ,
3 3 8. Prove that sin 2 q + cosec2q ≥ 2 .
p p 2p 5p 9. Prove that sec2 q + cosec2q ≥ 4 .
Hence, the values of q are , , , .
6 3 3 6 10. If sec q + tan q = 3 q.
Ex-20. Let f k (q ) = sin k (q ) + cos k (q ),
1
1 1 11. If cosecq - cot q = q.
f 6 (q ) - f 4 (q ) . 5
6 4
12. If x = a cos q + b sin q and y = a sin q - b cos q , then
Soln. We have f 6 (q ) = sin 6 q + cos6 q
prove that x2 + y2 = a2 + b2.
= 1 - 3 sin 2 q cos 2 q 13. If 3sin q + 5 cos q = 5, then prove that
5sin q – 3cos q = ±3.
Also, f 4 (q ) = sin 4 q + cos 4 q 14. If x = r cos q cos f, y = r cos q sin f, z = r sin q, then
prove that x2 + y2 + z2 = r2.
= 1 - 2 sin 2 q cos 2 q
15. If cos q + sin q = 2 cos q, than prove that
1 1 cos q – sin q = 2 sin q.
Now, f 6 (q ) - f 4 (q )
6 4 16. If tan2 A = 1 + 2 tan2 B, then
prove that cos2 B = 2 cos2A.
=
1
6
( 1
) (
1 - 3 sin 2 q cos 2 q - 1 - 2 sin 2 q cos 2 q
4
) 17. If tan2 q = 1 – e2, then prove that
( )
3/ 2
sec q + tan 3 q . cosecq = 2 - e2
1 1 2 1 1
= - sin q cos 2 q - + sin 2 q cos 2 q
6 2 4 2 18. If sin2 q + sin q = 1, then
1 1 prove that cos4 q + cos2 q = 1.
= - 19. If sin2 q + sin q
6 4
the value of cos12 q +3 cos10 q + 3 cos8 q + cos6 q.
1
= - . 20. If cos4 q + cos2 q = 1, then
12 prove that tan4 q + tan2 q = 1.
21. If sin 4 q + sin 2 q = 1, then
EXERCISE 2 prove that cos8 q + 2 cos6 q + cos 4 q =1.
cot q - tan q 22. If sin q + sin 2 q + sin 3 q = 1, then prove that,
1. Prove that = sec q cosec q.
1 - 2 sin 2 q cos6 q - 4 cos 4 q + 8 cos 2 q = 1 .
23. If sin q1 + sin q 2 + sin q3 = 3 ,
1 - sin q
2. Prove that tan q + = sec q. then prove that, cos q1 + cos q 2 + cos q3 = 0 .
1 + sin q
The Ratios and Identities 11

24. If x =
2 sin q
, then
1.9 MEASUREMENT OF THE ANGLES
1 + cos q + sin q OF DIFFERENT T-RATIOS
1 - cos q + sin q
prove that, = x. 1.9.1 Recognization of the quadrants
1 + sin q
We have introdcuced six t-ratios. P
sin q 2
1 + cos q sin q Signs of these t-ratios depend
25. Prove that 1 - + - = cos q .
1 + cos q sin q 1 - cos q upon the quadrant in which the r y
26. Prove that, terminal side of the angle lies.
We always take the length of OP
3 (sin x - cos x )4 + 6 (sin x + cos x )2 q
vector denoted by ‘r’, which is M
(
+ 4 sin 6 x + cos6 x = 13) always positive.
O x

y x
27. If a3 = cosecq - sin q and Thus, sinq = has the sign of y, cosq = has the sign of x
r r
b3 = sec q - cos q, then y
and tanq = depends on the signs of both x and y.
(
prove that a 2b 2 a 2 + b 2 = 1 .) x
Similarly, the signs of other trigonometric functions can
28. If x sin 3 a + y cos3 a = sin a cos a and be obtained by the signs of x and y.
x sin a = y cos a then prove that x 2 + y 2 = 1. x > 0, y > 0
29. If tan q + sin q = m, tan q – sin q = n, P

then prove that m2 – n2 = 4 mn .


r y
cos 4 x sin 4 x
30. If + = 1,
cos 2 y sin 2 y q
cos 4 y sin 4 y O x M
then prove that + =1
cos 2 x sin 2 x

sin 4 a cos 4 a 1
31. If + = , then
a b a+b
sin 8 a cos8 a 1 y r
prove that + = (i) sinq = > 0, cosecq = >0
a 3
b 3
( a + b )3 r y
y r
sin 4 q cos 4 q 1 (ii) cosq = > 0, secq = >0
32. If + = , then r y
2 3 5 y y
sin 8 q cos8 q 1 (iii) tanq = > 0, cotq = >0
prove that + = . x x
8 27 125
-
33. Let f k (q ) = sin k (q ) + cos k (q ) . Then prove
ed by ‘ALL’.
1 1 1
that f 6 (q ) - f 4 (q ) = - . (B) In the second quadrant, we have
6 4 12 x < 0 and y > 0
Y
34. If f n (q ) = sin n q + cos n q , prove that P

2 f 6 (q ) - 3 f 4 (q ) + 1 = 0 .

sin A cos A
35. If = p, = q , prove that
sin B cos B
q
p Ê q2 - 1ˆ X
tan A .tan B = Á . M O
q Ë 1 - p 2 ˜¯
12 Comprehensive Trigonometry with Challenging Problems & Solutions for Jee Main and Advanced

y y (E) Rotation
(i) sinq = > 0, cosecq = > 0
r r 90

x r
(ii) cosq = < 0, secq = < 0
r x
y y 180 0, 360
(iii) tanq = < 0, cotq = < 0
x x
Thus, in the second quadrant all t-raios are negative
other than sin and cosec. Due to this reason, second
270
quadrant is denoted by ‘SIN’.
- 270
(C) In the third quadrant, we have
x < 0 and y < 0
Y

-180 0, -360
M O
X¢ X
q

- 90
P

1.9.2 T-ratios of the angle (–q), in terms of q,
for all values of q.
y r
(i) sinq = < 0, cosecq = < 0 1. (i) sin (–q) = –sin q
r y
(ii) cos (–q) = cos q
x r
(ii) cosq = < 0, secq = < 0 (iii) tan (–q) = –tan q
r x (iv) cosec (–q) = – cosec q
y y (v) sec (–q) = sec q
(iii) tanq = > 0, cotq = > 0
x x (vi) cot (–q) = –cot q
1.9.3 T-ratios of the different angles in terms of q,
Thus, in the third quadrant all t-raios are negative other
than tan and cot. Due to this reason, third quadrant is
for all values of q.
denoted by ‘TAN’. 2. (i) sin (90 – q) = sin (90∞ ¥ 1 - q ) = cos q
(D) In the fourth quadrant, we have
(ii) sin (90 + q) = sin (90∞ ¥ 1 + q ) = cos q
x > 0 and y < 0
Y (iii) sin (180 – q) = sin (90∞ ¥ 2 - q ) = sin q
(iv) sin (180 + q) = sin (90∞ ¥ 2 + q ) = - sin q
M

O
X (v) sin (270 – q) = sin (90∞ ¥ 3 - q ) = - cos q
q
(vi) sin (270 + q) = sin (90∞ ¥ 3 + q ) = - cos q
(vii) sin (360 – q) = sin (90∞ ¥ 4 - q ) = - sin q
P (viii) sin (360 + q) = sin (90∞ ¥ 4 + q ) = sin q

3. (i) cos (90 – q) = cos (90∞ ¥ 1 - q ) = sin q
y r
(i) sinq = < 0, cosecq = < 0 (ii) cos (90 + q) = cos (90∞ ¥ 1 + q ) = - sin q
r y
(iii) cos (180 – q) = cos (90∞ ¥ 2 - q ) = - cos q
x r
(ii) cosq = > 0, secq = > 0 (iv) cos (180 + q) = cos (90∞ ¥ 2 + q ) = - cos q
r x
(v) cos (270 – q) = cos (90∞ ¥ 3 - q ) = - sin q
y y
(iii) tanq = < 0, cotq = < 0 (vi) cos (270 + q) = cos (90∞ ¥ 3 + q ) = - sin q
x x
(vii) cos (360 – q) = cos (90∞ ¥ 4 - q ) = cos q
Thus, in the fourth quadrant all t-raios are negative
other than cos and sec. Due to this reason, fourth (viii) cos (360 + q) = cos (90∞ ¥ 4 + q ) = cos q .
quadrant is denoted by ‘COS’.
The Ratios and Identities 13

4. (i) tan (90 – q) = tan (90∞ ¥ 1 - q ) = cot q (iii) sin 210∞


(iv) sin 225∞
(ii) tan (90 + q) = tan (90∞ ¥ 1 + q ) = - cot q (v) sin 300∞
(iii) tan (180 – q) = tan (90∞ ¥ 2 - q ) = - tan q (vi) sin 330∞
(iv) tan (180 + q) = tan (90∞ ¥ 2 + q ) = tan q (vii) sin 405∞
(viii) sin 650∞
(v) tan (270 – q) = tan (90∞ ¥ 3 - q ) = cot q
(ix) sin 1500∞
(vi) tan (270 + q) = tan (90∞ ¥ 3 + q ) = - cot q (x) sin 2013∞
(vii) tan (360 – q) = tan (90∞ ¥ 4 - q ) = - tan q Soln.
(viii) tan (360 + q) = tan (90∞ ¥ 4 + q ) = tan q . (i) sin (120∞) = sin (90 ¥ 1 + 30∞)
Note: All the above results can be remembered by the 3
= cos (30∞) =
following simple rule. 2
1. If q be measured with an even multiple of 90∞ by + or (ii) sin (150∞) = sin (90 ¥ 2 - 30∞)
– sign, then the T-ratios remains unaltered (i.e. sine re- 1
= sin (30∞) =
mains sine and cosine remains cosine, etc.) and treating 2
q as an acute angle, the quadrant in which the associated (iii) sin ( 210∞) = sin (90 ¥ 2 + 30∞)
angle lies, is determined and then the sign of the T-ratio
1
is determined by the All – Sin – Tan – Cos formula. = - sin (30∞) = -
2. If q be associated with an odd multiple of 90 by +ve or 2
–ve sign, then the T-ratios is altered in the form (i.e. sine (iv) sin ( 225∞) = sin (90 ¥ 2 + 45∞)
becomes cosine and cosine becomes sine, tangent 1
becomes cotangent and conversely, etc.) and the sign = - sin ( 45∞) = -
2
of the ratio is determined as in the previous paragraph.
(v) sin (300∞) = sin (90 ¥ 3 + 30∞)
3. If the multiple of 90 is more than 4, then divide it by
3
= - cos (30∞) = -
degree lies on the right of x-axis, if remainder is 1, 2
then the degree lies on the +ve y-axis, if remainder (vi) sin (330∞) = sin (90 ¥ 3 + 60∞)
is 2, then the degree lies on the –ve of x-axis and if
1
the remainder is 3, then the degree lies on the –ve of = - cos (60∞) = -
y–axis, respectively. 2
For examples: (vii) sin ( 405∞) = sin (90 ¥ 4 + 45∞)
1
(i) sin (570∞) = sin ( 45∞) =
= sin (90 ¥ 6 +30∞) 2
1 (viii) sin (660∞) = sin (90 ¥ 7 + 30∞)
= – sin 30∞ = - 1
2 = sin (30∞) =
(ii) tan (1950∞) 2
(ix) sin (1500∞) = sin (90 ¥ 16 + 60∞)
= tan (90 ¥ 22 –30∞)
3
= – tan (30∞) = –
1
. = sin (60∞) =
3 2
(iii) cos (2310∞) (x) sin ( 2013∞) = sin (90 ¥ 22 + 33∞)
= cos (90 ¥ 25 + 60∞) = - sin (33∞) .
3 Ex-2. Find the value of
= – sin (60∞) = – .
2 cos (1∞) .cos ( 2∞) .cos (3∞) ......cos (189∞) .

1.10 SOME SOLVED EXAMPLES Soln. We have,


cos (1∞) .cos ( 2∞) .cos (3∞) ..........cos (189∞) .
Ex-1. Find the value of
(i) sin 120∞ = cos (1∞) .cos ( 2∞) .cos (3∞) .........cos (89∞)
(ii) sin 150∞
14 Comprehensive Trigonometry with Challenging Problems & Solutions for Jee Main and Advanced

cos (90∞) cos (91∞) .........cos (189∞) . Ex-6. Find the value of
cos (10∞) + cos ( 20∞) + cos (30∞)
= cos (1∞) .cos ( 2∞) .cos (3∞) .....cos (89∞)
+ cos ( 40∞) + .......... + cos (360∞) .
¥ 0 ¥ cos (91∞) ................cos (189∞) .
Soln. We have, cos (10∞) + cos ( 20∞) + cos (30∞)
= 0.
+ cos ( 40∞) + .......... + cos (360∞)
Ex-3. Find the value of
tan (1∞) .tan ( 2∞) .tan (3∞).......tan (89∞) . = cos 20∞ + cos 30∞ + cos 40∞ + .........
+ cos 140∞ + cos 150∞ + cos 160∞ + cos 170∞
Soln. We have,
+ cos 180∞ + (cos 190∞ + cos 200∞ +
tan (1∞) .tan ( 2∞) .tan (3∞) .....tan (89∞)
cos 210∞ + cos 220∞ +..............+ cos 360∞)
= tan (1∞) .tan ( 2∞) .tan (3∞).......tan ( 44∞) = cos 10∞ + cos 20∞ + cos 30∞ + cos 40∞ +......
tan ( 45∞) tan ( 46∞) ...tan (87∞) tan (88∞) tan (89∞) – cos 40∞ – cos 50∞ – cos60
= {tan (1∞) ¥ tan (89∞)}.{tan ( 2∞) ¥ tan (88∞)}. – cos70∞ + cos 180∞ + (cos 190∞
......{tan ( 44∞) ¥ tan ( 46∞)}.tan ( 45∞) + cos 200∞ + cos 210∞ + cos 220∞ +
........... + cos 360∞)
= 1.
= cos 180∞ + cos 360∞
Ex-4. Find the value of
= –1 + 1
tan 35∞.tan 40∞.tan 45∞.tan 50∞.tan 55∞
Soln. We have, = 0.
tan 35∞.tan 40∞.tan 45∞.tan 50∞.tan 55∞ Ex-7. Find the value of
sin 2 5∞ + sin 2 10∞ + sin 2 15∞ + ....... + sin 2 90∞
= {tan 35∞ ¥ tan 55∞} {tan 40∞ ¥ tan 50∞}
¥ tan 45∞ Soln. We have,

= {tan 35∞ ¥ cot 35∞}. {tan 40∞ ¥ cot 40∞} sin 2 5∞ + sin 2 10∞ + sin 2 15∞ + .... + sin 2 90∞
¥ tan 45∞ = sin 2 5∞ + sin 2 10∞ + sin 2 15∞ +
= 1.
...... + sin 2 40 + sin 2 45
Ex-5. Find the value of sin (10∞) + sin ( 20∞) + sin (30∞)
+ sin ( 40∞) + ........... + sin (360∞) . + sin 2 50 + sin 2 80 + sin 2 85 + sin 2 90∞
Soln. We have, sin (10∞) + sin ( 20∞) + sin (30∞) (
= sin 2 5∞ + sin 2 85∞ )
+ sin ( 40∞) + ........... + sin (360∞) (
+ sin 2 10∞ + sin 2 80∞ )
= sin (10∞) + sin ( 20∞) + sin (30∞)
+ (sin 2 15∞ + sin 2 75∞)
+ sin ( 40∞) + .......... + sin (150∞)
+ ......... + (sin 2 40∞ + sin 2 50∞)
+ sin (340∞) + sin (350∞) + sin (360∞)
= sin (10∞) + sin ( 20∞) + sin (30∞) + (sin 2 45∞ + sin 2 90∞)

+ sin ( 40∞) + .......... + sin (80∞) = (sin 2 5∞ + cos 2 5∞)


+ sin (90∞) + sin (100∞) + (sin 2 10∞ + cos 2 10∞)
+ sin (360∞ - 40∞) + sin (360∞ - 30∞) + (sin 2 15∞ + cos 2 15∞) + ..........
+ sin (360∞ - 20∞) + sin (360∞ - 10∞)
.......... + (sin 2 40∞ + cos 2 40∞)
+ sin (360∞)
+ (sin 2 45∞ + sin 2 90∞)
= sin (10∞) + sin ( 20∞) + sin (30∞)
= (1 + 1 + ............. 8 times) + ÊÁ + 1ˆ˜
1
+ sin ( 40∞) + .......... + sin (80∞)
Ë2 ¯
+ sin (90∞) + sin (100∞)
= ÊÁ 8 + 1 + ˆ˜
1
- sin ( 40∞) - sin (30∞) Ë 2¯
- sin ( 20∞) - sin (10∞) + sin (180∞) = 9 .
1
= 0. 2
The Ratios and Identities 15

Ex-8. Find the value of Thus, sin (q1 ) = 1,sin (q 2 ) = 1,sin (q3 ) = 1
p p 4p 7p ˆ
sin 2 ÊÁ ˆ˜ + sin 2 ÊÁ ˆ˜ + sin 2 ÊÁ ˆ˜ + sin 2 ÊÁ ˜ . p p p
Ë 18 ¯ Ë 9¯ Ë 9¯ Ë 18 ¯ q1 = , q 2 = , q3 =
p p 2 2 2
Soln. We have, sin 2 ÊÁ ˆ˜ + sin 2 ÊÁ ˆ˜
Ë 18 ¯ Ë 9¯ Hence, the value of cos (q1 ) + cos (q 2 ) + cos (q3 )

p p p
4p 7p
+ sin 2 ÊÁ ˆ˜ + sin 2 ÊÁ ˆ˜ = cos ÊÁ ˆ˜ + cos ÊÁ ˆ˜ + cos ÊÁ ˆ˜
Ë 9¯ Ë 18 ¯ Ë 2¯ Ë 2¯ Ë 2¯

p 2p = 0.
= sin 2 ÊÁ ˆ˜ + sin 2 ÊÁ ˆ˜
Ë 18 ¯ Ë 18 ¯
EXERCISE 3
2 Ê 8p ˆ 2 Ê 7p ˆ
+ sin Á ˜ + sin Á ˜
Ë 18 ¯ Ë 18 ¯ 1. Find the values of
p p 7p (i) sin (135°) (ii) cos (150°)
= sin 2 ÊÁ ˆ˜ + sin 2 ÊÁ - ˆ˜ (iii) tan (120°) (iv) sin (225°)
Ë 18 ¯ Ë 2 18 ¯
(v) sec (240°) (vi) tan (300°)
p p 7p
+ sin 2 ÊÁ - ˆ˜ + sin 2 ÊÁ ˆ˜
(vii) sin (330°) (viii) tan (315°)
Ë 2 18 ¯ Ë 18 ¯ (ix) cos (315°) (x) sin (405°)
2. Find the values of
p 7p
= sin 2 ÊÁ ˆ˜ + cos 2 ÊÁ ˆ˜ (i) sin (675°) (ii) cos (1230°)
Ë 18 ¯ Ë 18 ¯ (iii) tan 1020° (iv) cosec (1305°)
(v) sec (–1035°) (vi) tan (–1755°)
p 7p
+ cos 2 ÊÁ ˆ˜ + sin 2 ÊÁ ˆ˜ (vii) sin (1410°) (viii) cos (1450°)
Ë 18 ¯ Ë 18 ¯
(ix) tan (2010°) (x) sin (1950°)

{ p
Ë 18 ¯
p
= sin 2 ÊÁ ˆ˜ + cos 2 ÊÁ ˆ˜
Ë 18 ¯ } 3. Express in terms of ratios of smallest +ve angles.
(i) sin 240°
(iii) sin (–1358°)
(ii) cos 780°
(iv) cosec (–1150°)

{ 7p
Ë 18 ¯
7p
+ cos 2 ÊÁ ˆ˜ + sin 2 ÊÁ ˆ˜
Ë 18 ¯ } (v) tan (–1750°)
4. If q =
23p
q – tan q.
=1+1 6
= 2. sin(270 + A) cos(90 - A)
5. Simplify: .
Ex-9. Find the value of sin(180 - A) cos(180 - A)
tan ( 20∞) tan ( 25∞) tan ( 45∞) tan (65∞) tan (70∞) . 6. Simplify: tan 25° . tan 35° . tan 45° tan 55° . tan 65°
Soln. We have 7. Prove that
tan ( 20∞) tan ( 25∞) tan ( 45∞) tan (65∞) tan (70∞) p 3p 5p 7p
sin 2 + sin 2 + sin 2 + sin 2 = 2.
8 8 8 8
= tan ( 20∞) tan ( 25∞) tan ( 45∞)
8. Prove that
tan (90∞ - 25∞) tan (90∞ - 20∞)
p 3p 5p 7p
sin 2 + sin 2 + sin 2 + sin 2 = 2.
= tan ( 20∞) tan ( 25∞) tan ( 45∞) cot ( 25∞) cot ( 20∞) 4 4 4 4
= tan ( 45∞) 9. Find the value of
sin 2 5∞ + sin 2 10∞ + .... + sin 2 90∞ .
= 1. 10. Find the value of
Ex-10. Find the value of cos (q1 ) + cos (q 2 ) + cos (q3 ) sin 2 6∞ + sin 2 12∞ + .... + sin 2 90∞
if sin (q1 ) + sin (q 2 ) + sin (q3 ) = 3 . 11. Find the value of
sin 2 10∞ + sin 2 20∞ + .... + sin 2 90∞
Soln. Given sin (q1 ) + sin (q 2 ) + sin (q3 ) = 3 12. Find the value of
It is possible only when each term of the above
sin 2 9∞ + sin 2 18∞ + .... + sin 2 90∞
equation will provide the maximum value
16 Comprehensive Trigonometry with Challenging Problems & Solutions for Jee Main and Advanced

13. Find the value of Characteristics of cosine function:


tan 1∞.tan 2∞.tan 3∞....tan 89∞
1. It is an even function, since
14. Find the value of
cos(–x) = cosx
cos1∞.cos 2∞.cos 3∞......cos189∞
2. It is a periodic function with period 2p.
15. Solve for q; 2 sin 2 q + 3 cos q = 0 3. cos x = 1 fi x = 2np, n ΠI
where 0 < q < 360∞ . p
16. Solve for q; cos q + 3 sin q = 2 , 4. cos x = 0 fi x = ( 2n + 1) , n ΠI
2
where 0 < q < 360∞ . 5. cos x = –1 fi x = ( 2n + 1) p , n Œ I
17. If 4n a = p, then prove that
tan a tan 2a tan 3a ....... tan(2n – 1)a = 1. 3. Graph of f (x) = tan x.
Q. If A, B, C, D be the angles of a cyclic quadrilateral Y
ABCD, then prove that
18. cos A + cos B + cos C + cos D = 0
19. tan A + tan B + tan C + tan D = 0.
20. sin 2A + sin 2B + sin 2C + sin 2D = 0
21. Find the value of X¢ X
O
cos (18∞) + cos ( 234∞) + cos (162∞) + cos (306∞) .
22. Find the value of
cos ( 20∞) + cos ( 40∞) + cos (60∞) + .... + cos (180∞)
23. Find the value of Y¢
sin ( 20∞) + sin ( 40∞) + sin (60∞) + .... + sin (360∞)
Characteristics of tangent function:
Graph of Trigonometric functions: 1. It is an odd function, since
tan (–x) = –tan x
1. Graph of f (x) = sin x.
2. It is a periodic function with period p
Y
p
3. tan x = 1 fi x = ( 4n + 1) , n ΠI
Y=1
4
4. tan x = 0 fi x = np, n ΠI
X¢ X
0 p p
5. tan x = –1 fi x = ( 4n - 1) , n Œ I
Y = -1 4
4. Graph of f (x) = cot x

Y
Characteristics of sine function:
1. It is an odd function, since sin (–x) = – sin x
2. It is a periodic function with period 2p
p
3. sin x = 1 fi x = ( 4n + 1) , n Œ I X¢ X
2 O
4. sin x = 0 fi x = np, n ΠI
p
5. sin x = –1 fi x = ( 4n - 1) , n Œ I
2
Graph of f (x) = cos x:

Y
Characteristics of cotangent function:
1. It is an odd function, since
X¢ X cot (–x) = –cot x
O
2. It is a periodic function with period p
p
3. cot x = 1 fi x = ( 4n + 1) , n ΠI
4

The Ratios and Identities 17

p 3. f (x) = –sin x
4. cot x = 0 fi x = ( 2n + 1) , n ŒI
2 4. f (x) = 1 – sin x
p 5. f ( x) = -1 - sin x
5. cot x = –1 fi x = ( 4n - 1) , n ŒI
4 6. f ( x ) = sin 2 x ,sin 3x

7. f ( x ) = sin 2 x
Graph of f (x) = sec x
Y 8. f ( x ) = cos 2 x
9. f ( x ) = max {sin x , cos x}
y=1 10. f ( x ) = min {sin x , cos x}
X¢ X
y = -1
{ 1
11. f ( x ) = min sin x , , cos x
2 }
12. f ( x ) = max {tan x , cot x}
13. f ( x ) = min {tan x , cot x}

Q. Find the number of solutions of
Characteristics of secant function 1
1. sin x = , " x Œ[0, 6]
2
1. It is an even function, sec(–x) = sec x
2. It is a periodic function with period 2p 3
2. cos x = , " x Œ[0, 10]
3. sec x can never be zero. 2
4. sec x = 1 fi x = 2np , n Œ I 3. 4 sin 2 x - 1 = 0 , " x Œ[0, 10]
5. sec x = –1 fi x = ( 2n + 1) p , n Œ I
4. sin 2 x - 3 sin x + 2 = 0, " x Œ[0, 10]
Graph of f (x) = cosec x
Y 5. cos 2 x - cos x - 2 = 0, "x Œ[0, 10]

1.11 T-RATIOS OF COMPOUND


y=1

X¢ X
ANGLES
O
y = -1
The algebraic sum or difference of two or more angles is
called a compound angle such as
Y¢ A + B, A – B, A + B + C, A + B – C etc.

1.11.1 The Addition Formula


Characteristics of cosecant function:
1. sin (A + B) = sin A cos B + cos A sin B
1. It is an odd function, since 2. cos (A + B) = cos A cos B – sin A sin B
cosec(–x) = –cosec x tan A + tan B
2. It is a periodic function with period 2p 3. tan ( A + B ) =
1 - tan A.tan B
p
3. cosec x = 1 fi x = ( 4n + 1) , n ΠI Proof:
2 Y
4. cosec x can never be zero. Q

p
5. cosecx = -1 fi x = ( 4n - 1) , n ΠI
2 R P

EXERCISE 4
B
Q. Draw the graphs of A
X
1. f (x) = sin x + 1 O S M
2. f (x) = sin x – 1
18 Comprehensive Trigonometry with Challenging Problems & Solutions for Jee Main and Advanced

Let –POX = A & –POQ = B tan A + tan B


=
Draw PM and QS perpendicular on OX and 1 - tan A.tan B
PR is parallel to OX.
Clearly, –PQR = A . Subtraction formulae:
1. sin (A + B) 1. sin (A – B) = sin A cos B – cos A sin B
2. cos (A – B) = cos A cos B + sin A sin B
QS SR + QR
= = tan A - tan B
OQ OQ 3. tan (A – B) = .
1 + tan A tan B
PM + QR Proof:
=
OQ
Y
PM RQ P R
= +
OQ OQ
PM OP QR PQ Q
= . + .
OP OQ PQ OQ
A
B
= sin A cos B + cos A. sin B. X
O M S
2. cos ( A + B )
OS OM - MS
= =
OQ OQ Let –POX = A and –QOX = B
Clearly, –POQ = A - B
OM MS Draw PM, RS perpendicular on OX and PR is parallel
= -
OQ OQ to OX.
OM MS From geometry, we can say that, –PQR = A
= - In DQOS ,
OQ OQ
OM OP PR PQ 1. sin ( A - B )
= . - .
OP OQ PQ OQ QS RS - RQ PM - RQ
= = =
= cos A .cos B – sin A. sin B. OQ OQ OQ
3. tan(A + B) PM RQ
= -
QS QR + RS OQ OQ
= =
OS OS PM OP RQ PQ
= . - .
QR + RS OP OQ PQ OQ
=
OM - SM
= sin A . cos B – cos A . sin B.
QR RS
+ 2. cos ( A - B )
= OM OM
SM OQ OM + MS OM + PR
1- = = =
OM OQ OQ OQ
QR RS OM PR
+ = +
= OM OM OQ OQ
PR PQ
1- . OM OP PR PQ
PQ OM = . + .
OP OQ PQ OQ
PM PQ
+
= cos A . cos B + sin A . sin B.
= OM OP
PM PQ
1- . 3. tan ( A - B )
OQ OP
The Ratios and Identities 19

= QS = RS - QR = RS - QR {
= cos 2 A cos 2 B - sin 2 A sin 2 B}
OS OM + MS OM + PR
= {cos A (1 - sin B ) - (1 - cos A) sin 2 B}
2 2 2
PL QR
-
= {cos 2 A - cos 2 A sin 2 B - sin 2 B + cos 2 A sin 2 B}
= OM OM
PR
1+ = cos 2 A - sin 2 B
OM
PL QR Deduction 3.
-
= OM OM cot (A + B) = cot A cot B - 1
PR PM cot B + cot A
1+ .
PM OM Proof: We have, cot ( A + B )
PM PQ cos ( A + B )
-
OM OM =
= sin ( A + B )
PM PQ
1+ . cos A cos B - sin A sin B
OM OM =
tan A - tan B sin A cos B + cos A sin B
= .
1 + tan A.tan B cos A cos B sin A sin B
-
Note: sin A sin B sin A sin B
=
p 1 + tan A sin A cos B cos A sin B
1. tan ÊÁ + Aˆ˜ = +
sin A sin B sin A sin B
Ë4 ¯ 1 - tan A
cot A cot B - 1
p 1 - tan A
2. tan ÊÁ - Aˆ˜ =
=
Ë4 ¯ 1 + tan A cot B + cot A

Deduction 4.
1.12 SOME IMPORTANT DEDUCTIONS cot A cot B + 1
cot (A – B) =
Deduction 1. cot B - cot A
sin (A + B) sin (A – B) Proof: We have, cot (A – B)
= sin2 A – sin2 B = cos2 B – cos2 A cos ( A - B )
=
Proof: We have sin ( A + B ) sin ( A - B ) sin ( A - B )
= {sin A cos B + cos A sin B} cos A cos B + sin A sin B
=
sin A cos B - cos A sin B
¥ {sin A cos B - cos A sin B}
cos A cos B sin A sin B
{
= sin 2 A cos 2 B - cos 2 A sin 2 B } +
sin A sin B sin A sin B
= {sin A (1 - sin B ) - (1 - sin A) sin 2 B}
2 2 2 =
sin A cos B cos A sin B
-
= {sin 2 A - sin 2 A sin 2 B - sin 2 B - sin 2 A sin 2 B} sin A sin B sin A sin B
cot A cot B + 1
= sin 2 A - sin 2 B =
cot B - cot A
( ) (
= 1 - cos 2 A - 1 - cos 2 B ) Deduction 5.
= cos B - cos A
2 2
sin (A + B + C)
Deduction 2. = cos A cos B cos C [ tan A + tan B + tan C
cos (A + B) cos (A – B) - tan A tan B tan C ]
= cos A – sin B = cos B – sin A
2 2 2 2
Proof: We have sin ( A + B + C )
Proof: We have, cos ( A + B ) cos ( A - B ) = sin ( A + B ) cos C + cos ( A + B ) sin C
= {cos A cos B + sin A sin B} = {sin A.cos B + cos A.sin B} cos C
¥ {cos A cos B - sin A sin B}
+ {cos A cos B - sin A sin B} sin C
20 Comprehensive Trigonometry with Challenging Problems & Solutions for Jee Main and Advanced

= sin A.cos B.cos C + sin B.cos A.cos C


3 -1
+ sin C.cos A cos B - sin A.sin B.sin C =
2 2
= cos A.cos B.cos C [ tan A + tan B + tan C (ii) cos(15°) = cos(45° – 30°)
- tan A.tan B.tan C ] = cos(45°) cos(30°) + sin(45°) sin(30°)
1 3 1 1
Deduction 6. = . + .
2 2 2 2
cos (A + B + C)
= cos A cos B cos C 3 +1
=
¥ [1 - tan A tan B - tan B tan C - tan C tan A] 2 2
(iii) tan (15°) = tan (45° – 30°)
Proof: We have, cos ( A + B + C )
tan 45∞ - tan 30∞
= cos ( A + B ) cos C - sin ( A + B ) sin C =
1 + tan 45∞.tan 30∞
= {cos A cos B - sin A sin B} cos C 1
1-
- {sin A cos B + cos A sin B} sin C =
3
= cos A cos B cos C - sin A sin B cos C 1
1+
3
{- sin A sin C cos B - cos A sin B sin C }
= cos A cos B cos C 3 -1
=
¥ [1 - tan A tan B - tan B tan C - tan C tan A] 3 +1

( )
2
Deduction 7. 3 -1
=
tan (A + B + C) 3 -1
tan A + tan B + tan C - tan A tan B tan C 3 +1- 2 3
= =
1 - tan A tan B - tan B tan C - tan C tan A 2
Proof: We have, tan ( A + B + C ) 4-2 3
=
sin ( A + B + C ) 2
=
cos ( A + B + C ) = 2- 3
cos A cos B cos C {tan A + tan B + tan C
Note:
- tan A tan B tan C} 1 1
= (i) cot (15∞) = =
cos A cos B cos C{1 - tan A tan B - tan(15∞) 2 - 3
tan B tan C - tan C tan A}
= 2+ 3
Ê tan A + tan B + tan C - tan A tan B tan C ˆ
= Á (ii) tan (105∞) = - cot(15∞) = -(2 + 3 )
Ë 1 - tan A tan B - tan B tan C - tan C tan A ˜¯
(iii) cot (105∞) = - tan(15∞) = -(2 - 3 )

1.13 SOME SOLVED EXAMPLES = 3-2.


Ex-1. Find the values of Ex-2. Find the value of tan (75°) + cot (75°)
(i) sin (15°), Soln. We have, tan(75°) + cot(75°)
(ii) cos (15°), = cot(15°) + tan(15°)
(iii) tan (15°)
= (2 + 3 ) + (2 - 3 )
Soln. We have,
(i) sin(15°) = sin(45° – 30°) = 4.
= sin(45°)cos(30°) – cos(45°)sin(30°) Ex-3. Prove that cos(9°) + sin(9°) = 2 sin(54°)
1 3 1 1 Soln. We have, cos(9°) + sin(9°)
= . - .
2 2 2 2
Ê 1 1 ˆ
= 2Á cos (9°) + sin (9°)˜
Ë 2 2 ¯
The Ratios and Identities 21

= {(1 + tan 245°)(1 + tan (–200°))}


= 2 (sin (45°) cos (9°) + cos (45°) sin (9°) )
{(1 + tan 250°)(1 + tan (–205°))}
= 2 (sin (45° + 9°))
{(1 + tan 260°)(1 + tan (–215°))}
= 2 sin(54∞) . =2¥2¥2
Ex-4. Prove that tan(70°) = 2 tan(50°) + tan(20°) = 8.
Soln. We have, tan(70°) = tan(50° + 20°)
Ex-8. If tan a + tan b = a, cot a + cot b = b and tan(a +
tan 50∞ + tan 20∞ b) = c a, b and c
fi tan(70∞) =
1 - tan 50∞ tan 20∞ Soln. We have, tan (a + b) = c
fi tan (70°) – tan(70°) tan (50°) tan(20°)
tan a + tan b
= tan 50° + tan 20° fi =c
1 - tan a tan b
fi tan(70°) – tan(70°) tan(50°) cot(70°) a
= tan50° + tan 20° fi =c
1 - tan a tan b
fi tan(70°) – tan(50°) = tan 50° + tan 20° a a-c
fi tan a tan b = - 1= (i)
fi tan(70°) = 2 tan 50° + tan 20° c c
Ex-5. If A + B Now, cot a + cot b = b
value of (1 + tan A)(1 + tan B) 1 1
Soln. We have, A + B = 45° fi + =b
tan a tan b
fi tan(A + B) = tan(45°)
tan a + tan b
fi tan(A + B) = 1 fi =b
tan a . tan b
tan A + tan B
fi =1 fi a
1 - tan A.tan B tan a . tan b = (ii)
b
fi tan A + tan B = 1 – tanA . tanB
a-c a
fi tan A + tan B + tan A . tan B = 1 From (i) and (ii), we get, =
c b
fi 1 + tan A + tan B + tanA . tanB = 1 + 1 = 2 fi ac + bc = ab
fi (1 + tan A) + tan B(1 + tan A) = 2 Which is the required relation.
fi (1 + tan A)(1 + tan B) = 2 n sin a cos a
Ex-9. If tan b = ,
Ex-6. Find the value of 1 - n sin 2 a
(1 + tan 20°)(1 + tan 24°)
then prove that tan (a – b) = (1 – n) tan a Soln.
(1 + tan 25°)(1 + tan21°)
tan a - tan b
Soln. We have, We have, tan (a - b ) =
1 + tan a .tan b
(1 + tan 20°) (1 + tan 24°) (1 + tan 25°) (1 + tan 21°)
sin a Ê n sin a cos a ˆ
= {(1 + tan 20°)(1 + tan 25°)} -Á ˜
¥ {(1 + tan24°)(1 + tan 21°)} cos a Ë 1 - n sin 2 a ¯
=
sin a Ê n sin a cos a ˆ
=2¥2 1+ ¥Á ˜
cos a Ë 1 - n sin 2 a ¯
= 4.
Ex-7. Find the value of ( )
sin a 1 - n sin 2 a - n sin a cos 2 a
cos a (1 - n sin a ) + n sin a .cos a
= 2 2
(1 + tan 245°)(1 + tan 250°)

sin a - n sin a (sin a + cos a )


(1 + tan 260°)(1 – tan 200°) 2 2
(1 – tan 205°)(1 – tan 215°) =
cos a - n sin 2 a . cos a + n sin 2 a .cos a
Soln. We have, (1 + tan 245°)(1 + tan 250°)
sin a - n sin a
(1 + tan 260°)(1 – tan 200°) =
cos a
(1 – tan 205°)(1 – tan 215°)
22 Comprehensive Trigonometry with Challenging Problems & Solutions for Jee Main and Advanced

(1 - n)sin a sin b cos b


= =
cos a 2 + sin 2 b
= (1 – n) tan a. 2 sin b cos b
=
Ex-10. If x + y + z = 0, then prove that, 4 + 2 sin 2 b
cot(x + y – z). cot(y + z – x)
sin 2b
+ cot (y + z – x). cot (z + x – y) =
+ cot (z + x – y). cot (x + y – z) = 1 4 + 1 - cos 2b

Soln. Let A = x + y – z, B = y + z – x, C = x + y – z sin 2b


=
Then, A + B + C 5 - cos 2b
= (x + y – z) + (y + z – x) + (x + y – z)
= (x + y + z) = 0
EXERCISE 5
fi A+B=–C
fi cot(A + B) = cot(–C) 1. Find the values of sin15°, cos15°,
sin 75°, cos 75°.
cot A cot B - 1
= cot ( -C ) 2. Find the values of tan 15°, cot 15°, tan 75°, cot 75°.
fi cot A + cot B 3. Prove that
cot A cot B - 1 (i) tan 15° + cot 15 ° = 4
= - cot C (ii) tan 75 ° + cot 75 ° = 4
fi cot A + cot B
(iii) cot 15 ° – tan 15 ° = 2 3
fi cotA cotB – 1 = –cotA cotC – cotB cotC
fi cotA cotB + cotA cotC + cotB cotC = 1 (iv) tan 75° – cot 75 ° = 2 3 .

fi cot(x + y – z). cot(y + z – x) 4. Prove that cos 18° – sin 18° = 2 sin 27°.
+ cot (y + z – x). cot (z + x – y) 5. Prove that sin(n + 1)x sin (n + 2) x
+ cos (n + 1)x cos(n + 2)x = cos x.
+ cot (z + x – y). cot (x + y – z) = 1.
Ex-11. If 2 tan a = 3 tan b, then show that, Ê xˆ
6. Prove that 1 + tan x .tan Á ˜ = sec x .
sin 2b Ë 2¯
tan (a - b ) =
5 - cos 2b 7. Prove that cot x – cot 2x = cosec2x.
Soln. We have, 8. Prove that tan69° + tan66° = -1 .
tan a - tan b 1 - tan69°tan66°
tan (a - b ) = 9. Prove that tan 70° = 2 tan 50° + tan 20°.
1 + tan a . tan b
p
10. If A + B = , then prove that
3 4
tan b - tan b
= 2 (1 + tanA)(1 + tanB) = 2.
3
1 + tan b tan b cos 20∞ - sin 20∞
2 11. Prove that = tan 25°.
cos 20∞ + sin 20∞
tan b cos 7∞ + sin 7∞
= 12. Prove that = tan 52°.
2 + 3 tan 2 b cos 7∞ - sin 7∞
13. Prove that tan20° + tan 25° + tan 20° tan 25° = 1.
sin b
14. Prove that tan 13 A – tan 9 A – tan 4A
cos b
= = tan 4A . tan9A . tan13A.
sin 2 b
2+3 15. Prove that tan 9A – tan 7A – tan 2A
cos 2 b
= tan 2A . tan 7A. tan 9A.
sin b cos b 16. Prove that cot x cot 2 x – cot 2 x cot 3 x
=
2 cos 2 b + 3 sin 2 b – cot 3x cot 2 x = 1.
The Ratios and Identities 23

m 1 29. If a and b are the roots of


17. If tan a = , tan b = , a tan q + b tan q = c, then show that
m +1 2m + 1
2ac
p tan (a + b ) = 2
then prove that a + b = . a - c2
4
30. If tan (p cos q) = cot(p sin q), prove that
18. Find the value of pˆ
Ê 1
(i) sin2 75° – sin2 15° cos Á q - ˜ = ±
Ë 4¯ 2 2.
(ii) cos2 75° – sin2 15°.
Êp ˆ Êp ˆ x sin j y sin q
19. Prove that cos 2 Á + q ˜ - sin 2 Á - q ˜ = 0 31. If tan q = and tan j = ,
Ë4 ¯ Ë 4 ¯ 1 - x cos j 1 - y cos q
20. Prove that sin2 B then prove that, x sin j = y sin q.
= sin2 A + sin2 (A – B) – 2 sinAcosB sin (A – B).
21. Prove that cos(2x + 2y)
= cos2x cos2y + cos2 (x + y) – cos2 (x– y).
1.14 TRANSFORMATION FORMULAE
x- y 1.14.1 Transformation of products into sums or differences
22. If sin b = ,
x+ y 1. 2 sin A cos B = sin (A + B) + sin (A – B)
Êp qˆ x 2. 2 cos A sin B = sin (A + B) – sin (A – B)
then prove that tan Á + ˜ = + .
Ë 4 2¯ y 3. 2 cosA cos B = cos (A + B) + cos (A – B)
4. 2 sin A sin B = cos (A – B) – cos (A + B)
n sin a cos a
23. If tan b = , Proof: As we know that
1 - n sin 2 a
sin (A + B) = sin A. cos B + cos A. sin B (i)
prove that tan (a – b) = (1 – n) tan a .
sin (A – B) = sin A. cos B – cos A. sin B (ii)
Q sin b
24. If tan a = , Adding (i) and (ii), we get
P + Q cos b 1. 2 sinA. cosB = sin (A + B) + sin (A – B)
P sin a Subtracting (i) and (ii), we get,
prove that tan (b - a ) =
Q + P cos a 2. 2 cosA. sin B = sin (A + B) – sin (A – B)
Also, we have,
3
25. If cos(a – b) + cos(b – g) + cos(g – a) = - cos (A + B) = cosA. cosB – sinA. sinB (iii)
2
prove that cosa + cosb + cos g = 0 cos(A – B) = cosA. cosB + sinA. sinB (iv)
and sina + sinb + sing = 0. Adding, (iii) and (iv), we get,
26. If tan (a – q) = n tan (a – q), show that 3. 2 cosA. cosB = cos (A + B) + cos (A – B)
sin 2q n - 1 Subtracting (iv) from (iii), we get,
= 4. 2 sinA. sinB = cos (A – B) – cos (A + B) .
sin 2a n + 1
1.14.2 Transformations of sums or
27. If sina + sinb = a and cosa + cosb = b, differences into Products
then show that
C+D C-D
b2 - a 2 1. sin C + sin D = 2 sin cos
(i) cos (a + b ) = 2 2
b2 + a 2
C+D C-D
2ab 2. sin C – sin B = 2 cos sin
(ii) sin (a + b ) = 2 2
b + a2
2
C+D C-D
28. If a and b are the roots of 3. cos C + cos D = 2 cos cos
2 2
a cos q + b sin q = c, then prove that
C-D C-D
a 2 - b2 4. cos C – tan D = – 2 sin sin .
(i) cos (a + b ) = 2 2
a 2 + b2
Proof: As we know that,
(ii) cos (a - b ) =
(
2c - a 2 + b 2 ) sin (A + B) + sin (A – B) = 2 sin A cos B (i)
a 2 + b2 sin (A + B) – sin (A – B) = 2 cos A sin B (ii)
24 Comprehensive Trigonometry with Challenging Problems & Solutions for Jee Main and Advanced

cos (A + B) + cos (A – B) = 2 cos A cos B (iii) Ê 40∞ + 20∞ ˆ Ê 40∞ - 20∞ ˆ


+2 sin Á ˜¯ cos ÁË ˜¯
cos (A + B) – cos(A – B) = –2 sin A sin B (iv) Ë 2 2
Put A+B=C&A–B=D – sin (70°) – sin (80°)
C+D C-D = 2 sin (30°) cos (20°) + 2 sin (30°) cos (10°)
A= &B= – sin (70°) – sin (80°)
fi 2 2
= cos (20°) + cos (10°) – sin (70°) – sin (80°)
From (i), we get,
= cos (20°) + cos (10°) – cos (20°) – cos (10°)
Ê C + Dˆ Ê C - Dˆ
1. sin C + sin D = 2 sin Á ˜ cos Á = 0.
Ë 2 ¯ Ë 2 ˜¯ sin A + sin 2 A + sin 4 A + sin 5 A
From (ii), we get, Ex-4. Prove
cos A + cos 2 A + cos 4 A + cos 5 A
Ê C + Dˆ Ê C - Dˆ = tan 3 A
2. sin C - sin D = 2 cos Á sin Á
Ë 2 ˜¯ Ë 2 ˜¯
From (iii), we get, Soln. We have, sin A + sin 2 A + sin 4 A + sin 5 A
Ê C + Dˆ Ê C - Dˆ cos A + cos 2 A + cos 4 A + cos 5 A
3 cos C + cos D = 2 cos Á cos Á
Ë 2 ¯ ˜ Ë 2 ˜¯ (sin 5 A + sin A) + (sin 4 A + sin 2 A)
=
From (iv), we get, (cos 5 A + cos A) + (cos 4 A + cos 2 A)
Ê C + Dˆ Ê C - Dˆ 2 sin 3 A cos 2 A + 2 sin 3 A cos A
4. cos C - cos D = -2 sin Á
Ë 2 ˜¯
sin Á
Ë 2 ˜¯ =
2 cos 3 A cos 2 A + 2 cos 3 A cos A
sin 3 A (cos 2 A + cos A)
1.14.3 Some solved examples =
cos 3 A (cos 2 A + cos A)
Ex-1. Prove that sin (47°) + cos (77°) = cos (17°)
Soln. We have, sin (47°) + cos(77°) sin 3 A
=
= sin (47°) + sin (13°) cos 3 A
Ê 47∞ + 13∞ ˆ Ê 47∞ - 13∞ ˆ = tan 3A.
= 2 sin Á
Ë ˜¯ cos ÁË ˜¯
2 2 Hence, the result.
= 2 sin (30°) cos (17°) Ex-5. Prove that
= 2 ¥ 1 ¥ cos(17∞) cosa + cosb + cosg + cos (a + b + g)
2 Êa + bˆ Êb +g ˆ Êg +aˆ
= 4 cos Á
Ë 2 ˜¯
cos Á
Ë 2 ˜¯
cos Á
= cos (17°) Ë 2 ˜¯
Ex-2. Prove that
cos (80°) + cos (40°) – cos (20°) = 0. Soln. We have
Soln. We have, cos (80°) + cos (40°) – cos (20°) cosa + cosb + cosg + cos (a + b + g)
Ê 80∞ + 40∞ ˆ Ê 80∞ - 40∞ ˆ = (cos a + cos b) + (cos (a + b + g) + cos g)
= 2 cos Á ˜ cos Á ˜¯ – cos (20°)
Ë 2 ¯ Ë 2 Êa + bˆ Êa - bˆ
= 2 cos Á ˜ cos Á
= 2 cos (60°) cos (20°) – cos (20°) Ë 2 ¯ Ë 2 ˜¯
= 2 ¥ 1 ¥ cos(20∞) - cos(20∞) Êa + b +g +g ˆ Êa + b +g -g ˆ
2 +2 cos Á ˜ cos Á ˜¯
Ë 2 ¯ Ë 2
= cos (20°) – cos (20°)
= 0. Êa + bˆ Êa - bˆ
= 2 cos Á ˜ cos Á
Ex-3. Prove that Ë 2 ¯ Ë 2 ˜¯
sin (10°) + sin (20°) + sin (40°) + sin (50°) Ê a + b + 2g ˆ Êa + bˆ
+2 cos Á cos Á
– sin (70°) – sin (80°) = 0. Ë 2
˜
¯ Ë 2 ˜¯
Soln. We have, sin (10°) + sin (20°) + sin (40°)
+ sin (50°) – sin (70°) – sin (80°) Êa + bˆ
= 2 cos Á ¥
= {sin (50°) + sin (10°)} + {sin (40°) + sin (20°)} Ë 2 ˜¯
– sin (70°) – sin (80°)
Ï Êa - bˆ Ê a + b + 2g ˆ ¸
Ê 50∞ + 10∞ ˆ Ê 50∞ - 10∞ ˆ Ìcos Á ˜ + 2 cos Á ˜¯ ˝
= 2 sin Á ˜ cos Á ˜¯ Ó Ë 2 ¯ Ë 2 ˛
Ë 2 ¯ Ë 2
The Ratios and Identities 25

Ê a - b a + b + 2g ˆ (iv) sin 75° cos 15°


+ (v) cos 75° cos 15°
Êa + bˆ Á ˜
= 2 cos Á ˜ ¥ cos Á 2 2
˜ 2. Express as a product:
Ë 2 ¯ 2
Á ˜ (i) cosf – cos 5f
Ë ¯
(ii) cos 45° + sin 75°
Ê a - b a + b + 2g ˆ
- (iii) cos 6q – cos 8q
Á 2 2 ˜ (iv) cos 4q + cos 8q
cos Á ˜
2 (v) sin 6q – sin 2q
Á ˜
Ë ¯ 3. Prove that:
Êa + bˆ Ï Êa +g ˆ Ê b +g ˆ¸ sin 5 A - sin 3 A
¥ Ìcos cos Á ˝ (i) = tan A
Ë 2 ˜¯ Ó ÁË 2 ˜¯ Ë 2 ˜¯ ˛
= 2 cos Á
cos 5 A + cos 3 A
sin A + sin 3 A
1 1 (ii) = tan 2A
Ex-6. If sin A - sin B = and cos A - cos B = , cos A + cos 3 A
2 3
sin A + sin B Ê A + Bˆ
Ê A + Bˆ (iii) = tan ÁË ˜
tan Á cos A + cos B 2 ¯
Ë 2 ˜¯
4. Prove that :
1 (i) sin 38° + sin 22° = sin 82°
Soln. Given sin A - sin B = .............(i)
2 (ii) sin 105° + cos 105° = cos 45°
1 ..............(ii) (iii) cos 55° + cos 65° + cos 175° = 0
and cos A - cos B = (iv) cos 20° + cos 100° + cos 140° = 0
3
(v) sin 50° – sin 70 + sin 10° = 0
Dividing (i) by (ii), we get,
5. Prove that : (cosa + cosb)2 + (sina + sinb)2
sin A - sin B 1 / 2 3 Êa - bˆ
= = = 4 cos 2 Á
cos A - cos B 1 / 3 2 Ë 2 ˜¯

Ê A + Bˆ Ê A - Bˆ 6. Prove that : (cosa – cosb)2 + (sina – sinb)2


2 cos Á
Ë 2 ¯ ˜ Ë 2 ˜¯ 3
sin Á Êa - bˆ
= 4 sin 2 Á

Ê A + Bˆ
=
Ê A - Bˆ 2 Ë 2 ˜¯
-2 sin Á
Ë 2 ˜¯
sin Á
Ë 2 ˜¯ 7. Prove that: (sina – sinb)2 + (cosa – cosb)2
Êa - bˆ
Ê A + Bˆ = 4 sin 2 Á
Ë 2 ˜¯
Ë 2 ˜¯
cos Á
3
fi =- 1
Ê A + Bˆ 2 8. Prove that cos 20 co 40 cos 80 =
Ë 2 ˜¯
sin Á 8
3
9. Prove that sin 20 . sin 40 sin 80 =
Ê A + Bˆ 3 8
fi cot Á =-
Ë 2 ˜¯ 2 3
10. Prove that sin 10 sin 50 . sin 60 . sin 70 =
Ê A + Bˆ 2 16
fi tan Á =- .
Ë 2 ˜¯ 3 3
11. Prove that cos 10 . cos 30 . cos 50 . cos 70° =
16
1 1
12. If cos A + cos B = , sin A + sin B = ,
EXERCISE 6 2 4
Ê A + Bˆ 1
1. Express as a sum or difference: then prove that tan Á =
(i) 2 sin3a cos 2b Ë 2 ˜¯ 2
1 13. Prove that :
(ii) cos 10a cos 20b
4 (i) sin A + sin 3 A + sin 5 A = tan 3A
(iii) 2 sin 5qcosq cos A + cos 3 A + cos 5 A
26 Comprehensive Trigonometry with Challenging Problems & Solutions for Jee Main and Advanced

Put B = A, we get,
(ii) cos 4 x + cos 3x + cos12 x = cot 3x
sin 4 x + sin 3x + sin 2 x cos 2 A = cosA. cos A – sin A. sinA
sin A + sin 3 A + sin 5 A + sin 7 A = cos2 A – sin2 A.
(iii) = tan 4 A
cos A + cos 3 A + cos 5 A + cos 7 A = cos2 A – (1 – cos2 A)
14. If cosec A + sec A = cosec B + sec B, then = 2 cos2 I – 1
Ê A + Bˆ
prove that tan A tan B = cot Á = 2 (1 – sin2 A) – 1
Ë 2 ˜¯
= 1 – 2 sin2 A.
15. If sin 2A = l sin 2B, then prove that
tan( A + B) l + 1 3. As we know that,
=
tan( A - B) l - 1 tan A + tan B
tan ( A + B ) =
1 - tan A.tan B
16. Find the value of 3 cot(20∞) - 4 cos(20∞) .
Put B = A, we get,
17. If sin A + sin B = a and cos A + cos B = b, tan A + tan A 2 tan A
tan 2 A = =
A + B). 1 - tan A.tan A 1 - tan 2 A
1 1
18. If 2 cos A = x + , 2 cos B = y + ,
x y 1.14.3 T-ratios of angle 2A
A – B). 2 tan A
19. Prove that 4. sin2 A = ,
1 + tan 2 A
sin(47°) + sin(61°) – sin(11°) – sin(25°) = cos(7°).
m 1 1 - tan 2 A
20. If tan a = , and tan b = , 5. cos 2A =
m +1 2m + 1 1 + tan 2 A
a + b). 6. 1– cos 2A = 2sin2A,
21. Find the number of integral values of k for which 7. 1 + cos 2A = 2 cos2A
7 cosx + 5 sinx = 2k + 1 has a solution. sin 2 A
8. tan A = ,
1 + cos 2 A
1.15 MULTIPLE ANGLES 9. tan A =
1 - cos 2 A
sin 2 A
1.15.1 Definition
Proof: 4. As we know that, sin 2A
An angle of the form nA, n ΠZ is called a multiple angle = 2 sin A cos A
of A. Such as 2A, 3A, 4A etc. are each multiple angles of A. 2 sin A cos A
=
1.15.2 Trigonometrical ratios of 2A 1
in terms of t-ratio of A 2 sin A cos A
=
cos 2 A + sin 2 A
1. sin 2A = 2 sin A cos A.
2. cos 2A = cos2 A – sin2 A 2 sin A cos A
= 2 cos2 A – 1 = 1 – 2 sin2 A. = cos 2 A
2 tan A sin 2 A
3. tan 2A = 1+
1 - tan 2 A cos 2 A
Proof:
2 tan A
1. As we know that, = .
1 + tan 2 A
sin (A + B) = sinA cosB + cosA sinB
5. Also, we have, cos 2 A
Put B = A, we get, = cos2 A – sin2 A
sin 2 A = sin A . cos A. + sin A. cos A cos 2 A - sin 2 A
=
= 2 sin A. cos A. 1
2. As we know that, cos 2 A - sin 2 A
=
cos (A + B) = cos A. cos B – sin A. sin B cos 2 A + sin 2 A
The Ratios and Identities 27

sin 2 A 2 tan A + tan A - tan 3 A


1- =
= cos 2 A 1 - tan 2 A - 2 tan 2 A
sin 2 A
1+ 3 tan A - tan 3 A
cos 2 A =
1 - 3 tan 2 A
1 - tan 2 A
= .
1 + tan 2 A
6. we have, 1 – cos2 A
1.16 SOME IMPORTANT DEDUCTIONS
= 1 – (1 – 2 sin2 A) Deduction 1
= 2 sin2 A 1
sin2 A = (1 – cos2 A)
7. Also, we have, 1 + cos 2 A 2
Proof: We have,
= 1 + (2 cos2 A – 1)
1 1
= 2 cos2 A. sin2 A = (2 sin 2 A) = (1 - cos 2 A)
8. tan A Deduction 2 2 2
sin A 2 sin A cos A sin 2 A 1
= = = cos2A = (1 + cos 2A)
cos A 2 cos A2
1 + cos 2 A 2
9. tan A Proof: We have, cos 2 A
1 1
sin A 2 sin 2 A 1 - cos 2 A = (2 cos 2 A) = (1 + cos 2 A)
= = = 2 2
cos A 2 sin A cos A sin 2 A
Deduction 3
1.14.4 Trigonometrical ratios of 3A 1
in terms of t-ratio of A. sin3 A = (3 sin A – sin 3 A)
4
10. sin 3A = 3 sin A – 4 sin3 A Proof: We have, sin 3A = 3 sin A – 4 sin3 A
11. cos 3A = 4 cos3 A – 3 cos A fi 4 sin3 A = 3 sin A – sin 3A
3 tan A - tan 3 A 1
12. tan 3 A = fi sin3 A = (3 sin A - sin 3 A)
1 - 3 tan 2 A 4
Proof: 10. We have, sin 3A Deduction 4.
= sin (2A + A) 1
cos 3A = (cos 3A + 3 cos A)
= sin 2 A cos A + cos 2A . sin A
Proof: We have, cos3A =4 4 cos3A – 3 cosA
= 2 sin A .cos A.cos A + (1 – 2 sin2 A) sin A
fi 4cos3 A = cos 3A + 3 cosA
= 2 sin A . cos2 A + (1 – 2 sin2 A) . sin A
= 2 sin A (1 – sin2 A) + (1 – 2 sin2 A) . sin A 1
fi cos3 A = (cos 3 A + 3 cos A)
= 2 sin A – 2 sin3 A + sin A – 2 sin3 A 4
= 3 sin A – 4 sin3 A.
11. We have, cos 3 A Deduction 5
= cos (2A + A) 1
sin A sin (60 – A) . sin (60 + A) = sin 3 A
= cos 2A . cos A – sin 2A . sin A Proof: We have, 4
= (2 cos2 A – 1) cos A – 2 sin2 A . cos A
sin A. sin (60° – A) sin (60° + A)
= (2 cos2 A – 1) cos A – 2 cos A (1 – cos2 A)
= 2 cos3 A – cos A – 2 cos A + 2 cos3 A. = sin A. (sin2 60° – sin2 A)
= 4 cos3 A – 3 cos A. Ê3 ˆ
12. we have, tan 3 A = sin A. Á - sin 2 A˜
Ë4 ¯
= tan (2A + A)
=
tan 2 A + tan A = sin A (3 - 4 sin 2 A)
1 - tan 2 A.tan A 4
2 tan A 1
+ tan A = (3 sin A - 4 sin 3 A)
4
= 1 - tan A
2

2 tan A 1
1- .tan A = ¥ sin 3 A
1 - tan 2 A 4
28 Comprehensive Trigonometry with Challenging Problems & Solutions for Jee Main and Advanced

Deduction 6 = 1 – 2 sin2 (2A)


1 = 1 – 2 (2 sin A . cos A)2
cos A. cos (60 – A) . cos (60 + A) = cos 3A
4
= 1 – 8 sin2 A . cos2 A
Proof: We have,
= 1 – 8 sin2 A (1 – sin2 A)
cosA. cos (60° – A). cos (60° + A)
= 1 – 8 sin2 A + 8 sin4 A
= cosA. (cos260° – sin2A)
Deduction 10
Ê1 ˆ
= cos A Á - 1 + cos 2 A˜ 4 tan A - 4 tan 3 A
Ë4 ¯ tan 4 A =
1 - 6 tan 2 A + tan 4 A
Ê 3 ˆ
= cos A Á - + cos 2 A˜ Proof: We have, tan 4 A
Ë 4 ¯
= tan2.(2A)
cos A
=
4
(
. -3 + 4 cos 2 A ) =
2 tan 2 A
1 + tan 2 2 A
=
1
4
(
-3 cos A + 4 cos3 A ) 4 tan A
1 - tan 2 A
1 =
= (4 cos3 A - 3 cos A) Ê 2 tan A ˆ
2
4 1- Á
Ë 1 - tan 2 A ˜¯
1
= ¥ cos 3 A .
4 4 tan A(1 - tan 2 A)
=
(1 - tan 2 A)2 - 4 tan 2 A
Deduction 7
tan A . tan (60 – A) . tan (60 + A) = tan 3A 4 tan A - 4 tan 3 A
= .
Proof: We have, 1 - 6 tan 2 A + tan 4 A
tanA . tan (60° – A). tan (60° + A) Deduction 11
sin A.sin(60∞ - A).sin(60∞ + A) sin 5 A = 16 sin5 A – 20 sin3 A + 5 sin A
=
cos A.cos(60∞ - A).cos(60∞ + A) Proof: We have, sin 5A
1
sin 3 A = sin (3A + 2A)
= 4 = sin 3A cos 2A + cos 3A. sin 2A
1
cos 3 A = (3 sin A – 4sin3A) (1 – 2 sin 2A)
4
sin 3 A + 2(4 cos3A – 3 cos A) sin A cos A
=
cos 3 A = (3 sin A – 4sin3A) (1 – 2 sin2 A)
= tan 3A. + 2 (4 cos2A – 3)sin A cos2A

Deduction 8 = (3 sin A – 4 sin3A) (1 – 2 sin2A)


sin 4A = 4 sin cos A – 8 cos A sin3A + 2(1 – 4 sin2A) (sin A – sin3A)
Proof: We have, sin 4A = (3 sin A – 4 sin3A – 6 sin3A + 8 sin5A)
= 2 sin 2A . cos 2A + 2 (sin A – 4 sin3A – sin3A + 4 sin5A)
= 2 (2 sin A cos A) (1 – 2 sin2 A) = 5 sin A – 20 sin3 A + 16 sin5A
= 4 sin A . cos A (1 – 2 sin 2 A) = 16 sin5 A – 20 sin3 A + 5 sin A.
= 4 sin A . cos A – 8 sin 3 A . cos A Deduction 12
Deduction 9 cos 5 A = 16 cos5 A – 20 cos3 A + 5 cos A
cos 4 A = 1 – 8 sin A + 8 sin A
2 4
Proof: We have, cos 5 A
Proof: We have, cos 4A = cos (3A + 2 A)
= cos 2 (2 A) = cos 3A cos 2A – sin 3A sin 2A
The Ratios and Identities 29

= (4 cos3 A – 3 cos A) (2 cos2 A – 1) Ex-3. Prove that cotq – tanq = 2 cot(2q)


– (3 sinA – 4 sin3A) (2 sin A cos A) Soln. We have, (cotq – tanq)
= 8 cos5 A – 6 cos3 A – 4 cos3 A Ê cos q sin q ˆ
= Á -
+ 3 cos A – (3 – 4 sin2 A) 2 cos A (1– cos2 A) Ë sin q cos q ˜¯
= 8 cos5 A – 10 cos3 A + 3 cos A –
cos 2q - sin 2 q
(4 cos2 A – 1) (2 cos A – 2 cos3 A) =
sin q cos q
= 8 cos5 A – 10 cos3 A + 3 cos A – 8 cos3 A
2 cos 2q
+ 2 cos A + 8 cos5 A – 2 cos3 A =
= 16 cos5 A – 20 cos3 A + 5 cos A. 2 sin q cos q
2 cos 2q
Deduction 13 =
sin 2q
sin 6 A = (6 sin A – 32 sin3 A + 32 sin5 A) cos A.
= 2 cot2 q.
Proof: We have sin 6 A
= sin 2 (3A) b
Ex-4. If tanq = ,
= 2 sin 3A . cos 3A a
= 2 (3 sin A – 4 Sin3 A) (4 cos3 A – 3 cos A) prove that , a cos (2q) + b sin (2q) = a
= 2 (3 sin A – 4 sin3 A) (1 – 4 sin2 A) cos A Soln. We have, a cos(2q) + b sin (2q)
= 2 (3 sin A – 4 sin3A – 12sin3A + 16 sin 5 A) cosA Ê 1 - tan 2 q ˆ Ê 2 tan q ˆ
= aÁ + bÁ
= 2 (3 sin A – 16 sin3 A + 16 sin5 A) cos A 2 ˜
Ë 1 + tan q ¯ Ë 1 + tan 2 q ˜¯
= (6 sin A – 32 sin3A + 32 sin5A) cos A.
Ê b2 ˆ Ê b ˆ
1 -
Á a ˜ + b ÁÁ a ˜˜
2˜ 2
Deduction 14
= aÁ
cos 6 A = 32 cos6 A – 48 cos4 A + 18 cos2 A – 1 Á b2 ˜ Á b2 ˜
ÁË 1 + ˜ ÁË 1 + ˜
Proof: We have, cos 6A a2 ¯ a2 ¯
= cos 2 (3A) Ê a 2 - b2 ˆ Ê 2ab ˆ
= 2 cos2 (3A) – 1 = aÁ 2 2˜
+ bÁ 2
Ë a + b 2 ˜¯
Ëa +b ¯
= 2 (4 cos3 A – 3 cos A)2 – 1
= 2 (16 cos6 A - 24 cos4 A + 9 cos2 A) – 1 a(a 2 - b 2 + 2b 2 )
=
= 32 cos6 A – 48 cos4 A + 18 cos2 A – 1 a 2 + b2
a(a 2 + b 2 )
=
1.16.1 Some solved examples a 2 + b2
Ê 1 - cos 2q ˆ = a.
Ex-1. Prove that Á = tan q
Ë sin 2q ˜¯ Ex-5. Prove that 3 cos ec (20∞) - sec(20∞) = 4
Soln. We have, Soln. We have, 3 cos ec (20∞) - sec(20∞)
1 - cos 2q 2 sin q
2
3 1
= = -
sin 2q 2 sin q cos q sin(20∞) cos(20∞)
sin q
= = tanq
cos q Ê 3 ˆ
Ê 1 + cos 2q ˆ

Ë 2
( )
1
cos 20O - sin 20O ˜
2 ¯
( )
= cot q =
2sin ( 20 ) cos (20 )
Ex-2. Prove that Á
Ë sin 2q ˜¯ Ÿ O

Soln. We have,
Ê 3 1 ˆ
4Á cos(20∞) - sin(20∞)˜
1 + cos 2q 2 cos q
2
Ë 2 2 ¯
= =
sin 2q 2 sin q cos q 2 sin (20∞) cos(20∞)
cos q 4(sin(60∞) cos(20∞) - cos(60∞)sin( 20∞))
= = cotq. =
sin q 2 sin (20∞) cos(20∞)
30 Comprehensive Trigonometry with Challenging Problems & Solutions for Jee Main and Advanced

4(sin(60∞ - 20∞)) 2 sin 4 A cos 4 A sin 4 A


= = ¥
sin (40∞) cos 8 A 2 sin 2 2 A
= 4. sin 8 A 2 sin 2 A cos 2 A
= ¥
Ex-6. Prove that cos 8 A 2 sin 2 2 A
tan(9°) – tan(27°) – tan(63°) + tan(81°) = 4 = sin 8 A ¥ cos 2 A
cos 8 A sin 2 A
Soln. We have,
= tan 8A ´ cot 2A
tan(9°) – tan(27°) – tan(63°) + tan(81°)
= {tan(9°) + tan(81°)} – {tan(27°) + tan(63°)} = tan 8 A .
tan 2 A
= {tan(9°) + cot(9°)} – {tan(27°) + cot(27°)} Ex-8. Prove that
Ï sin(9∞) cos(9∞) ¸ Ï sin(27∞) cos(27∞) ¸ tanq + 2tan (2q) + 4tan (4q) + 8cot8q = cotq
= Ì + ˝-Ì + ˝
Ó cos(9∞) sin(9∞) ˛ Ó cos(27∞) sin(27∞) ˛ Soln. We have,
ÏÔ sin (9∞) + cos (9∞) ¸Ô ÏÔ sin (27∞) + cos (27∞) ¸Ô
2 2 2 2
tan q + 2 tan ( 2q ) + 4 tan ( 4q ) + 8 cot 8q
= Ì ˝-Ì ˝
ÓÔ sin(9∞) cos(9∞) ˛Ô ÓÔ sin(27∞) cos(27∞) ˛Ô = cot q - (cot q - tan q ) + 2 tan 2q
Ï 2 ¸ Ï 2 ¸ + 4 tan 4q + 8 cot 8q
= Ì ˝-Ì ˝ = cot q - 2 cot 2q + 2 tan 2q + 4 tan 4q + 8 cot 8q
Ó 2 sin(9∞) cos(9∞) ˛ Ó 2 sin(27∞) cos(27∞) ˛
Ï 2 ¸ Ï 2 ¸ = cot q - 2 (cot 2q - tan 2q ) + 4 tan 4q + 8 cot 8q
= Ì ˝-Ì ˝ = cot q - 4 cot 4q + 4 tan 4q + 8 cot 8q
Ó sin(18∞) ˛ Ó sin(54∞) ˛
= cot q - 4 (cot 4q - tan 4q ) + 8 cot 8q
Ï ¸ Ï ¸
Ô Ô Ô = cot q - 8 cot 8q + 8 cot 8q
Ô 2 Ô Ô 2 ÔÔ
= Ì ˝-Ì ˝ = cot q.
Ô Ê 5 - 1ˆ Ô Ô 5 + 1 Ô
Ô ÁË 4 ˜¯ Ô ÔÓ 4 Ô˛ Ex-9. Prove that
Ó ˛ Ê 2p ˆ Ê 2p ˆ 3
cos 2 (q ) + cos 2 Á - q ˜ + cos 2 Á + q˜ =
Ï 8 ¸ Ï 8 ¸ Ë 3 ¯ Ë 3 ¯ 2
Ì ˝-Ì ˝ Soln. We have,
= Ó 5 - 1˛ Ó 5 + 1˛
Ê 2p Ê 2p
Ô (
Ï8 5 + 1 - 5 + 1 ¸
Ô ) cos 2 (q ) + cos 2 Á
Ë 3
ˆ
- q ˜ + cos 2 Á
¯ Ë 3
ˆ
+ q˜
¯
= Ì 5 -1
˝
ÔÓ Ô˛ 1Ê Ê 2p ˆ Ê 2p ˆˆ
= Á 2 cos 2 (q ) + 2 cos 2 Á - q ˜ + 2 cos 2 Á + q˜˜
= 4. 2Ë Ë 3 ¯ Ë 3 ¯¯
Ê sec 8 A - 1ˆ tan 8 A
Ex-7. Prove that Á =
Ë sec 4 A - 1˜¯ tan 2 A =
1
2
(1 + cos (2q )) + 12 ÊÁË1 + cos ÊÁË 43p - 2q ˆ˜¯ ˆ˜¯
Ê sec 8 A - 1ˆ 1Ê Ê 4p ˆˆ
Soln. We have, Á
Ë sec 4 A - 1˜¯ + Á

1 + cos Á
Ë 3
+ 2q ˜ ˜
¯¯
1
-1 1Ê Ê Ê 4p ˆ Ê 4p ˆˆˆ
3 + Á cos 2 q + cos Á - 2q ˜ + cos Á + 2q ˜ ˜ ˜
= cos 8 A = Á
2Ë Ë Ë 3 ¯ Ë 3 ¯¯¯
1
-1
cos 4 A 1Ê Ê Ê 4p ˆ ˆˆ
= Á 3 + Á cos 2q + 2 cos Á ˜ cos ( 2q )˜ ˜
= 1 - cos 8 A ¥ cos 4 A 2Ë Ë Ë 3¯ ¯¯
1 - cos 4 A cos 8 A
1Ê Ê Ê 1ˆ ˆˆ
2 sin 2 4 A
cos 4 A = Á 3 + Á cos 2q + 2 Á - ˜ cos ( 2q )˜ ˜
= ¥ 2Ë Ë Ë 2¯ ¯¯
2 sin 2 A cos 8 A
2
3
= .
2
The Ratios and Identities 31

Ex-10 Prove that 1


3 = (sin(60∞))
sin q + sin (120∞ + q ) + sin (120∞ + q ) =
2 2 2 4
2
3
Soln. We have, = .
8
sin2q + sin2 (120° + q) + sin2 (240° + q)
Ex-13. Prove that
1Ê Ê 2p ˆˆ 4 cos(q) . cos(60° – q). cos(60° + q)
= Á 2 sin 2 q + 2 sin 2 Á + q˜˜
2Ë Ë 3 ¯¯
= cos(3q)
1Ê Ê 4p ˆˆ Soln. We have,
+ Á 2 sin 2 Á + q˜˜
2Ë Ë 3 ¯¯ 4 cos(q). cos(60° – q). cos(60° + q)
1Ê Ê 4p ˆˆ = 4 cos(q). (cos(60° – q). cos(60° + q))
= Á (1 - cos 2q ) + Á1 - cos ÊÁË + 2q ˆ˜¯ ˜ ˜ = 4 cos(q). (cos2(60°) – sin2 q)
2Ë Ë 3 ¯¯
Ê1 ˆ
1 ÊÊ Ê 8p ˆˆˆ = 4 cos (q ) . Á - 1 + cos 2 q ˜
+ Á Á1 - cos Á + 2q ˜ ˜ ˜ Ë4 ¯
2 ËË Ë 3 ¯¯¯
Ê 3 ˆ
= 4 cos (q ) . Á - + cos 2 q ˜
3 1 1 È Ê 4p ˆ Ê 8p ˆ˘ Ë 4 ¯
= - cos 2q - Ícos Á + 2q ˜ + cos Á + 2q ˜ ˙
2 2 2Î Ë 3 ¯ Ë 3 ¯˚ = cos(q) . (–3 + 4 cos2q)
= 3 - 1 cos 2q - 1 (2 cos(120∞) cos 2q ) = (4 cos3q – 3cos(q))
2 2 2 = cos(3q).
= 3 - 1 cos 2q + 1 cos (2q ) Ex-14. Prove that
2 2 2 3
cos (10∞). cos(50∞). cos(70∞) =
3 8
= .
2 Soln. We have, cos(10°) . cos(50°) . cos(70°)
Ex-11. Prove that = cos(10°) cos(60° – 10°) cos(60° + 10°)
4sin (q) sin (60° + q) sin (60° – q) = sin 3q =
1
(4 cos(10∞).cos(60∞ - 10∞).cos(60∞ + 10∞))
Êp ˆ Êp ˆ 4
Soln. We have, 4 sin (q ) sin Á - q ˜ sin Á + q ˜ 1
Ë3 ¯ Ë3 ¯ = (4 cos(3.10∞))
4
Ê Êp ˆ Êp ˆˆ
= 4 sin (q ) ¥ Á sin Á - q ˜ sin Á + q ˜ ˜ 3
Ë Ë3 ¯ Ë3 ¯¯ = .
8
Ê Êpˆ ˆ
= 4 sin (q ) ¥ Á sin 2 Á ˜ - sin 2 q ˜ Ex-15. Prove that
Ë Ë 3¯ ¯ tan(q) + tan (60° + q) – tan (60° – q)
Ê3 ˆ
= 4 sin (q ) ¥ Á - sin 2 q ˜
= 3tan(3q)
Ë4 ¯ Soln. We have,
Ê3 ˆ tan (q) – tan (60° – q) + tan (60° + q)
= 4 ¥ Á sin q - sin 3 q ˜
Ë4 ¯ tan(60∞) - tan (q )
= tan (q ) -
= (3 sinq – 4 sin q) 1 + tan(60∞).tan (q )
3

= sin (3q) tan(60∞) + tan (q )


+
Ex-12. Prove that sin (20°) sin (40°) sin (80°) =
3 1 - tan(60∞).tan (q )
8
Soln. We have, sin (20°) sin (40°) sin (80°) 3 - tan (q ) 3 + tan (q )
= tan (q ) - +
1 1 + 3.tan (q ) 1 - 3.tan (q )
= (sin(3.20∞))
4
32 Comprehensive Trigonometry with Challenging Problems & Solutions for Jee Main and Advanced

- 3 + 3 tan (q ) + tan (q ) - 3 tan 2 (q ) 1 Ê Êpˆ Ê p ˆˆ Ê 2p ˆ Ê 4p ˆ


= - Á 2 sin Á ˜ cos Á ˜ ˜ cos Á ˜ cos Á ˜
+ 3 + 3 tan (q ) + tan (q ) + 3 tan 2 (q ) Êpˆ Ë Ë 7¯ Ë 7 ¯¯ Ë 7¯ Ë 7 ¯
2 sin Á ˜
Ë 7¯
= tan(q)
(1 - 3 tan 2 q )
1 Ê Ê 2p ˆ Ê 2p ˆ ˆ Ê 4p ˆ
8 tan (q ) = - Á 2 sin Á ˜ cos Á ˜ ˜ cos Á ˜
Ë 7¯ Ë 7 ¯¯ Ë 7¯
= tan (q ) + Êpˆ Ë
22 sin Á ˜
1 - 3 tan 2 (q ) Ë 7¯
tan (q ) - 3 tan 3 (q ) + 8 tan (q ) 1 Ê Ê 4p ˆ Ê 4p ˆ ˆ
= = - Á 2 sin Á ˜ cos Á ˜ ˜
1 - 3 tan (q )
2 p
Ê ˆ
23 sin Á ˜
Ë Ë 7 ¯ Ë 7 ¯¯
Ë 7¯
Ê 3 tan (q ) - tan 3 (q ) ˆ
= 3 ¥ Á ˜ Ê 8p ˆ
Ë 1 - 3 tan (q ) ¯
2 sin Á ˜
Ë 7¯
= -
= 3 ¥ tan (3q). Êpˆ
23 sin Á ˜
Ex-16. Prove that Ë 7¯
cos(q) cos(2q) cos(22q) cos(23q)... cos(2n–1q) Ê pˆ

=
( )
sin 2 q n
= -
sin Á p + ˜
Ë 7¯
2 sin q n p
Ê ˆ
23 sin Á ˜
Ë 7¯
Soln. We have,
cos(q) cos(2q) cos(22q) cos(23q)... cos(2n–1q) Êpˆ
sin Á ˜
Ë 7¯
=
1
2 sin q
( ( ) ( ))
(2 sin q cos q ) cos 2q .cos 22 q ...cos 22n-1q =
Êpˆ
8 sin Á ˜
Ë 7¯
= 2
1
2 sin q
( ( ) ( ))
(2 sin 2q cos 2q ) cos 22 q .....cos 22n-1q
=
1
.
8
= 3
1
2 sin q
( ( ) ( ))
(2 sin 4q cos 4q ) cos 23q .....cos 22n-1q Ê 2p ˆ Ê 4p ˆ Ê 6p ˆ
Ex-18. Prove that cos Á ˜ + cos Á ˜ + cos Á ˜ = -
1
Ë 7¯ Ë 7¯ Ë 7¯
) ( ( ) ( ))
2
= 4
1
2 sin q
(
2 sin 23 q cos 23 q cos 24 q .....cos 22 n -1q
Ê 2p ˆ Ê 4p ˆ Ê 6p ˆ
Soln. Let z = cos Á ˜ + cos Á ˜ + cos Á ˜
.............. ........... .............. Ë 7¯ Ë 7¯ Ë 7¯
.............. ........... .............. Êpˆ
fi 2 z sin Á ˜
Ë 7¯
=
2 n
1
sin q
(2 sin 2 n -1
q cos 2n -1q ) Êpˆ Ê 2p ˆ Êpˆ Ê 4p ˆ
= 2 sin Á ˜ cos Á ˜ + 2 sin Á ˜ cos Á ˜
Ë 7¯ Ë 7¯ Ë 7¯ Ë 7¯
= n
2 sin q
1
(sin 2 q )
n
Êpˆ Ê 6p ˆ
+2 sin Á ˜ cos Á ˜
Ë 7¯ Ë 7¯
sin(2n q )
= Ê 3p ˆ Êpˆ Ê 5p ˆ Ê 3p ˆ
2n sin q = sin Á ˜ - sin Á ˜ + sin Á ˜ - sin Á ˜
Ë 7¯ Ë 7¯ Ë 7¯ Ë 7¯
Ê 2p ˆ Ê 4p ˆ Ê 8p ˆ 1
Ex-17. Prove that cos Á ˜ cos Á ˜ cos Á ˜ = Ê 7p ˆ Ê 5p ˆ
Ë 7¯ Ë 7¯ Ë 7¯ 8 + sin Á ˜ - sin Á ˜
Ë 7¯ Ë 7¯
Ê 2p ˆ Ê 4p ˆ Ê 8p ˆ
Soln. We have cos Á ˜ cos Á ˜ cos Á ˜
Ë 7¯ Ë 7¯ Ë 7¯ Êpˆ
= - sin Á ˜
Ë 7¯
Ê 2p ˆ Ê 4p ˆ Ê pˆ
= cos Á ˜ cos Á ˜ cos Á p + ˜ Êpˆ Êpˆ
Ë 7¯ Ë 7¯ Ë 7¯ Thus, 2 z sin Á ˜ = - sin Á ˜
Ë 7¯ Ë 7¯
Êpˆ Ê 2p ˆ Ê 4p ˆ 1
= - cos Á ˜ cos Á ˜ cos Á ˜ fi z=-
Ë 7¯ Ë 7¯ Ë 7¯ 2
The Ratios and Identities 33

Ê 2p ˆ Ê 4p ˆ Ê 6p ˆ 1 1 + sin 2q + cos 2q
fi cos Á ˜ + cos Á ˜ + cos Á ˜ = - . 3. = cot q
Ë 7¯ Ë 7¯ Ë 7¯ 2 1 + sin 2q - cos 2q
4. cotq – tanq = 2 cot 2q
Ex-19. If M = a cos q + b sin q
2 2 2 2

5. 2 + 2 + 2 cos 4q = 2 cos q
+ a 2 sin 2 q + b 2 cos 2 q
6. Prove that:
such that max (M2) = m1 and min (M2) = m2,
m1 – m2. cosec 10° – 3 sec 10° = 4.
b
Soln. Given 7. If tanq = , then prove that
a
M = a 2 cos 2 q + b 2 sin 2 q + a 2 sin 2 q + b 2 cos 2 q
a cos 2q + b sin 2q = a
fi M2 = a2 cos2q + b2 sin2q + a2 sin2q + b2 cos2q +
Q. Prove that:
Êpˆ Ê 3p ˆ Ê 5p ˆ Ê 7p ˆ
2 (a 2
)(
cos 2 q + b 2 sin 2 q a 2 sin 2 q + b 2 cos 2 q ) 8. cos 2 Á ˜ + cos 2 Á ˜ + cos 2 Á ˜ + cos 2 Á ˜ = 2
Ë 8¯ Ë 8¯ Ë 8¯ Ë 8 ¯

fi M2= a 2 + b 2 + 2 (a 2
cos 2 q + b 2 sin 2 q ) Êpˆ Ê 3p ˆ Ê 5p ˆ Ê 7p ˆ
9. sin 2 Á ˜ + sin 2 Á ˜ + sin 2 Á ˜ + sin 2 Á ˜ = 2
Ë 8¯ Ë 8¯ Ë 8¯ Ë 8 ¯
(a 2 sin 2 q + b 2 cos 2 q )
sec 8 A - 1 tan 8 A
10. Prove that: =
(
fi M = a + b + 2 È a 4 + b 4 sin 2 q cos 2 q
2
Î
2 2
) sec 4 A - 1 tan 2 A
11. If cosec 2A + cosec 2 B + cosec 2 C = 0, then prove
( )
1/ 2
fi a b sin q + cos q ˘
2 2 4 4 that
˚ tan A + tan B + tan C + cot A + cot B + cot C = 0
(
M2 = a 2 + b 2 + 2 È a 4 + b 4 sin 2 q cos 2 q
Î ) 12. If tan 25° = a, then prove that:
tan 155∞ - tan 115∞ 1 - a2
=
( ) 1 + tan 155∞.tan 115∞ 2a
1/ 2
fi + a 2b 2 1 - 2sin 2 q cos 2 q ˘
˚
13. If tan2q = 2 tan2f + 1, then prove that
(( ) ) cos 2q + sin2f = 0
1/ 2
fi M2= a 2 + b 2 + 2 a 4 + b 4 - 2a 2b 2 sin 2 q cos 2 q + a 2b 2
14. If 2 tana = 3 tan b, then prove that:
sin 2b
fi M2= a 2 + b 2 + (4 (a 4
)
+ b 4 - 2a 2b 2 sin 2 q cos 2 q + 4a 2b 2 ) tan (a – b) =
5 - cos 2b
Thus,max (M2) = a2 + b2 + (a2 + b2) = 2(a2 + b2) 1
15. If a + = 2 cosq, then prove that :
a
and min (M2) = a2 + b2 + 2ab = (a + b)2
1
Hence, the value of m1 – m2 a4 + 4 = 2 cos 4q.
a
16. Prove that:
= max (M2) – min (M2)
tan a + 2 tan 2a + 4 tan 4a + 8 cot 8a = cot a
= 2(a2 + b2) – (a + b)2. 17. Prove that 3(sin x – cos x)4 +
6(sin x + cos x)2 + 4(sin6 x + cos6 x) = 13.
18. If a and b are the roots of
3 cosx + 4 sin x cot (a + b )
EXERCISE 7
19. If a and b are the solutions of the equation
Q. Prove that: a tan q + b cot q = c tan (a + b ).
sin 2q 20. If tan b = 3 tan a , then prove that
1. = tan q
1 + cos 2q 2 sin 2b
tan (a + b ) =
cos 2q Êp ˆ 1 + 2 cos 2b
2. = tan ÁË - q ˜¯ 21. Find the value of
1 + sin 2q 4
cot(91∞) cot(92∞) cot(93∞)....cot(179∞) .
34 Comprehensive Trigonometry with Challenging Problems & Solutions for Jee Main and Advanced

22. Find the value of 38. Prove that:


Ê Ê p ˆˆ Ê Ê 3p ˆ ˆ Ê Ê 5p ˆ ˆ Ê Ê 7p ˆ ˆ (1 + sec2¸ ) (1 + sec22¸ ) (1 + sec 23¸ )...(1 + sec 2n ¸ ) .
ÁË1 + cos ÁË 8 ˜¯ ˜¯ ÁË1 + cos ÁË 8 ˜¯ ˜¯ ÁË1 + cos ÁË 8 ˜¯ ¯˜ ÁË1 + cos ÁË 8 ˜¯ ˜¯
39. Prove that:
23. If a + b = 90∞
value of cosa cosb (1 + sec 2q ) (1 + sec 22 q ) (1 + sec 23q )...(1 + sec 2n q )
24. Prove that:
tan 2n q
Ê 2p ˆ Ê 2p ˆ 3 =
cos 2 q + cos 2 Á - q ˜ + cos 2 Á + q˜ = tan q
Ë 3 ¯ Ë 3 ¯ 2
25. Prove that:
Êp ˆ
cos 2 q + cos 2 Á - q ˜ + cos 2 Á + q ˜ =
Êp ˆ 3 1.17 THE MAXIMUM AND MINIMUM
Ë3 ¯ Ë3 ¯ 2
VALUES OF f (x) = a cos x
26. Prove that: 3 cosec20∞ - sec 20∞ = 4 + b sin x + c
We have f ( x ) = a cos x + b sin x + c
1
27. Prove that: sinq cos3q – sin3q cosq = sin 3q
28. Prove that: 4 Let a = r sin q & b = r cos q
Êp ˆ Êp ˆ a
tan q + tan Á + q ˜ + tan Á - q ˜ = 3 tan (3q ) Then r = a 2 + b 2 and tan (q ) =
Ë3 ¯ Ë3 ¯ b
29. Prove that: Now, f (x) = a cos x + b sin x + c
Êp ˆ Ê 2p ˆ = r (sin q cos x + cos q sin x )
cot q + cot Á + q ˜ + cot Á + q ˜ = 3 cot 3q
Ë3 ¯ Ë 3 ¯ = r sin (q + x )
4 tan q - 4 tan 3 q As we know that, -1 £ sin (q + x ) £ 1
30. Prove that: tan 4q = .
1 - 6 tan q + tan q 2 4 fi - r + c £ r sin (q + x ) + c £ r + c
Ê 2p ˆ Ê 4p ˆ fi -r + c £ f ( x) £ r + c
31. Prove that: sin 3 a + sin 3 Á + a ˜ + sin 3 Á + a˜
Ë 3 ¯ Ë 3 ¯
fi - a + b + c £ f ( x) £ a 2 + b2 + c
2 2

3
= - sin 3a . Thus, the maximum value of
4
f (x) isa 2 + b2 + c
32. Prove that:
and the minimum values of f (x) is - a 2 + b 2 + c .
( ) ( )
2n cos (q ) cos ( 2q ) .cos 22 q .cos 23 q ...cos 2n -1q ( )
( )
Ex-1. Find the max and min values of
sin 2n q
= f (x) = 3 sin x + 4 cos x + 10.
sin q
Soln. Here, a = 3 , b = 4 and c = 10
p Thus, the minimum values of f (x)
33. If q = , then prove that
n
2 +1
n n -1
= - a 2 + b 2 + c = – 5 + 10 = 5
2 cos q .cos 2q . cos 2 q ....cos 2 q = 1 .
2
and the maximum values of
34. If cos 6q = A cos6 q + B cos 4 q +C cos 2 q + D ,
A + B + C + D + 2. f (x) =
a 2 + b 2 + c = 5 + 10 = 15 .
35. If sin 5q = A sin q + B sin 3 q + C sin 5 q Ex-2. Find the range of f (x) = sin x + cos x + 3
value of A + B + C + 10. Soln. R f = ÈÎmin f ( x ) , max f ( x )˘˚
36. Prove that:
Ê sin x sin 3x sin 9 x ˆ 1 = ÈÎ- 2 + 3, 2 + 3˘˚
ÁË + + ˜ = ( tan 27 x - tan x )
cos 3x cos 9 x cos 27 x ¯ 2
Ex-3. Let A = sin 4 q + cos 4 q A
Ê 2p ˆ Ê 4p ˆ Ê 6p ˆ 1 Soln. We have , A = sin 4 q + cos 4 q
37. Prove that cos Á ˜ + cos Á ˜ + cos Á ˜ = - .
Ë 7¯ Ë 7¯ Ë 7¯ 2
( ) + (cos q )
2 2
= sin q
2 2
The Ratios and Identities 35

Soln. Now, A
(sin2)2
= 1- = cos 2 q + sin 4 q
2
(sin 2q )2 1
( 1
) ( )
2
= 1- = 2 cos 2 q + 2 sin 2 q
2 2 4

= 1+
{- sin 2
(2q ) } = (1 + cos ( 2q )) + (1 - cos ( 2q ))
1 1 2

2 2 4

As we know that, -1 £ - sin ( 2q ) £ 0


2
{ } 1
2
1
4
(
= (1 + cos ( 2q )) + 1 - 2 cos ( 2q ) + cos 2 ( 2q ) )

1
- £
- sin 2 { (2q ) }
£0
1 1
2 2
1 1
4 2
1
= + cos (2q ) + - cos (2q ) + cos 2 ( 2q )
4
( )
2 2
1 {
- sin 2( 2q ) }
3 1
(
= + cos 2 ( 2q )
4 4
)
fi - +1£ £ 0 +1
2 2 3 1
1 Max value of A = m1 = + .1 = 1
fi £ A £1. Also, B 4 4
2
Ex-4. Find the max and min values of = sin 2 q + cos 4 q
f (q ) = sin 6 q + cos6 q
Soln. We have, f (q ) = sin q + cos q
6 6
=
1
2
( 1
4
) (
2 sin 2 q + 4 cos 4 q )
= (sin q ) + (cos q )
1
( 1
) ( )
3 3 2
2 2 = 2 sin 2 q + 2 cos 2 q
2 4

= (sin q + cos q ) - 3 sin q cos q (sin )


2
(1 - cos ( 2q )) + (1 + cos ( 2q ))
2 2 2 2 2
q + cos 2 q 1 1 2
=
2 4
= 1 - 3 sin q cos q
2 2

=
1
( 1
(
1 - cos ( 2q )) + 1 + 2 cos ( 2q ) + cos 2 ( 2q ) )
3
(
= 1 - 4 sin 2 q cos 2 q
4
) 2
1 1 1
4

= + + cos 2 ( 2q )
3
(
= 1 - sin 2 2q
4
) 2 4 4
3 1
= + cos 2 ( 2q )
3
(
= 1 + - sin 2 2q
4
) 4 4
Thus, the minimum value of B
As we know that, -1 £ - sin 2q £ 0
2
( ) = m2 =
3 1
+ .0 =
3


3 3 - sin
- £
2
( 2q )
£0
4 4
Now, the value of
4

4 4
m12 + m22 + m1m2
fi -
3 3 - sin 2q
£
(
£0
2
) 9 3
4 4 = 1+ +
16 4
1
fi £ f (q ) £ 1 =
37
4 16
Hence, the maximum value = 1 and
1 Ex-9. Find the minimum value of
the minimum value = .
4 x 2 sin 2 x + 4
f ( x) = ,
Ex-5. If A = cos 2 q + sin 4 q and x sin x
B = cos 4 q + sin 2 q such that Ê pˆ
where x ŒÁ 0, ˜
m1 = Max of A and m2 = Min of B Ë 2¯
m12 + m22 + m1m2
36 Comprehensive Trigonometry with Challenging Problems & Solutions for Jee Main and Advanced

4. Find the max and min values


x 2 sin 2 x + 4
Soln. We have, f ( x ) = of f ( x ) = 3 sin x + 5
x sin x
5. Find the greatest and the least values of
4
= x sin x + ≥4 2 sin2q + 3 cos2q.
x sin x
6. Find the least value of
Hence, the minimum values of f (x) is 4. cosec2 x + 25 sec2 x.
Ex-10. Find the minimum value of 7. Find the ratio of the greatest value of

f (a, b, c, d ) =
(a + 1) (b + 1) (c + 1) (d
2 2 2 2
+1 ) 8.
2 – cos x + sin2 x to its least value.
If y = 4 sin2 q – cos2 q, then y lies in the
abcd interval.............
9. If m is the minimum value of g ( x ) = 3 - 2sin x
where a, b, c, d > 0 and n is the maximum value of ( m + n + 2)
Soln. We have f (a, b, c, d ) m + n + 2).
10. Find the maximum and minimum values of

=
(a + 1) (b + 1) (c + 1) (d
2 2 2 2
)
+1
11.
f (x) = sin4x + cos4 x.
Find the maximum and minimum values of
abcd f (x) = sin6 x + cos6 x.

=
(a + 1) ¥ (b + 1) ¥ (c + 1) ¥ (d
2 2 2 2
+1 ) 12. Find the max and min values of
f ( x ) = (sin x + cos x + cos ec 2 x )
3
a b c d
Ê 1ˆ Ê 1ˆ Ê 1ˆ Ê 1ˆ Ê pˆ
= ÁË a + ˜¯ ÁË b + ˜¯ ÁË c + ˜¯ ÁË d + ˜¯ where x ŒÁË 0, ˜¯
a b c d 2
13. Find the max and min values of
≥ 2.2.2.2 = 16
f ( x ) = log x y + log y x .
Hence, the minmimum value is 16.
14. Find the max and min values of
5
f ( x) =
sin q - 6 sin q cos q + 3 cos 2 q
2
EXERCISE 8
15. Find the maximum and the minimum values of
1. Find the maximum and minimum values of
f ( x ) = sin 2 x + cos 4 x
(i) f (x) = 3 sinx + 4 cosx + 5
(ii) f (x) = 3 sin (100)x + 4 cos (100)x + 10 16. Find the maximum and the minimum values of
(iii) f (x) = 3 sinx + 4 f ( x ) = cos 2 x + sin 4 x
(iv) f (x) = 2 cosx + 5
17. Find the minimum value of
(v) f (x) = sinx + cosx
a2 b2 Ê pˆ
(vi) f (x) = sinx – cosx f ( x) = + 2 ,
x ŒÁ 0, ˜
cos x sin x
2 Ë 2¯
(vii) f (x) = sin (sin x)
(viii) f (x) = cos (cos x) 18. Find the minimum values of
(ix) f (x) = sin (sin x) + cos (sinx) f ( x ) = 2 log10 x - log x (0.01) , x > 1
(x) f (x) = cos (sin x) + sin (cos x). 19. Find the minimum value of
2. Find the max and min value of
f ( x, y , z ) =
( x + 1) ( y + 1) ( z + 1) , x, y, z > 0
2 2 2

3 sin2x + 4cos 2x + 3 .
xyz
3. Prove that
20. Find the minimum value of

Ê
–4 £ 5 cosq + 3 cos Á q + ˜ + 3 £ 10.
Ë 3¯ f ( x, y , z ) =
(x 3
)( )(
+ 2 y3 + 2 z3 + 2 ) , x, y , z > 0
.
xyz
The Ratios and Identities 37

1.18 SUB–MULTIPLE ANGLES fi 2 sin A cos A = 4 cos3 A - 3 cos A


1.18.1 Definition fi 2 sin A = 4 cos 2 A - 3
A fi 2 sin A = 4 - 4 sin 2 A - 3 = 1 - 4 sin 2 A
An angle is of the form , n Œ Z (π 0), is called a sub-
n
A A A A fi 4 sin 2 A + 2 sin A - 1 = 0
multiple angle of A. Thus, , , , etc. are each a
sub-multiple angle of A. 2 3 4 5
fi -2 ± 20 -2 ± 2 5
sin A = =
8 8
Ê Aˆ Ê Aˆ -1 ± 5
1.18.2 T-ratios of angle Á ˜ and Á ˜ fi sin A =
Ë 2¯ Ë 3¯ 4
Ê Aˆ 5 -1 - 5 -1
Ê Aˆ Ê Aˆ
2 tan Á ˜
Ë 2¯ fi sin A = ,
1. sin A = 2 sin ÁË ˜¯ cos ÁË ˜¯ = 4 4
2 2 Ê Aˆ
1 + tan 2 Á ˜ 5 -1
Ë 2¯ fi sin (18∞) = ,
4
2 Ê Aˆ 2 Ê Aˆ
2. cos A = cos ÁË ˜¯ - sin ÁË ˜¯
2 2
1
2ÊAˆ 2. cos 18° = 10 + 2 5
= 2 cos ÁË ˜¯ - 1 4
2
Proof: We have, cos(18°)
2Ê Aˆ
1 - tan Á ˜

2Ê Ë 2¯ = 1 - sin 2 (18∞)
= 1 - 2 sin Á ˜ =
Ë 2¯ Ê Aˆ
1 + tan 2 Á ˜ Ê 5 - 1ˆ
2
Ë 2¯ = 1- Á ˜
Ë 4 ¯
Ê Aˆ
2 tan Á ˜
Ë 2¯ Ê 5 +1- 2 5ˆ
3. tan A = . = 1- Á
2 Ê Aˆ ˜
1 - tan Á ˜ Ë 16 ¯
Ë 2¯
Ê Aˆ Ê 16 - 5 - 1 + 2 5 ˆ
3 Ê Aˆ =
4. sin A = 3 sin ÁË ˜¯ - 4 sin ÁË ˜¯ Á
Ë 16 ˜
¯
3 3
3 Ê Aˆ Ê Aˆ 1
5. cos A = 4 cos ÁË ˜¯ - 3 cos ÁË ˜¯ = 10 + 2 5
3 3 4
Ê Aˆ Ê Aˆ 5 -1
3 tan Á ˜ - tan 3 Á ˜ 3. tan 18° =
Ë 3¯ Ë 3¯ 10 + 2 5
6. tan A =
Ê Aˆ
1 - 3 tan 2 Á ˜ Proof: We have, tan(18°)
Ë 3¯
sin (18∞)
=
1.18.3 Values of sin 18°, cos 18° and tan 18° cos (18∞)

Ê 5 - 1ˆ Ê 5 - 1ˆ
1. sin(18∞) = Á 4 ˜ Á 4 ˜
Ë ¯
Ë ¯ =
Proof: Let A = 18° 10 + 2 5
4
fi 5 A = 90∞
Ê 5 -1 ˆ
fi 2 A = 90∞ - 3 A Á ˜
=
fi sin 2 A = sin(90∞ - 3 A) = cos 3 A Ë 10 + 2 5 ¯
38 Comprehensive Trigonometry with Challenging Problems & Solutions for Jee Main and Advanced

Notes: Proof: We have tan(36∞)


1
(i) sin 72° = cos 18° = 10 + 2 5 sin(36∞)
4 =
cos(36∞)
5 +1
(ii) cos 72° = sin 18° = 10 - 2 5
4
= 4
1.18.4 Values of sin 36°, cos 36° and tan 36° 5 +1
4
5 +1
1. cos 36° = = 10 - 2 5
4
Proof: We have, ( 5 +1 )
= cos(36∞)
= 1 - 2 sin 2 (18∞) =
( )
5 -1 ¥ ( 10 - 2 5 )
4
2
Ê 5 - 1ˆ
= 1- 2 Á ˜ Notes:
Ë 4 ¯
5 +1
Ê 5 +1- 2 5ˆ i) sin 54° = cos 36° =
4
= 1- 2 Á ˜
Ë 16 ¯ 1
ii) cos 54° = sin 36° = 10 - 2 5 .
Ê 8 - 5 -1+ 2 5ˆ 4
= Á ˜
Ë 8 ¯ 1.18.5 Some Important Deductions
Ê 2 + 2 5ˆ Deduction 1:
= Á
Ë 8 ˜¯
Ê 1° ˆ
tan Á 7 ˜ = 6 - 4 - 3 + 2
Ê 5 + 1ˆ Ë 2¯
= Á ˜
Ë 4 ¯ 1 - cos ( 2q )
Proof: As we know that, tan q =
sin ( 2q )
1
2. sin 36° = 10 - 2 5 1∞
4 Put q = 7 , then
2
Proof: We have, sin (36∞) = 1 - cos (36∞)
2
Ê 1∞ ˆ 1 - cos (15∞)
tan Á 7 ˜ =
Ê 5 + 1ˆ
2
Ë 2¯ sin (15∞)
= 1- Á ˜
Ë 4 ¯ 3 +1
1-
= 2 2
Ê 5 +1+ 2 5ˆ
1- Á 3 -1
= ˜
Ë 16 ¯ 2 2
Ê 16 - 5 - 1 - 2 5 ˆ 2 2 - 3 -1
= Á ˜ =
Ë 16 ¯ 3 -1

Ê 10 - 2 5 ˆ (2 2 - 3 -1 )( 3 +1 )
= Á 16 ˜
Ë ¯ = ( 3 -1 )( 3 +1 )
=
1
4
10 - 2 5
=
(2 2 - 3 -1 )( 3 +1 )
2
1
3. tan 36° = ¥ ( 5 - 1 ) ¥ 10 - 2 5 . 2 6 - 3 - 3 + 2 2 - 3 -1
4 =
2
The Ratios and Identities 39

2 ( 6- 4- 3+ 2 ) Ê 1 ˆ
= 1- Á
= Ë 2 ˜¯
2

= ( 6- 4- 3+ 2 ) =
2 -1
2
Deduction 2:
Ê 1∞ ˆ 2 -1
Ê 1° ˆ fi sin Á 22 ˜ = ±
cot Á 7 ˜ = 6 + 4 + 3 + 2 Ë 2¯ 2 2
Ë 2¯
1 + cos ( 2q ) Ê 1∞ ˆ 2 -1 1∞
Proof: As we know that, cot (q ) = fi sin Á 22 ˜ = since, 22
sin ( 2q ) Ë 2¯ 2 2 2
1∞ .
Put q =7 ,
2
Ê 1° ˆ 1
Ê 1∞ ˆ fi sin Á 22 ˜ = 2- 2 .
cot Á 7 ˜ Ë 2¯ 2
Now, Ë 2¯
1 + cos (15∞) Deduction 4:
=
sin (15∞) Ê 1° ˆ 1
cos Á 22 ˜ = 2+ 2
3 +1 Ë 2¯ 2
1+
2 2 Proof: As we know that, 2 cos (q ) = 1 - cos 2q
2
=
3 -1
2 2 1∞
Put, q = 22 ,
2
2 2 + 3 +1 Ê 1° ˆ
= fi 2 cos 2 Á 22 ˜ = 1 + cos (45∞)
3 -1 Ë 2¯

(2 2 + 3 +1 )( 3 +1 ) Ê 1 ˆ
= 1+ Á
Ë 2 ˜¯
( )( )
=
3 -1 3 +1
2 +1
=
(2 6 + 3 + 3 + 2 2 + 3 +1 ) =
2
2
Ê 1∞ ˆ 2 -1

=
2 ( 6+ 4+ 3+ 2 ) fi cos Á 22 ˜ = ±
Ë 2¯ 2 2
2 2 +1
Ê 1∞ ˆ
fi cos Á 22 ˜ =
= ( 6+ 4+ 3+ 2 ) Ë 2¯ 2 2
1∞
Deduction 3: Since 22
2
Ê 1° ˆ 1 Ê 1° ˆ 1
sin Á 22 ˜ = 2- 2 thus, cos Á 22 ˜ = 2+ 2 .
Ë 2¯ 2 Ë 2¯ 2
Proof: As we know that,
Deduction 5:
2 sin (q ) = 1 - cos 2q
2
Ê 1° ˆ
tan Á 22 ˜ = 2 - 1
1∞ Ë 2¯
Put, q = 22 ,
2 1 - cos ( 2q )
Proof: As we know that, tan q =
Ê 1∞ ˆ sin ( 2q )
2 sin 2 Á 22 ˜ = 1 - cos (45∞)
Ë 2¯
1∞ tan Ê 22 1∞ ˆ = 1 - cos (45∞)
Put q = 22 , ÁË ˜
2 2¯ sin (45∞)
40 Comprehensive Trigonometry with Challenging Problems & Solutions for Jee Main and Advanced

1 Deduction 9:
1-
1∞ ˆ
( )
= 2 = 2 -1 Ê
1 tan Á112 ˜ = - 2 +1
Ë 2¯
2
Ê 1∞ ˆ
Deduction 6: Proof: We have, tan ÁË112 ˜¯
2
Ê 1° ˆ
cot Á 22 ˜ = 2 + 1 Ê 1∞ ˆ
Ë 2¯ = tan ÁË 90∞ ¥ 1 + 22 ˜¯
2
Proof: As we know that, Ê 1∞ ˆ
1 + cos ( 2q ) = - cot ÁË 22 ˜¯
cot (q ) = 2
sin ( 2q )
1∞ = - ( 2 +1 )
Put q = 22 ,
2
Ê 1∞ ˆ 1 + cos (45∞) 1.19 SOME SOLVED EXAMPLES
fi cot Á 22 ˜ =
Ë 2¯ sin (45∞)
Ex-1. Prove that: sin 2 (24∞) - sin 2 (6∞) =
( 5 -1 )
1
1+ 8
= 2 = 2 +1
1 Soln. We have, sin 2 (24∞) - sin 2 (6∞)
2 = sin(24∞ + 6∞) ¥ sin(24∞ - 6∞)
Deduction 7: = sin(30∞) ¥ sin(18∞)
Ê 1∞ ˆ 1
sin Á112 ˜ = 2+ 2 1
¥ sin(18∞)
Ë 2¯ 2 =
2
Ê 1∞ ˆ 1 5 -1
Proof: We have, sin ÁË112 ˜¯ = ¥
2 2 4
Ê 1∞ ˆ
= sin ÁË 90∞ ¥ 1 + 22 ˜¯
2 =
( 5 -1)
8
Ê 1° ˆ
= cos ÁË 22 ˜¯
2 Ex-2. Prove that sin 2 (48∞) - cos 2 (12∞) =
( 5 +1 )
8
1
= 2+ 2
2 Soln. We have, sin 2 (48∞) - cos 2 (12∞)
Deduction 8: = cos(48∞ + 12∞) ¥ cos(48∞ - 12∞)
Ê 1∞ ˆ 1 = cos(60∞) ¥ cos(36∞)
cos Á112 ˜ = 2- 2
Ë 2¯ 2
1 5 +1
Ê 1∞ ˆ = ¥
Proof: We have, cos ÁË112 ˜¯ 2 4
2
5 +1
Ê 1∞ ˆ =
= cos ÁË 90∞ ¥ 1 + 22 ˜¯ 8
2
1
Ê 1∞ ˆ Ex-3. Prove that sin(12∞).sin(48∞).sin(54∞) =
= - sin ÁË 22 ˜¯ 8
2
Soln. We have, sin(12∞).sin(48∞).sin(54∞)
1
= - 2+ 2 1
2 = (sin(12∞).sin(48∞).sin(72∞)) (sin(54∞))
sin(72∞)
The Ratios and Identities 41

1 Ê Ê 24∞ + 84∞ ˆ Ê 24∞ - 84∞ ˆ ˆ


= = 4 Á 2 sin ÁË ˜ cos Á ˜¯ ˜¯
4 sin(72∞) Ë 2 ¯ Ë 2
(4 sin(60∞ - 12∞).sin(12∞).sin(60∞ + 12∞)) = 8 ( sin(54∞) cos(30∞) )
(cos(36∞)) = 8 (cos(36∞) cos(30∞) )
1
= (.sin(36∞).cos(36∞)) Ê 5 +1 1ˆ
¥ ˜
4 sin(72∞) = 8Á
Ë 4 2¯
1
(2 sin(36∞).cos(36∞))
=
8 sin(72∞) = ( 5 +1 )
1
= (sin(72∞)) Ex-6. Prove that:
8 sin(72∞)
tan(6∞).tan(42∞).tan(66∞).tan (78∞) = 1
1
= Soln. We have,
8
tan(6∞).tan(42∞).tan(66∞).tan(78∞)
Ex-4. Prove that:
1 = (tan(6∞).tan(66∞)) ¥ (tan(42∞).tan(78∞))
sin(6°).sin(42°).sin(66°).sin(78°) =
16 1
Soln. We have,
= tan(54∞) ( tan(6∞).tan(54∞).tan(66∞) )
sin(6°).sin(42°).sin(66°).sin(78°)
¥ ( tan(42∞).tan(78∞) )
1
=
4 sin(54∞) 1
(4 sin(6∞).sin(60∞ - 6∞).sin(60∞ + 6∞).) = tan(54∞)
( tan(6∞).tan(60∞ - 6∞).tan(60∞ + 6∞))
¥(sin(78∞).sin(42∞)) ¥ ( tan(42∞).tan(78∞) )
1
= (sin(18∞)sin(72∞).sin(42∞)) 1
4 cos(36∞) (tan(18∞)) ¥ (tan(42∞).tan(78∞))
= tan(54∞)
1
=
16 cos(36∞) 1

(4 sin(18∞)sin (60∞ + 18∞).sin(60∞ - 18)) = tan(54∞)

1 ( tan(60∞ - 18∞).tan(18∞).tan(60∞ + 18∞))


= (sin(54∞))
16 cos(36∞)
1
1 ¥ (.tan(54∞) )
= (cos(36∞)) = tan(54∞)
16 cos(36∞)
=1
1
= Ex-8. Prove that:
16
Ex-5. Prove that: Ê Ê p ˆˆ Ê Ê 3p ˆ ˆ
ÁË1 + cos ÁË 8 ˜¯ ˜¯ . ÁË1 + cos ÁË 8 ˜¯ ˜¯
(
4 (sin (24°) + cos(6°) ) = 1 + 5 ) Ê Ê 5p ˆ ˆ Ê Ê 7p ˆ ˆ 1
Soln. We have, 4 (sin (24°) + cos(6°) ) = Á1 + cos ÁË ˜¯ ˜ . Á1 + cos ÁË ˜¯ ˜ =
Ë 8 ¯ Ë 8 ¯ 8

= 4 (sin (24°) + sin (84°) ) Ê Ê p ˆˆ Ê Ê 3p ˆ ˆ


Soln. We have, Á1 + cos Á ˜ ˜ . Á1 + cos Á ˜ ˜
Ë Ë 8 ¯¯ Ë Ë 8 ¯¯
42 Comprehensive Trigonometry with Challenging Problems & Solutions for Jee Main and Advanced

Ê Ê 5p ˆ ˆ Ê Ê 7p ˆ ˆ Ê 2Êpˆ 2 Ê p ˆˆ
. Á1 + cos Á ˜ ˜ . Á1 + cos Á ˜ ˜ = Á 2 - 2 sin ÁË ˜¯ .2 cos ÁË ˜¯ ˜
Ë Ë 8 ¯¯ Ë Ë 8 ¯¯ Ë 8 8 ¯
Ê Ê p ˆˆ Ê Ê 7p ˆ ˆ Ê Ê 2 Ê p ˆˆ Ê 2 Ê p ˆˆˆ
= Á1 + cos Á ˜ ˜ . Á1 + cos Á ˜ ˜
Ë 8 ¯¯ Ë Ë 8 ¯¯ = Á 2 - ÁË 2 sin ÁË 8 ˜¯ ˜¯ . ÁË 2 cos ÁË 8 ˜¯ ˜¯ ˜
Ë Ë ¯
Ê Ê 3p ˆ ˆ Ê Ê 5p ˆ ˆ Ê Ê Ê p ˆˆ Ê Ê p ˆˆˆ
. Á1 + cos Á ˜ ˜ . Á1 + cos Á ˜ ˜ = Á 2 - Á1 - cos Á ˜ ˜ . Á1 + cos Á ˜ ˜ ˜
Ë Ë ¯
8 ¯ Ë Ë 8 ¯¯ Ë Ë Ë 4¯¯ Ë Ë 4¯¯¯

Ê Ê p ˆˆ Ê Ê p ˆˆ Ê Ê 1 ˆ Ê 1 ˆˆ
= Á1 + cos Á ˜ ˜ . Á1 - cos Á ˜ ˜ = Á 2 - Á1 - ˜ . Á1 + ˜
Ë Ë 8 ¯¯ Ë Ë 8 ¯¯ Ë Ë 2 ¯ Ë 2 ¯ ˜¯
Ê Ê 1ˆ ˆ
Ê Ê 3p ˆ ˆ Ê Ê 3p ˆ ˆ = Á 2 - Á1 - ˜ .˜
. Á1 + cos Á ˜ ˜ . Á1 - cos Á ˜ ˜ Ë Ë 2¯ ¯
Ë Ë 8 ¯¯ Ë Ë 8 ¯¯
3
Ê =
Ê p ˆˆ Ê Ê 3p ˆ ˆ 2
= Á1 - cos 2 Á ˜ ˜ . Á1 - cos 2Á ˜ ˜
Ë Ë 8 ¯¯ Ë Ë 8 ¯¯
Ex-10. Prove that:
Ê Ê p ˆˆ Ê Ê 3p ˆ ˆ Êpˆ Ê 3p ˆ Ê 5p ˆ Ê 7p ˆ 3
= Á sin 2 Á ˜ ˜ . Á sin 2Á ˜ ˜ cos 4 Á ˜ + cos 4 Á ˜ + cos 4 Á ˜ + cos 4 Á ˜ =
Ë Ë ¯
8 ¯ Ë Ë 8 ¯¯ Ë 8¯ Ë 8¯ Ë 8¯ Ë 8 ¯ 2
1Ê Ê p ˆˆ Ê Ê 3p ˆ ˆ Soln. We have,
= 2 sin 2 Á ˜ ˜ . Á 2 sin 2Á ˜ ˜
4 ÁË Ë 8 ¯¯ Ë Ë 8 ¯¯
Êpˆ Ê 3p ˆ Ê 5p ˆ Ê 7p ˆ
cos 4 Á ˜ + cos 4 Á ˜ + cos 4 Á ˜ + cos 4 Á ˜
1Ê Ê p ˆˆ Ê Ê 3p ˆ ˆ Ë 8¯ Ë 8¯ Ë 8¯ Ë 8 ¯
= Á1 - cos Á ˜ ˜ . Á1 - cos Á ˜ ˜
4Ë Ë 4¯¯ Ë Ë 4 ¯¯
Êpˆ Ê 3p ˆ Ê 3p ˆ Êpˆ
= cos 4 Á ˜ + cos 4 Á ˜ + cos 4 Á ˜ + cos 4 Á ˜
1Ê 1 ˆ Ê 1 ˆ Ë 8¯ Ë 8¯ Ë 8¯ Ë 8¯
= Á1 - ˜ . Á1 + ˜
4Ë 2¯ Ë 2¯ Ê 3p ˆ Êpˆ
+ cos 4 Á ˜ + cos 4 Á ˜
1 Ê 1ˆ Ë 8¯ Ë 8¯
= Á1 - ˜ .
4 Ë 2¯
Ê Êpˆ Ê 3p ˆ ˆ
1 = 2 Á cos 4 Á ˜ + cos 4 Á ˜ ˜
= . Ë Ë 8¯ Ë 8 ¯¯
8
Ex-9. Prove that: Ê Êpˆ Ê p ˆˆ
= 2 Á cos 4 Á ˜ + sin 4 Á ˜ ˜
Ë Ë 8¯ Ë 8 ¯¯
Êpˆ Ê 3p ˆ Ê 5p ˆ Ê 7p ˆ 3
sin 4 Á ˜ + sin 4 Á ˜ + sin 4 Á ˜ + sin 4 Á ˜ =
Ë 8¯ Ë 8¯ Ë 8¯ Ë 8 ¯ 2 Ê Êpˆ Ê p ˆˆ
= 2 Á1 - 2 cos 2 Á ˜ .sin 2 Á ˜ ˜
Ë Ë 8¯ Ë 8 ¯¯
Soln. We have,
Êpˆ Ê 3p ˆ Ê 5p ˆ Ê 7p ˆ Ê Ê 2Êpˆ ˆ Ê 2 Ê p ˆˆˆ
sin 4 Á ˜ + sin 4 Á ˜ + sin 4 Á ˜ + sin 4 Á ˜ = Á 2 - Á 2 cos Á ˜ ˜ . Á 2 sin Á ˜ ˜ ˜
Ë 8¯ Ë 8¯ Ë 8¯ Ë 8 ¯ Ë Ë Ë 8¯ ¯ Ë Ë 8 ¯¯¯
4Êpˆ 4 Ê 3p ˆ 4 Ê 3p ˆ 4Êpˆ
= sin ÁË ˜¯ + sin ÁË ˜¯ + sin ÁË ˜¯ + sin ÁË ˜¯ Ê Ê Êpˆ ˆ Ê Ê p ˆ ˆˆ
8 8 8 8 = Á 2 - Á 1 + cos Á ˜ ˜ . Á 1 + cos Á ˜ ˜ ˜
Ë Ë Ë 4¯ ¯ Ë Ë 4¯ ¯¯
Ê 4Êpˆ 4 Ê 3p ˆ ˆ
= 2 Á sin ÁË ˜¯ + sin ÁË ˜¯ ˜ Ê Ê 1 ˆ Ê 1 ˆˆ
Ë 8 8 ¯ .Á 1 -
= Á2 - Á 1+ ˜ ˜
Ê 4Êpˆ Ë Ë 2 ¯ Ë 2 ¯ ˜¯
4 Ê p ˆˆ
= 2 Á sin ÁË ˜¯ + cos ÁË ˜¯ ˜
Ë 8 8 ¯ Ê Ê 1 ˆˆ
= Á 2 - Á 1- ˜˜
Ê 2Êpˆ 2 Ê p ˆˆ Ë Ë 2 ¯¯
= 2 Á1 - 2 sin ÁË ˜¯ .cos ÁË ˜¯ ˜
Ë 8 8 ¯ q
=
2
The Ratios and Identities 43

EXERCISE 9 17. If m = sin 6∞ sin 42∞ and n = sin 66∞ sin 72∞ ,
16 mn + 10.
Q Prove that:
18. If m = sin 8∞ sin 52∞ sin 66∞ sin 68∞ and
q q
1. tan + cot = 2 cosec q 8n = cos 42∞ , then prove that m = n.
2 2
q 19. If sin 17∞ sin 43∞ sin 77∞ = m cos 49∞ ,
2. cosecq – cotq = tan m.
2
Ê 2p ˆ Ê 4p ˆ Ê 8p ˆ
1 + sin q - cos q q 20. If m = sin ÁË ˜¯ + sin ÁË ˜¯ + sin ÁË ˜¯
3. = tan 7 7 7
1 + sin q + cos q 2

Êp qˆ
( 2m + 2 - 7 ) .
4. secq + tanq = tan ÁË + ˜¯ 21. If m = cos13∞ cos 47∞ cos 73∞
4 2
1
5. (cos A + cos B)2 + (sin A + sin B)2 and n = sin 51∞ , then prove that m = n.
4
2 Ê A - Bˆ
= 4 cos ÁË ˜ 22. Prove that:
2 ¯
sin 55∞ - sin 19∞ + sin 53∞ - sin 17∞ = cos1∞
6. Prove that:
2p 4p 6p 1
5 -1 23. Prove that: cos . cos . cos = .
2 2 7 7 7 8
sin 24 – sin 6 = .
8
2p 4p 6p 1
7. Prove that: 24. Prove that: cos + cos + cos = -
7 7 7 2
5 +1
sin2 48° – cos2 12° = – . Ê 1∞ ˆ Ê 1∞ ˆ
8 25. Find the value of tan Á 7 ˜ + cot Á 7 ˜ .
8. Prove that: Ë 2¯ Ë 2¯
Ê pˆÊ 3p ˆ Ê 5p ˆ Ê 5p ˆ 1 Ê xˆ Ê xˆ
ÁË1 + cos ˜¯ ÁË1 + cos ˜¯ ÁË1 + cos ˜¯ ÁË1 + cos ˜¯ = 26. If cos Á ˜ - 3 sin Á ˜ takes its minimum value
8 8 8 8 8 Ë 2¯ Ë 2¯
1 x.
9. Prove that: sin12° sin 48° sin 54° =
10. Prove that: 8
27. If a and b be two different roots
sin(47∞) + sin(61∞) - sin(11∞) - sin( 25∞) = cos(7∞) a cosq + b sinq = c, then prove that
11. Prove that: 2ab
Êpˆ Êpˆ Êpˆ Êpˆ 3 sin (a + b) = 2 .
cos 4 Á ˜ + cos 4 Á ˜ + cos 4 Á ˜ + cos 4 Á ˜ = a + b2
Ë 8¯ Ë 8¯ Ë 8¯ Ë 8¯ 2
1.20 CONDITIONAL TRIGONOMETRICAL
12. Prove that:
Êpˆ Ê 3p ˆ Ê 5p ˆ Ê 7p ˆ 3
IDENTITIES
sin 4 Á ˜ + sin 4 Á ˜ + sin 4 Á ˜ + sin 4 Á ˜ =
Ë 8¯ Ë 8¯ Ë 8¯ Ë 8 ¯ 2 Here, we shall deal with trigonometrical identities involving
two or more angles. In establishing such identities we
13. Prove that: tan 20∞ tan 80∞ = 3 tan 50∞ will be frequently using properties of supplementary and
complementary angles and hence students are advised
14. Prove that: tan 10∞ tan 70∞ = 3 tan 40∞
to go through all the above formulae from Ist topics to
15. If m = tan 12∞ tan 48∞ and n = tan 6∞ tan 66∞ , previous topic.
Êm ˆ We have certain trigonometrical identities like, sin2q
ÁË + 10˜¯ . + cos 2q = 1 and sec2q = 1 + tan2q, etc. Such identities are
n
identities in the sense that they hold for all values of the
16. If m = sin 10∞ sin 50∞ sin 70∞ and angles which satisfy the given condition amongst them and
n = cos 20∞ cos 60∞ cos 80∞ , they are called Conditional Identities.
If A, B, C denote the angles of a triangle ABC, then the
Êm ˆ relation A + B+ C = p enables us to establish many important
ÁË + 2012.˜¯
n identities involving trigonometric ratio of these angles.
44 Comprehensive Trigonometry with Challenging Problems & Solutions for Jee Main and Advanced

(i) If A + B + C = p, then A + B = p – C, Ê C + Dˆ Ê C - Dˆ
(iv) sin C – sin D = 2 cos Á
Ë 2 ˜¯
sin Á
Ë 2 ˜¯
B + C = p – A and C + A = p – B .
(ii) If A + B + C = p, then
sin (A + B) = sin (p – C) = sinC
similarly, sin (B – C) = sin (p – A) = sin A
1.21 SOME SOLVED EXAMPLES
and sin (C + A) = sin (p – B) = sin B Ex-1. If A + B + C = p, then prove that,
(iii) If A + B + C = p, then sin 2A + sin2B + sin 2C = 4sin A sin B sinC
cos (A + B) = cos (p – C) = –cos C Soln. We have, sin 2A + sin2B + sin 2C
Similarly, cos (B + C) = cos (p – A) = –cos A
and cos (C + A) = cos (p – B) = –cos B = (sin 2 A + sin 2 B ) + sin 2C
(iv) If A + B + C = p, then
tan(A + B) = tan (p – C) = –tan C = 2 (sin ( A + B ) cos ( A - B )) + sin 2C
Similarly, tan (B + C) = tan (p – A) = –tan A = 2 (sin (p - C ) .cos ( A - B )) + 2 sin C cos C
and tan (C + A) = tan (p – B) = –tan B
A+ B p C = 2 (sin C.cos ( A - B )) + 2 sin C cos C
v) If A + B + C = p, then = – ,
2 2 2 = 2sin C (cos ( A - B ) + cos C )
B+C p A C+A p B
2
=
2

2
and
2
=
2

2
(
= 2sin C cos ( A - B ) + cos (p - ( A + B )) )
Therefore, = 2 sin C (cos ( A - B ) - cos ( A + B ))
Ê A + Bˆ Êp Cˆ Ê Cˆ
sin Á ˜ = sin Á - ˜ = cos ÁË ˜¯ = 2 sin C ( 2 sin A sin B )
Ë 2 ¯ Ë 2 2¯ 2
Ê Cˆ = 4 sin A.sin B.sin C
Ê A + Bˆ Êp Cˆ
cos Á = cos Á - ˜ = sin ÁË ˜¯
Ë 2 ˜¯ Ë 2 2¯ 2 Ex-2. If A + B + C = p, then prove that,
Ê A + Bˆ Êp Cˆ Ê Cˆ cos 2A + cos 2B + cos 2C
tan Á ˜ = tan Á - ˜ = cot ÁË ˜¯ .
Ë 2 ¯ Ë 2 2¯ 2
= –1 – 4 cos A cos B cos C
Note: Dear students, please recollect the following formulae
Soln. We have cos 2A + cos 2B + cos 2C
from basic trigonometry
Step I: = (cos 2 A + cos 2 B ) + cos 2C
(i) sin 2 A = 2 sin A cos A
= 2 cos ( A + B ) cos ( A - B ) + cos 2C
(ii) cos 2 A = cos A - sin A
2 2

(iii) cos 2 A = 2 cos A - 1


2
= 2 cos {p - C } cos ( A - B ) + cos 2C
(iv) cos 2 A = 1 - 2 sin A
2
= -2 cos C cos ( A - B ) + 2 cos C - 1
2

= -1 - 2 cos C ( cos ( A - B ) - cos C )


Step II:
(i) 1 + cos 2 A = 2 cos A
2

(ii) 1 - cos 2 A = 2 sin A


2 = -1 - 2 cos C ( cos ( A - B ) + cos ( A + B ))
2 Ê Aˆ
(iii) 1 + cos A = 2 cos Á ˜ = -1 - 2 cos C ( 2 cos A.cos B )
Ë 2¯
= -1 - 4 cos A .cos B .cos C
2 Ê Aˆ
(iv) 1 - cos A = 2 sin Á ˜
Ë 2¯ Ex-3. If A + B + C = p , then prove that
Step III:
sin 2 A + sin 2 B - sin 2 C = 2 sin A sin B cos C
(i) cos (A+B) + cos (A–B) = 2 cos A cosB
(ii) cos (A – B)– cos (A+B) = 2 sin A sin B. Soln. We have, sin 2 A + sin 2 B - sin 2 C
Step IV:
(i) cos C + cos D = 2 cos Á
Ê C + Dˆ Ê C - Dˆ (
= sin 2 A + sin 2 B - sin 2 C )
Ë 2 ¯ ˜ cos Á
Ë 2 ˜¯
= sin 2 A + sin ( B + C ) sin ( B - C )
Ê C + Dˆ Ê D - Cˆ
(ii) cos C – cos D = 2 sin Á
Ë 2 ˜¯
sin Á
Ë 2 ˜¯ = sin 2 A + sin (p - A) sin ( B - C )

= sin A + sin A sin ( B - C )


2
Ê C + Dˆ Ê C - Dˆ
(iii) sin C + sin D = 2 sin Á cos Á
Ë 2 ¯ ˜ Ë 2 ˜¯
The Ratios and Identities 45

= sin A (sin A + sin ( B - C )) fi


1
+
1
+
1
=1
tan B.tan C tan A.tan C tan A.tan B
= sin A (sin ( B + C ) + sin ( B - C ))
fi cot B.cot C + cot A.cot C + cot A.cot B = 1
= sin A (2 sin B cos C )
Ex-7. If A, B, C and D be the angles of a quadrilateral, then
= 2sin A.sin B.cos C
prove that,
Ex-4. If A + B + C = p , then prove that, tan A + tan B + tan C + tan D
sin A + sin B + sin C = 2 + 2 cos A cos B cos C
2 2 2 cot A + cot B + cot C + cot D
= tan A.tan B.tan C.tan D
Soln. We have, sin 2 A + sin 2 B + sin 2 C
Soln. We have, A + B + C + D = 2p
= 1 - cos 2 A + sin 2 B + sin 2 C
fi A + B + C + D = 2p
( ) (
= 1 - cos A - sin B + 1 - cos C
2 2 2
) fi A + B = 2p - (C + D )

fi tan ( A + B ) = tan {2p - (C + D )}


= 2 - (cos 2 A - sin 2 B) - cos 2 C
fi tan ( A + B ) = - tan (C + D )
= 2 - (cos( A + B ).cos( A - B )) - cos 2 C
tan A + tan B tan C + tan D
fi =-
= 2 - (cos(p - C ).cos( A - B )) - cos 2 C 1 - tan A.tan B 1 - tan C.tan D
= 2 + cos C (.cos( A - B) - cos C ) fi ( tan A + tan B ) (1 - tan C.tan D )
= 2 + cos C (.cos( A - B) + cos ( A + B)) = - ( tan C + tan D ) (1 - tan A.tan B )
= 2 + cos C ( 2 cos A.cos B ) fi tan A + tan B - tan A.tan C.tan D
= 2 + 2 cos A.cos B.cos C - tan B.tan C.tan D
Ex-5. In a triangle DABC, prove that,
= –tan C – tan D + tan A. tan B tan C
tan A + tan B + tan C = tan A.tan B.tan C + tan A. tan B. tan D
Soln. We have, A + B + C = p
fi tan A + tan B + tan C + tan D
fi A+ B =p -C
= tan A . tan C. tan D + tan B. tan C. tan D
fi tan( A + B) = tan(p - C ) + tan A. tan B. tan D + tan A. tan B. tan D
tan A + tan B tan A + tan B + tan C + tan D
fi = - tan C
1 - tan A.tan B fi
tan A.tan B.tan C.tan D
fi tan A + tan B = - tan C (1 - tan A.tan B ) tan A.tan B.tan C
=
fi tan A + tan B = - tan C + tan A.tan B.tan C tan A.tan B.tan C.tan D
tan A.tan C.tan D
fi tan A + tan B + tan C = tan A.tan B.tan C +
tan A.tan B.tan C.tan D
Ex-6. In a triangle DABC, prove that, tan A.tan B.tan D
+
cot A.cot B + cot B.cot C + cot C.cot A = 1 tan A.tan B.tan C.tan D
Soln. As we know that, tan B.tan C.tan D
+
tan A + tan B + tan C = tan A.tan B.tan C tan A.tan B.tan C.tan D
Dividing both the sides by tan A + tan B + tan C + tan D

‘tanA. tanB. tanC’, we get, tan A.tan B.tan C.tan D
tan A tan B 1 1 1 1
+ = + + +
= tan A tan B tan C tan D
tan A.tan B.tan C tan A.tan B.tan C
tan A + tan B + tan C + tan D
tan C fi
+ =1 tan A.tan B.tan C.tan D
tan A.tan B.tan C
= cot A + cot B + cot C + cot D
46 Comprehensive Trigonometry with Challenging Problems & Solutions for Jee Main and Advanced

tan A + tan B + tan C + tan D p


fi fi (A + B + C) =
cot A + cot B + cot C + cot D 2
tan A.tan B.tan C.tan D Now,
Ex-8. In a triangle ABC, prove that, x y z
L.H.S = + +
1- x 2
1- y 2
1 - z2
(cot A + cot B ) (cot B + cot C )
tan A tan B tan C
(cot C + cot A) = cosec A cosec B cosec C. = + +
1 - tan A 1 - tan B
2 2
1 - tan 2 C
cos A cos B
Soln. We have, (cot A + cot B ) = + 1 Ê 2 tan A 2 tan B 2 tan C ˆ
sin A sin B = ÁË + + ˜
2 1 - tan A 1 - tan B 1 - tan 2 C ¯
2 2
cos A sin B + sin A. cos B
= 1
sin A sin B = ( tan 2 A + tan 2 B + tan 2C )
2
sin( A + B) sin C
= = .
sin A sin B sin A sin B 1
= ( tan 2 A.tan 2 B.tan 2C )
sin A 2
Similarly, (cot B + cot C ) =
sin B sin C 1 Ê 2 tan A 2 tan B 2 tan C ˆ
= Á . . ˜
sin B 2 Ë 1 - tan 2 A 1 - tan 2 B 1 - tan 2 C ¯
and (cot C + cot A) =
sin A sin C
Ê 4 tan A.tan B.tan C ˆ
Thus, (cot A + cot B ) (cot B + cot C ) = Á ˜
Ë (1 - tan A)(1 - tan B)(1 - tan C ) ¯
2 2 2

(cot C + cot A) 4 xyz


=
sin C sin B sin A (1 - x )(1 - y 2 )(1 - z 2 )
2
= ¥ ¥
sin A sin B sin A sin C sin B sin C
Hence, the result.
1 Ex-10. If xy + yz + zx = 1, then prove that,
=
sin A sin B sin C x y z 2
+ + =
=
x
+
y
+
z
=
4 xyz 1+ x 2
1+ y 2
1+ z 2
(1 + x ) (1 + y ) (1 + z )
2 2 2

1- x 2
1- y 2
1- z 2
(1 - x ) (1 - y ) (1 - z )
2 2 2

Soln. Put x = tan A, y = tan B and z = tan C


Ex-9. If x y + y z + z x = 1, then prove that, Given, x y + y z + z x = 1
x y z 4 xyz
+ + = fi tan A.tan B + tan B.tan C + tan C.tan A = 1
1- x 2
1- y 2
1- z 2
(1 - x )(1 - y 2 )(1 - z 2 )
2

Soln. Put x = tan A, y = tan B and z = tan C p


fi A+ B+C =
Given, x y + y z + z x = 1 2
fi tan A.tan B + tan B.tan C + tan C.tan A = 1 Now, L.H.S

fi tan B.tan C + tan C.tan A = 1 - tan A.tan B x y z


= + +
1+ x 2
1+ y 2
1 + z2
fi tan C ( tan B + tan A) = 1 - tan A.tan B
tan A + tan B 1 1Ê 2x 2y 2z ˆ
= = Á + + ˜
fi 2 Ë1+ x 2
1+ y 2
1 + z2 ¯
1 - tan A.tan B tan C
Êp ˆ 1 Ê 2 tan A 2 tan B 2 tan C ˆ
fi tan ( A + B ) = cot C = tan Á - C ˜ = ÁË + + ˜
Ë2 ¯ 2 1 + tan A 1 + tan B 1 + tan 2 C ¯
2 2

p
( A + B ) = ÊÁË ˆ 1
fi - C˜ = (sin 2 A + sin 2 B + sin 2C )
2 ¯ 2
The Ratios and Identities 47

1 Ê Aˆ Ê Bˆ Ê Cˆ
= (4 cos A.cos B.cos C ) = 1 - 2 cos ÁË ˜¯ cos ÁË ˜¯ cos ÁË ˜¯
2 2 2 2
= 2 cos A.cos B.cos C p
Q. If A + B + C = , then prove that:
2
2
= 11. sin2 A + sin2 B + sin2 C = 1 – 2 sin A sin B sin C
sec A.sec B.sec C
12. cos2 A + cos2 B + cos2 C = 2 + 2 sin A sin B sin C
2 Q. If A + B + C = p, then prove that:
=
(1 + tan A) (1 + tan B) (1 + tan C )
2 2 2 13. tan A + tan B + tan C = tan A tan B tan C.
14. tan 2A + tan2B + tan 2C = tan 2A tan 2B tan 2C.
2 15. cot A cot B + cot B cot C + cot C cot A = 1
=
(1 + x ) (1 + y ) (1 + z )
2 2 2 16. cot (A/2) cot (B/2)+cot (B/2) cot (C/2)
+ cot (C/2) cot (A/2) = 1.
Hence, the result. p
Q. If A + B + C = , then prove that:
2
17. cot A + cot B + cot C = cot A cot B cot C
EXERCISE 10 18. tan A tan B + tan B tan C + tan C tan A = 1
19. If xy + yz + zx = 1, then prove that:
1. If A + B + C = p, then prove that:
x y z 4 xyz
sin 2A + sin 2 B + sin 2C = 4 sin A sin B sin C. + + =
2. If A + B + C = p, then prove that:
1- x 2
1- y 2
1- z 2
( )(
1 - x 1 - y2 1 - z2
2
)( )
sin 2A + sin 2 B – sin 2C = 4 cos A cos B sin C. 20. Prove that:
3. If A + B + C = p, then prove that: tan (a - b ) + tan (b - g ) + tan (g - a )
cos 2 A + cos 2 B + cos 2C = tan (a - b ) tan (b - g ) tan (g - a )
= 1 - 4 cos A cos B cos C
21. In a DABC, if cot A + cot B + cot C = 3 ,
4. If A + B + C = p, then prove that: then prove that the triangle is an equilateral.
cos 2 A + cos 2 B - cos 2C 22. If x + y + z = x y z, then prove that:
= 1 – 4 sin A sin B cos C.
3 x - x3 3 y - y3 3z - z 3
5. If A + B + C = p, then prove that: + +
1 - 3x 2 1 - 3y2 1 - 3z 2
A B C
cos A + cos B + cos C = 1 + 4 sin sin sin
2 2 2 3x - x3 3 y - y 3 3z - z 3
= . .
6. If A + B + C = p, then prove that: 1 - 3x 2 1 - 3 y 2 1 - 3z 2
cos 2 A + cos 2 B + cos 2 C + 2 cos A cos B cos C = 1 23. Prove that 1 + cos 56° + cos 58° – cos 66°
= 4cos 28° cos 29° sin 33°.
7. If A + B + C = p, then prove that:
cos2 A + cos2 B + cos2 C = 1 – 2 sin A sin B cos C.
8. If A + B + C = p, then prove that
1.22 TRIGONOMETRICAL SERIES
sin2 A + sin2 B + sin2 C = 2 (1 + cos A cos B cos C) 1.21.1 Introduction
9. If A + B + C = p, then prove that: In this section, we are mainly connected with different
sin 2 A + sin 2 B + sin 2C
cos A + cos B + cos C - 1 series.
To find out the sum of diffrent trigonometrical
A B C
= 8 cos cos cos
2 2 2 trigonometrical terms. We must observe whether the angles
10. If A + B + C = p, then prove that: form any sequence or not? If they form any sequences, then
Ê Aˆ Ê Bˆ Ê Cˆ we must check, what kind of sequence it is? We also bserve
sin 2 Á ˜ + sin 2 Á ˜ - sin 2 Á ˜
Ë 2¯ Ë 2¯ Ë 2¯
the series.
48 Comprehensive Trigonometry with Challenging Problems & Solutions for Jee Main and Advanced

So, our main attempt will be Ê (n - 1)b ˆ Ê bˆ


(i) to express each term as a diffrence of the two terms = 2 sin ÁË a + ˜¯ ¥ sin ÁË ˜¯
2 2
directly or by manupulation and then add or
(ii) to arrange the series in such a way that it follows some Ê nb ˆ
sin Á ˜
standard trigonometrical expansion. Ë 2¯ Ê (n - 1)b ˆ
Thus S = ¥ sin Á a + ˜
b
Ê ˆ Ë 2 ¯
sin Á ˜
Ë 2¯
1.23 DIFFERENT TYPES OF
THE SUMMATION OF A EXERCISE 11
TRIGONOMETRICAL SERIES
1. A trigonometrical series involved with the terms of sines 1. Prove that: sin a + sin 2a + sin 3a + .......... + sin na
or cosines. Ê na ˆ
sin Á ˜
Rule: Whenever angles are in A.P. and the trigonometrical Ë 2 ¯ Êaˆ
= ¥ sin ( n + 1) Á ˜
terms involved with sines or cosines having power 1. Êaˆ Ë 2¯
1. We must multiply each term by sin Á ˜
Ë 2¯
Ê common diffrence of angles ˆ
2 sin Á ˜¯ 2. Prove that:
Ë 2
sin 2 nq
2. and then express each term as a diffrenec of two terms. sin q + sin 3q + ...... + sin (2n - 1)q =
3. Finally add them. sin q
Ex.-1 Prove that: 3. Prove that: sin 2 a + sin 2 (a + b ) + sin 2 (a + 2b ) + ...
sin a + sin (a + b ) + sin (a + 2b ) ...... + sin 2 (a + ( n - 1) b )
+ sin (a + 3b ) + ......... + sin (a + (n - 1)b ) n 1 sin nb
= - ¥ ¥ cos [ 2a + (n - 1)b ]
2 2 sin b
Ê nb ˆ
sin Á ˜ 2p ˆ
Ë 2¯ È b˘ 2Ê
4. Prove that: sin a + sin ÁË a + ˜¯
2
= ¥ sin Ía + ( n - 1) ˙ n
Ê bˆ Î 2˚
sin Á ˜
Ë 2¯ Ê 2p ˆ n
+ sin 2 Á a + ˜ + ..... to - n - terms =
Ë n¯
Proof: Let S = sin a + sin (a + b ) + sin (a + 2b ) 2

+ sin (a + 3b ) + ......... + sin (a + (n - 1)b ) 5. If 4 nq = p , then prove that:


Now, sin 2 q + sin 2 3q + sin 2 5q + ....to - n - terms = n
Ê bˆ Ê bˆ Ê bˆ 6. Prove that:
2 sin a sin Á ˜ = cos Á a - ˜ - cos Á a + ˜
Ë 2¯ Ë 2¯ Ë 2¯ sin 3 q + sin 3 3q + sin 3 5q + ....to - n - terms
Ê bˆ Ê bˆ Ê 3b ˆ
2 sin (a + b ) .sin Á ˜ = cos Á a + ˜ - cos Á a + ˜ 1 È 3 sin 2 nq 3 sin 2 3nq ˘
Ë 2¯ Ë 2¯ Ë 2¯ -
= Í ˙.
4 Î sin q sin 3q ˚
Ê bˆ Ê 3b ˆ Ê 5b ˆ
2 sin (a + 2b ) .sin Á ˜ = cos Á a + ˜ - cos Á a + ˜
Ë 2¯ Ë 2 ¯ Ë 2¯ 7. Prove that: sin a + cos(a + b )
---------- ------------- --------- ---- --------
--- ------------ ----- - sin(a + 2)b - cos(a + 2b )
Adding, we get + sin(a + 4b ) + ....to - n - terms
Ê bˆ
2 sin (a + (n - 1)b ) .sin Á ˜ Ê n( 2b + p ) ˆ
Ë 2¯ sin Á ˜¯
Ë 4 Ï (n - 1)(2b + p ) ¸
= ¥ sin Ìa + ˝
Ê (2n - 3)b ˆ Ê (2n - 1)b ˆ Ê 2 b + p ˆ Ó 4 ˛
= cos ÁË a + ˜¯ - cos ÁË a + ˜¯
Ë 4 ˜¯
sin Á
2 2
The Ratios and Identities 49

8. Prove that: Ê (2n - 1)b ˆ Ê (2n - 3)b ˆ


2p ˆ
3Ê Ê 4p ˆ = sin Á a + ˜¯ - sin ÁË a + ˜¯
sin a + sin Á a + ˜ + sin 3 Á a + ˜
3 Ë 2 2
Ë n¯ Ë n¯
Adding all, we get,
+.........to - n - terms = 0
Ê bˆ Ê 2n - 1 ˆ
9. Prove that: 2 sin Á ˜ ¥ S = sin Á a + b ˜ - sin (a - b )
sin q .sin 2q + sin 2q .sin 3q Ë 2¯ Ë 2 ¯

+ sin 3q .sin 4q + .....to - n - terms Ê bˆ


fi 2 sin Á ˜ ¥ S
n 1 sin nq Ë 2¯
= cos q - ¥ ¥ cos ( n + 2)q
2 2 sin q Ê n -1 ˆ Ê nb ˆ
= 2 cos Á a + b ˜ ¥ sin Á ˜
4Ê 2p ˆ Ë 2 ¯ Ë 2¯
10. Prove that: sin x + sin ÁË x + ˜¯
4
n
Ê nb ˆ
sin Á ˜
Ê 2p ˆ 3p Ë 2¯ Ê n -1 ˆ
+ sin 4 Á x + ˜ + .....to - n - terms = fi S= ¥ 2 cos Á a + b˜
Ë n¯ 8 b
Ê ˆ Ë 2 ¯
sin Á ˜
2. A Trigonometrical series involved with the terms of Ë 2¯
cosines:
cos a + cos(a + b ) + cos(a + b )
EXERCISE 12
+ cos(a + b ) + .... + cos(a + (n - 1)b )
Ê nb ˆ 1. Prove that, cos a + cos 2a + cos 3a + .......
sin Á ˜
Ë 2¯ Ê bˆ Ê na ˆ
= ¥ cos Á a + (n - 1) ˜ sin Á ˜
Ê bˆ Ë 2¯ Ë 2 ¯ Ê (n + 1)a ˆ
sin Á ˜ .... + cos na = ¥ cos Á ˜
Ë 2¯ Êaˆ Ë 2 ¯
sin Á ˜
Ë 2¯
Proof: Let S = cos a + cos(a + b ) + cos(a + b )
+ cos(a + b ) + .... + cos(a + (n - 1)b ) Ê 2p ˆ
2. Prove that, cos a + cos ÁË a + ˜¯
n
Ê bˆ
Now, 2 cos a sin Á ˜
Ë 2¯ Ê 2p ˆ
+ cos Á a + ˜ + ....to - n - terms
Ë n¯
Ê bˆ Ê bˆ
= sin Á a + ˜ - sin Á a - ˜ =0
Ë 2¯ Ë 2¯ 3. If B be the exist for angles of a regular polygon of
Ê bˆ n-sides and A is any constant, then prove that
2 cos (a + b ) sin Á ˜ cos A + cos ( A+B) + cos ( A+2B)
Ë 2¯
+..............to n terms = 0.
Ê 3b ˆ Ê bˆ 4. Show that, if n > 2,
= sin Á a + ˜ - sin Á a + ˜
Ë 2¯ Ë 2¯
Êpˆ Ê 3p ˆ Ê 5p ˆ
cos 2 Á ˜ + cos 2 Á ˜ + cos 2 Á ˜
Ê bˆ Ë n¯ Ë n¯ Ë n¯
2 cos (a + 3b ) sin Á ˜
Ë 2¯
n
+..........to - n - terms =
Ê 5b ˆ Ê 3b ˆ 2
= sin Á a + ˜ - sin Á a + ˜
Ë 2 ¯ Ë 2¯
5. Prove that, 1 - sin 2q + 1 - sin 4q
--------------- --- ---------
+ 1 - sin 6q + ......to - n - terms
------------------- ---------
Ê nq ˆ
sin Á ˜
Ê bˆ Ë 2¯ È q q˘
2 cos (a + (n - 1)b ) sin Á ˜ = ¥ cos ( n + 1) - sin ( n + 1) ˙ .
Ë 2¯ Ê q ˆ ÍÎ 2 2˚
sin Á ˜
Ë 2¯
50 Comprehensive Trigonometry with Challenging Problems & Solutions for Jee Main and Advanced

Thus, t1 = cot 2x – cot 3x


6. Prove that, cos 2 x + cos 2 3x + cos 2 5 x + ...
........to - n - terms t2 = cot3x – cot 4 x
t3 = cot 4x – cot 5 x
1 È sin 4nx ˘
= ¥ Ín + ˙. ........ .......... ..........
2 Î 2 sin 2 x ˚
7. Prove that, cos x + sin 3x + cos 5x ....... ........... ..........
+ sin 7x + ............+ sin (2n+1)x tn = cot(n + 1)x – cot(n + 2)x.
sin 2nx Adding all,
= ¥ [cos(2n - 1) x + sin(2n + 1) x ] .
sin 2 x we get, S = cot 2x – cot (n + 2) x.
3Ê 2p ˆ
8. Prove that, cos q + cos ÁË q - ˜¯
3
n
EXERCISE 13
3Ê 4p ˆ Ê 6p ˆ
+ cos Á q - ˜ + cos3 Á q - ˜
Ë n ¯ Ë n¯ 1. Prove that,
cos ec q .cos ec 2 q + cos ec 2 q .cos ec 3q
+....to - n - terms = 0
+cosec 3q .cosec 4q + ....to - n - terms
9. Prove that, cos a + cos (a + b )
2 2
= cosecq [cot q - cot(n + 1) q ]
+ cos 2 (a + 2b ) + .......to - n - terms
1 1
n 1 sin nb 2. Prove that, +
= + ¥ ¥ cos [ 2a + (n - 1)b ] cos q + cos 2q cos q + cos 5q
2 2 sin b
1
+ + ....to - n - terms
10. Prove that, cos q cos 2 q + cos 3q cos 4 q cos q + cos 7q
+ cos 5q cos 6 q + ......to - n - terms = cosecq ÈÎ tan (n + 1)q - tan q ˘˚
1 È sin 2nq ˘
= ¥ Ín cos q + ¥ cos(2n + 1)q ˙ . sin x sin x
2 Î sin 2q ˚ 3. Prove that, +
cos x + cos 2 x cos x + cos 4 x
3. A Trigonomeytrical Series Based Method of difference:
sin x
Rules: 1. Express each term of the series as a diffreence of + + .....to - n - terms.
two expressions. cos x + cos 6 x
2. Finally added them and we shall get the required 1 Ê nˆ È x Ê xˆ ˘
= ¥ cos ec Á ˜ ¥ Ísec(2n + 1) - sec Á ˜ ˙ .
result. 4 Ë 2 ¯ Î 2 Ë 2¯ ˚
Ex-1. Find the sum of n-terms of the series
sin x sin x 4. Prove that, cosec q + cosec 2q + cosec 22 q
+
sin 2 x.sin 3x sin 3x.sin 4 x
+cosec 24 q + .....to - n - terms
sin x
+ + ....to - n - terms Êqˆ
sin 4 x.sin 5 x n -1
= cot ÁË ˜¯ - cot 2 q .
2
( )
sin x
Soln. Let tn =
sin(n + 1) x.sin(n + 2) x Êqˆ Êqˆ
5. Prove that, cosecq + cosec ÁË ˜¯ + cosec ÁË 2 ˜¯
sin[(n + 2) x - (n + 1) x] 2 2
fi tn =
sin(n + 1) x.sin(n + 2) x Êqˆ
+cosec Á 3 ˜ + to - n - terms
sin(n + 2) x cos(n + 1) x Ë2 ¯
=
sin(n + 1) x. sin(n + 2) x
Êqˆ
cos(n + 2) x sin(n + 1) x = cot ÁË n ˜¯ - cot q .
- 2
sin(n + 1) x. sin(n + 2) x
6. Prove that,
= cot(n + 1) x - cot(n + 2) x
tan x . tan 2x + tan 2x. tan 3x
The Ratios and Identities 51

+ .......+ tan nx . tan (n + 1)x Ex-2 Prove that,


= cot x [tan (n + 1) x – tan x] – n . tan 9∞ - tan 27∞ - tan 63∞ + tan 81∞ = 4
-1 Ê x ˆ Ê x ˆ Soln. We have tan 9∞ - tan 27∞ - tan 63∞ + tan 81∞
7. Prove that, tan ÁË 2˜
+ tan -1 Á
1 + 2x ¯ Ë 1 + 6 x 2 ˜¯
= ( tan 9∞ + tan 81∞) - ( tan 27∞ + tan 63∞)
Ê x ˆ
+ tan -1 Á = ( tan 9∞ + cot 9∞) - ( tan 27∞ + cot 27∞)
Ë 1 + 12 x 2 ˜¯ +...............
Ê 1 ˆ Ê 1 ˆ

-1 Ê ˆ = Á
x Ë sin 9∞ cos 9∞ ¯ Ë sin 27∞ cos 27∞ ˜¯
˜
..........+ tan ÁË 1 + n(n + 1) x 2 ˜¯
Ê 2 ˆ Ê 2 ˆ
-1 -1 = ÁË ˜¯ - ÁË ˜
= tan (n + 1) x - tan x 2 sin 9∞ cos 9∞ 2 sin 27∞ cos 27∞ ¯
Ê 2 2 ˆ
-1 Ê 1 ˆ1ˆ -1 Ê
-1 Ê 1 ˆ = ÁË - ˜
8. Prove that, tan ÁË ˜¯ + tan ÁË ˜¯ + tan ÁË ˜¯ sin 18∞ sin 54∞ ¯
7 13 19
Ê 2 2 ˆ
-1 Ê 1 ˆ = ÁË - ˜
+..... + tan Á 2 sin 18∞ cos 36∞ ¯
Ë n + 3n + 3 ˜¯

= tan ( n + 2) - tan 2 .
-1 -1
= Á
(
Ê 8 5 +1- 5 +1 ˆ
˜
)
ÁË ( )( )
5 - 1 5 + 1 ˜¯
Ê 1 ˆ Ê 2 - 1ˆ
9. Prove that, sin -1 Á ˜ + sin -1 Á ˜ Ê 8 ( 5 + 1 - 5 + 1) ˆ
Ë 2¯ Ë 2 ¯ = Á ˜
ÁË ( 5 - 1) ( 5 + 1) ˜¯
Ê 3 - 2ˆ
+ sin -1 Á p
˜ + ......to• = . Ê 8 ¥ 2ˆ
Ë 12 ¯ 2 = Á =4
Ë 4 ˜¯
Ê 1ˆ Ê 2ˆ
10. Prove that, tan -1 Á ˜ + tan -1 Á ˜
Ë 3¯ Ë 9¯
Ex-3 Prove that:
Ê 4ˆ sin x sin 3x sin 9 x
+ tan -1 Á ˜ + ......to• = p .
1
+ + = ( tan 27 x - tan x )
Ë 23 ¯ 4 cos 3x cos 9 x cos 27 x 2
Soln. We have
sin x sin 3x sin 9 x
PROBLEMS FOR JEE ADVANCED EXAM + +
cos 3x cos 9 x cos 27 x
Ex-1 Prove that,
1 Ê 2 sin x cos x 2 sin 3x cos 3x
tan a + 2 tan 2a + 4 tan 4a + 8 cot 8a = cot a = 2 Á cos 3x cos x + cos 3x cos 9 x
Ë
Soln. We have
2 sin 9 x cos 9 x ˆ
tan a + 2 tan 2a + 4 tan 4a + 8 cot 8a + ˜
cos 27 x cos 9 x ¯
= cot a - (cot a - tan a ) + 2 tan 2a 1 Ê sin 2 x sin 6 x sin 18 x ˆ
+4 tan 4a + 8 cot 8a = 2 Á cos 3x cos x + cos 3x cos 9 x + cos 9 x cos 27 x ˜
Ë ¯
= cot a - 2 (cot 2a - tan 2a ) + 4 tan 4a + 8 cot 8a
1 Ê sin (3x - x ) sin (9 x - 3x ) sin (27 x - 9 x ) ˆ
= cot a - 2.2 cot 4a + 4 tan 4a + 8 cot 8a = 2 Á cos 3x cos x + cos 3x cos 9 x + cos 9 x cos 27x ˜
Ë ¯
= cot a - 4 (cot 4a - tan 4a ) + 8 cot 8a 1 Ê sin 3x cos x - cos 3x sin x
= cot a - 4.2 cot 8a + 8 cot 8a = 2 ÁË cos 3x cos x
= cot a - 8 cot 8a + 8 cot 8a sin 9 x cos 3x - cos 9 x sin 3x
+
= cot a cos 3x cos 9 x
52 Comprehensive Trigonometry with Challenging Problems & Solutions for Jee Main and Advanced

sin 27 x cos 9 x - cos 27 x sin 9 x ˆ


+ ˜¯
cos 9 x cos 27 x 4 5
1 4 5 ¥ 27
= ( tan 3x - tan x + tan 9 x - tan 3x tan ( x + y ) = 3 3 = = 15
2 15 12 ¥ 3 3
1-
+ tan 27 x - tan 9 x ) 27
Ex-5
1
= ( tan 27 x - tan x ) Êpˆ Ê 3p ˆ Ê 5p ˆ Ê 7p ˆ 3
2 sin 4 Á ˜ + sin 4 Á ˜ + sin 4 Á ˜ + sin 4 Á ˜ =
Ë 16 ¯ Ë 16 ¯ Ë 16 ¯ Ë 16 ¯ 2
sin x 1 cos x 3
Ex-4. If = , = , where x, y ΠR Soln.
sin y 2 cos y 2
Êpˆ Ê 3p ˆ Ê 5p ˆ Ê 7p ˆ
tan ( x + y ) . sin 4 Á ˜ + sin 4 Á ˜ + sin 4 Á ˜ + sin 4 Á ˜
Ë 16 ¯ Ë 16 ¯ Ë 16 ¯ Ë 16 ¯
sin x 1 cos x 3 Êpˆ Ê 3p ˆ
Soln. = , = = sin 4 Á ˜ + sin 4 Á ˜
sin y 2 cos y 2 Ë 16 ¯ Ë 16 ¯
tan x 1 Ê p 3p ˆ Êp p ˆ
= + sin 4 Á - ˜ + sin 4 Á - ˜
tan y 3 Ë 2 16 ¯ Ë 2 16 ¯
tan ( x + y ) Êpˆ Ê 3p ˆ Ê 3p ˆ Êpˆ
= sin 4 Á ˜ + sin 4 Á ˜ + cos 4 Á ˜ + cos 4 Á ˜
tan x + tan y Ë 16 ¯ Ë 16 ¯ Ë 16 ¯ Ë 16 ¯
=
1 - tan x tan y Ê Êpˆ Ê p ˆˆ Ê Ê 3p ˆ Ê 3p ˆ ˆ
= Á sin 4 Á ˜ + cos 4 Á ˜ ˜ + Á sin 4 Á ˜ + cos 4 Á ˜ ˜
tan x + 3 tan x Ë Ë 16 ¯ Ë 16 ¯ ¯ Ë Ë 16 ¯ Ë 16 ¯ ¯
=
1 - tan x .3 tan x Êpˆ Êpˆ Ê 3p ˆ Ê 3p ˆ
= 2 - 2 sin 2 Á ˜ .cos 2 Á ˜ - 2 sin 2 Á ˜ .cos 2 Á ˜
4 tan x Ë 16 ¯ Ë 16 ¯ Ë 16 ¯ Ë 16 ¯
= ......(i)
1 - 3 tan 2 x
1 ÊÊ Ê 3p ˆ ˆ ˆ
2 2
Êpˆ Ê p ˆˆ Ê Ê 3p ˆ
2
sin y = 2 sin x , cos y = cos x = 2 - 2 Á ÁË 2 sin ÁË 16 ˜¯ cos ÁË 16 ˜¯ ˜¯ + ÁË 2 sin ÁË 16 ˜¯ cos ÁË 16 ˜¯ ˜¯ ˜
3 Ë ¯

sin 2 y + cos 2 y 1Ê Êpˆ Ê 3p ˆ ˆ


= 2 - Á sin 2 Á ˜ + sin 2 Á ˜ ˜
2Ë Ë 8 ¯ Ë 8 ¯¯
4
= 4 sin 2 x + cos 2 x 1Ê Êpˆ Ê p p ˆˆ
9 = 2 - Á sin 2 Á ˜ + sin 2 Á - ˜ ˜
2Ë Ë 8¯ Ë 2 8 ¯¯
36 sin 2 x + 4 cos 2 x
= 1Ê Êpˆ Ê p ˆˆ
9 = 2 - Á sin 2 Á ˜ + cos 2 Á ˜ ˜
2Ë Ë 8¯ Ë 8 ¯¯
32 sin 2 x + 4
= 1
9 = 2-
2
32 sin 2 x + 4
fi =1 3
9 = .
2
fi 32 sin 2 x + 4 = 9
3
fi 32 sin 2 x = 5 Ex-6 If cos (a - b ) + cos (b - g ) + cos (g - a ) = - , then
2
5 cos a + cos b + cos g = 0
fi sin 2 x =
32
sin a + sin b + sin g = 0
5
fi sin x = Soln.
4 2 3
cos (a - b ) + cos (b - g ) + cos (g - a ) = -
5 2
fi tan x = ........(ii)
3 3
The Ratios and Identities 53

fi 2 (cos (a - b ) + cos (b - g ) + cos (g - a )) + 3 = 0 fi tan ( b + g ) = tan (90∞ - a )

fi 2 (cos a cos b + cos b cos g + cos g cos a ) fi tan ( b + g ) = cot a

+ 2 (sin a sin b + sin b sin g + sin g sin a ) + 3 = 0 tan b + tan g


fi = cot a
1 - tan b tan g
( ) (
fi cos 2 a + sin 2 a + cos 2 b + sin 2 b ) tan b + tan g
fi = cot b
(
+ cos g + sin g
2 2
) 1 - tan b tan g
fi tan b + tan g = cot b - tan b .cot b .tan g
2 (cos a cos b + cos b cos g + cos g cos a )
fi tan b + tan g = cot b - tan g
+ 2 (sin a sin b + sin b sin g + sin g sin a ) = 0
fi tan b + tan g = cot (90∞ - a ) - tan g
fi cos a + cos b + cos g
2 2 2
fi tan b + tan g = tan a - tan g
+2 cos a cos b + 2 cos b cos g fi tan a = tan b + 2 tan g
2 cos g cos a + sin a + sin b + sin g
2 2 2
Êpˆ
+2 sin a sin b + 2 sin b sin g Ex-9 If tan Á ˜ =
Ë 24 ¯ ( a- b )( )
c - d , where

+2 sin g sin a = 0 a, b, c, d

fi (cos a + cos b + cos g )


2 ( a + b + c + d + 2)
+ (sin a + sin b + sin g ) = 0
2 Ê p ˆ sin (p / 24)
Soln. tan Á ˜ =
Ë 24 ¯ cos (p / 24)
fi (cos a + cos b + cos g ) = 0
2 sin (p / 24) cos (p / 24)
(sin a + sin b + sin g ) = 0 =
2 cos 2 (p / 24)
Ex-7. If sin a sin b - cos a cos b + 1 = 0
sin (p / 12)
1 + cot a tan b = 0 =
1 + cos (p / 12)
Soln. sin a sin b - cos a cos b + 1 = 0
fi cos a cos b - sin a sin b = 1 3 -1
=
2 2 + 3 +1
fi cos (a + b ) = 1

=
3 -1 (2 2 - (
¥
3 +1 ))
sin (a + b ) = 1 - cos 2 (a + b ) = 1 - 1 = 0 (2 2 + ( 3 + 1)) (2 2 - ( 3 + 1))

1 + cot a .tan b ( 3 - 1) (2 2 - ( 3 + 1))


=
cos a .sin b Ê8 -
( 3 +1 ˆ)
2
= 1+ Ë ¯
sin a .cos b
sin a .cos b + cos a . sin b 2 6 - 3 - 3 - 2 2 + 3 +1
= =
sin a .cos b
(4 - 2 3 )
sin (a + b )
= 2 6 -2-2 2
sin a .cos b =
= 0. (4 - 2 3 )
Ex-8: If a + b = 90∞ b +g =a 6 -1- 2
=
tan a = tan b + 2 tan g (2 - 3 )
Soln.

b +g =a
tan ( b + g ) = tan a
= ( 6 - 2 -1 2 + 3)( )
= 2 6 - 2 2 - 2 + 18 - 6 - 3
54 Comprehensive Trigonometry with Challenging Problems & Solutions for Jee Main and Advanced

= 6+ 2- 4- 3 Ê 3 + 1ˆ
= = Á 8 2 ˜
6 -2- 3+ 2 Ë ¯

= ( 3- 2 )( 2 -1 ) Ê 3 + 1ˆ
= Á 128 ˜
Ë ¯
a = 3, b = 2, c d=1
a = 3, b c = 128
( a + b + c + d + 2).
( + b + c - 2)
a
=3+2+2+1+2
= 3 + 1 + 128 – 2
= 10.
= 130.
x y z
Ex-10 If = = 3 cot ( 20∞) - 4 cos (20∞) .
cos q Ê 2p ˆ Ê 2p ˆ Ex-12
cos Á q + ˜ cos Á q - ˜
Ë 3¯ Ë 3¯ Soln. 3 cot ( 20∞) - 4 cos (20∞)
x + y + z.
3 cos(20∞) - 4 sin(20∞) cos(20∞)
Soln. =
sin(20∞)
x y z
= = =m Ê 3 ˆ
cos q Ê 2p ˆ Ê 2p ˆ 2Á cos(20∞) - 2 sin(20∞) cos(20∞)˜
cos q + cos Á q - ˜
x + y +ÁË z 3 ˜¯ Ë 3¯ Ë 2 ¯
=
sin(20∞)
Ê Ê 2p ˆ Ê 2p ˆ ˆ
= m ¥ Á cos q + cos Á q + ˜ + cos Á q - ˜ ˜ 2 (sin(60∞) cos(20∞) - sin(40∞))
Ë Ë 3¯ Ë 3 ¯¯ =
sin(20∞)
Ê Ê 2p ˆ ˆ
= m ¥ Á cos q + 2 cos q . cos Á ˜ ˜ (2 sin(60∞) cos(20∞) - 2 sin(40∞))
Ë Ë 3 ¯¯ =
sin(20∞)
Ê Ê 1ˆˆ
= m ¥ Á cos q + 2 cos q . Á - ˜ ˜
Ë Ë 2¯ ¯ ( sin(80∞) + sin(40∞) - 2 sin(40∞))
=
= m ¥ (cos q - cos q ) sin(20∞)

=0
( sin(80∞) - sin(40∞))
=
sin(20∞)
x+y+z
2 cos(60∞)sin(20∞)
a+ b =
Ex-11 If sin ( 25∞) sin (35∞) sin (85∞) = sin(20∞)
c
+
= 1.
where a, b, c ΠI
Ex-13.
( a + b + c - 2) sin(2∞) + sin(4∞) + sin(6∞) +
Soln. sin ( 25∞) sin (35∞) sin (85∞) sin(8∞) + ............. + sin(180∞) = cot(1∞)
= sin (35∞) sin ( 25∞) sin (85∞) Soln. sin(2∞) + sin(4∞) + sin(6∞)
+ sin (8∞) + ............. + sin (180∞)
=
1
(4 sin (60∞ - 25) sin (25∞) sin (60∞ + 25∞))
4 = sin ( 2∞) + sin ( 2∞ + 2∞) + sin ( 2∞ + 2.2∞)
1
= ¥ sin (3 ¥ 25∞) + sin ( 2∞ + 3.2∞) +.... + sin (2∞ + (90 - 1) 2∞)
4
1 Ê 90∞.2∞ ˆ
= ¥ sin (75∞)
Ë 2 ˜¯
sin Á
4 Ê 2∞ ˆ
= ¥ sin Á 2∞ + (90∞ - 1) ˜
=
1
¥ cos (15∞) Ê 2∞ ˆ Ë 2¯
sin Á ˜
4 Ë 2¯
1 Ê 3 + 1ˆ
¥
= 4 ÁË 2 2 ˜¯
The Ratios and Identities 55

1 2 cos 2 (q ) + sin (2q )


= sin 1∞ ¥ sin (91∞) fi tan ( y ) =
( ) 2 cos 2 (q ) - sin (2q )
cos (1∞) 2 cos 2 (q ) + 2 sin (q ) cos (q )
= sin 1∞
( ) fi tan ( y ) =
2 cos 2 (q ) - 2 sin (q ) cos (q )
= cot (1∞)
cos (q ) + sin (q )
fi tan ( y ) =
Ex-14. cos (q ) - sin (q )
Ê p ˆ Ê 3p ˆ Ê 5p ˆ
sin Á + sin Á + sin Á Êp ˆ
Ë 2013 ˜¯ Ë 2013 ˜¯ Ë 2013 ˜¯ fi tan ( y ) = tan Á - q ˜
Ë4 ¯
Ê 7p ˆ
+ sin Á + ...upto (2013) trems p
Ë 2013 ˜¯ fi y= -q
4
Ê À 2.2Àˆ fi 4 y = p - 4q
+sin Á +
Soln. Ë 2013 2013 ˜¯ fi sin ( 4 y ) = sin (p - 4q ) = sin ( 4q ) = y
Ê p 2.2p ˆ
+ sin Á +
Ë 2013 2013 ˜¯
Ê p yˆ Ê p xˆ
Ê p 1006.2p ˆ Ex-16 If tan Á + ˜ = tan 3 Á + ˜ ,
+...... + sin Á + ˜ Ë 4 2¯ Ë 4 2¯
Ë 2013 2013 ¯
sin y 3 + sin 2 x
Ê Ê 2p ˆ ˆ =
Á Á ˜˜ sin x 1 + 3 sin 2 x
sin Á 2013. Á 2013 ˜ ˜
Á 2 ˜ Soln.
Á ˜
Ë Ë ¯¯
= Êp ˆ 1 + tan a cos a + sin a
Ê 2p ˆ tan Á + a ˜ = =
Ë4 ¯ 1 - tan a cos a - sin a
Á ˜
sin Á 2013 ˜
2 Êp ˆ 1 + sin 2a
Á ˜ fi tan 2 Á + a ˜ =
Ë ¯ Ë4 ¯ 1 - sin 2a
Ê Ê 2p ˆ ˆ
Ê p yˆ Ê p xˆ
Á p Á ˜˜ tan Á + ˜ = tan 3 Á + ˜ ,
¥ sin Á + (2012) Á 2013 ˜ ˜ Ë 4 2¯ Ë 4 2¯
Á 2013 2 ˜
Á ˜
Ë Ë ¯¯ 1 + sin y (1 + sin x )
3
fi =
= 0. 1 - sin y (1 - sin x )3

Ê1+ 1+ y ˆ
Ex-15. If tan y = Á ˜,
Ë1+ 1- y ¯
fi =
(
2 sin y 2 3 sin x + sin x
3
)
sin ( 4 y ) = y 2 (
2 1 + 3 sin 2 x )
Soln. y = sin ( 4q ) (3sin x + sin x) 3
fi sin y =
Then, tan ( y ) =
1 + 1 + sin (4q ) (1 + 3sin x) 2

1 + 1 - sin (4q )
tan a + tan g
1+ (cos (2q ) + sin (2q ))2 Ex-17 If tan b = , then
fi tan ( y ) = 1 + tan a tan g
1+ (cos (2q ) - sin (2q ))2 sin ( 2a ) + sin (2g )
sin ( 2b ) =
1 + cos (2q ) + sin (2q ) 1 + sin ( 2a ) .sin ( 2g )
fi tan ( y ) =
1 + (cos (2q ) - sin (2q ))
56 Comprehensive Trigonometry with Challenging Problems & Solutions for Jee Main and Advanced

tan a + tan g
Soln. tan b =
1 + tan a tan g Ë (
fi 4 sin(27∞) = Ê 5 + 5 - ) (3 - 5 ) ˆ¯
sin a cos g + cos a sin g a = 5, b = 5, c d=5
= a+b+c+d +2
cos a cos g + sin a sin g
=5+5+3+5+2
sin (a + g )
= = 20.
cos (a - y )
5
sin ( 2b ) =
2 tan b Ex-19 If (1 + sin q ) (1 + cos q ) =
4
1 + tan 2 b
(1 - sin q )(1 - cos q) .
Ê sin (a + g ) ˆ
2Á 5
Ë cos (a - g ) ˜¯ Soln. (1 + sin q ) (1 + cos q ) =
= 4
sin 2 (a + g )
1+ 5
cos 2 (a - g ) fi 1 + sin q + cos q + sin q cos q =
4
2 sin (a + g ) cos (a - g ) Ê t 2 - 1ˆ 5
= fi 1+ t + Á ˜=
cos (a - g ) + sin (a + g )
2 2
Ë 2 ¯ 4
sin ((a + g ) + (a - g )) + sin ((a + g ) - (a - g )) (sin q + cos q = t , say)
=
1 + sin (a + g ) - sin (a - g )
2 2
Ê t 2 - 1ˆ 1
fit +Á ˜=
sin ( 2a ) + sin ( 2g ) Ë 2 ¯ 4
=
1 + sin (a + g + a - g ) sin (a + g - a + g ) 1
fi t 2 + 2t - 1 =
sin ( 2a ) + sin ( 2g ) 2
=
1 + sin ( 2a ) sin ( 2g ) fi 2t2 + 4t - 3 = 0

-4 ± 16 + 24
fit =
( ) ( )
1/ 2 1/ 2
Ex-18 If 4 sin(27∞) = a + b - c- d 4
where a, b, c, d Œ N -4 ± 2 10 1
= = -1 ± 10
( a + b + c + d + 2) . 4 2
1
Soln. 16 sin 2 (27∞) fi t = -1 + 10
2
= 8 ¥ 2 sin 2 (27∞) 1
fi sin q + cos q = -1 + 10
= 8 ¥ (1 - cos(54∞)) 2

Ê (1 - sin q ) (1 - cos q )
10 - 2 5 ˆ
= 8 ¥ Á1 - ˜ = 1 - sin q - cos q + sin q cos q
ÁË 4 ˜¯
= 1 - (sin q + cos q ) + sin q cos q
(
= 8 - 2 10 - 2 5 ) Ê 10 ˆ 1 Ê 10 ˆ
= 1 - Á -1 + ˜ + 2 ÁË 4 - 10 ˜¯
Ë (
= Ê8 - 2 5 + 5 3 - 5 ˆ
¯ )( ) Ë 2 ¯
Ê 5ˆ
(
= Ê 5+ 5 + 3- 5 - 2 5+ 5
Ë ) ( ) ( ) (3 - 5 ) ˆ¯ = Á 2 + ˜ - 10
Ë 4¯

( ) ( )
2
Ê 13 ˆ
= Ê 5+ 5 - 3- 5 ˆ = Á - 10 ˜
Ë ¯ Ë4 ¯
The Ratios and Identities 57

p
Ex-20 If 3 sin x + 4 cos x = 5 , where x ŒÊÁ 0, ˆ˜ fi sin 2 A =
3± 5
Ë 2¯ 2
2 sin x + cos x + 4 tan x 6±2 5
fi sin 2 A =
Soln. x x=5 4
Let y x x Ê 5 - 1ˆ
2

fi sin A = Á
2
˜
y2 + 52 Ë 2 ¯
x x x x)2 Ê 5 - 1ˆ
2
x 2
x x x fi sin A = Á ˜
Ë 2 ¯
2 2
x x x x
2
Ê 5 - 1ˆ
fi y + 25 = 25 fi sin A = 2 Á ˜ = 2 sin(18∞)
Ë 4 ¯
fi y2 = 0
fi y=0
sin B = 2 sin(18∞) = sin C .
fi x x=0
Ex-22.
fi x x
Êpˆ Ê 3p ˆ Ê 5p ˆ Ê 7p ˆ
fi x = 3/4. tan 2 Á ˜ + tan 2 Á ˜ + tan 2 Á ˜ + tan 2 Á ˜
Ë 16 ¯ Ë 16 ¯ Ë 16 ¯ Ë 16 ¯
x x x
Soln.
Ê 3ˆ Ê 4 ˆ Ê 3ˆ
= 2Á ˜ + Á ˜ + 4Á ˜ = 2 + 3 = 5 .
Ë 5¯ Ë 5 ¯ Ë 4¯ Êpˆ Ê 3p ˆ Ê 5p ˆ Ê 7p ˆ
tan 2 Á ˜ + tan 2 Á ˜ + tan 2 Á ˜ + tan 2 Á ˜
Ë 16 ¯ Ë 16 ¯ Ë 16 ¯ Ë 16 ¯
Ex-21 If cos A = tan B , cos B = tan C , cos C = tan A p
Ê ˆ Ê ˆ
3p
= tan 2 Á ˜ + tan 2 Á ˜
sin A = sin B = sin C = 2 sin 18o ( ) Ë 16 ¯ Ë 16 ¯
Soln. cos A = tan B Ê p 3p ˆ Êp p ˆ
+ tan 2 Á - ˜ + tan 2 Á - ˜
fi cos A = tan B
2 2 Ë 2 16 ¯ Ë 2 16 ¯

fi cos 2 A = sec 2 B - 1 Êpˆ Êpˆ Ê 3p ˆ Ê 3p ˆ


= tan 2 Á ˜ + cot 2 Á ˜ + tan 2 Á ˜ + cot 2 Á ˜
Ë 16 ¯ Ë 16 ¯ Ë 16 ¯ Ë 16 ¯
fi 1 + cos 2 A = sec 2 B
2 2
fi 1 + cos 2 A = sec 2 B = cot 2 C Ê Êpˆ Ê p ˆˆ Ê Ê 3p ˆ Ê 3p ˆ ˆ
= Á tan Á ˜ + cot Á ˜ ˜ + Á tan Á ˜ + cot Á ˜ ˜ - 4
Ë Ë 16 ¯ Ë 16 ¯ ¯ Ë Ë 16 ¯ Ë 16 ¯ ¯
fi 1 + cos 2 A = cot 2 C
= 1 1
cos 2 C cos 2 C + -4
fi 2 - sin 2 A = = Ê p ˆ Ê p ˆ Ê 3p ˆ 2 Ê 3p ˆ
sin 2 C 1 - cos 2 C 2 2 2
sin Á ˜ cos Á ˜ sin Á ˜ cos Á ˜
Ë 16 ¯ Ë 16 ¯ Ë 16 ¯ Ë 16 ¯
tan 2 A
fi 2 - sin 2 A = 4 4
1 - tan 2 A = 2
+ 2
-4
Ê Êpˆ Ê p ˆˆ Ê Ê 3p ˆ Ê 3p ˆ ˆ
sin A2
ÁË 2 sin ÁË 16 ˜¯ cos ÁË 16 ˜¯ ˜¯ ÁË 2 sin ÁË 16 ˜¯ cos ÁË 16 ˜¯ ˜¯
fi 2 - sin 2 A =
cos A - sin 2 A
2

= 4 4
sin 2 A + -4
fi 2 - sin A = 2Êpˆ 2 Ê 3p ˆ
2
1 - 2 sin 2 A sin Á ˜ sin Á ˜
Ë 8¯ Ë 8¯
fi 2 - 4 sin 2 A - sin 2 A + 2 sin 4 A = sin 2 A 4 4
= + -4
fi 2 sin 4 A - 6 sin 2 A + 2 = 0 Êpˆ Êp pˆ
sin 2 Á ˜ sin 2 Á - ˜
fi sin 4 A - 3 sin 2 A + 1 = 0 Ë 8¯ Ë 2 8¯
3± 9- 4 =
4
+
4
-4
fi sin 2 A =
2 Êpˆ Êpˆ
sin 2 Á ˜ cos 2 Á ˜
Ë 8¯ Ë 8¯
58 Comprehensive Trigonometry with Challenging Problems & Solutions for Jee Main and Advanced

Ê ˆ fi (1 + tan(1∞)) (1 + tan(44∞)) (1 + tan(2∞)) (1 + tan(43∞))


Á 1 1 ˜
= 4Á + ˜ -4 ......(1 + tan(22∞))(1 + tan(23∞)) ¥ (1 + tan(24∞)) = 2n
Á sin 2 Ê p ˆ cos 2 Ê p ˆ ˜
ÁË ÁË ˜¯
8
ÁË ˜¯ ˜¯
8 fi 222 ¥ (1 + 1) = 2n

4 fi 2n = 223
= -4
2Êpˆ 2Êpˆ
fi n = 23.
sin Á ˜ cos Á ˜
Ë 8¯ Ë 8¯ n
8 Ex-25.
= 2
-4
Ê Êpˆ Ê p ˆˆ Ê Ê p ˆˆ Ê Ê 3p ˆ ˆ Ê Ê 7p ˆ ˆ
ÁË 2 sin ÁË 8 ˜¯ cos ÁË 8 ˜¯ ˜¯ ÁË1 + cos ÁË 10 ˜¯ ˜¯ ÁË1 + cos ÁË 10 ˜¯ ˜¯ ÁË1 + cos ÁË 10 ˜¯ ˜¯

8
-4 Ê Ê 9p ˆ ˆ 1
=
Êpˆ ÁË1 + cos ÁË 10 ˜¯ ˜¯ = 16
sin 2 Á ˜
Ë 4¯
Soln.
8 Ê 7p ˆ Ê 3p ˆ Ê 3p ˆ
= - 4 = 16 - 4 = 12 cos Á ˜ = cos Á p - ˜ = - cos Á ˜
1 Ë 10 ¯ Ë 10 ¯ Ë 10 ¯
2
Ê 9p ˆ Ê pˆ Êpˆ
cos Á ˜ = cos Á p - ˜ = - cos Á ˜
Ex-23 If sin(1∞).sin(3∞).sin(5∞)....sin(89∞) =
1 Ë 10 ¯ Ë 10 ¯ Ë 10 ¯
2n
n.
Ê Ê p ˆˆ Ê Ê 3p ˆ ˆ
Soln. sin(1∞).sin(3∞).sin(5∞)....sin(89∞) ÁË1 + cos ÁË 10 ˜¯ ˜¯ ÁË1 + cos ÁË 10 ˜¯ ˜¯
= sin(1∞).sin(3∞).sin(5∞)....sin(44∞)sin(45∞)
Ê Ê 7p ˆ ˆ Ê Ê 9p ˆ ˆ
sin(46∞).sin(47∞).sin(48∞)....sin(89∞) ÁË1 + cos ÁË 10 ˜¯ ˜¯ ÁË1 + cos ÁË 10 ˜¯ ˜¯

= sin(1∞).sin(2∞).sin(3∞)....sin(44∞)sin(45∞) Ê Ê p ˆˆ Ê Ê 3p ˆ ˆ
= Á1 + cos Á ˜ ˜ Á1 + cos Á ˜ ˜
cos(44∞) cos(43∞) cos(42∞)....cos(1∞) Ë Ë 10 ¯ ¯ Ë Ë 10 ¯ ¯
= sin(1∞)sin(2∞)sin(3∞)...sin(44∞)sin(45∞) Ê Ê 3p ˆ ˆ Ê Ê p ˆˆ
ÁË1 - cos ÁË 10 ˜¯ ˜¯ ÁË1 - cos ÁË 10 ˜¯ ˜¯
cos(1∞) cos(2∞) cos(3∞)...cos(44∞)
1 1 Ê Ê p ˆˆ Ê Ê 3p ˆ ˆ
= ¥ 44
(sin(2∞)sin(4∞)sin(6∞)...sin(88∞)) = Á1 - cos 2 Á ˜ ˜ Á1 - cos 2 Á ˜ ˜
Ë Ë 10 ¯ ¯ Ë Ë 10 ¯ ¯
2 2

=
2
1
89 / 2 (sin (2 ) sin (4 ) sin (6 )...sin (88 ))
o o o o Êpˆ Ê 3p ˆ
= sin 2 Á ˜ sin 2 Á ˜
Ë 10 ¯ Ë 10 ¯
sin(2∞)sin(4∞)sin(6∞)...sin(88∞) Ê 5 - 1ˆ Ê 5 + 1ˆ
2 2

= Á ˜ Á ˜
1 Ë 4 ¯ Ë 4 ¯
= 89 / 2
2 2 2
Ê 5 - 1ˆ Ê 5 + 1ˆ
89 = Á ˜ Á ˜
n= . Ë 4 ¯ Ë 4 ¯
2
2
Ê 5 - 1ˆ 1
Ex-24. If (1 + tan(1∞) ) (1 + tan(2∞) ) (1 + tan(3∞)) = Á =
Ë 16 ˜¯ 16
......... (1 + tan(45∞)) = 2n n. Ex-27
Soln. (1 + tan(1∞)) (1 + tan(2∞)) (1 + tan(3∞)) 1
cos(60∞) cos(36∞) cos(42∞) cos(78∞) =
......... (1 + tan(45∞)) = 2n 16
The Ratios and Identities 59

Soln.
cos(60∞) cos(36∞) cos(42∞) cos(78∞)
(
= 1 + sin 2 (sin q ) - sin 2 (cos q ) )
f (q )
1 Ê 5 + 1ˆ 1
= Á . ( 2 cos 78∞ cos 42∞)
2 Ë 4 ˜¯ 2 Ê 2 Ê Ê p ˆˆ 2Ê Ê p ˆˆˆ
= 1 + Á sin Á sin Á ˜ ˜ - sin Á cos Á ˜ ˜ ˜
Ë Ë Ë 2¯¯ Ë Ë 2¯¯¯
1 Ê 5 + 1ˆ
= (cos120∞ + cos 36∞) = 1 + sin (1)
2
4 ÁË 4 ˜¯
f (q )
1 Ê 5 + 1ˆ Ê 1 5 + 1ˆ
= Á - +
4Ë 4 ¯Ë 2˜ Á 4 ˜¯ (
= 1 + sin (sin (0)) - sin (cos (0))
2 2
)
1 Ê 5 + 1ˆ Ê 5 - 1ˆ = 1 - sin (1)
2
=
4 ÁË 4 ˜¯ ÁË 4 ˜¯ Ex-30.
(5 - 1) 4 1 f (q ) = (3 sin (q ) - 4 cos (q ) - 10)
= = =
64 64 16 (3 sin (q ) + 4 cos (q ) - 10)
Ex-28. Let f k (q ) = sin (q ) + cos (q ) .
k k
Soln. f (q ) = (3 sin (q ) - 4 cos (q ) - 10)
1 1
f 6 (q ) - f 4 (q )
6 4 (3 sin (q ) + 4 cos (q ) - 10)
f 6 (q ) = sin 6 q + cos6 q
( )
Soln.
= 9 sin (q ) - 16 cos (q )
2 2

= 1 - 3 sin q cos q
2 2

-10 (3 sin q + 4 cos q ) -10 (3 sin q - 4 cos q )


f 4 (q ) = sin 4 q + cos 4 q
= 1 - 2 sin 2 q cos 2 q (
= 9 sin 2 (q ) - 16 cos 2 (q ) )
1 1
f 6 (q ) - f 4 (q ) -10 (3 sin q + 4 cos q + 3 sin q - 4 cos q )
6 4
( )
= 9 sin (q ) - 16 cos (q ) - 60 sin (q )
2 2
=
1
6
( 1
) (
1 - 3 sin 2 q cos 2 q - 1 - 2 sin 2 q cos 2 q
4
)
= 25 sin 2 q - 60 sin (q ) - 16
1 1 2 1 1
= - sin q cos 2 q - + sin 2 q cos 2 q
= (5 sinq - 6) - 36 - 16
2
6 2 4 2
1 1
= - = (5 sinq - 6) - 52
2
6 4
1
= -
12
f (q ) = 121 – 52 = 69
Ex-29.
Ex-31. A = sin 2010 q + cos 2014 q
f (q ) = sin (sin q ) + cos (cos q )
2 2

Soln. 0 < sin 2010 q £ sin 2 q ...........(i)


Soln. sin (sin q ) + cos (cos q )
2 2
0 < cos 2014
q £ cos q 2
...........(ii)
= sin (cos q ) + cos (cos q )
2 2

+ sin 2 (sin q ) - sin 2 (cos q ) 0 < sin 2012 q + cos 2014 q £ sin 2 q + cos 2 q

(
= sin (cos q ) + cos (cos q )
2 2
) fi 0< A £ 1
+ sin 2 (sin q ) - sin 2 (cos q )
(0, 1 ]
60 Comprehensive Trigonometry with Challenging Problems & Solutions for Jee Main and Advanced

32. If
sin A
sin B
= p,
cos A
cos B
=q
tan B = ±
(q - 1)
2

p Ê q - 1ˆ
2
(1 - p )
2

tan A.tan B =
q ÁË 1 - p 2 ˜¯ tan (a - b ) sin 2 g
. Ex-33. If + =1
tan a sin 2 a
sin A cos A
Soln. = p, =q
sin B cos B tan 2 g = tan a .tan b
sin A sin B p Soln.
fi / =
cos A cos B q tan (a - b ) sin 2 g
+ =1
tan A p tan a sin 2 a
fi =
tan B q sin 2 g tan (a - b )
fi =1-
tan A tan B sin a
2
tan a
fi = =l
p q sin 2 g
sin A cos A

sin a
2
=
1
tan a
(tan a - tan (a - b ))
= p, =q
sin B cos B sin 2 a Ê tan a - tan b ˆ
fi sin 2 g = Á tan a -
sin A cos A tan a Ë 1 + tan a tan b ˜¯
fi = pq
sin B cos B
2 sin A cos A fi sin g =
2
Ê
sin 2 a tan b 1 + tan a
Á
(
2
)ˆ˜
fi = pq tan a ÁË (1 + tan a tan b ) ˜¯
2 sin B cos B
sin 2 A sin 2 a Ê tan b ˆ
fi = pq fi sin 2 g (1 + tan a tan b ) = Á ˜
sin 2 B tan a Ë cos 2 a ¯
2 tan A 2 tan B sin 2 a Ê tan b ˆ
fi / = pq fi sin 2 g (1 + tan a tan b ) = Á ˜
1 + tan A 1 + tan 2 B
2
cos 2 a Ë tan a ¯
2 pl 2q l Ê tan b ˆ
fi / = pq fi sin 2 g (1 + tan a tan b ) = tan 2 a Á
1 + p l 1 + q2l 2
2 2
Ë tan a ˜¯


p
¥
(1 + q l ) = pq
2 2
fi sin 2 g (1 + tan a tan b ) = tan a tan b
(1 + p l )
2 2 q fi (
sin 2 g = tan a tan b 1 - sin 2 g )

(1 + q l ) = q
2 2
2 fi
sin 2 g
= tan a tan b
(1 + p l )
2 2 cos 2 g

fi (1 + q l ) = q (1 + p l )
2 2 2 2 2 fi tan 2 g = tan a tan b

fi l (1 - p ) q = q - 1
2 2 2 2

Êqˆ 1- e Êjˆ
Ex-34. If tan Á ˜ = tan Á ˜ , then
fi l =
2 (q - 1)
2
Ë 2¯ 1+ e Ë 2¯

(1 - p ) q 2 2
cos j =
cos q - e
1 - e cos q
1 ( q - 1)
2

fi l=± Êqˆ 1- e Êjˆ


q (1 - p ) 2 Soln. tan Á ˜ =
Ë 2¯
tan Á ˜
Ë 2¯
1+ e

p (q - 1)
2 Ê q ˆ Ê 1 - eˆ Êjˆ
fi tan 2 Á ˜ = Á ˜ tan 2 Á ˜
tan A = ± Ë 2¯ Ë 1 + e¯ Ë 2¯
q (1 - p ) 2
The Ratios and Identities 61


1 Ê 1 - eˆ

1 1
=
(a + b)
˜ fi
Ê j ˆ Ë 1 + e¯
tan 2 Á ˜ ( a - b ) tan (j / 2)
Êqˆ Êqˆ 2
tan 2 Á ˜ tan 2 Á ˜
Ë 2¯ Ë 2¯ Ë 2¯
Êjˆ Êqˆ Êqˆ tan 2 (j / 2) (a + b)
1 - tan 2 Á ˜ 1 - e - tan 2 Á ˜ - e tan 2 Á ˜ fi =

Ë 2¯
=
Ë 2¯ Ë 2¯ tan (q / 2) ( a - b )
2

Êjˆ Êqˆ Êqˆ


1 + tan 2 Á ˜ 1 - e + tan 2 Á ˜ + e tan 2 Á ˜ (a + b) 2
Ë 2¯ Ë 2¯ Ë 2¯ fi tan 2 (q / 2) = tan (j / 2)
(a - b)
Ê 2 Ê q ˆˆ Ê 2 Ê q ˆˆ
ÁË1 - tan ÁË 2 ˜¯ ˜¯ - e ÁË1 + tan ÁË 2 ˜¯ ˜¯ (a + b)
fi cos j = fi tan (q / 2) = tan (j / 2)
Ê 2 Ê q ˆˆ Ê 2 Ê q ˆˆ
(a - b)
ÁË1 + tan ËÁ 2 ˜¯ ˜¯ - e ÁË1 - tan ÁË 2 ˜¯ ˜¯

Ê 2 Ê q ˆˆ Ex-36. If sin x + sin y = a , cos x + cos y = b


ÁË1 - tan ÁË 2 ˜¯ ˜¯
-e Ê x - yˆ 4 - a 2 - b2
Ê 2 Ê q ˆˆ tan Á = ±
ÁË1 + tan ÁË 2 ˜¯ ˜¯ Ë 2 ˜¯ a 2 + b2
fi cos j =
Ê 2 Ê q ˆˆ Soln. a 2 + b2
ÁË1 - tan ÁË 2 ˜¯ ˜¯
= (sin x + sin y ) + (cos x + cos y )
2 2
1- e
Ê 2 Ê q ˆˆ
ÁË1 + tan ÁË 2 ˜¯ ˜¯ = 2 + 2 (cos x cos y + sin x sin y )
= 2 + 2 cos ( x - y )
cos q - e
fi cos j =
1 + e cos q fi (
cos ( x - y ) = a 2 + b 2 - 2 / 2 )
a cos j + b Ê A ˆ 1 - cos A
Ex-35. If cos q = tan 2 Á ˜ =
a + b cos j Ë 2 ¯ 1 + cos A

Êqˆ a-b Êjˆ Ê x - y ˆ 1 - cos ( x - y )


tan Á ˜ = fi tan 2 Á =
Ë 2¯ a+b
tan Á ˜
Ë 2¯ Ë 2 ˜¯ 1 + cos ( x - y )

Soln. cos q =
a cos j + b
fi tan 2 Á =
(
Ê x - yˆ 1- a + b - 2 / 2
2 2
)
a + b cos j Ë 2 ˜¯ 1 + a 2 + b 2 - 2 / 2
( )
Ê 1 - tan 2 (j / 2) ˆ
2 Êqˆ Ê x - y ˆ 2 - (a + b2 - 2)
2
1 - tan Á ˜ a Á ˜ +b
Ë 1 + tan (j / 2) ¯ fi ˜=
2 2
Ë 2¯ tan ÁË
fi = 2 ¯ 2 + (a 2
+ b2 - 2)
1 + tan Á ˜ a + b 1 - tan (j / 2)
2 Êqˆ Ê 2 ˆ
Ë 2¯ Á ˜ Ê x - yˆ 4 - a - b
2 2
Ë 1 + tan (j / 2) ¯
2
fi tan 2 Á ˜ =
Ë 2 ¯ a 2 + b2
Êqˆ
1 - tan 2 Á ˜
Ë 2 ¯ (a + b ) - (a - b ) tan 2 (j / 2) Ê x - yˆ 4 - a 2 - b2
fi = fi tan Á ˜ =±
Ë 2 ¯
1 + tan Á ˜ (a + b ) + (a - b ) tan (j / 2)
2 Êqˆ
2 a 2 + b2
Ë 2¯
2 (a + b) Êpˆ
( ) ( )
2 1/ 2
fi = Ex-37. If tan Á ˜ = a + b 2 - c + d , where
2 Êqˆ -2 ( a - b ) tan (j / 2)
2 Ë 16 ¯
-2 tan Á ˜ a, b, c, d
Ë 2¯
(a + b + c + d + 1) .
62 Comprehensive Trigonometry with Challenging Problems & Solutions for Jee Main and Advanced

Soln. cos q sin q 1


Soln. + =
1 - cos 2q a b c
tan q =
sin 2q fi bc cos q + ac sin q = ab
Ê 1∞ ˆ
q = Á 22 ˜ Ê 1 - tan 2 (q / 2) ˆ Ê 2 tan (q / 2) ˆ
Ë 2¯ fi bc Á ˜ + ac Á ˜ = ab
Ë 1 + tan (q / 2) ¯ Ë 1 + tan (q / 2) ¯
2 2
Ê 1∞ ˆ 1 - cos 45∞
fi tan Á 22 ˜ =
Ë 2¯ sin 45∞ fi ( )
bc 1 - tan 2 (q / 2) + 2ac tan (q / 2)

=
1-1/ 2
= 2 -1 = ab (1 + tan 2
(q / 2))
1/ 2
1∞ fi (ab + bc ) tan 2 (q / 2) - 2ac tan (q / 2)
Let A = 11
4 + ( ab - bc ) = 0
1∞ tan (a / 2) and tan ( b / 2)
fi 2 A = 22
2 2ac
Ê 1∞ ˆ tan (a / 2) + tan (b / 2) =
fi tan (2 A) = tan Á 22 ˜ = 2 - 1 b (a + c )
Ë 2¯
(a - c )
2 tan A tan (a / 2) .tan (b / 2) =

1 - tan 2 A
= 2 -1 (a + c )
2y Êa + bˆ
tan Á
fi = 2 -1 Ë 2 ˜¯
1 - y2
2 tan (a / 2) + tan (b / 2)
fi 1 - y2 = y =
1 - tan (a / 2) .tan (b / 2)
( 2 -1)
2ac / b ( a + c )
fi 1 - y2 = 2 ( 2 +1 y ) =
1 - (a - c ) / (a + c )
fi ( 2 + 1) y - 1 = 0
y2 + 2
=
2ac / b
a+c-a+c
-2 ( 2 + 1) ± 4 ( 2 + 1)
2
+4 2ac a
fi y= = =
2 2bc b

( ) ( 2 + 1) + 1 Êa + bˆ b
2
fi y=- 2 +1 + =
Ë 2 ˜¯ a .
cot Á

fi y = -( 2 + 1) + 4 + 2 2 Êpˆ
Ex-39. sin Á ˜
Ë 14 ¯
fi y = 4 + 2 2 - ( 2 + 1)
8 x3 - 4 x 2 - 4 x + 1 = 0
Ê 1∞ ˆ
fi tan Á11 ˜ = 4 + 2 2 - ( 2 + 1)
Ë 4¯ p
Soln. Let =q
a = 4, b = 2, c = 2, d =1 14
p
(a + b + c + d + 1) fi 7q =
2
= 4 + 2 + 2 + 1 +1
p
= 10. fi 4q = - 3q
2
Ex-38. If a b q Êp ˆ
cos q sin q 1 fi sin ( 4q ) = sin Á - 3q ˜ = cos (3q )
+ = Ë2 ¯
aa + b b b c
Ê
cot Á
ˆ
= fi sin ( 4q ) = cos (3q )
Ë 2 ˜¯ a
The Ratios and Identities 63

fi 2 sin ( 2q ) cos ( 2q ) = 4 cos3 q - 3 cos q fi


tan 2 A + tan 2 B
= - tan 2C
1 - tan 2 A.tan 2 B
fi (
4 sin q cos q 1 - 2 sin 2 q ) fi tan 2 A + tan 2 B + tan 2C = tan 2 A.tan 2 B.tan 2C
(
= cos q 4 cos q - 3 2
) fi
2 tan A
+
2 tan B
+
2 tan C
1 - tan A 1 - tan B 1 - tan 2 C
2 2
fi ( ) ( )
4 sin q 1 - 2 sin 2 q = 1 - 4 sin 2 q
2 tan A 2 tan B 2 tan C

4 sin q - 8 sin q = (1 - 4 sin q )
. .
fi 3 2
1 - tan A 1 - tan B 1 - tan 2 C
2 2

fi 8 sin 3 q - 4 sin 2 q - 4 sin q + 1 = 0 2x 2y 2z


fi + +
sinq = x 1- x 2
1- y 2
1 - z2
2x 2y 2z
fi . .
8 x3 - 4 x 2 - 4 x + 1 = 0 1 - x 1 - y 1 - z2
2 2

Ex-40. 2x 2y 2z
= . .
(tan q + tan 2q + tan 3q ) , 1 - x 1 - y 1 - z2
2 2
2 2 2

¥ (cot q + cot 2q + cot 3q ) = 105


2 2 2
Ex-42. If cos a + cos b + cos g = 0
p sin a + sin b + sin g = 0
where q =
7
cos (3a ) + cos (3b ) + cos (3g ) = 3 cos (a + b + g )
Ex-41 If x + y + z = xyz
2x 2y 2z sin (3a ) + sin (3b ) + sin (3g ) = 3 sin (a + b + g )
+ +
1 - x2 1 - y2 1 - z2
2x 2y 2z Soln. Let a = cos a + i sin a , b = cos b + i sin b
= . .
1- x 1- y 1- z
2 2 2
c = cos g + i sin g
Then a + b + c
Soln. Let x = tan A , y = tan B , z = tan C
tan A + tan B + tan C = tan A.tan B.tan C = (cos a + cos b + cos g ) + i (sin a + sin b + sin g )
fi tan A + tan B = - tan C + tan A.tan B.tan C = 0 + i.0 = 0
fi tan A + tan B = - tan C (1 - tan A.tan B ) a3 + b3 + c3 = 3abc
tan A + tan B
fi = - tan C fi (cos a + i sin a )3 + (cos b + i sin b )3
(1 - tan A.tan B )
+ (cos g + i sin g )
3

fi tan ( A + B ) = - tan C
= 3 (cos a + i sin a ) (cos b + i sin b ) (cos g + i sin g )
fi tan ( A + B ) = tan (p - C )

fi ( A + B ) = (p - C ) = 3 (cos (a + b + g ) + i sin (a + b + g ))

fi A+ B+C =p fi (cos 3a + i sin 3a ) + (cos 3b + i sin 3b )


fi 2 A + 2 B + 2C = 2p + (cos 3g + i sin 3g )

fi 2 A + 2 B = 2p - 2C = 3 (cos (a + b + g ) + i sin (a + b + g ))
fi tan ( 2 A + 2 B ) = tan ( 2p - 2C ) fi (cos 3a + cos 3b + cos 3g )
fi tan (2 A + 2 B ) = - tan 2C +i (sin 3a + sin 3b + sin 3g )
64 Comprehensive Trigonometry with Challenging Problems & Solutions for Jee Main and Advanced

= 3 (cos (a + b + g ) + i sin (a + b + g )) Ê 3 1 ˆ
= cos x + 5 Á sin x - cos x˜
Ë 2 2 ¯

cos (3a ) + cos (3b ) + cos (3g ) = 3 cos (a + b + g ) = ÊÁ1 - ˆ˜ cos x +


5 5 3
sin x
Ë 2¯ 2
sin (3a ) + sin (3b ) + sin (3g ) = 3 sin (a + b + g )
3 5 3
= - cos x + sin x
2 2
9 75 84
Ex-43. + = = 21
4 4 4
tan 9∞ - tan 27∞ - tan 63∞ + tan 81∞ = 4
9 75 84
- + =- = - 21
4 4 4
Soln. tan 9∞ - tan 27∞ - tan 63∞ + tan 81∞
a = - 21, b = 21
= ( tan 9∞ + tan 81∞) - ( tan 27∞ + tan 63∞)

= ( tan 9∞ + cot 9∞) - ( tan 27∞ + cot 27∞)


Ex-45. If A = cos 2 q + sin 4 q q
Ê sin 9∞ cos 9∞ ˆ Ê sin 27∞ cos 27∞ ˆ
= Á + - + A
Ë cos 9∞ sin 9∞ ˜¯ ÁË cos 27∞ sin 27∞ ˜¯
1 1 Soln. A = cos 2 q + sin 4 q
= -
sin 9∞ cos 9∞ sin 27∞ cos 27∞ 1
( ) ( 1
)
2
= 2 cos 2 q + sin 2 q
1¥ 2 1¥ 2 2 4
= -
2 sin 9∞ cos 9∞ 2 sin 27∞ cos 27∞ 1 1
= (1 + cos 2q ) + (1 - cos 2q )2
2 2 2 4
= -
sin 18∞ sin 54∞ =
1
2
1
(
(1 + cos 2q ) + 1 - 2 cos 2q + cos2 2q
4
)
2 2
= - 1 1 1
Ê 5 - 1ˆ Ê 5 + 1ˆ = + + cos 2 2q
Á 4 ˜ Á 4 ˜ 2 4 4
Ë ¯ Ë ¯
3 1
8 8 = + cos 2 2q
= - 4 4
( 5 -1 ) ( 5 +1 ) 3 1
+ .1 = 1
Ê 5 + 1 - 5 + 1ˆ 4 4
= 8Á ˜ 3 1 3
Ë 5 -1 ¯ + .0 =
4 4 4
16
= È3 ˘
4 Í 4 ,1˙ .
Î ˚
=4
Ex-46. p
Ex-44. a b q
Ê pˆ p
a £ cos x + 5 sin Á x - ˜ £ b x.
Ë 6¯ q

qx3 - px 2 + (1 + q ) x - p = 0
Ê pˆ
Soln. cos x + 5 sin Á x - ˜
Ë 6¯
Soln. sin A sin B sin C = p
Ê Êpˆ Ê p ˆˆ
= cos x + 5 Á sin x cos Á ˜ - cos x sin Á ˜ ˜ cos A cos B cos C = q
Ë Ë 6¯ Ë 6 ¯¯
The Ratios and Identities 65

p f ¢ ( x ) = -6 sin 2 x + 8 cos 2 x
tan A tan B tan C =
q 8 3
f ¢ ( x) = 0 tan 2 x = - =-
A+ B+C =p 6 4
tan A + tan B + tan C = tan A tan B tan C 3
tan 2 x = - = tan a
p 4
tan A + tan B + tan C = Ê 3ˆ
q 2 x = n p + a , where a = tan -1 Á - ˜
Ë 4¯
tan A tan B + tan B tan C + tan C tan A
Ex-50. If ei q - log cos( x -iy ) = 1 x y
1+ q q.
=
q
Soln. ei q - log cos( x -iy ) = 1
Ê pˆ Ê1+ qˆ Ê pˆ fi eiq = cos ( x - iy )
x3 - Á ˜ x 2 + Á ˜ x-Á ˜ =0
Ë q¯ Ë q ¯ Ë q¯
fi (cos q + i sin q )
q x3 - p x 2 + (1 + q ) x - p = 0 .
= cos x cos (iy ) + sin x sin (iy )
= cos x cos (hy ) + i sin x sin (hy )
Ex-48 If x = cos10∞ cos 20∞ cos 40∞ , cos q = cos x cos (hy )
x
sin q = sin x sin ( h y )
Soln. x = cos10∞ cos 20∞ cos 40∞
cos 2 q sin 2 q
1 - =1
= (2 sin 10∞ cos10∞) cos 20∞ cos 40∞ cos 2 x sin 2 x
2 sin 10∞
cos 2 q sin 2 q
1 fi - = 1, sin x = p
= (2 sin 20∞ cos 20∞) cos 40∞ 1 - p2 p2
4 sin 10∞

=
1
(2 sin 40∞ cos 40∞)
fi ( )
p 2 1 - p 2 = p 2 cos 2 q + p 2 - 1 sin 2 q ( )
8 sin 10∞
= p - sin q
2 2

1
= (sin 80∞) fi p 4 = sin 2 q
8 sin 10∞
p = ± (sin q )
1/ 2
1 fi
= (sin 10∞)
8 sin 10∞ sin x = ± (sin q )
1/ 2

x = 2np ± (sin q )
1 1/ 2
= cot 10∞ fi
8
sin q
sin ( hy ) = = ± (sin q )
1/ 2

± (sin q )
1/ 2

x 27cos 2 x.81sin 2 x
( ( ))
Ex-49.
y = h -1 sin -1 ± (sin q )
1/ 2

Soln. Let y = 27cos 2 x.81sin 2 x LEVEL I


= 33 cos 2 x + 4 sin 2 x (PROBLEMS BASED ON FUNDAMENTALS)

y = 35 = 243
3 (sin x - cos x ) + 6 (sin x + cos x )
4 2
1
y = 3-5 =
243
cos 2 x
(
+4 sin 6 x + cos6 x )
y = 27 .81sin 2 x
2. If m = sin 6 x + cos6 x m.
Let f ( x ) = 3 cos 2 x + 4 sin 2 x 3. If cos ecq - cot q = q q.
66 Comprehensive Trigonometry with Challenging Problems & Solutions for Jee Main and Advanced

p 3p 5p 7p 9p tan 9∞ - tan 27∞ - tan 63∞ + tan 81∞ .


20 20 20 20 20 Ê pˆ Ê 2p ˆ
23. If tan a + tan Á a + ˜ + tan Á a + ˜
2 sin q Ë 3¯ Ë 3¯
5. If x =
1 + cos q + sin q = l tan 3a l.
1 - cos q + sin q 1 3
– .
1 + sin q sin 10∞ cos10∞
2
x x 1 1
q=x+ f=y+
cos x + 2 cos6 x + cos 4 x
8
x y
q1 q2 q3 (q - j ) .
q1 q2 q3.
sec 8q - 1
x x 2
x 3 q q.
sec 4q - 1
cos x - 4 cos 4 x + 8 cos 2 x
6
a b
9. If cos x + cos y + cos a = 0 = sin x + sin y + sin a a tan q + b sec q = c a + b).
Ê x + yˆ 28. If a q+b q=c a b
ÁË ˜.
2 ¯
a b
pˆ 2Ê Ê pˆ
10. If f ( x ) = sin x + sin Á x + ˜ + cos x cos Á x + ˜
2 a b.
Ë 3 ¯ Ë 3¯
A B=a A B = b,
Ê 5ˆ A + B)
gÁ ˜ =1 ( gof )( x )
Ë 4¯
2
( ) (
30. If 1 + 1 + y tan y = 1 + 1 - y , then )
x x y = y.
2 4
x x.
12. If A 2
q 4
q A.
(2 cos q - 1) (2 cos 2q - 1) (2 cos 22 q - 1)
( )
Ê 2 cos 2n q + 1ˆ
(i) 3 cos x + 4 sin x + 8 (
.... 2 cos 2n-1q - 1 = Á ) ˜
ÁË 2 cos q + 1 ˜¯
Ê pˆ
(ii) 5 cos x + 3 cos Á x + ˜ + 3
Ë 3¯ Ê 3+ 5 ˆ
cos (9∞) + sin (9∞) = Á ˜
p ÁË 2 ˜¯
2
q q.
7 È p˘
q q = a3 q q = b3 33. If sin x + cos x = , where x Œ Í0, ˙
Î 4˚
( )
2
a 2b2 a 2 + b2 .
Ê x ˆ Ê 7 - 2ˆ
tan Á ˜ = Á
x q q y q q Ë 2 ¯ Ë 3 ˜¯
xy + x – y + 1 = 0.
18. If a cos q - b sin q = c sin x 1 cos x 3
34. If = = , where
a sin q + b cos q sin y 2 cos y 2
x y z Ê pˆ
19. If = = , x, y ŒÁ 0, ˜ tan ( x + y ) = 15 .
cosq Ê 2p ˆ Ê 2p ˆ Ë 2¯
cos Á q - ˜ cos Á q + ˜
Ë 3¯ Ë 3¯
35. If sec ( x + y ) + sec ( x - y ) = 2 sec x , where
x + y + z.
Ê pˆ Ê yˆ
x, y ŒÁ 0, ˜ cos x = 2 cos Á ˜ .
b Ë 2¯ Ë 2¯
q= q+b q.
a
The Ratios and Identities 67

LEVEL II
1 + sin 2 x cos 2 x 4 sin 2 x
(MIXED PROBLEMS)
1 14. Let f (x) = sin x
2
1 + cos x 2
4 sin 2 x ,
1. If sec x = p + x x
p sin 2 x cos 2 x 1 + 4 sin 2 x
1 4
p p (c) . f (x
4p p
x x = a3 x x = b3, then a2 2
2 2 x
(a + b
ab. cos (cos x ) + sin 2 (sin x )
2
2
1
x x 2
3 3 3 3
x x x x
Êpˆ Ê 5p ˆ Ê 7p ˆ
16. If a = sin Á ˜ sin Á ˜ sin Á ˜ x
Ë 18 ¯ Ë 18 ¯ Ë 18 ¯
Ê 2p ˆ Ê 4p ˆ Ê 6p ˆ
cos Á ˜ + cos Á ˜ + cos Á ˜ y = 2 [x y = 3[x
Ë 7¯ Ë 7¯ Ë 7¯
a
x x x x 2.
8 8
x x

sin 4 q cos 4 q 1 sin 8 q cos8 q


18. If + = , then +
A x x a b a+b a3 b3
A 1
1
a 3 + b3 ( a + b )3
(c) – 2 , 2 2.
1
p (c)
8. If A + B = , A, B ( a - b )3
3
A B Êpˆ Ê 2p ˆ Ê 3p ˆ
tan Á ˜ tan Á ˜ tan Á ˜
Ë 7¯ Ë 7¯ Ë 7¯
x+b x 1
(c) 7
x 7
a+b a 2 + b2 2
20. If a b x+a x+b
a b a, b 2
x+c x+d a + b)

∞ ∞ 2bd a2 + c2
∞ ∞. b2 + d 2 2ac
5 3
x=a x+b x+c x + d, then 2ac
(c) b + d
2 2
a c d = 2. .
2bd a + c2
2
3
x x = c0 + c1 x + c2 x + c3 x 21. If sec q + tan q = 1
+ .......... + cn nx, then
n a (b - c ) x 2 + b (c - a ) x + c ( a - b ) = 0
c0 = 1/8 q q q q.
(c) c2 = –c4 22. If a
c1 = c3 = c5.
x 2 - ax + 12 = 0 , x - b x + 15 = 0
2

13. If f ( x ) = cos [p ] x + sin [p ] x


x 2 - ( a + b ) x + 36 = 0
Êpˆ cos x + cos 2 x + cos 3x = 0 , then
fÁ ˜
Ë 2¯
sin x + sin 2 x + sin 3x
68 Comprehensive Trigonometry with Challenging Problems & Solutions for Jee Main and Advanced

(4 + sec 20∞) sin 20∞


q
2
cos (cos q ) + sin 2 (sin q )
2
(c) 3 2 3
1 + sin 2 1
(c) 1 + cos 2 1 1 - cos 2 1 33. If (1 + tan 1∞) (1 + tan 2∞) ... (1 + tan 45∞) = 2n
n
24. If a + b = p / 2 b + g = a , then a
2 ( tan b + tan g )
( tan b + tan g ) (a1 , a2 , a3 )
(c) ( tan b + 2 tan g ) a1 + a2 cos ( 2 x ) + a3 sin x = 0
2

(2 tan b + tan g )

cos a1.cos a 2 .cos a 3 ..............cos a n 35. If sin (p cos q ) = cos (p sin q )


p sin ( 2q ) is
0 £ a1 , a 2 , a 3 ,................, a n £
2
cot a1. cot a 2 .cot a 3 ...............cot a n = 1
1 1 1 8 x3 - 6 x - 1 = 0
n/ 2 n (c) n
2 2 2 Êpˆ Êpˆ
p cos Á ˜ cos Á ˜
26. If A B A+ B = Ë 5¯ Ë 9¯
A B 3
Êpˆ Êpˆ
1 1 1 1 (c) cos Á ˜ cos Á ˜
(c) Ë 18 ¯ Ë 36 ¯
2 3 3 2
tan b = 2 sin a .sin g .cos ec (a + g ) , ( 3 cot (20∞) - 4 cos (20∞) )
then cot a , cot b , cot g

(c) - 3 3
2 2
sin a + sin b + sin g , where a , b , g
Ê p q ˆ a
38. If tan 2 Á + ˜ = , then sin (q )
a +b +g =p Ë 4 2¯ b
Ê a - bˆ Ê a - bˆ
ÁË ˜ -Á
a + b¯ Ë a + b ˜¯
4 cos 20∞ - 3 cot 20∞
Ê a + bˆ Ê a + bˆ
(c) Á -Á
Ë a - b ˜¯ Ë a - b ˜¯
Ê xˆ Ê xˆ cosec2 x + 25 sec2 x
4 sin 2 x + 3 cos 2 x + sin Á ˜ + cos Á ˜
Ë 2¯ Ë 2¯

4+ 2 3+ 2
sin 3 x cos3 x p
(c) 4 - 2 40. Let y = - ,0 < x <
cos x sin x 2

( 3 sin 75∞ - cos 75∞ )


1 1 tan (55∞) tan (65∞) tan (75∞)
2 2
cot ( x∞) , 0 < x < 90 , then x
(c) 2
The Ratios and Identities 69

42. If x1 and x2 Ê 2Êpˆ 2 Ê 2p ˆ 2 Ê 3p ˆ ˆ


1 ÁË tan ÁË 7 ˜¯ + tan ÁË 7 ˜¯ + tan ÁË 7 ˜¯ ˜¯
x 2 + (1 - sin q ) x - cos 2 q = 0
2
Ê Êpˆ Ê 2p ˆ Ê 3p ˆ ˆ
x12 + x22 ¥ Á cot 2 Á ˜ + cot 2 Á ˜ + cot 2 Á ˜ ˜ = 105
Ë Ë 7¯ Ë 7¯ Ë 7 ¯¯
3 + cot(76∞) cot(16∞)
= cot(44∞)
cot(76∞) + cot(16∞)
Êpˆ Ê 3p ˆ Ê 5p ˆ Ê 7p ˆ
cos 2 Á ˜ + cos 2 Á ˜ + cos 2 Á ˜ + cos 2 Á ˜ 10. If cos x + cos y + cos z = 0
Ë 8¯ Ë 8¯ Ë 8¯ Ë 8 ¯
cos (3x ) + cos (3 y ) + cos (3z ) = 12 cos x cos y cos z
Êpˆ Êpˆ Êpˆ
tan 6 Á ˜ - 33 tan 4 Á ˜ + 27 tan 2 Á ˜ = 3
Êp ˆ Ë 9¯ Ë 9¯ Ë 9¯
44. If tan x = a cot Á - a˜ 12. If cos A + cos B + cos C = 0 = sin A + sin B + sin C
Ë4 ¯ 2 2 2
A B C
Ê a - 1ˆ Ê a 2 - 1ˆ 2 2 2 3
ÁË ˜ Á 2 ˜ A B C= .
a + 1¯ Ë a + 1¯ 2
Ê a 2 + 1ˆ p
Ê a + 1ˆ 13. Let A, B, C A= B.
(c) Á 2 ˜ ÁË ˜ 4
Ë a - 1¯ a - 1¯
C=p p A, B,
1 C
45. If sin q + cos q = , 0 £ q £ p q
5
tan 3A
=k sin 3 A 2k
14. If = k
tan A sin A k - 1

LEVEL III 15. If A + B + C = p


(TOUGHER PROBLEMS FOR JEE ADVANCED) cot A + cot B + cot C - cosec A cosec B cosec C
= cot A.cot B.cot C
tan x tan 2 x + tan 2 x tan 3x + ... + tan x tan (n + 1) x 1 x
16. If tan a = tan b =
= cot x tan (n + 1) x - (n + 1)
(
x x2 + x + 1 ) (x 2
+ x +1 )
cosec x + cosec2 x + cosec4 x + ....to n terms
tan g =
(x 2
+ x +1 ) a +b =g
Ê xˆ
(
n -1
= cot ÁË ˜¯ - cot 2 x .
2
) x x

cot (16∞) cot ( 44∞) + cot ( 44∞) cot (76∞) 3 cos 2b - 1


a b cos 2a = ,
3 - cos 2b
- cot (76∞) cot (16∞) = 3
tan a : tan b = 2 : 1.
p
5. If q = n
2 cosq
2n - 1
3Êa pˆ Êb pˆ
18. If tan Á + ˜ = tan Á + ˜
(
cos (2q ) .cos ( 4q ) .cos (8q ) ....cos 2 n -1
)
q = -1 Ë 2 4¯ Ë 2 4¯

sin b =
(3 + sin a ) sin a 2

Ê 2p ˆ Ê 4p ˆ Ê 8p ˆ 7
sin Á ˜ + sin Á ˜ + sin Á ˜ = 1 + 3 sin 2 a
Ë 7¯ Ë 7¯ Ë 7¯ 2 .
1
19. If sin b = sin (2a + b )
5
Êpˆ Ê 2p ˆ Ê 7p ˆ
tan 2 Á ˜ + tan 2 Á ˜ + .... + tan 2 Á ˜ = 35 3
Ë 16 ¯ Ë 16 ¯ Ë 16 ¯ tan (a + b ) = tan a
2
70 Comprehensive Trigonometry with Challenging Problems & Solutions for Jee Main and Advanced

20. If sin x + sin y = 3 (cos x - cos y ) 6 Ê Ê 2k p ˆ Ê 2k p ˆ ˆ


 Á sin ÁË ˜¯ - i cos ÁË ˜
sin (3x ) + sin (3 y ) = 0 k =1 Ë 7 7 ¯ ˜¯
where i = -1 .
21. If sec (j - a ) ,sec j , sec (j + a )
Êaˆ 34. If cos q + cos j = a sin q + sin j = b
cos (j ) = 2 cos Á ˜
Ë 2¯ Êqˆ Êjˆ
tan Á ˜ + tan Á ˜
Ê x + yˆ Ê x - yˆ Ë 2¯ Ë 2¯
22. If tan Á ˜ , tan z , tan Á
Ë 2 ¯ Ë 2 ˜¯ tan q 1
35. If =
cos ( x ) = cos ( y ) cos ( 2 z ) tan q - tan 3q 3
cot q
sec2 q - tan q .
cot (q ) - cot (3q )
sec2 q + tan q
q.
INTEGER TYPE QUESTIONS
p
24. If q = x y z
2n + 1 1. If = = ,
cos q Ê 2p ˆ Ê 2p ˆ
( )
2n cos (q ) cos ( 2q ) cos 22 q ...cos 2n -1q ( ) cos Á q - ˜
Ë 3¯
cos Á q + ˜
Ë 3¯
( x + y + z + 4)
tan (6∞) tan ( 42∞) tan (66∞) tan (78∞)
9
Ê p rˆ
tan (a + b - g ) tan g  sin 2 ÁË ˜
26. If = 18 ¯
tan (a - b + g ) tan b r =0

sin x 1 cos x 3 Ê pˆ
sin ( b - g ) = 0 sin 2a + sin 2b + sin 2g = 0 . 3. If = and = , where x, y ŒÁ 0, ˜ .
Ë 2¯
sin y 2 cos y 2
A+ B+C =p tan 2 ( x + y )
.
sin A sin B 5
cot A + = cot B +
sin B sin C sin A sin C 4. If cos ( x - y ) , cos x , cos ( x + y )
= cot C + sin C Ê yˆ
sin A sin B sec x.cos Á ˜ = m
Ë 2¯ (m 2
)
+2 .
sin (q + A) sin (2 A)
28. If = , then Êp ˆ Ê 2p ˆ
sin (q + B ) sin (2 B ) 5. If tan x + tan Á + x˜ + tan Á + x˜
Ë3 ¯ Ë 3 ¯
tan 2 q = tan A tan B . = k tan 3x k.
29. If cos ( x - y ) = -1 Ê 2p ˆ Ê 4p ˆ
6. Let f (q ) = sin 2 q + sin 2 Á + q ˜ + sin 2 Á + q˜ .
x y x y = 0. Ë 3 ¯ Ë 3 ¯
30. If 2 cos A = cos B + cos B 3
2 A B Êpˆ
2fÁ ˜.
1 Ë 15 ¯
3
B sin ( A - B ) = ±
3 m = 3 cosec ( 20∞) - sec ( 20∞)

sin (9∞) =
1
4 ( 3+ 5 - 5- 5 ) n = sin (12∞) sin ( 48∞) sin (54∞) where
m, n ΠN
Ê p2 ˆ
f ( x ) = sin Á - x2 ˜ ( m + 8n + 2 ) .
ÁË 36 ˜¯
The Ratios and Identities 71

8. Let tan (15∞) and tan (30∞) p 6


2. If a = ’ (cos (ra ))
x 2 + px + q = 0 13 r =1

(2 + q - p ) .
44 Êpˆ Ê 3p ˆ Ê 5p ˆ Ê 7p ˆ
 cos(n∞) sin Á ˜ sin Á ˜ sin Á ˜ sin Á ˜
Ë 14 ¯ Ë 14 ¯ Ë 14 ¯ Ë 14 ¯
n =1
9. Let x = 44
 sin(n∞) Ê 9p ˆ Ê 11p ˆ Ê 13p ˆ
sin Á ˜ sin Á ˜ sin Á
n =1 Ë 14 ¯ Ë 14 ¯ Ë 14 ˜¯

[ x + 3]
Êpˆ Ê 5p ˆ Ê 7p ˆ
sin(25∞)sin(35∞)sin(85∞) sin Á ˜ sin Á ˜ sin Á
Ë 18 ¯ Ë 18 ¯ Ë 18 ˜¯

a + b where a, b, c ΠN
c
Êpˆ Êpˆ Êpˆ Êpˆ Êpˆ
Ê c ˆ 64 3 sin Á ˜ cos Á ˜ cos Á ˜ cos Á ˜ cos Á ˜
ÁË + 2˜ Ë 48 ¯ Ë 48 ¯ Ë 24 ¯ Ë 12 ¯ Ë 6¯
a+b ¯
17
Ê kpˆ
11. Let m = Â cos Á
k =1 Ë 9 ˜¯
PASSAGE II
(m 2
+m+2 . ) Êpˆ Ê 3p ˆ Ê 5p ˆ
If cos Á ˜ , cos Á ˜ , cos Á ˜
tan(55∞) tan(65∞) tan(75∞) Ë 7¯ Ë 7¯ Ë 7¯
x∞ m 8 x3 - 4 x 2 - 4 x + 1 = 0 .
tan (27∞) + tan (18∞) + tan (27∞) tan (18∞)
(m + x + 1) .
Êpˆ Ê 3p ˆ Ê 5p ˆ
sec Á ˜ + sec Á ˜ + sec Á ˜
LINK COMPREHENSION TYPE Ë 7¯ Ë 7¯ Ë 7¯
(FOR JEE ADVANCED EXAM ONLY)

PASSAGE I Êpˆ Ê 3p ˆ Ê 5p ˆ
sin Á ˜ sin Á ˜ sin Á ˜
Ë 14 ¯ Ë 14 ¯ Ë 14 ¯

cos a .cos 2a .cos 22 a ....cos 2n-1a 7 7


.
4 8
Ï sin 2n a
Ô n :if a π np
Êpˆ Ê 3p ˆ Ê 5p ˆ
Ô 2 sin a cos Á ˜ cos Á ˜ cos Á ˜
Ô1 p Ë 14 ¯ Ë 14 ¯ Ë 14 ¯
=Ì n : if a = n
Ô2 2 +1 7 7
Ô 1 p
Ô - n : if a = n
4 8
Ó 2 2 -1
,
where n Êpˆ Ê 3p ˆ Ê 5p ˆ
tan 2 Á ˜ , tan 2 Á ˜ , tan 2 Á ˜
Ë 7¯ Ë 7¯ Ë 7¯
x3 - 35 x 2 + 7 x - 21 = 0
Ê 2p ˆ Ê 4p ˆ Ê 6p ˆ
cos Á ˜ cos Á ˜ cos Á ˜ x3 - 35 x 2 + 21x - 7 = 0
Ë 7¯ Ë 7¯ Ë 7¯
(c) x3 - 35 x 2 + 35 x - 7 = 0
x3 - 21x 2 + 7 x - 35 = 0
72 Comprehensive Trigonometry with Challenging Problems & Solutions for Jee Main and Advanced

PASSAGE VI
3 Ï Ê 2r - 1ˆ ¸ Ï 2 Ê 2r - 1ˆ ¸
3
22 p
Â Ì tan 2 ÁË ˜ p ˝ ¥  Ìcot ÁË ˜p˝ Let sec x + tan x = , where 0 < x <
r =1 Ó 7 ¯ ˛ r =1 Ó 7 ¯ ˛ 7 2
Then
Ê xˆ
tan Á ˜
Ë 2¯
PASSAGE III
Let x 2 + y 2 = 1 x, y in R.
Then
Ê 1 - cos x ˆ
( ) + (3 y - 4 y )
2 3 2
P = 3 x - 4 x3 Á1 - 1 + cos x ˜
Ë ¯

Q = x6 + y 6
(cosec x + cot x )
R = x2 + y 4

MATCH MATRIX
PASSAGE IV

Column I Column II
P ( x ) = ( x - cos 36∞) ( x - cos 84∞) ( x - cos156∞) p
Then (A) If q + j = , where q f
2
x2


Ê 5 - 1ˆ (sin q + sin j ) sin ÊÁË ˜ (P) 1
Á 2 ˜ 4¯
Ë ¯

x (B) If sin q - sin j = a


cos q + cos j = b , then (R) 3

P(x a 2 + b2
Ê 5 - 1ˆ Ê 5 - 1ˆ (C) If 3 sin q + 5 cos q = 5, (q π 0),
Á 4 ˜ Á 16 ˜
Ë ¯ Ë ¯
Ê 5 + 1ˆ 5 sin q - 3 cos q
1
(c) Á ˜
Ë 16 ¯ 16 Column I Column II
3 /8
PASSAGE V cos(20∞).cos(40∞).cos(80∞)
a, b Œ(0,1)
3 /16
If a sin x + b cos x = 1 a 2 + b2 = 1 cos(20∞).cos(40∞).cos(60∞)
Then .cos(80∞)
x
3 / 32
a b sin(20∞).sin(40∞).sin(80∞)
(c) a/b b/a
sin(20∞).sin(40∞).sin(60∞)
a b
(c) a/b b/a .sin(80∞)
x (T) 1/8
a b
(c) a/b b/a
The Ratios and Identities 73

Column I Column II tan(6∞).tan(42∞).tan(66∞).tan(78∞)

7 + 6 tan q - tan 2 q
(P) l + m = 2 Column I ColumnII
1 + tan 2 q
p
Ê pˆ If A + B = (1 + tan A) (1 + tan B )
q Áπ ˜ 4
Ë 2¯
l m
l-m=6
(1 + tan 21∞).(1 + tan 22∞).(1 + tan 23∞)
. (1 + tan 24∞)

Ê pˆ
5 cos q + 3 cos Á q + ˜ + 3
Ë 3¯ (1 + tan 2058∞).(1 - tan 2013∞)
q l+m=6
l m
Ê Êp ˆˆ Ê Ê p ˆˆ
ÁË1 + tan ÁË 8 - x˜¯ ˜¯ . ÁË1 + tan ÁË x + 8 ˜¯ ˜¯ .

Êp ˆ Êp ˆ
1 + sin Á + q ˜ + 2 cos Á - q ˜
Ë4 ¯ Ë4 ¯ (1 + tan 235∞).(1 - tan 190∞). (T) –4
q
l l - m = 14
Column I Column II
Column I Column II
ABC 2 tan (50∞) + tan ( 20∞)
A B C
A B tan ( 40∞) + 2 tan (10∞)
C
A B C tan(20∞).tan(40∞).tan(60∞).tan(80∞)
A
B C x x
(C) sin 2 A + sin 2 B + sin 2 C
A x x x
B C
(D) cos 2 A + cos 2 B + cos 2 C Column I Column II
A
B C 2 sin 2 q + 3 cos 2 q

Column I ColumnII sin 2 q + cos 4 q

Êpˆ Ê 3p ˆ Ê 5p ˆ
cos 4 Á ˜ + cos 4 Á ˜ + cos 4 Á ˜ sin 4 q + cos 2 q
Ë 8¯ Ë 8¯ Ë 8¯
Ê 7p ˆ sin 2014 q + cos 2010 q
+ cos 4 Á ˜
Ë 8 ¯
Column I Column II
If a b
sin(12∞).sin(48∞).sin(54∞)
a cos q + b sin q = c , then
c2 - b2
sin a + sin b
sin(6∞).sin(42∞).sin(66∞).sin(78∞) a 2 + b2
74 Comprehensive Trigonometry with Challenging Problems & Solutions for Jee Main and Advanced

2ac A tan a + 2 tan (2a )


sin a .sin b
a +b
2 2
+4 tan (4a ) + 8 tan (8a ) -16 cot (16a ) = cot a
c2 - a2
cos a + cos b R cot a - tan a = 2 cot 2a
a 2 + b2 A B (c) C D
2bc A
cos a .cos b
a 2 + b2 Ê pˆ Ê 4p ˆ
cos 2 a + cos 2 Á a + ˜ + cos 2 Á a + ˜
Ë 3¯ Ë 3¯
Column I ColumnII
Ê 2p ˆ Ê 4p ˆ
= 3 cos a cos Á a + ˜ cos Á a + ˜
Ë 3 ¯ Ë 3¯
a + b + c = 0 , then a3 + b3 + c3 = 3abc
A B (c) C D

Êpˆ Êpˆ Êpˆ


A tan (5q ) - tan (3q ) - tan (q )
2 tan Á ˜ + 3 sec Á ˜ - 4 cos Á ˜
Ë 10 ¯ Ë 10 ¯ Ë 10 ¯ = tan (5q ) .tan (3q ) .tan (q )
R x = y + z, then
x y z x y z
3 cot(20∞) - 4 cos(20∞) A B (c) C D
A sin q + cos q
tan(20∞) + 2 tan(50∞) - tan(70∞) R
sinq cosq
A B (c) C D
ASSERTION & REASON n
A ’ cos (a i )
CODES i =1

A R R p 1
0 £ a1 , a 2 , a 3 ,..., a n £ n/ 2
A. 2 2
A R R n
A. R ’ cot (a i ) = 1
(C) A R i =1

(D) A R A B (c) C D
1 A A+B+C=p
A sinq = x + x ΠR - {0}.
x
R A.M ≥ G.M A B 3 3 C
A B (c) C D R A.M ≥ G.M
A A DABC , then A B (c) C D
B C>1 4 xy
A sec q =
2

R DABC , tan A =
tan B + tan C ( x + y )2
tan B tan C - 1 x y x=y
A B (c) C D R sec q ≥ 1 .
2

Ê 2p ˆ Ê 4p ˆ Ê 6p ˆ 1 A B (c) C D
A sin Á ˜ + sin Á ˜ + sin Á ˜ = -
Ë 7 ¯ Ë 7¯ Ë 7¯ 2
(QUESTIONS ASKED IN
Ê 2p ˆ Ê 2p ˆ th
R cos Á ˜ + i sin Á ˜ PAST IIT-JEE EXAMS)
Ë 7¯ Ë 7¯
A B (c) C D sin x sin y sin ( x - y ) + sin y sin z sin ( y - z )
+ sin z sin x sin ( z - x )
+ sin ( x - y ) sin ( y - z ) sin ( z - x ) = 0
The Ratios and Identities 75

4 5 3 cosec(20∞) - sec(20∞)
2. If cos (a + b ) = , sin (a - b ) = a, b
5 13
p 2 sin 20∞
a. sin 40∞
4
4 sin 20∞
A = sin 2 q + cos 4 q q, then .
£A£ £A£1 sin 40∞
(c) 13/6 £ A £ £ A £ 13/6.

1 - cos B tan a + 2 tan 2a + 4 tan 4a + 8 cot 8a = cot a .


4. If tan A = A B
sin B
n
sin 3 x sin 3x = Â Cm cos (m x )
m=0 Êpˆ Ê 3p ˆ Ê 5p ˆ Ê 7p ˆ
sin Á ˜ sin Á ˜ sin Á ˜ sin Á ˜
in x, when C0 , C2 ,.........., Cn Cn π 0 Ë 14 ¯ Ë 14 ¯ Ë 14 ¯ Ë 14 ¯
n
Ê 9p ˆ Ê 11p ˆ Ê 13p ˆ
sin Á ˜ sin Á ˜ sin Á
Ë 14 ¯ Ë 14 ¯ Ë 14 ˜¯
1
sin 12∞ sin 54∞ sin 48∞ =
8
a +b +g =p f ( x ) = cos ÈÎp 2 ˘˚ x + cos ÈÎ-p 2 ˘˚ x , where

sin 2 a + sin 2 b - sin 2 g = 2 sin a .sin b .sin g


Êpˆ f (p ) = 1
f Á ˜ =1
Ë 2¯
Êpˆ
Ê 2p ˆ Ê 4p ˆ Ê 8p ˆ Ê 16p ˆ
16 cos Á ˜ cos Á ˜ cos Á ˜ cos Á =1 (c) f ( -p ) = 0 f Á ˜ =1
Ë 15 ¯ Ë 15 ¯ Ë 15 ¯ Ë 15 ˜¯ Ë 4¯

Column I Column II
Ê Ê p ˆˆ Ê Ê 3p ˆ ˆ Ê Ê 5p ˆ ˆ
ÁË1 + cos ÁË 8 ˜¯ ˜¯ ÁË1 + cos ÁË 8 ˜¯ ˜¯ ÁË1 + cos ÁË 8 ˜¯ ˜¯ Ê 13p 14p ˆ
ÁË , ˜
48 48 ¯
Ê Ê 7p ˆ ˆ
ÁË1 + cos ÁË 8 ˜¯ ˜¯ Ê 14p 18p ˆ
ÁË , ˜
48 48 ¯
p 1+ 2
cos . Ê 18p 23p ˆ
8 2 2 (C) ÁË , ˜
48 48 ¯
Ê pˆ
(D) ÁË 0, ˜¯
2
È Ê 3p ˆ ˘
3 Ísin 4 Á - a ˜ + sin 4 (3p + a )˙
Î Ë 2 ¯ ˚ Êpˆ Ê 5p ˆ Ê 7p ˆ
19. If k = sin Á ˜ sin Á ˜ sin Á ˜ , then
È Êp ˆ ˘ Ë 18 ¯ Ë 18 ¯ Ë 18 ¯
– 2 Ísin 6 Á + a ˜ + sin 6 (5p - a )˙
Î Ë 2 ¯ ˚ k

p
sin 4a + cos a . 20. If A > 0, B A+ B =
A B 3

p
21. Let o < x < , then (sec 2 x - tan 2 x)
4
76 Comprehensive Trigonometry with Challenging Problems & Solutions for Jee Main and Advanced

Ê pˆ Êp ˆ £0 q £0
tan Á x - ˜ tan Á - x˜
Ë 4¯ Ë4 ¯
cos a1.cos a 2 ......cos a n
Êp ˆ Êp ˆ
(c) tan Á + x˜ tan 2 Á + x˜
Ë4 ¯ Ë4 ¯ p
0 £ a1 , a 2 ,......., a n £
2
cot a1.cot a 2 ......cot a n = 1
x x)4 x x)2 6
x 6
x 1 1
2n/ 2 2n
1
(c)
sin a + sin b + sin g , where a , b , g 2n
a +b +g =p
p
31. If a + b = a = b +g a
2
2 ( tan b + tan g ) tan b + tan g
Ê 4 xy ˆ (c) ( tan b + 2 tan g ) 2 tan b + tan g
24. sec2 q = Á 2˜
Ë (x + y) ¯
Ê pˆ
x+y x = y, x π 0 32. If a ŒÁ 0, ˜
Ë 2¯
(c) x = y x π 0, y π 0.
tan 2 a
x x 2
(x y = x2 + x +
2
1) with x2 + x

2 tan a
2
1) sec2 a

Êp 2 ˆ
1
q f sinq = ,
ÁË , - sin 1˜¯ x 2
2
1
cosj = (q + j )
3
sin15∞ cos15∞
(c) sin 15∞ cos15∞ sin 15∞ cos 75∞
Êp pˆ Ê p 2p ˆ
ÁË , ˜¯ ÁË , ˜¯
n 3 2 2 3
Êqˆ Ê 2p 5p ˆ Ê 5p ˆ
f n (q ) = tan Á ˜ (1 + sec q ) (1 + sec 2q ) ...... (c) Á ÁË , p ˜¯
Ë 3 6 ˜¯
,
Ë 2¯ 6
(1 + sec 4q )........(1 + sec 2n q ) , then
t
Êpˆ Êpˆ 1 - 2 x + 5x 2
Ê p pˆ
f2 Á ˜ = 1 f3 Á ˜ = 1 2 sin t = , t ŒÁ - , ˜
Ë 16 ¯ Ë 32 ¯ Ë 2 2¯
3x - 2 x - 1
2

Êpˆ Ê p ˆ
(c) f 4 Á ˜ = 1 f5 Á =1
Ë 64 ¯ Ë 128 ˜¯ 1
35. cos (a - b ) = 1 and cos (a + b ) = , where
e
a , b Œ[ -p , p ] a, b
28 Let f (q ) = sin q (sin q + sin 3q ) , then f (q )
≥ 0 when q ≥ 0
£0 ≥0
(c) £ 0 q £0
The Ratios and Identities 77

Ê pˆ tan q cot q QÀP


36. Let q ŒÁ 0, ˜ t1 = ( tan q ) , t2 = ( tan q )
Ë 4¯ (c) P À Q
tan q cot q P=Q
t3 = (cot q ) , t4 = (cot q ) , then
n
t1 > t2 > t3 > t4 t4 > t3 > t1 > t2
(c) t3 > t1 > t2 > t4 t2 > t3 > t1 > t4 1 1 1
= +
Êpˆ Ê 2p ˆ Ê 3p ˆ
sin Á ˜ sin Á ˜ sin Á ˜
Ë n¯ Ë n¯ Ë n ¯
sin 4 x cos 4 x 1 , then
+ =
2 3 5
41. Let f : ( -1,1) Æ R
2
f (cos 4q ) =
2 sin 8 x cos8 x 1 2 - sec 2 q
tan 2 x = + =
3 8 27 125 Ê pˆ Êp pˆ
q ŒÁ 0, ˜ » Á , ˜
(c) sin x + cos x = 2
8 8
1 Ë 4¯ Ë 4 2¯
tan 2 x =
8 27 125 3 Ê 1ˆ
fÁ ˜
Ë 3¯

1 3 3
1- 1+
sin q + 3 sin q cos q + 5 cos q
2 2
2 2

{
39. Let P = q : sin q - cos q = 2 cos q } (c) 1 -
2
3
1+
2
3
{
Q = q : sin q + cos q = 2 sin q }
PÃQ Q-Pπj

ANSWERS
EXERCISE 1 EXERCISE 2
5 10. 3/5.
1. p
12 11. 5/13.
20p
2. EXERCISE 3
3
4. 3
5. –1
6. 1
6. 81∞, 9∞ 1
p 4p 5p 2p 9. 9
, , , . 2
3 9 9 3
10. 8
p p 5p 11. 5
8. , ,
9 3 9 1
12. 5
9. 6p 2
15. q = 120∞, 240∞

12. p/2 5p
16. q =
13. 1080 ft 3
21. 0
4p 22. –1
15. .
3 23. 0
78 Comprehensive Trigonometry with Challenging Problems & Solutions for Jee Main and Advanced

EXERCISE 4 8. [ -1, 5]
1. 6 9. 9
2. 10 V = 1, Min V = 1/2
3. 20 V = 1, Min V = 1/4
4. 5 12. Min V V
5. 10 13. Min V
5 5
EXERCISE 6 V= , Min V =
2 + 10 2 - 10
16. –1.
V = 1, Min V = 3/4
Ê b2 - a 2 ˆ
Á 2 ˜. V = 1, Min V = 3/4
Ë b + a2 ¯
V = (a + b)
2

1Ê 1ˆ 18. Min V = 4
18. Á xy + ˜ .
2Ë xy ¯ 19 Min V = 8
20 Min V
20. 1.
EXERCISE 9
15. 11
EXERCISE 7 16. 2013
18. 5/12
2c 19. 1/4
19. 25. 2
1 - c2
4p
21 0 26. x =
3
22. 1/8
Level I
23. 1/2
1. 13
34. 3
È1 ˘
35. 11. 2. Í ,1˙
Î4 ˚
EXERCISE 8 1 Ê 1ˆ
V = 10, Min V = 0
3. ÁË q + q ˜¯
2
V = 15, Min V = 5 4. 1
V V=1 5. x
V V=2 6. 1
V= 2 , Min V = - 2 8. 4
V= 2 , Min V = - 2 a
10. 1
V V
11. 1
V V=0
È3 ˘
12. Í ,1˙
V= 2 , Min V = - 2 Î4 ˚
V= 2 - sin 1 , Min V = - 2 + sin 1 13. (i) 3, 13 (ii) – 4, 10
V = 8, Min V = –2 14. P 2 - p
2

V = 4 + 10 , Min V = 4 - 10 15. 4
V = 4, Min V =2 16. 1
V = 10 18. ± (a 2 + b 2 - c 2 )
19. 0
The Ratios and Identities 79

È 1˘
21. b 32. R f = Í0, ˙
22. 4 Î 2˚
23. 3 33. i
24. 4 Ê 4b ˆ
34. Á 2
1 Ê x + yˆ Ë a + 2a + b 2 ˜¯
25. Á ˜
2 Ë y x¯ 35. 2/3
2ac Integer Type Questions
1. 4 2. 5 3. 3 4. 4
a2 - c2
2b c-a
28. (i) (ii)
c+a c+a Comprehensive Link Passages
b2 - a 2
29.
b2 + a 2
Level II

(Match Matrix)
1. (A) Æ Æ (S, T); (C) Æ (R)
2. (A) Æ T; (B) Æ S; (C) Æ P; (D) Æ
3. (A) Æ (R, S); (B) Æ (R, T); (C) Æ
4. (A) Æ P; (B) Æ Æ R, (D) Æ S
5. (A) Æ (R); (B) Æ (P); (C) Æ (S), (D) Æ (T)
6. (A) Æ Æ (P); (C) Æ (P), (D) Æ (P)
Æ (R); (B) Æ (S); (C) Æ (P), (D) Æ
8. (A) Æ (R); (B) Æ (P); (C) Æ Æ (P)
9. (A) Æ (S); (B) Æ (R); (C) Æ Æ (P)
Level III 10. (A) Æ (S); (B) Æ (P); (C) Æ Æ (P)
13. p Œ (- •, 3 - 2 2] » [3 + 2 2 , •) . Assertion and Reason
16. q x - p x + (1 + q ) x - p = 0
3 2

HINTS AND SOLUTIONS ..... ..... ......


..... ..... ......
LEVEL III
tan n x tan (n + 1) x = cot x ( tan (n + 1) x - tan nx ) - 1
x
= tan (2x - x )
tan 2 x - tan x = cot x ( tan (n + 1) x - tan x ) - n
=
1 + tan x tan 2 x = cot x ( tan (n + 1) x ) - (n + 1)
tan x (1 + tan x tan 2 x ) = tan 2 x - tan x
(1 + tan x tan 2 x ) = cot x ( tan 2 x - tan x ) cosec x + cosec2 x + cosec4 x + ....to n terms
tan x tan 2 x = cot x ( tan 2 x - tan x ) - 1 Ê xˆ
(
= cot Á ˜ - cot 2n -1 x .
Ë 2¯ )
tan 2 x tan 3x = cot x ( tan 3x - tan 2 x ) - 1
tan 3x tan 4 x = cot x ( tan 4 x - tan 3x ) - 1 cosec x + cosec2 x + cosec4 x + ....to n terms
80 Comprehensive Trigonometry with Challenging Problems & Solutions for Jee Main and Advanced

1 sin ( x / 2) 1 È cos (46∞) - cos (44∞) ˘


cos ec x = = = Í ˙
sin x sin x.sin ( x / 2) 2 Î sin (16∞)sin(44∞)sin (76∞) ˚
xˆ =0
Ê
sin Á x - ˜
Ë 2¯
=
sin x.sin ( x / 2)
sin x cos ( x / 2) - cos x sin ( x / 2) ( ) ( )
cos q .cos (2q ) .cos 22 q cos 23 q ...cos 2n -1q ( )
=
sin x.sin ( x / 2)
=
( )
sin 2n q
= cot ( x / 2) - cot x 2n sin (q )
cosec ( 2 x ) = cot ( x ) - cot ( 2 x )
sin (p + q )
cosec ( 4 x ) = cot ( 2 x ) - cot ( 4 x ) =
2n sin (q )
.... .... .... ....
sin ( q )
.... .... .... .... = -
2n sin (q )
( ) ( )
cosec 2n -1 x = cot 2n - 2 x - cot 2n -1 x ( ) 1
= -
2n
cosec x + cosec2 x + cosec4 x + ....to n terms
Ê xˆ
(
n -1
= cot ÁË ˜¯ - cot 2 x
2
) ( ) ( )
2n cos q .cos ( 2q ) .cos 22 q cos 23 q ...cos 2n -1q ( )
cos ( A ± B ) = –1.
cot ( A) cos ( B ) - 1 =
sin A sin B
cos(60∞) Ê 2p ˆ Ê 4p ˆ Ê 8p ˆ
6. Let y = sin Á ˜ + sin Á ˜ + sin Á ˜
cot(16∞) cot(44∞) - 1 = Ë 7¯ Ë 7¯ Ë 7¯
sin (16∞)sin(44∞)
cos(120∞) Ê Ê 2p ˆ Ê 4p ˆ Ê 8p ˆ ˆ
2
cot(76∞) cot(44∞) - 1 = y 2 = Á sin Á ˜ + sin Á ˜ + sin Á ˜ ˜
sin (76∞)sin(44∞) Ë Ë 7¯ Ë 7¯ Ë 7 ¯¯
cos(60∞)
cot(76∞) cot(16∞) - 1 = Ê 2p ˆ Ê 4p ˆ Ê 8p ˆ
sin (76∞)sin(16∞) = sin 2 Á ˜ + sin 2 Á ˜ + sin 2 Á ˜
Ë 7¯ Ë 7¯ Ë 7¯
Ê Ê2p ˆ Ê 4p ˆ Ê 4p ˆ Ê 8p ˆ Ê 2p ˆ Ê 8p ˆ ˆ
cos(60∞) cos(120∞) cos(60∞)) + 2 Á sin ÁË ˜¯ sin ÁË ˜¯ + sin ÁË ˜¯ sin ÁË ˜¯ + sin ÁË ˜¯ sin ÁË ˜¯ ˜¯
= + - Ë 7 7 7 7 7 7
sin (16∞)sin(44∞) sin (76∞)sin(44∞) sin (76∞)sin(16∞)
= y1 + y2 (say )
cos(60∞)sin (76∞) + cos(120∞)sin (16∞) - cos(60∞)sin (44∞)
=
sin (16∞)) sin(44∞)sin (76∞) Ê 2p ˆ Ê 4p ˆ Ê 8p ˆ
y1 = sin 2 Á ˜ + sin 2 Á ˜ + sin 2 Á ˜
Ë 7¯ Ë 7¯ Ë 7¯
1 È sin (76∞) - sin (16∞) - sin (44∞) ˘
= Í ˙ Êpˆ Ê 2p ˆ Ê 4p ˆ
2 Î sin (16∞)sin(44∞)sin (76∞) ˚ = sin 2 Á ˜ + sin 2 Á ˜ + sin 2 Á ˜
Ë 7¯ Ë 7¯ Ë 7¯
1 È (sin (76∞) - sin (16∞)) - sin (44∞) ˘
= Í ˙ 1È Êpˆ Ê 2p ˆ Ê 4p ˆ ˘
2 Î sin (16∞)sin(44∞)sin (76∞) ˚ = 2 sin 2 Á ˜ + 2 sin 2 Á ˜ + 2 sin 2 Á ˜ ˙
2 ÍÎ Ë 7¯ Ë 7¯ Ë 7 ¯˚
1 È 2 sin (30∞) - cos (46∞) - sin (44∞) ˘
= Í ˙ 1È Ê 2p ˆ Ê 4p ˆ Ê 8p ˆ ˘
2 Î sin (16∞)sin(44∞)sin (76∞) ˚ = 3 - cos Á ˜ - cos Á ˜ - cos Á ˜ ˙
Í
2Î Ë 7¯ Ë 7¯ Ë 7 ¯˚
1 È cos (46∞) - sin (44∞) ˘
= Í ˙ 1È Ê 2p ˆ Ê 4p ˆ Ê 6p ˆ ˘
2 Î sin (16∞)sin(44∞)sin (76∞) ˚ = 3 - cos Á ˜ - cos Á ˜ - cos Á ˜ ˙
Í
2Î Ë 7¯ Ë 7¯ Ë 7 ¯˚
The Ratios and Identities 81

1È Ê 2p ˆ Ê 4p ˆ Ê 6p ˆ ˘ Ê Êpˆ Ê p ˆˆ Ê
2
Ê 2p ˆ Ê 2p ˆ ˆ
2
= Í3 - cos Á ˜ + cos Á ˜ + cos Á ˜ ˙ = Á tan Á ˜ + cot Á ˜ ˜ + Á tan Á ˜ + cot Á ˜ ˜
2Î Ë 7¯ Ë 7¯ Ë 7 ¯˚ Ë Ë 16 ¯ Ë ¯
16 ¯ Ë Ë 16 ¯ Ë 16 ¯ ¯
1 È Ê 1ˆ ˘ 2
= 3- Á- ˜ Ê Êpˆ Ê p ˆˆ
2 ÍÎ Ë 2 ¯ ˙˚ + Á tan Á ˜ + cot Á ˜ ˜ - 2 - 2 - 2 + 1
Ë Ë 16 ¯ Ë 16 ¯ ¯
7
= 2 2
4 Ê ˆ Ê ˆ
Á 1 ˜ Á 1 ˜
y2 = Á ˜ +Á ˜
Á sin Ê p ˆ cos Ê p ˆ ˜ Á sin Ê p ˆ cos Ê p ˆ ˜
Ê 2p ˆ Ê 4p ˆ Ê 4p ˆ Ê 8p ˆ Ê 2p ˆ Ê 8p ˆ ˆ ÁË ÁË 16 ˜¯ ÁË ˜¯ ˜¯ ÁË ÁË 16 ˜¯ ÁË ˜¯ ˜¯
= 2 Á sin ÊÁ ˜¯ sin ÁË ˜¯ + sin ÁË ˜¯ sin ÁË ˜¯ + sin ÁË ˜¯ sin ÁË ˜¯ ˜¯
16 16
Ë Ë 7 7 7 7 7 7
2
Ê ˆ
È Ê 2p ˆ Ê 6p ˆ Ê 4p ˆ Ê 12p ˆ Á ˜
= Ícos Á ˜ - cos Á ˜ + cos Á ˜ - cos Á
Ë 7 ˜¯
1
Î Ë 7¯ Ë 7¯ Ë 7 ¯ +Á ˜ -5
Á sin Ê p ˆ cos Ê p ˆ ˜
Ê 6p ˆ Ê 10p ˆ ˘ ÁË ÁË 16 ˜¯ ÁË ˜¯ ˜¯
+ cos Á ˜ - cos Á 16
Ë 7¯ Ë 7 ˜¯ ˙˚
4 4 4
= + + -5
p
Ê ˆ Ê ˆ
2p Ê 3p ˆ
È Ê 2p ˆ Ê 6p ˆ Ê 4p ˆ Ê 2p ˆ sin 2 Á ˜ sin 2 Á ˜ sin 2 Á ˜
= Ícos Á ˜ - cos Á ˜ + cos Á ˜ - cos Á ˜ Ë 8¯ Ë 8 ¯ Ë 8¯
Î Ë 7¯ Ë 7¯ Ë 7 ¯ Ë 7 ¯
4 4 4
Ê 6p ˆ Ê 4p ˆ ˘ = + + -5
+ cos Á ˜ - cos Á ˜ ˙ Êpˆ Êpˆ Ê 3p ˆ
Ë 7¯ Ë 7 ¯˚ sin 2 Á ˜ sin 2 Á ˜ sin 2 Á ˜
=0 Ë 8¯ Ë 4¯ Ë 8¯

È 4 4
Ê 2p ˆ Ê 4p ˆ Ê 8p ˆ ˘ = + +8-5
y 2 = Ísin 2 Á ˜ + sin 2 Á ˜ + sin 2 Á ˜ ˙ Êpˆ Êpˆ
Î Ë 7¯ Ë 7¯ Ë 7 ¯˚ sin 2 Á ˜ cos 2 Á ˜
Ë 8¯ Ë 8¯
fi 7
y2 = 4
4 = +3
Êpˆ Êpˆ
fi 7 sin 2 Á ˜ cos 2 Á ˜
y2 = Ë 8¯ Ë 8¯
4
16
= +3
Ê 2p ˆ Ê 4p ˆ Ê 8p ˆ 7 Êpˆ Êpˆ
sin Á ˜ + sin Á ˜ + sin Á ˜ 4 sin 2 Á ˜ cos 2 Á ˜
Ë 7¯ Ë 7¯ Ë 7¯ 2 Ë 8¯ Ë 8¯
16
= 2
+3

2Ê 2 Ê 2p ˆ 2 Ê 7p ˆ Ê Êpˆ Ê p ˆˆ
= tan ÁË ˜¯ + tan ÁË ˜¯ + .... + tan ÁË ˜¯ ÁË 2 sin ÁË 8 ˜¯ cos ÁË 8 ˜¯ ˜¯
16 16 16

Ê Êpˆ Ê 7p ˆ ˆ Ê Ê 2p ˆ Ê 6p ˆ ˆ 16
= Á tan 2 Á ˜ + tan 2 Á ˜ ˜ + Á tan 2 Á ˜ + tan 2 Á ˜ ˜ = +3
Ë Ë 16 ¯ Ë 16 ¯ ¯ Ë Ë 16 ¯ Ë 16 ¯ ¯ Êpˆ
sin 2 Á ˜
Ë 4¯
Ê Êpˆ Ê 7p ˆ ˆ Ê Ê 4p ˆ ˆ
+ Á tan 2 Á ˜ + tan 2 Á ˜ ˜ + Á tan 2 Á ˜ ˜
Ë Ë 16 ¯ Ë 16 ¯ ¯ Ë Ë 16 ¯ ¯ = 32 + 3 = 35 .

Ê Êpˆ Ê p ˆˆ Ê Ê 2p ˆ Ê 2p ˆ ˆ p
= Á tan 2 Á ˜ + cot 2 Á ˜ ˜ + Á tan 2 Á ˜ + cot 2 Á ˜ ˜ 8. Let q =
Ë Ë 16 ¯ Ë 16 ¯ ¯ Ë Ë 16 ¯ Ë 16 ¯ ¯ 7
4q = p - 3q
Ê Êpˆ Ê p ˆˆ Ê Ê p ˆˆ
+ Á tan 2 Á ˜ + cot 2 Á ˜ ˜ + Á tan 2 Á ˜ ˜ tan ( 4q ) = tan (p - 3q )
Ë Ë 16 ¯ Ë 16 ¯ ¯ Ë Ë 4¯¯
82 Comprehensive Trigonometry with Challenging Problems & Solutions for Jee Main and Advanced

tan ( 4q ) = - tan (3q ) cos(76∞) cos(16∞)


2 +1+
sin(76∞)sin(16∞)
4 tan q - 4 tan 3 q 3 tan q - tan 3 q =
=- cos(76∞) cos(16∞)
+
1 - 6 tan 2 q + tan 4 q 1 - 3 tan 2 q sin(76∞) sin(16∞)
4 x - 4 x3
=-
3x - x3 2 (sin(76∞)sin(16∞) ) + cos(76∞ - 16∞)
=
1 - 6 x2 + x4 1 - 3x 2 cos(76∞)sin(16∞) + sin(76∞) cos(16∞)
2 (sin(76∞)sin(16∞) ) + cos(76∞ - 16∞)
=
x 6 - 21x 4 + 35 x 2 - 7 = 0 sin(76∞ + 16∞)
y 3 - 21 y 2 + 35 y - 7 = 0 , y = x 2 = tan 2 q 2 (sin(76∞)sin(16∞) ) + cos(60∞)
=
.......(i) sin(92∞)

tan 2 q , tan 2 ( 2q ) , tan 2 (3q ) cos(60∞) - cos(92∞) + cos(60∞)


=
sin(92∞)
tan 2 q + tan 2 ( 2q ) + tan 2 (3q ) = 21 1 1
- cos(92∞) +
Êpˆ Ê 2p ˆ Ê 3p ˆ = 2 2
tan 2 Á ˜ + tan 2 Á ˜ + tan 2 Á ˜ = 21 sin(92∞)
Ë 7¯ Ë 7 ¯ Ë 7¯
1 - cos(92∞)
y y =
sin(92∞)
1 21 35 2 sin 2 (46∞)
- + -7=0 =
y 3
y 2
y 2 sin(46∞) cos(46∞)
-7 y 3 + 35 y 2 - 21y + 1 = 0 = tan(46∞)
= cot(44∞)
7 y 3 - 35 y 2 + 21 y - 1 = 0
10. Let a = cos x , b = cos y , c = cos z
cot 2 (q ) , cot 2 (2q ) , cot 2 (3q ) a+b+c =0

cot 2 (q ) + cot 2 ( 2q ) + cot 2 (3q ) =


35
=5 then a3 + b3 + c3 = 3 abc
7
cos (3x ) + cos (3 y ) + cos (3z ) .
2Êpˆ 2p ˆ Ê 3p ˆ

cot Á ˜ + cot Á ˜ + cot 2 Á ˜ = 5
Ë 7¯ Ë 7¯ Ë 7¯ ( )
= 4 cos3 x + cos3 y + cos3 z - 3 (cos x + cos y + cos z )

(
= 4 cos3 x + cos3 y + cos3 z )
Ê 2Êpˆ 2 Ê 2p ˆ 2 Ê 3p ˆ ˆ = 4.3 cos x cos y cos z
ÁË tan ÁË 7 ˜¯ + tan ÁË 7 ˜¯ + tan ÁË 7 ˜¯ ˜¯
x z
Ê Êpˆ Ê 2p ˆ Ê 3p ˆ ˆ p
¥ Á cot 2 Á ˜ + cot 2 Á ˜ + cot 2 Á ˜ ˜ 11. Let q =
Ë Ë 7¯ Ë 7¯ Ë 7 ¯¯ 9
= 35 ¥ 3 = 105 . p
fi 3q =
3 + cot(76∞) cot(16∞) 3
Êpˆ
cot(76∞) + cot(16∞) fi tan (3q ) = tan Á ˜ = 3
Ë 3¯
2 + 1 + cot(76∞) cot(16∞)
= 3 tan q - tan 3 q
cot(76∞) + cot(16∞) fi = 3
1 - 3 tan 2 q
The Ratios and Identities 83

p = tan B tan C
(3 tan q - tan q ) ( )
2 2
fi 3
= 3 1 - 3 tan 2 q
Ê 3p ˆ
= tan B tan Á - B˜
fi (
9 tan q - 6 tan q + tan q = 3 1 - 6 tan q + tan q
2 4 6 2 4
) Ë 4 ¯
Ê -1 - tan B ˆ
fi tan 6 q - 9 tan 4 q + 27 tan 2 q = 3 = tan B Á
Ë 1 - tan B ˜¯
Ê - tan B - tan 2 B ˆ
12. Let a = cos A + i sin A , b = cos B + i sin B , = Á ˜
Ë 1 - tan B ¯
c = cos C + i sin C
a+b+c=0 fi tan 2 B + (1 - p ) tan B + p = 0
1 1 1 D≥0
+ +
a b c
fi (1 - p )2 - 4 p ≥ 0
= (cos A - sin A) + (cos B - sin B )
fi p2 - 6 p + 1 ≥ 0
+ (cos C - sin C ) = 0
fi ( p - 3)2 - 8 ≥ 0
(a + b + c ) 2
=0
fi ( p - 3 + 2 2 )( p - 3 - 2 2 ) ≥ 0
fi a 2 + b 2 + c 2 + 2 ( ab + bc + ca ) = 0
fi p£ 3- 2 2 , p≥ 3+ 2 2
Ê 1 1 1ˆ
fi a + b + c + 2abc Á + + ˜ = 0
2 2 2
tan 3 A
Ë a b c¯ k=
tan A
fi a 2 + b2 + c2 = 0 tan 3 A tan 3 A - tan A
fi k -1 = -1 =
fi (cos A + i sin A)2 + (cos B + i sin B )2 tan A tan A
+ (cos C + i sin C ) = 0
2 sin ( 2 A)
fi k -1 =
cos 3 A sin A
fi (cos 2 A + i sin 2 A) + (cos 2 B + i sin 2 B ) 2 cos A
fi k -1 =
+ (cos 2C + i sin 2C ) = 0 cos 3 A
2k 2 tan 3 A cos 3 A
fi (cos 2 A + cos 2 B + cos 2C ) = .
k -1 tan A 2 cos A
+i (sin 2 A + sin 2 B + sin 2C ) = 0
2k sin 3 A
fi =
fi (cos 2 A + cos 2 B + cos 2C ) = 0 k - 1 sin A
(sin 2 A + sin 2 B + sin 2C ) = 0 sin 3 A 2k
fi =
fi (cos 2 A + cos 2 B + cos 2C ) = 0 sin A k - 1

fi 2 cos 2 A - 1 + 2 cos 2 B - 1 + 2 cos 2 C - 1 = 0 3 sin A - 4 sin 3 A 2k


=
3 sin A k -1
fi cos 2 A + cos 2 B + cos 2 C = 2k
2 fi 3 - 4 sin 2 A =
3 k -1
fi 1 - sin 2 A + 1 - sin 2 B + 1 - sin 2 C =
2 fi 2k k -3
4 sin 2 A = 3 - =

3 3
sin A + sin B + sin C = 3 - =
2 2 2 k -1 k -1
2 2 k -3
fi sin 2 A =
4 ( k - 1)
p k -3
A= 0£ £1
4 4 (k - 1)
p 3p
fi B+C =p - =
4 4
84 Comprehensive Trigonometry with Challenging Problems & Solutions for Jee Main and Advanced

1 1 - tan 2 a 1 - 2 tan 2 b
k£ and k ≥ 3 fi =
3 1 + tan 2 a 1 + 2 tan 2 b
cos ( A + B + C ) = cos (p ) = -1
2 2
fi =
fi cos A cos B cos C [1 - tan A tan B -2 tan a 2
-4 tan 2 b
- tan B tan C - tan C tan A] = -1 1 1
fi =
fi cos A cos B cos C + 1 = cos A sin B sin C - tan a 2
-2 tan 2 b
+ cos B sin C sin A + cos C sin A sin B fi tan 2 a = 2 tan 2 b
A B C
tan 2 a
cosecA cosecB cosec C + cot A cot B cot C fi =2
tan 2 b
= cot A + cot B + cot C
fi cot A + cot B + cot C - cosec A cosec B cosec C tan a 2
fi = 2=
= cot A cot B cot C tan b 1

tan (a + b ) Êp ˆ 1 + tan a cos a + sin a


tan Á + a ˜ = =
Ë4 ¯ 1 - tan a cos a - sin a
tan a + tan b
=
1 - tan a tan b Êp ˆ 1 + sin ( 2a )
tan 2 Á + a ˜ =
Ë4 ¯ 1 - sin ( 2a )
1 x
+

=
(
x x + x +1 2
) (x 2
)
+ x +1 Êa pˆ Êb pˆ
tan 3 Á + ˜ = tan Á + ˜
Ë 2 4¯ Ë 2 4¯
1
1-
x2 + x + 1 (1 + sin a )3 1 + sin b
fi =
(1 + x ) ( x (1 - sin a )3 1 - sin b
=
2
+ x +1 )
(x 2
+x ) x (x 2
+ x +1 )
(
2 sin b 2 3 sin a + sin a
=
3
)
= (x 2
+ x +1 ) l 2 (
2 1 + 3 sin 2 a )
x x
sin b =
(3 sin a + sin 3 a )
a +b =g (
1 + 3 sin 2 a )
3 cos 2b - 1
cos 2a =
3 - cos 2b
1
sin b = sin ( 2a + b )
Ê 1 - tan b ˆ 2 5
3Á ˜ -1 sin b
1 - tan a 2
Ë 1 + tan 2 b ¯ =
1
fi = fi
1 + tan a 2
Ê 1 - tan b ˆ 2 sin ( 2a + b ) 5
3-Á ˜
Ë 1 + tan 2 b ¯ sin b + sin (2a + b ) 1 + 5
fi =
sin b - sin (2a + b ) 1 - 5
1 - tan 2 a 3 - 3 tan 2 b - 1 - tan 2 b
fi =
1 + tan 2 a 3 + 3 tan 2 b - 1 + tan 2 b 2 sin (a + b ) cos (a ) 3
fi =-
2 cos (a + b ) sin ( -a ) 2
1 - tan 2 a 2 - 4 tan 2 b
fi =
1 + tan 2 a 2 + 4 tan 2 b 3
fi tan (a + b ) cot a =
2
The Ratios and Identities 85

fi tan (a + b ) =
3
tan a Ê x + yˆ Ê x - yˆ
tan Á , tan z , tan Á
2 Ë 2 ˜¯ Ë 2 ˜¯
sin x + sin y = 3 (cos x - cos y )
Ê x + yˆ Ê x - yˆ
fi tan 2 z = tan Á
Ë 2 ˜¯
tan Á
fi 3 cos x + sin x = 3 cos y - sin y Ë 2 ˜¯
3 = r cos a ,1 = r sin a Ê xˆ Ê yˆ
sin 2 Á ˜ - sin 2 Á ˜
1 Ë 2 ¯ Ë 2¯
fi r = 10 and tan a = fi tan 2 z =
3 Ê xˆ Ê yˆ
cos 2 Á ˜ - sin 2 Á ˜
Ë 2¯ Ë 2¯
3 cos x + sin x = 3 cos y - sin y
r cos ( x - a ) = r cos ( y + a ) cos y - cos x
fi fi tan 2 z =
cos y + cos x
fi cos ( x - a ) = cos ( y + a )
tan 2 z cos y - cos x
fi (x - a ) = ± ( y + a ) fi =
1 cos y + cos x
fi x = - y , x = y + 2a
1 cos y + cos x
fi x = -y fi =
tan z 2
cos y - cos x
fi 3x = -3 y
fi sin (3x ) = sin ( -3 y )
1 - tan 2 z 2 cos x
fi sin (3x ) = - sin (3 y ) =
1 + tan z 2
2 cos y
fi sin (3x ) + sin (3 y ) = 0 cos x
fi cos (2z ) =
cos y
sec (j - a ) ,sec j , sec (j + a ) fi cos x = cos y cos ( 2 z )
fi 2sec j = sec (j - a ) + sec (j + a )
sec2 q - tan q
23. Let y =
2 1 1 sec2 q + tan q
fi = +
cos j cos (j - a ) cos (j + a )
tan 2 q - tan q + 1
2 cos (j + a ) + cos (j - a ) fi y=
fi = tan 2 q + tan q + 1
cos j cos (j - a ) cos (j + a )
x2 - x + 1
1 cos j cos a fi y= , x = tanq
fi = x2 + x + 1
cos j cos (j ) - sin 2 (a )
2
x2 - x + 1
fi y=
fi cos (j ) - sin (a ) = cos j cos a
2 2 2
x2 + x + 1
fi cos 2 (j ) (1 - cos a ) = sin 2 (a ) fi (x 2
) (
+ x + 1 y = x2 - x + 1 )
sin 2 (a )
fi cos 2 (j ) = fi ( y - 1) x 2 + ( y + 1) x + ( y - 1) = 0
(1 - cos a )
q, D ≥ 0
4 sin 2 (a / 2) cos 2 (a / 2)
fi cos 2 (j ) = ( y + 1)2 - 4 ( y - 1)2 ≥ 0
2 sin 2 (a / 2) fi

fi cos 2 (j ) = 2 cos 2 (a / 2) fi ( y + 1)2 - (2 y - 2)2 ≥ 0


fi ( y + 1 + 2 y - 2) ( y + 1 - 2 y + 2) ≥ 0
fi cos (j ) = 2 cos (a / 2)
fi (3 y - 1) (- y + 3) ≥ 0
86 Comprehensive Trigonometry with Challenging Problems & Solutions for Jee Main and Advanced

fi (3 y - 1) ( y - 3) £ 0 sin ( 2a ) sin ( b + g )
=-
sin 2 ( b - g ) sin ( b - g )
fi 1£ y£3
3 sin ( 2a ) sin ( b + g )
=-
1 Ê sec q - tan q ˆ
2 2 sin ( b - g ) cos ( b - g ) sin ( b - g )
fi £ £3
3 ÁË sec 2 q + tan q ˜¯
sin (b - g ) ÈÎsin (2a ) + 2 cos (b - g ) sin (b + g )˘˚ = 0

sin ( b - g ) ÈÎsin (2a ) + sin (2b ) + sin ( 2g )˘˚ = 0


( ) ( ) (
cos q .cos (2q ) .cos 22 q cos 23 q ...cos 2n -1q ) sin ( b - g ) = 0 , ÈÎsin (2a ) + sin ( 2b ) + sin (2g )˘˚ = 0

=
sin 2 q( )
n

2 sin (q )
n
sin A
cot A +
sin B sin C
sin (p - q )
=
2n sin (q ) sin (p - ( B + C ))
= cot A +
sin B sin C
sin ( q )
=
2n sin (q ) sin ( B + C )
= cot A +
sin B sin C
1
= sin B cos C + cos B sin C
2n = cot A +
sin B sin C
sin B cos C cos B sin C
( ) ( ) (
2n cos q .cos ( 2q ) .cos 22 q cos 23 q ...cos 2n -1q ) = cot A + +
sin B sin C sin B sin C
= 1.
cos B sin C sin B cos C
tan(6∞) tan(42∞) tan(66∞) tan(78∞) = cot A + +
sin B sin C sin B sin C
= {tan(6∞) tan(66∞)}{tan(42∞) tan(78∞)}
= cot A + cot B + cot C
1
= {tan(54∞) tan(6∞) tan(66∞)}
tan(54∞) tan(18∞)
sin (q + A) sin (2 A)
¥ {tan(42∞) tan(18∞) tan(78∞)} =
sin (q + B ) sin (2 B )
tan(54∞) tan(18∞)
= sin q cos A + cos q sin A sin A cos A
tan(54∞) tan(18∞) fi =
sin q cos B + cos q sin B sin B cos B
= 1.
tan q cos A + sin A sin A cos A
fi =
tan q cos B + sin B sin B cos B
sin ( A ± B )
tan A ± tan B =
cos A cos B fi (
tan q cos A sin B cos B - cos B sin A cos A )
tan (a + b - g ) tan g
=
tan (a - b + g ) tan b
(
= sin B sin A cos A - sin A sin B cos B )
fi tan q cos A cos B ( cos A sin B - cos B sin A )
tan (a + b - g ) + tan (a - b + g ) tan g + tan b
= = sin A sin B ( cos A sin B - cos B sin A )
tan (a + b - g ) - tan (a - b + g ) tan g - tan b
sin A sin B
sin (a + b - g + a - b + g ) sin ( b + g ) fi tan q = = tan A tan B
=- cos A cos B
sin (a + b - g - a + b - g ) sin ( b - g )
The Ratios and Identities 87

fi tan q = tan A tan B (3 cos (2B ) - 1) (cos (2B ) + 3) = 0


fi tan 2 q = tan A tan B (3 cos (2B ) - 1) = 0 , (cos (2B ) + 3) = 0
(3 cos (2B ) - 1) = 0
cos ( x - y ) = -1
1
fi cos x cos y + sin x sin y = -1 cos (2 B ) =
3
fi 2 cos x cos y + 2 sin x sin y = -2 1 2 2
sin ( 2 B ) = ± 1 - =± ....(i)
fi (1 + 2 cos x cos y ) + (1 + 2 sin x sin y ) = 0 9 3

fi (cos 2
x + cos 2 y + 2 cos x cos y ) 1
sin ( A - B ) = - sin (2 B ) ....(ii)
(
+ sin x + sin y + 2 sin x sin y = 0
2 2
) 2 2

fi (cos x + cos y )2 + (sin x + sin y )2 = 0


1
sin ( A - B ) = ±
fi (cos x + cos y ) = 0 , (sin x + sin y ) = 0
2 2
3

fi (cos x + cos y ) = 0 , (sin x + sin y ) = 0


16 sin 2 (9∞)

30. If 2 cos A = cos B + cos3 B (


= 8 2 sin (9∞)
2
)
2 sin A = sin B - sin B = 8 (1 - cos(18∞))
3

1 Ê 10 + 2 5 ˆ
sin ( A - B ) = ± -
3 8
= ÁÁ 1 ˜
Ë 4 ˜¯
2 cos A = cos B + cos3 B

2 sin A = sin B - sin 3 B (


= 8 - 2 10 + 2 5 )
(
= Ê8 - 2 3 + 5 5 - 5 ˆ
Ë ¯ )( )
(cos B + cos B) + (sin B - sin B) = 2
3 2 3 2

ÊÊ ˆ
( ) ( ) ( )( )
2 2
ˆ Ê ˆ
(sin B + cos B) - 2 (sin B - cos B) + 1 = 2
6 6 4 4 = ÁË 3 + 5 ¯ + Ë 5 - 5 ¯ - 2 3 + 5 5 - 5 ˜
Ë ¯

(1 - 3sin B cos B) - 2 (sin B - cos B) + 1 = 2


2 2 2 2

(
= ÊÁ Ê 3 + 5 ˆ - Ê 5 - 5 ˆˆ˜ ) ( )
2

ËË ¯ Ë ¯¯
(3sin B cos B) + 2 (sin B - cos B) = 0
2 2 2 2

(3sin B cos B) = 2 cos (2B)


2 2
Ê ˆ
4 sin(9∞) = Á Ê 3 + 5 ˆ - Ê 5 - 5 ˆ˜
ËË ¯ Ë ¯¯( ) ( )
3
4
(sin 2B) = 2 cos (2B)
2 fi

4 Ë Ë ¯ Ë ¯
ˆ
sin(9∞) = Á Ê 3 + 5 ˆ - Ê 5 - 5 ˆ˜
¯ ( ) ( )
3 - 3 cos 2 2 B = 8 cos ( 2 B )
f (x
3 cos 2 2 B + 8 cos (2 B ) - 3 = 0 p
if x x=
3 cos 2 (2 B ) + 9 cos ( 2 B ) - cos ( 2 B ) - 3 = 0
6

3 cos (2 B ) (cos (2 B ) + 3) - (cos ( 2 B ) + 3) = 0 È Êpˆ ˘


= Í f Á ˜ , f ( 0 )˙
Î Ë 6 ¯ ˚
88 Comprehensive Trigonometry with Challenging Problems & Solutions for Jee Main and Advanced

È Êp ˆ˘ Êq +jˆ Êq -jˆ
= Í0,sin Á ˜ ˙
Ë 2 ˜¯ Ë 2 ˜¯ b
2 sin Á cos Á
Î Ë 6¯˚
fi =
Êq +jˆ Êq -jˆ a
Ë 2 ˜¯
cos Á
Ë 2 ˜¯
È 1˘ 2 cos Á
= Í0, ˙ .
Î 2˚
Êq +jˆ
Ë 2 ˜¯ b
2 sin Á
6 Ê Ê 2k p ˆ Ê 2k p ˆ ˆ fi =
 Á sin ÁË ˜¯ - i cos ÁË ˜
k =1 Ë 7 7 ¯ ˜¯ Êq +jˆ a
Ë 2 ˜¯
2 cos Á
6 Ê
Ê 2k p ˆ Ê 2k p ˆ ˆ
= -i  Á cos Á ˜¯ + i sin ÁË ˜
k =1 Ë Ë 7 7 ¯ ˜¯ Êq +jˆ b
fi tan Á =
Ë 2 ˜¯ a
Ê 2k p ˆ
6 iÁ
Ë 7 ˜¯
= -i  e
k =1
Êq -jˆ a 2 + b2
Ê i 2p 4p 12p ˆ cos Á =
i i Ë 2 ˜¯ 2
= -i e 7
Á +e 7 + .... + e 7 ˜
Ë ¯
Êq +jˆ a 2 + b2
cos Á =
i
2p Ê i
2p
i
4p
i
10p ˆ Ë 2 ˜¯ a 2 + b2 + 2
= -i e 7
Á 1 + e 7 + e 7 + .... + e 7
˜
Ë ¯ Êqˆ Êjˆ
tan Á ˜ + tan Á ˜
Ë 2¯ Ë 2¯
Ê 12p ˆ
2p i

= -i e
i
7 Á1- e 7 ˜ Êq +jˆ
Á ˜
Ë 2 ˜¯
2p sin Á
ÁË 1 - ei 7 ˜¯ =
Êqˆ Êjˆ
cos Á ˜ cos Á ˜
Ê i 2p i
12p 2p
+i ˆ Ë 2¯ Ë 2¯
Á e 7 -e 7 7
˜
= -i Á
i
2p ˜ Êq +jˆ
ÁË 1- e 7 ˜¯ Ë 2 ˜¯
2 sin Á
=
Êqˆ Êjˆ
Ê i 2p ˆ 2 cos Á ˜ cos Á ˜
Á e 7 - e i ( 2p ) ˜ Ë 2¯ Ë 2¯
= -i Á 2p ˜
ÁË 1 - ei 7 ˜¯ Êq +jˆ
Ë 2 ˜¯
2 sin Á
=
Ê i 2p ˆ Êq +jˆ Êq -jˆ
+ cos Á
˜ Ë 2 ˜¯
e 7 - 1˜ cos Á
Á
= -i Á Ë 2 ¯
2p ˜
ÁË 1 - ei 7 ˜¯
Êq +jˆ
Ë 2 ˜¯
2 tan Á
Ê 2p ˆ
i
- e ˜ =i Êq -jˆ
= i ÁÁ
1 7
Ë 2 ˜¯
cos Á
2p ˜
ÁË 1 - ei 7 ˜¯ = 1+
Êq +jˆ
Ë 2 ˜¯
cos Á
cos q + cos j = a
sin q + sin j = b a 2 + b2
= 1+ 2
sin q + sin j b a 2 + b2
=
cos q + cos j a
a 2 + b2 + 2
The Ratios and Identities 89

a 2 + b2 + 2 = k [cos q - cos q ]
= 1+
a 2 + b2 =0
Ê 2 ˆ ( x + y + z + 4) = 4.
= Á2 + ˜
Ë a 2 + b2 ¯
9
Ê p rˆ
 sin 2 ÁË ˜
r =0 18 ¯
tan q 1
=
tan q - tan 3q 3 Êpˆ Ê 2p ˆ Ê 3p ˆ Ê 9p ˆ
= sin 2 Á ˜ + sin 2 Á ˜ + sin 2 Á ˜ + .... + sin 2 Á ˜
Ë 18 ¯ Ë 18 ¯ Ë 18 ¯ Ë 18 ¯
1 1
fi =
tan 3q 3 = sin 2 (10∞) + sin 2 (20∞) + sin 2 (30∞) + .... + sin 2 (90∞)
1-
tan q
1 1 = 4 ¥1+1 = 5
fi =
3 - tan q 2
3 tan x 1
1- =
1 - 3 tan 2 q2 tan y 3
1 - 3 tan q 1
fi = tan x tan y
1 - 3 tan q - 3 + tan q
2 2
3 =
1 3
1 - 3 tan 2 q 1
fi =- 9
2 + 2 tan q 2
3 1-
1 4 =1 5
tan x =
fi 2 + 2 tan 2 q = -3 + 9 tan 2 q 3 1 3 3
-1
4
fi 7 tan 2 q = 5 tan x + tan y
tan ( x + y ) =
5 1 - tan x tan y
fi tan 2 q =
7 tan x + 3 tan x
=
cot q 1 - tan x. 3 tan x
cot q - cot (3q )
4 tan x
=
1 1 - 3 tan 2 x
=
Ê cot (3q ) ˆ
1- Á
Ë cot q ˜¯
4 5
= 3 3
1 2 3 5
= = 1- ¥
1 3 9 3
1+
2 4 5 9
= ¥
3 3 4
INTEGER TYPE QUESTIONS
x y z 3 5
= = =k = = 15
cos q Ê 2p ˆ Ê 2p ˆ 3
cos Á q - ˜ cos Á q + ˜
Ë 3¯ Ë 3¯ tan 2 ( x + y )
x+ y+z = 3.
5
È Ê 2p ˆ Ê 2p ˆ ˘ cos ( x - y ) , cos x , cos ( x + y )
= k Ícos q + cos Á q - ˜ + cos Á q + ˜ ˙
Î Ë ¯ Ë 3 ¯˚
2 cos ( x - y ) cos ( x + y )
3
fi cos x =
È Ê 2p ˆ ˘ cos ( x - y ) + cos ( x + y )
= k Ícos q + 2 cos q cos Á ˜ ˙
Î Ë 3 ¯˚
cos 2 x + cos 2 y
fi cos x =
È 1˘ 2 cos x cos y
= k Ícos q - 2 cos q . ˙
Î 2˚
90 Comprehensive Trigonometry with Challenging Problems & Solutions for Jee Main and Advanced

fi 2 cos 2 x cos y = 2 cos 2 x + 2 cos 2 y - 2 Êp ˆ Êp ˆ


= sin 2 q + sin 2 Á - q ˜ + sin 2 Á + q ˜
Ë3 ¯ Ë 3 ¯
fi cos 2 x cos y = cos 2 x + cos 2 y - 1
1È Ê 2p ˆ Ê 2p ˆ˘
fi cos 2 x (cos y - 1) = cos 2 y - 1 = Í1 - cos ( 2q ) + 1 - cos Á - 2q ˜ + 1 - cos Á + 2q ˜ ˙
2Î Ë 3 ¯ Ë 3 ¯˚
fi cos 2 x = cos y + 1 1È Ê 2p ˆ Ê 2p ˆ˘
= Í3 - cos (2q ) - cos Á - 2q ˜ - cos Á + 2q ˜ ˙
Ê yˆ 2Î Ë 3 ¯ Ë 3 ¯˚
fi cos 2 x = 2 cos 2 Á ˜
Ë 2¯
1È Ê 2p ˆ ˘
= Í3 - cos (2q ) - 2 cos Á ˜ cos ( 2q )˙

2Ê 2Î Ë 3 ¯ ˚
fi cos x sec Á ˜ = 2
2
Ë 2¯
1È 1 ˘
= 3 - cos (2q ) - 2 ¥ - ¥ cos ( 2q )˙

Ê yˆ
cos x sec Á ˜ = 2 2 ÍÎ 2 ˚
Ë 2¯
3
=
m= 2. 2

(m 2
)
+ 2 = 4. Êpˆ 3
2 f Á ˜ = 2¥ = 3.
Ë 15 ¯ 2
Êp ˆ Ê 2p ˆ
tan x + tan Á + x˜ + tan Á + x˜ m = 3 cosec(20∞) - sec(20∞) = 4
Ë3 ¯ Ë 3 ¯
1
Êp ˆ Ê p ˆ n = sin(12∞)sin(48∞)sin(54∞) =
= tan x + tan Á + x˜ + tan Á p - + x˜ 8
Ë3 ¯ Ë 3 ¯ ( m + 8n + 2 )
Êp ˆ Êp ˆ =4+1+2
= tan x + tan Á + x˜ - tan Á - x˜
Ë3 ¯ Ë3 ¯

3 + tan x 3 - tan x 8. Here, tan(15∞) + tan(30∞) = - p


= tan x + -
1 - 3 tan x 1 + 3 tan x tan(15∞) + tan(30∞) = q
8 tan x tan(45∞) = 1
= tan x +
1 - 3 tan 2 x fi tan(30∞ + 15∞) = 1
tan x - 3 tan x + 8 tan x
3
tan(30∞) + tan(15∞)
= fi =1
1 - 3 tan x2
1 - tan(30∞) tan(15∞)
9 tan x - 3 tan 3 x -p
= fi =1
1 - 3 tan 2 x 1- q
fi - p =1- q
=
(
3 3 tan x - tan 3 x ) fi q - p =1
1 - 3 tan 2 x
fi 2 + q - p = 2 +1 = 3
= 3 tan (3x )
(2 + q - p )
k = 3. 44
2p ˆ 2Ê Ê 4p ˆ Â cos(n∞)
f (q ) = sin q + sin Á
2
+ q ˜ + sin 2 Á + q˜ n =1
Ë 3 ¯ Ë 3 ¯ x= 44

2p p
 sin(n∞)
2Ê ˆ Ê ˆ n =1
= sin q + sin Á
2
+ q ˜ + sin 2 Á p + + q ˜
Ë 3 ¯ Ë 3 ¯
Ê 43∞ ˆ
cos Á1∞ + ˜
Ê 2p ˆ Êp ˆ Ë 2 ¯
= sin 2 q + sin 2 Á + q ˜ + sin 2 Á + q ˜ =
Ë 3 ¯ Ë3 ¯ Ê 43∞ ˆ
sin Á1∞ + ˜
Ë 2 ¯
The Ratios and Identities 91

Ê 45∞ ˆ =
1
¥ tan(15∞) ¥ cot(15∞)
Ë 2 ˜¯
cos Á
tan(5∞)
=
Ê 45∞ ˆ 1
Ë 2 ˜¯
sin Á =
tan(5∞)
Ê 45∞ ˆ = cot(5∞)
2 cos 2 Á
Ë 2 ˜¯
= x=5
sin(45∞)
tan(27∞) + tan(18∞) + tan(27∞) tan(18∞)
(1 + cos 45∞)
=
sin(45∞)
= ( )
2 +1 = tan (27∞ + 18∞)
= tan (45∞)
[ x + 3] =1
= ÎÈ 2 + 1 + 3˘˚ m=1

= ÎÈ 2 ˘˚ + 4 = 1 + 4 = 5
(m + x + 1)
sin(25∞)sin(35∞)sin(85∞) QUESTIONS ASKED IN IIT-JEE EXAMS
=
1
[4 sin(35∞)sin(25∞)sin(85∞)] sin x sin y sin ( x - y )
4
1
1
= ¥ sin(75∞) = (2sin x sin y ) sin ( x - y )
4 2

= (cos ( x - y ) - cos ( x + y )) sin ( x - y )


1 1
= ¥ cos(15∞) 2
4
= ( 2 cos ( x - y ) sin ( x - y ) - 2 sin ( x - y ) cos ( x + y ))
1
1 3 +1 6+ 2
= ¥ = 4
4 2 2 16
= (sin ( 2 x - 2 y ) - sin 2 x + sin 2 y )
1
Ê c ˆ 4
ÁË + 2˜ = 4 .
a+b ¯ sin y sin z sin ( y - z )
17
Ê kpˆ
m = Â cos Á
k =1 Ë 9 ˜¯ =
1
4
(sin (2 y - 2 z ) - sin 2 y + sin 2 z )
Êpˆ Ê 2p ˆ Ê 3p ˆ Ê 17p ˆ
= cos Á ˜ + cos Á ˜ + cos Á ˜ + ... + cos Á sin z sin x sin ( z - x )
Ë 9¯ Ë 9¯ Ë 9¯ Ë 9 ˜¯
Ê 17p ˆ
=
1
4
(sin (2 z - 2 x) - sin 2 z + sin 2 x)
Ë 18 ˜¯
sin Á
Ê p 16p ˆ
= ¥ cos Á +
Êpˆ Ë 9 18 ˜¯ 1
sin Á ˜
Ë 18 ¯ (sin 2 A + sin 2 B + sin 2C ) + sin A.sin B.sin C
4
Ê 18p ˆ Where A = x - y, B = y - z , C = z - x
= cos Á = -1
Ë 18 ˜¯
1
= (sin 2 A + sin 2 B + sin 2C ) + sin A.sin B.sin C
(m 2
)
+ m + 2 = 4. 4

= ( 2 sin ( A + B ) cos ( A - B ) + sin 2C )


1
tan(55∞) tan(65∞) tan(75∞)
4
1 + sin A.sin B.sin C
= ¥ [ tan(55∞) tan(5∞) tan(65∞) ] ¥ tan(75∞)
tan(5∞)
=
1
4
(2 sin (-C ) cos ( A - B ) + sin 2C )
1
= ¥ tan(15∞) ¥ tan(75∞) + sin A.sin B.sin C
tan(5∞)
=
1
4
(-2 sin (C ) cos ( A - B ) + 2 sin C cos C )
92 Comprehensive Trigonometry with Challenging Problems & Solutions for Jee Main and Advanced

+ sin A.sin B.sin C


1 - cos B
tan A =
= ( -2 sin (C ) cos ( A - B ) + 2 sin C cos C )
1
sin B
4
+ sin A.sin B.sin C 2 sin 2 ( B / 2) Ê Bˆ
tan A = = tan Á ˜
= - ¥ 2sin C ( cos ( A - B ) - cos ( A + B ))
1 2 sin ( B / 2) cos ( B / 2) Ë 2¯
4
+ sin A.sin B.sin C tan 2A

1 2 tan A
= - ¥ 2 sin C ¥ 2 sin A sin B + sin A.sin B.sin C =
4 1 - tan 2 A
= - sin A sin B sin C + sin A.sin B.sin C 2 tan ( B / 2)
=
=0 1 - tan 2 ( B / 2)
= tan B
tan 2a

= tan ((a + b ) + (a - b )) n=6


tan (a + b ) + tan (a - b ) sin 3 x sin 3x
=
1 - tan (a + b ) tan (a - b ) ( )
= sin x (sin 3x sin x )
2

Ê3 5ˆ
=
ÁË + ˜¯
4 12 =
1
4
( )
2 sin 2 x (2 sin 3x sin x )
Ê3 5ˆ
1- Á . ˜ 1
Ë 4 12 ¯ = (1 - cos 2 x ) (cos 2 x - cos 4 x )
4
36 + 20
=
48 - 15 =
1
4
(
cos 2 x - cos 2 2 x - cos 4 x + cos 2 x.cos 4 x )
56 1
= = È2 cos 2 x - 2 cos 2 2 x - 2 cos 4 x
33 8Î
+2 cos 4 x.cos 2 x ]
A = sin 2 q + cos 4 q
1
= È2 cos 2 x - (1 + cos 4 x ) - 2 cos 4 x
1
( ) (
1
) 8Î
2
= 2 sin 2 q + 2 cos 2 q
2 4
+ cos 6 x + cos 2 x ]
1 1
= (1 - cos 2q ) + (1 + cos 2q )
2
1
2 4 = (-1 + 3 cos 2 x - 3 cos 4 x + cos 6 x )
8
=
1
2
1
(
(1 - cos 2q ) + 1 + 2 cos 2q + cos2 2q
4
) n = 6.

1 1 1 sin 12∞ sin 54∞ sin 48∞


= + + cos 2 2q
2 4 4 = (sin 12∞ sin 48∞) sin 54∞
3 1
= + cos 2 2q =
1
(4 sin 48∞ sin 12∞ sin 72∞) sin 54∞
4 4 4 sin 72∞
3 1
+ .1 = 1 =
1
(sin 36∞ cos 36∞)
4 4 4 sin 72∞
3 1 3
+ .0 = 1
(2 sin 36∞ cos 36∞)
4 4 4 =
4 ¥ 2 sin 72∞
3
£ A £1
4
The Ratios and Identities 93

1
= (sin 72∞) Ê Ê p ˆˆ Ê Ê 3p ˆ ˆ Ê Ê 5p ˆ ˆ
4 ¥ 2 sin 72∞
ÁË1 + cos ÁË 8 ˜¯ ˜¯ ÁË1 + cos ÁË 8 ˜¯ ˜¯ ÁË1 + cos ÁË 8 ˜¯ ˜¯
1
= Ê Ê 7p ˆ ˆ
ÁË1 + cos ÁË 8 ˜¯ ˜¯
8
sin 2 a + sin 2 b - sin 2 g
Ê p ˆÊ 3p ˆ Ê 3p ˆ Ê p ˆ
= Á1 + cos ÊÁ ˆ˜ ˜ Á1 + cos ÊÁ ˆ˜ ˜ Á1 - cos ÊÁ ˆ˜ ˜ Á1 - cos ÊÁ ˆ˜ ˜
= sin a + sin (b + g ) sin (b - g )
2
Ë Ë 8 ¯¯ Ë Ë 8 ¯¯ Ë Ë 8 ¯¯ Ë Ë 8 ¯¯

= sin a + sin (p - a ) sin (b - g )


2
Ê Ê p ˆˆ Ê Ê 3p ˆ ˆ
= Á1 - cos 2 Á ˜ ˜ Á1 - cos 2 Á ˜ ˜
Ë Ë 8 ¯¯ Ë Ë 8 ¯¯
= sin a + sin a sin ( b - g )
2

Êpˆ Ê 3p ˆ
= sin a (sin a + sin ( b - g )) = sin 2 Á ˜ sin 2 Á ˜
Ë 8¯ Ë 8¯
(
= sin a sin (p - ( b + g )) + sin ( b - g ) ) 1Ê Ê p ˆˆ Ê Ê 3p ˆ ˆ
= Á 2 sin 2 Á ˜ ˜ Á 2 sin 2 Á ˜ ˜
= sin a (sin ( b + g ) + sin ( b - g )) 4Ë Ë 8 ¯¯ Ë Ë 8 ¯¯

= sin a ¥ 2 sin b sin g 1Ê Ê p ˆˆ Ê Ê 3p ˆ ˆ


= Á 1 - cos Á ˜ ˜ Á1 - cos Á ˜ ˜
4Ë Ë 4¯¯ Ë Ë 4 ¯¯
= 2sin a sin b sin g
1Ê 1 ˆÊ 1 ˆ
= Á 1- ˜ Á 1+ ˜
4Ë 2¯ Ë 2¯
Ê 2p ˆ Ê 4p ˆ Ê 8p ˆ Ê 16p ˆ 1 Ê 1ˆ
16 cos Á ˜ cos Á ˜ cos Á ˜ cos Á = Á1 - ˜
Ë 15 ¯ Ë 15 ¯ Ë 15 ¯ Ë 15 ˜¯ 4 Ë 2¯
Êpˆ Ê 2p ˆ Ê 4p ˆ Ê 8p ˆ 1 1
= -16 cos Á ˜ cos Á ˜ cos Á ˜ cos Á ˜ = ¥
Ë 15 ¯ Ë 15 ¯ Ë 15 ¯ Ë 15 ¯ 4 2
-16 Ê 2p ˆ Ê 2p ˆ Ê 4p ˆ Ê 8p ˆ 1
= sin Á ˜ cos Á ˜ cos Á ˜ cos Á ˜ =
Êpˆ Ë 15 ¯ Ë 15 ¯ Ë 15 ¯ Ë 15 ¯ 8
2 sin Á ˜
Ë 15 ¯

-8 Ê 4p ˆ Ê 4p ˆ Ê 8p ˆ
= sin Á ˜ cos Á ˜ cos Á ˜
Êpˆ Ë 15 ¯ Ë 15 ¯ Ë 15 ¯
2 sin Á ˜
Ë 15 ¯ ( ) (
3 cos 4 a + sin 4 a - 2 cos6 a + sin 6 a )
-4 Ê 8p ˆ Ê 8p ˆ
= sin Á ˜ cos Á ˜ = 3 (1 - 2 sin 2
a cos a ) - 2 (1 - 3 sin
2 2
a cos 2 a )
p
Ê ˆ Ë 15 ¯ Ë 15 ¯
2 sin Á ˜
Ë 15 ¯ = 3- 2

-1 Ê 16p ˆ =1
= sin Á
Êpˆ Ë 15 ˜¯
sin Á ˜
Ë 15 ¯
3 cosec(20∞) - sec(20∞)
-1 Ê pˆ
= sin Á p + ˜
Êpˆ Ë 15 ¯ 3 1
sin Á ˜
Ë 15 ¯ = -
sin(20∞) cos(20∞)
-1 Êpˆ 3 cos(20∞) - sin(20∞)
= ¥ - sin Á ˜ = 1 =
Êpˆ Ë 15 ¯
sin Á ˜ sin(20∞) cos(20∞)
Ë 15 ¯
94 Comprehensive Trigonometry with Challenging Problems & Solutions for Jee Main and Advanced

Ê 3 1 ˆ Ê Êpˆ Ê 3p ˆ Ê 5p ˆ ˆ
2
4Á cos(20∞) - sin(20∞)˜ = Á sin Á ˜ sin Á ˜ sin Á ˜ ˜
Ë 2 2 ¯ Ë Ë 14 ¯ Ë 14 ¯ Ë 14 ¯ ¯
=
2 sin(20∞) cos(20∞) 2
Ê Êp p ˆ Ê p 3p ˆ Ê p 5p ˆ ˆ
4 (sin(60∞) cos( 20∞) - cos(60∞)sin(20∞) ) = Á cos Á - ˜ cos Á - ˜ cos Á - ˜ ˜
Ë Ë 2 14 ¯ Ë 2 14 ¯ Ë 2 14 ¯ ¯
=
sin(20∞)
2
Ê Ê 6p ˆ Ê 4p ˆ Ê 2p ˆ ˆ
4 (sin(60∞ - 20∞) ) = Á cos Á
Ë 14 ˜¯
cos Á ˜ cos Á ˜ ˜
Ë 14 ¯ Ë 14 ¯ ¯
= Ë
sin(40∞)
2
Ê Ê 3p ˆ Ê 2p ˆ Ê p ˆˆ
4 (sin(40∞) ) = Á cos Á ˜ cos Á ˜ cos Á ˜ ˜
= Ë Ë 7 ¯ Ë 7¯ Ë 7 ¯¯
sin(40∞)
2
=4 Ê Êpˆ Ê 2p ˆ Ê 4p ˆ ˆ
= Á cos Á ˜ cos Á ˜ cos Á ˜ ˜
Ë Ë 7¯ Ë 7¯ Ë 7 ¯¯
14. tan a + 2 tan 2a + 4 tan 4a + 8 cot 8a
2
= cot a - (cot a - tan a ) + 2 tan 2a + 4 tan 4a + 8 cot 8a Ê 1ˆ
= Á ˜
Ë 8¯
= cot a - ( 2 cot 2a - 2 tan 2a ) + 4 tan 4a + 8 cot 8a
1
= cot a - 2 ( cot 2a - tan 2a ) + 4 tan 4a + 8 cot 8a =
64
= cot a - 2.2 cot 4a + 4 tan 4a + 8 cot 8a
= cot a - 4 (cot 4a - tan 4a ) + 8 cot 8a f ( x ) = cos ÈÎp 2 ˘˚ x + cos ÈÎ-p 2 ˘˚ x
= cot a - 4.2 cot 8a + 8 cot 8a = cos 9 x + cos10 x
= cot a - 8 cot 8a + 8 cot 8a
Êpˆ Ê 9p ˆ
f Á ˜ = cos Á ˜ + cos (5p )
= cot a Ë 2¯ Ë 2¯
= 0 – 1 = –1
1
f (p ) = cos (9p ) + cos (10p ) = –1 + 1 = 0
64
Êpˆ Ê 3p ˆ Ê 5p ˆ Ê 7p ˆ f ( -p ) = cos (-9p ) + cos ( -10p )
sin Á ˜ sin Á ˜ sin Á ˜ sin Á ˜
Ë 14 ¯ Ë 14 ¯ Ë 14 ¯ Ë 14 ¯
= cos (9p ) + cos (10p )
Ê 9p ˆ Ê 11p ˆ Ê 13p ˆ =–1+1=0
sin Á ˜ sin Á
Ë 14 ˜¯
sin Á
Ë 14 ¯ Ë 14 ˜¯
Êpˆ Ê 9p ˆ Ê 10p ˆ
f Á ˜ = cos Á ˜ + cos Á
Êpˆ Ê 3p ˆ Ê 5p ˆ Êpˆ
= sin Á ˜ sin Á ˜ sin Á ˜ sin Á ˜ Ë 4¯ Ë 4¯ Ë 4 ˜¯
Ë 14 ¯ Ë 14 ¯ Ë 14 ¯ Ë 2¯
1 1
Ê 9p ˆ Ê 11p ˆ Ê 13p ˆ = +0=
sin Á ˜ sin Á 2 2
Ë 14 ˜¯
sin Á
Ë 14 ¯ Ë 14 ˜¯
Æ C; (ii) Æ A.
Êpˆ Ê 3p ˆ Ê 5p ˆ Êpˆ
= sin Á ˜ sin Á ˜ sin Á ˜ sin Á ˜ Êpˆ Ê 5p ˆ Ê 7p ˆ
Ë 14 ¯ Ë 14 ¯ Ë 14 ¯ Ë 2¯ k = sin Á ˜ sin Á ˜ sin Á ˜
Ë 18 ¯ Ë 18 ¯ Ë 18 ¯
Ê 5p ˆ Ê 3p ˆ Ê pˆ
sin Á p - ˜ sin Á p - ˜ sin Á p - ˜ = sin(10∞)sin(50∞)sin(70∞)
Ë 14 ¯ Ë 14 ¯ Ë 14 ¯
= sin(50∞)sin(10∞)sin(70∞)
Êpˆ Ê 3p ˆ Ê 5p ˆ
= sin Á ˜ sin Á ˜ sin Á ˜ 1
Ë 14 ¯ Ë 14 ¯ Ë 14 ¯ = ¥ ( 4 sin(60∞ - 10∞)sin(10∞)sin(60∞ + 10∞) )
4
Êpˆ Ê 3p ˆ Ê 5p ˆ
sin Á ˜ sin Á ˜ sin Á ˜
Ë 14 ¯ Ë 14 ¯ Ë 14 ¯
The Ratios and Identities 95

1 p
= ¥ ( sin(3 ¥ 10∞) ) fi A=
4 6
1 1 p p p p p
= ¥ When A = , B= - A= - =
4 2 6 3 3 6 6
1 A B
=
8 1 1
= .
20. Let y = tan A tan B 3 3
1
Êp ˆ =
= tan A tan Á - A˜ 3
Ë3 ¯
Ê 3 - tan A ˆ
= tan A Á ˜
(sec 2 x - tan 2 x )
Ë 1 + 3 tan A ¯
Ê 1 - sin 2 x ˆ
= ÁË ˜
Ê 3 tan A - tan 2 Aˆ cos 2 x ¯
= Á ˜
Ë 1 + 3 tan A ¯ Ê (cos x - sin x )2 ˆ
= Á 2 ˜
Ë cos x - sin x ¯
2
Ê 3 t - t2 ˆ
= Á ˜ Ê cos x - sin x ˆ
Ë 1+ 3t ¯ = Á
Ë cos x + sin x ˜¯


dy 1 + 3t
=
( )( ) (
3 - 2t - 3t - t 2 ) 3 Ê 1 - tan x ˆ
= Á
Ë 1 + tan x ˜¯
dt
(1 + 3t )
2

Êp ˆ
= tan Á - x˜
3 + 3t - 2t - 2 3t 2 - 3t + 3t 2 Ë4 ¯
=
(1 + 3t )
2

3 (sin x - cos x ) + 6 (sin x + cos x )


4 2

3 - 2t - 3t 2
= (
+ 4 sin x + cos x
6 6
)
(1 + 3t )
2

= 3 ÈÎsin x - 4 sin x cos x + 6 sin x cos x


4 3 2 2

3t + 2t - 3
2
= - -4 sin x cos3 x + cos 4 x ˘˚ +6 (1 + 2sin x cos x )
(1 + 3t )
2

= -
3t 2 + 3t - t - 3 (
+4 1 - 3 sin 2 x cos 2 x )
(1 + 3t ) 3 È(sin ) ( )
2
4
x + cos 4 x - 4 sin x cos x sin 2 x + cos 2 x
Î
3t (t + 3 ) - (t + 3 ) +6 sin 2 x cos 2 x ˚˘ +6 (1 + 2sin x cos x )
-
=
(1 + 3t ) ( )
2
+4 1 - 3 sin 2 x cos 2 x

-
( 3t - 1) (t + 3 ) 3 ÈÎ1 - 2 sin 2 x cos 2 x + 4 sin x cos x
=
(1 + 3t )
2
+6 sin 2 x cos 2 x ˚˘ + 6 (1 + 2 sin x cos x )
dy
=0 t=
1
,- 3 (
+4 1 - 3 sin 2 x cos 2 x )
dt 3
=3+6+4
1
t= = 13.
3
1
fi tan A =
3
96 Comprehensive Trigonometry with Challenging Problems & Solutions for Jee Main and Advanced

(sin a + sin b + sin g ) < sin (a + b + g ) =


1
( sin 30∞)
2
= sin (p ) = 0 1
=
sin a + sin b + sin g < 0 4
a, b, c, d)
Ê 4 xy ˆ
sec2 q = Á 2˜ Êqˆ
Ë (x + y) ¯ f n (q ) = tan Á ˜ (1 + sec q ) (1 + sec 2q ) ......
Ë 2¯
Ê 4 xy ˆ
fi 1 + tan 2 q = Á 2˜ (1 + sec 4q )........(1 + sec 2n q )
Ë (x + y) ¯
Ê 4 xy ˆ Êqˆ
sin Á ˜
fi tan q = Á
2
- ˜ Ë 2 ¯ Ê 1 + cos q ˆ
˜ (1 + sec 2q )
1 = ¥Á
Ë (x + y)
2
¯ Ê q ˆ Ë cos q ¯
cos Á ˜
Ê 4 xy - ( x + y )2 ˆ 2 Ë 2¯
Ê x - yˆ
fi tan 2 q = Á ˜ = -
Ë (x + y)
2
¯
ÁË x + y ˜¯
(1 + sec 4q )........(1 + sec 2n q )
2
Ê x - yˆ Êqˆ Êqˆ
fi -Á = tan 2 q ≥ 0 2 sin Á ˜ cos Á ˜
Ë x + y ˜¯ Ë 2¯ Ë 2¯
(1 + sec 2q ) (1 + sec 4q )
=
cos q
- (x - y) ≥ 0
2

(1 + sec 8q )......(1 + sec 2n q )
fi (x - y) £ 0
2

sin q Ê 1 + cos 2q ˆ
fi ( x - y )2 = 0 = ¥Á ˜ (1 + sec 4q )
cos q Ë cos 2q ¯
fi x–y=0
(1 + sec 8q )......(1 + sec 2n q )
fi x=y
x= yπ0 2 sin q cos q Ê 1 + cos 4q ˆ
= ¥Á
cos 2q Ë cos 4q ˜¯
y = cos x cos ( x + 2) - cos 2 ( x + 1) (1 + sec 8q )......(1 + sec 2n q )
=
1
2
(
2 cos x cos ( x + 2) - 2 cos 2 ( x + 1) ) = sin 2q ¥ 2 cos 2q (1 + sec 8q ) ... 1 + sec 2n q
2
( )
cos 2q cos 4q
=
1
(
2 cos ( 2 x + 2) + cos (1) - (1 + cos ( 2 x + 2)) ) =
sin 4q Ê 1 + cos 8q ˆ
( n
)
2 ÁË ˜¯ ...... 1 + sec 2 q
cos 4q cos 8q
= - (1 - cos (1))
1
sin 4q Ê 2 cos 2 4q ˆ
2
1
=
cos 4q ÁË cos 8q ˜¯
(
...... 1 + sec 2n q )
= - ¥ 2 sin 2 (1)
2
= sin 8q ........... 1 + sec 2n q
( )
= - sin (1)
2
cos 8q

Êp 2 ˆ
ÁË , - sin 1˜¯ =
( )
sin 2n q
2 cos ( 2 q )
n

x
= tan (2 q )
n

sin 15∞ cos15∞


Êpˆ Ê pˆ Êpˆ
f 2 Á ˜ = tan Á 22 ¥ ˜ = tan Á ˜ = 1
1
= ( 2 sin 15∞ cos15∞) Ë 16 ¯ Ë 16 ¯ Ë 4¯
2
The Ratios and Identities 97

Êpˆ Ê pˆ Êpˆ Ê 2 tan 2 a ˆ


f3 Á ˜ = tan Á 23 ¥ ˜ = tan Á ˜ = 1
Ë 32 ¯ Ë 32 ¯ Ë 4¯ Á x +x+ ˜
Ë x2 + x ¯
fi ≥ tan a
Êpˆ Ê pˆ Êpˆ 2
f 4 Á ˜ = tan Á 24 ¥ ˜ = tan Á ˜ = 1
Ë 64 ¯ Ë 64 ¯ Ë 4¯ Ê 2 tan 2 a ˆ
fi Á x + x + ˜ ≥ 2 tan a
Ê p ˆ Ê p ˆ Êpˆ Ë x2 + x ¯
f5 Á = tan Á 25 ¥ ˜ = tan ÁË ˜¯ = 1
Ë 128 ˜¯ Ë 128 ¯ 4

1 p
sinq = q=
f (q ) = sin q (sin q + sin 3q ) 2 6

= sin q (sin 3q + sin q )


1 1
cosj = <
3 2
= sin q ¥ 2 sin 2q cos q
p p
= 2 sin q cos q ¥ sin 2q fi <j <
2 3
= sin 2q ¥ sin 2q p p p p
+ <q +j < +
= sin 2 2q 6 3 6 2
≥ q p 2p
fi <q +j <
2 3
cot a1.cot a 2 ......cot a n = 1 1 - 2 x + 5x2
34. Let y =
fi cos a1.cos a 2 ....cos a n = sin a1.sin a 2 .....sin a n 3x 2 - 2 x - 1

fi (cos a1.cos a 2 ....cos a n )2 fi 3x 2 y - 2 xy - y = 1 - 2 x + 5 x 2

= (cos a1 sin a1 )(cos a 2 sin a 2 ) ... (cos a n sin a n ) fi (3 y - 5) x 2 + 2 (1 - y ) x - (1 + y ) = 0


1 x ( y - 1)2 + (3 y - 5) ( y + 1) ≥ 0
= (2 cos a1 sin a1 )(2 cos a 2 sin a 2 )...(2 cos a n sin a n )
( )
n
2 y2 - 2 y + 1 + 3y2 - 5 y + 3y - 5 ≥ 0

1
£ fi 4 y2 - 4 y - 4 ≥ 0
2n
1 fi y2 - y - 1 ≥ 0
fi (cos a1.cos a 2 ....cos a n ) £
2n / 2
(2 y - 1)2 ≥ ( )
2
fi 5

fi (2 y - 1) ≥ 5 or ( 2 y - 1) £ - 5
a = b +g
Ê 5 + 1ˆ Ê1- 5ˆ
fi tan a = tan (b + g ) fi y≥Á ˜ or y £ Á ˜
Ë 2 ¯ Ë 2 ¯
tan b + tan g
fi tan a = Ê 5 + 1ˆ Ê 5 - 1ˆ
1 - tan b tan g fi y≥Á or y £ - Á
˜ ˜
Ë 2 ¯ Ë 2 ¯
fi tan a - tan a .tan b .tan g = tan b + tan g
fi tan a - tan g = tan b + tan g Ê 5 + 1ˆ Ê 5 - 1ˆ
fi 2 sin t ≥ Á ˜ or 2 sin t £ - Á ˜
fi tan a = tan b + 2 tan g Ë 2 ¯ Ë 2 ¯
Ê 5 + 1ˆ Ê 5 - 1ˆ
fi sin t ≥ Á ˜ or sin t £ - Á ˜
A.M ≥ G.M Ë 4 ¯ Ë 4 ¯
Ê 2 tan 2 a ˆ fi sin t ≥ sin(54∞) or sin t £ sin(-18∞)
Á x +x+ ˜
Ë x2 + x ¯ tan a2
Ê 3p ˆ Ê pˆ
fi ≥ x2 + x. fi sin t ≥ sin Á ˜ or sin t £ sin Á - ˜
2 x +x
2 Ë 10 ¯ Ë 10 ¯
98 Comprehensive Trigonometry with Challenging Problems & Solutions for Jee Main and Advanced

3p p p p Ê 3 2
2
fi £ t £ or - £ t £ - 2 2 ˆ
10 2 2 10 fi Á 2 sin x - 3 cos x˜ = 0
È p p ˘ È 3p p ˘ Ë ¯
t Œ Í- , - ˙ » Í , ˙
Î 2 10 ˚ Î 10 2 ˚ Ê 3 2 2 2 ˆ
fi Á 2 sin x - 3 cos x˜ = 0
Ë ¯
1
cos (a - b ) = 1 and cos (a + b ) = fi
3 2 2
e sin x = cos 2 x
2 3
fi cos (a - b ) = cos (0)
sin 2 x cos 2 x 1
fi a=b fi = =
2 3 5
1
cos (a + b ) = 2
e fi tan 2 x =
3
1
fi cos (2a ) = sin 8 x cos8 x
e +
-p < a < p 8 27

(sin x) + (cos x)
4 4
-2p < 2a < 2p 2 2
=
8 27
1
(a, b cos (2a ) = ( 2 / 5) ( 3 / 5 )4
4
e = +
8 27
p =
2
+
3
0<q < 4
4 5 54
fi 0 < tan q < 1 and cot q > 1 2+3 1 1
= = 3=
54 5 125
0 < x < 1 and 0 < a < b
38. Let f (q ) = sin q + 3 sin q cos q + 5 cos q
2 2
a b
Ê 1ˆ Ê 1ˆ
fi then xb < x a < Á ˜ < Á ˜ = 1 + 3 sin q cos q + 4 cos 2 q
Ë x¯ Ë x¯
3
fi ( tan q )cot q < ( tan q )tan q < (cot q )tan q < (cot q )cot q = 1 + sin 2q + 2 (1 + cos 2q )
2
fi t2 < t1 < t3 < t4 3
= 3 + sin 2q + 2 cos 2q
2

sin 4 x cos 4 x 1 9 5 11
+ = +4 +3= +3=
2 3 5 4 2 2
5 4 5 9 5 1
fi sin x + cos 4 x = 1 - +4 +3= - +3=
2 3 4 2 2
Ê 3ˆ 4 Ê 2ˆ
ÁË1 + ˜¯ sin x + ÁË1 + ˜¯ cos x = 1
4
fi 1 2
2 3
sin q + 3 sin q cos q + 5 cos q
2 2
11
fi ( Ê3
) 2 ˆ
sin x + cos x + Á sin 4 x + cos 4 x˜ = 1
4 4
Ë2 3 ¯ P : sin q - cos q = 2 cos q


Ê3 2 ˆ
1 - 2 sin 2 x cos 2 x + Á sin 4 x + cos 4 x˜ = 1
Ë2 ¯
fi sin q = ( )
2 + 1 cos q
3
sin q
cos q =

Ê3 4 2 2 ˆ
ÁË sin x + cos x - 2 sin x cos x˜¯ = 0
2 3
4 2 fi
( 2 +1 )
The Ratios and Identities 99

cos q =
sin q
¥
( 2 - 1) fi 7q = p
p

( 2 + 1) ( 2 - 1) fi 7. = p
n
cos q = ( 2 - 1) sin q
fi n

1
cos q + sin q = 2 sin q 42. Let cos 4q =
fi 3
Q : cos q + sin q = 2 sin q 1
fi 2 cos 2 ( 2q ) - 1 =
3
P=Q
1 4
1 1 1 fi 2 cos 2 ( 2q ) = 1 + =
= + 3 3
Êpˆ Ê 2p ˆ Ê 3p ˆ
sin Á ˜ sin Á ˜ sin Á ˜ 2
Ë n¯ Ë n¯ Ë n ¯ fi cos 2 ( 2q ) =
3
p
Let =q 2
n fi cos (2q ) = ±
3
1 1 1
Then = + 2
sin q sin 2q sin 3q fi 2 cos 2 q - 1 = ±
1 1 1 3
fi - =
sin q sin 3q sin 2q 2
fi 2 cos 2 q = 1 ±
sin 3q - sin q 1 3
fi =
sin q sin 3q sin 2q 2 2 cos 2 q
f (cos 4q ) = =
2 cos 2q sin q 1 2 - sec 2 q 2 cos 2 q - 1
fi =
sin q sin 3q sin 2q Ê 2ˆ
2 Á1 ±
fi 2 cos 2q
=
1
Ê 1ˆ Ë 3 ˜¯
sin 3q sin 2q fi fÁ ˜=
Ë 3¯ Ê 2ˆ
2 Á1 ± -1
fi sin 4q = sin 3q Ë 3 ˜¯
fi sin 4q = sin (p - 3q )
3
fi 4q = p - 3q = 1± .
2
CHAPTER 2
Graphs of Trigonometric Functions

2.1 INTRODUCTION Ex-1. Let f (x) = x + 2


Y
There are six trigonometric ratios. In this section, we shall
describe each trigonometric function and their characteristics

2
and graph.

+
x
=
1. Sine function: A function f : R Æ R is

y
X¢ X
O
f (x) = sin x
Graph of f (x) = sin x:

x
=
Y
y

Y=1
X¢ X
-2 - 0 2
Y = -1 Ex-2 Let f (x) = x2 + 1
Y

Characteristics of sine function:
1. Df = R
X¢ X
2. Rf = [–1, 1] O
3. It is an odd function.
4. It is a periodic function
5. It is non-monotonic function
p Y¢
6. If sin x = 1 fi x = (4n + 1) , n ΠI
4
p
7. sin x = Р1 fi x = (4n Р1) , n ΠI Ex-3. Let f (x) = x3 + 1
4
8. sin x = 0 fi x = np, n ΠI Y
9. If sin x > 0 fi x Π(2n p, (2n + 1)p), n ΠI
10. If sin x < 0 fi x Π((2n Р1)p, 2np), n ΠI.
Ê p pˆ
11. If x > y fi sin x > sin y, " x, y ŒÁ - , ˜
Ë 2 2¯ X¢ X
O
2. RULE TO DRAW THE GRAPHS OF
DIFFERENT TYPES OF FUNCTIONS
Rule I: y = f (x) transforms to y = f (x) + a
Rule: Lift the graph of y = f (x), a-units upwards. Y¢
Graphs of Trigonometric Functions 101

Ex-4. Let f (x) = ex + 1 Ex-3. Let f (x) = x3 – 1


Y Y

X¢ X X¢ X
O O

Y¢ Y¢

Ex-5. Let f (x) = sin x + 2 Ex-4. Let f (x) = 2x – 1


Y Y

Y=3

Y=1 X¢ X
X¢ O
X
0


Rule II: y = f (x) transforms to y = f (x) – b Ex-5. Let y = sin x – 3


Rule: Down the graph of y = f (x), b-units down-
Y
wards.

Ex-1. Let f (x) = x – 2 y=1


X¢ X
Y O
y = -1
y = -2

y = -4
X¢ X
O

Rule III: y = f (x) transforms to y = f (x – a)



Rule: Shift the graph of y = f (x), a-units right-
wards.
Ex-2 Let f (x) = x2 – 1
Y Ex-1. Let f (x) = (x – 1)2
Y

X¢ X
O
X¢ X
O



102 Comprehensive Trigonometry with Challenging Problems & Solutions for Jee Main and Advanced

Ex-2. Let f (x) = (x – 2)3 Ex-1. Let f (x) = (x + 2)2


Y Y

X¢ X X¢ X
O O

Y¢ Y¢

Ex-2. Let f (x) = (x + 1) 3

Ex-3. Let f (x) = |x – 2| Y


Y

X¢ X
X¢ X
O O

Y¢ Y¢

Ex-3. Let f (x) = |x + 2|


Ê pˆ
Ex-4. Let f ( x ) = sin Á x - ˜ Y
Ë 4¯
Y

y=1
X¢ X
X¢ X O
O
y = -1


Ex-5. Let f (x) = ex–1 Ê pˆ


Ex-4. Let f ( x ) = sin Á x - ˜
Y Ë 4¯
Y

1
y=1
X¢ X
O 1 X¢ X
O O
y = -1

Y¢ Y¢

Ex-5. Let f (x) = ex + 1


Rule IV: y = f (x) transforms to y = f (x + b) Rule V: y = f (x) transforms to y = – f (x)
Rule: Shift the graph of y = f (x), b units leftwards. Rule: Take the image of the graph of y = f (x) with
respect to x-axis.
Graphs of Trigonometric Functions 103

Ex-1. Let f (x) = – x -x


Ex-1. f ( x ) = e
Y

X¢ X
O

Ex-2. Let f (x) = – |x|


Y

Ex-2. f ( x ) = log e ( - x )

X¢ O X


x
Ex-3. Let f (x) = – e
Y

X¢ X
O
Ex-3. f ( x ) = 2- x

Ex-4. Let f (x) = – logex


Y

X¢ X
O


Ex-4. f ( x ) = e1- x
Ex-5. Let f (x) = – sin x
Y Y

y=1

X¢ X 1
X¢ X
y = -1 O 1



Rule VI: y = f (x) transforms to y = f (–x)
Rule: Take the image of the graph of y = f (x) with
respect to y-axis.
104 Comprehensive Trigonometry with Challenging Problems & Solutions for Jee Main and Advanced

Ex-5. f ( x ) = sin ( - x ) = - sin ( x ) Ex-2. f ( x) = 2 x2


Y Y

y=1 y=1

X¢ X
O
y = -1 X¢ X
-p O p

y = -1

Rule VII: when y = f (x) transforms to y = f (ax), Y¢


where a > 1
Rule: Shrink the graph of y = f (x), a-times along Rule IX: when y = f (x) transforms to y = af (x),
the x-axis where a > 1.
Rule: Stretch the graph of y = f (x), a –times along
Ex-1. Let f (x) = sin 2x the y-axis.
Y
Ex-1. Let f (x) = 2x
y=1 Y
X¢ X

2x
y = -1

y=
Y¢ X¢ X
O
Ex-2. Let f (x) = sin 3x
x
Y =
y

y=1 Y¢
X¢ X
- 3p O 3p
y = -1
Ex-2. Let f (x) = 2x2
Y

Rule VIII: when y = f (x) transforms to y = f (ax),


where 0 < a < 1 y = 2x2 y = x2
1 X¢ X
Rule: Stretch the graph of y = f (x), – times O
along the x-axis. a
Ê xˆ
Ex-1. Let f (x) = sin ÁË ˜¯
2 Y¢
Y

y=1 Ex-3. Let f (x) = 2x3


Y

X¢ -p p X
O

y = -1
X¢ X
O

y = 2x3

Graphs of Trigonometric Functions 105

Ex-4. Let f (x) = 2sin x | x|


Y
Ex-3. Let f (x) =
2
Y
y=2
y=1
X¢ X
-2p -p O p 2p
y = -1
X¢ X
y = -2 O

Ex-5. Let f (x) = 3 sin x Y¢


Y

1
Ex-4. Let f (x) = sinx
y=3 2
Y
y=2
y=1
y = 1/2
X¢ X
-2p -p O p 2p X¢
y = -1 X
-2p -p O p 2p
y = -2 y = -1/2

y = -3

Y¢ 1
Ex-5. Let f (x) = sin x
Rule X: when y = f (x) transforms to y = a f (x), 3
where 0 < a < 1. Y
1
Rule: Shrink the graph of y = f (x), –times along
the y-axis. a y = 1/3
X¢ X
Ê xˆ -2p -p O p 2p
Ex-1. Let f (x) = ÁË ˜¯ y = -1/3
2
Y

x
=
y

Rule XI: when y = f (x) transforms to y = | f (x)|.


x/2
y= Rule : 1. First, we draw the graph of y = f (x)
X¢ X
O : 2. Leave the part of y = f (x) as it is which
lies above x-axis
x
=

: 3. Take the image of the graph of y = f (x) with


y

respect to x-axis, which lies below x-axis.



Ex-1. Let y = |x|
x2
Ex-2. Let f (x) = Y
2
Y

X¢ X
O
X¢ X
O



106 Comprehensive Trigonometry with Challenging Problems & Solutions for Jee Main and Advanced

Ex-2. Let y = |ex| Ex-1. Let y = e|x|


Y Y

X¢ X X¢ O X
O

Y¢ Y¢

Ex-2. Let y = e–|x|


Ex-3. Let y = |loge x| Y
Y

X¢ X
O
X¢ X
O



Ex-3. Let y = loge|x|
Ex-4. Let y = |sin x| Y

X¢ X
O
X¢ X
O



Ex-4. Let y = sin|x|
Ex-5. Let y = |x – 1|
2 Y

X¢ X
O
X¢ O X

Y¢ Ex-5. Let y = |2 – |x – 1||


Y
Rule XI: when y = f (x) transforms to y = f (|x|).
Rule : 1. First, we draw the graph of y = f (x)
: 2. Remove the part of y = f (x), which lies
left of y-axis X¢ X
O
: 3. Take the image of the part of y = f (x)
which lies right of y-axis with respect to
y-axis.

Graphs of Trigonometric Functions 107

Rule XII: when y = max{ f (x), g(x)} Ex-5. Let y = max{sin x, – sin x}, "x Œ[-2p , 2p ]
Rule : 1. First, we draw the graph of y = f (x) and Y
y = g(x) independently on the same co-
ordinate axes.
: 2. Take the upper part between the graphs
of y = f (x) and y = g(x). X¢ X
O
Ex-1. Let y = max{–x, x}
Y

Rule XIII: when y = min{f (x), g(x)}


X¢ X Rule : 1. First, we draw the graph of y = f (x) and
O y = g(x) independently on the same co-
ordinate axes.
: 2. Take the lower part between the graphs
Y¢ of y = f (x) and y = g(x).
Ex-1. Let y = min{–x, x}
Ex-2. Let y = max{x , x}
3
Y
Y

X¢ X
O
X¢ X
O

Ex-2. Let y = min{x , x} 3

Y

Ex-3. Let y = max{x2, x}


Y

X¢ X
O

X¢ X
O

Ex-3. Let y = min{x3, x2}


Y

Ï 1¸
Ex-4. Let y = Ìsin x , ˝ , " x Œ[ -2p , 2p ]
Ó 2˛
Y X¢ X
O

y = 1/2
X¢ X
O


108 Comprehensive Trigonometry with Challenging Problems & Solutions for Jee Main and Advanced

Ï 1¸ Ex. 4. Draw the graph of f (x) = sin3x, " x Œ [–2p, 2p]


Ex-4. Let y = min Ìsin x, ˝ , "x Œ[-2p , 2p ] Y
Ó 3˛
Y
y=1

y = 1/3
X¢ X
X¢ X - 2p - 3p - p -p O p p 3p 2p
O
2 2 2 2
y = -1

Y¢ Y¢

Ex-5. Let y = min{x3, 1, x2}. Ex. 5. Draw the graph of f (x) = xsin x
Y
y

X¢ X
O


-3p -2p -p O p 2p 3p

2.1.1 Some solved examples


Ï 1¸
Ex. 1. Let f (x) = min Ìsin x, ˝ , " x[-2p , 2p ]
Ó 2˛ y¢

Y
Ex. 6. Draw the graph of f (x) = x + sin x
y = 1/2
As we know that
X¢ X – 1 £ sin x £ 1
O
fi x – 1 £ x + sin x £ x + 1
fi x – 1 £ f (x) £ x + 1

y

{Ê pˆ
Ex. 2. Let f ( x ) = min sin x,sin Á x - ˜ , " x [ -2p , 2p ]
Ë 4¯ }
Y
x¢ x

X¢ X
- 2p - 3p - p -p O p p 3p 2p
2 2 2 2 y¢

Ex. 7. Draw the graph of


Y¢ f (x) = 2sin x, " x Œ[-2p , 2p ]
y
Ex. 3. Draw the graph of f (x) = sin2x, " x Œ [–2p, 2p]
Y

x¢ x
-2p - 3p -p -p O p p 3p 2p
X¢ X
- 2p - 3p -p -p O p p 3p 2p 2 2 2 2
2 2 2 2


Graphs of Trigonometric Functions 109

Ex. 8. Draw the graph of f (x) = sin|x| + |x| 3. Cosine function: A function f : R Æ R f (x)
y = cos x.
Graph of f (x) = cos x:
2

x¢ x 1
-p -p O p p
2 2

-2p -3p -p -p p p 3p 2p
2 2 2 2

-1

Ex. 9. Draw the graph of f (x) = sin|x| – |x|


y -2

2.2 CHARACTERISTICS OF
-p p

O
x
CO-SINE FUNCTION
1. Df = R
2. Rf = [–1, 1]

3. It is an even function.
Ex. 10. Draw a right graph of f (x) = sin (p x) 4. It is a periodic function
y 5. It is non-monotonic function
6. If cos x = 1 fi x = 2np, n ΠI
7. cos x = –1 fi x = (2n + 1)p, n Œ I
y=1 p
8. cos x = 0 fi x = (2n + 1) , n ΠI
x¢ x 2
-3 -2 -1 O 1 2 3
Ê p pˆ
y = -1 9. If cos x > 0 fi x = Á (2n - 1) , (2n + 1) ˜ , n Œ I
Ë 2 2¯
10. If cos x < 0
y¢ Ê p pˆ
fi x Œ Á (2n + 1) , (2n + 3) ˜ , n Œ I
Ë 2 2¯
Ex. 11 Draw the graph of f (x) = sin( x)
y 11. If x > y fi cos x < cos y, " x, y Π(0, p)

y=1 Ex. 1. Draw the graph of y = cos x + 1.


y

x¢ x
-3p -2p -p O p 2p 3p y=2

x¢ y=1

x
O
Ex. 12. Draw the graph of f (x) = sin (x – [x]), [,] = G.I.F
y y¢

Ex. 2. Draw the graph of y = cos x – 1.


y = sin 1 y

x¢ x O x
O x¢

y=-1

y¢ y=-2


110 Comprehensive Trigonometry with Challenging Problems & Solutions for Jee Main and Advanced

Ex. 3. Draw the graph of y = cos x - 1 . 1


Ex. 8. Draw the graph of y = cos x.
y 2
y

y=1
y = 1/2
x¢ x
O
y = -1 x¢ x
-p -3p -p -p O p p 3p 2p
2 2 2 2
y = -1/2

Ex. 4. Draw the graph of y = 1 – cos x. y¢


y
Ex. 9. Draw the graph of y = cos2x.
y
y = -1

y=1
y=1
x¢ x
O x¢ x
-p -3p -p -p O p p 3p 2p
y¢ 2 2 2 2

Ex. 5. Draw the graph of y = cos 2x.


y y¢

y=1 Ex. 10. Draw the graph of y = cos3x.


y

x¢ x
-p -3p -p -p O p p 3p p y=1
4 2 4 4 2 4
y = -1
x¢ x
-2p - 3p -p -p O p p 3p 2p

2 2 2 2
y = -1
Ex. 6. Draw the graph of y = 2 cos x.
y y¢

y=2 Ex. 11. Draw the graph of y = |cos x|.


y
x¢ x
-2p -3p -p -p O p p 3p 2p
2 2 2 2
y = -2 y=1

x¢ x
y¢ -2p -3p -p -p O p p 3p 2p
2 2 2 2
Ê xˆ
Ex. 7. Draw the graph of y = cos ÁË ˜¯ .
2

y

Ex. 12. Draw the graph of y = cos |x|.


y=1 y

y=1
x¢ x
-2p -3p -p -p O p p 3p 2p
2 2 2 2
y = -1 x¢ x
O

y = -1


Graphs of Trigonometric Functions 111

Ex. 13. Draw the graph of y = |cos |x|| 4. Tangent Function


y A function f : R Æ R
f (x) = tan x.
Graph of f (x) = tan x
y=1

x¢ x y = tan x
-2p -3p -p -p O p p 3p 2p
2 2 2 2


-
3p -p - p p p 3p
Ex. 14. Draw the graph of y = max{sin x, cos x} " x Π2 2 2 2
(– 2p, 2p).
y

y=1

x¢ x 2.3 CHARACTERISTICS OF TANGENT


-2p -3p -p -p O p p
FUNCTION
3p 2p
2 2 2 2

y = -1 p
1. Df = R Р(2n + 1) , n ΠI
2. Rf = R 2

3. It is an odd function.
Ex. 15. Draw the graph of y = min{sin x, cos x} " x Π4. It is a periodic function
(– 2p, 2p). 5. It is monotonic function
y p
6. If tan x = 1 fi x = (4n + 1) , n ΠI
4
y=1 p
7. If tan x = –1 fi x = (4n – 1) , n Œ I
4
x¢ x 8. If tan x = 0 fi x = np, n Œ I
-2p -3p -p -p O p p 3p 2p
2 2 2 2
9. If tan x > 0
Ê pˆ
y = -1 fi x Œ Á np , ( 2n + 1) ˜ , n Œ I
Ë 2¯
y¢ 10. If tan x < 0
Ê p ˆ
fi x Œ Á ( 2n - 1) , n p ˜ , n Œ I
Ex. 16. Draw the graph of Ë 2 ¯
Ï 1 ¸ 11. If x > y fi tan x > tan y " x, y Œ R – np, n Œ I
y = max Ìsin x, , cos x ˝ " x Œ (–2p, 2p).
Ó 2 ˛
y Ex. 1. Draw the graph of f (x) = |tan x|
y
y=1

x¢ x
-2p -3p -p -p O p p 3p 2p
2 2 2 2 x¢ x
-2p -3p -p -p O p p 3p 2p
y = -1 2 2 2 2


112 Comprehensive Trigonometry with Challenging Problems & Solutions for Jee Main and Advanced

4. Co-tangent function: A function Ex. 2. Draw the graph of


f:RÆR f (x) = cot x f (x) = min{tan x, cot x}, " x Œ [–2p, 2p]
Graph of f (x) = cot x y
y
y = cot x

x¢ x
-2p -3p -p -p O p p 3p 2p
x
-2p -3p -p -p O p p 3p 2p 2 2 2 2
2 2 2 2

5. Co-secant function: A function


f:RÆR f (x) = cosec x
2.4 CHARACTERISTICS OF Graph of f (x) = cosec x
CO-TANGENT FUNCTION y

1. Df = R Рnp, n ΠI
2. Rf = R
3. It is an odd function. y=1
4. It is a periodic function
x¢ x
5. It is monotonic function -2p -3p -p -p O p p 3p 2p
p 2 2 2 2
y = -1
6. If cot x = 1 fi x = (4n + 1) ,nŒI
4
p
7. If cot x = –1 fi x = (4n – 1) , n Œ I y¢
4
p
8. If cot x = 0 fi x = (2n +1) , n ΠI
9. If cot x > 0
2 2.5 CHARACTERISTICS OF
Ê pˆ CO-SECANT FUNCTION
fi x Œ Á np , (2n + 1) ˜ , n Œ I
Ë 2¯ 1. Df = R – np, n Œ I
10. If cot x < 0
Ê p ˆ 2. R f = ( -•, - 1] » [1, • )
fi x Œ Á ( 2n - 1) , n p ˜ , n Œ I
Ë 2 ¯ 3. It is an odd function
11 x > y fi cot x < cot y, 4. It is a periodic function
p
" x, y ΠR Р(2n + 1) , n ΠI 5. It is non-monotonic function
2
Ex. 1. Draw the graph of p
f (x) = max{tan x, cot x}, " x Œ [–2p, 2p] 6. If cosec x = 1 fi x = (4n + 1) ,nŒI
4
y
p
7. If cosec x = –1 fi x = (4n – 1) ,nŒI
4
8. cosec x can never be zero
9. If cosec x > 0 fi x Π(2np, (2n + 1)p), n ΠI
x¢ x
-2p -3p -p -p O p p 3p 2p 10. If cosec x < 0 fi x Π((2n Р1)p, 2np), n ΠI.
2 2 2 2

6. Secant function: A function


f: R Æ R f (x) = sec x
y¢ Graph of f (x) = sec x
Graphs of Trigonometric Functions 113

y Ex. 3. Draw the graph of y = 2sin x


y

y=2
y=1
x¢ x y = 1/2
-2p -3p -p -p O p p 3p 2p
2 2 2 2 x¢ x
-2p -3p -p -p O p p 3p 2p
2 2 2 2

y¢ Ex. 4. Draw the graph of y = 2–sin x


y

2.6 CHARACTERISTICS OF SECANT y=2

FUNCTION: y=1
1. Df = R Р(2n + 1), n ΠI y = 1/2
x¢ x
2. R f = ( -•, - 1] » [1, • ) -2p -3p -p -p O p p 3p 2p
3. It is an even function 2 2 2 2
4. It is a periodic function
5. It is non-monotonic function y¢
6. If sec x = 1 fi x = 2np, n ΠI
7. If sec x = –1 fi x =( 2n + 1)p, n Œ I
8. sec x can never be zero Ex. 5. Draw the graph of y = 2|cos x|
p pˆ y
Ê
9. If sec x > 0 fi x ŒÁ ( 4n - 1) , ( 4n - 1) ˜ , n Œ I
Ë 2 2¯
y=2
10. If sec x < 0
Ê p pˆ y=1
fi x Œ Á (4n + 1) , (4n + 3) ˜ , n Œ I
Ë 2 2¯ x¢ x
10. Some solved examples: -2p -3p -p -p O p p 3p 2p
2 2 2 2
Ex. 1. Draw the graph of y = 2cos x
y y¢

y=2
Ex. 6. Draw the graph of y = 2|sin x|
y=1 y
y = 1/2
x¢ x
-2p -3p -p -p O p p 3p 2p y=2
2 2 2 2
y=1

x¢ x
Ex. 2. Draw the graph of y = 2–cos x -2p -3p -p -p O p p 3p 2p
y 2 2 2 2

y=2 y¢

y=1
y = 1/2
x¢ x
-2p -3p -p -p O p p 3p 2p
2 2 2 2

114 Comprehensive Trigonometry with Challenging Problems & Solutions for Jee Main and Advanced

Ex. 7. Draw the graph of y = sin x + |sin x| Ex. 10. Draw the graph of y = tan x ◊ cot x
We have y = sin x + |sin x| We have y = tan x ◊ cot x
Ïsin x + sin x : sin x ≥ 0 Ï p ¸
= Ì = 1, x Œ Ì(2n + 1) - np ˝ , n Œ I
Ó sin x - sin x :sin x < 0 Ó 2 ˛
y
ÏÔ2 sin x : x ŒÈÎ2np , ( 2n + 1)p ˘˚ , n Œ I
= Ì
ÔÓ 0 : x Œ(( 2n - 1)p , 2np ) , n Œ I
y=1

y
x¢ x
-2p -3p -p -p O p p 3p 2p 5p
y=2 2 2 2 2 2

y=1

x¢ x
-2p -3p -p -2p -p -p O p p p p
2p 3p
|sin x|
2 2 2 2 2 2 Ex. 11. Draw the graph of y =
sin x
y¢ |sin x|
We have y =
Ex. 8. Draw the graph of y = sin x + sin |x| sin x
We have y = sin x + sin |x| Ï sin x
Ï2 sin x : x ≥ 0 ÔÔ sin x : sin x ≥ 0
= Ì = Ì
Ó0 :x < 0 Ô- sin x : sin x < 0
y ÔÓ sin x
y=2 ÏÔ1 : x Œ ÈÎ2np , (2n + 1) p ˘˚ , n Œ I
= Ì
ÓÔ -1 : x Œ(2np , (2n + 1) p ) , n Œ I
y=1
x¢ x
O p p 3p 2p
2 2 y = -1 y
y = -2

y¢ y=1

Ex. 9. Draw the graph of y = cos x + |cos x| x¢ x


-3p -2p -p O p 2p 3p
We have y = cos x + |cos x| y = -1
Ïcos x + cos x :cos x ≥ 0
= Ì
Ócos x - cos x :cos x < 0 y¢

Ï È p p˘ Ex. 12. Draw the graph of y = 2x sin x


Ô2 cos x : x Œ Í( 2n - 1) 2 , (2n + 1) 2 ˙ , n Œ I
Ô Î ˚ We have –1 £ sin x £ 1
= Ì
Ô 0 : x ŒÊ Ê (2n + 1) p ˆ , ( 2n + 3) p ˆ ,n Œ I fi –2x £ 2x sin x £ 2x
ÔÓ ÁË ÁË 2¯
˜
2 ˜¯ fi –2x £ f (x) £ 2x
y

y=2 Y

y=1

x¢ x
-5p -p -3p -p -p O p p 3p p 5p X¢ X
2 2 2 2 2 2 O



Graphs of Trigonometric Functions 115

sin x Ê 1ˆ
Ex. 13. Draw the graph of y = fi – x £ xsin Á ˜ £ x
x Ë x¯
We have – 1 £ sin x £ 1
Ê 1ˆ
1 sin x 1 Hence, the curve y = x sin Á ˜ lies between
fi - £ £ Ë x¯
x x x y = x and y = – x.
Ê Ê 1ˆˆ Ê Ê 1ˆˆ
Ê sin x ˆ Also, Lt + Á x sin Á ˜ ˜ = 0 = Lt - Á x sin Á ˜ ˜
Also, lim Á ˜ =1 xÆ0 Ë Ë ¯
x ¯ xÆ0 Ë Ë x¯¯
xÆ0 Ë x ¯

sin x 1
Hence, y = lies between the curves It cuts the x-axis at x = , n ΠI Р{0}
x np
y
1 1
y= and y = –
x x
It is now periodic curve and cuts the x-axis at
x = np, n Œ I –{0} and does not cut the y-axis
x¢ x
y O

x¢ x LEVEL I
O
(PROBLEMS BASED ON FUNDAMENTALS)
Q. Draw the graphs of
1. f (x) = sin 2x
2. f (x) = sin 3x
Ê xˆ
3. f (x) = sin Á ˜
y¢ Ë 2¯
Ê 1ˆ Ê xˆ
Ex. 14. Draw the graph of y = sin Á ˜ 4. f (x) = sin Á ˜
Ë x¯ Ë 3¯
We have –1 £ sin x £ 1 1
p 5. f (x) = sin x
fi –1 £ sin ÊÁ x - ˆ˜ £ 1 2
Ë 6¯
1
1 6. f (x) = sin x –
It cuts the x-axis at x = , n Œ –I {0} 2
np
⎛ π⎞
But it does not cut the y-axis. 7. f (x) = sin ⎜ x − ⎟
y ⎝ 6⎠
Ê pˆ
8. f (x) = sin Á x + ˜
y = 1/2 Ë 6¯
2
p
1
p
9. f (x) = sin (x – 1)

1 O
x 10. f (x) = sin (x + 1)
p 11. f (x) = – sin x
y = - 1/2 1
12. f (x) = – sin x
2

Ê pˆ
13. f (x) = sin Á x - ˜ + 1
Ê 1ˆ Ë 4¯
Ex. 15. Draw the graph of y = x sin Á ˜ pˆ
Ë x¯ Ê
14. f (x) = sin Á x - ˜ – 1
Ê 1ˆ Ë 4¯
We have – 1 £ sin Á ˜ £ 1
Ë x¯ 15. f (x) = sin |x|
116 Comprehensive Trigonometry with Challenging Problems & Solutions for Jee Main and Advanced

16. f (x) = sin |x| + 1 58. f (x) = |tan x|


17. f (x) = sin |x| – 1 59. f (x) = cot 2x
18. f (x) = 1 – sin |x | 60. f (x) = |cot x|
19. f (x) = sin |x| 61. f (x) = tan x ◊ cot x
20. f (x) = sin |x – 1| 1 - cos 2 x
21. f (x) = sin |x + 1| 62. f (x) =
sin 2 x
22. f (x) = sin2 x
23. f (x) = sin2 x + 1 1 + cos 2 x
63. f (x) =
24. f (x) = sin2 x – 1 sin 2 x
25. f (x) = 1 – sin2 x sin 2 x
26. f (x) = sin2 x + cos2x 64. f (x) =
1 + cos 2 x
27. f (x) = |sin |x||
f (x) = |sin |x|| + 1 sin 2 x
28. 65. f (x) =
29. f (x) = |sin |x|| – 1 1 - cos 2 x
30. f (x) = cos 2x 66. f (x) = cosec 2x
31. f (x) = cos 4x 67. f (x) = sec 2x
Ê xˆ 68. f (x) = tan x + cot x
32. f (x) = cos Á ˜
Ë 2¯ 69. f (x) = cot x – tan x
Ê xˆ 70. f (x) = sin 2 x
33. f (x) = cos Á ˜
Ë 3¯ 71. f (x) = cos 2 x
34. f (x) = – cos x 72. f (x) = sin 2 x
35. f (x) = 1 – cos x
Ê pˆ 73. f (x) = - sin x
36. f (x) = cos Á x - ˜
Ë 4¯ - cos x
74. f (x) =
Ê pˆ
37. f (x) = cos Á x + ˜ Ê pˆ
Ë 4¯ 75. f (x) = sin Á x - ˜
Ë 4¯
38. f (x) = cos (x – 1)
39. f (x) = cos (x + 1) LEVEL II
Ê pˆ (PROBLEMS FOR JEE MAIN)
40. f (x) = cos Á x - ˜ + 1
Ë 4¯ Q. Draw the graphs of
1. f (x) = x sin x
Ê pˆ
41. f (x) = cos Á x + ˜ – 1 2. f (x) = x + sin x
Ë 4¯ 3. f (x) = x – sin x
42. f (x) = |cos x| 4. f (x) = 2x sin x
43. f (x) = – |cos x| 5. f (x) = ex sin x
44. f (x) = 1 – |cos x| 6. f (x) = sin x – cos x
45. f (x) = ||cos x|| 7. f (x) = sin x + cos x
46. f (x) = |cos (x – 1)| 8. f (x) = x cos x
47. f (x) = cos |x| 9. f (x) = x + cos x
48. f (x) = cos |x| + 1 10. f (x) = x – cos x
49. f (x) = cos2 x 11. f (x) = sin x
50. f (x) = cos3x
51. f (x) = cos2 x – sin2x 12. f (x) = - sin x
1 13. f (x) = sin x + - sin x
52. f (x) = cos2 x
2
53. f (x) = sin x + cos x 14. f (x) = sin x - - sin x
54. f (x) = sin x – cos x 15. f (x) = sin x + |sin x|
55. f (x) = tan 2x 16. f (x) = sin x – |sin x|
56. f (x) = tan2x 17. f (x) = sin x + sin |x|
57. f (x) = tan3x 18. f (x) = sin x – sin |x|
Graphs of Trigonometric Functions 117

19. f (x) = cos x + |cos x| (x) f (x) = 3cos x + 5


20. f (x) = cos x – |cos x| (xi) f (x) = –2 cos x + 3
21. f (x) = cos x + cos |x| (xii) f (x) = 3sin2 x + 4
22. f (x) = cos x – cos |x| (xiii) f (x) = 2 – 4sin2 x
Ï 1¸ (xiv) f (x) = 3sin2 x + 4cos2 x
23. f (x) = max Ìsin x, ˝ (xv) f (x) = 2sin2 x – 3cos2 x
Ó 2˛
(xvi) f (x) = 3sin2 x + 4cos2 x + 10sin x cos x
Ï Ê pˆ Ê pˆ ¸ 1
26. f (x) = max Ì sin Á x - ˜ , sin Á x + ˜ ˝ (xvii) f (x) =
Ó Ë 4¯ Ë 4¯ ˛ 3 sin x + 2 cos 2 x
2

27. f (x) = tan x + tan|x| 1


28. f (x) = tan x – tan|x| (xviii) f (x) =
3 sin x - 2 cos 2 x
2
29. f (x) = tan x ◊ cot x
(xix) f (x) = 25 sin x + 6 cos x
2 2
30. f (x) = sin x ◊ cosec x
31. f (x) = cos x ◊ sec x (xx) f (x) = log2(3sin2x + 1)
32. Find the number of solutions of 34. Find the domains and ranges of
(i) sin x = x (i) f (x) = 2sin x + 4
(ii) sin x = |x| (ii) f (x) = sin x + cos x
(iii) sin x = 2|x| (iii) f (x) = sin x + cos x + 1
1 (iv) f (x) = 3sin x + 4cos x + 10
(iv) sin x = |x|
2 Ê pˆ
(v) f (x) = sin Á x - ˜ + cos x
1 Ë 4¯
(v) sin x = x + , x > 0
x Ê pˆ Ê pˆ
(vi) f (x) = sin Á x - ˜ + cos Á x - ˜
1 Ë 4 ¯ Ë 6¯
(vi) sin x = x + , x < 0
x Ê pˆ Ê pˆ
(vii) sin x = x + x + 1
2 (vii) f (x) = sin Á x - ˜ + cos Á x - ˜
Ë 4¯ Ë 4¯
(viii) sin x = – x2 + x – 1
(ix) sin x = 2x + 2–x (viii) f (x) = 3sin2x + 2
(x) sin x = 2x – 2–x (ix) f (x) = 3sin2 x + 2cos2 x
3 (x) f (x) = 3sin2 x – 4cos2 x
(xi) 2sin x = , " x Œ [–2p, 2p]
4 LEVEL III
3
(xii) 2cos x = , " x Œ [–2p, 2p] (PROBLEMS FOR JEE ADVANCED)
4 Q. Draw the graphs of
5 Ê 1ˆ
(xiii) 2|sin x| = , " x Œ [–2p, 2p] 1. f (x) = sin Á ˜
6 Ë x¯
|cos x|
(xiv) 2 = 1, " x Œ [–2p, 2p]
Ê 1ˆ
1 2. f (x) = xsin Á ˜
(xv) 3sin x = , " x Œ [–2p, 2p] Ë x¯
2
3. f (x) = sin x + |x|
(xvi) 2sin x = |sin x|, " x Œ [–2p, 2p]
4. f (x) = sin x – |x|
(xvii) 2cos x = |sin x|, " x Œ [–2p, 2p]
5. f (x) = |x| + sin x
(xix) 2cos x = |cos x|, " x Œ [–2p, 2p]
6. f (x) = |x| – sin |x|
33. Find the maximum and minimum values of
7. f (x) = |x| + cos |x|
(i) f (x) = sin x + cos x
8. f (x) = |x| – cos |x|
(ii) f (x) = sin x – cos x
9. f (x) = log2 (sin x)
(iii) f (x) = 3sin x + 4cos x + 10
10. f (x) = log2 (cos x)
(iv) f (x) = 5sin x + 12cos x + 12
11. f (x) = [sin x], where [,] = G.I.F
(v) f (x) = sin (x – 1) + cos x
12. f (x) = [2sin x], [,] = G.I.F
(vi) f (x) = sin x + cos (x – 1)
13. f (x) = [sin x + cos x], [,] = G.I.F
(vii) f (x) = 3sin x + 4
14. f (x) = max{sin x, cos x}," x Œ [–2p, 2p]
(viii) f (x) = 2 – 4sin x
(ix) f (x) = – 3 – 2sin x Ï 1 ¸
15. f (x) = max Ìsin x , , cos x ˝ ," x Œ [–2p, 2p]
Ó 2 ˛
118 Comprehensive Trigonometry with Challenging Problems & Solutions for Jee Main and Advanced

sin x + cos x sin x - cos x (iii) f (x) = sin4 x + cos4 x


16. f (x) = - ," x Œ [–2p, 2p] (iv) f (x) = sin6 x + cos6 x
2 2
22. Find the number of solutions of
sin x + cos x sin x - cos x (i) 2sin x = sin x, " x Π[0, 4p]
17. f (x) = + ," x Œ [–2p, 2p] (ii) 2sin x = |sin x|, " x Œ [0, 4p]
2 2
(iii) 2cos x = |sin x|, " x Π[0, 8p]
Ê 1ˆ (iv) 2cos x = |cos x|, " x Œ [0, 10p]
18. f (x) = x sin Á ˜ (v) 2cos x = cos x, " x Œ [0, 20p]
Ë x¯
(vi) xsin x Р1 = 0, " x Π[0, 2p]
19. f (x) = sgn (sin x) (vii) xsin2 x Р1 = 0, " x Π[0, 2p]
20. f (x) = sgn (cos x) (viii) sin x = 2x + 2x, " x Π[0, 2p]
21. Find the maximum and minimum values of (ix) cos x = 2x Р2x, " x Π[0, 2p]
(i) f (x) = sin2 x + cos4 x (x) x2cos x Р2 = 0, " x Π[0, 2p]
(ii) f (x) = sin4 x + cos2 x

ANSWERS
Level II Ê 13 101 ˆ
32. (i) 1 (ii) 1 (iii) 1 Min V = Á -
(iv) 3 (v) 0 (vi) 0 Ë2 2 ˜¯
(vii) 0 (viii) 0 (ix) 0 (xvii) Max V = 1/2 and Min V = 1/3
(x) 3 (xi) 4 (xii) 4 (xviii) Max V = 1/3 and Min V = –1/2
(xiii) 0 (xiv) 4 (xv) 4 (xix) Max V = 26 and Min V = 25
(xvi) 4 (xvii) 8 (xviii) 4 (xx) Max V = 2 and Min V = 0
33. (i) Max V = 2 and Min V = – 2 33. (i) Df = R and Rf = [2, 6]
(ii) Max V = 2 and Min V = – 2 (ii) Df = R and Rf = [- 2 , 2 ]
(iii) Max V = 15 and Min V = 5
(iv) Max V = 25 and Min V = –1 (iii) Df = R and Rf = [1 - 2 ,1 + 2 ]
Ê Ê 1ˆ Ê 1ˆˆ (iv) Df = R and Rf = [5, 15]
(v) Max V = 2 Á cos Á ˜ - sin Á ˜ ˜
Ë Ë 2 ¯ Ë 2¯ ¯ (v) Df = R and Rf = [- 5 + 2 2 , 5 + 2 2 ]
Ê Ê 1ˆ Ê 1ˆˆ
Min V = - 2 Á cos Á ˜ - sin Á ˜ ˜ (vi) Df = R and Rf = ÈÍ- 5 + 2 + 6 , 5 + 2 + 6 ˘˙
Ë Ë 2¯ Ë 2¯ ¯ Î ˚
(vii) Df = R and Rf = [- 2 , 2 ]
Ê Ê 1ˆ Ê 1ˆˆ
(vi) Max V = 2 Á cos Á ˜ + sin Á ˜ ˜ (viii) Df = R and Rf = [2, 5]
Ë Ë 2¯ Ë 2¯ ¯
(ix) Df = R and Rf = [2, 3]
Ê Ê 1ˆ Ê 1ˆˆ (x) Df = R and Rf = [–4, 3]
Min V = - 2 Á cos Á ˜ + sin Á ˜ ˜
Ë Ë 2¯ Ë 2¯ ¯
Level-III
(vii)Max V = 7 and Min V = 1
21. (i) Max V = 1 and Min V = 3/4
(viii)Max V = 6 and Min V = – 2
(ii) Max V = 1 and Min V = 3/4
(ix)Max V = –1 and Min V = – 5
(iii) Max V = 1 and Min V = 1/2
(x)Max V = 8 and Min V = 2
(iv) Max V = 1 and Min V = 1/4
(xi)Max V = 5 and Min V = 1
22. (i) 0 (ii) 4
(xii)Max V = 7 and Min V = 4
(iii) 16 (iv) 10
(xiii)Max V = 2 and Min V = – 2
(v) 0 (vi) 2
(xiv) Max V = 4 and Min V = 3
(vii) 2 (viii) 0
(xv) Max V = 2 and Min V = – 3
(ix) 1 (x) 2
Ê 13 101 ˆ
(xvi) Max V = Á +
Ë2 2 ˜¯
CHAPTER 3
The Trigonometric Equation

3.1 DEFINITION 2. cos q = 0 fi p


q = (2n + 1) , n ΠI
An equation involving one or more trigonometrical ratios of 2
unknown angle is called trigonometrical equation. 3. tan q = 0 fi q = np , n ΠI
For examples, sin2 x – cos x – 2 = 0, tan x = 1, Step II
cos2 x + cos x – 2 = 0 etc. are trigonomteric equations. p
1. sinq = 1 fi q = ( 4n + 1) , n ŒI
2
3.2 SOLUTION OF A TRIGONOMETRIC 2. cosq = 1 fi q = 2np , n ΠI
EQUATION 3. tanq = 1 fi p
q = ( 4n + 1) , n ΠI
4
trigonometrical equation is called a solution or root of the Step III
p
equation 1. sinq = -1 fi q = ( 4n - 1) , n ŒI
1 p 5p 2
For example, sinq = fiq = , .
2 6 6 q = (2n + 1) p , n ΠI
2. cosq = -1 fi
Types of Solutions: p
(i) Principal Solution 3. tanq = -1 fi q = ( 4n - 1) , n ŒI
4
(ii) General Solution.
Step IV
n
PRINCIPAL SOLUTION: 1. sin q = sin a fi q = np + ( -1) a , n ΠI
2. cos q = cos a fi q = 2np ± a , n Œ I
given equation is called the principal solutions.
3. tan q = tan a fi q = np + a, n ŒI
GENERAL SOLUTIONS: Step V
Since trigonometric functions are periodic function, 1. sin2 q = sin2 a fi q = np ± a , n Œ I
therefore solution of trigonometric equations can be 2. cos2 q = cos2 a fi q = np ± a , n Œ I
genralised with the help of periodicity of a trigonometrical 3. tan2 q = tan2 a fi q = np ± a , n Œ I
function. The solution consisting of all possible solution of
a trigonometrical equation is called the general solution.
3.4 RANGES OF TRIGONOMETRIC
3.3 GENERAL SOLUTION OF FUNCTIONS
TRIGONOMETRIC EQUATIONS Step I
The general solution of the equations are as follows: 1. -1 £ sin q , cos q £ 1
2. -1 £ - sin q , - cos q £ 1
Step I
q = np , n ŒI 3. - • < tan q , cot q < •
1. sin q = 0 fi
4. - • < - tan q , - cot q < •
120 Comprehensive Trigonometry with Challenging Problems & Solutions for Jee Main and Advanced

5. cosecq ,sec q Œ( - • ,1] » [1, • ) Ex-5. Solve for q: sin (9q ) = sin q .
6. -cosecq , - sec q Œ(- • ,1] » [1, • ) Soln. We have, sin (9q ) = sin q
Step II fi sin (9q ) - sin q = 0
1. 0 £ sin 2 q , cos 2 q £ 1 Ê 9q + q ˆ Ê 9q - q ˆ
fi 2 cos Á =0
Ë 2 ˜¯
sin Á
2. 0 £ tan 2 q , cot 2 q < • Ë 2 ˜¯
3. 1 £ cosec2q ,sec 2 q < • fi 2 cos (5q ) sin ( 4q ) = 0
Step III fi cos (5q ) = 0 & sin ( 4q ) = 0
1. -1 £ - sin 2 q , - cos 2 q £ 0 p
fi (5q ) = (2n + 1) & (4q ) = np
2. - • < - tan q , - cot q £ 0
2 2 2
p Ê np ˆ
3. - • < -cosec2q , - sec2 q £ -1 fi q = ( 2n + 1) & q =Á ˜ .
10 Ë 4¯
where n ΠI .
3.5 SOME SOLVED EXAMPLES
Ex-6. Solve for q: 5 sin 2 q + 3 cos 2 q = 4
Ex-1. Solve for q: sin 3q = 0 .
Soln. We have, sin 3q = 0 Soln. We have, 5 sin 2 q + 3 cos 2 q = 4
fi 3q = n p fi ( )
2 sin 2 q + 3 sin 2 q + cos 2 q = 4
np fi 2 sin q + 3 = 4
2
fi q= , where n ΠI
3
fi 2 sin 2 q = 1
Ex-2. Solve for q: cos (5q ) = 0 .
2
1
Soln. We have, cos (5q ) = 0 fi sin 2 q =
2
2
Êpˆ 2
fi cos 2 (5q ) = cos 2 Á ˜ Ê 1 ˆ Êpˆ
Ë 2¯ fi sin 2 q = Á = sin 2 Á ˜
Ë 2 ˜¯ Ë 4¯

fi (5q ) = np ± ÊÁË ˜ fi
Êpˆ
q = np ± Á ˜ , where n Œ I
2¯ Ë 4¯
1Ê Ê p ˆˆ
fi q = Á np ± Á ˜ ˜ , where n Œ I . Ex-7. Solve for q: tan(q - 15∞) = 3 tan(q + 15∞) .
5Ë Ë 2¯¯
Soln. We have, tan(q - 15∞) = 3 tan(q + 15∞)
Ex-3. Solve for q: tanq = 3 . tan(q - 15∞) 3
fi =
Soln. We have, tanq = 3 tan(q + 15∞) 1
Êpˆ tan (q - 15∞) + tan (q + 15∞) 3 + 1
fi tan q = tan Á ˜ fi =
Ë 3¯ tan (q - 15∞) - tan (q + 15∞) 3 - 1


Êpˆ
q = np + Á ˜ , where n Œ I . sin (q + 15∞ + q - 15∞) 3 + 1
Ë 3¯ fi =
sin (q + 15∞ - q + 15∞) 3 - 1
Ex-4. Solve for q: sin 2q = sin q . fi 2 sin ( 2q ) = 2
Soln. We have, sin 2q = sin q
fi sin ( 2q ) = 1
fi 2sin q cos q = sin q
p
fi sin q (2 cos q - 1) = 0 q = (4n + 1) , n ΠI .
4
fi sin q = 0 & ( 2 cos q - 1) = 0 Ex-8. Solve for q: tan (q ) + cot (q ) = 2
2 2

Soln. We have, tan (q ) + cot (q ) = 2


1 2 2
fi sin q = 0 & cos q =
2 1
p fi tan 2 (q ) + =2
fi q = np & q = 2np ± ,where n Œ I . tan 2 (q )
3
The Trigonometric Equation 121

fi tan 4 (q ) - 2 tan 2 (q ) + 1 = 0 tan (2 q ) + tan (q )


fi =1
1 - tan (q ) tan (2q )
(tan (q ) - 1)
2
fi 2
=0
fi tan (3q ) = 1
fi (tan 2
(q ) - 1) = 0

Êpˆ
tan (3q ) = tan Á ˜
Ë 4¯
Êpˆ
fi q = np ± Á ˜ , n Œ I pˆ
Ë 4¯ fi (3q ) = np + ÊÁË ˜

Ex-9. Solve for q : cos (q ) + cos (2q ) + cos (3q ) = 0
Ê np ˆ Ê p ˆ
Soln. We have, fi q = Á ˜ + Á ˜ , n ŒI
Ë 3 ¯ Ë 12 ¯
fi (cos (3q ) + cos (q )) + cos (2q ) = 0
Ex-12. Solve for q : tan (q ) + tan ( 2q ) + tan (3q )
fi (cos (3q ) + cos (q )) + cos (2q ) = 0 = tan (q ) .tan (2q ) .tan (3q ) .
fi 2 cos ( 2q ) cos (q ) + cos ( 2q ) = 0 Soln. We have,
tan (q ) + tan ( 2q ) + tan (3q )
fi cos (2q ) (2 cos (q ) + 1) = 0
= tan (q ) .tan (2q ) .tan (3q )
fi cos (2q ) = 0 & ( 2 cos (q ) + 1) = 0
fi tan (q ) + tan ( 2q )

fi cos (2q ) = 0 & cos (q ) = -


1 = - tan (3q ) + tan (q ) .tan (2q ) .tan (3q )
2
fi tan (q ) + tan ( 2q )
Êpˆ Ê 2p ˆ
fi q = ( 2n + 1) Á ˜ & q = np ± Á ˜
Ë 4¯ Ë 3¯ = - tan (3q ) (1 - tan (q ).tan (2q ))
Ê tan (q ) + tan ( 2q ) ˆ
Ex-10. Solve for q: sin ( 2q ) + sin ( 4q ) + sin (6q ) = 0 . fi Á ˜ = - tan (3q )
Ë (1 - tan (q ) .tan (2q )) ¯
Soln. We have, sin ( 2q ) + sin ( 4q ) + sin (6q ) = 0
fi tan (3q ) = - tan (3q )
fi sin (6q ) + sin (2q ) + sin (4q ) = 0 fi 2 tan (3q ) = 0
fi 2 sin ( 4q ) .cos ( 2q ) + sin (4q ) = 0 fi (3q ) = np
Ê np ˆ
fi sin ( 4q ) ( 2 cos ( 2q ) + 1) = 0 fi q = Á ˜ , n ŒI .
Ë 3¯
fi sin ( 4q ) = 0 & ( 2 cos ( 2q ) + 1) = 0

fi (4q ) = np & cos (2q ) = -


1 EXERCISE 1
2
Solve the general values of q :
Ê np ˆ Ê 2p ˆ 1. sin 2q = 0
fi q = Á ˜ & ( 2q ) = np ± Á ˜
Ë 4¯ Ë 3¯
2. cos 3q = 0
Ê np ˆ Ê np ˆ Ê p ˆ 3. tan 5q = 0
fi q = Á ˜ & q = Á ˜ ± Á ˜ , n ŒI
Ë 4¯ Ë 2 ¯ Ë 3¯
Ê 5q ˆ
4. sin Á ˜ = 0
Ex-11. Solve for q : Ë 2¯
tan (q ) + tan (2q ) + tan (q ) tan (2q ) = 1 .
Ê 7q ˆ
5. cos Á ˜ = 0
Soln. We have, Ë 2¯
tan (q ) + tan ( 2q ) + tan (q ) tan (2q ) = 1 Ê 7q ˆ
6. tan Á ˜ = 0
fi tan (2 q ) + tan (q ) = 1 - tan (q ) tan (2q ) Ë 3¯
122 Comprehensive Trigonometry with Challenging Problems & Solutions for Jee Main and Advanced

7. sin (7q ) = 0
2
37. (1 - tan q ) (1 + sin 2q ) = 1 + tan q

8. cos (3q ) = 0 38. 2 sin 2 q + sin 2 2q = 2


2

39. sin 3q = 4 sin q .sin(q + a ).sin(q - a ) , a π np , p n Œ Z


9. sinq = 3
2 40. 4 sin q sin 2q sin 4q = sin 3q

10. cosq = 1
2 3.6 A TRIGONOMETRIC EQUATION
11. tanq = 2 IS OF THE FORM
12. 2 sinq + 1 = 0 a cos q ± b sin q = c
1 Rule: 1. Divide by a 2 + b 2 on both the sides
13. cos 3q = -
2 2. Reduce the given equation into
14. tan 3q + 1 = 0 either sin (q ± a ) or cos (q ± a )
3. simplify the given equation.
15. 2 sinq - 1 = 0
Ex-1. Solve for q : sin (q ) + cos (q ) = 1
16. 2 cosq - 3 = 0
Soln. We have, sin (q ) + cos (q ) = 1
17. 3 tanq = 1
Ê 1 1 ˆ
18. sec (q ) + 2 = 0 fi 2Á sin (q ) + cos (q )˜ = 1
Ë 2 2 ¯
19. sin 2q = sin q
Ê 1 1 ˆ 1
fi ÁË sin (q ) + cos (q )˜ =
20. cos 3q = cos q 2 2 ¯ 2
21. sin (3q ) = sin (q ) Ê pˆ 1
fi sin Á q + ˜ =
Ë ¯
22. sin (5q ) = cos ( 2q ) 4 2

23. 7 sin 2 q + 3 cos 2 q = 4 Ê p ˆ Ê Ê p ˆˆ


fi sin Á q + ˜ = Á sin Á ˜ ˜
Ë 4¯ Ë Ë 4¯¯
24. sin q + sin 3q + sin 5q = 0
Ê pˆ Ê n Ê p ˆˆ
25. cos q + cos 3q - 2 cos 2q = 0 fi ÁË q + ˜¯ = ÁË np + (-1) ÁË ˜¯ ˜¯
4 4
26. sin 2q + sin 4q + sin 6q = 0
Ê n Êpˆ pˆ
fi q = Á np + (-1) Á ˜ - ˜ , n Œ I
27. cot q + 3 + 3 = 0
2 Ë Ë 4¯ 4¯
sin q
28. 2 tan q - cot q = -1 Ex-2. Solve for q : 3 sin (q ) + cos (q ) = 2

29. tan 2 q + (1 - 3 ) tan q - 3 = 0 Soln. We have, 3 sin (q ) + cos (q ) = 2

30. tan q + tan 2q + tan q tan 2q = 1 3 1


fi sin q + cos q = 1
31. tan q + tan 2q + tan 3q = tan q tan 2q tan 3q 2 2
Ê pˆ
Ê pˆ Ê 2p ˆ fi sin Á q + ˜ = 1
32. tan q + tan Á q + ˜ + tan Á q + ˜ = 3 Ë 6¯
Ë 3 ¯ Ë 3¯
Ê pˆ Êpˆ
33. 3 tan(q - 15∞) = tan(q + 15∞) fi sin Á q + ˜ = sin Á ˜
Ë 6 ¯ Ë 2¯
34. tan q + tan 2q + tan 3q = 0 pˆ
Ê n Êpˆ
fi ÁË q + ˜¯ = np + (-1) ÁË ˜¯
35. cos 2q cos 4q = 1 6 2
2 n Êpˆ p
36. cot q - tan q = cos q - sin q fi q = np + (-1) Á ˜ -
Ë 2¯ 6
The Trigonometric Equation 123

Ex-3. Solve for q : 1Ê n Ê p ˆˆ


sin ( 2q ) + cos ( 2 q ) + sin (q ) + cos (q ) + 1 = 0 & q = Á np + (-1) Á ˜ ˜ , where n Œ I .
2Ë Ë 6 ¯¯
Soln. We have,
sin ( 2q ) + cos ( 2 q ) + sin (q ) + cos (q ) + 1 = 0
EXERCISE 2
fi (sin (q ) + cos (q )) + (1 + sin (2q ))
+ cos (2 q ) = 0 Q Solve for q :
1. sin q + 3 cos q = 2
fi (sin (q ) + cos (q )) + (sin (q ) + cos (q ))2 2. 2 sec q + tan q = 1
( )
+ cos 2 q - sin 2 q = 0
3. cotq + cosecq = 3
fi (sin (q ) + cos (q )) + (sin (q ) + cos (q )) 2
4. sin q + cos q = 2
+ (cos q + sin q ) (cos q - sin q ) = 0 5. 3 cos q + sin q = 1
6. sin q + cos q = 1
fi (sin (q ) + cos (q ))
7. cosecq = 1 + cot q
(1 + (sin (q ) + cos (q )) + (cos q - sin q )) = 0 8. tan q + sec q = 3
fi (sin (q ) + cos (q )) (1 + 2 cos q ) = 0 9. cos q + 3 sin q = 2 cos 2q
fi (sin (q ) + cos (q )) = 0 & (1 + 2 cos q ) = 0 10. 3 (cos q - 3 sin q ) = 4 sin 2q .cos 3q
Ê Êp ˆˆ 1
fi ÁË sin ÁË 4 + q ˜¯ ˜¯ = 0 & cos q = - 2 3.7 PRINCIPAL VALUE
Êp ˆ Ê 2p ˆ The numerically least angle is called the principal value.
fi ÁË + q ˜¯ = np & q = 2np ± ÁË ˜¯
4 3 1
For example, sinq =
2
p Ê 2p ˆ Then,
fi q = np - & q = 2np ± Á ˜ , n Œ I
4 Ë 3¯ p 5p 13p 17p 11p 7p
q= , , , ,- ,-
6 6 6 6 6 6
Ex-4. Solve for q : sin 3 q + sin q cos q + cos3 q = 1 p
Among all these values of q, is the numerically smallest.
Soln. We have, sin 3 q + sin q cos q + cos3 q = 1 6
p
(sin q + cos q ) + sin q cosq = 1
1

3 3 So principal value of sinq = is q =
2 6
fi (sin q + cos q ) (1 - sin q cos q ) + sin q cos q = 1 3.8 METHOD TO FIND OUT THE
fi (sin q + cos q ) (1 - sin q cos q ) = (1 - sin q cos q ) PRINCIPAL VALUE
fi (sin q + cos q - 1) (1 - sin q cos q ) = 0 (i) First draw a trigonometric circle and mark the quadrant
fi (sin q + cos q - 1) = 0 & (1 - sin q cos q ) = 0 in which the angle may lie.
1 (ii) Select anti-clockwise direction for 1st and 2nd quad-
fi (sin q + cos q ) = 1 & sin (2q ) = rant and select clockwise direction for 3rd and 4th
2
quadrants.
Ê Ê p ˆˆ Êpˆ
fi ÁË sin ÁË q + 4 ˜¯ ˜¯ = sin ÁË 2 ˜¯ (iv) Select the numerically least angle among these two val-
ues. The angle thus formed will be the principal value.
Êpˆ
& sin ( 2q ) = sin Á ˜ (v) In case, two angles, one with +ve sign and the other
Ë 6¯ with -ve sign, qualify for the numerically least angle,
then it is the conventional of mathematics, to consider
n Êpˆ Êpˆ
fi q = np + (-1) Á ˜ - Á ˜ the angle with +ve signs as a principal value.
Ë 2¯ Ë 4¯
124 Comprehensive Trigonometry with Challenging Problems & Solutions for Jee Main and Advanced

1 Rule: (i) Find the common value of q between 0 and


Ex-1. Find the principal value of sin (q ) = - 2p
2
(ii) Add 2 np to this common value
1
Soln. We have, sin (q ) = - (iii) then we shall get the general value of the given
2 two equations.
p 1 1
fi q =- Ex-1. If sin (q ) = and cos (q ) = -
6 2 2
Ê pˆ the general values of q.
Hence, the principal value of q is ÁË - ˜¯ 1
Soln. Now, sin (q ) =
6
1 2
Ex-2. Find the principal value of sin (q ) = p 3p
2 fi q= ,
4 4
1
Soln. We have, sin (q ) = 1
2 and cos (q ) = -
2
p 3p
fi q= , 3p
4 4 fi q=
p 4
Hence, the principal value of q is
4 3p
Thus, the commoon value of q is
Ex-3. Find the principal value of tan (q ) = - 3
4
Ê 3p ˆ
Hence, the general value of q is Á 2 np + ˜
Soln. We have, tan (q ) = - 3 Ë 4¯
p 1
fi (q ) = - Ex-2. If sin (q ) = and tan (q ) = -1 , then find the
3 2
p general values of q.
Hence, the principal value of q is - 1
3 Soln. We have, sin (q ) =
2
p 3p
EXERCISE 3 fi q= ,
4 4
Find the principal values of Also, tan (q ) = -1
1. tanq = -1 3p 7p
fi q= ,
1 4 4
2. cosq = 3p
2 Thus, the common value of q is
4
1 3p ˆ
3. cosq = - Ê
2 Hence, the general values of q is Á 2 np + ˜ ,
Ë 4¯
4. tanq = - 3 where n ΠI .
5. secq = 2
Ex-3. If (1 + tan A) (1 + tan B ) = 2
of A + B.
3.9 SOLUTIONS IN CASE OF TWO
Soln. We have, (1 + tan A) (1 + tan B ) = 2
EQUATIONS ARE GIVEN:
fi 1 + tan A + tan B + tan A.tan B = 2
of q which may satisfy both the given equations like fi tan A + tan B = 1 - tan A.tan B
cos q = cos a , sin q = sin a and tan q = tan a .
Ê tan A + tan B ˆ
The common solution is q = 2np + a , where n Œ Z fi ÁË ˜ =1
1 - tan A.tan B ¯
Similarly, sin q = sin a , tan q = tan a .
The common solution is q = 2np + a , n ΠZ fi tan ( A + B ) = 1
The Trigonometric Equation 125

Êpˆ Dividing (i) and (ii), we get,


fi tan ( A + B ) = tan Á ˜
Ë 4¯ sin A sin B
=
cos A cos B

fi ( A + B ) = np + ÊÁË ˜ , where n Œ I . fi tan A = tan B

fi A = np + B, where n ΠI
Ex-4. If sin (p cos q ) = cos (p sin q ) , then prove that,
Ex-7. If A and B are acute +ve angles satisfying the
Ê pˆ 1
cos Á q ± ˜ = equations 3 sin 2 A + 2 sin 2 B = 1 and
Ë 4¯ 2 2 3 sin 2 A - 2 sin 2 B = 0 A + 2B
Soln. We have, sin (p cos q ) = cos (p sin q ) Soln. Given equations are
3 sin 2 A + 2 sin 2 B = 1 .................(i)
Êp ˆ
fi sin (p cos q ) = sin Á - p sin q ˜ and 3 sin 2 A – 2 sin 2 B = 0
Ë2 ¯ ...............(ii)

p From (ii), we get,


fi (p cos q ) = ÊÁË ˆ
- p sin q ˜
¯ 3 sin 2 A = 2 sin 2 B
2
Ê1 ˆ sin 2 A sin 2 B
fi cos q = Á - sin q ˜ fi =
Ë2 ¯ 2 3
sin 2 B 3
1 fi =
fi cos q + sin q = sin 2 A 2
2
1 1 1 From (i), we get,
fi cos q + sin q =
2
Ê pˆ 1
2 2 2 3
2
( ) (
2 sin 2 A + 2 sin 2 B = 1)
fi cos Á q + ˜ =
Ë 4¯ 2 2 3
fi (1 - cos 2 A) + (1 - cos 2 B ) = 1
Similarly, we can prove that, 2
3 3
Ê pˆ 1 fi cos 2 A + cos 2 B =
cos Á q - ˜ = 2 2
Ë 4 ¯ 2 2
sin 2 B sin 2 B
fi cos 2 A + cos 2 B =
Ex-5. If tan (p cos q ) = cot (p sin q ) , sin 2 A sin 2 A
p fi sin 2 B cos 2 A + sin 2 A cos 2 B = sin 2 B
then prove that cos ÊÁ q - ˆ˜ =
1
Ë 4¯ 2 2 fi sin (2 A + 2 B ) = sin 2 B
Soln. We have, tan (p cos q ) = cot (p sin q ) fi sin (2 A + 2 B ) = sin (p - 2 B )


Êp ˆ
tan (p cos q ) = tan Á - p sin q ˜
fi (2 A + 2 B ) = (p - 2 B )
Ë2 ¯ fi (2 A + 4 B ) = p
p
fi (p cos q ) = ÊÁË ˆ
- p sin q ˜
¯ fi ( A + 2 B) =
p
2 2
1 p
fi cos (q ) + sin (q ) = Ex-8. Solve: x + y = and tan x + tan y = 1
2 4
1 1 1 p
fi cos (q ) + sin (q ) = Soln. Given x + y = and tan x + tan y = 1
2 2 2 2 4
Ê pˆ 1 Êpˆ
fi cos Á q - ˜ = fi tan ( x + y ) = tan Á ˜
Ë 4¯ 2 2 Ë 4¯
Ex-6. If sin A = sin B and cos A = cos B tan x + tan y
fi =1
of A in terms of B. 1 - tan x.tan y
Soln. Given sin A = sin B ...............(i) fi 1 - tan x .tan y = 1
and cos A = cos B ..............(ii)
126 Comprehensive Trigonometry with Challenging Problems & Solutions for Jee Main and Advanced

fi tan x .tan y = 0 fi (2 sin q + 3) (2 sin q - 1) = 0


fi tan x = 0 & tan y = 0 3 1
fi x = np = y fi sin q = - ,
2 2
Thus, no values of x and y satisfying the given 1
equations. fi sinq =
2
Therefore, the given equations have no solutions.
p 5p
Ex-9. Solve: sin x + sin y = 1, cos 2 x – cos 2 y = 1. fi q= ,
Soln. Given sin x + sin y = 1 ...........(i) 6 6
and cos 2 x – cos 2 y = 1 ..........(ii) Ex-13. Find the set of values of x for which
From (ii), we get, cos 2 x – cos 2 y = 1
tan 3x - tan 2 x
=1
fi 1 - 2 sin 2 x - 1 + sin 2 y = 1 1 + tan 3x.tan 2 x
fi sin 2 x - sin 2 y = -
1 tan 3x - tan 2 x
Soln. We have, =1
2 1 + tan 3x.tan 2 x

n Ê 3ˆ
y = n p + (-1) sin -1 Á ˜ , fi tan (3x - 2 x ) = 1
Ë 4¯
fi tan x = 1
where n ΠI
p
1 fi x = np + , where n ΠI
fi sin x - sin y = - .............(iii) 4
2
Adding (i) and (iii), we get, But the values of x do not satisfy the given equation.
Hence, the set of values of x is f.
1
2 sin x = Ex-14. Find the number of solutions of the equation
2 tan x + sec x = 2 cos x lying in the interval
1
fi sin x =
4
[0, 2 p ]
n Ê 1ˆ Soln. Given equation is tan x + sec x = 2 cos x
fi x = n p + (-1) sin -1 Á ˜ , n Œ I ,
Ë 4¯ fi (1 + sin x ) = 2 cos2 x
Subtracting (i) and (iii), we get,
fi (1 + sin x ) = 2 (1 - sin 2 x )
3
2 sin y = fi (1 + sin x ) = 2 (1 + sin x ).(1 - sin x )
2
fi sin y =
3 fi (1 + sin x ) (1 - 2 + 2 sin x ) = 0
4
fi (1 + sin x ) (2 sin x - 1) = 0
n 3ˆ-1 Ê
fi y = np + (-1) sin Á ˜ , n Œ I . fi (1 + sin x ) = 0 & (2 sin x - 1) = 0
Ë 4¯
1
Ex-10. If r sinq = 3 and r = 4 (1 + sin q ) , where 0 £ q £ 2p , fi sin x = -1 & sin x =
q. 2
Soln. Given equations are p p 5p
fi x= , ,
r sinq = 3 ...........(i) 2 6 6
p
and r = 4 (1 + sin q ) ..........(ii) But x = doest not satisfy the given equation.
2
Eliminating (i) and (ii), we get, p 5p
Thus, the values of x are and .
4 (1 + sin q ) sin q = 3 6 6
Hence, the number of solutions is 2.
fi 4 sin 2 q + 4 sin q - 3 = 0
Ex-15. Find the number of values of x in the interval [0, 3p ]
fi 4 sin 2 q + 6 sin q - 2 sin q - 3 = 0 satisfying the equation.
fi 2 sin q ( 2 sin q + 3) - 1( 2 sin q + 3) = 0 2 sin 2 x + 5 sin x - 3 = 0
The Trigonometric Equation 127

Soln. Given equation is 2 sin 2 x + 5 sin x - 3 = 0 fi (4 x + 20∞) = 260∞


fi 4 x = 260∞ - 20∞ = 240∞
fi 2 sin 2 x + 6 sin x - sin x - 3 = 0
fi x = 60∞
fi 2 sin x (sin x + 3) - 1(sin x + 3) = 0 Hence, the smallest positive value of x is 60∞ .
fi (sin x + 3) (2 sin x - 1) = 0
1 EXERCISE 4
fi sin x = -3,
2
1 1
fi sin x = 1. If cosq = and tanq = -1
2 2
p 5p 13p 17p value of q.
fi x= , , , . 2. Find the most general value of q which satisfy the
6 6 6 6
1 1
Hence, the number of values of x is 4 . equations sinq = and tanq = .
2 3
Ex-16. Find the smallest positive value of x such that
3. If A and B are acute +ve angles satisfying the equations
tan( x + 20∞) = tan( x - 10∞).tan x.tan( x + 10∞) .
3 sin 2 A + 2 sin 2 B = 1 and 3 sin 2 A - 2 sin 2 B = 0, then
Soln. Given, A + 2B.
tan( x + 20∞) = tan( x - 10∞).tan x.tan( x + 10∞)
2
4. If tan (A – B) = 1 and sec ( A + B ) =
tan( x + 20∞)
fi = tan( x - 10∞).tan( x + 10∞) 3
tan x smallest +ve values of A and B and their most genral
sin ( x + 20∞) cos x sin( x - 10∞)sin( x + 10∞) values.
fi =
cos( x + 20∞)sin x cos( x - 10∞∞).cos( x + 10∞) 2p
5. Solve: x + y = and sin x = 2 sin y.
sin ( x + 20∞) cos x + cos( x + 20∞)sin x 3

sin ( x + 20∞) cos x - cos( x + 20∞)sin x 2p 3
6. Solve: x + y = and cos x + cos y = .
3 2
sin ( x - 10∞)sin ( x + 10∞) + cos ( x - 10∞).cos ( x + 10∞)
= p
sin ( x - 10∞)sin ( x + 10∞) - cos ( x - 10∞).cos ( x + 10∞) 7. Solve: x + y = and tan x + tan y = 1.
4
sin ( x + 20∞ + x ) cos( x + 10∞ - x + 10∞) 8. Solve: r sinq = 3 and r = 4 (1 + sin q ), 0 £ q £ 2p
fi sin x + 20∞ - x = - cos( x + 10∞ + x - 10∞)
( ) 9. If sin A = sin B and cos A = cos B ,
sin(2 x + 20∞) cos (20∞) A in terms of B.
fi =- 10. Solve: sin x + sin y = 1
sin(20∞) cos (2 x)
cos 2 x - cos 2 y = 1 .
fi sin (2 x + 20∞) cos (2 x) = - sin (20∞) cos (20∞)
11. Find the co-ordinates of the point of inter section of
fi 2 sin ( 2 x + 20∞) cos (2 x) = - 2 sin (20∞) cos (20∞) the curves
y = cos x & y = sin 2 x
fi sin ( 4 x + 20∞) + sin(20∞) = - sin (40∞)
12. Find all points of x, y that satisfying the equations
fi sin ( 4 x + 20∞) = - sin (40∞) - sin(20∞)
3
cos x + cos y + cos ( x + y ) = - .
fi sin (4 x + 20∞) = - 2 sin(30∞) cos (10∞) 2
fi sin ( 4 x + 20∞) = - cos (10∞) 13. If 0 < q , j < p and 8 cos q cos j cos(q + j ) + 1 = 0 ,
fi sin (4 x + 20∞) = - sin (80∞) q &j .
fi sin ( 4 x + 20∞) = sin (-80∞) 1 1
14. Solve: 4sin x + 3 cos y = 11 , 5.16sin x + 2.3cos y = 2 .
fi sin ( 4 x + 20∞) = sin (p - (-80∞)) p p
if - £ x £
fi (4 x + 20∞) = (p - (-80∞)) 2 2
128 Comprehensive Trigonometry with Challenging Problems & Solutions for Jee Main and Advanced

3.10 SOME IMPORTANT REMARKS TO fi (5 cos x + 3) = 0 , (2 cos x - 1) = 0


KEEP IN MIND WHILE SOLVING A 3
fi cos x = - = cos a ,
TRIGONOMETRIC EQUATION 5
1. Squaring the equation at any stage should be avoided 1 Êpˆ
as far as possible. If squaring is necessary, check the cos x = = cos Á ˜
2 Ë 3¯
solutions for extraneous roots.
2. Never cancel terms containing unknown terms on both Êpˆ
fi x = 2np ± a = 2np ± cos -1 Á ˜ ,
the sides, which are in product form. It may cause roots Ë 3¯
loose.
p
3. Domain should not be changed. If it is changed, nec- x = 2np ± , n ŒZ .
essary correction must be made. 3
4. Check the denominator is non zero at any stage, while
solving equations.
5. The answer should not contain such values of angles, EXERCISE 5
1. 4 sin 4 x + cos 4 x = 1
those in the package in their notations.This leads to 2. 4 cos 2 x sin x - 2 sin 2 x = 2 sin x
the different methods of solving the same problem.
Whenever we come across such situation, we must check 3. sin x – cos x – 4 cos2 x sin x = 4 sin2 x
authenticity. This will ensure that our answer is correct. 4. 2 cos 2 x + 2 sin x = 2
7. Sometimes the two soltuion set consists partly of
common values. In all such cases the common part 3
5. 1 + sin 2 x + cos 2 x = sin 2 x
must be presented only once. 2
7
3.11 TYPES OF TRIGONOMETRIC 6. sin 6 x + cos6 x =
16
EQUATIONS 17
7. sin 8 x + cos8 x = cos 2 2 x
16
TYPE 1
A trigonometric equation reduces to Quadratic/Higher 8. 2 sin 3 x = cos 2 3x
degree Equations.
9. cos 4 x = cos 2 3x
Rules: 1. Transform the terms to be a only one trigono-
metric ratio involving angles in same form. 10. cos 2 x = 6 tan 2 x - 2 cos 2 x
2. Factorize the equation and express it in f(x) ¥
g(x) = 0 TYPE 2.
A trigonometric equation is solved by factorization
fi f ( x) = 0 or g(x) = 0.
method.
3. Solve both the equations one by one to get the Rule: Simply reduces to a single trigonometric ratio of the
general value of the variables. unknown angles and factorize by basic algebraic
method.
Ê xˆ
Ex-1. Solve: 5 cos 2 x + 2 cos 2 Á ˜ + 1 = 0 . Ex.-1 Solve: (2 sin x – cos x) (1 + cos x) = sin2 x.
Ë 2¯
Soln. The given equation can be expressed as Soln. The given equation can be written as
(2 sin x – cos x) (1 + cos x)
( )
5 2 cos 2 x - 1 + (1 + cos x ) + 1 = 0
= (1 – cos x) (1 + cos x)
fi 10 cos x + cos x - 3 = 0
2 fi (1 + cos x)(2 sin x – cos x – 1 + cos x)=0

fi (5 cos x + 3)(2 cos x - 1) = 0 fi (1 + cos x) (2 sin x – 1) = 0.


The Trigonometric Equation 129

fi cos x = –1, sin x = 1/2 p


fi x = 0,
1 Êpˆ 3
fi cos x = -1 = cos p , sin x = = sin Á ˜ .
2 Ë 6¯
EXERCISE 7
p n
fi x = 2np ± p , x = np + (-1) , n Œ Z .
6 Q. Solve for x
1. cos x – cos 2x = sin 3x
p
EXERCISE 6 2. sin 7x + sin 4x + sin x = 0, 0 £ x £
2
3. cos 3x + cos 2x
1. 2 sin 2 x + sin x - 1 = 0, where 0 £ x £ 2p
Ê 3x ˆ Ê xˆ
= sin ÁË ˜¯ + sin ÁË ˜¯ , 0 £ x £ 2p
2. 5 sin 2 x + 7 sin x - 6 = 0 ,where 0 £ x £ 2p 2 2

1 4. sin x + sin 2 x + sin 3x


3. sin 2 x - cos x = , where 0 £ x £ 2p
4 = cos x + cos 2 x + cos 3x , -p £ x £ p
5. cos 2x + cos 4x = 2 cos x
4. tan 2 x - 2 tan x - 3 = 0
6. sin 2 x + cos 2 x + sin x + cos x + 1 = 0
5. 2 cos 2 x - 3 sin x + 1 = 0
7. tan x + tan 2 x + tan 3x = 0

TYPE 3. 8. tan 3x + tan x = 2 tan 2 x


A trigonometric equation is solved by transformation as a 9. (1 - tan x ) (1 + sin 2 x ) = (1 + tan 4 x )
sum or difference into a product.
10. sin x - 3 sin 2 x + sin 3x = cos x - 3 cos 2 x + cos 3x
Rules:
1. The given equation is reducible to TYPE 4.
f ( x) ¥ g ( x) = 0 A trigonometric equation is solved by transformation as a
product into a sum or difference:
fi f ( x) = 0, g ( x) = 0
Rule:
2. Solve both the equations one by one to get the general
1. The given equation can be reduced to f ( x) = 0, g ( x) = 0
value of the variable x.
2. Solve both the equation one-by-one to get the genral
Ex.-1 Solve: sin x + sin 3 x + sin 5 x = 0,
value of the variable x.
p Ex.-1. Solve: 4 sin x . sin 2 x . sin 4x = sin 3 x
0£ x£ .
2 Soln. The given equation can be written as
Soln. The given equation can be written as (2 sin 2x . sinx ) . 2 sin 4x – sin 3x = 0
(sinx + sin5x) + sin 3x = 0. fi 2 (cos x – cos 3x ) sin 4x – sin 3x = 0

fi 2 sin 3x . cos 2x + sin 3 x = 0 fi 2 sin 4x cos x – 2sin 4x cos 3x – sin 3x = 0


fi (sin 5x + sin 3x) – (sin 7x + sin x ) – sin 3x = 0
fi sin 3x (2 cos 2x + 1 ) = 0
fi (sin 7x – sin 5x ) + sin x = 0
fi sin 3x = 0, cos 2x = –1/2 fi sin x (2 cos 6x + 1 ) = 0
fi sin x = 0, cos 6x = –1/2,
Ê 2p ˆ
fi sin 3x = 0, cos 2 x = cos ÁË ˜¯ Ê 2p ˆ
3 fi sin x = 0, cos 6 x = cos Á ˜
Ë 3¯
2p
fi 3x = np , 2 x = 2np ± ,nŒZ 2p
3 fi x = np , 6 x = 2np ± , n ŒZ
np p 3
fi x= , x = np ± , n Œ Z
p
3 3 fi x = np , x = (3n ± 1) , n Œ Z
9
130 Comprehensive Trigonometry with Challenging Problems & Solutions for Jee Main and Advanced

EXERCISE 8 m, n, p, q ΠN, can be reduced to a rational function with


respect to sin x, cos x, tan x and cot x.
1. cos x . cos 2x . cos 3x = 1/4, 0 £ x £ p Rule: 1. Use half angle formulae of tangents:
2. sin 3a = 4 sin a .sin ( x + a ) .sin ( x - a ) Ê xˆ
2 tan Á ˜
3. sin 2x . sin 4x . + cos 2x = cos 6x Ë 2¯
sin x = ,
4. secx . cos 5x + 1 = 0., 0 £ x £ p 2 Ê xˆ
5. cos x . cos 6x = –1 1 + tan Á ˜
Ë 2¯
TYPE 5 Ê xˆ
A trigonometric equation is of the form 1 - tan 2 Á ˜
Ë 2¯
b0 sin n x + b1 sin n -1 x.cos x + cos x =
Ê xˆ
1 + tan 2 Á ˜
b2 sin n - 2 x.cos 2 x + ....... + bn cos n x = 0 Ë 2¯

where, b0 , b1 , b2 ,..........., bn Œ R is a homogenous equation Ê xˆ


2 tan Á ˜
of sin x and cos x, where cos x is nonzero. Ë 2¯
tan x = ,
2 Ê xˆ
Rule: 1. Divide both the sides by highest power of cos x. 1 - tan Á ˜
Ë 2¯
2. The given equation can be reduced to
b0 tan n x + b1 tan n -1 x + ......... + bn = 0 Ê xˆ
1 - tan 2 Á ˜
3. and then use the factorization method. Ë 2¯
cot x =
Ê xˆ
2 tan Á ˜
Ex.-1 Solve: 2 sin 2 x - 5 sin x cos x - 8 cos 2 x = -2 Ë 2¯
Soln. The given equation can be written as
Ê xˆ
2 sin 2 x - 5 sin x cos x - 8 cos 2 x 2. Substitute tan Á ˜ = t , and then solve it.
Ë 2¯
(
= -2 sin 2 x + cos 2 x ) Ê xˆ
sec2 Á ˜
fi 2 tan x - 5 tan x - 8 = -2(tan x + 1)
2 2 Ë 2¯
Ex.-1 Solve: 1 + 2cosecx = -
2
fi 4 tan 2 x - 5 tan x - 6 = 0 Soln. The given equation can be written as
fi ( tan x - 2) (4 tan x + 3) = 0 2 1Ê Ê xˆˆ
1+ = - Á1 + tan 2 Á ˜ ˜
3 sin x 2Ë Ë 2¯ ¯
fi tan x = -2, tan x = -
4 Ê Ê xˆˆ
x = np + a , x = np + b , fi 2(sin x + 2) = - Á1 + tan 2 Á ˜ ˜ sin x
fi Ë Ë 2¯ ¯
Ê 3ˆ Ê x ˆ
where a = tan -1 ( 2) , b = tan -1 Á - ˜ , n Œ Z . 2 tan
Ë 4¯ Á 2 ˜
fi 2Á + 2˜
x
Á 1 + tan 2 ˜
Ë 2 ¯
EXERCISE 9 x ˆ
Ê
2 tan
Q. Solve for x: Ê xˆ Á 2 ˜
= - Á1 + tan 2 ˜¯ . Á ˜
Ë 2 x
1. 5 sin 2 x - 7 sin x cos x + 6 cos 2 x = 4 Á 1 + tan 2 ˜
Ë 2¯
2. 2 sin 2 x - 5 sin x cos x - 8 cos 2 x = -3
Ê 2t Ê 2t ˆ
3. sin 3x.cos x + sin 2 x cos 2 x, 0 £ x £ 2p fi 2Á
Ë1+ t2
ˆ
¯ (
+ 2˜ = - 1 + t 2 ¥ Á )
Ë 1 + t 2 ˜¯
,
Type 6
A Trigonometric equation is of the form R (sin m x, cos n where t = tan (x/2)
x, tan p x, cot q x ) = 0, where R is a rational function and fi t 3 + 2t 2 + 3t + 2 = 0
The Trigonometric Equation 131

fi t 3 + t 2 + t 2 + t + 2t + 2 = 0 Ê t 2 - 1ˆ
t - 2 2Á ˜ =0
fi (t + 1)(t 2 + t + 2) = 0 Ë 2 ¯

fi t + 1 =0, t 2 + t + 2 π 0 fi 2t 2 - t - 2 = 0


Ê xˆ Ê -p ˆ
tan Á ˜ = -1 = tan Á ˜ fi ( )(
2t + 1 t - 2 = 0 )
Ë 2¯ Ë 4 ¯
1
fi t = 2, -
fi x p 2
= np - , n ΠZ
2 4
When sin x + cos x = 2
fi p
x = 2np - , n ΠZ . 1 1
2 fi sin x + cos x = 1
2 2
Ê pˆ Êpˆ
EXERCISE 10 fi sin Á x + ˜ = 1 = sin Á ˜
Ë 4¯ Ë 2¯
1. Solve: (cos x - sin x ) ( 2 tan x + sec x ) + 2 = 0 fi np p
x+ = np + (-1) n , n ΠZ
x x 4 2
sin 3 - cos3
2. Solve: 2 2 = cos x When sin x + cos x = -
1
2 + sin x 3 2
Ê xˆ Ê xˆ 1 1 1
3. Solve: cot Á ˜ - cosec Á ˜ = cot x fi sin x + cos x = -
Ë 2¯ Ë 2¯ 2 2 2
4. If q1 , q 2 , q3 , q 4 be the four roots of the equation Ê pˆ Ê pˆ
fi sin Á x + ˜ = sin Á - ˜
sin (q + a ) = k sin 2q , no two of which differ by a Ë 4¯ Ë 6¯
multiple of 2p, then Ê pˆ nÊ pˆ
fi ÁË x + ˜¯ = np + (-1) ÁË - ˜¯
prove that q1 + q 2 + q 3 + q 4 = ( 2n + 1) p , n ΠZ . 4 6
4
p p
TYPE 7 fi x = np - ( -1) n - , n ΠZ.
6 4
A trigonometric equation is of the form R(sinx + cosx, sinx
. cos x) = 0, where R is a rational function of the argument
of sinx and cosx.
Rule: 1. Put sin x + cos x = t
EXERCISE 11
2. Use the identity
Q. Solve for x:
(sin x + cos x )2 = 1 + 2 sin x cos x 1. sin 3 x + sin x cos x + cos3 x = 1
t2 -1 2. sin x + cos x = 1 – sin x cos x
fi sin x cos x =
2
3. 1 + sin 3 x + cos3 x = 3 sin 2 x
Ê t 2 - 1ˆ 2
3. So, the given equation reduces to R Á t ,
Ë 2 ˜¯ 4. sin 2x – 12 (sin x – cos x) + 12 = 0, 0 £ x £ 2p
and then solve it.
TYPE 8
Ex.-1 Solve: sin x + cos x – 2 2 sin x cos x = 0. A trigonometrical equation is based on extreme values of
Soln. Let sin x + cos x = t sin x and cos x.
Rule: 1. Whenever terms are sin, cos in power 1 and
t2 -1 all terms connected with plus sign and number
fi sin x . cos x =
2 of terms in L.H.S (with +ve or –ve sign ) then
So, the given equation can be reduced to each term must have in the extreme value.
132 Comprehensive Trigonometry with Challenging Problems & Solutions for Jee Main and Advanced

2. In such problems, each term will be +1 when 3 5


81+ cos x + cos x + cos x + cos x + cos x +..........to• = 64
2 4

the value of R.H.S is +ve and each term will be


(–1) when the value of R.H.S is –ve. Soln. The given equation can be written as
3 5
81+ cos x + cos x + cos x + cos 4 x + cos x +..........to•
2
Ex.-1 Solve: sin 6x + cos 4x + 2 = 0. = 82
Soln. The given equation can be written as 3
fi 1 + cos x + cos x + cos x + cos x + ..to• = 2
2 4
sin 6x + cos 4x = –2
1
fi sin 6x = –1 and cos 4x = –1 fi =2
1 - cos x
3p
fi sin 6 x = sin , cos 4x = cos p 1
2 fi cos x = ±
2
fi 6 x = 2np + 3p , 4 x = 2np + p , n ΠZ 1
2 When cos x =
2
fi np p np p
x= + ,x = + , n ŒZ Êpˆ
3 4 2 4 fi cos x = cos Á ˜
Ë 3¯
fi p 5p 7p 11p
x= , , , ,...
4 4 12 12 fi p
x = 2np ±
, n ŒZ
p 3p 5p 7p 3
...x = , , , ,.....
4 4 4 4 1 Ê 2p ˆ
When cos x = - = cos Á ˜
fi p 5p 2 Ë 3¯
x= ,
4 4
fi 2p
Hence, the general solution will be, x = 2np ± , n ŒZ
3
fi x = 2np +
p
, 2np +
5p
, n ŒZ Hence, the values of x are ± p , ± 2p .
4 4 3 3
p p
fi x = 2np + , ( 2n + 1) p + , n ΠZ EXERCISE 13
4 4
fi p Q. Solve for x:
x = mp + , m ΠZ
4 3 5
1. 21+ cos x + cos x + cos x + cos 4 x + cos x +........to •
2
=4
EXERCISE 12 2. 1 + sin q + sin q + sin q + ...........to •
2 3

= 4+2 3
Q. Solve for x: 3 1
sin 2 x - sin x +
1. sin 6 x = 1 + cos 4 3x 3. cos x 2 2 =1
2. sin x = 1 + tan x
4 8
4. e sin x
-e - sin x
-4=0
3. sin 2 x + cos 2 y = 2 sec2 z [sin 2 x + sin 4 x +sin 6 x +................to• ]log e 2
5. If e
4. sin 3x + cos 2x + 2 = 0
5. cos 4x + sin 5x = 2. equations x 2 - 9 x + 8 = 0
cos x p
TYPE 9 , 0< x<
A trigonometrical equation involving with exponential, cos x + sin x 2
logarithmic and modulii terms: 6. log cos x tan x + logsin x cot x = 0
Rule: Whenever equation contains power term, then we 7. 3sin 2 x + 2 cos
2
x
+ 31-sin 2 x + 2 sin
2
x
= 28
should use the following method.
1. Equate the base if possible 8. log cos x sin x + logsin x cos x = 2 , where x > 0
2. If it is not possible to equate the base, take log of both TYPE 10
the sides and make its R.H.S is zero, then we proceed A trigonometrical equation involving the terms of two sides
further. are of different nature:
Ex.-1 Find the values of x in ( -p , p ) which satisfy the Rule: 1. Let y = each side of the equation and break the
equation. equation in two parts.
The Trigonometric Equation 133

2. Find the inequality for y taking L.H.S of the Ê pˆ 1


equation and also for the R.H.S of the equation. fi sin Á x - ˜ =
Ë 4¯ 2 2
3. Use the A.M ≥ G.M. on the right hand side of
the equation. Ê pˆ n -1 Ê 1 ˆ
4. If there is any value of y satisfying both the
fi ÁË x - ˜¯ = np + (-1) .sin ÁË ˜
4 2 2¯
inequalities, then the equation will have real
Ê np n Ê 1 ˆˆ
solution, otherwise no solution. fi x=Á + np + (-1) .sin -1 Á , n ŒI
Ë 4 Ë 2 2 ˜¯ ˜¯
EXERCISE 14 when t = -2
fi sin x - cos x = 2
Q. Solve for x:
It is impossible, since the maximum value of
Ê xˆ 1 p
1. 2 cos 2 Á ˜ sin 2 x = x 2 + 2 , 0 < x <
Ë 2¯ x 2 (sin x - cos x ) is 2
Ê x2 + x ˆ Ex-2. Solve for x:
x -x
2. 2 cos 2 Á ˜ =2 +2 sin 2 x + 12 = 12 (sin x - cos x )
Ë 6 ¯
Soln. The given equation is
PROBLEMS FOR JEE ADVANCED EXAM sin 2 x + 12 = 12 (sin x - cos x )
4
Ex-1. Solve for x: sec x - cosecx = Put (sin x - cos x ) = t
3
Soln. The given equation is fi 1 - 2 sin x cos x = t 2

sec x - cosecx =
4
fi ( )
2 sin x cos x = 1 - t 2
3
(i) reduces to (1 - t ) + 12 = 12t
2
1 1 4
fi - =
cos x sin x 3 fi (1 - t ) = 12 (t - 1)
2

fi 3 (sin x - cos x ) = 4 sin x cos x ....(i)


fi (t + 1) (t - 1) = -12 (t - 1)
Put (sin x - cos x ) = t
fi (t - 1) (t + 1 + 12) = 0
fi 1 - 2 sin x cos x = t 2 fi (t - 1) (t + 13) = 0
1- t 2
fi t = 1, - 13
fi sin x cos x =
2 when t = 1, then sin x - cos x = 1
Ê1- t 2ˆ
(i) reduces to 3t = 4 Á Ê 1 1 ˆ 1
˜ fi ÁË sin x - cos x˜ =
Ë 2 ¯ 2 2 ¯ 2
fi (
3t = 2 1 - t 2 ) fi
Ê pˆ Êpˆ
sin Á x - ˜ = sin Á ˜
Ë 4 ¯ Ë 4¯
fi 2t 2 + 3t - 2 = 0
Ê pˆ n Êpˆ
fi 2t 2 + 4t - t - 2 = 0 fi ÁË x - ˜¯ = np + (-1) ÁË ˜¯
4 4
fi 2t (t + 2) - (t + 2) = 0
p n Êpˆ
fi (t + 2) (2t - 1) = 0 fi x = np + + (-1) Á ˜ , n Œ I
4 Ë 4¯
1
fi t = ,- 2 when t = -13
2
sin x - cos x = -13
1
when t = It is impossible, since the maximum value of
2
1 (sin x - cos x ) is 2 .
fi sin x - cos x =
2
Ex-3. Solve for x: sec x + tan x = sec x + tan x

Ê 1
sin x -
1 ˆ
cos x˜ =
1 in [0, 2p ] .
ÁË ¯ 2 2
2 2
134 Comprehensive Trigonometry with Challenging Problems & Solutions for Jee Main and Advanced

Soln. The given equation is Ê a - 4ˆ


fi -1 £ Á £1
sec x + tan x = sec x + tan x Ë 2 ˜¯
fi sec x.tan x ≥ 0 fi -2 £ ( a - 4) £ 2
sin x fi 2£a£6
fi ≥0
cos x2
fi a Œ[ 2, 6]
fi sin x ≥ 0, cos x π 0
Ex-6. Solve for x: sin100 x - cos100 x = 1
Ïp ¸ Soln. The given equation is
fi x Œ[0,p ] - Ì ˝
Ó2˛
sin100 x - cos100 x = 1
È pˆ Êp ˘ It is possible only when sin x = 1, cos x = 0
fi x Œ Í0, ˜ » Á , p ˙
Î 2¯ Ë 2 ˚
Hence, the general solution is
È pˆ Êp ˘ p
Hence, the solution set is Í0, ˜¯ » ÁË , p ˙ . x = np + , n ŒI
Î 2 2 ˚
2
Ex-4. Let n be a positive integer such that
Ex-7. Solve for x:
Êpˆ Êpˆ n
n. 29
sin Á ˜ + cos Á ˜ = sin10 x + cos10 x = cos 4 2 x
Ë 2n ¯ Ë 2n ¯ 2 16
Soln. The given equation is Soln. The given equation is
Êpˆ Êpˆ n 29
sin Á ˜ + cos Á ˜ = sin10 x + cos10 x = cos 4 2 x
Ë 2n ¯ Ë 2n ¯ 2 16
5 5
Êpˆ Êpˆ n Ê 1 - cos 2 x ˆ Ê 1 - cos 2 x ˆ 29

1
sin Á ˜ +
1
cos Á ˜ = fi ÁË ˜¯ + ÁË ˜¯ = cos 4 2 x
Ë 2 n ¯ Ë 2 n ¯ 2 2 16
2 2 2 2


Êp p ˆ
cos Á - ˜ =
n fi
2
32
(
1 + 10 cos 2 2 x + 5 cos 4 2 x =
29
16
)
cos 4 2 x
Ë 4 2n ¯ 2 2
n = 6 only.
fi (1 + 10 cos 2
)
2 x + 5 cos 4 2 x = 29 cos 4 2 x

Ex-5. If cos 2 x + a sin x = 2a - 7 possesses a solution fi 24 cos 4 2 x - 10 cos 2 2 x - 1 = 0


a.
Soln. The given equation is cos 2 x + a sin x = 2a - 7
fi (2 cos 2
)( )
2 x - 1 12 cos 2 2 x + 1 = 0

fi 1 - 2 sin 2 x + a sin x = 2a - 7 fi (2 cos 2


2 x - 1) = 0 , since (12 cos 2
)
2x + 1 π 0
1
fi 2 sin 2 x - a sin x + ( 2a - 8) = 0 fi cos 2 2 x =
2
a ± a 2 - 16 (a - 4) fi 2 cos 2 2 x - 1 = 0
fi sin x =
4 fi cos 4 x = 0
a ± a - 16a + 64
2 p
fi sin x = fi 4 x = ( 2n + 1) , n ΠI
4 2
p
a± (a - 8)2 fi x = (2n + 1) , n Œ I
fi sin x = 8
4 p
a ± ( a - 8) Hence, the solution is x = (2n + 1) , n Œ I
fi sin x = 8
4 Ex-8. Solve for x:
3 1
2a - 8 cos x
sin 2 x - sin x +
2 =1
fi sin x = ,2 2
4
Soln. The given equation is
a-4
fi sin x = 3
sin 2 x - sin x +
1
2 cos x 2 2 =1
The Trigonometric Equation 135

Ê 2 3 1ˆ Soln. The given equation is


fi ÁË sin x - sin x + ˜¯ log cos x = 0 Êpˆ
2 2 x 4 - 2 x 2 sin 2 Á ˜ x + 1 = 0
Ë 2¯
Ê 2 3 1ˆ
fi ÁË sin x - sin x + ˜¯ = 0, log cos x = 0 Ê 2 2Êpˆ ˆ
2
4Êpˆ
2 2
ÁË x - sin ÁË 2 ˜¯ x˜¯ + 1 - sin ÁË 2 ˜¯ x = 0
Ê 3 1ˆ
when Á sin 2 x - sin x + ˜ = 0 It is possible only when
Ë 2 2¯
Ê 2 2Êpˆ ˆ 4Êpˆ
fi (2 sin 2
)
x - 3 sin x + 1 = 0 ÁË x - sin ÁË 2 ˜¯ x˜¯ = 0 , 1 - sin ÁË 2 ˜¯ x = 0

fi (2 sin 2
)
x - 2 sin x - sin x + 1 = 0 Êpˆ
when 1 - sin 4 Á ˜ x = 0
Ë 2¯
fi 2 sin x (sin x - 1) - (sin x - 1) = 0
4Êpˆ
fi (2 sin x - 1) (sin x - 1) = 0 fi sin ÁË ˜¯ x = 1
2
fi (2 sin x - 1) = 0 , (sin x - 1) = 0 4Êpˆ 2Êpˆ
1 fi sin ÁË ˜¯ x = sin ÁË ˜¯
fi sin x = ,1 2 2
2
Êpˆ Êpˆ
1 fi ÁË ˜¯ x = np ± ÁË ˜¯
fi sin x = , since cos x = 0 2 2
2
fi x = (2n ± 1) , n Œ I
n Êpˆ
fi x = np + (-1) Á ˜ , n Œ I
Ë 6¯ Ê Êpˆ ˆ
when Á x 2 - sin 2 Á ˜ x˜ = 0
Ë Ë 2¯ ¯
when log cos x = 0
fi log cos x = log 1 Êpˆ
fi x 2 = sin 2 Á ˜ x
Ë 2¯
fi cos x = 1
fi x2 = 1
fi cos x = ±1
fi x = ±1
when cos x = 1
Hence, the number of solutions is 2.
fi x = 2np
Ex-11. If cos 4 x + a cos 2 x + 1 = 0 has atleast one real
when cos x = -1 a.
x = ( 2n + 1) p Soln. The given equation is
Hence, the solution is cos 4 x + a cos 2 x + 1 = 0 .......(i)
n Êpˆ Let cos 2 x = t
x = 2np , ( 2n + 1) p , np + (-1) Á ˜ , n Œ I
Ë 6¯
Then t Œ[0,1]
Ex-9. Find the number of solutions of
(i) reduces to t 2 + at + 1 = 0
( ) (
cos p x - 4 .cos p x = 1 ) since it has at-least one real root in [0,1] , so
Soln. It is possible only when a 2 - 4 ≥ 0 and 1 + a + 1 £ 0
( )
cos p x - 4 = 1. and cos p x = 1 ( ) fi a ≥ 2 , a £ -2
x = 4 and x = 0 fi a ≥ 2 , a £ -2 ; a £ -2
x = 0 does not satisfy the equation simultaneously. fi a £ -2
Hence, the solution is x = 4 fi a Œ ( - •, - 2 ] .
Therefore, the number of solution is 1
Ex-10. Find the number of solution of Ex-12. If the equation tan 4 x - 2 sec 2 x + b 2 = 0 has
Êpˆ b.
x 4 - 2 x 2 sin 2 Á ˜ x + 1 = 0 Soln. The given equation is
Ë 2¯
136 Comprehensive Trigonometry with Challenging Problems & Solutions for Jee Main and Advanced

tan 4 x - 2 sec 2 x + b 2 = 0 fi x = ±1

fi (
tan 4 x - 2 1 + tan 2 x + b 2 = 0 ) Then y = 4 - 1 = 3

fi y = ±3
tan 4 x - 2 tan 2 x + 1 = 3 - b 2
Thus, the possible ordered pairs are
(tan x - 1) = 3 - b
2 2 2
fi (1, 3) , (1, - 3) , (-1, 3) , (-1, - 3)
(3 - b ) = (tan x - 1)
2
≥0 Ex-15 Find the number of values of x in ( -2p , 2p )
2 2

fi (3 - b ) ≥ 0 2 and satisfying log cos x sin x + log sin x cos x = 2 .


Soln. The given equation is
fi b2 £ 3 log cos x sin x + log sin x cos x = 2
fi b£ 3 It is possible only when
sin x = cos x π 1
Ex-13. If a, b Œ[0, 2p ] and the equation
fi tan x = 1
x 2 + 4 + 3 sin ( ax + b ) = 2 x has at-least one
p 3p 5p 7p
(a + b) . fi x=± ,± ,± ,±
Soln. The given equation is 4 4 4 4
Hence, the number of values of x is 8.
x 2 + 4 + 3 sin ( ax + b ) = 2 x
Ex-16 The number of solutions of tan x + sec x = 2 cos x
fi (x 2
)
- 2 x + 1 + 3 + 3 sin (ax + b ) = 0 in [0, 2p ) is
(a) 2 (b) 3
( x - 1) + 3 (1 + sin ( ax + b )) = 0
2
fi (c) 0 (d) 1.
It is possible only when [JEE MAIN - 2002]
( x - 1) = 0, (1 + sin (ax + b)) = 0 Soln. Ans. (a)
The given equation is
fi x = 1, sin (ax + b ) = -1
tan x + sec x = 2 cos x
fi sin ( a + b ) = -1 sin x + 1
fi = 2 cos x
p cos x
fi (a + b) = (4n - 1) , n ŒI
2 fi sin x + 1 = 2 cos 2 x
3p 7p 1 + sin x = 2 (1 - sin x ) (1 + sin x )
fi (a + b) = , . fi
2 2
fi (1 + sin x ) (1 - 2 + 2 sin x ) = 0
Ex-14. Find the number of ordered pairs ( a, b ) satisfying
fi (1 + sin x ) = 0 , (2 sin x - 1) = 0
Ê p x2 ˆ
the equations x + y = 4 and sin Á 2 ˜ =1. 1
Ë ¯ fi sin x = -1,sin x =
Soln. The given equation is 2
x + y =4 3p p 5p
fi x= ;x = ,
2 6 6
fi - 4 £ x, y £ 4
p 5p
x £4, y £4 fi x= ,
fi 6 6
Ê p x2 ˆ Ï p 5p ¸
Hence, the solutions are Ì , ˝ .
Also, sin Á ˜ =1 Ó6 6 ˛
Ë 2 ¯
Note. No questions asked in between 2003 to 2005.
Ê p x2 ˆ p Ex-17. The number of values of x in the interval [0, 3p ]
fi Á 2 ˜ = ( 4n + 1) 2 , n Œ I
Ë ¯ satisfying the equation 2 sin 2 x + 5 sin x - 3 = 0
fi x 2 = ( 4n + 1) (a) 4 (b) 6
(c) 1 (d) 2.
fi x2 = 1 [JEE MAIN - 2006]
The Trigonometric Equation 137

Soln. The given equation is Ex-19. Find the general solution to the following equation
2 sin 2 x + 5 sin x - 3 = 0 2 (sin x - cos 2 x ) - sin 2 x (1 + 2 sin x ) + 2 cos x = 0 .
[Roorkee – 1987]
fi 2 sin 2 x + 6 sin x - sin x - 3 = 0 Soln. The given equation is
fi 2 sin x (sin x + 3) - (sin x + 3) = 0 2 (sin x - cos 2 x ) - sin 2 x (1 + 2 sin x ) + 2 cos x = 0

fi (sin x + 3) (2 sin x - 1) = 0 fi 2 sin x - 2 cos 2 x - 2 sin x cos x


(sin x + 3) = 0 , (2 sin x - 1) = 0 - sin 2 x + 2 cos x = 0

fi 2 sin x (1 - cos x ) + 4 cos3 x
fi (2 sin x - 1) = 0
-4 cos 2 x - 2 cos x + 2 = 0
1
fi sin x = fi 2 sin x (1 - cos x ) + 4 cos 2 x (cos x - 1)
2
-2 (cos x - 1) = 0
p 5p 11p 17p
fi x= , ,
6 6 6
,
6 fi (cos x - 1) (4 cos2 x - 2 - 2 sin x ) = 0
Hence, the number of solutions is 4.
Note. No questions asked in between 2007 to 2015. fi (cos x - 1) (2 sin 2 x + sin x - 1) = 0
Ex-18. Find all the angles q between p and –p that satisfy fi (cos x - 1) (sin x + 1) (2 sin x - 1) = 0
the equation
Êqˆ 1
5 cos ( 2q ) + 2 cos 2 Á ˜ + 1 = 0 . [Roorkee – 1984] fi cos x = 1,sin x = -1,sin x =
Ë 2¯ 2
Soln. The given equation is p n Êpˆ
fi x = 2np , (4n - 1) , np + (-1) Á ˜
Êqˆ 2 Ë 6¯
5 cos ( 2q ) + 2 cos 2 Á ˜ + 1 = 0
Ë 2¯ Ex-20. Solve for x and y;

fi ( )
5 2 cos q - 1 + (1 + cos q ) + 1 = 0
2 x cos3 y + 3x cos y sin 2 y = 14 ,
x sin 3 y + 3x cos 2 y sin y = 13 .
fi 10 cos 2 q + cos q - 3 = 0 [Roorkee – 1988]
Soln. The given equations are
fi 10 cos q + 6 cos q - 5 cos q - 3 = 0
2

x cos3 y + 3x cos y sin 2 y = 14 ,


fi 2 cos q (5 cos q + 3) - 1(5 cos q + 3) = 0
x sin 3 y + 3x cos 2 y sin y = 13
fi (2 cos q - 1) (5 cos q + 3) = 0
Adding and subtracting, we get,
1 3
fi cos q = , cos q = - x (cos y + sin y ) = 27
3
.........(i)
2 5
x (cos y - sin y ) = 1
1 3
When cosq = ........(ii)
2
Dividing (ii) by (i), we get,
p p
Then q = - ,
3 3 (cos y + sin y )3
= 27
When cosq = -
3 (cos y - sin y )3
5 (cos y + sin y )
fi =3
p -1 Ê 3 ˆ
Then q = + cos ÁË - ˜¯
(cos y - sin y )
2 5 1 + tan y
fi =3
p -1 Ê 3 ˆ 1 - tan y
and - - cos ÁË - ˜¯
2 5 1
Hence, the solutions are fi tan y =
2
p p Ê 3ˆ p Ê 3ˆ Ê 1ˆ
q=± , + cos -1 Á - ˜ , - - cos -1 Á - ˜ . fi y = tan -1 Á ˜
3 2 Ë 5 ¯ 2 Ë 5¯ Ë 2¯
Note. No questions asked in between 1985 to 1986.
138 Comprehensive Trigonometry with Challenging Problems & Solutions for Jee Main and Advanced

-1 Ê 1 ˆ
Put the value of y = tan ÁË ˜¯ into (ii), we get fi (sin 2 x - 1)2 = 2a + 3
2
2a + 3 = (sin 2 x - 1) ≥ 0
2
3 fi
Ê 2 1 ˆ
xÁ - ˜ =1 3
Ë 5 5¯ fi a≥-
2
3
Ê 1 ˆ
Also, (sin 2 x - 1) ≥ 0
2
fi xÁ ˜ =1
Ë 5¯
fi (sin 2 x - 1) ≥ 0
fi x=5 5
fi sin 2 x ≥ 1
Hence, the solutions are
fi sin 2 x = 1
-1 Ê 1 ˆ
x = 5 5 and y = tan ÁË ˜¯ n Êpˆ
2 fi 2 x = n p + (-1) Á ˜
Ë 2¯
Ex-21. Solve for x; 4 sin 4 x + cos 4 x = 1 [Roorkee – 1989] np n Êpˆ
fi x= + ( -1) Á ˜ , n Œ I
Soln. The given equation is 4 sin x + cos x = 1 4 4 2 Ë 4¯
Note. No questions asked in 1991.
( )
2
fi 4 sin 4 x + cos 4 x = sin 2 x + cos 2 x
Ex-23. Find the general solution of the equation
fi 4sin 4 x + cos 4 x = sin 4 x + cos 4 x + 2sin 2 x cos 2 x
( )
3 - 1 sin q + ( )
3 + 1 cos q = 2
fi 3 sin x - 2 sin x cos x = 0
4 2 2
[Roorkee – 1992]
Soln. The given equation is
fi 3 sin x - 2 sin x + 2 sin x = 0
4 2 4

fi 5 sin 4 x - 2 sin 2 x = 0
( )
3 - 1 sin q + ( )
3 + 1 cos q = 2
Ê 3 - 1ˆ Ê 3 + 1ˆ
(
fi sin 2 x 5 sin 2 x - 2 = 0 ) Á
Ë 2 2 ¯
˜ sin q + Á
Ë 2 2 ¯
˜ cos q =
1
2
(
fi sin 2 x = 0 , 5 sin 2 x - 2 = 0 ) 1
fi sin q cos(75∞) + cos q sin(75∞) =
2 2
fi sin x = 0 , sin 2 x =
5 Ê 5p ˆ 1
fi sin Á q + ˜ =
Ê 2ˆ Ë 12 ¯ 2
fi x = n p , np ± a , where a = sin -1 Á ˜
Ë 5¯ Ê 5p ˆ 1 Êpˆ
fi sin Á q + ˜ = = sin Á ˜
Ex-22. Find all the values of ‘a’ for which the equation Ë 12 ¯ 2 Ë 4¯
sin 4 x + cos 4 x + sin 2 x + a = 0 is valid. Ê 5p ˆ n Êpˆ
fi ÁË q + ˜¯ = np + (-1) ÁË ˜¯ , n Œ I
12 4
[Roorkee Main – 1990]
Ê n Êpˆ 5p ˆ
Soln. The given equation is fi q = Á np + (-1) Á ˜ - ˜ , n Œ I
Ë Ë 4 ¯ 12 ¯
sin 4 x + cos 4 x + sin 2 x + a = 0
Note. No questions asked in 1993.
fi 1 - 2 sin 2 x cos 2 x + sin 2 x + a = 0
4
Ex-24. Solve for q secq - cosec q =

1
( )
1 - 4 sin 2 x cos 2 x + sin 2 x + a = 0
2
3
[Roorkee Main – 1994]
1
1 - (sin 2 x ) + sin 2 x + a = 0
2 4
fi Soln. The given equation is secq - cosec q =
2 3
2 - (sin 2 x ) + 2 sin 2 x + 2a = 0 3 (sin q - cos q ) = 4 sin q cos q
2
fi ......(i)
fi (sin 2 x )2 - 2 sin 2 x - 2 = 2a Let sin q - cos q = t ’
1- t2
fi (sin 2 x - 1)2 - 3 = 2a Then sin q cos q =
2
The Trigonometric Equation 139

Equation (i) reduces to (2 cos a + 1) (cos a - 2) = 0



Ê1- t 2ˆ
fi 3t = 4 ¥ Á
Ë 2 ¯
˜ = 2 1- t2 ( ) fi (2 cos a + 1) = 0, (cos a - 2) = 0
1
fi 2t 2 + 3t - 2 = 0 fi cos a = - , 2
2
fi ( 2t - 1) (t + 2) = 0 fi cosa = 2 is not possible.
1
fi t = ,- 2 1
2 Also, when cosa = -
2
when t = -2 , (sin q - cos q ) = -2
fi 2p
It is not possible. x = 2np ±
3
1 1 Note. No questions asked in 1997.
when t = , sin q - cos q =
2 2 Ex-26. Find the general values of x and y and satisfying the
pˆ equations 5 sin x cos y = 1, 4 tan x = tan y.
Ê 1
fi sin Á q - ˜ = [Roorkee Main – 1998]
Ë 4 ¯ 2 2 Soln. Given equations are
Ê pˆ 1 5 sin x cos y = 1 and 4 tan x = tan y.
fi sin Á q - ˜ = = sin a
Ë 4 ¯ 2 2 fi 5 sin x cos y = 1 ...........(i)
Ê pˆ n Ê 1 ˆ and 4sin x cos y = cos x sin y ..........(ii)
fi Á q - ˜ = np + (-1) a , where a = sin -1 Á
Ë 4¯ Ë 2 2 ˜¯ Dividing (i) by (ii), we get,
p
fi q = ÊÁ np + (-1)n a + ˆ˜ , n Œ I 4
cos x sin y = ........(iii)
Ë 4¯ 5
Note. No questions asked in 1995. Adding (i) and (iii), we get,
Ex-25. If 32 tan 8 q = 2 cos 2 a - 3 cos a and 3 cos 2q = 1 , sin ( x + y ) = 1
a. p
[Roorkee Main – 1996] fi ( x + y ) = (4n + 1), n ŒI ........(iv)
2
Soln. Given 3 cos 2q = 1 Subtracting (iii) from (i), we get,
1 3
fi cos 2q = sin ( x - y ) = - = sin a
3 5
1 - tan 2 q 1 fi ( x - y ) = mp + (-1)n a , m ΠI ........(v)
fi =
1 + tan q 2
3
From (iv) and (v), we get,
fi 1 + tan 2 q = 3 - 3 tan 2 q p m
2 x = ( 2n + m ) p +
+ (-1) a
fi 4 tan q = 2
2
2
p p ma Ê 3ˆ
fi x = ( 2n + m ) + + (-1)
1 , a = sin -1 Á - ˜
fi tan 2 q = Ë 5¯
2 2 4 2
Also, it is given that, and
fi 32 tan 8 q = 2 cos 2 a - 3 cos a p m Ê 3ˆ
2 y = ( 2n - m ) p + - (-1) a , a = sin -1 Á - ˜
4 2 Ë 5¯
Ê 1ˆ
fi 32 Á ˜ = 2 cos 2 a - 3 cos a p p ma
Ë 2¯ fi y = ( 2n - m ) + - (-1)
2 4 2
fi 2 cos 2 a - 3 cos a = 2
Note. No questions asked in 1999.
fi 2 cos 2 a - 3 cos a - 2 = 0 Ex-27. Find the smallest positive value of x and y
fi 2 cos a - 4 cos a + cos a - 2 = 0
2 p
satisfying ( x - y ) = , cot x + cot y = 2.
fi 2 cos a (cos a - 2) + 1(cos a - 2) = 0 4
[Roorkee Main – 2000]
140 Comprehensive Trigonometry with Challenging Problems & Solutions for Jee Main and Advanced

p n Êpˆ
Soln. Given equations are ( x - y ) = , fi x = np + (-1) Á ˜ , n Œ I
4 Ë 6¯
and cot x + cot y = 2
2
fi cos x sin y + sin x cos y = 2 sin x sin y Again, sec y =
3
fi sin ( x + y ) = cos ( x - y ) - cos ( x + y ) 3
fi cos y =
fi sin ( x + y ) + cos ( x + y ) = cos ( x - y ) 2
3 Êpˆ
1 1 1 fi cos 2 y = = cos 2 Á ˜
fi sin ( x + y ) + cos ( x + y ) =
cos ( x - y ) 4 Ë 6¯
2 2 2
Êpˆ
1
sin ( x + y ) +
1
cos ( x + y ) =
1 1 fi y = np ± Á ˜ , n Œ I
fi . Ë 6¯
2 2 2 2
Hence, the solutions are
Ê pˆ 1
fi sin ÁË x + y + ˜¯ = Ï n Êpˆ
Ô x = np + ( -1) ÁË 6 ˜¯ , n Œ I
4 2
p ˆ 5p Ô
Ê Ì
fi ÁË x + y + ˜¯ = Ô y = np ± Ê p ˆ , n Œ I
4 6 ÁË ˜¯
ÔÓ 6
5p p 7p
fi (x + y) = - =
6 4 12
p
Also, ( x - y ) =
LEVEL I
4 (QUESTIONS BASED ON FUNDAMENTALS)
5p p 1. Solve: 7 cos 2 q + 3 sin 2 q = 4
Thus, x = &y=
12 6 2. Solve: sin 2q + sin 4q + sin 6q = 0
Ex-28. Solve the following equations for x and y;
3. Solve: tan 2 q + (1 - 3 ) tan q - 3 = 0
5
(cosec x -sec y ) = 1
2 2

4. Solve: tan q + tan 2q + tan q tan 2q = 1

2
(2cosec x +
= 64
3 sec y ) [Roorkee Main – 2001] 5. Solve: 3 tan(q - 15∞) = tan(q + 15∞)
6. Solve: tan q + tan 2q + tan 3q = 0
Soln. The given equations are
7. Solve: 4 sin q sin 2q sin 4q = sin 3q
5
(cosec x -sec y ) = 1
2 2
........(i)
8. Solve: 2 sec q + tan q = 1
2
(2cosec x + 3 sec y ) = 64 ........(ii) 9. Solve: sin ( 2013 )q + cos ( 2013)q = 2
From (i), we get, 10. Solve: cos q + 3 sin q = 2 cos 2q
cosec x - 3 sec y = 0
2 2 2p 3
11. Solve: x + y = and cos x + cos y =
3 2
fi cosec2 x = 3 sec2 y
p
12. Solve: x + y = and tan x + tan y = 1
fi cosec x = 3 sec y .........(iii) 4
Also, from (ii), we get, 13. Solve: r sinq = 3 and r = 4 (1 + sin q ) , 0 £ q £ 2p

2
(2cosec x + 3 sec y ) = 64 = 26 14. Solve: sin x + sin y = 1 , cos 2 x - cos 2 y = 1
15. If A and B are acute +ve angles satisfying the equations
fi 2cosec x + 3 sec y = 6
3 sin 2 A + 2 sin 2 B = 1 and 3 sin 2 A = 2 sin 2 B , then
fi 2cosec x + cosec x = 6 , from (iii)
( A + 2B) .
fi 3 cosec x = 6
2
cosec x = 2 16. If tan (A – B) = 1 and sec ( A + B ) =
fi 3
1 smallest +ve values of A and B and their most general
fi sin x = values.
2
The Trigonometric Equation 141

17. If sin A = sin B and cos A = cos B


of A in terms of B. ( )
3. If tan 2 x + 1 - 3 tan x - 3 = 0 , then x is
18. Solve: 4 sin 4 x + cos 4 x = 1 p p p p
(a) np + (b) np - (c) np + (d) np -
19. Solve: 4 cos x sin x - 2 sin x = 2 sin x
2 2 3 3 4 4
7 4. If tan 2 q + cot 2 q = 2 , then q is
20. Solve: sin 6 x + cos6 x =
16 p p p p
(a) np + (b) np - (c) np + (d) np -
p 6 6 4 4
21. Solve: sin 7x + sin 4x + sin x = 0, 0 £ x £
2 5. If tan q + cot q = 2 , then q is
22. Solve: cos 3x + cos 2x p p p
Ê 3x ˆ Ê xˆ (a) np + (b) np - (c) (d) np - .
= sin ÁË ˜¯ + sin ÁË ˜¯ , 0 £ x £ 2p 4 4 3
2 2 6. The set of values of x for which
23. Solve: sin x + sin 2 x + sin 3x tan 3x - tan 2 x
= 1 is
= cos x + cos 2 x + cos 3x 1 + tan 3x tan 2 x
24. Solve: cos 2 x + cos 4 x = 2 cos x p p p
(a) f (b) (c) np + (d) 2np +
25. Solve: sin 2 x + sin x + cos 2 x + cos x + 1 = 0 4 3 4
26. Solve: cos x cos 2 x cos 4 x =
1 7. If sin 5x + sin 3x + sin x = 0, then the value of x other
4 p
than zero, lying between 0 £ x £
0£ x£p 2
27. Solve: sin 3a = 4 sin a .sin ( x + a ) .sin ( x - a ) (a)
p
(b)
p
(c)
p
(d)
p
28. Solve: sin 2 x sin 4 x + cos 2 x = cos 6 x 6 12 3 4
8. If a and b are acute positive angles satisfying the equa-
29. Solve: sin 3x.cos x + sin 2 x cos 2 x
tion 3 sin 2 a + 2 sin 2 b = 1 and 3 sin 2a - 2 sin 2b = 0,
+ sin x.cos3 x = 1, 0 £ x £ 2p then a + 2b is
30. If q1 , q 2 , q3 , q 4 be the four roots of the equation p p p
(a) 0 (b) (c) (d)
sin (q + a ) = k sin 2q , no two of which differ by a 4 3 2
multiple of 2p , then prove that 9. If 2 sin 2 x + sin 2 2 x = 2 , -p < x < p , then x is
p p p 3p
q1 + q 2 + q 3 + q 4 = ( 2n + 1) , n Œ Z (a) ± (b) ± (c) ± (d) None
4 2 4 4

LEVEL II 10. The real roots of the equation cos7 x + sin 4 x = 1 in


(MIXED PROBLESMS) (-p , p ) are
Solve the following equations and tick the correct one. p p p
(a) - ,0 (b) - , 0,
1 2 2 2
1. sin 2 q - cos q = , 0 £ q £ 2p
2 p p p
(c) 0, (d) 0, ,
2p p p 5p 2 4 2
(a) , (b) ,
3 3 3 3 11. The general solution of cos5 x - sin 5 x - 1 = 0 is
p 2p 2p 5p (a) np (b) 2np
(c) - , (d) ,
3 3 3 3 p p
(c) np + (d) 2np +
2. If 3 tan q - 2 sin q = 0 , then q is
2 2 2
p 12. If 4 sin 4 x + cos 4 x = 1 , then x is
(a) np (b) np + (-1) n
6
2
p p (a) np (b) np ± sin -1
n
(c) np - (-1) (d) np + 5
6 3 2np p
(c) (d) 2np ±
3 4
142 Comprehensive Trigonometry with Challenging Problems & Solutions for Jee Main and Advanced

13. The number of points of intersection of 2y = 1 and 24. The minimum value of 2sin x + 2cos x is
p p
y = cos x in - £ x £ is 1-
1
2 2 (a) 1 (b) 2 2
(a) 1 (b) 2 1
(c) 3 (d) 4 - Ê 1 ˆ
(c) 2 2 (d) Á 2 - ˜
14. The number of values of x in the interval [0, 3p ] Ë 2¯
satisfying the equation 2 sin 2 x + 5 sin x - 3 = 0 is 25. If cos pq + cos qq = 0 , then the different values of q
(a) 6 (b) 1 (c) 2 (d) 4 are in A.P., whose common difference is
p p
15. The number of values of x in [0, 5p ] satisfying the (a) (b)
p+q p-q
equation 3 sin 2 x - 7 sin x + 2 = 0 is
(a) 0 (b) 5 (c) 6 (d) 10 2p 3p
(c) (d)
p±q p±q
16. The number of solution of the equation
1 26. If tan 2x. tan x = 1, then x is
cot x = cot x + , 0 < x < 2p , is p p
sin x (a) (b) (6n ± 1)
(a) 1 (b) 2 (c) 3 (d) 4 3 6
p p
17. The number of solution of cos x = sin x such that (c) ( 4n ± 1) (d) ( 2n ± 1)
6 6
0 < x < 2p is
(a) 2 (b) 4 27. The maximum value of 5 sin q + 3sin(q – a) is 7, then
(c) 8 (d) None the set of all possible values of a is

18. The number of solution of the equation tan x.tan 4 x = 1 , Ê pˆ Ê 2p ˆ


(a) Á 2np ± ˜ (b) Á 2np ± ˜
0 < x < p , is Ë 3¯ Ë 3¯
(a) 1 (b) 2
È p 2p ˘
(c) 5 (d) 8 (c) Í , ˙ (d) None
Î3 3 ˚
19. The number of solution of the equation
12 cos3 x - 7 cos 2 x + 4 cos x - 9 = 0 , is Êp ˆ Êp ˆ
28. If tan Á sin q ˜ = cot Á sin q ˜ , then sin q + cos q is
(a) 0 (b) 2 Ë2 ¯ Ë2 ¯
(a) 2n – 1 (b) 2n + 1 (c) 2n (d) n
20. The sum of all solution of the equation
Êp ˆ Êp ˆ
Êp ˆ Êp ˆ 1 29. If sin Á cot q ˜ = cos Á tan q ˜ , then q is
cos q .cos Á + q ˜ .cos Á - q ˜ = is Ë4 ¯ Ë4 ¯
Ë3 ¯ Ë3 ¯ 4
Ê pˆ Ê pˆ
100p (a) ÁË np + ˜¯ (b) ÁË 2np ± ˜¯
(a) 15p (b) 30p (c) (d) None 4 4
3
Ê pˆ Ê pˆ
21. The number of solution of 16sin
2
x
+ 16cos
2
x
= 10 , (c) ÁË np - ˜¯ (d) ÁË 2np ± ˜¯
4 6
0 £ x £ 2p , is
(a) 2 (b) 4 (c) 6 (d) 8. 30. If tan (p cos q ) = cot (p sin q ) , then the values of

22. The smallest positive value of x such that Ê pˆ


cos Á q - ˜ is (are)
tan( x + 20∞) = tan( x + 10∞).tan x.tan( x - 10∞) , is Ë 4¯
(a) 30∞ (b) 45∞ (c) 60∞ (d) 75∞ 1 1 1
(a) (b) (c) ± (d) None
23. The maximum value of 2 2 2 2
Ê pˆ Ê pˆ Ê pˆ 31. If 3 tan(q - 15∞) = tan(q + 15∞) , then q is
sin Á x + ˜ + cos Á x + ˜ in ÁË 0, ˜¯ is attained at
Ë 6¯ Ë 6¯ 2
Ê pˆ Ê pˆ
(a) Á np + ˜ (b) Á np + ˜
p p p p Ë 4¯ Ë 8¯
(a) (b) (c) (d)
12 6 3 2
Ê pˆ
(c) Á np + ˜ (d) None
Ë 3¯
The Trigonometric Equation 143

Ê pˆ Ê 2p ˆ 42. |tan x + sec x| = |tan x| + |sec x|, x Œ [0, 2p],


32. If tan q + tan Á q + ˜ + tan Á q + ˜ = 3 , then q is if x belongs to that interval
Ë 3¯ Ë 3¯
È pˆ Êp ˘
Ê pˆ (a) [0, p] (b) Í0, ˜ » Á , p ˙
p Î 2¯ Ë 2 ˚
(a) (2n + 1) (b) ÁË np ± ˜¯
3 È 3p ˆ Ê 3p ˘
12 (c) Í0, ˜ » Á , 2p ˙ (d) (p, 2p].
p Î 2¯ Ë 2 ˚
(c) ( 4n + 1) (d) None 5
12 43. The number of solutions of  cos (rx ) = 5 in the
33. The equation a sin2x + cos2x = 2a – 7 posses a solution interval [0, 2p] is r =1
if (a) 0 (b) 1 (c) 5 (d) 10
(a) a > 6 (b) 2 £ a £ 6 44. If f (x) = max {tan x, cot x}. The number of roots of
(c) a > 2 (d) None 1
the equation f ( x ) = in (0, 2p) is
34. The number of all possible triplets ( a1 , a2 , a3 ) such 2+ 3
that a1 + a2 cos 2 x + a3 sin 2 x = 0 for all x, the number (a) 0 (b) 2 (c) 4 (d) •
of possible 5-tuplets is 45. If sin x + cos x + tan x + cot x + sec x + cosec x = 7
(a) 0 (b) 1 (c) 2 (d) None and sin 2x = a - b c , then a – b + 2c is
3
35. If a1 + a2 sin x + a3 cos x + a4 sin 2 x + a5 cos 2 x = 0 (a) 0 (b) 14 (c) 2 (d)
2
holds for all x, then the number of possible 5-tuplets 46. If sin4x + cos4x + 2 = 4 sin x cos y and
is
p
0 < x, y < , then sin x + cos y is
2
36. The number of solution of the equation 3
x (a) –2 (b) 0 (c) 2 (d)
1 + sin x .sin 2 = 0 in [ -p , p ] is 2
2 47. The equation cos4x – (l + 2) cos2x – (l + 3) = 0
(a) 0 (b) 1 (c) 2 (d) 3 possesses a solution if
37. The solution of (a) l > – 3 (b) l < – 2
(c) –3 < l < – 2 (d) l Œ z+
sin 4 x + cos 4 x + sin 2 x + a = 0 is solvable for
1 1 48. If 0 < q < 2p and 2 sin 2 q - 5 sin q + 2 > 0 , then
(a) - £ a £ (b) -3 £ a £ 1 the range of q is
2 2
Ê pˆ Ê 5p ˆ
(c) -
3
£a £
1
(d) -1 £ a £ 1 (a) ÁË 0, ˜¯ » ÁË , 2p ˜¯
2 2 6 6
Ê 5p ˆ
38. The equation sin 4 x - 2 cos 2 x + a 2 = 0 is solvable for (b) ÁË 0, ˜¯ » (p, 2p)
6
(a) - 3 £ a £ 3 (b) - 2 £ a £ 2
Ê pˆ
(c) -1 £ a £ 1 (d) None (c) ÁË 0, ˜¯ » (p, 2p)
6
39. The number of pairs (x, y) satisfying the equations (d) None.
sin x + sin y = sin (x + y ) and x + y = 1 , is 49. The number of values of x for which
sin 2x + cos 4x = 2 is
(a) 0 (b) 1 (c) 2 (d) •
40. The value of ‘a’ for which the equation
50. The number of solutions of the equation
4cosec2 [p (a + x )] + a2 - 4a = 0 , has a real solution, x3 + x2 + 4x + 2 sin x = 0 in 0 < x < 2p is
if (a) 0 (b) 1 (c) 2 (d) 4
(a) a = 1 (b) a = 2 (c) a = 3 (d) None 51. The number of solutions of the equation
1 tan x + sec x = 2cos x lying in the interval [0, 2p] is
41. If sin x + cos x = y + , x Π[0, p], then
y (a) 0 (b) 1 (c) 2 (d) 3
p
(a) x = , y = 1 (b) y = 0 52. The number of solutions of the equation
4
3p 2(sin4 2x + cos4 2x) + 3 sin2 x cos2 x = 0 is
(c) y = 2 (d) x = (a) 0 (b) 1 (c) 2 (d) 3
4
144 Comprehensive Trigonometry with Challenging Problems & Solutions for Jee Main and Advanced

53. cos 2x + a sin x = 2a – 7 possesses a solution for 63. The general solution of the equation
(a) all a (b) a > 6 sin100 x – cos100 x = 1, is
(c) a < 2 (d) a Π[2, 6] p p
2 2 (a) 2np + , n Πz (b) np + , n Πz
54. If 0 < x <2 p and 81sin x + 81cos x = 30, then x is 3 2
p p p p p
(a) (b) (c) p (d) (c) np + p, n Πz (d) 2np , n Πz
6 2 4 4 3
55. If 1 + sinq + sin2q + ...... to • = 4 + 2 3 , 64. The number of solution of the equation 2cosx = |sin x|
p in [–2p, 2p] is
0 < q < p, q π , then (a) 1 (b) 2 (c) 3 (d) 4
2
65. The2 general solution of the equation
p p 2
(a) q = (b) q = 2cos x + 1 = 3.2–sin x is
6 3 (a) np . n Πz
p p p 2p (b) (n + 1)p, n Πz
(c) q = or (d) or
3 6 3 3 (c) (n Р1)p, n Πz
56. If tan (pcosq) = cot (psinq), then the value of (d) None
Ê pˆ 66. If x Œ (0, 1), the greatest root of the equation
cos Á q - ˜ is sin2px = 2 cospx is
Ë 4¯
1 1 1 1 1 3
(a) (b) (c) (d) None (a) (b) (c) (d) None
2 2 2 2 4 2 4
67. The number of solutions of tan (5p cos a)
57. The most general values of x for which
= cot (5p sin a) for a Π(0, 2p) is
min
sin x + cos x = a ŒR {1, a2 – 4a + 6} are given by (a) 7 (b) 14 (c) 21 (d) 3
(a) 2np, n ΠN 68. The number of solution of the equation
p Ê xˆ
(b) 2np + , n Œ N 1 + sinx . sin2 ÁË ˜¯ = 0 in [–p, p] is
2 2
p p (a) 0 (b) 1 (c) 2 (d) 3
(c) np + (–1)n – ,nŒN 69. The number of solution of the equation
4 4
1
(d) None |cot x| = cot x + , " x Π[0, 2p] is
58. If x Π[0, 2p] and sin x + sin y = 2 then the value of sin x
x + y is (a) 0 (b) 1 (c) 2 (d) 3
p 70. The real roots of the equation
(a) p (b) (c) 3p (d) None cos7x + sin4x = 1 in (–p, p) are
2
p p p
59. The number of roots of the equation (a) – , 0 (b) – , 0,
p 2 2 2
x + 2 tan x = in the interval [0, 2p] is p p p
2 (c) ,0 (d) 0, ,
(a) 1 (b) 2 (c) 3 (d) • 2 4 2
60. The number of solutions of the equation cos(p x - 4 )
LEVEL III
cos (p x ) = 1 is
(PROBLEMS FOR JEE ADVANCED EXAM)
(a) None (b) 1 (c) 2 (d) >2
Solve the following trigonometric equations:
61. The number of solutions of the equation
Ê xˆ Ê xˆ
Ê px ˆ 1. cot Á ˜ - cosec Á ˜ = cot x
sin Á = x2 – 2 3 x + 4 Ë 2¯ Ë 2¯
Ë 2 3 ˜¯
(a) forms an empty set (b) only one 2. 8 cos x.cos 2 x.cos 4 x = sin 6 x
sin x
(c) is only two (d) is greater then two
tan x tan 2 x
62. Number of real roots of the equation 3. + +2=0
tan 2 x tan x
sec q + cosec q = 15 lying between 0 and 2p is
4. Solve: cos x cos (6x) = –1
(a) 8 (b) 4 (c) 2 (d) 0
5. Solve: cos(4x) + sin(5x) = 2
The Trigonometric Equation 145

6. sin2x + 5 cos x + 5 sin x + 1 = 0


sin 4 x - (k + 2) sin 2 x - ( k + 3) = 0 has a solution, then
7. sinx + sin 2x +sin 3x = cos x + cos 2x +cos 3x
k.
in the interval 0 £ x £ p 24. Find the number of principal solutions of the equation
8. sin 2 x tan x + cos 2 x cot x - sin 2 x x
= 26 sin x .
2
4.16sin
= 1 + tan x cot x 25. Find the general solution of
2 2 4
9. sin 4x + cos x = 2 sin 4x cos x sec x = 1 + cos x + cos 2 x + cos3 x + ......
7
10. sin 4 x + cos 4 x = sin x cos x INTEGER TYPE QUESTIONS
2
11. sin 4 x + cos 4 x 1. Find the number of values of x in (0, 5p ) satisfying
the equation 3 sin 2 x - 7 sin x + 2 = 0
Ê pˆ Ê pˆ
= 2 cos Á 2 x + ˜ cos Á 2 x - ˜ 2. Find the number of integral values of k, for which the
Ë 6 ¯ Ë 6¯ equation 2 cos x + 3 sin x = k + 1 has a solution.
Ê pˆ 1 3. Find the number of distinct real roots of
12. sin 4 x + sin 4 Á x + ˜ =
Ë 4¯ 4 sin x cos x cos x
Ê pˆ cos x sin x cos x = 0 in È- p , p ˘
Í 4 4˙
13. If cos ÁË x + ˜¯ + cos x = a cos x cos x sin x Î ˚
3
that the equation has a real solution. 4. Find the number of pairs (x, y ) satisfying the equations
1 sin x + sin y = sin (x + y) and x + y = 1
14. Find the number of roots of cos x - x + = 0 lies in 5. Find the maximum value of
2
Ê 0, p ˆ
ÁË ˜¯ 1 + sin 2 x cos 2 x 4 sin 2 x
2
f ( x) = sin x 2
1 + cos x 2
4 sin 2 x
15. Find the number of integral ordered pairs satisfy the
Ïcos ( xy ) = x sin x cos x 1 + 4 sin 2 x
2 2
equations Ì
Ó tan ( xy ) = y x
6. Find the number of solutions of sin x =
16. Find the number of real solutions of 10
sin 2016 x - cos 2016 x = 1 in [0, 2p ] 7. Find the number of solutions of
tan x + cot x = 2 cosec x in [ -2p , 2p ] .
17. Find the number of ordered pairs which satisfy the 8. Find the number of soltions of
equation x 2 + 2 x sin ( xy ) + 1 = 0 for y Œ[0, 2p ] . 1
cos x .cos 2 x .cos 4 x = in [0, p ] .
18. Find the number of solution of the equation 4
sin 5x . cos 3x = sin 6x . cos 2x in [0,p ] . 9. If x , y Œ[0, 2p ]
19. Find the number of solution of the equation (x, y) satisfying the equation sin x . cos y = 1.
È p˘ 10. If x Œ[0, 3p ] x
cos 3x. tan 5x = sin 7x lying in Í0, ˙ .
Î 2˚ 1
20. The angles B and C (B > C ) of a triangle satisfying satisfying the equation cot x = cot x + .
sin x
(
the equation 2 tan x - l 1 + tan 2 x = 0 ) 11. Find the number of solutions of
angle A, if 0 < l < 1 . tan x tan(4x) = 1, for 0 < x < p
12. Find the number of integral values of n for which the
21. Determine all values of ‘a’ for which the equation equation sin(sin x + cos x) = n has atleast one solution.
cos 4 x - (a + 2) cos 2 x - (a + 3) = 0 has a solution and 13. Find the number of real solutions of
sin{x} = cos{x} in [0, 2p]
22. Find all the solution of the equation 14. Find the number of solutions of
p È 5p 7p ˘
( ) ( )
2x 2x
sin x + sin Ê (1 - cos x ) + sin 2 x ˆ = 0 in Í , ˙. 3 +1 + 3 -1 = 23 x
2
8 Ë ¯ Î 2 2 ˚
15. Find the number of values of y in [–2p, 2p] for which
23. If the equation |sin(2x)| + |cos(2x)| = |sin(y)|
146 Comprehensive Trigonometry with Challenging Problems & Solutions for Jee Main and Advanced

LINKED COMPREHENSION TYPE Ê pˆ Ê pˆ


(FOR JEE ADVANCED EXAMS ONLY) (c) Á 2np + ˜ , Á 2np + ˜ , n Œ I
Ë 4¯ Ë 2¯
PASSAGE I
(d) None
An equation is of the form f (sin x ± cos x, ± sin x cos x ) = 0
5. If (sin x + cos x ) = 2 2 sin x cos x , then x is
can be solved by changing the variable.
Let sin x ± cos x = t Ê pˆ Ê pˆ
(a) Á 2np + ˜ , n Œ I (b) Á 2np - ˜ , n Œ I
Ë 4¯ Ë 4¯
fi sin 2 x + cos 2 x ± 2 sin x cos x = t 2 Ê pˆ Ê pˆ
(c) Á np + ˜ , n Œ I (d) Á np - ˜ , n Œ I
fi 1 ± 2 sin x cos x = t 2 Ë 4¯ Ë 4¯
Thus, the given equation is reducible to PASSAGE II
Ê t 2 - 1ˆ
a is a root of the equation ( 2 sin x - cos x ) (1 + cos x ) = sin x
2
f Át , =0
Ë 2 ˜¯ b is a root of the equation 3 cos3 x - 10 cos x + 3 = 0
On the basis of above information, answer the following and g is a root of the equation 1 - sin 2 x = cos x - sin x ,
questions . p
1. If 1 - sin 2 x = cos x - sin x , then x is 0 £ a , b , g £ . Then on the basis of above information,
2
Ê pˆ answer the following questions.
(a) 2np , ÁË 2np - ˜¯ , n Œ I
2
1. cos a + cos b + cos g is equal to
Ê pˆ
(b) 2np , ÁË np + ˜¯ , n Œ I 3 6 +2 2 +6
4 (a) (b) 3 3 + 8
6 2 6
Ê pˆ Ê pˆ
(c) ÁË 2np - ˜¯ , ÁË np + ˜¯ , n Œ I
2 4 (c) 3 3 + 2 (d) None
(d) None 6
2. If sin x + cos x = 1 + sin x cos x , then x is 2. sin a + sin b + sin g is equal to
Ê pˆ 14 + 3 2 5
(a) 2np , Á 2np + ˜ , n Œ I (a) (b)
Ë 2¯ 6 2 6
Ê pˆ 1+ 2
(b) 2np , Á np + ˜ , n Œ I (c) 3 + 4 2 (d)
Ë 4¯ 6 2
Ê pˆ Ê pˆ
(c) Á 2np - ˜ , Á np + ˜ , n Œ I 3. sin (a - b ) is equal to
Ë 2¯ Ë 4¯ (a) 1 (b) 0
(d) None 1- 2 6 3-2 2
(c) (d)
3. If sin 4 x + cos 4 x = sin x cos x , then x is 6 6
p
(a) n p , n ΠI (b) (6n + 1) , n ΠI PASSAGE III
6
p Solutions of equations a sin x ± b cos x = c. General value
(c) ( 4n + 1) , n ΠI (d) None satisfying two equations.
4
a cos q ± b sin q = c, where q satisfying two equations.
3
4. If 1 + sin 3 x + cos3 x = sin 2 x , then x is
2 a = r cosq, b = r sinq.
Ê pˆ (b) Satisfying two equations. Find the common value
(a) ( 2n + 1) p , Á 2np + ˜ , n Œ I
Ë 4¯ of lying between 0 and 2p and then add 2np.
On the basis of above information, answer the following

(b) (2n + 1) p , ÊÁË 2np - ˜ , n ŒI questions.
2¯ 1. The number of integral values of k for which the
equation 7 cos x + 5 sin x = 2k + 1 has a solution is
(a) 4 (b) 8 (c) 10 (d) 12
The Trigonometric Equation 147

Ê 7p ˆ PASSAGE V
2. If cos 3x + sin Á 2 x - ˜ = -2 , then x is Whenever the terms of two sides of the equation are of
Ë 6¯
different nature, then equations are known as non-standard
p (b) (6n - 1) p , n ΠI
(a) (6n + 1), n ŒI form, some of them are in the form of an ordinary equation
3 3 but can not be solved by standard procedures.
p
(c) ( 2n + 1) , n ΠI (d) None Non standard problems require high degree of logic,
3 they also require the use of graphs, inverse properties of
3. The value of x such that -p < x < p and satisfying functions, in-equalities.
3 On the basis of above information, answer the following
the equation 81+ cos x + cos x + cos x +....... = 43 , then x is
2

questions:
p 2p p 2p 1. The number of solutions of the equation
(a) (b) (c) – (d) –
Ê xˆ
3 3
4. The number of solutions of the equation
3 3
(
2 cos Á ˜ = 3x + 3- x is
Ë 2¯ )
esin x - e- sin x - 4 = 0 is (a) 1 (b) 2 (c) 3 (d) None
(a) 1 (b) 2 (c) 4 (d) 0 2. The number of solution of the equation
Ê xˆ p
PASSAGE IV Ë 2¯ ( )
2 cos 2 Á ˜ sin 2 x = x 2 + x -2 , 0 £ x £ is
2
Suppose equation is f ( x ) - g ( x ) = 0 or y = f ( x ) = g ( x ) , (a) 1 (b) > 1 (c) 0 (d) None
say, then draw the graphs of y = f (x) and y = g (x).
3. The number of real solutions of the equation
If graphs of y = f (x) and y = g (x) cuts at one, two,
three,………, no points, then number of solutions are one, ( )
sin e x = 5 x + 5- x is
two, three, ……, zero, respectively. (a) 0 (b) 1
On the basis of above information, answer the following
questions:
y2
4. If 0 £ x £ 2p and 2cosec x ¥
2
x - y + 1 £ 2 , then
1. The number of solutions of sin x = is 2
10
the number of ordered pairs of (x, y) is
(a) 4 (b) 6 (c) 5 (d) None
(a) 1 (b) 2
2. Total number of solutions of the equation
5p
3x + 2 tan x = , x Œ[0, 2p ] , is 5. The number of solutions of the equation sin x = x2 +
2
x + 1 is
(a) 1 (b) 2 (c) 3 (d) 4
(a) 0 (b) 1 (c) 2 (d) None
3. Total number of solutions of sin { x} = cos { x} , where
{ } = F.P.F, in [0, 2p ] is MATCH MATRIX
(a) 3 (b) 5 (c) 7 (d) None. 1. Match the following columns:
Column - I Column - II
3 p
4. If 1 - sin x = x - + a has no solution, when (A) The equation
2 2 sin x + cos x = 2 has (P) One solution
+
a ΠR , then (B) The equation
3 p 3 sin x + cos x = 4 has (Q) Two solution
(a) a ΠR + (b) a > + (C) The equation
2 3
3 sin x + 4 cos x = 6 has (R) Three solution
Ê 3 p ˆ Ê3 3 p ˆ (D) The equation
(c) a ŒÁ 0, + ˜ (d) a ŒÁ , + ˜. sin x cos x = 2 has (S) No solution
Ë 2 3¯ Ë2 2 3¯
5. Total number of solutions of cos 2x = sin x , 2. Match the following columns:
Column - I Column - II
p
where - < x < p , is cos ( A + B ) - sin ( A + B ) cos (2 B )
2
(A) If sin A cos A sin B = 0
(a) 3 (b) 4 (c) 5 (d) 6
- cos A sin A cos B
then B is (P) np
148 Comprehensive Trigonometry with Challenging Problems & Solutions for Jee Main and Advanced

cos q sin q cos q 5. Match the following columns:


(B) If Column - I Column - II
- sin q cos q sin q = 0
(A) The number of real roots of
- cos q - sin q cos q cos7 x + sin 4 x = 1 (P) 8
p
then q is (Q) (2n + 1) in ( -p , p ) , is
2 (B) The number of real (Q) 4
1 + sin q 2
cos q 2
4 sin 4q roots of cosec x = 1 + cot x
(C) If in ( -2p , 2p ) , is
sin q 2
1 + cos q 2
4 sin 4q =0,
(C) The number of integral (R) 3
sin 2 q sin 2 q 1 + 4 sin 4q values of k for which,
the equation 7 cos x + 5 sin x
p
then q is (R) (2n - 1) = 2 k + 1 has a solution is
2 (D) The number of solutions (S) 2
7p of the pair of equations (T) 7
(S)
24 2 sin 2 q - cos 2q = 0 and
3. Match the following columns;
Column - I Column -II 2 cos 2 q - 3 sin q = 0 in
(A) If 4 sin 4 x + cos 4 x = 1 , [0, 2p ] is
p
then x is (P) 6. Match the following columns:
4
(B) If sec x.cos (5 x ) + 1 = 0 ,
Column - I Column - II
p (A) If cos 3x.cos x + sin 3x ,
3
where 0 < x < 2p, (Q) Ê pˆ
6 .sin 3 x = 0 then x is (P) ÁË n p ± ˜¯ , n Œ I
then x is 3

(C) If 81sin
2
x
+ 81cos
2
x
= 30 , (B) If sin 3a = 4 sin a ,

where 0 < x < 2p, (R) -


p sin ( x + a ) sin ( x - a )
4 Ê pˆ
then x is then a is, where a π np (Q) Á n p + ˜ , n Œ I
Ë 4¯
(D) If 2 sin 2 x + sin 2 2 x = 2
where 0 < x < 2p, (S) np, n ΠI (C) If 2 tan x - 1 ,
then x is Ê np p ˆ
+ 2 cot x - 1 = 2 then x is (R) Á + ˜ , n ŒI
4. Match the following columns; Ë 4 8¯
Column - I Column -II (D) If sin10 x + cos10 x ,
(A) If cos q + cos 3q + cos 5q ,
29 Ê np p ˆ
np p = cos 4 ( 2 x ) then x is (S) Á ± , n ŒI
+ cos 7q = 0 then q is (P) + , n ŒI 16 Ë 2 4 ˜¯
2 6
7. Match the following columns:
(B) If sin x - 3 sin 2 x + sin 3x If a & b are the roots of a cos q + b sin q = c , then
= cos x - 3 cos 2 x + cos 3x Column - I Column - II
p 2b
then x is (Q) n p + , n ŒI (A) sin a + sin b is (P)
2 a+c
(C) If sin 4q - sec 2q = 2 , c-a
(B) sin a .sin b is (Q)
np p c+a
then q is (R) + , n ŒI
5 10 a
Ê ˆ b
Ê ˆ 2bc
(C) tan Á ˜ + tan Á ˜ is (R) 3
(D) If tan( x + 100∞) Ë 2¯ Ë 2¯ a + b3
= tan( x + 50∞).tan x Êaˆ Ê bˆ c2 - a2
np (D) tan Á ˜ .tan Á ˜ is (S) 2 .
.tan( x - 50∞) then x is (S) , n ŒI Ë 2¯ Ë 2¯ a + b2
5
8. Match the following columns:
Êpˆ
(T) ± ÁË ˜¯ Column - I Column - II
3 p
(A) If sin 5 x = 16 sin 5 x , (P) ( 2n + 1) , n ŒI
then x is 4
The Trigonometric Equation 149

(a) A (b) B (c) C (d) D


(B) If 4 cos 2 x.sin x - 2 sin 2 x ,
= 3 sin x then x is (Q) n p , n Œ I 5. Assertion (A): a cos x + b cos 3x £ 1 for every x in R
(C) If tan ( 2 x ) + cot ( 2 x )
2 2 Reason (R): since b £ 1
p (a) A (b) B (c) C (d) D
+2 tan (2 x ) + 2 cot ( 2 x ) = 0 (R) np + , n ΠI
6 6. Assertion (A): The set of values of x for
(D) If tan x.tan 3x.tan 4 x
2 2
tan 3x - tan 2 x
which = 1 is f
= tan 2 x - tan 2 3x + tan 4 x 1 + tan 3x.tan 2 x
Ê n pˆ Reason (R): since tanx
then x is (S) Á np + (-1) ˜
Ë 8¯ p
9. Observe the following columns: x = ( 2n + 1) , n ΠI
2
Column - I Column - II
(a) A (b) B (c) C (d) D
(A) If cos(6q ) + cos(4q )
7. Assertion (A): The number of solutions of the equation
+ cos(2q) + 1 = 0, (P) 2 n p , n ΠI tan x + sec x = 2 cos x lying in the interval [0, 2p ] is 2
then q is Reason (R): The number of solutions of the equation
(B) If 3 - 2 cos q - 4 sin q , 3 sin 2 x - 7 sin x + 2 = 0 in [0, 5p ] is 6
Рcos 2q + sin 2q = 0, (Q) np , n ΠI (a) A (b) B (c) C (d) D
then q is 3
8. Assertion (A): The number of solutions of
1
(C) If cos q .cos 2q .cos 3q = , tan x. tan 4 x = 1 in (0, p ) is 5
4 Reason (R): The number of solutions of
p
then q is (R) ( 4 n + 1) , n ΠI cos x = sin x in [0, 4p ] is 4
2
(a) A (b) B (c) C (d) D
(D) If sin (5q ) + sin (q )
p 9. Assertion (A):
= sin(3q), then q is (S) ( 2n + 1)
12 Êp ˆ Êp ˆ
If tan Á sin q ˜ = cot Á cos q ˜ , then
ASSERTION AND REASON Ë2 ¯ Ë2 ¯
Codes: sin q + cos q = ± 2
(A) Both A and R are individually true and R is the Reason (R): - 2 £ sin q + cos q £ 2
correct explanation of A. (a) A (b) B (c) C (d) D
(B) Both A and R are individually true and R is not
the correct explanantion of A. 10. Assertion (A): sin A = sin B = sin C = 2 sin(18∞)
(C) A is true but R is false. Reason (R): If cos A = tan B,
(D) A is false but R is true. cos B = tan C , cos C = tan A
1. Assertion (A): The number of real solutions (a) A (b) B (c) C (d) D
of sin x = x 2 + x + 1 is 1
Reson (R): since sin x £ 1 SELF ASSESMENT I
CH: TRIGONOMETRIC EQUATIONS
(a) A (b) B (c) C (d) D
2. Assertion (A): The number of real solutions Time: 3 Hrs. Max. Marks: 100.
Give answers of the following questions:
of cos x = 3x + 3- x
1. Solve for x: sinx + cosx
Reason (R): since cos x £ 1
(a) A (b) B (c) C (d) D
= min
a ŒR
{1, a 2 - 4a + 6 . }
2. Solve for x: cos x = cos x - 2 sin x .
3. Assertion (A): The maximum value of 3. Find all values of a for which the equation
3 sin x + 4 cos x + 10 is 15
sin 4 x + cos 4 x + sin 2 x + a = 0
Reason (R): The least value of 2 sin 2 x + 4 is 4 general solution of the equation.
(a) A (b) B (c) C (d) D 4. Find the smallest positive root of the equation
4. Assertion (A): The greatest value of sin(1 - x) = cos x .
sin 4 x + cos 2 x is 1 5. Solve for x and y:
Reason (R): The range of the function
3sin x + cos y = 1 , 25sin x + cos 2 y
2
=5.
f ( x ) = sin 2 x + cos 2 x is 1
150 Comprehensive Trigonometry with Challenging Problems & Solutions for Jee Main and Advanced

6. Solve for x and y: 9. The general solutions of sin x - 3 sin 2 x + sin 3x


1 - 2 x - x 2 = tan 2 ( x + y ) + cos 2 ( x + y ) . = cos x - 3 cos 2 x + cos 3x is
7. Solve for x and y: x 2 + 2 x sin( xy ) + 1 = 0 . p
(a) np + , n ΠI
8. Find the range of y such that the equation in x, 8
y + cos x = sin x has a real solution. For y x np p
(b) + , n ŒI
such that 0 < x < 2p . 2 8
9. For what values of k, the equation n np p
sinx + cos (k + x) + cos (k – x) = 2 (c) ( -1) + , n ŒI
2 8
has real solutions. Ê 2ˆ
10. If sin 5q = a sin 5 q + b sin 3 q + c sin q + d (d) 2n p + cos -1 Á ˜ , n Œ I [IIT-JEE-1989]
Ë 3¯
for every q ŒR , then prove that 10. No questions asked in between 1990 to 1992.
(i) a + b + c + d = 0 (ii) a + b + c = 1 11. Number of solutions of the equation
(iii) 5a + 4b = 0 (iv) b + 4c = 0. tan x + sec x = 2 cos x lying in the interval [0, 2 p ] is
(a) 0 (b) 1
QUESTIONS ASKED IN PAST IIT-JEE EXAMS
(c) 2 (d) 3. [IIT-JEE-1993]
1. The genral solution of the trigonometric 12. Determine the smallest +ve value of x (in degrees) for
equation sin x + cos x = 1 is given by which
(a) x = 2np , n Œ I tan( x + 100∞) = tan( x + 50∞) tan x tan( x - 50∞) .
Ê pˆ [IIT-JEE-1993]
(b) x = Á 2 n p + ˜ , n Œ I 13. Let n be a +ve integer such that
Ë 2¯
Êpˆ Êpˆ n
Ê n pˆ sin Á ˜ + cos Á ˜ = . Then
(c) x = Á n p + (-1) ˜ , n Œ I Ë 2n ¯ Ë 2n ¯ 2
Ë 4¯
(a) 6 £ n £ 8 (b) 4 < n £ 8
(d) None of these [IIT-JEE - 1981] (c) 4 £ n £ 8 (d) 4 < n < 8
2. Find the point of intersections of the curves [IIT-JEE-1994]
y = cos x & y = sin 3x
14. Let 2 sin 2 x + 3 sin x - 2 ≥ 0 and x 2 - x - 2 < 0 (x is
È p p˘ measured in radians). Then x lies in the interval
where x Œ Í- , ˙ [IIT-JEE-1982]
Î 2 2˚ Ê p 5p ˆ Ê 5p ˆ
3. Find all solutions of (a) Á , ˜ (b) Á -1, ˜
Ë6 6 ¯ Ë 6¯
4 cos 2 x sin x - 2 sin 2 x = 3 sin x . [IIT-JEE-1983]
Êp ˆ
4. There exist a value of q between 0 and 2p which (c) (-1, 2) (d) Á , 2˜
Ë6 ¯
[IIT-JEE-1994]
sin 4 q - 2 sin 2 q - 1 = 0? [IIT-JEE-1984] 15. Find the smallest +ve value of p for which the equation.
5. No questions asked in 1985. cos ( p sin x ) = sin ( p cos x ) has a solution
6. Find the solution set of for x Œ[0, 2p ] [IIT-JEE-1995]
2p 3
x+ y= , cos x + cos y = , where x and y are real. È p p˘
3 2 16. Find all values of q in the interval Í- , ˙ satisfying
[IIT-JEE-1986] Î 2 2˚
the equation
7. Find the set of all x in the interval [0, p ] (1 - tan q ) (1 + tan q ) sec2 q + 2tan
=0
2
q

for which 2 sin 2 x - 3 sin x + 1 ≥ 0 . [IIT-JEE - 1987] [IIT-JEE-1996]


8. The smallest +ve root of the equation 17. Find the general value of q satisfying the
tan x - x = 0 lies in equation tan 2 q + sec 2q = 1 [IIT-JEE-1997]
Ê pˆ Êp ˆ 18. Find the real roots of the equation
(a) Á 0, ˜ (b) Á , p ˜
Ë 2¯ Ë 2 ¯ cos7 x + sin 4 x = 1 in the interval ( -p , p )
Ê 3p ˆ Ê 3p ˆ [IIT-JEE-1997]
(c) Á p , ˜ (d) Á , 2p ˜ 19. The number of values of x in the interval [0, 5p ]
Ë 2 ¯ Ë 2 ¯
satisfying the equation 3 sin 2 x - 7 sin x + 2 = 0 is
[IIT-JEE-1987]
(a) 0 (b) 5
(c) 6 (d) 10 [IIT-JEE-1998]
The Trigonometric Equation 151

n 28. If sin q = cos j , then the possible values of


20. Let n be an odd integer. If sin ( nq ) = Â br sin r q for
each value of q then r =0 1Ê pˆ
ÁË q ± j - ˜¯ are ..... [IIT-JEE-2008]
(a) b0 = 1, b1 = 3 p 2
(b) b0 = 0, b1 = n p
29. For 0 < q < , the solutions of
(c) b0 = -1, b1 = n 2
(d) b0 = -1, b1 = n 2 - 3n + 3 [IIT-JEE-1998] 6 Ê (m - 1) p ˆ Ê mp ˆ
21. No questions asked in between 1999 to 2001.  cosec Á q + ˜¯ cosec ÁË q + 4 ˜¯ = 4 2
m =1 Ë 4
22. The number of values of k for which the equation
is (are)
7 cos x + 5 sin x = 2k + 1 has a solution is
p p
(a) 4 (b) 8 (a) (b)
(c) 10 (d) 12 [IIT-JEE-2002] 4 6
23. No questions asked in between 2003 to 2004. p 5p
(c) (d) [IIT-JEE-2009]
24. Let ( a, b ) Œ[-p , p ] be such that cos (a - b ) = 1 and 12 12
1 30. The number of values of q in the interval
cos (a + b ) = . The number of pairs of a, b satisfying
e Ê p pˆ np
the system of equations is ÁË - , ˜¯ such that j π for n Œ I
2 2 2
(a) 0 (b) 1
and tan q = cot 5q as well as sin ( 2q ) = cos (4q ) is...
(c) 2 (d) 4 [IIT-JEE-2005]
È p p˘ [IIT-JEE-2010]
25. Find the values of t Œ Í- , ˙ so that 31. The +ve integer value of n > 3 satisfying
Î 2 2˚
the equation
5x2 - 2 x + 1 Ï 1¸
2 sin t = 2 , "x Œ R - Ì1, - ˝ 1
=
1
+
1
is...
3x - 2 x - 1 Ó 3˛ Êpˆ Ê 2p ˆ Ê 3p ˆ
sin Á ˜ sin Á ˜ sin Á ˜
[IIT-JEE-2005] Ë n¯ Ë n¯ Ë n¯ [IIT-JEE-2011]
26. If 0 £ q £ 2p , 2 sin 2 q - 5 sin q + 2 > 0 , then the range 32. No questions asked in between 2012 to 2013.
of q is
33. For x Œ(0,p ) , the equation
Ê p ˆ Ê 5p ˆ Ê 5p ˆ
(a) Á 0, ˜ » Á , 2p ˜ (b) Á 0, ˜ » (p , 2p ) sin x + 2 sin 2 x - sin 3x = 3 has
Ë 6¯ Ë 6 ¯ Ë 6¯
Ê pˆ (b) three solutions
(c) Á 0, ˜ » (p , 2p ) (d) None of these
Ë 6¯ (c) one solution
[IIT-JEE- 2006] (d) no solution [IIT-JEE-2014]
27. The number of solutions of the pair of equations 34. The number of distinct solutions of the equation
5
2 sin 2 q - cos 2q = 0 and 2 cos 2 q - 3 sin q = 0 in the cos 2 2 x + cos 4 x + sin 4 x + cos6 x + sin 6 x = 2
4
interval [0, 2p ] is
in the interval [0, 2p ] is ...... [IIT-JEE-2015]
(a) 0 (b) 1
(c) 2 (d) 4 [IIT-JEE-2007]

ANSWERS

Exercise 1
np
1. q = , n ŒI 4. q = 2np , n Œ I
2 5
2. q = (2n + 1) p , n ΠI 5. q = ( 2n + 1) p
, n ŒI
6 7
np
6. q = (
3. q = , n ŒI 2n + 1) 3p
3 , n ŒI
14
152 Comprehensive Trigonometry with Challenging Problems & Solutions for Jee Main and Advanced

7. q = n p , n ΠI 30. q =
np p
+ , n ŒI
3 3 12
8. q = p ± p , n Œ I
n
31. q =
np
, n ŒI
3 6 3
9. q = np + (-1)n p , n ΠI 32. q =
np p
+ , n ŒI
3 3 12
p np
10. q = 2np ± , n Œ I n Êpˆ
3 33. q = + ( -1) Á ˜ , n Œ I
2 Ë 4¯
( )
11. q = np + a , a = tan -1 2 , n ΠI
p Ê 1 ˆ
34. q = n p , n p ± , n p ± tan -1 Á , n ŒI
n +1 Ê p ˆ Ë 2 ˜¯
12. q = n p + ( -1)
3
ÁË ˜¯ , n Œ I
6 np p
35. q = , ( 2n + 1) , ( 2n + 1) p , n ΠI
13. q = 2n p ± p , n Œ I 6 4 12
3 4 p
36. q = 2np + , 2 n p , n ΠI
np p 2
14. q = - , n ŒI
3 12 p
37. q = np - , np , n ŒI
np
15. q = n p + (-1) , n ΠI
4
6 p
38. q = ( 2n + 1) , n ΠI
p 2
16. q = 2n p ± , n Œ I
6 p
39. q = np + , n ŒI
p 3
17. q = n p + , n ΠI
6 p
40. q = np + , n ŒI
2p 3
18. q = 2n p ± , n ŒI
3 Exercise 2
p
19. q = np , 2n p ± , n Œ I np p
3 1. q = np + (-1) - , n ŒI
p p Ê pˆ4 3
20. q = (2n + 1) , ( 2n ± 1) , n Œ I 2. q = Á 2np - ˜ , n Œ I
2 2 Ë 4¯
p p p
21. q = n p , n p ± , n Œ I 3. q = ( 4n + 1) - , n ŒI
3 2 6
2 np p p p
22. q = ± , n ŒI 4. q = ( 4n + 1) - , n Œ I
7 14 2 4
p Ê pˆ p
23. q = np ± , n Œ I 5. q = Á 2np ± ˜ - , n Œ I
3 Ë 3¯ 6
np 2p
24. q = , 2 np ± , n ŒI 6. q = np + (-1)n p - p , n Œ I
3 3 4 4
p p p
25. q = (2n + 1) , 2 np n ΠI
n
7. q = np + (-1) - , n ŒI
4 4 4
np p Ê pˆ p
26. q = , np ± , n Œ I 8. q = Á 2np ± ˜ - , n Œ I
4 3 Ë 3¯ 6
p np
27. q = ( 4n - 1) , np + (-1) , n Œ I Ê p ˆ Ê 2np p ˆ
2 6 9. q = - Á 2np + ˜ Á + ˜ , n ŒI
Ë 3¯ Ë 3 9¯
-1 Ê ˆ
1 3p
28. q = np + tan ÁË ˜¯ , np +
2 4
, n ŒI 10. q = np ± p , n Œ I
2 18
p 3p
29. q = n p + , np + , n ŒI
3 4
The Trigonometric Equation 153

Exercise 3 p 3p
2. x = np , x = np + (-1) n , x = np - (-1) n , n ŒZ
1. q = 3p 16 16
4
Ê 1ˆ
p 3. x = np + a , where a = tan -1 Á - ˜ , n Œ Z
2. q = Ë 3¯
3
p
4. x = np + (-1) n , n ŒZ
3. q = 2p 16
3
p 3p
5. x = 2np + ± , n ŒZ
4. q = 2p 4 4
3
np
p 6. x = np + (-1) , n ΠZ
5. q = 6
4
p
7. x = (2n + 1) , n ΠZ
Exercise 4 8
1. q = 2np + 7p , n ΠZ p
4 8. x = np + , n ΠZ
4
2. q = 2np + 7p , n ΠZ np p
6 9. x = np , x = ±
2 2
3. q = 2np + p , n ΠZ np p
6 10. x= ± , n ŒZ
2 6
1 È p p˘
4. A = ¥ Í(2m + n)p + ± ˙ , Exercise 6
2 Î 4 6˚
p 5p 7p
1 È p p˘ 1. x = , ,
B = ¥ Í(2m - n)p - ± ˙ , m, n Œ Z 6 6 12
2 Î 4 6˚
2. x = sin -1 (0.6) , x = p - sin -1 (0.6)
p p
5. x = + np , y = - np , n ΠZ p 5p
2 6 3. x = ,
2 3
6. No. Solution., x = j = y
p
p p 4. x = np ± , n Œ Z
7. x = np , y = - np or, x = - np , y = np 3
4 4
np
p 5p 5. x = np + (-1) , n ŒZ
8. r = 6,q = or 3
6 6
9. A РB = 2np , n ΠZ Exercise 7
p p
p n p 1. x = ( 4n + 1) , ( 4n - 1) , n ΠZ
10. x = np + (-1) , y = 2mp ± , m, n Œ Z 2 2
6 2
p p 2p 4p
Ê 3p 3p ˆ Ê p pˆ Êp 1 ˆ 2. x = 0, , . ,
11. Á - , cos ˜ , Á , cos ˜ , Á , 4 2 9 9
Ë 8 8 ¯ Ë8 8 ¯ Ë 4 2 ˜¯ p 5p 9p 13p
2p 2p 3. x = , .p , ,
12. x = 2mp ± , y = 2(m - n)p ± , m, n Œ Z 7 7 7 7
3 3 p 5p 2p 4p 9p 13p
4. x = , , , , ,
p 2p 8 8 3 3 8 8
13. x = or =j
3 3 p 2np
5. x = (2n + 1) , , n ŒZ
p 2 3
14. x = np + (-1) n
6 p 2p
6. x = np - , 2np ± , n ŒZ
p 4 3
& y = 2mp ± , m, n Œ Z
3 7. x = np ± a, n Œ Z
Exercise 5 np
8. x = np , , n ŒZ
2 2
1. q = np + a , where sin a = , n ŒZ
5
154 Comprehensive Trigonometry with Challenging Problems & Solutions for Jee Main and Advanced

p p 2p
9. x = np - , n ŒZ 2. q = ,
4 3 3
p p
10. x = (4n + 1) , n Œ Z 3. x = np , np + (-1) n , n ŒI
8 6
Exercise 8
p p 3p 5p 2p 7p
4.
1
2
(3 -1 )
1. x = , , , , ,
6 3 8 8 3 8 5. No solution
p p
2. x = np ± , n Œ Z 6. x = 2np + , n Œ I
3 4
np np 3p Ê pˆ p
3. x = , + (-1) n , n ŒZ 7. x = ÁË np - ˜¯ , ( 2n + 1) , n Œ I
2 8 16 4 2
p p 3p 5p 7p 11p p
4. x = , , , , , 8. x =
6 4 4 6 4 6 4
5. x = ( 2n + 1) p , n ΠZ Exercise 14
Exercise 9 1. No Solution
2. x = 0
1. x = np + tan -1 (3), np + tan -1 (4), n ΠZ
2. No Solution Level I
p
3. No solution 1. q = np ± , n ŒI
3
Exercise 10
p np p
1. x = 2np ± , n ŒZ 2. q = , np ± , n Œ I
3 4 3
p p p
2. x = (4n + 1) , n ΠZ 3. q = np + , np - , n ΠI
2 3 4
2p np p
3. x = 4np ± , n ŒZ 4. q = + , n ŒI
3 3 12
p
Exercise 11 5. q = (4n + 1) , n ΠI
4
p
1. x = 2n p , 2np + , n ŒZ p , n p ± tan -1 Ê 1 ˆ , n Œ I
3 6. q = n p , n p ± ÁË ˜
p 3 2¯
2. x = 2np , ( 4n + 1) , n ΠZ
np n Êpˆ
2
7. q = + ( -1) Á ˜ , n Œ I
p 3p 3 Ë 9¯
3. x = 2np ± ± , n ŒZ
4 4 p
8. q = 2np - , n ŒI
p 4
4. x = , p
2 9. q = np , n ΠI
Exercise 12 2np p Ê p ˆ
10. q = + , - Á + 2np ˜ n Œ I
1. No Solution 3 9 Ë 3 ¯
2. No Solution
11. x = p + np , y = p - np , n ΠZ
3. No Solution 2 6
p 12. x = j = y
4. x = 2np + , n ΠZ
2
p 5p
p 13. r = 6,q = or
5. x = 2np + , n ΠZ 6 6
2
n -1 Ê 1 ˆ
Exercise 13 14. x = n p + (-1) sin ÁË ˜¯ , n Œ I
4
p 2p
1. ± , ±
n Ê 3ˆ
3 3 y = np + (-1) sin -1 Á ˜ , n Œ I
Ë 4¯
The Trigonometric Equation 155

p 53. (d) 54. (a) 55. (d) 56. (c)


15. 57. (c) 58. (a) 59. (c) 60. (b)
2
61. (b) 62. (b) 63. (b) 64. (d)
1 È p p˘ 65. (a) 66. (c) 67. (b) 68. (a)
16. A = ¥ Í(2m + n)p + ± ˙ ,
2 Î 4 6˚ 69. (c) 70. (b)
1 È p p˘
B = ¥ Í(2m - n)p - ± ˙ , m, n Œ Z Level III
2 Î 4 6˚ 2p
1. x = 4np ± , n ŒZ
17. A = np + B , n ΠI 3
p
18. q = np + a , where sin a =
2 2. x = (2n + 1)) , x = np , n ΠI
, n ŒZ 14
5
p
p 3. x = np ± , n Œ I
19. x = np , x = np + (-1) n 3
16
4. x = 2np + p = (2n + 1)p , n ΠI
3p
, x = np - (-1)n , n ŒZ Ê pˆ p
16 5. x = Á 2np + ˜ = (4n + 1) , n Œ I
np
Ë 2¯ 2
20. x = np + (-1) , n ΠZ p
6 6. x = np - , n ΠI
p p 2p 4p 4
21. x = 0, , . , p 5p 7p 13p 2p 4p
4 2 9 9 7. x = , , , , ,
p 5p 9p 13p 8 8 8 8 3 3
22. x= , ,p, , np a
7 7 7 7 8. x = + (-1)n
p 5p 2p 4p 9p 13p 2 2
23. x= , , , , , p
8 8 3 3 8 8 9. x = (2n + 1)
p 2np 2
24. x = (2n + 1) , , n ŒZ np p
2 3 10. x = + (-1) n , n ΠI
p 2p 2 12
25. x = np - , 2np ± , n ŒZ
np Ê 1ˆ
4 3
11. x = + cos -1 Á ˜ , n Œ Z
p p 3p 5p 2p 7p 2 Ë 3¯
26. x= , , , , ,
6 3 8 8 3 8 p
12. x = np , np - , n ŒZ
p 4
27. x = np ± , n Œ Z
3 13. - 3 £ a £ 3
np np 3p 14. 1
28. x= , + (-1)n , n ŒZ
2 8 16 15. 1
29. No solution 16. 1
Level II Ê pˆ Ê 3p ˆ
1. (b) 2. (a, b) 3. (a, d) 4. (c) 17. ÁË1, ˜¯ , ÁË -1, ˜¯
2 2
5. (a) 6. (a) 7. (c) 8. (d)
9. (a, b, c) 10. (b) 11. (a) 12. (a, b) 18. 5
13. (b) 14. (d) 15. (c) 16. (b) 19. 2
17. (b) 18. (c) 19. (c) 20. (b) 20. 90∞
21. (b) 22. (c) 23. (a) 24. (b) 21. -3 £ a £ -2
25. (c) 26. (b) 27. (a) 28. (b)
29. (a) 30. (c) 31. (a) 32. (c) 22. x = 13p
33. (b) 34. (d) 35. (b) 36. (a) 4
37. (c) 38. (b) 39. (c) 40. (b) 23. [–3, –2]
41. (a) 42. (b) 43. (b) 44. (a) 24. 3
45. (c) 46. (c) 47. (c) 48. (a) p
25. x = 2np ± , n ŒI
49. (a) 50. (b) 51. (c) 52. (a) 3
156 Comprehensive Trigonometry with Challenging Problems & Solutions for Jee Main and Advanced

INTEGER TYPE QUESTIONS 3p


7. x = ±1 , y = 2np + .
1. 6 2
2. 7 p
3. 2 8. x = p ,.
2
4. 4
5. 6 p p
9. np - £ k £ np + , n Œ Z .
6. 6 4 4
7. 6
8. 6 HINTS AND SOLUTIONS
9. 3
EXERCISE 1
10. 2
27. Given equation is
COMPREHENSIVE LINK PASSAGE 3
cot 2 q + +3=0
Passage-I: 1. (d) 2. (a) 3. (c) 4. (b) 5. (a) sin q
Passage-II: 1. (a, b) 2. (a, c ) 3. (c)
fi cot 2 q + 3 (1 + cosecq ) = 0
Passage-III: 1. (b) 2. (b) 3. (a, b, c, d) 4. (d)
Passage-IV: 1. (b)
Passage-V: 1. (a)
2. (c)
2. (c)
3. (b)
3. (a)
4. (b) 5. (b)
4. (b) 5. (a)
fi (cosec q - 1) + 3(1 + cosecq ) = 0
2

fi (cosec q - 1 + 3) (1 + cosecq ) = 0
(MATCH MATRIX) fi (cosec q + 2) (1 + cosecq ) = 0
1. (A) Æ (S), (B) Æ (S), (C) Æ (S), (D) Æ (S) fi cosecq = -1, - 2
2. (A) Æ (Q), (B) Æ (R), (C) Æ (S) .
3. (A) Æ (S), (B) Æ (P), (C) Æ (Q),(D) Æ (P, Q) fi sin q = -1,sin q = -
1
4. (A) Æ (S), (B) Æ (P), (C) Æ (Q, R),(D)Æ (T) 2
5. (A) Æ (R), (B) Æ (S), (C) Æ (P), (D) Æ (S) p nÊ pˆ
6. (A) Æ (S), (B) Æ (P), (C) Æ (Q), (D) Æ (R) fi q = ( 4n - 1) , q = np + (-1) Á - ˜ , n Œ I .
2 Ë 6¯
7. (A) Æ (R), (B) Æ (S), (C) Æ (P), (D) Æ (Q)
8. (A) Æ (Q), (B) Æ (R), (C) Æ (S), (D) Æ (P) 28. Given equation is
9. (A) Æ (S), (B) Æ (R), (C) Æ (Q), (D) Æ (P). 2 tan q - cot q = -1
fi 2 tan q = cot q - 1
ASSERTION AND REASON
1
1. (a) 2. (a) 3. (b) 4. (b) fi 2 tan q = -1
5. (a) 6. (a) 7. (b) 8. (b) tan q
9. (d) 10. (a) fi 2 tan 2q + tan q - 1 = 0
SELF ASSESMENT I fi 2 tan 2q + 2 tan q - tan q - 1 = 0
p p fi 2 tan q ( tan q + 1) - ( tan q + 1) = 0
1. x = np + (-1) n - , n ΠZ
4 4 fi (2 tan q - 1) ( tan q + 1) = 0
p
2. x = 2mp , (2m + 1)p + , m ΠZ .
4
fi (2 tan q - 1) = 0 , ( tan q + 1) = 0
np (-1) n 1
3. x =
2
+
2
sin -1 {1 - }
2a + 3 , where fi tan q = -1,
2
È 3 1˘ Ê pˆ Ê 1ˆ
a Œ Í- , ˙ fi q = Á np - ˜ , q = n p + a , a = tan -1 Á ˜
Ë 4¯ Ë 2¯
Î 2 2˚
1 7p 29. Given equation is
4. x = +
2 4 tan 2 q + (1 - 3 ) tan q - 3 = 0
Ê 7p p ˆ Ê 7p 5p ˆ Ê 11p p ˆ Ê 11p 5p ˆ
5. ÁË , ˜,Á , ˜ , ÁË , ˜ ,Á , ˜ fi tan 2 q + tan q - 3 ( tan q + 1) = 0
6 3¯ Ë 6 3 ¯ 6 3¯ Ë 6 3 ¯
p fi tan q ( tan q + 1) - 3 ( tan q + 1) = 0
6. x = -1 , y = np ± + 1 , n ŒZ
4
The Trigonometric Equation 157

fi (tan q - 3 ) (tan q + 1) = 0 fi tan q = 0 ,1 - tan 2 q = -2

fi tan q = 3 , tan q = -1 fi tan q = 0 , tan 2 q = 3


p p fi p
fi q = np + , q = np - , n Œ I q = np , q = np ± , n ŒI
3 4 3
32. Given equation is Ê 1 ˆ
when Á1 + ˜ =0
Ê pˆ Ê 2p ˆ Ë 1 - tan (2q ) tan (q ) ¯
tan q + tan Á q + ˜ + tan Á q + ˜ = 3
Ë 3¯ Ë 3¯ 1
fi = -1
Êp ˆ Ê Êp ˆˆ 1 - tan q tan 2q
fi tan q + tan Á + q ˜ + tan Á p - Á - q ˜ ˜ = 3
Ë3 ¯ Ë Ë3 ¯¯ fi 1 - tan q tan 2q = -1
Êp ˆ Êp ˆ fi tan q tan 2q = 2
fi tan q + tan Á + q ˜ - tan Á - q ˜ = 3
Ë3 ¯ Ë3 ¯
Ê 2 tan q ˆ
fi tan q Á =2
3 + tan q 3 - tan q Ë 1 - tan 2 q ˜¯
fi tan q + - =3
1 - 3 tan q 1 + 3 tan q fi tan 2 q = 1 - tan 2 q
8 tan q
fi tan q + =3 1 Ê 1 ˆ
1 - 3 tan 2 q fi tan 2 q = = tan 2 a , a = tan -1 Á
2 Ë 2 ˜¯
9 tan q - 3 tan 3 q fi q = np ±a , n ŒI
fi =3
1 - 3 tan 2 q
36. Given equation is
3 tan q - tan 3 q cot q - tan q = cos q - sin q
fi =1
1 - 3 tan 2 q Ê (cos q + sin q ) ˆ
tan (3q ) = 1
fi (cos q - sin q ) Á - 1˜ = 0
fi Ë sin q cos q ¯
fi p fi (cos q - sin q ) = 0 , (cos q + sin q ) = sin q cos q
3q = n p +
4
fi tan q = 1, (cos q + sin q ) = sin q cos q
fi np p
q= + , n ŒI when tanq = 1
3 12
34. Given equation is p
fi q = np + , n ŒI
tan q + tan 2q + tan 3q = 0 4
fi tan q + tan 2q + tan ( 2q + q ) = 0 when (cos q + sin q ) = sin q cos q
tan ( 2q ) + tan (q ) No real value of q
fi tan q + tan 2q + =0 37. Given equation is
1 - tan (2q ) tan (q )
(1 - tan q ) (1 + sin 2q ) = 1 + tan q
Ê 1 ˆ
fi ( tan q + tan 2q ) Á1 + =0 fi (cos q - sin q ) (cos q + sin q )2 = (cos q + sin q )
Ë 1 - tan (2q ) tan (q ) ˜¯
fi (cos q + sin q ) (cos 2q - 1) = 0
when ( tan q + tan 2q ) = 0
fi tan (q ) = -1,sin 2 q = 0
2 tan q
fi tan q + =0 fi tan (q ) = -1,sin (q ) = 0
1 - tan 2 q
fi p
Ê 2 ˆ q = np - ,q = n p , n Œ I
fi tan q Á1 + =0
Ë 1 - tan 2 q ˜¯
4
38. Given equation is
Ê 2 ˆ
fi tan q = 0 , Á1 + =0 2 sin 2 q + sin 2 2q = 2
Ë 1 - tan 2 q ˜¯
fi 2 sin 2 q + 4 sin 2 q cos 2 q = 2
2
fi tan q = 0 , = -1
1 - tan 2 q fi sin 2 q + 2 sin 2 q cos 2 q = 1
fi 2 sin 2 q cos 2 q = 1 - sin 2 q
158 Comprehensive Trigonometry with Challenging Problems & Solutions for Jee Main and Advanced

fi 2 sin 2 q cos 2 q = cos 2 q Ê pˆ


fi ÁË q - ˜¯ = 2n p ± 2q
fi (2 sin 2
)
q - 1 cos 2 q = 0
3
Taking +ve one, we get,
fi (2 sin 2
q - 1) = 0 , cos q = 0
2
Ê pˆ
q = - Á 2n p + ˜
1 Ë 3¯
fi sin 2 q = , cos q = 0
2 Taking –ve one, we get,
p p 2n p p
fi q = ( 2n + 1) , q = np ± , n Œ I fi q= + , n ŒI
2 4 3 9
39. Given equation is
10. Given equation is
sin (3a ) = 4 sin q sin (q + a ) sin (q - a )
3 (cos q - 3 sin q ) = 4 sin 2q .cos 3q
fi (
sin (3a ) = 4 sin q sin 2 q - sin 2 a ) fi 3 cos q - 3 sin q = 2 (sin 5q - sin q )
fi 3
(
3 sin a - 4 sin a = 4 sin a sin q - sin a 2 2
) fi 3 cos q - sin q = 2 (sin 5q )
It is possible only when
3 1
3 fi cos q - sin q = (sin 5q )
sin 2 q = 2 2
4
Êp ˆ
2 fi sin Á - q ˜ = sin 5q
Ê 3ˆ Ë3 ¯
fi sin q = Á ˜
2
Ë 2 ¯ n Êp ˆ
fi 5q = n p + (-1) Á - q ˜
p Ë3 ¯
fi q = np ± , n ŒI
3 when n is even
40. Given equation is
Êp ˆ
4 sin q sin 2q sin 4q = sin 3q 5q = 2k p + Á - q ˜
Ë3 ¯
fi 4 sin q sin (3q - q ) sin (3q + q ) = sin 3q p
fi 6q = 2k p +
fi 4 sin q ÈÎsin (3q ) - sin (q )˘˚ = sin 3q
2 2 3
fi kp p
fi 4 sin q ÈÎsin 2 (3q ) - sin 2 (q )˘˚ = 3 sin q - 4 sin 3 q q= + , k ŒI
3 18
fi sin q ÈÎ4 sin 2 (3q ) - 4 sin 2 (q ) + 4 sin 2 q - 3˘˚ = 0 when n is odd
Êp ˆ
fi sin q ÈÎ4 sin 2 (3q ) - 3˘˚ = 0 5q = ( 2k + 1) p - Á - q ˜
Ë3 ¯
fi sin q = 0 , ÈÎ4 sin 2 (3q ) - 3˘˚ = 0 fi p
4q = ( 2k + 1) p -
3
3
fi sin q = 0 ,sin 2 (3q ) = p p
4 fi q = ( 2k + 1) - , k ΠI
p 4 12
fi q = n p ,q = n p ± , n ŒI
3 EXERCISE 4
EXERCISE 2 4. Given tan ( A - B ) = 1
9. Given equation is p 5p
fi ( A - B) = ,
4 4
cos q + 3 sin q = 2 cos 2q
2
1 3 Also, sec ( A + B ) =
fi cos q + sin q = cos 2q 3
2 2
3
Ê pˆ fi cos ( A + B ) =
fi cos Á q - ˜ = cos 2q 2
Ë 3¯ p 11p
fi ( A + B) = ,
6 6
The Trigonometric Equation 159

Here, we observe that A – B is +ve 3


So, A>B Now cos x + cos y =
2
fi A+B>A–B Ê 2p ˆ 3
fi cos x + cos Á - x˜ =
Ï 11p Ï 11p Ë 3 ¯ 2
ÔÔ A + B = 6 ÔÔ A + B = 6
Ì or Ì 1 3 3
fi cos x - cos x + sin x =
Ô A- B = p Ô A - B = 5p 2 2 2
ÔÓ 4 ÔÓ 4 1 3 3
On solving, we get, fi cos x + sin x =
2 2 2
Ï 25 p Ï 19 p fi cos x + 3 sin x = 3
ÔÔ A = 24 ÔÔ A = 24
Ì or Ì It is not possible, since the maximum value of L.H.S
Ô B = 19p Ô B = 7p is 2.
ÔÓ 24 ÔÓ 24
So, the given system of equations has no solutions.
General values of tan ( A - B ) = 1
8. Given equations are
p
is ( A - B) = n p + , n ΠI ......(i) r sinq = 3 ...........(i)
4
and r = 4 (1 + sin q ) ..........(ii)
2
General values of sec ( A + B ) = Eliminating (i) and (ii), we get,
3
4 (1 + sin q ) sin q = 3
p
is ( A + B ) = 2n p + , n ŒI .....(ii)
fi 4 sin 2 q + 4 sin q - 3 = 0
6
On solving (i) and (ii), we get fi 4 sin 2 q + 6 sin q - 2 sin q - 3 = 0
Ï p p fi 2 sin q ( 2 sin q + 3) - 1( 2 sin q + 3) = 0
ÔÔ A = ( 2n + m ) 2 + 24
Ì . fi (2 sin q + 3) (2 sin q - 1) = 0
Ô B = ( 2n - m ) p - 5 p
ÔÓ 2 24 fi sin q = - ,
3 1
2 2
5. Given sin x = 2 sin y 1
fi sinq =
Ê 2p ˆ 2
fi sin x = 2 sin Á - x˜
Ë 3 ¯ p 5p
fi q= ,
Ê 3 ˆ 6 6
1
fi sin x = 2 Á cos x + sin x˜ p
Ë 2 2 ¯ 7. Given x + y = and tan x + tan y = 1
4
fi sin x = 3 cos x + sin x Êpˆ
fi tan ( x + y ) = tan Á ˜
fi 3 cos x = 0 Ë 4¯
fi cos x = 0 tan x + tan y
fi =1
p 1 - tan x.tan y
fi x = ( 2n + 1) fi 1 - tan x .tan y = 1
2
p p fi tan x .tan y = 0
when x = ( 2n + 1) , then y = n p -
2 6 fi tan x = 0 & tan y = 0
Hence, the solutions are
fi x = np = y .
Ï p
ÔÔ x = (2n + 1) 2 Thus, no values of x and y satisfying the given
Ì , n ŒI equations.
Ô y = np - p Therefore, the given equations have no solutions.
ÔÓ 6 10. Given equations are
2p 3
6. Given x + y = and cos x + cos y = sin x + sin y = 1 ....(i)
3 2 and cos 2 x - cos 2 y = 1
160 Comprehensive Trigonometry with Challenging Problems & Solutions for Jee Main and Advanced

Now, cos 2 x - cos 2 y = 1 Ê x - yˆ


fi cos 2 Á =1
fi 1 - 2 sin x - 1 + 2 sin y = 1
2 2 Ë 2 ˜¯

fi -2 sin 2 x - 1 + 2 sin 2 y = 0 Ê x - yˆ
fi cos 2 Á ≥1
Ë 2 ˜¯
fi ( )
2 sin 2 x - sin 2 y = -1
Ê x - yˆ
fi ÁË ˜ =0
1 2 ¯
fi (sin x + sin y ) (sin x - sin y ) = -
2 fi x=y
1
fi (sin x - sin y ) = - ....(ii) The given equation
2
On solving, we get, Ê x + yˆ Ê x - yˆ Ê x + yˆ
4 cos 2 Á + 4 cos Á cos Á +1= 0
Ë 2 ˜¯ Ë 2 ˜¯ Ë 2 ˜¯
sin x = 0 ,sin y = 1
reduces to 4 cos 2 ( x ) + 4 cos ( x ) + 1 = 0
p
fi x = n p , y = ( 4n + 1) , n ΠI
2 fi (2 cos ( x) + 1)2 = 0
Hence, the solutions are
1
fi cos ( x ) = -
Ï x = np 2
Ô
Ì p , n ŒI fi 2p
ÔÓ y = (4n + 1) 2 x=
3
=y

11. Given curves are y = cos x & y = sin 2 x 13. Given equation is
8 cos q cos j cos(q + j ) + 1 = 0
Thus, sin 2x = cos x
1
fi 2sin x cos x = cos x fi 2 cos q cos j cos (q + j ) = -
4
fi (2 sin x - 1) cos x = 0 fi 4 ÈÎcos (q + j ) + cos (q - j )˘˚ cos (q + j ) + 1 = 0
fi (2 sin x - 1) = 0 , cos x = 0 fi 4 cos 2 (q + j ) + 4 cos (q - j ) cos (q + j ) + 1 = 0
1
fi sin x = , cos x = 0 For all real 0 < q , j < p ,
2

p 5p p 3p
x= , , , 16 cos 2 (q - j ) - 16 ≥ 0
6 6 2 2
fi cos 2 (q - j ) ≥ 1
3 3
then y= ,- ,0 .
2 2 fi cos 2 (q - j ) = 1
Hence, the solutions are fi q -j = 0
Ê p 3 ˆ Ê 5p 3 ˆ Ê p ˆ Ê 3p ˆ fi q =j
Á 6 , 2 ˜ , Á 6 , - 2 ˜ , ÁË 2 , 0˜¯ , ÁË 2 , 0˜¯
Ë ¯ Ë ¯ when q = j , then the equation
12. Given equation is 4 cos 2 (q + j ) + 4 cos (q - j ) cos (q + j ) + 1 = 0
3 reduces to
fi cos x + cos y + cos ( x + y ) = -
2 fi 4 cos 2 ( 2q ) + 4 cos ( 2q ) + 1 = 0
fi 2 (cos x + cos y ) + 2 cos ( x + y ) + 2 = 1
fi (2 cos (2q ) + 1)2 = 0
Ê x + yˆ Ê x - yˆ Ê x + yˆ
fi 4 cos Á
Ë 2 ¯ ˜ cos Á
Ë 2 ¯ ˜ + 4 cos 2 Á
Ë 2 ˜¯
=1 fi (2 cos (2q ) + 1) = 0
Ê x + yˆ Ê x - yˆ Ê x + yˆ 1
fi 4 cos 2 Á + 4 cos Á cos Á +1= 0 fi cos (2q ) = -
Ë 2 ˜¯ Ë 2 ˜¯ Ë 2 ˜¯ 2
2p
For real x and y fi (2q ) =
3
Ê x - yˆ
16 cos 2 Á - 16 ≥ 0 p
Ë 2 ˜¯ fi q= =j
3
The Trigonometric Equation 161

EXERCISE 5
13 - 1
1. Given equation is fi sin x = 0 ,sin x =
4
4 sin 4 x + cos 4 x = 1 n Ê 13 - 1ˆ
fi x = n p , x = n p + ( -1) a , a = sin -1 Á ˜.
fi 4 sin x = 1 - cos x
4 4
Ë 4 ¯
fi (
4 sin 4 x = 1 + cos 2 x sin 2 x ) 4. Given equation is

fi (
sin 2 x 4 sin 2 x - cos 2 x - 1 = 0 ) 2 cos 2 x + 2 sin x = 2
fi 2 sin x = 2 (1 - cos 2 x )
fi (
sin 2 x = 0 , 5 sin 2 x - 2 = 0 ) 2 sin x = 4 sin 2 x

2
fi sin x = 0 ,sin 2 x = sin x = 2 2 sin 2 x
5 fi


Ê 2ˆ
x = n p , x = np ± a , a = sin -1 Á ˜
fi (
sin x 1 - 2 2 sin 3/ 2 x = 0 )
Ë 5¯ 1
2. Given equation is fi sin x = 0 ,sin 3/ 2 x =
2 2
4 cos x sin x - 2 sin x = 2 sin x
2 2
sin x = 0 ,sin x =
1

fi ( )
4 1 - sin 2 x sin x - 2 sin 2 x = 2 sin x 2
n Êpˆ
x = n p , n = np + (-1) Á ˜ , n Œ I
fi 2 (1 - sin x ) sin x - sin
2 2
x = sin x fi Ë 4¯
fi 2 sin x - 2 sin 3 x - sin 2 x - sin x = 0 5. Given equation is
3
fi sin x - 2 sin 3 x - sin 2 x = 0 1 + sin 3 x + cos3 x = sin 2 x
2
fi 2 sin 3 x + sin 2 x - sin x = 0
( )
fi 1 + sin 3 x + cos3 x = 3 sin x cos x
fi (
sin x 2 sin x + sin x - 1 = 0
2
)
fi 1 + (sin 3
x + cos3 x ) - 3 sin x cos x.1 = 0
fi (
sin x = 0 , 2 sin 2 x + sin x - 1 = 0 ) fi (sin x + cos x + 1) (2 - sin x cos x
-1 ± 3
fi sin x = 0 ,sin x = - sin x - cos x) = 0
2
fi sin x = 0 ,sin x = 1,sin x = -2 fi (sin x + cos x + 1) = 0
fi sin x = 0 ,sin x = 1 fi sin x + cos x = -1
p 1 1 1
fi x = n p , x = ( 4n + 1) , n ΠI fi sin x + cos x = -
2 2 2 2
3. Given equation is Ê pˆ 1
fi sin Á x + ˜ = -
sin 3x + cos 2 x = 1 Ë 4¯ 2
fi sin 3x = 1 - cos 2 x Ê p ˆ nÊ pˆ
fi Á x + ˜ = np + (-1) Á - ˜
Ë 4¯ Ë 4¯
fi ( )
sin x 3 - 4 sin 2 x = 2 sin 2 x
nÊ pˆ p
fi x = n p + (-1) Á - ˜ - , n Œ I
fi sin x (3 - 4 sin x - 2 sin x ) = 0
2
Ë 4¯ 4
6. Given equation is
fi sin x = 0 , ( 4 sin x + 2 sin x - 3) = 0
2
7
sin 6 x + cos6 x =
-2 ± 4 + 48 16
fi sin x = 0 ,sin x =
8 7
fi 1 - 3 sin 2 x cos 2 x =
-1 ± 13 16
fi sin x = 0 ,sin x = 7 9
4 fi 3 sin 2 x cos 2 x = 1 - =
16 16
162 Comprehensive Trigonometry with Challenging Problems & Solutions for Jee Main and Advanced

3 fi sin x = -1
fi sin 2 x cos 2 x =
16 p
fi x = (4n - 1) , n ΠI
3 2
fi 4 sin x cos x =
2 2
4 9. Given equation is
3
fi sin ( 2 x ) =
2
cos 4 x = cos 2 3x
4
fi 2 cos 2 2 x - 1 = cos 2 3x
p
fi (2 x ) = n p ± fi 2 cos 2 2 x = 1 + cos 2 3x
3
np p It is possible only when
fi x= ± , n ŒI
3 6 2 cos 2 2 x = 1 + cos 2 3x
7. Given equation is It is true for x = 0.
17 Hence, the solution is x = n p , n ΠI
sin 8 x + cos8 x = cos 2 2 x
16 10. Given equation is

( ) 17
2
fi sin 4 x + cos 4 x - 2 sin 4 x cos 4 x = cos 2 2 x cos 2 x = 6 tan 2 x - 2 cos 2 x
16
Ê sin 2 x ˆ

( ) 17 2 cos 2 x - 1 = 6 Á - 2 cos 2 x
2
fi 1 - 2 sin 2 x cos 2 x - 2 sin 4 x cos 4 x = cos 2 2 x 2 ˜
16 Ë cos x ¯

fi ( ) 17
1 - 4 sin 2 x cos 2 x + 2 sin 4 x cos 4 x = cos 2 2 x
16
fi 2 cos 4 x - cos 2 x = 6 - 6 cos 2 x - 2 cos 4 x
fi 4 cos 4 x + 5 cos 2 x - 6 = 0
(
fi 16 1 - 4 sin x cos x + 2 sin x cos x
2 2 4 4
) fi 4 cos 4 x + 8 cos 2 x - 3 cos 2 x - 6 = 0
(
= 17 cos 4 x + sin 4 x - 2 sin 2 x cos 2 x) fi ( ) (
4 cos 2 x cos 2 x + 2 - 3 cos 2 x + 2 = 0 )
= 17 (1 - 4 sin )
x cos 2 x
(4 cos x - 3) (cos x + 2) = 0
2
fi 2 2

fi 32 sin 4 x cos 4 x + 4 sin 2 x cos 2 x - 1 = 0 fi (4 cos 2


x - 3) = 0
fi 2 sin ( 2 x ) + sin 2 x - 1 = 0
4 2
3
fi cos 2 x =
-1 ± 5 4
fi sin 2 ( 2 x ) = p
4 fi x = np ± , n ŒI
6
5 -1
fi sin 2 ( 2 x ) =
4 EXERCISE 6
Ê 5 - 1ˆ 1. Given equation is
fi 2 x = n p ± a , a = sin -1 Á ˜
ÁË 4 ˜¯ 2 sin 2 x + sin x - 1 = 0,
Ê -1 ± 3 1
np a 5 - 1ˆ fi sin x = = ,-1
fi x= ± , a = sin -1 Á ˜ 4 2
2 2 ÁË 4 ˜¯
1
fi sin x = ,sin x = -1
8. Given equation is 2
2 sin 3 x + 2 = cos 2 3x p 5p 3p
fi x= , ,
6 6 2
fi 2 sin 3 x + 2 = 1 - sin 2 3x 2. Given equation is
fi 2 sin 3 x + sin 2 3x + 1 = 0 5 sin 2 x + 7 sin x - 6 = 0
( )
2
fi 2 sin 3 x + 3 sin x - 4 sin 3 x +1 = 0 fi 5 sin 2 x + 10 sin x - 3 sin x - 6 = 0
fi 5 sin x (sin x + 2) - 3 (sin x + 2) = 0
fi 2 sin 3 x + 9 sin 2 x - 24 sin 4 x + 16 sin 6 x + 1 = 0
fi 16 sin 6 x - 24 sin 4 x + 2 sin 3 x + 9 sin 2 x + 1 = 0
fi (5 sin x - 3) (sin x + 2) = 0
The Trigonometric Equation 163

fi (5 sin x - 3) = 0 , (sin x + 2) = 0 3
fi sin x =
fi (5 sin x - 3) = 0 2
n Êpˆ
fi sin x =
3
fi x = n p + (-1) Á ˜ , n Œ I
5 Ë 3¯
n Ê 3ˆ
fi x = np + ( -1) a , a = sin -1 Á ˜ EXERCISE 7
Ë 5¯
1. Given equation is
Hence, the solution is
cos x – cos 2 x = sin 3 x
Ê 3ˆ Ê 3ˆ
x = sin -1 Á ˜ , p - sin -1 Á ˜ Ê 3x ˆ Ê xˆ Ê 3x ˆ Ê 3x ˆ
Ë 5¯ Ë 5¯ fi 2 sin Á ˜ sin Á ˜ = 2 sin Á ˜ cos Á ˜
Ë 2¯ Ë 2¯ Ë 2¯ Ë 2¯
3. Given equation is
Ê 3x ˆ Ê Ê x ˆ Ê 3x ˆ ˆ
1 fi 2 sin Á ˜ Á sin Á ˜ - cos Á ˜ ˜ = 0
sin x - cos x =
2 Ë 2 ¯ Ë Ë 2¯ Ë 2 ¯¯
4
Ê 3x ˆ Ê Ê xˆ Ê 3x ˆ ˆ
fi 4 sin x - 4 cos x - 1 = 0
2 fi sin Á ˜ = 0 , Á sin Á ˜ - cos Á ˜ ˜ = 0
Ë 2¯ Ë Ë 2 ¯ Ë 2 ¯¯
fi 4 - 4 cos 2 x - 4 cos x - 1 = 0 Ê 3x ˆ
when sin Á ˜ = 0
fi 3 - 4 cos x - 4 cos x = 0
2 Ë 2¯
fi 4 cos 2 x + 4 cos x - 3 = 0 3x
Then = np
2
-4 ± 8 1 3
fi cos x = = ,- 2n p
8 2 2 fi x=
1 3
fi cos x =
2 Ê Ê xˆ Ê 3x ˆ ˆ
when Á sin Á ˜ - cos Á ˜ ˜ = 0
p 5p Ë Ë 2¯ Ë 2 ¯¯
fi x= ,
3 3 Ê Ê xˆ Ê 3x ˆ ˆ
4. Given equation is fi ÁË sin ÁË 2 ˜¯ = cos ÁË 2 ˜¯ ˜¯
tan 2 x - 2 tan x - 3 = 0 Ê xˆ Ê 3x ˆ
fi sin Á ˜ = cos Á ˜
fi ( tan x - 3) ( tan x + 1) = 0 Ë 2¯ Ë 2¯
fi ( tan x - 3) = 0 , ( tan x + 1) = 0 fi
Ê 3x ˆ Ê p xˆ
cos Á ˜ = cos Á - ˜
fi ( tan x - 3) = 0 , ( tan x + 1) = 0 Ë 2¯ Ë 2 2¯

fi tan x = -1, tan x = 3 Ê 3x ˆ Ê p xˆ


fi ÁË ˜¯ = 2n p ± ÁË - ˜¯
p 2 2 2
fi x = np - , x = n p + a , a = tan -1 (3) . Taking +ve sign, we get,
4
5. Given equation is p
2 x = 2n p +
2 cos 2 x - 3 sin x + 1 = 0 2
p
fi 2 - 2 sin 2 x - 3 sin x + 1 = 0 fi x = np +
4
fi 3 - 2 sin 2 x - 3 sin x = 0 Taking –ve sign, we get,
fi 2 sin 2 x + 3 sin x - 3 = 0 p
x = 2n p - , n ŒI
- 3 ± 27 - 3 ± 3 3 2
fi sin x = = 2. Given equation is
4 4
-4 3 2 3 sin7x + sin 4 x + sin x = 0
fi sin x = ,
4 4 fi (sin 7 x + sin x ) + sin 4 x = 0
3 fi 2 sin 4 x cos 3x + sin 4 x = 0
fi sin x = - 3,
2
164 Comprehensive Trigonometry with Challenging Problems & Solutions for Jee Main and Advanced

fi sin 4 x ( 2 cos 3x + 1) = 0 6. Given equation is

fi sin 4 x = 0 , (2 cos 3x + 1) = 0 sin 2 x + cos 2 x + sin x + cos x + 1 = 0


1 fi (1 + sin 2 x ) + (sin x + cos x ) + cos 2 x = 0
fi sin 4 x = 0 , cos 3x = -
2 fi (sin x + cos x )2 + (sin x + cos x )
fi 4 x = n p , 3x = 2np ±
2p
3
( )
+ cos 2 x - sin 2 x = 0

np 2np 2p fi (sin x + cos x ) (2 cos x + 1) = 0


fi x= , x= ± , n ŒI
4 3 9 fi (sin x + cos x ) = 0 , (2 cos x + 1) = 0
Hence, the solutions are 1
fi tan x = -1, cos x = -
2p p p 2
x = 0, , ,
9 4 2 p 2p
fi x = np - , x = 2n p ± , n ŒI
3. Given equation is 4 3
Ê 3x ˆ Ê xˆ 7. Do yourself.
cos 3x + cos 2 x = sin Á ˜ + sin Á ˜ 8. Given equation is
Ë 2¯ Ë 2¯
Ê 5x ˆ Ê xˆ Ê xˆ tan 3x + tan x = 2 tan 2 x
fi 2 cos Á ˜ cos Á ˜ = 2 sin x cos Á ˜
Ë 2¯ Ë 2¯ Ë 2¯ sin 4 x 2 sin 2 x
fi =
cos 3x cos x cos 2 x
Ê Ê 5x ˆ ˆ Ê xˆ
fi 2 Á cos Á ˜ - sin x˜ cos Á ˜ = 0 2 sin 2 x cos 2 x 2 sin 2 x
Ë Ë 2¯ ¯ Ë 2¯ fi =
cos 3x cos x cos 2 x
Ê Ê 5x ˆ ˆ Ê xˆ Ê cos 2 x 1 ˆ
fi 2 Á cos Á ˜ - sin x˜ = 0 , cos Á ˜ = 0 2 sin 2 x Á - =0
Ë Ë 2¯ ¯ Ë 2¯ fi Ë cos 3x cos x cos 2 x ˜¯
Ê 5x ˆ Ê xˆ Ê cos 2 x 1 ˆ
fi cos Á ˜ = sin x , cos Á ˜ = 0 fi 2 sin 2 x = 0 , Á =
Ë 2¯ Ë 2¯ Ë cos 3x cos x cos 2 x ˜¯
Ê 5x ˆ Êp ˆ Ê xˆ fi sin 2 x = 0 , 2 cos 2 2 x = cos 4 x + cos 2 x
fi cos Á ˜ = cos Á - x˜ , cos Á ˜ = 0
Ë 2¯ Ë2 ¯ Ë 2¯
fi sin 2 x = 0 , 2 cos 2 2 x = 2 cos 2 2 x - 1 + cos 2 x
Ê 5x ˆ Êp ˆ Ê xˆ p
fi ÁË ˜¯ = 2np ± ÁË - x˜¯ , ÁË ˜¯ = ( 2n + 1) fi sin 2 x = 0 , cos 2 x = 1
2 2 2 2
fi 2 x = n p , 2 x = 2np
4n p
fi x= ± (p - 2 x ) , x = ( 2n + 1) p np
5 fi x= , x = np , n ΠI
2
4n p
fi x= ± (p - 2 x ) , x = ( 2n + 1) p 9. Given equation is
5
p p 3p 13 p 17p 7p 5p 29p (1 - tan x ) (1 + sin 2 x ) = (1 + tan x )
fi x= , , , , , , ,
2 tan x ˆ
(1 - tan x ) ÊÁË1 +
3 5 5 15 15 5 3 15
fi ˜ = (1 + tan x )
4. Do yourself. 1 + tan 2 x ¯
5. Given equation is
fi (1 - tan x ) (1 + tan x )2 = (1 + tan x ) (1 + tan 2 x )
cos 2 x + cos 4 x = 2 cos x
fi 2 cos 3x cos x = 2 cos x fi (1 - tan x) (1 + tan x) = (1 + tan x) (1 + tan x)
2 2

fi (2 cos 3x - 1) cos x = 0 fi ((1 - tan x) - (1 + tan x)) (1 + tan x) = 0


2 2

fi (2 cos 3x - 1) = 0 , cos x = 0
fi tan 2 x (1 + tan x ) = 0
1
fi cos 3x = , cos x = 0 tan 2 x = 0 , (1 + tan x ) = 0
2 fi
2np 2p p fi tan 2 x = 0 , tan x = -1
fi x= ± , x = ( 2n + 1)
3 9 2
The Trigonometric Equation 165

p 5. Given equation is
fi x = np , x = np - , n ŒI
4 cos (6 x ) cos x = -1
10. Given equation is 2 cos (6 x ) cos x = -2

sin x - 3 sin 2 x + sin 3x = cos x - 3 cos 2 x + cos 3x
fi cos 7 x + cos 5 x = -2
fi (sin 3x + sin x ) - 3 sin 2 x It is possible only when
= (cos 3x + cos x ) - 3 cos 2 x
cos (7 x ) = -1, cos (5 x ) = -1
fi 2 sin 2 x cos x - 3 sin 2 x = 2 cos 2 x cos x - 3 cos 2 x
p p
sin 2 x ( 2 cos x - 3) = ( 2 cos x - 3) cos 2 x fi x = (2n + 1) , x = ( 2n + 1) , n ΠI
fi 7 5
sin 2 x
fi (2 cos x - 3) = (2 cos x - 3) EXERCISE 9
cos 2 x
1. Given equation is
sin 2 x
fi =1 5 sin 2 x - 7 sin x cos x + 10 cos 2 x = 4
cos 2 x
fi tan 2 x = 1 fi 5 tan 2 x - 7 tan x + 10 = 4 sec 2 x
p fi 5 tan 2 x - 7 tan x + 10 = 4 + 4 tan 2 x
fi 2x = n p +
4
fi tan 2 x - 7 tan x + 6 = 0
np p
fi x=
2
+ , n ŒI
8 fi ( tan x - 1) ( tan x - 6) = 0
fi tan x = 1, 6
EXERCISE 8
p
1. Do yourself. fi x = np + , x = n p + a , a = tan -1 (5)
4
2. Do yourself.
3. Given equation is 2. Given equation is
sin 4 x sin 2 x = cos 6 x - cos 2 x 2 sin 2 x - 5 sin x cos x - 8 cos 2 x = -3
fi sin 4 x sin 2 x = -2 sin 4 x sin 2 x fi 2 tan 2 x - 5 tan x - 8 = -3 sec2 x
fi 3 sin 4 x sin 2 x = 0 fi 2 tan 2 x - 5 tan x - 8 = -3 - 3 tan 2 x
fi sin 4 x = 0 ,sin 2 x = 0
fi 5 tan 2 x - 5 tan x - 5 = 0
fi 4x = n p , 2x = n p , n ΠI
fi tan 2 x - tan x - 1 = 0
np np
fi x= ,x = , n ŒI 1± 5
4 2 fi tan x =
4. Given equation is 2
sec x cos 5 x + 1 = 0 Ê1± 5ˆ
fi x = n p + a , a = tan -1 Á ˜
fi cos 5 x + cos x = 0 Ë 2 ¯
fi 2 cos (3x ) cos ( 2 x ) = 0 3. Given equation is

fi 2 cos (3x ) = 0 , cos (2 x ) = 0 sin 3 x cos x + sin 2 x cos 2 x + sin x cos3 x = 1

fi cos (3x ) = 0 , cos ( 2 x ) = 0 fi sin x cos x ÈÎsin 2 x + sin x cos x + cos 2 x ˘˚ = 1


p p
3x = (2n + 1) , 2 x = (2n + 1) , n ΠI fi sin x cos x [1 + sin x cos x ] = 1
2 2
fi 2 sin x cos x [ 2 + 2 sin x cos x ] = 4
p p
fi x = (2n + 1) , x = (2n + 1) , n ΠI
6 4 fi sin ( 2 x ) ( 2 + sin ( 2 x )) = 4
Hence, the solutions are
fi sin 2 ( 2 x ) + 2 sin ( 2 x ) - 4 = 0
p p p 3p 5p
x= , , , , -2 ± 20
6 4 2 4 6 fi sin ( 2 x ) =
2
166 Comprehensive Trigonometry with Challenging Problems & Solutions for Jee Main and Advanced

-2 ± 2 5 Ê xˆ
fi sin ( 2 x ) = = -1 ± 5 when sin Á ˜ = 0
Ë 2¯
2
It is not possible . fi x = 2n p , n ΠI
So, it has no solution.
Ê Ê xˆ ˆ
when Á 2 sin Á ˜ + cot x˜ = 0
EXERCISE 10 Ë Ë 2¯ ¯
1. Given equation is Ê xˆ cos x
fi 2 sin Á ˜ = -
(cos x - sin x ) (2 tan x + sec x ) + 2 = 0 Ë 2¯ sin x

fi (cos x - sin x ) (2 sin x + 1) + 2 cos x = 0 Ê 2 Ê xˆ 2 Ê xˆˆ


ÁË cos ÁË 2 ˜¯ - sin ÁË 2 ˜¯ ˜¯
Ê xˆ
Ê 1 - tan 2 ( x / 2) 2 tan ( x / 2) ˆ fi 2 sin Á ˜ = -
fi - Ë 2¯ Ê xˆ Ê xˆ
Á ˜
Ë 1 + tan ( x / 2) 1 + tan ( x / 2) ¯
2 2 2 sin Á ˜ cos Á ˜
Ë 2¯ Ë 2¯
Ê 4 tan ( x / 2) ˆ Ê 1 - tan 2 ( x / 2) ˆ Ê xˆ Ê xˆ Ê xˆ Ê xˆ
Á + 1˜ + 2 Á ˜ =0 fi 4 sin 2 Á ˜ cos Á ˜ + cos 2 Á ˜ - sin 2 Á ˜ = 0
Ë 1 + tan ( x / 2) ¯
2
Ë 1 + tan ( x / 2) ¯
2 Ë 2¯ Ë 2¯ Ë 2¯ Ë 2¯

Ê xˆ Ê xˆ Ê xˆ Ê xˆ Ê xˆ
Put tan Á ˜ = t and then solve it. fi cos 2 Á ˜ + 4 sin 2 Á ˜ cos Á ˜ - sin 2 Á ˜ = 0
Ë 2¯ Ë 2¯ Ë 2¯ Ë 2¯ Ë 2¯
2. Given equation is For all real x,
x x Ê xˆ Ê xˆ
sin 3 - cos3 16 sin 4 Á ˜ + 4 sin 2 Á ˜ ≥ 0
2 2 = cos x Ë 2¯ Ë 2¯
2 + sin x 3 Ê xˆ Ê xˆ
fi 4 sin 4 Á ˜ + sin 2 Á ˜ = 0
Ê Ê xˆ Ê x ˆ ˆ Ê sin x ˆ Ë 2¯ Ë 2¯
ÁË sin ÁË 2 ˜¯ - cos ÁË 2 ˜¯ ˜¯ ÁË1 + 2 ˜¯
fi Ê xˆ Ê Ê xˆ ˆ
2 + sin x fi sin 2 Á ˜ Á 4 sin 2 Á ˜ + 1˜ = 0
Ë 2¯ Ë Ë 2¯ ¯
Ê 2 Ê xˆ 2 Ê xˆˆ Ê xˆ Ê Ê xˆ ˆ
ÁË cos ÁË 2 ˜¯ - sin ÁË 2 ˜¯ ˜¯ fi sin 2 Á ˜ = 0 , Á 4 sin 2 Á ˜ + 1˜ = 0
= Ë 2¯ Ë Ë 2¯ ¯
3
Ê xˆ
fi sin 2 Á ˜ = 0
3 Ê Ê xˆ xˆ Ë 2¯
fi - = Á cos Á ˜ + sin ˜
2 Ë Ë 2 ¯ 2¯ Ê xˆ
It is not possible. fi sin Á ˜ = 0
Ë 2¯
So, it has no solution.
3. Given equation is fi x = 2n p , n ΠI
4. Given equation is
Ê xˆ Ê xˆ
cot Á ˜ - cosec Á ˜ = cot x sin (q + a ) = k sin ( 2q )
Ë 2¯ Ë 2¯
Ê xˆ sin q cos a + cos q sin a = k sin ( 2q )
cos Á ˜ - 1 fi
Ë 2¯
fi = cot x Ê Êqˆ ˆ Ê
Ê xˆ 2 Êqˆ ˆ
sin Á ˜
Ë 2¯ Á 2 tan ÁË 2 ˜¯ ˜ Á 1 - tan ÁË 2 ˜¯ ˜
fi Á ˜ cos a + Á ˜ sin a
Ê xˆ Ê xˆ Á 1 + tan 2 Ê q ˆ ˜ Á 1 + tan 2 Ê q ˆ ˜
fi 2 sin 2 Á ˜ = - sin Á ˜ cot x ÁË ÁË ˜¯ ˜¯ ÁË ÁË ˜¯ ˜¯
2 2
Ë 2¯ Ë 2¯
Ê Êqˆ ˆ Ê 2 Êqˆ ˆ
Ê Ê xˆ ˆ Ê xˆ Á 2 tan ÁË 2 ˜¯ ˜ Á 1 - tan ÁË 2 ˜¯ ˜
fi ÁË 2 sin ÁË 2 ˜¯ + cot x˜¯ sin ÁË 2 ˜¯ = 0 = 2k Á ˜Á ˜
Á 1 + tan 2 Ê q ˆ ˜ Á 1 + tan 2 Ê q ˆ ˜
Ê Ê xˆ ˆ Ê xˆ ÁË ÁË ˜¯ ˜¯ ÁË ÁË ˜¯ ˜¯
2 2
fi ÁË 2 sin ÁË 2 ˜¯ + cot x˜¯ = 0 ,sin ÁË 2 ˜¯ = 0
The Trigonometric Equation 167

Ê1- t2 ˆ fi sin x + cos x = 1


Ê 2t ˆ
fi ÁË ˜ cos a + Á ˜ sin a
1+ t2 ¯ Ë1+ t2 ¯ fi
1
sin x +
1
cos x =
1
2 2 2
Ê 2t ˆ Ê 1 - t ˆ
2
= 2k . Á Ê pˆ 1
Ë 1 + t 2 ˜¯ ÁË 1 + t 2 ˜¯ fi sin Á x + ˜ =
Ë 4¯ 2
fi ( ) ( ) (
2t 1 + t 2 cos a + 1 - t 4 sin a = 4k t 1 - t 2 ) fi
n Êpˆ p
x = n p + (-1) Á ˜ - , n Œ I
Ë 4¯ 4
fi (sin a ) t 4 - (4k + 2 cos a ) t 3
2. Given equation is
+ ( 4k - 2 cos a ) t - sin a = 0 .
3 ....(i)
1 + sin 3 x + cos3 x = sin 2 x
Let t1, t2, t3 and t4 are its four roots 2
4k + 2 cos a fi 1 + (sin x + cos x ) (1 - sin x cos x ) = 3 sin x cos x
 t1 = = s1
sin a Put sin x + cos x = t
 t1t2 = 0 = s2 t2 -1
fi sin x .cos x =
2 cos a - 4k 2
 t1t2t3 = = s3
Now, equation (i) becomes
sin a
sin a Ê t 2 - 1ˆ 3 2
 t1t2t3t4 = -
sin a
= -1 = s4 1 + t Á1 -
Ë
= t -1
2 ˜¯ 2
( )
Ê q + q 2 + q3 + q 4 ˆ s1 - s3
Now, tan Á 1
Ë 2 ˜¯ = 1 - s + s
2 4
fi ( ) (
2 + t 3 - t2 = 3 t2 -1 )
Ê q1 + q 2 + q 3 + q 4 ˆ fi 2 + 3t - t 3 - 3t 2 + 3 = 0
fi tan Á ˜¯ = •
Ë 2 fi 3t - t 3 - 3t 2 + 5 = 0
Ê q + q 2 + q3 + q 4 ˆ Êpˆ fi
fi tan Á 1 ˜¯ = tan ÁË 2 ˜¯ t 3 + 3t 2 - 3t - 5 = 0
Ë 2
fi t 3 + t 2 + 2t 2 + 2t - 5t - 5 = 0
Ê q1 + q 2 + q 3 + q 4 ˆ Êpˆ
fi ÁË ˜ = np + Á ˜ fi t 2 (t + 1) + 2t (t + 1) - 5 (t + 1) = 0
2 ¯ Ë 2¯
fi (q1 + q 2 + q3 + q 4 ) = (2n + 1) p , n ΠI . fi (t 2
)
+ 2t - 5 (t + 1) = 0

fi -2 ± 24
EXERCISE 11 t = -1, t =
2
1. Given equation is
fi -2 ± 24
sin x + cos x = 1 - sin x cos x .....(i) sin x + cos x = -1,
2
Put sin x + cos x = t fi sin x + cos x = -1
1 1 1
fi sin x + cos x = -
fi sin x .cos x = t - 1
2
2 2 2
2
Ê p ˆ 1
Now, equation (i) becomes fi sin Á x + ˜ = -
Ë 4¯ 2
t2 -1
t =1- nÊ pˆ p
2 fi x = n p + (-1) Á - ˜ - , n Œ I
Ë 4¯ 4
fi 2t = 2 - t 2 + 1
3. Given equation is
fi t 2 + 2t - 3 = 0
sin 2 x - 12 (sin x - cos x ) + 12 = 0
fi (t + 3) (t - 1) = 0
2 sin x cos x - 12 (sin x - cos x ) + 12 = 0
fi (t + 3) = 0 , (t - 1) = 0
sin x cos x - 6 (sin x - cos x ) + 6 = 0 ...(i)
sin x + cos x = 1,sin x + cos x = -3
Put sin x + cos x = t
168 Comprehensive Trigonometry with Challenging Problems & Solutions for Jee Main and Advanced

It is possible only when


t2 -1
fi sin x .cos x =
2 sin 4 x = 1, tan 8 x = 0
Now, equation (i) becomes fi sin 2 x = 1, tan x = 0
fi t2 -1 p
- 6t + 6 = 0 fi x = np ± , x = np , n Œ I
2 2
fi t 2 - 1 - 12t + 12 = 0
fi above equations
t 2 - 12t + 11 = 0
Hence, the equation has no solution.
fi (t - 1) (t - 11) = 0
3. Given sin 2 x + cos 2 y = 2 sec2 z
fi sin x + cos x = 1
Here, L.H.S £ 2 and R.H.S ≥ 2
Ê pˆ 1
fi sin Á x + ˜ = It is possible only when
Ë 4 ¯ 2
n Êpˆ p sin 2 x = 1, cos 2 y = 1,sec 2 z = 1
fi x = n p + (-1) Á ˜ - , n Œ I
Ë 4¯ 4 fi cos 2 x = 0 ,sin 2 y = 0 , cos 2 z = 1
fi p fi cos 2 x = 0 ,sin 2 y = 0 ,sin 2 z = 1
x = 0 , , 2p
2
fi cos x = 0 ,sin y = 0 ,sin z = 0
Hence, the solution is
p
3p . fi x = ( 2n + 1) , y = m p , z = kp
sin 6 x = sin 2
2
where, n , m , k ΠI
4. The given equation can be written as
4. Given equation is
sin 6 x + cos 4 x = – 2
sin 3x + cos 2x + 2 = 0
fi sin 6x = –1 and cos 4x = –1
It is possible only when
fi 3p ,
sin 6 x = sin cos 4x = cos p sin 3x = -1, cos 2 x = -1
2
3p 3p
fi 6 x = 2np + , 4 x = 2np + p , n ΠZ fi 3x = , 2x = p
2 2
np p np p p p
fi x= + ,x = + , n ŒZ fi x= , x =
3 4 2 4 2 2
p 5p 7p 11p Hence, the general solution is
fi x= , , , ,...
4 4 12 12 p
x = 2n p + , n ŒI
p 3p 5p 7p 2
...x = , , , ,.....
4 4 4 4 5. Given equation is
p 5p cos 4x + sin 5x = 2
fi x= ,
4 4 It is possible only when
Hence, the general solution will be, cos 4 x = 1,sin 5 x = 1
p 5p p
fi x = 2np + , 2np + , n ŒZ fi 4 x = 2n p , 5 x = ( 4n + 1)
4 4 2
p p 2n p p
fi x = 2np + , ( 2n + 1) p + , n ΠZ fi x= , x = (4n + 1)
4 4 4 10
p p
fi x = mp + , m ΠZ Thus, x =
4 2
EXERCISE 12 Hence, the solution is
2. Given equation is p p
x = 2n p + = ( 4n + 1) , n ΠI
2 2
sin 4 x = 1 + tan 8 x
The Trigonometric Equation 169

EXERCISE 13 n Êpˆ
1. Given equation is fi x = np + (-1) Á ˜ , x = 2np , x = (2n + 1) p
Ë 6¯
3 5
21+ cos x + cos x + cos x + cos 4 x + cos x +........to • 4. Given equation is
2
=4
1 esin x - e- sin x - 4 = 0
1- cos x
fi 2 = 4 = 22 1
fi t - - 4 = 0, t = esin x
1
=2 t

1 - cos x fi t - 4t - 1 = 0
2

fi 1 - cos x =
1 fi ( t - 2 )2 = 5
2
fi t = 2± 5
fi 1 1
cos x = 1 - = fi esin x = 2 ± 5
2 2

fi cos x = ±
1 fi sin x = log e 2 ± 5 ( )
(2 + 5 )
2
fi sin x = log e
p 2p
Hence, the values of x are ± , ±
3 3 fi sin x = log e (2 + 5 ) > 1
2. Given equation is It is not possible.
1 + sin q + sin q + ... = 4 + 2 3
2 So, it has no solution.

fi 1 5. We have
= 4+2 3
1 - sinq 2
x + sin 4 x +sin 6 x +................to• ]log e 2
e[sin
1
fi 1 - sinq = Ê sin 2 x ˆ
4+2 3 Á ˜ log e 2
Ë 1- cos 2 x ¯ 2
x log e 2 2
x
1 = e = e tan = 2 tan
fi sinq = 1 - 2
x
4+2 3 Let a = 2tan
1 2- 3 3 Thus, a 2 - 9a + 8 = 0
fi sinq = 1 - =1- =
4+2 3 2 2 fi (a - 1) (a - 8) = 0
n Êpˆ
fi q = n p + (-1) Á ˜ , n Œ I fi a = 1, 8
Ë 3¯ 2
x
when a = 1, then 2tan = 1 = 20
3. Given equation is 2
x
3 1
fi 2tan = 1 = 20
sin x - sin x +
2
cos x 2 2 =1 fi tan 2 x = 0
Ê 2 3 1ˆ fi x = np , n ŒI
fi ÁË sin x - sin x + ˜¯ log cos x = 0
2 2 2
x
when a = 8, then 2tan = 8 = 23
fi (2 sin 2
)
x - 3 sin x + 1 log cos x = 0
fi tan 2 x = 3
fi (sin x - 1) (2 sin x - 1) log cos x = 0 fi tan x = 3
fi (sin x - 1) = 0 , (2 sin x - 1) = 0 , log cos x =0
cos x
1 Now,
fi sin x = 1, sin x = , log cos x = 0 cos x + sin x
2 1 1
1 = =
fi sin x = , cos x = 1 1 + tan x 3 +1
( )
2
1 3 -1
fi sin x = , cos = 1, cos x = -1 =
2 2
170 Comprehensive Trigonometry with Challenging Problems & Solutions for Jee Main and Advanced

6. Given equation is EXERCISE 14


log cos x tan x + logsin x cot x = 0 1. Given equation is
Ê sin x ˆ Ê cos x ˆ Ê xˆ 1
fi log cos x Á + logsin x Á =0 2 cos 2 Á ˜ sin 2 x = x 2 + 2
˜
Ë cos x ¯ Ë sin x ˜¯ Ë 2¯ x
fi log cos x (sin x ) + logsin x (cos x ) = 2 p
Here, L.H.S. < 2 for 0 < x <
2
It is possible only when and R.H.S. ≥ 2
sin x = cos x So, it has no solutions
2. Given equation is
fi tan x = 1
Ê x2 + x ˆ
p x -x
fi x= 2 cos 2 Á ˜ =2 +2
4 Ë 6 ¯
It is possible only when x = 0.
7. Given equation is Hence, the solution is x = 0.
3sin 2 x + 2 cos x
+ 31-sin 2 x + 2 sin x
2 2
= 28
LEVEL III
3sin 2 x + 2 cos x
+ 31-sin 2 x + 2- 2 cos x
2 2
fi = 28
1. Given equation is
sin 2 x + 2 cos 2 x 33
fi 3 + = 28 Ê xˆ Ê xˆ
3sin 2 x + 2 cos
2
x cot Á ˜ - cosec Á ˜ = cot x
Ë 2¯ Ë 2¯
27
= 28 , a = 3sin 2 x + 2 cos x
2
fi a+ cos ( x / 2) - 1
a fi = cot x
sin ( x / 2)
fi a 2 - 28 a + 27 = 0
2 sin 2 ( x / 4)
fi (a - 27) (a - 1) = 0 fi - = cot x
sin ( x / 2)
fi a = 27, 1
2 sin 2 ( x / 4)
when a = 27, then 3sin 2 x + 2 cos x fi - = cot x
2
= 33
2 sin( x / 4) cos( x / 4)
fi sin 2 x + 2 cos 2 x = 3 fi tan( x /4) + cot x = 0
fi sin 2 x + 1 + cos 2 x = 3 sin( x / 4) cos x
fi + =0
fi sin 2 x + cos 2 x = 2 cos( x / 4) sin x
It is not possible. Ê xˆ
fi cos Á x - ˜ = 0
when a = 1, then 3 sin 2 x + 2 cos 2 x
=3 0 Ë 4¯
fi sin 2 x + 2 cos 2 x = 0 Ê 3x ˆ
fi cos Á ˜ = 0
Ë 4¯
fi sin 2 x + 1 + cos 2 x = 0
3x p
fi sin 2 x + cos 2 x = -1 fi = ( 2n + 1)
4 2
1 1 1
fi sin 2 x + cos 2 x = - p
2 2 2 fi x = ( 4n + 2 ) , n ΠI
3
Ê pˆ 1
fi sin Á 2 x + ˜ = - 2. Given equation is
Ë 4 ¯ 2
sin 6 x
Ê pˆ nÊ pˆ 8 cos x.cos 2 x.cos 4 x =
fi ÁË 2 x + ˜¯ = n p + (-1) ÁË - ˜¯ sin x
4 4 fi 4 sin 2 x cos 2 x cos 4 x = sin 6 x
np nÊ pˆ p fi 2 sin 4 x cos 4 x = sin 6 x
fi x= + ( -1) Á - ˜ - , n Œ I
2 Ë 8¯ 8
fi sin 8 x - sin 6 x = 0
8. Do yourself. fi 2 cos (7 x ) sin x = 0
The Trigonometric Equation 171

fi cos (7 x ) = 0 ,sin x = 0 Ê pˆ p
x = Á 2np + ˜ = (4n + 1) , n Œ I
p Ë 2¯ 2
fi (7 x ) = (2n + 1) , x = np
2 6. Given equation is
p
fi x = ( 2n + 1) , x = n p , n ΠI (1 + sin 2 x ) + 5 (sin x + cos x ) = 0
14
2. Given equation is fi (sin x + cos x )2 + 5 (sin x + cos x ) = 0

tan x tan 2 x
+ +2=0 fi ((sin x + cos x) + 5) (sin x + cos x) = 0
tan 2 x tan x
fi ((sin x + cos x) + 5) = 0 , (sin x + cos x) = 0
fi ( tan x + tan 2 x )2 = 0 fi (sin x + cos x ) = 0
fi ( tan x + tan 2 x ) = 0 fi tan x = -1
fi sin ( 2 x + x ) = 0 p
x = np - , n ŒI
fi sin (3x ) = 0 4
fi 3x = n p 7. Given equation is
sinx + sin 2x +sin 3x = cos x +cos 2x +cos 3x
np
fi x=
3
, n ŒI fi (sin 3x + sin x ) + sin 2 x = (cos 3x + cos x ) + cos 2 x
4. Given equation is fi 2 sin 2 x cos x + sin 2 x = 2 cos 2 x cos x + cos 2 x
cos x cos (6 x ) = -1 fi sin 2 x ( 2 cos x + 1) = cos 2 x ( 2 cos x + 1)

fi 2 cos (6 x ) cos x = -2 fi (sin 2 x - cos 2 x ) (2 cos x + 1) = 0


fi cos (7 x ) + cos (5 x ) = -2 fi (sin 2 x - cos 2 x ) = 0 , (2 cos x + 1) = 0
It is possible only when when (sin 2 x - cos 2 x ) = 0
cos (7 x ) = -1 , cos (5 x ) = -1 fi tan 2 x = 1

fi 7 x = ( 2k + 1) p , 5 x = ( 2m + 1) p p
fi 2 x = np +
4
p p
fi x = ( 2k + 1)
, x = ( 2m + 1) np p
7 5 fi x= +
when k = 3 and m = 2, then common 2 8
value of x is p. when 2 cos x + 1 = 0
Hence, the general solution is 1
fi cos x = -
x = 2n p + p = (2n + 1) p , n ΠI 2
5. Given equation is 2p
fi x = 2n p ±
3
cos ( 4 x ) + sin (5 x ) = 2
Hence, the solution is
It is possible only when
p 5p 9p 13p 2p 4p
cos (4 x ) = 1,sin (5 x ) = 1 x= , , , , ,
8 8 8 8 3 3
p 8. Given equation is
fi 4 x = 2k p , 5 x = (4 m + 1)
2
sin 3 x cos3 x
kp p + - 2 sin x cos x = 2
fi x= , x = (4m + 1) , k , m ΠI cos x sin x
2 10
when k = 1 and m = 1, then the sin 4 x + cos 4 x
fi = 2 sin x cos x + 2
p sin x cos x
common value of x is
2 fi 1 - 2 sin 2 x cos 2 x = 2 sin 2 x cos 2 x + sin (2 x )
Hence, the general solution is
fi 4 sin 2 x cos 2 x + sin (2 x ) - 1 = 0
172 Comprehensive Trigonometry with Challenging Problems & Solutions for Jee Main and Advanced

fi sin 2 2 x + sin ( 2 x ) - 1 = 0 fi (2 sin (2 x) + 1) = 0 , (sin (2 x) + 4) = 0


fi sin ( 2 x ) =
-1 ± 1 + 4 -1 ± 5
=
fi (2 sin (2 x) + 1) = 0
2 2 1
fi sin ( 2 x ) = -
-1 + 5 2
fi sin ( 2 x ) =
2 Ê pˆ
fi sin ( 2 x ) = sin Á - ˜
Ê -1 + 5 ˆ Ë 6¯
fi sin ( 2 x ) = sin a , a = sin -1 Á
Ë 2 ˜¯ nÊ pˆ
fi 2 x = np + (-1) Á - ˜
Ë 6¯
fi (2 x ) = n p + (-1)n a
np nÊ p ˆ
np na fi x= + (-1) Á - ˜ , n Œ I
fi x= + ( -1) 2 Ë 12 ¯
2 2
11. Given equation is
9. Given equation is
1
sin 4 x + cos 4 x = cos ( 4 x ) +
sin 4 x + cos x = 2 sin 4 x cos x
2 2 4
2
fi sin 2 4 x - 2 sin 4 x cos 4 x + cos 2 x = 0 1
fi 1 - 2 sin 2 x cos 2 x = cos ( 4 x ) +
2
(sin )
2
fi 2
4 x - cos 4 x + cos 2 x - cos8 x = 0
fi 2 - 4 sin 2 x cos 2 x = 2 cos (4 x ) + 1
(sin x) ( )
2
fi 2
4 x - cos 4 + cos 2 x 1 - cos6 x = 0 fi 2 - sin 2 2 x = 2 cos (4 x ) + 1
It is possible only when fi 2 cos ( 4 x ) + sin 2 2 x = 1
(sin 2
) (
4 x - cos 4 x = 0 , cos 2 x 1 - cos6 x = 0 ) fi ( )
2 1 - 2 sin 2 2 x + sin 2 ( 2 x ) = 1
Now, cos x = 0 , cos x = 1
2
3 sin 2 (2 x ) = 1

when cos x = 0 then x = ( 2n + 1) p 1
2 fi sin 2 ( 2 x ) =
3
Ê 1ˆ
So, sin 4 Á n + ˜ p = 0 fi
1
sin 2 ( 2 x ) = = sin 2 a
Ë 2¯ 3
which is true fi 2x = n p ± a
when cos 2 x = 1 , then x = n p
np a Ê 1ˆ
which will not satisfy the equation fi x= ± , n Œ I , a = sin -1 Á ˜
2 2 Ë 3¯
sin ( 4 x ) - cos 4 x = 0
12. Given equation is
p
Hence, the solution is x = ( 2n + 1) Ê pˆ 1
2 sin 4 x + sin 4 Á x + ˜ =
10. Given equation is Ë 4¯ 4
7 Ê pˆ
sin 4 x + cos 4 x = sin x cos x fi 4 sin 4 x + 4 sin 4 Á x + ˜ = 1
2 Ë 4¯
7 2
1 - 2 sin 2 x cos 2 x = sin ( 2 x ) Ê p ˆˆ

(2 sin x) Ê
2
4 fi 2
+ Á 2 sin 2 Á x + ˜ ˜ = 1
Ë Ë 4¯¯
fi 4 - 8 sin 2 x cos 2 x = 7 sin ( 2 x ) 2

fi 4 - 2 sin 2 x - 7 sin ( 2 x ) = 0
2 fi (1 - cos (2 x))2 + ÊÁË1 - cos ÊÁË 2 x + p2 ˆ˜¯ ˆ˜¯ =1

fi 2 sin 2 2 x + 7 sin ( 2 x ) - 4 = 0 fi (1 - cos (2 x))2 + (1 + sin (2 x))2 = 1


fi 2 sin 2 2 x + 8 sin ( 2 x ) - sin ( 2 x ) - 3 = 0 fi 1 - 2 cos ( 2 x ) + 1 + 2 sin (2 x ) = 0
fi (2 sin (2 x) + 1) (sin (2 x) + 4) = 0 fi 2 - 2 (cos ( 2 x ) - sin (2 x )) = 0
The Trigonometric Equation 173

It is possible only when


fi (cos (2 x) - sin (2 x)) = 1
Ê 1 1 ˆ 1 sin 2016 x = 1 , cos 2016 x = 0
fi ÁË cos ( 2 x ) - sin (2 x )˜ =
2 2 ¯ 2 fi sin x = 1 , cos x = 0
Ê pˆ 1 Hence, the solution is
fi cos Á 2 x + ˜ = p
Ë 4 ¯ 2 x = 2n p + , n Œ I
2
Ê pˆ p
fi ÁË 2 x + ˜¯ = 2np ± Thus, the number of solutions is 1.
4 4 17. Given equation is
p
fi x = np , x = np - , n ŒI x 2 + 2 x sin ( xy ) + 1 = 0
4
Ê pˆ fi ( x + sin ( xy ))2 + (1 - sin 2 ( xy )) = 0
13. We have a = cos Á x + ˜ + cos x
Ë 3¯ fi ( x + sin ( xy ))2 + cos2 ( xy ) = 0
Êpˆ Êpˆ It is possible only when,
= cos x cos Á ˜ - sin x sin Á ˜ + cos x
Ë 3¯ Ë 3¯
( x + sin ( xy ))2 = 0 , cos2 ( xy ) = 0
= 1 cos x - 3 sin x + cos x
2 2 fi ( x + sin ( xy )) = 0 , cos ( xy ) = 0
fi cos ( xy ) = 0
= 3 cos x - 3 sin x
p
x y = (2n + 1) , n ΠI
2 2

The equation will provide us a real solution if 2
3 3 p
cos x - sin x when x = 1, n = 0, then y =
2 2 2
fi 3p
- 3£a£ 3 when x = –1, n = 1, then y =
2
14. Let f ( x ) = cos x - x + 1 Hence, the number of ordered pairs are
2
Ê pˆ Ê 3p ˆ
Now, f (0) = 1 + 1 = 3 > 0 ÁË1, ˜¯ , ÁË -1, ˜¯
2 2 2 2
Êpˆ p 1 1 p 18. Given equation is
and f Á ˜ = 0 - + = - < 0 sin 5x . cos 3x = sin 6x . cos 2x
Ë 2¯ 2 2 2 2
By intermediate value theorem there is a root fi sin (5 x ) cos (3x ) = 2 sin (3x ) cos (3x ) cos ( 2 x )
Ê pˆ
lies in Á 0 , ˜ . fi (sin (5x) - 2 sin (3x) cos (2 x)) cos (3x) = 0
Ë 2¯
Hence, the number of roots is 1.
fi (sin (5x) - sin (5x) - sin ( x)) cos (3x) = 0
fi sin ( x ) cos (3x ) = 0
15. Now, cos ( xy ) tan ( xy ) = xy
fi sin ( x ) = 0 , cos (3x ) = 0
fi sin ( xy ) = xy
p
It is possible only when xy = 0 x = n p , x = ( 2n + 1) , n ΠI
6
fi x = 1 and y = 0 p p 5p
x = 0, p , , ,
Thus, the solution is (1, 0). 6 2 6
Hence, the number of integral ordered pairs is 1. Hence, the number of solutions is 5.
16. Given equation is 19. Given equation is
sin 2016 x - cos 2016 x = 1 cos 3x . tan 5x = sin 7x
fi fi cos (3x ) sin (5 x ) = sin (7 x ) cos (5 x )
sin 2016 x = cos 2016 x + 1
174 Comprehensive Trigonometry with Challenging Problems & Solutions for Jee Main and Advanced

2 cos (3x ) sin (5 x ) = 2 sin (7 x ) cos (5 x )



fi sin (8 x ) + sin ( 2 x ) = sin (12 x ) + sin (2 x )

Êpˆ
sin x + sin Á ˜
Ë 8¯ ( )
2 (1 - cos x ) = 0

fi sin (8 x ) = sin (12 x ) Êpˆ


fi sin x = - sin Á ˜ 2 (1 - cos x )
Ë 8¯
fi sin (12 x ) - sin (8 x ) = 0
2 cos (10 x ) sin (2 x ) = 0 Êpˆ
fi fi sin 2 x = 2 sin 2 Á ˜ (1 - cos x )
Ë 8¯
fi cos (10 x ) = 0 ,sin (2 x ) = 0
Ê 1 ˆ

p
10 x = ( 2n + 1) , 2 x = n p
fi sin 2 x = Á1 - ˜ (1 - cos x )
Ë 2¯
2
p np Ê xˆ Ê xˆ Ê 1 ˆ 2 Ê xˆ
fi x = ( 2n + 1) , x = , n ŒI fi 4 sin 2Á ˜ cos 2 Á ˜ = Á1 -
Ë 2¯ Ë 2¯ Ë ˜¯ 2 sin ÁË 2 ˜¯
20 2 2
p Ê xˆ Ê 1 ˆ
fi x = 0, fi 2 cos 2 Á ˜ = Á1 - ˜
20 Ë 2¯ Ë 2¯
Hence, the number of solutions is 2.
Ê 1 ˆ
20. Given equation is fi 1 + cos x = Á1 - ˜
Ë 2¯
(
2 tan x - l 1 + tan 2 x = 0 ) fi cos x = -
1
2
fi l tan x - 2 tan x + l = 0
2

Ê 3p ˆ
Let it has two roots, say, tan B and tan C fi cos x = cos Á ˜
Ë 4¯
2
Now, tan B + tan C = 3p
l fi x = 2n p ± , nŒI
fi tan B.tan C = 1 4
13 p
tan B + tan C Hence, the solution is x =
Now, tan ( B + C ) = 4
1 - tan B tan C 23. Given equation is
fi tan (p - A) = •
sin 4 x - (k + 2) sin 2 x - ( k + 3) = 0
p
fi (p - A) =
2 fi sin 4 x - 2 sin 2 x - 3 = k sin 2 x + 1 ( )
p sin 4 x - 2 sin 2 x - 3
fi A= fi k=
2
(sin 2
x +1 )
21. Given equation is
cos 4 x - (a + 2) cos 2 x - (a + 3) = 0 fi k=
(sin 2
)(
x + 1 sin 2 x - 3 )
(sin 2
x +1 )
fi (
cos 4 x - 2 cos 2 x - 3 = a 1 + cos 2 x )
fi (
k = sin 2 x - 3 )
fi ( )( ) (
cos 2 x - 3 cos 2 x + 1 = a 1 + cos 2 x ) fi k + 3 = sin 2 x
fi (cos 2
x - 3) = a fi 0 £ k + 3£1
fi -3 £ k £ -2
fi a + 3 = cos 2 x
Clearly, 0 £ a + 3 £ 1 24. Given equation is
x
= 26 sin x
2

fi -3 £ a £ -2 4.16sin
22. Given equation is fi x
= 43 sin x
2
4.42 sin
pÊ fi
(1 - cos x )2 + sin 2 x ˆ¯ = 0 41+ 2 sin x
= 43 sin x
2

sin x + sin

fi 1 + 2 sin 2 x = 3 sin x
The Trigonometric Equation 175

fi 2 sin 2 x - 3 sin x + 1 = 0 sin x + 2 cos x cos x cos x


fi 2 sin 2 x - 2 sin x - sin x + 1 = 0 sin x + 2 cos x sin x cos x = 0
sin x + 2 cos x cos x sin x
fi 2 sin x (sin x - 1) - (sin x - 1) = 0
fi (2 sin x -1) (sin x - 1) = 0 (C1 Æ C1 + C2 + C3 )
1 1 cos x cos x
fi sin x = ,1
2 (sin x + 2 cos x ) 1 sin x cos x = 0
p 5p p 1 cos x sin x
fi x= , ,
6 6 2
1 cos x cos x
Thus, the number of principal solutions is 3.
25. Given equation is (sin x + 2 cos x ) 0 sin x - cos x 0 =0
0 0 sin x - cos x
sec x = 1 + cos x + cos 2 x + cos3 x + ......

fi sec x =
1 (sin x + 2 cos x ) (sin x - cos x )2 = 0
1 - cos x tan x = 1, - 2
1 1 p
fi = So, there is only one solution x = in
cos x 1 - cos x 4
È p p˘
fi 2 cos x = 1 Í- 4 , 4 ˙
Î ˚
1
fi cos x = 4. Given equation is
2
p sin x + sin y = sin ( x + y )
fi x = 2n p ± , n ŒI
3 Ê x + yˆ Ê Ê x - yˆ Ê x + yˆˆ
- cos Á =0
Ë 2 ˜¯ ˜¯
cos Á
Ë 2 ˜¯
2 sin Á
Ë 2 ˜¯ ÁË
INTEGER TYPE QUESTIONS Ê x + yˆ Ê xˆ Ê yˆ
4 sin Á ˜ sin Á ˜ sin Á ˜ = 0
1. Given equation is Ë 2 ¯ Ë 2¯ Ë 2¯
3 sin 2 x - 7 sin x + 2 = 0 Ê x + yˆ Ê xˆ Ê yˆ
sin Á ˜ = 0 ,sin Á ˜ = 0 ,sin Á ˜ = 0
fi 3 sin x - 6 sin x - sin x + 2 = 0
2 Ë 2 ¯ Ë 2¯ Ë 2¯

fi 3 sin x (sin x - 2) - (sin x - 2) = 0 x + y = 0, x = 0, y = 0

fi (3 sin x - 1) (sin x - 2) = 0 It is also given that x + y = 1


1 when x = 0, then y = 1 fi y = ± 1
fi sin x = , 2
3
when y = 0, then x = 1 fi x = ± 1
1
fi sin x =
3 1 1
when y = - x , then x = fix = ±
Hence, the number of real solutions is 6. 2 2
2. Given equation is 1
and then y = ∓
2 cos x + 3 sin x = k + 1 2
fi - 13 £ ( k + 1) £ 13 Hence, the pairs of solutions are

(0,1) , (0, - 1) , (1, 0) , (-1, 0) , ÊÁË , - ˆ˜¯ , ÊÁË - , ˆ˜¯


1 1 1 1
fi - 13 - 1 £ k £ 13 - 1
2 2 2 2
fi k = –4, –3, –2, –1, 0, 1, 2 Thus, the numbers of pairs is 6
Hence, the number of integral values of k is 7 5. Given expression is
3. Given equation is
1 + sin 2 x cos 2 x 4 sin 2 x
sin x cos x cos x
cos x sin x cos x = 0 f ( x) = sin x
2
1 + cos x2
4 sin 2 x
cos x cos x sin x sin 2 x cos 2 x 1 + 4 sin 2 x
176 Comprehensive Trigonometry with Challenging Problems & Solutions for Jee Main and Advanced

1 + sin 2 x cos 2 x 4 sin 2 x ( )


4 cos3 2 x + 2 cos 2 2 x - 2 cos 2 x = 1
= sin x
2
1 + cos x2
4 sin 2 x 2 cos 2 2 x ( 2 cos 2 x + 1) = ( 2 cos 2 x + 1)
sin 2 x cos 2 x 1 + 4 sin 2 x
(2 cos 2
)
2 x - 1 ( 2 cos 2 x + 1) = 0
1 + sin x cos x 4 sin 2 x 4 cos 4 x ( 2 cos 2 x + 1) = 0
2 2

= -1
4 cos 4 x = 0 , ( 2 cos 2 x + 1) = 0
1 0
-1 0 1
1
cos 4 x = 0 , cos 2 x = -
Ê R2 Æ R2 - R1 ˆ 2
ÁË R Æ R - R ˜¯ p p
3 3 1 4 x = ( 4n + 1) , 2 x = 2np ±
(
= 4 sin 2 x + 1 + sin 2 x + cos 2 x ) p
2
p
3

= 4 sin 2 x + 2 x = (4n + 1) , x = np ± , n Œ I
8 6
So, the maximum value is 6. p 5p p 5p
x= , , ,
6 We have, -1 £ sin x £ 1 8 8 6 6
x Hence, the number of solutions is 4.
fi -1 £ £1 9. Given equation is sin x .cos y = 1
10
It is possible only when
fi -10 £ x £ 10
sin x = 1, cos y = 1

fi p 3p and y = 0, 2p
x= ,
2 2
-3p -2p -p p 2p 3p
Also, when sin x = -1, cos y = -1
3p
fi x=,y=p
2
Clearly, the number of solutions is 6.
Hence, the number of ordered pairs is 5
7. Given equation is
tan x + cot x = 2 cosec x Êp ˆ Êp ˆ Ê 3p ˆ Ê 3p ˆ Ê 3p ˆ
i.e. Á , 0˜ , Á , 2p ˜ , Á , 0˜ , Á , p ˜ , Á , 2p ˜
Ë2 ¯ Ë2 ¯ Ë 2 ¯ Ë 2 ¯ Ë 2 ¯
1 2
fi = 10. When cot x is positive
sin x cos x sin x
The equation becomes
1
fi cos x = 1
2 cot x = cot x +
sin x
fi p 5p
x =±
,± cosec x = 0
3 3
Thus, the number of solutions is 4. It is not possible,
8. Given equation is when cot x is negative.
The given equation becomes
1
cos x .cos 2 x .cos 3x = 1
4 fi - cot x = cot x +
sin x
2 (2 cos 3x cos x ) cos 2 x = 1
1
2 (cos 4 x + cos 2 x ) cos 2 x = 1 fi 2 cot x + =0
sin x
2 (cos 4 x ) cos 2 x + 2 cos 2 2 x = 1 fi 2 cos x + 1 = 0

( )
2 2 cos 2 2 x - 1 cos 2 x + 2 cos 2 2 x = 1 fi cos x = -
1
2
The Trigonometric Equation 177

2p x = 1.
fi x = 2n p ± , n ŒI Thus, the number of solutions is 1.
3
2p 4p 15. Here, 1 £ sin ( 2 x ) + cos ( 2 x ) £ 2 and sin ( y ) £ 1
fi x= ,
3 3 It is possible only when sin ( y ) = 1
Hence, the number of solutions is 2.
fi sin( y ) = ±1
11. Given equation is
p 3p
tan ( 4 x ) tan x = 1 fi y=± ,± .
2 2
cos ( 4 x ) cos x - sin ( 4 x ) sin x = 0 Thus, the number of values of y is 4
cos (5 x ) = 0
PAST IIT-JEE QUESTIONS
p
5 x = ( 2n + 1) 1. Ans. (a)
2
The given equation is sin x + cos x = 1
p
x = ( 2n + 1) , n ΠI 1 1 1
10 fi sin x + cos x =
2 2 2
p 3p 5p 7p 9 p
x= , , , , Ê pˆ Êpˆ
10 10 10 10 10 fi cos Á x - ˜ = cos Á ˜
Hence, the number of solutions is 5. Ë 4¯ Ë 4¯
12. Given equation is Ê pˆ Ê pˆ
sin x (sin x + cos x ) = n fi ÁË x - ˜¯ = ÁË 2np ± ˜¯
4 4
fi n = sin x (sin x + cos x ) Ê p pˆ
fi x = Á 2np ± + ˜
= sin 2 x + sin x cos x Ë 4 4¯

1 - cos ( 2 x ) sin ( 2 x ) p
= + fi x = 2np , 2np +
2 2 2
p
fi sin ( 2 x ) - cos ( 2 x ) = 2n - 1 But x = 2np + does not satisfy the given equation.
2
fi - 2 £ 2n - 1 £ 2 Therefore, the solution is x = 2np , n Œ I
fi 1- 2 1+ 2 2. We have cos x = sin 3x
£n£
2 2 Êp ˆ
fi n = 0, 1 cos x = sin 3x = cos Á - 3x˜
Ë2 ¯
Hence, the number of integral values of n is 2.
13. Given equation is Êp ˆ
fi x = 2 n p ± Á - 3 x˜
sin{x} = cos{x} Ë2 ¯
fi tan{x} = 1 Taking +ve sign, we get,
p p p p p p Êp ˆ
fi x= ,1 + , 2 + , 3 + , 4 + , 5 + x = 2 n p + Á - 3 x˜
4 4 4 4 4 4 Ë2 ¯
Hence, the number of solutions is 6. p
14. Given equation is fi 4x = 2 n p +
2
( ) +( )
2x 2x
3 +1 3 -1 = 23 x 2 np p
fi x= +
4 8
( 3 + 1) + ( 3 - 1) ( )
2x 2x 2x
fi = 2 2 np p
fi x= + , n ŒI
2x 2x 2 8
Ê 3 + 1ˆ Ê 3 - 1ˆ
fi Á ˜ +Á ˜ =1 Taking –ve sign, we get,
Ë 2 2 ¯ Ë 2 2 ¯
Êp ˆ
2x 2x fi x = 2 n p - Á - 3 x˜
Ê Ê 5p ˆ ˆ Ê Ê 5p ˆ ˆ Ë2 ¯
fi ÁË sin ÁË 12 ˜¯ ˜¯ + Á cos Á ˜ ˜ =1
Ë Ë 12 ¯ ¯
178 Comprehensive Trigonometry with Challenging Problems & Solutions for Jee Main and Advanced

p Êpˆ Ê x - yˆ 3
fi -2 x = 2np - fi 2 cos Á ˜ cos Á =
2 Ë 3¯ Ë 2 ˜¯ 2
p 1 Ê x - yˆ 3
fi x = -n p + , n ŒI fi 2¥ ¥ cos Á =
4 2 Ë 2 ˜¯ 2
p p p 3p p
As - £ x £ , x = , - , Ê x - yˆ 3
fi cos Á = .
2 2 8 8 4 Ë 2 ˜¯ 2
Thus, the point of intersections of two curves are
It is not possible, so the solution set is x = j .
Êp Ê p ˆ ˆ Ê 3p Ê 3p ˆ ˆ Ê p 1 ˆ 7. The given inequation is
ÁË 8 , cos ÁË 8 ˜¯ ˜¯ , ÁË - 8 , cos ÁË 8 ˜¯ ˜¯ , ÁË 4 , ˜
2¯ 2 sin 2 x - 3 sin x + 1 ≥ 0
3. The given equation is
fi (2 sin x - 1) (sin x - 1) ≥ 0
4 cos 2 x sin x - 2 sin 2 x = 3 sin x 1
fi sin x £ & sin x ≥ 1
fi ( )
4 1 - sin x sin x - 2 sin x = 3 sin x
2 2

1
2

fi 4 sin x - 4 sin 3 x - 2 sin 2 x = 3 sin x fi sin x £ & sin x = 1


2
fi sin x - 4 sin 3 x - 2 sin 2 x = 0 È p ˘ È 5p ˘ p
x Œ Í0, ˙ » Í , p ˙ and x =
Î 6 ˚ Î 6 ˚ 2
fi 4 sin 3 x + 2 sin 2 x - sin x = 0
Hence, the solution set is
fi (
sin x 4 sin 2 x + 2 sin x - 1 = 0 ) È p ˘ È 5p ˘ Ï p ¸
x Œ Í0, ˙ » Í , p ˙ » Ì ˝ .
fi (
sin x = 0, 4 sin 2 x + 2 sin x - 1 = 0 ) Î 6˚ Î 6
8. Ans. (c)
˚ Ó2˛

-2 ± 2 5 As we know that tan x ≥ x


fi sin x = 0, sin x =
8 Ê pˆ
So there is no root between Á 0, ˜
-1 ± 5 Ë 2¯
fi sin x = 0, sin x =
4 Êp ˆ Ê 3p ˆ
ÁË , p ˜¯ and ÁË , 2p ˜¯
n Ê -1 ± 5 ˆ 2 2
fi x = np , x = n p + ( -1) sin -1 Á
Ë 4 ˜¯ Ê 3p ˆ
But there is a root in Á p , ˜.
where n Œ I . Ë 2 ¯
4. The given equation is 9. Ans. (b)
The given equation is
sin 4 q - 2 sin 2 q - 1 = 0
sin x - 3 sin 2 x + sin 3x = cos x - 3 cos 2 x + cos 3x
( )
2
fi sin 2 q - 1 = 2 fi (sin x + sin 3x ) - 3 sin 2 x
= (cos x + cos 3x ) - 3 cos 2 x
fi (sin
q - 1) = ± 2
2
fi 2 sin 2 x cos x - 3 sin 2 x
fi sin q = (1 ± 2 )
2
= 2 cos 2 x cos x - 3 cos 2 x
fi (2 cos x - 3) (sin 2 x - cos 2 x ) = 0
fi sin q = (1 + 2 ) , (1 - 2 )
2

fi (sin 2 x - cos 2 x ) = 0, (∵ 2 cos x - 3 π 0)


since (1 + 2 ) > 1 and (1 - 2 ) < 0 , so tan 2 x = 1
there is no value of q satisfying the given equation. Êpˆ
fi tan 2 x = 1 = tan Á ˜
5. No questions asked in 1985. Ë 4¯
3 Ê pˆ
6. We have cos x + cos y = 2x = Á n p + ˜ , n Œ I
fi Ë
2 4¯
Ê x + y ˆ Ê x - y ˆ 3 Ê np p ˆ
cos Á =
Ë 2 ˜¯ 2
2 cos Á ˜ x=Á + ˜ , n ŒI
Ë 2 ¯ fi Ë 2 8¯
The Trigonometric Equation 179

10. No questions asked in between 1990 to 1992. p


11. Ans. (c) Let q =
2n
The given equation is The given equation reduces to
tan x + sec x = 2 cos x
n
fi sin x + 1 = 2 cos 2 x sin q + cos q =
2
fi (
sin x + 1 = 2 1 - sin 2 x ) fi (sin q + cos q )2 =
n
fi (sin x + 1) = 2 (1 - sin x ) (1 + sin x ) 4
n
fi (sin x + 1) (1 - 2 (1 - sin x )) = 0 fi 1 + 2 sin q cos q =
4
1 n Ê n - 4ˆ
fi sin x = -1, sin x = sin 2q = - 1 = Á
2 fi Ë 4 ˜¯
4
p 5p 3p
fi x= , , . As per choices, n ≥ 4
6 6 2
3p p
But x = does not satisfy the given equation. fi 0 < 2q <
2 2
Hence, the number of solutions is 2. fi 0 < sin 2q < 1
12. The given equation can be written as
tan( x + 100∞) cot x = tan( x + 50∞) tan( x - 50∞) Ê n - 4ˆ
fi 0<Á <1
Ë 4 ˜¯
sin( x + 100∞) cos x
fi fi 0 < ( n - 4) < 4
cos( x + 100∞)sin x
sin( x - 50∞)sin( x - 50∞) fi 4<n<8
= 14. The given in-equations are
cos( x + 50∞) cos( x - 50∞)
Applying compenendo and dividendo, we get, 2 sin 2 x + 3 sin x - 2 ≥ 0 and x 2 - x - 2 < 0


sin( x + 100∞) cos x + cos( x + 100∞)sin x fi (2 sin x - 1) (sin x + 2) ≥ 0
sin( x + 100∞) cos x - cos( x + 100∞) sin x (2 sin x - 1) ≥ 0 (∵ sin x + 2 > 0, " x Œ R )

sin( x + 50∞)sin( x - 50∞) + cos( x + 50∞) cos( x - 50∞)
= 1
sin( x + 50∞)sin( x - 50∞) - cos( x + 50∞) cos( x - 50∞) fi sin x ≥
2
sin( x + 100∞ + x) p 5p
fi £x£
sin( x + 100∞ - x) fi 6 6 .......(i)
cos ( x + 50∞ - x + 50∞) Also, x 2 - x - 2 < 0
= - cos x + 50∞ + x - 50∞
( ) ( x - 2) ( x + 1) < 0

fi sin(2 x + 100∞) cos 2 x = - sin(100∞) cos(100∞)
fi -1 < x < 2 ......(ii)
fi 2 sin(2 x + 100∞) cos 2 x = -2 sin(100∞) cos(100∞)
From (i) and (ii), we get,
fi sin(4 x + 100∞) + sin(100∞) = - sin(200∞)
Êp ˆ
fi sin(4 x + 100∞) = -(sin(200∞) + sin(100∞)) x ŒÁ , 2˜
Ë6 ¯
fi sin(4 x + 100∞) = -2 sin(150∞) cos(50∞)
15. The given equation is
1
fi sin(4 x + 100∞) = -2 ¥ ¥ sin(40∞) cos ( p sin x ) = sin ( p cos x )
2
Êp ˆ
fi sin(4 x + 100∞) = - sin(40∞) = sin(220∞) fi cos ( p sin x ) = cos Á - p cos x˜
Ë2 ¯
fi (4 x + 100∞) = (220∞)
p
fi 4 x = 120∞ fi ( p sin x ) = 2np ± ÊÁË ˆ
- p cos x˜
¯
2
fi x = 30∞
p
Hence, the result. fi p ( sin x + cos x ) = 2np ±
13. Ans. (d) 2
180 Comprehensive Trigonometry with Challenging Problems & Solutions for Jee Main and Advanced

Ê pˆ p fi x 2 - 3x = 0
fi 2 p sin Á x + ˜ = 2np ±
Ë 4¯ 2 fi x = 0, 3
Ê pˆ
As we know that, -1 £ sin Á x + ˜ £ 1 fi tan 2 q = 0, 3
Ë 4¯
pˆ tan q = 0 and tan q = ± 3
Ê
- 2 p £ 2 p sin Á x + ˜ £ 2 p
Ë 4¯ p
fi q = n p , q = np ± , n ŒI
fi p 3
- 2 p £ 2n p ± £ 2p
2 18 The given equation is cos7 x + sin 4 x = 1
fi p
2 p ≥ 2n p ± cos7 x = 1 - sin 4 x
2
fi p
2 p ≥ 2n p + , 2n p -
p fi (
cos7 x = 1 - sin 2 x 1 + sin 2 x)( )
2 2
As we require smallest +ve value of p, so we consider, fi (
cos7 x = cos 2 x 2 - cos 2 x )
x (cos x - ( 2 - cos x )) = 0
p
2p = fi cos 2 5 2
2
p fi cos 2 x = 0, (cos x - ( 2 - cos x )) = 0
5 2
fi p= .
2 2
fi cos 2 x = 0, cos5 x + cos 2 x = 2
p
For this value of p, x = is a solution of the given fi cos x = 0, cos5 x + cos 2 x = 2
equation. 4
16. The given equation can be written as fi cos x = 0, cos x = 1
p
(1 - tan q ) (1 + tan q ) + 2 = 0
2 2 tan 2 q fi x = ± , 0.
2
fi (1 - tan q ) (1 + tan q ) + 2
2 2 tan 2 q
=0 Ï p ¸
Hence, the real roots are ̱ , 0 ˝ .
Ó 2 ˛
fi (1 - tan q ) + 2 = 0
4 tan 2 q
19. The given equation is

fi 2 = ( tan q - 1)
tan 2 q 4 fi 3 sin 2 x - 7 sin x + 2 = 0
(3 sin x - 1) (sin x - 2) = 0
2 = ( x - 1) , where x = tan
x fi
fi 2 2
q
1
It is true for x = 3 fi sin x = , 2
3
Thus, tan 2 q = 3 1
fi sin x =
( 3)
2
tan 2 q = 3

There is 2 solutions in its period 2p.
fi tanq = ± ( 3 ) So, the number of solutions is 6.
n
20. We have sin ( nq ) = Â br sin q
r
Êpˆ
fi q = ±Á ˜ .
Ë 3¯ r =0

17. The given equation is fi sin ( nq ) = ÈÎb0 + b1 sin q + b2 sin 2 q


tan 2 q + sec 2q = 1 +b3 sin 3 q + ...... + bn sin n q ˚˘
Put q = 0 , then b0 = 0
1 + tan 2 q
fi tan q +2
=1 Thus,
1 - tan 2 q
1+ x sin ( nq ) = b1 sin q + b2 sin 2 q + ...... + bn sin n q
fi x+ = 1, where x = tan 2 q
1- x
sin ( nq )
fi x - x2 + 1 + x = 1 - x fi = b1 + b2 sin q + ...... + bn sin n-1 q
sin q
The Trigonometric Equation 181

Taking limit q Æ 0 , we get, Ê1- 5ˆ Ê1+ 5ˆ


b1 = n fi 2 sin t £ Á ˜ , 2 sin t ≥ Á ˜
Ë 2 ¯ Ë 2 ¯
Therefore, b0 = 0, b1 = n . Ê1- 5ˆ Ê1+ 5ˆ
fi sin t £ Á ˜ , sin t ≥ Á ˜
22. We have - 74 £ 7 cos x + 5 sin x £ 74 Ë 4 ¯ Ë 4 ¯
fi - 74 £ ( 2k + 1) £ 74 Ê pˆ Ê 3p ˆ
fi sin t £ sin Á - ˜ , sin t ≥ sin Á ˜
fi Ë 10 ¯ Ë 10 ¯
- 74 - 1 £ 2k £ 74 - 1
p p
fi (
-8 - 1 £ 2k £ 8 - 1 ∵ 74 < 9 ) As -
2
£ t £ , we get,
2
fi -9 £ 2k £ 7 p p 3p p
fi - £ t £ - and £t£
fi -4.5 £ k £ 3.5 2 10 10 2
fi k = –4, –3, –2, –1, 0, 1, 2, 3. È p p ˘ È 3p p ˘
Thus, t Œ Í - , - ˙ » Í , ˙ .
Thus, the number of integral values of k is 8. Î 2 10 ˚ Î 10 2 ˚
23. No questions asked in between 2003 to 2004. 26. The given in-equation is
24. Ans. (d). 2 sin 2 q - 5 sin q + 2 > 0
Given -p £ a, b £ p
fi (2 sin q - 1) (sin q - 2) > 0
fi -p £ a £ p , - p £ b £ p
fi (2 sinq - 1) < 0
fi -p £ a £ p , - p £ -b £ p
1
fi -2p £ a - b £ 2p fi sinq <
2
Given cos (a - b ) = 1 Ê p ˆ Ê 5p ˆ
fi q ŒÁ 0, ˜ » Á , 2p ˜ .
a=b Ë 6¯ Ë 6 ¯
1 27. The given equations are
Also, cos (a + b ) = 2 sin 2 q - cos 2q = 0 and 2 cos 2 q - 3 sin q = 0
e
fi cos (2a ) =
1
e
( )
Now, 2 sin 2 q - 1 - 2 sin 2 q = 0

It has one solution in its period p. fi 4 sin q = 1


2

So it has 4 solutions in [0 , 4p ] . Ê 1ˆ
2
Êpˆ
fi sin 2 q = Á ˜ = sin 2 Á ˜
Ë 2¯ Ë 6¯
5x2 - 2 x + 1
25. Let y = p
3x 2 - 2 x - 1 fi q = np ±
6
fi (3 y - 5) x 2 - 2 ( y - 1) x - ( y + 1) = 0 fi
p 5p 7p 11p
q= , , ,
As x is real, so 6 6 6 6
( y - 1)2 + (3 y - 5) ( y + 1) ≥ 0 Also, 2 cos 2 q - 3 sin q = 0
fi y2 - 2 y + 1 + 3y2 - 2 y - 5 ≥ 0 fi 2 - 2 sin 2 q - 3 sin q = 0
fi 4 y2 - 4 y - 4 ≥ 0 fi 2 sin 2 q + 3 sin q - 2 = 0
fi y2 - y - 1 ≥ 0 fi (2 sin q - 1) (sin q + 2) = 0
Ê Ê1- 5ˆˆ Ê Ê1+ 5ˆˆ fi
1
sin q = , 2
fi Áy-Á ˜ ˜Áy-Á ˜˜ ≥ 0 2
Ë Ë 2 ¯¯ Ë Ë 2 ¯¯
1
Ê1- 5ˆ Ê1+ 5ˆ fi sinq =
y£Á , y≥Á 2
fi ˜ ˜
Ë 2 ¯ Ë 2 ¯ p 5p
fi q= , .
6 6
182 Comprehensive Trigonometry with Challenging Problems & Solutions for Jee Main and Advanced

p 5p Ï p 5p ¸
Hence, the solutions are q = , . Hence, the solutions are Ì , ˝.
6 6 Ó12 12 ˛
Therefore, the number of solutions is 2.
30. We have tan q = cot 5q
28. The given equation is sin q = cos j
sin q cos 5q
Êp ˆ =
cos Á - q ˜ = cos j cos q sin 5q
Ë2 ¯
fi 2 cos 5q cos q = 2 sin 5q sin q
Êp ˆ fi cos 6q + cos 4q = cos 4q - cos 6q
fi ÁË - q ˜¯ = 2np ± j , n Œ I
2 fi 2 cos 6q = 0
Ê pˆ fi cos 6q = 0
fi -2np = Á q ± j - ˜
Ë 2¯ 5p 3p p
fi 6q = ± , ± ,±
Ê pˆ 2 2 2
fi ÁË q ± j - ˜¯ = -2n p 5p 3p p
2 fi q = ± ,± ,±
12 12 12
1Ê pˆ
fi ÁË q ± j - ˜¯ = -2 n 5p p p
p 2 fi q = ± ,± ,±
12 4 12
1Ê pˆ
Thus, ÁË q ± j - ˜¯ is an even integer. Also, sin ( 2q ) = cos (4q )
p 2
29. We have Êp ˆ
fi cos (4q ) = cos Á - 2q ˜
Ë2 ¯
6 Ê (m - 1) p ˆ Ê mp ˆ
 cosec Á q +
Ë 4 ˜
¯
cosec Á q +
Ë ˜ =4 2
4 ¯ fi
Êp ˆ
4q = 2np ± Á - 2q ˜
m =1
6
Ë2 ¯
1
fi [Â =4 2 Taking +ve sign, we get,
m =1 Ê
sin Á q +
( m - 1) p ˆ Ê mp ˆ
Ë 4 ˜¯ sin ÁË q + 4 ˜¯ Êp ˆ
4q = 2np + Á - 2q ˜
Ë2 ¯
ÊÊ
sin Á Á q +
mp ˆ Ê
- Áq +
(m - 1) p ˆ ˆ
˜ ˜¯ ˜¯ p
6
ËË 4 ¯ Ë 4 fi 6q = 2n p +
fi Â Ê ( m - 1) p ˆ Ê mp ˆ
=4 2
m =1
sin Á q + ˜¯ sin ÁË q + 4 ˜¯ np p
Ë 4 fi q= +
3 12
6
Ê Ê Ê m - 1ˆ ˆ ˆ
fi  ÁË cot ÁË q + ÁË 4 ˜¯ p ˜¯ - cot (q + mp )˜¯ = 4 fi q=
p 5p p
, ,-
m =1 12 12 4
Ê Ê 6p ˆ ˆ Taking –ve sign, we get,
fi ÁË cot q - cot ÁË q + 4 ˜¯ ˜¯ = 4

(cot q + tan q ) = 4 Êp ˆ
fi 4q = 2np - Á - 2q ˜
Ë2 ¯
1
fi =4 p
sin q cos q fi 2q = 2np -
2
1
fi =2 p
2 sin q cos q fi q = np -
4
1
fi =2 p
sin 2q fi q = np -
1 4
fi sin 2q = p 5p p
2 Hence, the solutions are q = , , - .
12 12 4
p 5p p
fi 2q = , 31. Let = q
6 6 n
p 5p Then the given equation becomes
fi q= , .
12 12
The Trigonometric Equation 183

1 1 1 sin x È3 - ( 2 cos x - 1) ˘ = 3
2
= + fi Î ˚
sin q sin ( 2q ) sin (3q )
It is possible only when
1 1 1
fi - = sin x = 1 and (2 cos x - 1) = 0
sin q sin (3q ) sin ( 2q )
p p
sin 3q - sin q 1 fi x= and x = .
fi = 2 3
sin q sin (3q ) sin ( 2q )
Thus, the values of x does not saitsfy the given
2 cos 2q sin q 1 equation.
fi =
sin q sin 3q sin 2q 34. Ans. 8
2 cos 2q 1 The given equation can be written as
fi =
sin 3q sin 2q 5
cos 2 2 x + cos 4 x + sin 4 x + cos6 x + sin 6 x = 2
fi sin 4q = sin 3q 4
fi sin 4q = sin (p - 3q ) fi
5
( )
cos 2 2 x + 1 - 2 sin 2 x cos 2 x
( )
4
+ 1 - 3 sin 2 x cos 2 x = 2
fi 4q = p - 3q
fi 7q = p 5 Ê 1 ˆ
fi cos 2 2 x + Á1 - sin 2 2 x˜
4 Ë 2 ¯
p Ê 3 ˆ
fi q= + Á1 - sin 2 2 x˜ = 2
7 Ë 4 ¯
p p 5 5 2
fi = fi cos 2 x - sin 2 x = 0
2
n 7 4 4
fi n = 7. fi cos 2 x - sin 2 2 x = 0
2

Hence, the integral value of n is 7. fi cos 2 2 x = sin 2 2 x


32. No questions asked in between 2012 to 2013.
33. Ans. (d) fi tan 2 2 x = 1
The given equation is p
fi 2 x = np ± , n Œ I
sin x + 2 sin 2 x - sin 3x = 3 4
np p
fi sin x + 4 sin x cos x - 3 sin x + 4 sin 3 x = 3 fi x= ± , n ŒI .
2 8

Î ( )
sin x È-2 + 4 cos x + 4 1 - cos 2 x ˘ = 3 .
˚
Hence, the solutions are
p 3p 5p 7p 9p 11p 13p 15p
fi sin x ÈÎ2 - ( 4 cos 2 x - 4 cos x + 1) + 1˘˚ = 3 x= , , , , , , , .
8 8 8 8 8 8 8 8
CHAPTER 4
Trigonometric In-Equation

4.1 TRIGONOMETRIC INEQUALITIES Ex-1. Solve sin x > 1/2.


Soln. Here, we should construct the graph of y = sinx and
Suppose we have to solve f (x) > k or f (x) < k. y = 1/2.
When we solve the inequation, we often use the graphs Y
of the functions y = f (x) and y = k.
Then, the solution of the inequality f (x) > k is the values
of x for which the point (x, f (x)) of the graph of y = f (x) lies y = 1/2
above the striaght line y = k.
X¢ X
Y O p 5p
6 6
y=k

X¢ X Y¢
O
Hence, the solution set is
Y¢ Ê p 5p ˆ
x = ∪ Á 2np + , 2np + ˜ .
nŒI Ë 6 6¯
Similarly, when we solve f (x) < k, then the solution of the
inequalition f (x) < k is the values of x for which the point (x, Ex-2: Solve: sin x > 1/3.
f (x)) of the graph of y = f (x) lies below the striaght line y = k.
Soln. Here, we should construct the graph of y = sin x
Y
Ê 1ˆ
and y = sin -1 Á ˜ .
y=k Ë 3¯
X¢ X Y
O


y = 1/3

Type - I: An inequation is of the form sin x > k. X¢


O
X
Ê 1ˆ Ê 1ˆ
sin-1 Á ˜ p - sin -1 Á ˜
Rule: Find the smallest values of x Ë 3¯ Ë 3¯
inequation and then add 2np with that values of x.
Y Y¢

y=k Hence, the solution set is


Ê Ê 1ˆ Ê 1ˆ ˆ

O
X x = ∪ Á 2np + sin -1 Á ˜ , ( 2n + 1) p - sin -1 Á ˜ ˜
nŒI Ë Ë 3¯ Ë 3¯ ¯


Trigonometric In-Equation 185

Ex-3. Solve: sin x ≥ 1. Ex-2. Solve: sin x < 1/5.


Soln. We have sin x ≥ 1. Soln. Here, we should draw the graphs of y = sin x and
Y y = 1/5.
Y

y=1
0 y = 1/2
X¢ X
p/2 5p/2 X¢ X
Ê 1ˆ 0 Ê 1ˆ
-p sin-1 Á ˜ sin-1 Á ˜
Ë 5¯ Ë 5¯



p Hence, the solution set is
fi x = ( 4n + 1) , n ŒI .
Ê -1 Ê 1 ˆ ˆ
Á ( 2n - 1) p - sin ÁË 5 ˜¯ , ˜
2
Ex-4. Solve: sin x > 0 x= ∪ Á ˜
Soln. We have, sin x > 0 nŒI Á -1 Ê 1 ˆ ˜
Y
ÁË 2np + sin ÁË 5 ˜¯ ˜¯

y=1 3
+ + + Ex-3. Solve: sin x £
X¢ X 2
0 p
y = -1 Soln. Here, we should draw the graphs of y = sin x

3
and y =
2
x = ∪ ( 2 n p , (2n + 1) p )
Y

nŒI

Type - II: An in-equation is of the form sin x < k. y=


3
2
Rule: Find the smallest values of x
X¢ X
in-equation and then add 2np with that values of x. -
4p 0 p
Y 3 3


y=k
Hence, the solution set is
X¢ X È 4p p˘
x = ∪ Í2np - , 2np + ˙
nŒI Î 3 3˚

Type - III: An in-equation is of the form cos x > k.



Rule: x
Ex-1. Solve: sin x < 1/2
the given in-equation and then add 2np with each
Soln. Here, we should draw the graph of y = sinx and
values of x.
y = 1/2
Y
Y

y=k
y = 1/2 X¢ X
O
X¢ X
7p 0 p
-
6 6

1
Y¢ Ex-1. Solve: cos x > .
2
Hence, the solution set is Soln. Here, we should draw the graphs of y = cos x
Ê 7p pˆ 1
x = ∪ Á 2 np - , 2 np + ˜ . and y = .
nŒI Ë 6 6¯ 2
186 Comprehensive Trigonometry with Challenging Problems & Solutions for Jee Main and Advanced

Y Hence, the solution set is


Ê -1 Ê 1 ˆ ˆ
Á 2np + cos ÁË 3 ˜¯ ,
1
y= ˜
2
X¢ X
Á ˜
p O p Á -1 Ê 1 ˆ ˜
-
4 4 x = ∪ Á 2 ( n + 1) p - cos Á ˜ ˜ .
nŒI Ë 3¯
Á ˜

Á ˜
ÁË ¯˜
Hence, the solution set is
Ê p pˆ Ex-2. Solve: cos x £
3
x = ∪ Á 2np - , 2np + ˜ . .
nŒI Ë 4 4¯ 2
Soln. Here, we should draw the graphs of y = cos x
Ex-2. Solve: cos x ≥ 1 .
2 3
and y = .
Soln. Here, we should draw the graphs of y = cos x and 2
y = 1/2 Y
Y
3
y=
2
y = 1/2 X¢ X
O p p
2p -
X¢ p p X 6 6
O
-
3 3


Hence, the solution set is
Hence, the solution set is È p p˘
x = ∪ Í2np + , 2 ( n + 1) p - ˙ .
È p p˘ nŒI Î 6 6˚
x = ∪ Í2np - , 2np + ˙ .
nŒI Î 3 3˚
Type - V: An in-equation is of the form tan x > k.
Type - IV: An in-equation is of the form cos x < k. Rule: x
Rule: x the given in equation and then add np with each
the given inequation and then add 2np with each values of x.
values of x. Y
Y

y=k
y=k
X¢ X X¢ p p X
O cos-1 - O
2p - cos-1
2 2
(k) (k)


1 Y¢
Ex-1. Solve: cos x < .
3 Ex-1. Solve: tan x > 1.
Soln. Here, we should draw the graphs of y = cos x Soln. Here, we should draw the graphs of y = tan x
1 and y = 1.
and y = . Y
3
Y

y=1
y =1/3
X¢ X X¢ p p p X
O ËÁ ¯˜ - O
1 2 4 2
ˆ1ˆ -1
2p - cos Á ˜
cos-1 ˜ ˜ Ë 3¯
¯3¯



Trigonometric In-Equation 187

Hence, the solution set is Ex-1. Solve: tan x < 1.


Ê p pˆ Soln. Here, we should draw the graphs of y = tan x
x = ∪ Á np + , n p + ˜ .
nŒI Ë 4 2¯ and y = 1.
Y
1
Ex-2. Solve: tan x ≥ .
3
Soln. Here, we should draw the graphs of y = tan x y=1
1
and y = . X¢ X
3 p O p p
-
Y 2 4 2

1
y=
3 Y¢
X¢ p p p X Hence, the solution set is
- O
2 6 2 p pˆ
Ê
x = ∪ Á np - , np + ˜ .
nŒI Ë 2 4¯
Ex-2. Solve: tan x £ 3 .

Hence, the solution set is Soln. Here, we should draw the graphs of y = tan x
È p pˆ and y = 3
x = ∪ Ínp + , n p + ˜ Y
nŒI Î 6 2¯
Ex-3. Solve: tan x > 2.
Soln. Here, we should draw the graphs of y = tanx
y= 3
and y = 2.
Y X¢ X
p O p p
-
2 3 2

y=2
p
2
X¢ p X
- O y = tan-1 (2) Y¢
2
Hence, the solution set is
È p p˘
x = ∪ Ínp - , np + ˙ .
Y¢ nŒI Î 2 3˚
Hence, the solution set is
Ê pˆ
x = ∪ Á np + tan -1 ( 2) , n p + ˜ . 4.2 SOME SOLVED EXAMPLES
nŒI Ë 2¯
Ex-1. Solve: sin x > cos x.
Type -VI: An inequation is of the form tan x < k. Soln. We have sin x > cos x
Rule: x fi sin x – cos x > 0
the given in equation and then add np with each 1 1
fi sin x – cos x > 0
values of x. 2 2
Y
Ê pˆ
fi sin Á x - ˜ > 0
Ë 4¯
y=k
Ê p pˆ
fi x ŒÁ 2np + , (2n + 1) p + ˜
X¢ p O p X Ë 4 4¯
-
2 2
Hence, the solution set is
Ê p pˆ
x = ∪ Á 2np + , (2n + 1) p + ˜ .
nŒI Ë 4 4¯

188 Comprehensive Trigonometry with Challenging Problems & Solutions for Jee Main and Advanced

Ex-2. Solve: cos x > sin x. Soln. We have, sin x sin 2x < sin 3x sin 4 x
Soln. We have cos x > sin x fi 2 sin x sin 2x < 2 sin 3x sin 4 x
fi cos x – sin x > 0 fi cos x – cos 3x < cos x – cos 7x
1 1 fi cos 3x > cos 7x
fi cos x – sin x > 0
2 2 fi cos 3x – cos 7x > 0
Ê pˆ
fi cos Á x + ˜ > 0 fi 2 sin 5x sin 2x > 0
Ë 4¯
fi sin 5 x > 0 (since sin 2x is +ve for 0 < x < p/2)
Ê 3p pˆ
fi x ŒÁ 2np - , 2np + ˜ fi 0 < 5x < p
Ë 4 4¯
p
Hence, the solution set is fi 0< x<
5
Ê 3p pˆ Hence, the solution set is
x = ∪ Á 2np - , 2np + ˜ .
nŒI Ë 4 4¯
Ê pˆ Ê pˆ
1 1 x = Á 0, ˜ » Á 0, ˜
Ex-3. Solve: - £ cos x < . Ë 5¯ Ë 2¯
2 2
1 Ex-6. Solve: cos x – sin x – cos 2x > 0, " x Œ(0, 2p )
1
Soln. We have cos x < and cos x ≥ - . Soln. We have cos x – sin x – cos 2x > 0
2 2
Ê p 7p ˆ fi (cos x – sin x) – (cos2 x – sin 2 x) > 0
fi x ŒÁ 2np + , 2np + ˜
Ë 4 4¯ fi (cos x – sin x) (1 – cos x – sin x) > 0
and fi (sin x – cos x) (sin x + cos x – 1) > 0
È p p˘ Ê pˆ
x Œ Í2np - , 2np + ˙ fi sin Á x - ˜ (sin x + cos x - 1) > 0
Î 3 3˚ Ë 4¯
Hence, the solution set is Hence, the solution set is
Ê p 7p ˆ Ê p ˆ Ê 5p ˆ
x = ∪ Á 2np + , 2np + ˜ x ŒÁ 0, ˜ » Á , 2p ˜
nŒI Ë 4 4¯ Ë 4¯ Ë 4 ¯

È p p˘ 5 2 1
sin x + sin 2 2 x > cos 2 x
» Í2np + , 2np + ˙ . Ex-7. Solve:
Î 3 3˚ 4 4
5 1
Soln. We have, sin 2 x + sin 2 2 x > cos 2 x
Ex-4: Solve: sin x + cos x = sin x + cos x . 4 4
Soln. We have sin x + cos x = sin x + cos x fi 5 (2 sin2 x) + 2 (sin2 2x) > 8 cos 2x
As we know that, if fi 5 (1 – cos 2x) + 2 (1 – cos2 2x) > 8 cos 2x
fi 5 – 5 cos 2x + 2 – 2 cos2 2x – 8 cos 2x >0
f ( x) + g ( x ) = f ( x) + g ( x )
fi 2 cos2 2x + 13 cos 2x – 7 < 0
then f ( x ) g ( x ) ≥ 0 fi 2 cos2 2x + 14 cos 2x – cos 2x – 7 < 0
Thus, sin x cos x ≥ 0 fi 2 cos 2x (cos 2x + 7) – (cos 2x +7) <0
fi sin 2x ≥ 0 fi (cos2x + 7) (2 cos 2x – 1) < 0
È p˘ fi 2 cos 2x – 1 < 0
fi x Œ Ín p , n p + ˙ fi cos 2x < 1/2
Î 2˚
Hence, the solution set is Ê p 5p ˆ
fi x ŒÁ np + , np + ˜
Ë 6 6¯
È p˘
x = ∪ Ín p , n p + ˙ . Hence, the solution set is
nŒI Î 2˚
Ex-5. Solve: sin x sin 2 x < sin 3 x sin 4 x, Ê p 5p ˆ
x = ∪ Á np + , np + ˜
Ê pˆ nŒI Ë 6 6¯
" x ŒÁ 0, ˜ .
Ë 2¯
Trigonometric In-Equation 189

Ex.-8 Solve: 6 sin 2 x - sin x cos x - cos 2 x > 2 . fi cos 4x > 1/2
Soln. We have, 6 sin 2 x - sin x cos x - cos 2 x > 2 Ê p pˆ
fi 4 x ŒÁ 2np - , 2np + ˜
fi 6 sin2 x – sin x cos x – cos2 x Ë 3 3¯
> 2 (sin2 x + cos2 x) Ê np p np p ˆ
fi x ŒÁ - , +
fi 4 sin2 x – sin x cos x – 3 cos2 x > 0 Ë 2 12 2 12 ˜¯
fi 4 tan2 x – tan x – 3 > 0 Hence, the solution set is
fi 4 tan x – 4 tan x + 3 tan x – 3 > 0
2
Ê np p np p ˆ
x= ∪Á - , + ˜
fi 4 tan x (tan x – 1) + 3 (tan x – 1) > 0 nŒI Ë 2 12 2 12 ¯
fi (tan x – 1) (4 tan x + 3) > 0 5
Ex-10. Solve: cos3 x.cos 3x - sin 3 x sin 3x >
fi tan x < – 3/4 and tan x > 1 8
Ê p Soln. The given inequation is
Ê 3ˆ ˆ
fi x ŒÁ np - , np - tan -1 Á ˜ ˜
Ë 2 Ë 4¯ ¯ cos3 x.cos 3x - sin 3 x sin 3x >
5
8
Ê p pˆ
and x ŒÁ np + , np + ˜ fi (cos 3x + 3 cos x ) cos 3x
Ë 4 2¯
5
Hence, the solution set is – (3 sin x - sin 3x ) sin 3x >
2
Ê p Ê 3ˆ ˆ
x = Á np - , np - tan -1 Á ˜ ˜ fi sin 2 3x + cos 2 3x
Ë 2 Ë 4¯ ¯
5
Ê p pˆ + 3 (cos 3x cos x - sin 3x sin x ) >
» ÁË np + , np + ˜¯ , n Œ I 2
4 2
5
13 fi 3 cos 4x + 1 >
Ex-9. Solve: sin x + cos x >
6 6 2
16
1
13 fi cos 4 x >
Soln. We have, sin x + cos x >
6 6 2
16
p p
fi 2np - < 4 x < 2np + , n ΠI
fi (
1 - 3 sin 2 x cos 2 x >
13
16
) 3 3
np p np p
fi - <x< + , n ŒI
Ê 3
( )(
fi Á1 - 2 sin 2 x 2 cos 2 x ˜ >
Ë 4
ˆ 13
¯ 16 ) 2 12 2 12
Hence, the solution set is
Ê 3 ˆ 13
fi ÁË1 - (1 - cos 2 x ) (1 + cos 2 x )˜¯ > Ê Ê np p np p ˆ ˆ
4 16 x = ∪ Á ÁË - , + ˜
nŒI Ë 2 12 2 12 ¯ ˜¯

Ê 3
( ˆ 13
ÁË1 - 1 - cos 2 x ˜¯ >
4
2
16
) Ex-11. Solve the inequality: sin 6 x + cos6 x >
13
16
Ê 3 2 ˆ 13
fi ÁË1 - sin 2 x˜¯ > Soln. The given inequation is
4 16
13
sin 6 x + cos6 x >

Ê 3
( ˆ 13
ÁË1 - 2 sin 2 x ˜¯ >
8
2
)
16
16

(sin )
3
fi 2
x + cos 2 x - 3 sin 2 x cos 2 x
Ê 3 ˆ 13
fi ÁË1 - (1 - cos 4 x )˜¯ >
8 16
(sin 2
x + cos 2 x) >
13
16
Ê5 3 ˆ 13
fi ÁË + cos 4 x˜¯ > 13
8 8 16 fi 1 - 3 sin 2 x cos 2 x >
16
fi 3 3
8
cos 4 x >
16 fi 1-
3
4
(
sin 2 2 x > )
13
16
190 Comprehensive Trigonometry with Challenging Problems & Solutions for Jee Main and Advanced

Soln. The given inequation is


fi 1-
3
8
( )
2 sin 2 4 x >
13
16 sin3x sin4x > sinx sin2x
3 13 fi 2 sin 3x sin 4x > 2 sin x sin 2x.
fi 1 - (1 - cos 4 x ) >
8 16 fi cos x – cos 7x > cos x – cos 3x
5 3 13
fi + cos 4 x > fi –cos 7x > – cos 3x
8 8 16
3 3 fi cos 7x < cos 3x
fi cos 4 x >
8 16 fi cos 7x – cos 3x < 0
1 fi –2 sin 5x sin 2 x < 0
fi cos 4 x >
2
fi sin 5x sin 2x > 0
p p
fi 2np - < 4 x < 2np + , n ΠI
3 3 Ê pˆ
fi sin 5x > 0, since sin 2x is +ve in Á 0, ˜
np p np p Ë 2¯
fi - <x< + , n ŒI fi 0 < 5x < p
2 12 2 12
Hence, the solution set is p
fi 0< x<
Ê np p np p ˆ 5
x = ∪ ÁË - , + ˜.
nŒI 2 12 2 12 ¯ Hence, the solution set is
5 1
Ex-12: Solve: sin 2 x + sin 2 2 x > cos 2 x Ê pˆ
x = Á 0, ˜
4 4 Ë 5¯
Soln. The given inequaton is
Ex-14: Solve: sin x + cos x = sin x + cos x
5 2 1
sin x + sin 2 2 x > cos 2 x
4 4 Soln. We have, sin x + cos x = sin x + cos x
fi 5 sin x + sin 2 x > 4 cos 2 x
2 2
fi sin x cos x ≥ 0

fi ( ) ( )
5 2 sin 2 x + 2 sin 2 2 x > 4 cos 2 x


2 sin x cos x ≥ 0
sin 2 x ≥ 0
fi ( )
5 (1 - cos 2 x ) + 2 1 - cos 2 2 x > 8 cos 2 x fi 2np £ 2 x £ 2np + p , n Œ I
fi 2 cos 2 2 x + 13 cos 2 x - 7 < 0 fi p
np £ x £ np + , n ŒI
2
fi 2 cos 2 2 x + 14 cos 2 x - cos 2 x - 7 < 0 Hence, the solution set is
fi 2 cos 2 x (cos 2 x + 7 ) - 1(cos 2 x + 7 ) < 0 Ê pˆ
x = ∪ Á np , np + ˜
fi (2 cos 2 x - 1) (cos 2 x + 7) < 0 nŒI Ë 2¯
1 Ex-15. Solve: sec x + tan x = sec x + tan x
fi -7 < cos 2 x <
2 Soln. We have, sec x + tan x = sec x + tan x
1 fi sec x.tan x ≥ 0
fi cos 2 x <
2
sin x
p p fi ≥0
fi 2np - < 2 x < 2np + , n ΠI cos 2 x
3 3
fi sin x ≥ 0, cos 2 x π 0
p p
fi np - < x < np + , n ΠI p
6 6 fi sin x ≥ 0, x π (2n + 1) , n Œ I
Hence, the solution set is 2
p
Ê p pˆ fi 2np £ x £ 2np + p , x π ( 2n + 1) , n Œ I
x = ∪ ÁË np - , np - ˜¯ . 2
nŒI 6 6
Hence, the solution set is
Ê pˆ
Ex-13: Solve: sin 3x sin 4x > sin x sin 2x " x ŒÁ 0, ˜ . p
Ë 2¯ x = ∪ ( 2np , ( 2n + 1) p ) - ( 2n + 1)
nŒI 2
Trigonometric In-Equation 191

Ex-16. Solve for x: sin2 x + sin x – 2 < 0 fi (tan x + 3) (tan x – 2) < 0


and x – 3x + 2 < 0.
2
fi –3 < tan x < 2
Soln. We have, sin x + sin x – 2 < 0
2
fi tan -1 ( -3) < x < tan -1 ( 2)
fi (sin x + 2) (sin x - 1) < 0 x -1
fi -2 < sin x < 1 Also, <0
5- x
fi sin x < 1 x -1
fi >0
3p p x-5
fi - <x< fi x < 1 and x > 5
2 2
Also, x - 3x + 2 < 0
( )
2
Hence, the solution set is x Π1, tan -1 ( 2)
fi ( x - 1) ( x - 2) < 0 Ex-20. Solve for x: [ sin x ] = 0, where [, ] = G.I.F
fi 1<x<2 Soln. We have, [ sin x ] = 0
p fi 0 £ sin x < 1
Hence, the solution set is 1 < x <
2
Case I: When sin x ≥ 0
Ex-17. Solve for x: 2 sin x + sin x – 1 < 0
2

and x2 + x – 2 < 0 fi 2np £ x £ ( 2n + 1) p , n Œ I


Soln. We have, 2 sin2 x + sin x – 1 < 0
Case II: When sin x < 1
fi (2 sin x – 1) (sin x + 1) < 0
fi –1 < sin x < 2 3p p
fi 2np - < x < 2np +
p p 2 2
fi - <x< Ê p Ê p ˆˆ
2 6 fi x ŒÁ ( 4n - 3) , Á (4n + 1) ˜ ˜
Ë 2 Ë 2¯¯
Also, x + x – 2 < 0
2
Hence, the solution set is
fi (x + 2) (x – 1) < 0
fi –2 < x < 1 Ê ÈÎ2np , (2n + 1) p ˘˚ ˆ
Á ˜
p p x= ∪Á Ê p p ˆ˜
Hence, the solution set is, - <x< nŒI Á » Á ( 4n - 3) , ( 4n + 1) ˜ ˜
2 6 Ë Ë 2 2¯¯
Ex-18. Solve for x: tan2 x – 5 tan x + 6 > 0
and x2 – 16 £ 0 LEVEL I
Soln. We have, tan2 x – 5 tan x + 6 > 0 (QUESTIONS BASED ON FUNDAMENTALS)
fi (tan x – 2) (tan x – 3) > 0 Q. Solve for x:
1
fi tan x < 2 and tan x > 3 1. sin x >
2
fi x < tan -1 ( 2) and x > tan -1 (3) 2. sin x ≥ 1
1
Also, x2 – 16 £ 0 3. sin x £
2
fi ( x + 4) ( x - 4) £ 0 3
4. sin x <
fi -4 £ x £ 4 2
3
Hence, the solution set is 5. cos x >
2
( ) (
x Œ -4, tan -1 (2) » tan -1 (3) , 4 ) 6. cos x ≥
1
x -1 2
Ex-19. Solve for x: < 0 and tan 2 x + tan x - 6 < 0 . 1
5- x 7. cos x <
2
Soln. We have, tan 2 x + tan x - 6 < 0
192 Comprehensive Trigonometry with Challenging Problems & Solutions for Jee Main and Advanced

1 LEVEL III
8. cos x <
3 (FOR JEE ADVANCED EXAM ONLY)
1 Q. Solve for x:
9. tan x >
3 sin x + cos x
1. > 3
10. tan x ≥ 1 sin x - cos x

11. sin 2 x <


1 2. sin x > cos x
2 sin x
3. cot x + ≥0
12. sin 3x <
3 cos x - 2
2 4. sin x + cos x > 2 cos 2 x
1 5. 4 sin x sin 2x sin 3x > sin 4x
13. cos 5 x ≥
2 cos 2 2 x
14. sin x + cos x > 1 6. ≥ 3 tan x
cos 2 x
15. sin x - cos x < 1
cos x + 2 cos 2 x + cos 3x
16. 3 sin x + cos x > 1 7. >1
cos x + 2 cos 2 x - 1
17. sin x - 3 cos x < 1
18. sin 2 x + sin x - 2 < 0
8. 2 ( )
2 - 1 sin x - 2 cos 2 x + 2 ( )
2 -1 < 0

19. sin 2 x + 3 sin x + 2 < 0 (


9. sin 2 x > 2 sin 2 x + 2 - 2 cos 2 x )
20. cos x - cos x > 0
2
, x Œ(0, 2p )
10. 1 + log 4 sin x + 2 log16 cos x > 0
LEVEL II
(FOR JEE MAIN & ADVANCED EXAMS ONLY) Comprehensive Link Passage
1. sin (3x - 1) > 0 PASSAGE I
2. cos (2 x - 3) < 0 If x1 , x2 , x3 ΠR , then

3. sin x £
1 f ( x1 ) + f ( x2 ) + f ( x3 ) Ê x + x + x3 ˆ
£ fÁ 1 2 ˜¯
2 3 Ë 3
1
4. cos x £ Then
2 1. The value of sin a + sin b + sin g is, where
5. sin 2 x + cos 2 x = sin 2 x + cos 2 x a + b + g = 180∞
6. 2 cos 2 x + cos x < 1 (a) £ 1 (b) £ 3
7. 4 sin 2 x - 1 £ 0 (c) £
3 3
(d) £
3
8. 4 cos 2 x - 3 ≥ 0 2 2
9. sin x > cos x 2. The value of cos a + cos b + cos g is, where
a + b + g = 180∞
10. cos x > sin x
3
11. sin x + cos x – cos 2 x > 0 (a) £ 2 (b) £
2
1
12. x 2 + x - 2 < 0 and sin x > 3
2 (c) £ 3 (d) £
1 2
13. x 2 - 1 £ 0 and cos x < 3. The value of cot a + cot b + cot g is, where
2
1 a + b + g = 180∞
14. 4 x 2 - 1 ≥ 0 and tan x ≥
3 3
(a) ≥ 1 (b) ≥ 3 (c) ≥ 3/2 (d) ≥
15. x 2 - 3x + 2 < 0 and (sin x ) - sin x > 0
2 2
4. The value of cot a cot b cot g is, where
a + b + g = 180∞
Trigonometric In-Equation 193

1 1 1 3 (C) The number of solutions (R) 4


(a) £ (b) £ (c) £ (d) £ of cos x > 1 in (0, 2013p ) is
3 3 3 2 3 2
(D) The number of solutions of (S) 2
5. The value of sin a + sin b + sin 2 g is, where
2 2
sin x + cos x = sin x + cos x
a + b + g = 180∞
in (0, 2p ) is
(a) £ 9/4 (b) £ 3/4 (c) £ 3/2 (d) £ 1/2
6. The value of sin a .sin b .sin g is, where 2. Match the following columns:
Column - I Column - II
a + b + g = 180∞
(A) If sin x.cos x > cos x.sin x,
3 3

È 3p ˘
(a) £
3
(b) £
3 3
(c) £
1
(d) £
1 x Œ[0, 2p ] Then x is (P) Í-p , - 4 ˙
4 8 2 3 3 2 Î ˚
7. The value of cot 2 a + cot 2 b + cot 2 g is, where È p p˘
» Í- , ˙
a + b + g = 180∞ Î 4 4˚
(a) ≥ 1 (b) ≥ 3 (c) ≥ (d) ≥ È 3p ˘
3 3/2 »Í , p˙
Î4 ˚
PASSAGE II (B) If 4 sin 2 x - 8 sin x + 3 £ 0 ,
If f ( x ) + g ( x ) = f ( x ) + g ( x ) , then f ( x ) .g ( x ) ≥ 0 È 3p ˘
x Œ[0, 2p ] then x is (Q) Í , 2p ˙ » {0}
On the basis of the above information answer the Î2 ˚
(C) If tan x £ 1, x Œ[ -p , p ]
following questions
1. If sec x + tan x = sec x + tan x " x Œ[0, 2p ] ,
Ê pˆ
then x does not satisfy the equation is then x is (R) Á 0, ˜
(a) 0 (b) p Ë 4¯
p (D) If cos x - sin x ≥ 1,
(c) (d) 2p
2 È p 5p ˘
x Œ[0, 2p ] then x is (S) Í , ˙
2. If x - 1 + x - 3 = 2 , then x is Î6 6 ˚
(a) x > 1 (b) x > 3
(c) x < 1 (d) 1 < x < 3
ASSERTION AND REASON
3. If sin x + cos x = sin x + cos x , " x Œ[0, 2p ] (FOR JEE ADVANCED EXAM ONLY)
then the solution set is Codes:
È p˘ (A) Both A and R are true and R is the correct
(a) Í0, ˙ explannation of A
Î 2˚
(B) Both A and R are true but R is not the correct
È p ˘ È 3p ˘
(b) Í0, ˙ » Íp , ˙ » {2p } explanation of A
Î 2˚ Î 2 ˚ (C) A is true but R is false
È 3p ˘ (D) A is false but R is true.
(c) Íp , ˙ » {2p }
Î 2˚ 1. Assertion (A): The value of tan 3a .cot a
(d) [0, 2p ] cannot lie between 3 and 1/3.
Reason (R): In a triangle ABC, the maximum
Ê Aˆ Ê Bˆ Ê Cˆ 1
MATCH MATRIX value of sin Á ˜ sin Á ˜ sin Á ˜ is
Ë 2¯ Ë 2¯ Ë 2¯ 8
(FOR JEE ADVANCED EXAM ONLY)
2. Assertion (A): The minimum value of
1. Match the following columns:
Column - I Column-II a 2 tan 2 q + b 2 cot 2 q is 2ab.
(A) The number of solutions (P) 6 Reason (R): For positive real numbers
1 A.M ≥ G.M
of sin x > in (0, 2p ) is 3. Assertion (A): The minimum value of
2
a2 b2
is ( a + b )
(B) The number of solutions (Q) 0 2
+
of tan x £ 1 in ( -p , p ) is cos x sin x
2 2
194 Comprehensive Trigonometry with Challenging Problems & Solutions for Jee Main and Advanced

Reason (R): The maximum value of Reason (R): In a triangle ABC, the maximum.
3 sin 2 x + 4 cos 2 x is 4 sin A + sin B + sin C 3
value of is
È p˘ cot A + cot B + cot C 2
4. Assertion (A): For all q Œ Í0, ˙ ,
Î 2˚ 5. Assertion (A): cot -1 x ≥ 2 fi x Œ( -•, 2]
cos (sin q ) > sin (cos q )
Reason (R): cot -1 x is a decreasing function.

ANSWERS
LEVEL I Ê 3p pˆ
18. x ŒÁ 2np - , 2np + ˜ , n Œ I
Ê p 5p ˆ Ë 2 2¯
1. x ŒÁ 2np + , 2np + ˜ , n Œ I
Ë 6 6¯ 19. x = j
2. x = (4n + 1) p , n Œ I Ê p 5p ˆ
20. x ŒÁ 2np + , 2np + ˜ , n Œ I
2 Ë 12 12 ¯
Ê 5p pˆ
3. x ŒÁ 2np - , 2np + ˜ , n Œ I LEVEL II
Ë 4 4¯
Ê 4p pˆ Ê 2np + 1 (2np + 1) p + 1ˆ
4. x ŒÁ 2np - , 2np + ˜ , n Œ I 1. x ŒÁ , ˜¯ , n Œ I
Ë 3 3¯ Ë 3 3

Ê p pˆ Ê p 3 p 3ˆ
5. x ŒÁ 2np - , 2np + ˜ , n Œ I 2. x ŒÁ ( 4n + 1) + , ( 4n + 3) + ˜ , n Œ I
Ë 6 6¯ Ë 4 2 4 2¯
Ê p pˆ Ê p pˆ
6. x ŒÁ 2np - , 2np + ˜ , n Œ I 3. x ŒÁ np - , np + ˜ , n Œ I
Ë 3 3¯ Ë 6 6¯
Ê 7p pˆ Ê 3p pˆ
7. x ŒÁ 2np - , 2np + ˜ , n Œ I 4. x ŒÁ np - , np - ˜ , n Œ I
Ë 4 4¯ Ë 4 4¯
Ê Ê 1ˆ Ê 1ˆ ˆ Ê pˆ
8. x ŒÁ 2np + cos -1 Á ˜ , 2 ( n + 1) p - cos -1 Á ˜ ˜ , n Œ I 6. x ŒÁ np - p , np - ˜ , n Œ I
Ë Ë 3¯ Ë 3¯ ¯ Ë 3¯

Ê p pˆ Ê p pˆ
9. x ŒÁ np + , np + ˜ , n Œ I 7. x ŒÁ np - , np + ˜ , n Œ I
Ë 6 2¯ Ë 6 6¯

Ê p pˆ Ê p pˆ
10. x ŒÁ np + , np + ˜ , n Œ I 8. x ŒÁ np - , np + ˜ , n Œ I
Ë 4 2¯ Ë 6 6¯

Ê p 5p ˆ ÏÊ p pˆ
11. x ŒÁ np + , np + ˜ , n Œ I 9. x Œ ∪ ÌÁ 2np - , np - ˜
Ë 12 12 ¯ nŒI ÓË 2 4¯
Ê 2np 4p 2np p ˆ Ê p pˆ¸
12. x ŒÁ - , + ˜ , n ŒI » Á np + , np + ˜ ˝
Ë 3 9 3 9¯ Ë 4 2¯˛
Ê 2np p 2np p ˆ Ê p pˆ
13. x ŒÁ - , + ˜ , n ŒI 10. x ŒÁ np - , np + ˜ , n Œ I
Ë 5 15 5 15 ¯ Ë 4 4¯
14. x Œ(2np , ( 2n + 1) p ) , n Œ I Ê 3p ˆ Ê 3p 7p ˆ
11. x ŒÁ 0, ˜ » Á , ˜
Ë 4¯ Ë 4 4¯
Ê pˆ
15. x ŒÁ ( 2n - 1) p , 2np + ˜ , n Œ I
Ë 2¯ 12. p < x < 1
6
Ê 2p ˆ
16. x ŒÁ 2np , 2np + ˜ , n Œ I 13. x = j
Ë 3¯
Èp p ˘
Ê 5p pˆ 14. x Œ Í , ˙
17. x ŒÁ 2np - , 2np + ˜ , n Œ I Î3 2˚
Ë 6 2¯
Trigonometric In-Equation 195

15. x Œ(p , 2p ) Ê p pˆ
8. ∪ Á 2np - , 2np + ˜
nŒI Ë 4 6¯
Ê 5p 5p ˆ
LEVEL III » Á 2np + , 2np + ˜
Ë 6 4¯
Ê p pˆ
1. ∪ Á np + , np + ˜ pˆ
nŒI Ë 4 12 ¯ Ê
9. Á tan -1
Ë ( 2 -1 , ) ˜

Ê p 3p ˆ
2. ∪ Á np + , np + ˜
nŒI Ë 4 4¯ Ê
» Á p + tan -1
Ë ( ) 54p ˆ˜¯
2 -1 ,
Ê p pˆ
3. ∪ Á 2np - , 2np + ˜ Ê p 5p ˆ
nŒI Ë 3 3¯ 10. ∪ Á 2np + , 2np + ˜
nŒI Ë 12 12 ¯
Ê p ˆ
» Á 2np + , ( 2n + 1) p ˜
Ë 3 ¯
COMPREHENSIVE LINK PASSAGES
Ê p 3p ˆ
4. ∪ Á 2np + , 2np + ˜ PASSAGE I
nŒI Ë 12 4¯
1. (c) 2. (b) 3. (b) 4. (b)
Ê 17p 7p ˆ
» Á 2np + , 2np + ˜ 5. (a) 6. (b) 7. (a)
Ë 12 4¯
Ê p ˆ Ê p 5p ˆ PASSAGE II
5. ∪ Á np - , np ˜ » Á np + , np + ˜
nŒI Ë 8 ¯ Ë 2 8¯ 1. (c) 2. (d) 3. (b)
Ê p 3p ˆ
» Á np + , np + ˜ MATCH MATRIX
Ë 8 8¯
1. (A) Æ (S); (B) Æ (S); (C) Æ (Q); (D) Æ (R).
Ê 7p pˆ 2. (A) Æ (R); (B) Æ (S); (C) Æ (P); (D) Æ (Q).
6. ∪ Á np - , np - ˜
nŒI Ë 12 2¯
p pˆ ASSERTION AND REASON
Ê
» Á np - , np + ˜ . 1. (B) 2. (A) 3. (B)
Ë 2 12 ¯
4. (B) 5. (A)
Ê p pˆ
7. ∪ Á 2np - , 2np + ˜
nŒI Ë 3 3¯
5
196 Comprehensive Trigonometry with Challenging Problems & Solutions for Jee Main and Advanced

CHAPTER

Logarithm

5.1 INTRODUCTION log a 0 = ± •


Case i: a>1
a -• = 0
fi -• = log a 0
Case ii: a
a• = 0
fi • = log a 0
log a 0 = ± •
log a • = •
Case i: a>1
NOTATIONAL CONVENTIONS

x a =•
e
x fi log a • = •
x
Step II
log m a + log m b = log m ( ab )
log m a = x , log m b = y, log m ( ab ) = z
BASIC FORMULAE ON LOGARITHM: m x = a, m y = b, m z = ab

Step I: ab = m z
ax = n x = log a n n > 0, fi m x .m y = m z
a aπ
fi m x+ y = m z
log a a = 1 fi x+ y= z
a1 = a fi log m a + log m b = log m ( ab )
fi 1 = log a a
Ê aˆ
log a 1 = 0, a π 1, a > 0 log m a - log m b = log m Á ˜
Ë b¯
a0 = 1 Ê aˆ
log m a = x , log m b = y, log m Á ˜ = z
Ë b¯
fi 0 = log a 1
Logarithm 197

Ê aˆ log a b = x , log m b = y , log m a = z


fi m x = a, m y = b, m z = Á ˜
Ë b¯
fi ax = b , m y = b , mz = a
Ê aˆ z
ÁË ˜¯ = m fi ax = b
b


mx
.= m z
fi (m )
z x
= my
y
m fi mz x = m y
fi m x- y = m z fi zx= y
fi x- y= z
y
Ê aˆ fi x=
fi log m a - log m b = log m Á ˜ z
Ë b¯
log m a
log m a n = n log m a fi logb a =
log m b
log m a n = x , log m a = y a log n b = blog n a
fi mx = an , m y = a log n b = x , log n a = y
fi mx = m ( )
y n
fi nx = b , n y = a

fi m x = mn y fi n = b1/ x , n = a1/ y
fi x=ny fi b1/ x = a1/ y
fi log m a n = n log m a fi by = ax
fi a log n b = blog n a
a log a n = n
log a b ¥ logb c ¥ log c d ¥ log d a = 1
log a n = x
log a b ¥ logb c ¥ log c d ¥ ....... ¥ log z a = 1
fi ax = n
Step IV
fi a log a n = n 1
log aa b = log a b
a
Step III
log a b ¥ logb a = 1 log aa b
log a b = x , logb a = y 1
=
fi x
a = b,b = a y
( )
logb aa

fi (b ) y x
=b =
1
a logb a
fi b xy = b = b1 1 1
x y =1 = ¥
fi a logb a
fi log a b ¥ logb a = 1 1
= ¥ log a b
1 a
log a b = b
logb a ( )
log aa b b = log a b
a
log a b ¥ logb a = 1
( )
log aa b b
1
fi log a b =
logb a = b log aa (b )
log m a b
logb a = = ¥ log a b
log m b a
198 Comprehensive Trigonometry with Challenging Problems & Solutions for Jee Main and Advanced

Step V = log36 (6)


x > y fi log a x > log a y a
Y
= log 62 (6)
= 1/2
Ex-4. log 2 4.log 4 5.log5 10.log10 32
Soln. log 2 4.log 4 5.log5 10.log10 32
X¢ X
O = log 2 (32)

= log 2 2
5
( )
=5
Y¢ log 2 (125)
Ex-6. 2 2

x > y fi log a x < log a y a log 2 (125)


Y
Soln. 2 2

log 2 (125)
= 2 2

2
log 2 (125)
= 23
2/3

O
X = 2log 2 (125)
2/3
= (125)

( )
3 2/3
= 5
Y¢ = 25
Ex-7. 3log5 2 - 2log5 3
5.2 SOME SOLVED EXAMPLES Soln. 3log5 2 - 2log5 3
Ex-1. log 2 (64) + log 4 ( 256)
= 2log5 3 - 2log5 3
Soln. log 2 (64) + log 4 ( 256)
=0
6 4
= log 2 (2 ) + log 4 (4 ) Ex-8. log a ( ab ) = x logb ( ab )
= 6 log 2 (2) + 4 log 4 (4) Soln. log a ( ab ) = x
=6+4
fi log a a + log a b = x
= 10
fi 1 + log a b = x
Ex-2. log8 64
fi log a b = x - 1
Soln. log8 64
logb ( ab ) = logb a + logb b
( )
= log 23 4 3
= logb a + 1
= log 2 ( 4) =
1
+1
x -1
( )
= log 2 22
x
=
=2 x -1
1 1
Ex-3. + x log 2 x + log 4 x + log8 x = 11
log 2 (36) log3 (36)
Ex-9.
Soln. log 2 x + log 4 x + log8 x = 11
1 1
Soln. + fi log 2 x + log 22 x + log 23 x = 11
log 2 (36) log3 (36)
= log36 ( 2) + log36 (3) fi
Ê 1 1ˆ
ÁË1 + + ˜¯ log 2 x = 11
2 3
= log36 ( 2.3)
Logarithm 199

Ê 11ˆ = log abc ( abc ) = 1


fi ÁË ˜¯ log 2 x = 11
6 log a log b log c
Ex-12. = =
fi log 2 x = 2 b-c c-a a-b
fi x = 26 = 32 a a .bb .c c = 1
x log a log b log c
Soln. = = =k
Ex-10. a = log 4 5 b = log5 6 log3 2 b-c c-a a-b
Soln. ab = log 4 5.log5 6 = log 4 6 a log a b log b c log c
fi = = =k
a (b - c ) b ( c - a ) c ( a - b )
= log 22 (6)
1 log a a log bb log c c
= log 2 (6) fi = = =k
2 a (b - c ) b (c - a ) c ( a - b )
1
= log 2 ( 2.3)
2
fi ( )
log a a + log bb + log c c ( ) ( )
1 = k (ab - bc + bc - ba + ca - cb ) = 0
= (log 2 ( 2) + log 2 3)
2 fi (
log a a .bb .c c = 0 )
1
= (1 + log 2 3)
2
fi (a .b .c ) = e
a b c 0
=1

fi 2ab = (1 + log 2 3) Ex-13. log 2 x + log 2 y ≥ 6


fi log 2 3 = (2ab - 1) x+y
Soln. log 2 x + log 2 y ≥ 6
1 1
fi = fi log 2 ( x y ) ≥ 6
log 2 3 ( 2ab - 1)
1 fi x y ≥ 26 = 64
fi log3 2 =
(2ab - 1) x+ y
≥ xy
Ex-11. x = log a bc , y = logb ca z = log c ab 2
x+ y
1 1 1 fi ≥ 64 = 8
+ + 2
1+ x 1+ y 1+ z fi x y ≥ 2.8 = 16
Soln. x = log a bc x+y
18 21 28
fi ax = b c Ex-14. x =y =z

fi a.a x = a b c 3, 3 log y x, 3 log z y, 7 log x z

fi a x+1 = a b c Soln. x18 = y 21 = z 28


fi x + 1 = log a ( abc ) fi ( ) ( )
log x18 = log y 21 = log z 28 ( )
1 1
fi = fi 18 log x = 21log y = 28 log z = k
x + 1 log a (abc )
log x 21 7
1 1 1 1 3 log y x = 3. = 3. =
= , = log y 18 2
y + 1 logb (abc ) z + 1 log c (abc ) log y 28
3 log z y = 3. = 3. = 4
1 1 1 log z 21
+ +
x +1 y +1 z +1 log z 18 9
7 log x z = 7. = 7. =
1 1 1 log x 28 2
= + +
log a ( abc ) logb ( abc ) log c (abc )
= log abc ( a ) + log abc (b ) + log abc (c )
200 Comprehensive Trigonometry with Challenging Problems & Solutions for Jee Main and Advanced

7ˆ 4log9 3 + 9log 2 4 = 10log x 83


Ex-15. ( Ê
log3 2, log3 2 x - 5 , log3 Á 2 x - ˜
Ë 2¯
) Ex-18.
Soln.
x
4log9 3 + 9log 2 4 = 10log x 83
x 1
log3 3
7ˆ + 92 log 2 2 = 10log x 83
Soln. ( Ê
log3 2, log3 2 x - 5 , log3 Á 2 x - ˜
Ë 2¯
) fi 42
fi 2 + 81 = 10log x 83

fi ( ) Ê
2 log3 2 x - 5 = log3 2 + log3 Á 2 x - ˜
Ë 2¯
fi 83 = 83log x 10
fi log x 10 = 1

( ) Ê
x 2
fi log3 2 - 5 = log3 2. Á 2 x - ˜ fi x = 10
Ë 2¯
x
(2 - 5) = 2.2 - 7
x 2 x
fi Ex-19. log x 10 = 1 , b = log36 24 c = log 48 36

fi (2 ) - 12.2 + 32 = 0
x 2 x Ê abc + 1ˆ
ÁË
bc ¯
˜ =2
fi a 2 - 12 a + 32 = 0, a = 2 x
Soln. a b c = log 24 12 ¥ log36 24 ¥ log 48 12
fi (a - 4) (a - 8) = 0, a = 2 x = log 48 36 ¥ log36 24 ¥ log 24 12
fi a = 4, 8 = log 48 12
x 2 abc+1
a=4fi 2 =4=2 fix=2
x 3
a=8fi 2 =8=2 fix=3 = log 48 12 + 1 = log 48 12 + log 48 48
x
= log 48 (12 ¥ 48)
x
Ex-16. a, b, c b c = log36 24 ¥ log 48 36
1 1 1 = log 48 36 ¥ log36 24
, ,
1 + log a 1 + log b 1 + log c
= log 48 24
Soln. a, b, c
Ê abc + 1ˆ log 48 (12 ¥ 48)
fi b2 = a c ÁË ˜=
bc ¯ log 48 24
fi ( )
log b 2 = log (a c )
= log 24 (12 ¥ 48)
fi 2 log (b ) = log ( a ) + log (c )
= log 24 ( 24 ¥ 24)
fi a b c
fi a b c
= log 24 242 ( )
1 1 1 =2
fi , ,
1 + log a 1 + log b 1 + log c
Ê Ê p ˆˆ 1
1
+
1
>x Ex-20. log10 Á sin Á x + ˜ ˜ = (log10 6 - 1)
Ex-17. x Ë Ë 4¯¯ 2
log3 p log 4 p
1 1 log10 sin x + log10 cos x
Soln. +
log3 p log 4 p Ê Ê p ˆˆ 1
Soln. log10 Á sin Á x + ˜ ˜ = (log10 6 - 1)
= logp 3 + logp 4 Ë Ë 4¯¯ 2

= logp (3.4) fi
Ê Ê p ˆˆ
2 log10 Á sin Á x + ˜ ˜ = (log10 6 - 1)
Ë Ë 4¯¯
= logp (12) > logp p 2 = 2 ( ) Ê 1 ˆ
x fi 2 log10 Á
Ë 2
(sin x + cos x )˜¯ = (log10 6 - 1)
Logarithm 201

Ê 1 ˆ fi 5x = 4
fi 2 log10 Á + 2 log10 (sin x + cos x ) = (log10 6 - 1)
Ë 2 ˜¯ fi x = 4/5

2 log10 (sin x + cos x ) = (log10 6 + 2 log10 2 - 1) x



Type 2:
2 Ê 24 ˆ
log10 (sin x + cos x ) = log10 Á ˜
fi Ë 10 ¯ { }
log f1 ( x ) log f 2 ( x ) f ( x) = 0
24 ˆ
fi (sin x + cos x )2 = ÊÁË ˜ fi
Ï f1 ( x) > 0, f1 ( x) π 1
f 2 ( x) = f ( x) Ì
10 ¯
Ó f 2 ( x) > 0, f 2 ( x) π 1
24
fi 1 + sin 2 x =
10 Ex-3. x log 2
x +6 x+6 { (
log 2 x 2 + 2 x +3 x 2 - 2 x )} = 0
24 14
fi sin 2 x = -1 = Soln.
10 10

fi sin x .cos x =
7 { (
log x 2 + 6 x + 6 log 2 x 2 + 2 x +3 x 2 - 2 x )} = 0
10
Ê 7ˆ
fi (x 2
)
- 2 x = 2 x2 + 2 x + 3
log10 (sin x cos x ) = log10 Á ˜
Ë 10 ¯ fi x2 + 4 x + 3 = 0
log10 (sin x ) + log10 (cos x ) = log10 7 - 1 fi ( x + 1) ( x + 3) = 0
fi x = –1, –3
5.3 LOGARITHMIC EQUATION x 2 + 6 x + 6 > 0, x 2 + 6 x + 6 π 1

Type 1: fi ( ) (
x Œ -•, - 3 - 3 » -3 + 3, • , x π -1, -5 )
log g ( x ) f ( x) = b 2 x 2 + 2 x + 3 > 0, x 2 + x + 1 π 0
fi f ( x) = g ( x)b , g ( x) > 0, g ( x) π 1 fi x ŒR
Ex-1 (
x log x 3x 2 + 10 x = 3 ) x =j

Soln. log x (3 x 2
)
+ 10 x = 3
x log 2
x + x +1 { (
log 2 x 2 +3 x +5 x 2 + 3 = 0 )}
fi (3 x 2
)
+ 10 x = x3 , x > 0, x π 1 Soln.

fi x3 - 3x10 - 10 x = 0 { ( )}
log x 2 + x +1 log 2 x 2 +3 x +5 x 2 + 3 = 0

fi (
x x 2 - 3x - 10 ) fi 2 x 2 + 3x + 5 = x 2 + 3
fi x 2 + 3x + 2 = 0
fi x ( x - 5) ( x + 2 )
fi ( x + 1) ( x + 2) = 0
fi x = 0, –2, 5
fi x = –1, –2
fi x=5
x 2 + x + 1 > 0, x 2 + 3 π 1
x fi x ŒR
Ex-2. (
x log x +1 x 2 - 3x + 5 = 2 ) 2 x 2 + 3x + 5 > 0, x 2 + 3 π 1
fi x ŒR
Soln. (
log x +1 x 2 - 3x + 5 = 2 ) x =f
fi ( ) 2
x 2 - 3x + 5 = ( x + 1) , x + 1 > 0, x π 0
Type 3:
fi (x 2
- 3x + 5) = ( x + 1)
2
log a f1 ( x) = log a f 2 ( x), a > 0, a π 1
f1 ( x ) = f 2 ( x ) , f1 ( x) > 0 or f 2 ( x) > 0
fi (x 2
- 3 x + 5) = x + 2 x + 1
2 fi
202 Comprehensive Trigonometry with Challenging Problems & Solutions for Jee Main and Advanced

Ex-5 ( )
x log5 x 2 - 4 x + 3 = log5 (3x + 21) Ex-7 x logÊ x +5 ˆ 3 = logÊ
ÁË ˜
3 ¯
ÁË
-1 ˆ
˜
x +1¯
3

Soln.
logÊ x +5 ˆ 3 = logÊ 3
( )
Soln. -1 ˆ
log5 x 2 - 4 x + 3 = log5 (3x + 21) ÁË ˜
3 ¯
ÁË ˜
x +1¯

fi (x 2
)
- 4 x + 3 = 3x + 21, 3x + 21 > 0 fi
Ê x + 5 ˆ Ê -1 ˆ
ÁË
3
˜¯ = ÁË
x +1
˜¯ , x + 5 > 0,
x+5
3
π1

fi (x 2
)
- 7 x - 18 = 0, x > -7
fi ( x + 1) ( x + 5) = -3, x > -5, x π -2
fi ( x - 9) ( x + 2) = 0, x > -7 fi x 2 + 6 x + 8 = 0, x > -5, x π -2
fi x x fi ( x + 2) ( x + 4) = 0, x > -5, x π -2
fi x = -2, - 4, x > -5, x π -2
x {-2, 9}
x = –4
Ê Ê 1ˆ x ˆ ÊÊ 1ˆ x ˆ
Ex-6: x log1/ 3 Á 2 Á ˜ - 1˜ = log1/ 3 Á Á ˜ - 4˜ Ex-8 x logÊ x + 2 ˆ 5 = logÊ 4 ˆ 5
Ë Ë 2¯ ¯ Ë Ë 4¯ ¯ ÁË ˜
3 ¯
ÁË ˜
x -3¯

Soln. Soln. logÊ x + 2 ˆ 5 = logÊ 4 ˆ 5


ËÁ 3 ¯˜ ËÁ x - 3 ¯˜
Ê Ê 1ˆ x ˆ ÊÊ 1ˆ x ˆ
log1/ 3 Á 2 Á ˜ - 1˜ = log1/ 3 Á Á ˜ - 4˜
Ë Ë 2¯ ¯ Ë Ë 4¯ ¯ Ê x + 2ˆ Ê 4 ˆ
fi ÁË ˜ =Á ˜ , x > –2, x + 2 π 3
3 ¯ Ë x - 3¯
x x
Ê 1ˆ Ê 1ˆ fi ( x + 2) ( x - 3) = 12, x > -2, x π 1
fi 2Á ˜ -1 = Á ˜ - 4
Ë 2¯ Ë 4¯
fi x 2 + 5 x - 6 = 0, x > -2, x π 1
2x x
Ê 1ˆ Ê 1ˆ ( x + 6) ( x - 1) = 0, x > -2, x π 1
fi ÁË ˜¯ - 2Á ˜ - 3 = 0 fi
2 Ë 2¯
x
fi x = -6,1, x > -2, x π 1
2 Ê 1ˆ x=1
fi a - 2a - 3 = 0 a=Á ˜
Ë 2¯
Type 5:
fi (a - 3) (a + 1) = 0
log f ( x ) g1 ( x) = log f ( x ) g 2 ( x)
fi a = 3, –1
fi a Ï g1 ( x) > 0, f ( x) > 0, π 1
Ô
x fi g1 ( x) = g 2 ( x) , Ì or
Ê 1ˆ Ô g ( x) > 0, f ( x) > 0, π 1
fi ÁË ˜¯ = 3 Ó 2
2
fi 2- x = 3 Ex-9 ( )
x log x 2 -1 x3 + 6 = log x 2 -1 4 x 2 - x ( )
fi - x = log 2 3
Soln. ( )
log x 2 -1 x3 + 6 = log x 2 -1 4 x 2 - x ( )
Ê 1ˆ
fi x = - log 2 3 = log 2 Á ˜ fi x3 + 6 = 4 x 2 - x, x3 + 6 > 0, x 2 - 1 > 0, π 1
Ë 3¯
Ê 1ˆ fi x3 - 4 x 2 + x + 6 = 0, x3 > -6, x 2 > 1, x π ± 2
x = log 2 Á ˜
Ë 3¯ x3 - 4 x 2 + x + 6 = 0
Type 4: fi x3 + x 2 - 5 x 2 - 5 x + 6 x + 6 = 0
log f1 ( x ) A = log f 2 ( x ) A fi x 2 ( x + 1) - 5 x ( x + 1) + 6 ( x + 1) = 0

Ï f1 ( x) > 0, f1 ( x) π 1 fi ( x + 1) ( x 2 - 5 x + 6) = 0
Ô
fi f1 ( x) = f 2 ( x) , Ì or
Ô f ( x) > 0, f ( x) π 1 fi ( x + 1) ( x - 2) ( x - 3) = 0
Ó 2 2
Logarithm 203

fi x = –1, 2, 3 144 100


= ,
3
x > -6 fi x > - 6 3 32 32
71 25
x 2 > 1 fi x Œ(-•, - 1) » (1, • ) = ,
16 4
x = 2, 3
x > 15/4
Type 6: x = 25/4
log g1 ( x ) f ( x) = log g2 ( x ) f ( x) , Ex-12. (
x 2 log 2 x = log 7 x - 2 - 2 x 2 )
Ï g1 ( x) > 0, π 1, f ( x) > 0
Ô
Soln. (
2 log 2 x = log 7 x - 2 - 2 x 2 )
fi g1 ( x ) = g 2 ( x ) , Ì or
Ô g ( x) > 0, π 1, f ( x) > 0 4 x2 = 7 x - 2 - 2 x2 , x > 0
Ó 2
6 x 2 - 7 x + 2 = 0, x > 0
Ex-10 ( ) (
x log x3 + x x 2 - 4 = log 4 x 2 - 6 x 2 - 4 )
fi 6 x 2 - 3x - 4 x + 2 = 0
Soln.
3x ( 2 x - 1) - 2 ( 2 x - 1) = 0
( ) (
log x3 + x x 2 - 4 = log 4 x 2 - 6 x 2 - 4 ) fi
fi (2 x - 1) (3x - 2) = 0
fi x3 + x = 4 x 2 - 6, x3 + x > 0, x3 + x π 1, x 2 - 4 > 0
1 2
fi x= ,
fi x3 - 4 x 2 + x + 6 = 0, x > 0, x3 + x π 1, x 2 > 4 2 3
fi ( x + 1) ( x - 2) ( x - 3) = 0 x>0
fi x = –1, 2, 3 1 2
x= ,
x>0 2 3
Type 8:
x 2 - 4 > 0 fi x Œ( -•, - 2) » ( 2, • )
(2n + 1) log a f1 ( x) = log a f 2 ( x) ,
x=3
a > 0, a π 1, n Œ N
Type 7:
fi f1( 2 n +1) ( x) = f 2 ( x) , f1 ( x) > 0
2n log a f1 ( x) = log a f 2 ( x) , a > 0, a π 1, n Œ N

fi f12 n ( x) = f 2 ( x) , f1 ( x) > 0 Ex-13 ( )


x log 8 - 10 x - 12 x 2 = 3 log ( 2 x - 1)
Soln.
x log3 2 x = 2 log3 ( 4 x - 15)
( )
Ex-11
log 8 - 10 x - 12 x 2 = 3 log ( 2 x - 1)
Soln. log3 2 x = 2 log3 ( 4 x - 15)
2 15
fi (8 - 10 x - 12 x ) = (2 x - 1) , x > 1 / 2
2 3

fi (4 x - 15) = 2 x, x >
4 fi 8 x3 + 16 x - 9 = 0
fi 16 x 2 - 122 x + 225 = 0 fi (2 x - 1) (4 x 2 + 2 x + 9) = 0
122 ± (122)2 - 64 ¥ 225 fi x = 1/2
fi x=
32 x > 1/2
x =j
122 ± 14884 - 14400
=
32 Type 9:
122 ± 484 log a f ( x) + log a g ( x) = log a m( x) ,
=
32 a > 0, a π 1
122 ± 22 Ï f ( x) > 0
= Ô
32 fi Ì g ( x) > 0
Ô f ( x ) g ( x ) = m( x )
Ó
204 Comprehensive Trigonometry with Challenging Problems & Solutions for Jee Main and Advanced

Ex-14 x fi ( x - 1) ( x - 8) = 0
( )
2 log3 x + log3 x 2 - 3 = log3 (0.5) + log3 8
fi x = 1, 8
Soln. fi x = 1, x
( )
2 log3 x + log3 x - 3 = log3 (0.5) + log3 8
2 x

fi ( )
x 2 x 2 - 3 = 4, x > 0, x 2 - 3 > 0
5.4 LOGARITHMIC INEQUATION
fi x 2
(x 2
- 3) = 4 Type I:
fi x 4 - 3x 2 - 4 = 0 Ï Ï g ( x) > 0
Ôlog a f ( x) > log a g ( x) Ô
fi ( x - 1) ( x
2 2
)
+4 =0 Ì
Ô a > 1
fi Ì a >1
Ô f ( x) > g ( x)
Ó Ó
fi x = ±1
Ex-1 x
x>0
log(2 x +1) x 2 < log(2 x +1) ( 2 x + 3) , x > –1
2
x - 3 > 0 fi x Œ -•, - 3 » ( ) ( 3, • )
x=1
log(2 x +1) x 2 < log(2 x +1) ( 2 x + 3)
Type 10:
log a f ( x) - log a g ( x) = log a h( x) - log a t ( x) , x 2 < 2 x + 3, 2 x + 1 > 1, 2 x + 3 > 0

a > 0, a π 1 x 2 - 2 x - 3 < 0, x > -1/ 2, x > -3 / 2


fi log a f ( x) + log a t ( x) = log a g ( x) + log a h( x) ( x - 3) ( x + 1) < 0, x > -1 / 2, x > -3 / 2
x x > –1/2, x > –3/2
Ï f ( x) > 0, t ( x) > 0, g ( x) > 0, h( x) > 0
fi Ì
f ( x).t ( x) = g ( x).h( x) Ê 1 ˆ
Ó x ŒÁ - , 3˜
Ë 2 ¯
Ex-15. x Type II:
Ê Ê 3p ˆ ˆ
Á sin ÁË 4 ˜¯ ˜ 1 Ï Ï f ( x) > 0
log 2 (3 - x ) - log 2 Á ˜ = + log 2 ( x + 7 ) Ôlog a f ( x) > log a g ( x) Ô
Á 5- x ˜ 2 Ì fi Ì 0< a < 1
ÁË ˜¯ Ô0 < a < 1 Ô f ( x) < g ( x)
Ó Ó
Soln.
Ex-2 x log 0.3 ( x - 1) < log 0.09 ( x - 1)
Ê Ê 3p ˆ ˆ
Á sin ÁË 4 ˜¯ ˜ 1 Soln. log 0.3 ( x - 1) < log 0.09 ( x - 1)
log 2 (3 - x ) - log 2 Á ˜ = + log 2 ( x + 7 )
Á 5- x ˜ 2 fi log 0.3 ( x - 1) < log ( x - 1)
ÁË ˜¯ (0.3)2
1
fi log 0.3 ( x - 1) < log ( x - 1)
Ê 1 ˆ 1 2 (0.3)
fi log 2 (3 - x ) - log 2 Á ˜ = + log 2 ( x + 7 )
Ë 2 (5 - x ) ¯ 2 fi 2 log 0.3 ( x - 1) < log(0.3) ( x - 1)
1
( )
2
fi log 2 (3 - x ) + log 2 2 (5 - x ) =
+ log 2 ( x + 7 ) fi log 0.3 ( x - 1) < log(0.3) ( x - 1)
2
fi log 2 (3 - x ) + 1 + log 2 (5 - x ) = 1 + log 2 ( x + 7 )
fi ( x - 1)2 > ( x - 1)
2 2 fi ( x - 1) ( x - 2) > 0
fi log 2 {(3 - x ) (5 - x )} = log 2 ( x + 7 ) fi x x>2
fi ( x - 3) ( x - 5) = ( x + 7) fi x>2
fi x 2 - 8 x + 15 - x - 7 = 0 fi x Œ( 2, • )
fi x2 - 9 x + 8 = 0
Logarithm 205

Type III:
Ê Ê 1 1 ˆˆ
fi log10 x ¥ M > 1 Á∵ M = Á - ˜
Ïlog a x > 0 Ïx> 0
fiÌ Ë Ë log10 3 log10 2 ˜¯ ¯
Ì
ÓÔa >1 Óa > 1 1
fi log10 x >
Ïlog a x > 0 Ï0 < x < 1 M
Ì fiÌ
ÔÓ0 < a < 1 Ó0 < a < 1 fi x > 101/ M
Ïlog a x < 0 Ï0 < x < 1
Ì fiÌ (
x Œ 101/ M , • )
ÓÔ a >1 Ó a >1
1 1
Ïlog a x < 0 Ï x >1 Ex-6. x - <1
Ì fiÌ log 2 x log 2 x - 1
ÓÔ0 < a < 1 Ó0 < a < 1
1 1
Ex-3 x log1/ 2 ( 2 x + 3) > 0 Soln. - <1
log 2 x log 2 x - 1
Soln. log1/ 2 ( 2 x + 3) > 0
1 1
0 fi - < 1, a = log 2 x
1 a a -1
fi (2 x + 3) < ÊÁË ˆ˜¯
2 1 1
fi -1- < 0,
fi (2 x + 3) < 1 a a -1
fi 2x 1- a 1
fi - < 0,
fi x a a -1
2
fi x - (1 - a ) - a
fi < 0,
(2 x + 3) > 0 fi x > -3 / 2 a (a - 1)
Ê 3 ˆ
x ŒÁ - , - 1˜ (1 - a )2 + a 0
Ë 2 ¯ fi >
a (a - 1)
Ex-4. x log1/ 2 x > log1/ 3 x a2 - a + 1
fi >0
Soln. log1/ 2 x > log1/ 3 x a (a - 1)
Ê log1/ 2 x ˆ 1
fi log1/ 2 x > Á ˜ fi >0
Ë log1/ 2 (1 / 3) ¯ a (a - 1)
Ê 1 ˆ fi a a
fi log1/ 2 x Á1 - ˜ >0
Ë log1/ 2 (1 / 3) ¯ fi log 2 x > 1 and log 2 x < 0
Ê Ê ˆ ˆ fi x x
1
fi log1/ 2 x > 0 Á∵Á1 - ˜ > 0˜ x x
Ë Ë log1/ 2 (1 / 3) ¯
2
¯
x x>2
0

Ê 1ˆ
x > 0 and x < Á ˜ x Œ(0,1) » (2, • )
Ë 2¯
2
fi 0 < x <1 Ex-7. x log(2 x +3) x < 1
Ex-5. x log1/ 2 x + log3 x > 1 Soln. log(2 x +3) x 2 < 1
Soln. log1/ 2 x + log3 x > 1 x π 0, 2 x + 3 > 0, 2 x + 3 π 1
fi - log 2 x + log3 x > 1 fi x π 0, x > -3 / 2, x π -1
fi log3 x - log 2 x > 1 Case I: x
log10 x log10 x x2 > 2 x + 3
fi - >1
log10 3 log10 2
fi x2 - 2 x - 3 > 0
Ê 1 1 ˆ
fi log10 x Á -
Ë log10 3 log10 2 ˜¯
>1 fi ( x - 3) ( x + 1) > 0
fi x x
206 Comprehensive Trigonometry with Challenging Problems & Solutions for Jee Main and Advanced

0 < 2x + 3 < 1 fi ( x + 1) > 20 = 1


3
fi - < x < -1 fi x
2
x Œ(1, • )
Ê 3 ˆ 1 1
x ŒÁ - , -1˜ Ex-10. x £
Ë 2 ¯ Ê x + 1 ˆ log 4 ( x + 3)
log 4 Á
Case II: x+3>1 Ë x + 2 ˜¯
x2 < 2 x + 3 1 1
Soln. £
fi x2 - 2 x - 3 < 0 Ê x + 1 ˆ log 4 ( x + 3)
log 4 Á
Ë x + 2 ˜¯
fi ( x - 3) ( x + 1) < 0
fi x Ê x +1ˆ
fi log 4 Á ≥ log 4 ( x + 3)
x Œ( -1, 3) Ë x + 2 ˜¯

Ê x +1ˆ
fi log 4 Á - log 4 ( x + 3) ≥ 0
Ê 3 ˆ Ë x + 2 ˜¯
x ŒÁ - - 1˜ » ( -1, 3) - {0}
Ë 2 ¯ Ê x +1 ˆ
fi log 4 Á ˜ ≥0
Ë ( x + 2) ( x + 3) ¯
log 2 x - 3 log x + 3
Ex-8. x <1 Ê ˆ
log x - 1 x +1
fi Á ( x + 2) ( x + 3) ˜ ≥ 1
Ë ¯
log 2 x - 3 log x + 3
Soln. <1 Ê x +1 ˆ
log x - 1
fi Á ( x + 2) ( x + 3) ˜ - 1 ≥ 0
Ë ¯
a 2 - 3a + 3
fi <1 a x Ê ( x + 1) - ( x + 2) ( x + 3) ˆ
a -1 fi Á ≥0
a 2 - 3a + 3
Ë ( x + 2) ( x + 3) ˜¯
fi -1< 0 Ê ( x + 2) ( x + 3) - ( x + 1) ˆ
a -1 £0
fi Á
a 2 - 3a + 3 - a + 1 Ë ( x + 2) ( x + 3) ˜¯
fi <0
a -1 Ê x2 + 4 x + 5 ˆ
fi Á (x + 2 (x + 3 ˜ £ 0

a 2 - 4a + 4
<0 Ë ) )¯
a -1 Ê ˆ
1

1
<0
fi Á ( x + 2) ( x + 3) ˜ £ 0
Ë ¯
a -1
fi a fi -2 < x < 3
fi 0 < log x < 1 x Œ(-2, 3)
fi 0 < log10 x < 1
fi 1 < x < 10
Ex-11. x
(x 2
-4 ) <0
x Œ(1,10) (
log 2 x - 1 2
)
log 2 ( x + 1)
Ex-9. x >0 (x 2
-4 )
( x - 1) Soln. <0
(
log 2 x 2 - 1 )
log 2 ( x + 1)
Soln >0 x2 - 4 > 0
( x - 1)
1
x > 1, log 2 ( x + 1) > 0 fi <0
x > –1 (
log1/ 2 x 2 - 1 )
log 2 ( x + 1) > 0 x2 - 4 > 0
Logarithm 207

fi ( x + 2) ( x - 2) > 0 2
= 1 - (log10 2) + (log10 2)
2

fi x Œ(-•, - 2) » ( 2, • ) =1
1 2 log 2 + log 3
<0 y=
( )
log1/ 2 x 2 - 1 log ( 48) - log ( 4)
log (12) log (12)
fi log ( x - 1) > 0 2
= = =1
1/ 2
Ê 48 ˆ log (12)
log
fi ( x - 1) < 1
2 Ë 4¯
x=y
fi x2 - 2 < 0
fi - 2<x< 2 Ex-3.
x2 - 1 > 0 2 log x - log (2 x - 75) = 2
fi ( x + 1) ( x - 1) > 0 Soln.
fi x Œ( -•, - 1) » (1, • ) 2 log x - log (2 x - 75) = 2

fi ( )
log x 2 - log ( 2 x - 75) = 2
(
x Œ( -•, - 2) » - 2 , -1 » 1, 2 » ( 2, • ) ) ( ) Ê x2 ˆ
fi log Á ˜ =2
Ë 2 x - 75 ¯
Ê x2 ˆ
PROBLEMS FOR JEE ADVANCED EXAM fi log10 Á ˜ = log10 10
Ë 2 x - 75 ¯
2
( )
Ex-1. 4 5
a b =1 (
log a a b5 4
)
4 5 Ê x2 ˆ
a b =1
Soln. fi Á 2 x - 75 ˜ = 100
Ë ¯
fi (
log a 4b5 = log(1) = 0 )
fi x 2 = 200 x - 7500
fi 4 log a + 5 log b = 0
fi x 2 - 200 x + 7500 = 0
fi 4 log a = -5 log b
log a 5
fi =-
log b 4 Ex-4. (
log x log18 ( 2+ 8 =- )) 1
2
x
5
fi logb a = -
4 Soln. (
log x log18 ( 2+ 8 =- )) 12
log a a5b 4 ( )
= 5 log a a + 4 log a b
fi (
log x log18 ( 2+2 2 )) = -
1
2
Ê ˆ 1
= 5 + 4 log a b fi log x Á log
Ë (3 2 )
2 ( 3 2 )˜ = -
¯ 2
4
= 5+
Ê 1ˆ Ê
logb a fi log x Á ˜ Á log 3
Ë 2¯ Ë ( 2 () 3 2 )ˆ˜¯ = - 12
4 ¥ 4 25 - 16 9
= 5- = =
5 5 5 Ê 1ˆ 1
fi log x Á ˜ = -
Ë 2¯ 2
2
Ex-2. x = log10 5 ¥ log10 20 + (log10 2) 1
fi - log x 2 = -
2 log 2 + log 3 2
y= x=y
log ( 48) - log ( 4) 1
fi log x 2 =
Soln. 2
2
x = log10 5 ¥ log10 20 + (log10 2) fi x=2
2
= (1 - log10 2) (1 + log10 2) + (log10 2) fi x=4
208 Comprehensive Trigonometry with Challenging Problems & Solutions for Jee Main and Advanced

Ex-5. log 6 9 - log9 27 + log8 x = log 64 x - log 6 4 3c 3a

x fi 4 log3 a = 3 + log3 a
Soln. log 6 9 - log9 27 + log8 x = log 64 x - log 6 4 fi 3 log3 a = 3
fi log8 x - log 64 x = log9 27 - log 6 9 - log 6 4 fi log3 a = 1
1 3
fi a=3
fi log8 x - log8 x = log32 (3) - log 6 36
2 log a b = 2
1 3

2
log8 x = - log 6 62
2
( ) fi

log3 b = 2
b = 32 = 9
1 3 1
fi log8 x = - 2 = - logb c = 2
2 2 2
fi log9 c = 2
1 1
fi log8 x = -
2 2 fi c = 92 = 81
fi log8 x = -1 (a + b + c ) + 7
1
fi x=
8
= 100
Ex-6. x = 12 + 6 3 + 12 - 6 3 7 3
Ex-8. log9 x + log 4 y = log9 x - log8 y = -
log36 x 2 2
log 4 ( x + y - 3)
Soln. 12 + 6 3
= 12 + 2.3. 3 7
Soln. log9 x + log 4 y =
2
( 3)
2
= 32 + + 2.3. 3
1 1 7
fi log3 x + log 2 y =
= (3 + 3 )
2
2 2 2
fi log3 x + log 2 y = 7
( )
2
12 - 6 3 = 3 - 3 3
log9 x - log8 y = -
2
x = 12 + 6 3 + 12 - 6 3
fi log3 x - log 2 y = -3
(
= 3+ 3 + 3- 3 ) ( )
=6 2 log3 x = 4
log36 x fi log3 x = 2
= log36 6 fi x = 32 = 9
= log 62 6
1 2 log 2 y = 10
= log 6 6
2
fi 2y =5
1
= fi y = 25 = 32
2
x y = 32
Ex-7. log a b = 2, logb c = 2 log3 c = 3 + log3 a
(a + b + c ) + 7 Ex-9. a = log10 2, b = log10 3 3x+ 2 = 45
Soln. log a b ¥ logb c = 2 ¥ 2 = 4 x a b
fi log a c = 4 Soln. 3x+ 2 = 45


log3 c
=4 fi x + 2 = log3 (45)
log3 a
Logarithm 209



x + 2 = log3 (5 ¥ 9)
x + 2 = log3 5 + log3 9
P = log5 Ê log3
Ë ( 9 )ˆ¯
5

fi x + 2 = log3 5 + 2 ( ( ))
= log5 log3 9
1/10

= log (log (3 ))
fi x = log3 5 2 /10
5 3
log10 5
fi x=
log10 3 Ê 1ˆ
= log5 Á ˜ (log3 (3))
Ê 10 ˆ Ë 5¯
log10 Á ˜
fi x=
Ë 2 ¯ log10 10 - log10 2
=
-1
( )
= log5 5 = -1
log10 3 log10 3
ÊM ˆ
1- a ÁË + P + 3˜¯
fi x= N
b 16
= -1+ 3
Ex-10. N = 6 log10 2 + log10 31 27
N 16
= +2
27
Soln. N = 6 log10 2 + log10 31
70
6 =
= log10 2 + log10 31 27
= log10 (64 ¥ 31) Ex-12. log3 ( x ) = a log 7 ( x ) = b
= log10 (1984) log 21 ( x )
< log10 (1000) = 3 1 1
Soln. +
N = log10 (1984) > log10 (10000) = 4 a b
= log x 3 + log x 7
= log x ( 21)
Ê 1ˆ 1
M = log 2 2 Á ˜ , N = log32 (8) log 21 ( x ) =
log x ( 21)
Ex-11.
Ë 4¯ 2

P = log5 Ê log3
Ë ( 9 )ˆ¯
5
=
1
=
ab
1 1 a+b
+
ÊM ˆ a b
ÁË + P + 3˜¯ x y
N Ex-13.
log8 x + log 4 y 2 = 5 log8 y + log 4 x 2 = 7
Ê 1ˆ
( (2 ))
2
2 -2
Soln. M = log 2 Á ˜ = log
Ë 4¯ 2 xy

Ê 2 ˆ
2 Soln. log8 x + log 4 y 2 = 5
= Á- log 2 2˜
Ë 1/ 2 ¯ 1 2
fi log 2 x + log 2 y = 5
2 3 2
= ( -4) = 16
fi log 2 x1/ 3 + log 2 y = 5
(8) = (log 2 2 (8))
3 3
N = log 2 2
fi (
log 2 x1/ 3 y = 5 )
3ˆ3
= Ê log
Ë 2 2 (2 2 ) ¯ fi (x y) = 2
1/ 3 5
= 32

log8 y + log 4 x 2 = 7
( (2 2 ))
3
= 3 log 2 2 1 2
3
fi log 2 y + log 2 x = 7
= (3) = 27 3 2
210 Comprehensive Trigonometry with Challenging Problems & Solutions for Jee Main and Advanced

fi log 2 y1/ 3 + log 2 x = 7 fi x ( x - 2) = 1

fi (
log 2 y1/ 3 x = 7 ) fi x2 - 2 x - 1 = 0

fi ( y x) = 2
1/ 3 7
= 128 fi x2 - 2 x + 1 = 2

fi ( x - 1)2 = 2
(x 4/3 4/3
y ) = 2 .2
5 7
= 212
fi ( x - 1) = ± 2

fi ( xy )4/ 3 = 212 fi x =1± 2


3
12 ¥
fi x =1+ 2 = y
fi ( xy ) = 2 4

x+ y-2 2
fi ( xy ) = 29
(2 xy ) = 210 = 1024
(
= 1+ 2 +1+ 2 - 2 2 )
fi =2
Ex-14. ( )
log10 x 2 + x = log10 x3 - x ( ) Ex-16. a, b ΠR + log 27 a + log9 b =
7
2

Soln. ( )
log10 x 2 + x = log10 x3 - x ( ) log 27 b + log9 a =
2
3
ab

fi ( x + 1) = ( x - 1) 2
Soln. log 27 a + log9 b =
7
( x + 1) = ( x + 1) ( x - 1) 2

7
fi ( x - 1) = 1 fi log33 a + log32 b =
2
fi x=2 1 1 7
fi log3 a + log3 b =
3 2 2
fi 2 log3 a + 3 log3 b = 21
Ex-15. log10 ( x - 2) + log10 y = 0 2
log 27 b + log9 a =
x + y-2 = x+ y 3
1 1 2
(x + y - 2 2 ) fi
3
log3 b + log3 a =
2 3
Soln. log10 ( x - 2) + log10 y = 0 fi 2 log3 b + 3 log3 a = 4

fi log10 ( y ( x - 2)) = log10 1

fi y ( x - 2) = 1 fi 4 log3 a - 9 log3 a = 42 - 12

x + y-2 = x+ y fi -5 log3 a = 30

fi log3 a = -6
fi x + y - 2 + 2 x ( y - 2) = x + y
fi a = 3-6
fi -2 + 2 x ( y - 2) = 0
2 log3 b - 18 = 4
fi -2 = -2 x ( y - 2)
fi 2 log3 b = 22
fi x ( y - 2) = 1
fi log3 b = 11
fi b = 311
x=y
x=y ab = 3-6 ¥ 311 = 35 = 243
Logarithm 211

Ex-17. Find the number of values of x satisfying the equation Soln. Given c ( a - b ) = a (b - c )
(
log tan x 2 + 4 cos x = 2 in [0, 2p ]
2
) fi ac - bc = ab - ac
2 ac = ab + bc = b (a + c )
Soln. Given equation is log tan x 2 + 4 cos 2 x = 2 ( ) fi
2 ac
b=
fi (2 + 4 cos x) = tan2 2
x fi
(a + c )
sin 2 x log ( a + c ) + log (a - 2b + c )
fi (2 + 4 cos 2 x = ) cos x2
Now,
log (a - c )

fi (4 cos 4
x + 2 cos 2 x - sin 2 x = 0 ) log {( a + c ) ( a + c - 2b )}
=
log ( a - c )
fi (4 cos 4
x + 3 cos 2 x - 1 = 0 )
ÔÏ Ê 4ac ˆ Ô¸
fi (4 cos 4
x + 4 cos 2 x - cos 2 x - 1 = 0 ) log Ì( a + c ) Á a + c -
ÔÓ Ë (a + c ) ˜¯ ˝Ô˛
( ) ( )
=
fi 4 cos 2 x cos 2 x + 1 - 1 cos 2 x + 1 = 0 log ( a - c )

fi (4 cos 2
x - 1) (cos x + 1) = 02

=
log {((a + c) - 4ac)} 2

fi (4 cos 2
x - 1) = 0 log (a - c )
2
Ê 1ˆ
2 log ( a - c )
2 2Êpˆ =
fi cos x = ÁË ˜¯ = cos ÁË ˜¯ log ( a - c )
2 3


p
x = n p ± , n = 0,1, 2 2 log ( a - c )
3 = =2
log ( a - c )
p 2p 4p 5p
fi x= , , ,
3 3 3 3 ( )
Ex-20. If x = log10 A + B is a solution of 10 x + 10- x = 4,
Hence, the number of values of x is 4
( A + B + 3) .
Ê2 ˆ
Ex-18. If cos(ln x ÁË ¥ log ( x ) + 10˜¯ Soln. Given 10 x + 10- x = 4
p
1
Soln. Given cos (ln x ) = 0 fi 10 x + =4
10 x
p
fi ln x =
2 fi (10 ) - 4 (10 ) + 1 = 0
x 2 x

fi x= e2 fi (10 ) = 4 ± 16
x
2
-4

Ê2 ˆ
Thus, the value of ÁË ¥ log ( x ) + 10˜¯
p
fi (10 ) = 4 ± 2 12
x

2 Ê pˆ fi (10 ) = (2 + 3 )
x

= ¥ log Á e 2 ˜ + 10
p Ë ¯ fi x = log ( 2 + 3 )
10

Ê2 p ˆ Thus A = 2, B = 3
= Á ¥ log (e) + 10˜
Ëp 2 ¯
Hence, the value of A + B + 3 = 2 + 3 + 3 = 8.
= 1 + 10 = 11 Ex-21. Solve for x:
Ex-19. If c ( a - b ) = a (b - c ) such that a π b π c
log ( a + c ) + log (a - 2b + c )
(
log10 98 + x3 - x 2 - 12 x + 36 = 2 )
the value of [Roorkee-1975]
log (a - c )
212 Comprehensive Trigonometry with Challenging Problems & Solutions for Jee Main and Advanced

Soln. The given equation is fi y = 10, - 10 / 7

( 3 2
log10 98 + x - x - 12 x + 36 = 2 ) when y = 10 , then 2 log10 x = 10
fi log10 x = 5
fi (98 + x - x - 12 x + 36 ) = 10
3 2 2
= 100
fi x = 105
fi x3 - x 2 - 12 x + 36 = 4 when y = -10 / 7 , then 2 log10 x = -10/ 7
3 2
fi x - x - 12 x + 32 = 0 fi log10 x = -5/ 7
fi x3 + 4 x 2 - 5 x 2 - 20 x + 8 x + 32 = 0 fi x = 10-5/ 7
fi x 2 ( x + 4) - 5 x ( x + 4) + 8 ( x + 4) = 0 Ex-23. Solve for x:

fi ( x + 4 ) ( x 2 - 5 x + 8) = 0 a
Ê xˆ
6 log a x log10 a log a 5-3 log10 ÁË 10 ˜¯
= 9log100 x + log 4 2
fi ( x + 4) = 0 5
[Roorkee-1988]
fi x = -4
Soln. We have
Hence, the solution set is {-4}
Ê xˆ
Ex-22. Solve for x and y: 6 log a x log10 a log a 5-3 log10 ÁË 10 ˜¯
a = 9log100 x + log 4 2
5
log10 x + log10 x1/ 2 + log10 x1/ 4 + .... = y log x log 5
fi 6 log a . log 10 .-3(log10 x -1) log100 x + log 2 2
1 + 3 + 5 + ..... + (2 y - 1) 20 a =9 2

and = 5
4 + 7 + 10 + ... + (3 y + 1) 7 log10 x 6 log a x log10 5.-3(log10 x -1) log 2 x + log 2 2
[Roorkee-1987] fi a = 9 10 2

5
Soln. We have log10 x + log10 x1/ 2 + log10 x1/ 4 + .... = y 6 log a x log10 5.-3(log10 x -1)
1
(log10 x +1)
fi a = 92
1 1 5
fi log10 x + log10 x + log10 x + .... = y
2 4 6 log a x log10 5.-3(log10 x -1)
= 3( 10 )
log x +1
fi a
Ê 1 1 ˆ 5
fi y = Á1 + + + ....˜ log10 x
Ë 2 4 ¯ 2 log a x log10 5 3-3 log10 x
fi a .a = 3log10 x
Ê ˆ 5
Á 1 ˜ 2 log10 x log a 5 3-3 log10 x
fi y=Á log x = 2 log10 x fi a .a = 3log10 x
1 ˜ 10 5
Á 1 - ˜
Ë 2¯ 2 b log a 5 3-3b
fi a .a = 3b , b = log10 x
1 + 3 + 5 + ..... + (2 y - 1) 20 5
Also, = It is possible only when, b = 1
4 + 7 + 10 + ... + (3 y + 1) 7 log10 x
Thus, log10 x = 1
y
(1 + 2 y - 1) 20 fi x = 101 = 10
2 =
fi y Ê yˆ
(4 + 3 y + 1) 7Á ˜ Hence, the solution is x = 10
2 Ë 2¯
Ex-24. Solve for x:
2y 40 log3 x 2 - 2 log x 9
fi = x -1 = ( x - 1)
7
( 3 y + 5 ) y7 [Roorkee-1990]
y 20 Soln. We have
fi =
( 3 y + 5 ) y7 x -1
log3 x 2 - 2 log x 9
= ( x - 1)
7

fi 7 y 2 = 60 y + 100 log3 x 2 - 2 log x 9 7


when x > 1, then ( x - 1) = ( x - 1)
2
fi 7 y - 60 y - 100 = 0
fi log3 x 2 - 2 log x 9 = 7
fi ( y - 10) (10 y + 7) = 0
Logarithm 213

fi log3 x 2 - 2 log x 32 = 7 Ê yˆ
fi Á x˜ =2
fi 2 log3 x - 4 log x 3 = 7 Ë ¯
4 y
fi 2 log3 x - =7 fi x =
log3 x 2
y
2 (log3 x ) - 7 log3 x - 4 = 0
2
fi fi x=± ........(ii)
2
fi 2a 2 - 7 a - 4 = 0 , a = log3 x From (i) and (ii), we get,
fi 2a 2 - 8 a + a - 4 = 0 20
y=± , ± 20
fi 2 a ( a - 4 ) + 1( a - 4 ) = 0 3


(a - 4) (2a + 1) = 0 10
and x = ± , ∓ 10
3
1 Hence, the solutions are
fi a = 4, -
2 10 20
Ï ¸
when a = 4 , then log3 x = 4 Ì x = , y = ; x = 10, y = -20 ˝
Ó 3 3 ˛
fi x = 34 = 81 Ex-26. Find all real number x which satisfy the equation
1 1
when a = - , then log3 x = -
2 2 (
2 log 2 log 2 x + log1/ 2 log 2 2 2 x = 1 )
[Roorkee-1999]
1
fi x = 3-1/ 2 =
3 Soln. We have 2 log 2 log 2 x + log1/ 2 log 2 2 2 x = 1 ( )
It is not possible, since x > 1
fi (
2 log 2 log 2 x - log 2 log 2 2 2 x = 1 )
Hence, the solution is {2, 81} .
Ex-25. Solve for x and y fi
2
( (
log 2 (log 2 x ) - log 2 log 2 2 2 x = 1 ))
1 Ê ˆ
log100 x + y =
2
and log10 y - log10 x = log100 4
log 2 Á ( log 2 x )
2
˜ =1
[Roorkee-1996]

Ë( (
Á log 2 2 x
2 )) ˜
¯
1
Soln. We have log100 x + y = Ê ˆ
Á ( 2 )
2
2 log x
˜ =2
fi log102 x + y =
1

Ë ( (
Á log 2 2 x
2 )) ˜
¯
2
Ê ˆ

1
log10 x + y =
1
Á (log 2 x )2 ˜ =2


2
log10 x + y = 1
2 fi
Ë ( ( )
Á log 2 2 + log x
2 2 ) ˜
¯

fi x + y = 10 Ê ˆ
Á (log 2 x )2 ˜ =2
( x + y ) = ±10
fi ..........(i) fi
Ë ( ( )
Á log 23/ 2 + log x
2 2 ) ˜
¯
Ê yˆ
Also, log10 Á ˜ = log100 4 Ê ˆ
Ë x¯
Á (log 2 x )2 ˜
Ê yˆ fi Á ˜ =2
fi log10 Á ˜ = log102 22 Á Ê 3 + log xˆ ˜
ÁË ÁË 2 2 ˜˜
Ë x¯ ¯¯
Ê yˆ Ê ˆ
fi log10 Á ˜ = log10 2 2
Ë x¯ Á a ˜
fi Á3 ˜ = 2 , a = log 2 x
Á + a˜
Ë2 ¯
214 Comprehensive Trigonometry with Challenging Problems & Solutions for Jee Main and Advanced

Ê 2a 2 ˆ Soln. We have y = log 2 x + log 4 x + log8 x + ....


fi Á 3 + 2a ˜ = 2 1 1
Ë ¯ fi y = log 2 x + log 2 x + log 2 x + ....
2 4
fi 2a 2 - 4a - 6 = 0 Ê 1 1 ˆ
fi y = Á1 + + + ....˜ log 2 x
fi a 2 - 2a - 3 = 0 Ë 2 4 ¯
Ê ˆ
fi (a - 3) (a + 1) = 0 Á 1 ˜
fi y=Á log 2 x = 2 log 2 x
fi a = -1, 3 1˜
Á1- ˜
1
Ë 2¯
when a = -1fi log 2 x = -1 fi x = 2-1 =
2 5 + 9 + 13 + ..... + (4 y + 1)
Also, = 4 log 4 x
1 + 3 + 5 + .... + (2 y - 1)
when a = 3 fi log 2 x = 3 fi x = 23 = 8
y
Hence, the solution is x = 8
2
(2.5 + ( y - 1) 4)
Ex-27. Solve for x; fi = 4 log 4 x
y
( ( ))
2 . + y - 1 2
( ) ( )
-1 2
log3/ 4 log8 x 2 + 7 + log1/ 2 log1/ 4 x 2 + 7 = -2
(10 + 4 y - 4) 4
[Roorkee-2000] fi = log 4 x
( 2 + 2 y - 2)
Soln. We have
(6 + 4 y ) 4
( ) ( )
-1 = log 4 x
log3/ 4 log8 x 2 + 7 + log1/ 2 log1/ 4 x 2 + 7 = -2 fi
(2 y )
(3 + 2 y ) 4
( ) ( )
-1
fi log3/ 4 log 3 x 2 + 7 + log -1 log -2 x 2 + 7 = -2 fi = log x = 2 log 2 x= y
2 2 2 22
y
Ê1
( ( 2 ˆ
))
Ê1
( (2 ˆ
fi log3/ 4 ÁË log 2 x + 7 ˜¯ - log 2 ÁË log 2 x + 7 ˜¯ = -2
3 2
)) fi y2 = 2 y + 3

fi y2 - 2 y - 3 = 0
Ê1 ˆ Ê1 ˆ
( (2
fi log3/ 4 ÁË y˜¯ - log 2 ÁË y˜¯ = -2 , y = log 2 x + 7
3 2
)) fi (y – 3)(y + 1) = 0
fi y = –1, 3
Ê yˆ Ê yˆ fi y = 3 (y = –1 is not possible)
fi log3/ 4 Á ˜ - log 2 Á ˜ = -2
Ë 3¯ Ë 2¯
when y = 3, then 2 log 2 x = 3
fi y = 4 , by trial
3
fi log 2 x =
fi ( 2
log 2 x + 7 = 4 ) 2
fi x = 23/ 2
fi (x 2
)
+ 7 = 24 = 16
Hence, the solutions are x = 23/ 2 and y = 3
fi x2 = 9
LEVEL I
fi x = ±3
(PROBLEMS BASED ON FUNDAMENTALS)
Hence, the solutions are {-3, 3}
EXERCISE 1
Ex-28. Solve for x and y:
1. Find the value of
log 2 x + log 4 x + log8 x + ..... = y
(i) log 2 32
5 + 9 + 13 + ..... + (4 y + 1) Ê 1 ˆ
= 4 log 4 x (ii) log3 Á
1 + 3 + 5 + .... + (2 y - 1) Ë 243 ˜¯
[Roorkee-2001]
(iii) log5 5
5
Logarithm 215

(iv) log100 (0.1) 11. If a = log 4 5 and b = log5 6 log3 2 .


(v) 10log10 m + log10 n 12. Find the value of log12 54 , where b = log12 24 .
(vi) log3 log5 log3 ( 243) 1
+
1
13. Find the value of
2. Find the value of log 2 36 log3 36
(i) log 2 log3 log 4 (64) 1 1
14. Prove that + >2
1
- log3 7 log3 p log 4 p
(ii) 3 3
16 25 81
(iii) 2 2 - log 2 5 15. Simplify: 7.log + 5.log + 3.log
15 24 80
log 2
(iv) 2log3 5 - 5 3 16. If a 2 + b 2 = 7ab , then prove that
(v) log9 27 - log 27 9 1 1
log ( a + b ) = (log a + log b )
3. Find the value of 3 2
(i) log10 tan 40∞ + log10 tan 41∞ + log10 tan 42∞
17. If a 2 + b 2 = 11ab , then prove that
+....... + log10 tan 50∞
Ê a - bˆ 1
log Á = (log a + log b )
(ii) log10 tan 1∞ + log10 tan 2∞ + log10 tan 3∞ Ë 3 ˜¯ 2
+............ + log10 tan 89∞
log a n
(iii) log3 4.log 4 5.log5 6.log 6 7.log 7 8.log8 9 18. Prove that = 1 + log a b .
log ab n
4. Find the value of 19. If log 225 = a and log 225 = b, then prove
(i) x, if log5 a .log a x = 2 , Ê Ê 1ˆ 2 ˆ Ê 1 ˆ
(ii) x, if log k x .log5 k = log k 5 , where that log Á Á ˜ ˜ + log Á = 2a - 3b - 1 .
Ë Ë 9¯ ¯ Ë 2250 ˜¯
k π 1, k > 0
20. If a, b, c are in G.P, then prove that, log a n, logb n, log c n
(iii) A + B + 10, if A = log 2 log 2 log 4 256 are in H.P.
and B = 2 log 2, 7ˆ
2
(iv) log 300 ., if a = log 5 , b = log 2
( Ê
)
21. If log3 2, log3 2 x - 5 , log3 Á 2 x - ˜ are in A.P.,
Ë 2¯
3 3 3
5. Find the minimum value of x.
1 1
(i) logb a + log a b 1 - log a x 1 - log y
22. If y = a , z=a a
, then prove
(ii) logb a + logc b + loga c.
1
1 1 1 1 - log z
6. Prove that + + that, x = a a
.
log 2 n log3 n log 4 n
1 1 23. If x = log c b + logb c , y = log a c + log c a
+............. + =
log 43 n log 43 ! n and z = logb a + log a b

7. If n = 1983, then prove that of x 2 + y 2 + z 2 - xyz.


1 1 1 1 log a log b log c
+ + + ...... + =1 24. If = = , then prove that a a .bb .c c = 1
log 2 n log3 n log 4 n log1983 n b-c c-a a-b
8. Determine b satisfying 1 4

(i) log b = 3 1
log5 3 log9 36 log 7 9
25. Find the value of 81 + 27 +3
8
3 26. Prove that
1
(ii) 1 1
log 8
b= 33 +
1 + logb a + logb c 1 + log c a + log c b
(iii) log a 2.logb 625 = log10 16.log a 10
1
(iv) log3 log 2 log (625) = b + =1
5 1 + log a b + log a c
9. If log a ab = x logb ab . 27. Find x, if log 2 x + log 4 x + log8 x = 11
10. If log10 2 = x 10 5.
216 Comprehensive Trigonometry with Challenging Problems & Solutions for Jee Main and Advanced

28. Find x, if log 2 x + log 4 x + log8 x + log16 x =


25 49. Find x, if 34 log9 ( x +1) = 22 log 2 x + 3
4 50. If a = log 24 12 , b = log36 24 and c = log 48 36
29. If log a x = a , logb x = b , log c x = g and log d x = d ,
Ê abc + 1ˆ
then prove that Á = 2.
log abcd x . Ë bc ˜¯
30. If x = log a bc , y = logb ca and z = log c ab Ê Ê p ˆˆ 1
51. If log10 Á sin Á x + ˜ ˜ = (log10 6 - 1)
1 1 1 Ë Ë 4¯¯ 2
the value of + +
1+ x 1+ y 1+ z the value of log10 sin x + log10 cos x .
1 52. If a, b, c are in G.P., then prove that
log 4
31. If y = 2 x
x. 1 1 1
, , are in H.P.
1 + log a 1 + log b 1 + log c
32. If N = 6log10 40. 5log10 36 N + 10 .
log
Ê1 1 1 ˆ
+ + +............to •˜
53. Find x, if 5log10 x = 50 - x log10 5
33. Find the value of (0.5) 2 ËÁ 4 8 16 ¯ .
54. Find x, if log5 ÈÎ2 + log (3 + x )˘˚ = 0 .
log10 3 log10 2
34. If x = 2 and y = 3 , then find a relation Q. Find the number of real solutions of
between x and y. 55. log 4 ( x - 1) = log 2 ( x - 3)
35. Find the value of
56. log 4 ( x - 2) = log 2 ( x - 2)
2log10 3 - log10 5 ¥ 3log10 5 - log10 2 ¥ 5log10 2 - log10 3.
57. log9 ( x - 1) = log3 ( x - 1) .
36. If a = log30 3 and b = log30 5
58. log 2 x + log 2 ( x + 3) = 1 / 4
log10 8 .
37. If a = log12 18 and b = log 24 54 , then prove that ( )
59. log 4 x + x - log 4 ( x + 1) = 2
2

ab + 5 ( a - b ) = 1 60. 1 + 2 log( x + 2) 5 = log5 ( x + 2)


38. If a = log 7 12 and b = log12 24 61. log 2 x + log 4 ( x + 2) = 2
log54 168 . 62. log10(x – 1)3 – 3 log10(x – 3) = log10 8

39. If a = log 6 30 , b = log15 24 , then prove that Q Solve for x:

Ê 2ab + 2a - 1ˆ
(
63. log5 x 2 - 3x + 3 > 0 )
log12 60 = Á
Ë ab + b + 1 ˜¯
( ( ))
64. log 7 log5 x 2 - 7 x + 15 > 0
40. Find x, if log 7 log5 ( ( x+5 + x =0 )) 65. log( ) (log ( x - 7 x + 17 )) > 0
1/ 2 5
2

log( ) (log (log ( x - 6 x + 40))) > 0


41. If log 2 x + log 2 y ≥ 6 2
x + y. 66. 1/ 2 5 2

log (log (log ( x - 9 x + 50))) > 0


42. Solve for x and y: 4 log x
=3 log y
, (3 x)log 3 =(4 y)log 4 2
67. 3 5 2
18 21 28
43. If x = y =z = k , then prove that,
Ê x - 2ˆ
68. log 6 Á >0
3, 3 log y x , 3 log z y, 7 log x z are in A.P. Ë 6 - x ˜¯
1 1 69. log(1/2)x > log(1/3)x
44. If + >x x.
log3 p log 4 p
45. If log 0.3 ( x - 1) < log 0.09 ( x - 1) x.
(
70. log 0.5 x 2 - 5 x + 6 > -1 )
46 If log e log5 ( 2x - 2 + 3 = 0 ) x.
2
(
71. log8 x - 4 x + 3 < 1 )
Ê 35 - x 2 ˆ 1
47. Find the least value of 2.log10 x - log x (0.01) for x > 1. 72. log(1/ 4) Á ≥-
˜
Ë x ¯ 2
48. Find x, if 4log9 3 + 9log 2 4 = 10log x 83
Logarithm 217

Q Solve for x: 4. If x = log3 5 and y = log 27 25 , then


( )(
) (2 x +5 x) ( x2 - 1)
73. log x3 + 6 x 2 - 1 = log 2 (a) x > y (b) x = y

74. log (3x 2 + x - 3) = 3 log (3x - 2) (c) x < y (d) x 2 = y

75. log x -1 ( x3 + 6) = log x -1 ( 2 x 2 + 5 x ) ( ) ( )


5. If log10 2 , log10 2 x + 1 , log10 2 x + 3 are in A.P.
( ) 2
( ) 2
then
(
76. log3 x 2 - 3x - 5 = log3 (7 - 2 x )) (a) x = 0 (b) x = 1
1
1 (c) x = log10 2 (d) x = log 2 5
77. log ( 2
)
x - 1 + log ( 2 x + 15) = 1 2
6. If log a ( ab ) = x , then logb ( ab ) is
( )
78. log(3 x + 4) 4 x 2 + 4 x + 1 + log(2 x +1) 6 x 2 + 11x + 4 = 4 ( ) (a)
x
(b)
x
79. 5 log10 x = 50 - x log10 5 1- x 1+ x
x
80. 4log9 x - 6.2log9 x + 2log3 27 = 0 (c) (d) None
x-2
Q (Solve the inequality, wherever base is not given, take 7. The value of 42 log9 3 is
it as 10) (a) 9 (b) 2
2 (c) 4 (d) 3
Ê Ê x5 ˆ ˆ
81. (log 2 x ) 4
- Á log1/ 2 Á ˜ ˜ - 20 log 2 x + 148 < 0 8. If log 7 2 = x , then log 49 ( 28) is
Ë Ë 4 ¯¯
Ê 1ˆ Ê 1ˆ
2
82. (log 100 x ) + (log 10 x ) + log x £ 14
2 (a) ÁË x + ˜¯ (b) ÁË x - ˜¯
2 2
83. log1/ 2 ( x + 1) > log 2 (2 - x ) Ê 1ˆ Ê 1ˆ
(c) - Á x - ˜ (d) - ÁË x + ˜¯
2
(
84. log1/ 5 2 x 2 + 5 x + 1 < 0 ) Ë 2¯

85. log1/ 2 x + log3 x > 1


( (
9. If log 2016 log5 ))
2 x - 2 + 3 = 0 , then x is

(a) 1/3 (b) 1/2


Ê 4x + 5ˆ
86. log x Á < -1
Ë 6 - 5 x ˜¯
(c) 3 (d) 2
1 1
10. If + > x , then x is
Ê x + 2ˆ log 2 p log 6 p
87. log 0.2 Á £1
Ë x ˜¯ (a) 2 (b) 3
( )
88. log10 x 2 - 16 £ log10 ( 4 x - 11) (c) 4 (d) 5
11. The value of 5log 2 7 - 7log 2 5 is
(a) 5 (b) 0
LEVEL II (c) 7 (d) 2
(MIXED PROBLEMS) 12. If log10 2 = x , then log10 5 is
1. If a, b, c are in G.P. then log 2016 a, log 2016 b, log 2016 c (a) 1 (b) 1 - x
are in (c) x + 1 (d) 2x
(a) G.P. (b) A.P. 13. The number of real solutions of
(c) H.P. (d) A.G.P. log 2 x + log 4 ( x + 2) = 2 is
1
log x 9 (a) 1 (b) 2
2. If y = 3 , then x is (c) 3 (d) 0
(a) y (b) y 14. The number of real solutions of
(c) y 2 (d) y 3 1 + log 2 ( x - 1) = log( x -1) 4 is
1 1 2 (a) 1 (b) 2
3. If + = , then a, b, c are in (c) 3 (d) 0
log a x log c x logb x log ( x - 2)
(a) A.P. (b) G.P. 15. The number of real solutions of x x
= 9 is
(c) H.P. (d) A.G.P. (a) 4 (b) 3
(c) 2 (d) 1
218 Comprehensive Trigonometry with Challenging Problems & Solutions for Jee Main and Advanced

Ê 1 10. Solve for x:


1 ˆ
1 1
16. The value of Á +
Ë log3 p log 4 p ˜¯
lies between
2
(
log5 ( x + 5) + log5 x - 3 = log5 ( 2 x + 1)
2
)
(a) (1, 2) (b) (2, 3) 11. Solve for x:
3
(c) (3, 4) (d) (0,1) 2 3
log 4 ( x + 2) + 3 = log 4 ( 4 - x ) + log 4 (6 + x )
3
2
17. The number of real roots of x ln x - 1 = 0 is 12. Solve for x:
(a) 2 (b) 1 1 + log 2 ( x - 4)
=1
18. The number of real roots of 2 - x - ln x = 0 is
(
2 log 2 x + 3 - x - 3 )
13. Solve for x:
(a) 1 (b) 2
Ê 1ˆ Ê x 3 + 27 ˆ
ÁË1 + ˜¯ log 3 = log Á
2x Ë 4 ˜¯
19. If 3x = 10 - log 2 x , then x is
14. Solve for x:
(a) 0 (b) 1 2
(c) 2 (d) 3 4log10 x +1 - 6log10 x - 2.3log10 x +2
=0
20 If 1 - log1/ 5 x + 2 = 3 - log1/ 5 x , then x is 15. Solve for x:

( ) ( )
2
(a) 2 (b) 5 log3 x+ x -1 = log3 4 3 - 3 + 4 x -1
(c) 1 (d) 3
COMPREHENSIVE LINK PASSAGES
LEVEL III
(TOUGHER PROBLEMS FOR JEE ADVANCED )
PASSAGE I
1. Solve for x: x + log10 1 + 2 ( x
) = x log 10 5 + log10 6 Let A be the sum of the roots of
1 4
+ =3
x2 - x - 1 5 - 4 log 4 x 1 + log 4 x
2. Solve for x: log =0
x2 + x - 2 B be the product of m and n, where
2m = 3 and 3n = 4
3. Solve for x: 4 + log1/ 7 x = 2 + 2 + log1/ 7 x and C be the sum of the integral roots of
4. Solve for x
Ê 3ˆ
log3 x Á ˜ + (log3 x ) = 1
2
Ê 4ˆ Ê 4 ˆ 2Ê 2 ˆ
log 2 Á1 + ˜ + log 2 Á1 - ˜¯ = 2 log ÁË - 1˜ Ë x¯
Ë x ¯ Ë x+4 x -1 ¯
Then
5. Solve the system of equations: 1 The value of A + B is
Ï 8 (a) 10 (b) 6
Ôlog y x - log x y =
Ì 3 (c) 8 (d) 4
ÔÓ xy = 16 . 2. The value of B + C is
(a) 6 (b) 2
6. Solve for x:
(c) 4 (d) 8
( )
log 3x 2 + 12 x + 19 - log (3x + 4) + log32 4 3. The value of ( A + C ∏ B ) is
= 1 - log1/16 ( 256 )
5 (a) 5
(c) 7
(b) 8
(d) 4
7. Solve for x:
Ê 1ˆ Ê 1ˆ PASSAGE II
log 2 ( 4 - x ) + log (4 - x ) .log Á x + ˜ - 2 log 2 Á x + ˜ = 0
Ë 2¯ Ë 2¯ A function f : R + Æ R f (x) = loga x, x > 0,
8. Solve for x: a > 0, a π 1

( ( )) (
log3/ 4 log8 x 2 + 7 + log1/ 2 Ê log1/ 4 x 2 + 7
Ë )
-1 ˆ
¯
= -2 Then D f = R + and R f = R
Ê x - 3ˆ
9. Solve for x: 1. If f ( x ) = log ÁË ˜ , then the domain of the
5 - x¯
( )
log10 x 2 - x - 6 - x = log10 ( x + 2) - 4
function f ( x ) is
Logarithm 219

(a) (3, 5) (b) ( -•, 5) (R) The value of


log 210 5 + log10 5.log10 20 + log 210 2 - 1 (3) 1
(c) (5, • ) (d) None
is
2. Let f ( x ) = ( - x + 3x - 2) . Then the domain of the
2
(S) The value of a for which (4) –1
function f(x) is log a 7
= log9 36 holds good is
(a) ( -1, 2) (b) (1, 2) log 6 7
codes:
(c) (-•,1] (d) [2, • ) P Q R S
3. Let f ( x) =
x - 2 + 4 - x . Then the range (A) 2 3 1 4
of the function f(x) is (B) 4 3 1 2
(C) 4 3 2 1
(a) È 2 , 2˘ (b) [1, 2]
Î ˚ (D) 3 4 2 1
(c) ( 2, 4) (d) [ 2, 4]
MATCH MATRIX
MATCHING LIST TYPE Column-I Column-II
(ONLY ONE OPTION IS CORRECT) (A) If a the root of (P) 2
3x log3 4 + 4log3 x = 64
This section contains four questions, each having two
matching list. Choices for the correct combination of then a + 1 is
elements from (B) The integral value of (Q) 4
List-I and List-II are given as options (A), (B), (C) and 2
of x in log 2 x - log 2 x - 2 = 0 is
(D), out of which ONE is correct.
1. Match the following lists (C) The value of 4 log 2 x is (R) 3
List I List II where x = 3 + 2 2 + 3 - 2 2
Ê log 2 32 ˆ
(P) The value of Á ˜ is (1) 2/7 (D) If a 2 + b 2 = 1 , then the value (S) 1
Ë log3 243 ¯
(Q) The value of (2) –2
(
of log ab a3b5 + a5b3 is )
Ê 2 log 2016 6 ˆ INTEGER TYPE QUESTIONS
ÁË log 12 + log 3 ˜ is
2016 2016 ¯ n -1
Ê r - 1ˆ 99
1. If  log 2 Á = ’ log ( r + 1)
Ê -2 Ë r + 1˜¯ r =10 r
Ê 1ˆ ˆ r =0
(R) The value of Á log1/ 4 Á ˜ ˜ is (3) 1 the value of n.
Ë Ë 16 ¯ ¯
2. Solve for x: 7log 2 x = 98 - x log 2 7
Ê log5 16 - log5 4 ˆ
(S) The value of Á is (4) 2 3. Solve for x: 4log3 x = 32 - x log3 4
Ë log5 128 ˜¯
4. If a and b are the roots of
codes:
P Q R S 3 log x 4 + 2 log 4 x 4 + 3 log16 x 4 = 0
(A) 2 3 1 4 1Ê 1 1ˆ
the value of +
2 ÁË a b ˜¯
(B) 4 2 1 3
(C) 4 3 2 1
5. Solve for x:
(D) 3 1 4 2
2. Match the following lists ( )
x + log10 2 x + 1 = log10 6 + x log10 5
List-I List-II
6. Solve for x:
Ê 2 log 2 + log 3 ˆ
(P) The vaue of ÁË log 48 - log 4 ˜¯ is (1) 3 ( )
log x -1 x3 - 9 x + 8 .log x -1 ( x + 1) = 3

1 Ê 64 ˆ 7. If a and b are the solutions of


log 3 Á
Ë 27 ˜¯
(Q) The value of (2) 0.
6 x-2 ( )
log 2 x3 - 3 log x 4
= ( x - 2)
3
2

the value of (a + b + 3)
220 Comprehensive Trigonometry with Challenging Problems & Solutions for Jee Main and Advanced

8. If a integral solutions of 7. Solve for x:


6 (log x 2 - log 4 x ) + 7 = 0 ( ) (
log(2 x +3) 6 x 2 + 23x + 21 = 4 - log(3 x + 7) 4 x 2 + 12 x + 9 )
Ê 2a - 1ˆ [IIT-JEE- 1987]
the value of Á
Ë 5 ˜¯
.
5
(3/ 4)(log 2 x )2 + log 2 x -
9. Let The number N = 6 log10 2 + log10 31 . 8. The equation x 4 = 2 has
If N (a) at least one real solution
their sum. (b) exactly three real solutions
10. Find the value of the expession (c) exactly one irrational solutions
Ê1 1 1 ˆ
(d) complex roots [IIT-JEE-1989]
(0.16)log 2.5 Á + + +......˜
Ë 3 32 33 ¯
.

Questions asked in PAST IIT-JEE EXAMS ( ) Ê
9. If log3 2, log3 2 x - 5 , log3 Á 2 x - ˜ are in A.P
Ë 2¯
x. [IIT-JEE-1990]
1. For a > 0, solve for x, the equation
10. The number log 2 7 is
2 log x a + log a x a + 3 log a 2 x a = 0 [IIT-JEE-1978] (a) an integer (b) a rational number
2. The least value of the expression (c) an irrational no. (d) a prime number.
2 log10 x - log x (0.01) , x > 1 is [IIT-JEE-1990]
11. The number of solutions of
(a) 10 (b) 2
log 4 ( x - 1) = log 2 ( x - 3) is
(c) –0.01 (d) None of these
[IIT-JEE - 1980] (a) 3 (b) 1
x (c) 2 (d) 0
3. y = 10 y = log10 x in the line [IIT-JEE-2001]
whose equation is ................
12. Let ( x0 , y0 ) be the solution of the following equations
[IIT-JEE-1982]
4. For 0 < a < x , the minimum value of (2 x )ln 2 = (3 y )ln 3 , 3ln x = 2ln 3 . Then x0 is
log x a + log a x is ...... [IIT-JEE-1984] 1 1
(a) (b)
5. If log 0.3 ( x - 1) < log 0.09 ( x - 1) , then x lies in 6 3
1
(a) (2, • ) (b) (1, 2) (c) (d) 6
2
(c) (-2, - 1) (d) None of these
13. The value of
[IIT-JEE-2011]

[IIT-JEE-1985]
Ê ˆ
6. The solution of the equation 1 1 1 1
6 + log3/ 2 Á 4- 4- 4- .......to • ˜
( (
log 7 log5 ))
x + 5 + x = 0 is ......
[IIT-JEE-1986]
ÁË 3 2 3 2 3 2 3 2 ˜¯
is..... [IIT-JEE-2012]

ANSWERS
LEVEL I 10. (1 - x )
1. (i) 5 (ii) –5 (iii) 2/3
1
(iv) –1/2 (v) m+n (vi) 0 11.
3. (i) 0 (ii) 0 (iii) 2 (2ab - 1)
4. (i) 25 (ii) 25 (iii) 12 13. 1/2
5. (i) 2 (ii) 3 15. log 2
8. (i) 32 (ii) 2 (iii) 24 (iv) 21 21. x=3
Ê x ˆ 25. 216
9. Á
Ë l - 1˜¯ 27. 64
28. 25/4
Logarithm 221

Ê 1 ˆ È 1 ˘
29. Á -1 82. x Œ Í ,10˙
-1 -1 -1 ˜
Ëa + b +g +d ¯ Î 10 ˚

83. -1 < x < 1 - 5 or 1 + 5 < x < 2


30. 1
31. x = y2 2 2
32. 226 84. x Œ(-•, -2.5) » (0, • )
33. 4 1
34. x=y 1- log 3
85. 0 < x < 3 (where base is 2)
40 x=4
86. 1/2 < x < 1
41. 16.
x=2 Ê 5˘
44. 87. x ŒÁ -•, - ˙ » (0, • )
45. (2, • ) Ë 2˚
46. x=3 88. x Œ( 4, 5]
47. 4
48. x = 10 LEVEL II
49. x=1 1. (b) 2. (c) 3. (a) 4. (c)
51. –1 5. (b) 6. (a) 7. (c) 8. (a)
53 x = 100 9. (c) 10. (a) 11. (b) 12. (b)
54. x=1 13. (a) 14. (b) 15. (d) 16. (b)
55. 1 17. (b) 18. (a) 19. (c) 20. (b)
56. 1
57. 1 LEVEL III
58. 2
1. 1
59. 1
Ï 3 1 3 Ô¸
60. 1 2. ÔÌ- , , ˝
61. 1 ÔÓ 2 2 2 Ô˛
62. 1
3. (0, 49]
63. (-•,1) » (2, • )
64. (-•, 2) » (5, • ) 4. { 2 or 6 }
65. (3, 4) 5. 5/3
6. {-1, 7}
66. (2, 4)
ÏÔ 7 3 + 24 ¸Ô
67. (-•, 3) » (6, • ) 7. Ì0, , ˝
68. (4, 6) ÓÔ 4 2 ˛Ô
8. x = 3 or –3
69. 0 < x < 1
70. (1, 4) (
9. x Œ(0,1) » 19834 , • )
71. (-1, 5) Ê 12 ˆ
10. Á , •˜
72. (-1, 0) » (5, • ) Ë 5 ¯
73. x = 3
COMPREHENSIVE LINK PASSAGES
74. x = 1, 5 ± 10
9 Passage-I: 1. (c) 2. (a) 3. (b)
75. x = 3 Passage-II: 1. (a) 2. (b) 3. (a)
76. x = –3
77. x = 5 MATCHING LIST
78. x = 3/4 1. (C)
79. 100 2. (D)
80. {9, 81} 3. (A) Æ (Q); (B) Æ (P); (C) Æ (Q); (D) Æ (R)
Ê 1 1ˆ
81. x ŒÁ , ˜ » (8,16)
Ë 16 8 ¯
222 Comprehensive Trigonometry with Challenging Problems & Solutions for Jee Main and Advanced

INTEGER TYPE QUESTIONS


fi x2 - x - 1 = x2 + x - 2
1. 3
2. x = 4
3. x = 9
fi (x 2
) (
- x - 1 = ± x2 + x - 2 )
4. 5 Taking +ve sign, we get,
5. 1
6. x = 3
fi (x 2
) (
- x - 1 = x2 + x - 2 )
7. 9 fi 2x = 1
8. 3 fi x =1/2
9. 7
Taking –ve sign, we get,
10. 4

HINTS AND SOLUTIONS


(x 2
) (
- x - 1 = - x2 + x - 2 )
fi 2 x2 = 3
LEVEL III
3
1. Given equation is fi x=±
2
( )
x + log10 1 + 2 x = x log10 5 + log10 6 Hence, the solutions are

Ê 1 + 2x ˆ ÏÔ 3 1 3 ¸Ô
x (1 - log10 5) + log10 Á Ì- , , ˝
fi ˜ =0 ÓÔ 2 2 2 Ô˛
Ë 6 ¯
Ê 1 + 2x ˆ 3. Given equation is
fi x (log10 2) + log10 Á ˜ =0 4 + log1/ 7 x = 2 + 2 + log1/ 7 x
Ë 6 ¯
Ê 1 + 2x ˆ fi 4 + log1/ 7 x = 2 + 2 + log1/ 7 x
fi log10 Á ˜ = - x (log10 2) 2 (2 + log1/ 7 x ) ≥ 0
Ë 6 ¯ fi
Ê 1 + 2x ˆ
log10 Á = log10 2- x fi (2 + log1/ 7 x ) ≥ 0
fi ˜
Ë 6 ¯ fi log1/ 7 x ≥ -2
Ê 1 + 2x ˆ -x 1 -2
fi Á 6 ˜ =2 = x Ê 1ˆ
Ë ¯ 2 fi x£Á ˜
Ë 7¯

fi (2 ) x 2
+ 2x - 6 = 0 fi x £ 49

fi a2 + a - 6 = 0 , a = 2x Hence, the value of x is (0, 49]

fi (a + 3) (a - 2) = 0 4. Given equation is
fi a = 2, - 3 Ê 4ˆ 2Ê 4 ˆ 2Ê 2 ˆ
log 2 Á1 + ˜¯ + log ÁË1 - ˜¯ = 2 log ÁË - 1˜
Ë x x+4 x -1 ¯
fi 2 x = 2, - 3
Ê x + 4ˆ Ê x ˆ Ê 2 ˆ
fi 2x = 2 fi log 2 Á ˜ + log 2 Á ˜ = 2 log 2 Á - 1˜
Ë x ¯ Ë x + 4¯ Ë x -1 ¯
fi x =1
Ê x + 4ˆ Ê x ˆ Ê 3 - xˆ
fi log 2 Á ˜ + log 2 Á ˜ = 2 log 2 Á
Hence, the solution is x = 1. Ë x ¯ Ë x + 4¯ Ë x - 1 ˜¯
2. Given equation is
Ê x ˆ Ê x + 4ˆ Ê 3 - xˆ
fi -2 log Á log Á = 2 log 2 Á
x2 - x - 1 Ë x + 4 ˜¯ Ë x ˜¯ Ë x - 1 ˜¯
log =0
x2 + x - 2
Ê x ˆ Ê x + 4ˆ Ê 3 - xˆ
fi - log Á log Á = log 2 Á
2
x - x -1 Ë x + 4 ˜¯ Ë x ˜¯ Ë x - 1 ˜¯
fi =1
x2 + x - 2 Ê x ˆ Ê 3 - xˆ
fi log 2 Á ˜ = log 2 Á
Ë x + 4¯ Ë x - 1 ˜¯
Logarithm 223

Ê x ˆ Ê 3 - xˆ Hence, the solutions set are


fi ÁË ˜ =Á ˜
x + 4¯ Ë x - 1¯ Ê1 ˆ
ÁË , 64˜¯ , (8, 2)
fi x 2 - x = 3x + 12 - x 2 - 4 x 4
fi 2 x 2 = 12 6. Given equation is
Ê 3x 2 + 12 x + 19 ˆ 1
fi x2 = 6
log Á
Ë 3x + 4 ˜
¯
+ log 25 4 = 1 + log 2
4
( 5
256 )
fi x=± 6
Ê 3x 2 + 12 x + 19 ˆ 2 1
{
Hence, the solution is - 6 , 6 } fi log Á
Ë 3x + 4 ˜ + = 1 + log 2
¯ 5 4
( 5
256 )
8 Ê 3x 2 + 12 x + 19 ˆ 3 1
5. We have log y x - log x y =
3 fi log Á
Ë 3x + 4
( )
˜¯ = 5 + 4 log 2 2
8/ 5

log x log y 8
fi - = Ê 3x 2 + 12 x + 19 ˆ 3 2
˜¯ = 5 + 5 log 2 (2)
log y log x 3 fi log Á
8 Ë 3x + 4
fi (log x )2 - (log y )2 = log x log y
3 Ê 3x 2 + 12 x + 19 ˆ
fi log Á ˜¯ = 1
3 (log x ) - -8 log x log y - 3 (log y ) = 0
2 2
fi Ë 3x + 4
fi 3a 2 - 8ab - 3b 2 = 0 Ê 3x 2 + 12 x + 19 ˆ
fi ÁË 3x + 4 ˜¯ = 10
where a = log x, b = log y
fi 3a 2 - 9ab + ab - 3b 2 = 0 fi 3x 2 + 12 x + 19 = 30 x + 40
fi 3a (a - 3b ) + b ( a - 3b ) = 0 fi 3x 2 - 18 x - 21 = 0
fi (a - 3b) (3a + b) = 0 fi ( x - 7) (3x + 3) = 0
fi (a - 3b) = 0 , (3a + b) = 0 fi x = -1, 7

fi a = 3b , b = -3a Hence, the solution set is {-1, 7}

fi log x = 3 log y , log y = -3 log x 7. Given equation is


Ê 1ˆ Ê 1ˆ
Ê xˆ log 2 ( 4 - x ) + log (4 - x ) .log Á x + ˜ - 2 log 2 Á x + ˜ = 0
fi Ëy ¯
( )
log Á 3 ˜ = 0 , log yx3 = 0 Ë 2¯ Ë 2¯
fi a 2 + ab - 2b 2 = 0 , where
Ê xˆ
fi ( )3
ÁË y 3 ˜¯ = 1, yx = 1 Ê 1ˆ
fi a = log ( 4 - x ) , b = log Á x + ˜
Ë 2¯
Now, x3 y = 1
fi (a - b) (a + 2b) = 0
fi x 2 ( xy ) = 1
fi a = b, - 2b
2 1 1
fi x = = when a = b
xy 16
Ê 1ˆ
1 fi log ( 4 - x ) = log Á x + ˜
fi x= Ë 2¯
4

when x =
1 1
, then y = 3 = 64 fi (4 - x ) = ÊÁË x + ˜
4 x 2¯
x 7
Also, =1 fi 2x =
y3 2
7
fi y 4 = xy = 16 fi x=
4
fi y=2
when a = -2b
when y = 2 , then x = 8
224 Comprehensive Trigonometry with Challenging Problems & Solutions for Jee Main and Advanced


Ê 1ˆ
log ( 4 - x ) = -2 log Á x + ˜
2
(
fi log 2 x + 7 = 4 )
Ë 2¯
-2
2
( 4
fi x + 7 = 2 = 16)
1
fi (4 - x ) = ÊÁË x + ˆ˜¯ fi x2 = 9
2
1 fi x = ±3
fi (4 - x ) = 2 Hence, the solutions are {-3, 3}
Ê 1ˆ
ÁË x + ˜¯
2 9. Given equation is

1
2
( )
log10 x 2 - x - 6 - x = log10 ( x + 2) - 4
fi (4 - x ) ÊÁË x + ˆ˜¯ = 1
2 Ê x2 - x - 6 ˆ
fi log10 Á = ( x - 4)
1ˆ Ë x + 2 ˜¯
fi (4 - x ) ÊÁË x 2 + x + ˜ =1

Ê ( x - 3) ( x + 2) ˆ
fi (
(4 - x ) 4 x 2 + 4 x + 1 = 4 ) fi log10 Á
Ë x+2 ˜¯ = ( x - 4)

fi 16 x 2 + 16 x + 4 - 4 x3 - 4 x 2 - x = 4 fi log10 (( x - 3)) = ( x - 4)
fi 12 x 2 + 15 x - 4 x3 = 0
fi ( x - 3) = 10( x - 4)
fi (
x 4 x 2 - 12 x - 15 = 0 ) Clearly x = 4 is the required solution by trial.
x = 0, 4 x 2 - 12 x - 15 = 0 10. Given equation is

1 1
fi x = 0, x =
12 ± 144 + 240 2
log 5 ( x + 5) + log 5 ( )
x-3 =
2
log 5 (2 x + 1)
8 1 1 1
fi log 5 ( x + 5) + log 5 ( x - 3) = log 5 (2 x + 1)
12 ± 384 2 2 2
fi x = 0, x =
8 fi log 5 ( x + 5) + log 5 ( x - 3) = log 5 ( 2 x + 1)
12 ± 4 24
fi x = 0, x = fi log 5 {( x + 5) ( x - 3)} = log 5 ( 2 x + 1)
8
3 ± 24 fi ( x + 5) ( x - 3) = (2 x + 1)
fi x = 0, x =
2 fi x 2 + 2 x - 15 = (2 x + 1)
3 + 24 fi x 2 = 16
fi x = 0, x =
2
fi x = ±4
Ï 7 3 + 24 ¸Ô
Hence, the solutions are ÔÌ0, , ˝ fi x = 4 is the required solution
ÓÔ 4 2 ˛Ô
11. Given equation is
( ) ( )
-1
2
8. log 3/ 4 log8 x + 7 + log1/ 2 log1/ 4 x + 7 2
= -2 3
log 4 ( x + 2) + 3 = log 4 ( 4 - x ) + log 4 (6 + x )
2 3 3

2
( ) ( )
2 2 -1
fi log 3/ 4 log 23 x + 7 + log 2-1 log 2-2 x + 7 = -2
fi 3 log 4 ( x + 2) + 3 = 3 log 4 (4 - x ) + 3 log 4 (6 + x )
Ê1 Ê1
( ( ˆ
)) ˆ
fi log 3/ 4 Á log 2 x 2 + 7 ˜ - log 2 Á log 2 x 2 + 7 ˜ = -2
Ë3 ¯ Ë2 ¯ ( ( )) fi log 4 ( x + 2) + 1 = log 4 ( 4 - x ) + log 4 (6 + x )

fi log 4 {4 ( x + 2)} = log 4 ( 4 - x ) (6 + x )


Ê1 ˆ Ê1 ˆ
3 2
2
fi log 3/ 4 ÁË y˜¯ - log 2 ÁË y˜¯ = -2 , y = log 2 x + 7 ( ( ))
fi 4 ( x + 2 ) = ( 4 - x ) (6 + x )
Ê yˆ Ê yˆ fi 4 x + 8 = 24 - 2 x - x 2
fi log 3/ 4 ÁË ˜¯ - log 2 ÁË ˜¯ = -2
3 2
fi x 2 + 6 x - 16 = 0
fi y = 4 , by trial.
Logarithm 225

fi ( x + 8) ( x - 2 ) = 0 Ï1 1 ¸
Hence, the solutions are Ì , ˝
fi x = 2, - 8 Ó2 4˛
14. Given equation is
Hence, the solution is x = 2. 2
4log10 x +1 - 6log10 x - 2.3log10 x +2
=0
12. The given equation is
2

1 + log 2 ( x - 4) fi 4.4log10 x - 6log10 x - 2.32.3log10 x = 0


=1
2 log 2 ( )
x+3- x-3 fi 4.4a - 6a - 18.32 a = 0 , a = log10 x
fi 4.4a - 6a - 18.32 a = 0
fi 1 + log ( x - 4) = 2 log ( x + 3 - x - 3 )
( ) ( )
2 2 2 2
fi 4 2a - 2a.3a - 18 3a =0
log {2 ( x - 4)} = log ( x + 3 - x - 3 )
2
fi 2 2
fi 4b 2 - bc - 18c 2 = 0, b = 2a , c = 3a ( )
2 ( x - 4) = ( x + 3 - x - 3 )
2
fi fi 4b 2 - bc - 18c 2 = 0

fi 2 ( x - 4) = x + 3 + x - 3 - 2 x 2 - 9 fi 4b 2 - 9bc + 8bc - 18c 2 = 0


fi b ( 4b - 9c ) + 2c ( 4b - 9c ) = 0
fi 2 x 2 - 9 = -8
fi x 2 - 9 = -4 fi (4b - 9c ) (b + 2c ) = 0
fi x 2 - 9 = 16 fi (4b - 9c ) = 0, (b + 2c ) = 0
fi 2
x = 25 fi 4b = 9c , b = -2c
fi x=±5 fi 4.2a = 9.3a , 2a = -2.3a
fi x=5 fi 4.2a = 9.3a
Hence, the solution is x = 5 fi 2a + 2 = 3a + 2
13. Given equation is a+2 0
Ê 2ˆ Ê 2ˆ
fi ÁË ˜¯ =1= Á ˜
Ê 1ˆ Ê x 3 + 27 ˆ 3 Ë 3¯
ÁË1 + ˜¯ log 3 = log Á
2x Ë 4 ˜¯ fi a+2=0
Ê 1ˆ
ÁË1+ ˜¯ Ê x 3 + 27 ˆ fi a = -2
2x
fi log 3 = log Á
Ë 4 ˜¯ fi log10 x = -2
Ê 1ˆ
Á1+ ˜¯ Ê x 3 + 27 ˆ 1
3Ë 2x
=Á fi x = 10-2 =

Ë 4 ˜¯ 100
Ê 1ˆ 1
Á1+ ˜¯
1
Hence, the solution is x = 10-2 =
fi 4. 3Ë 2x
= 3 x + 27 100
1 1 15. Given equation is
2x
fi 4. 3.3 = 3 x + 27
( x + x - 1 ) = log (4 3 - 3 + 4 x - 1 )
2
1 log 3 3
2x
fi 12. a = a 2 + 27 , a = 3
( x + x - 1 ) = (4 3 - 3 + 4 x - 1 )
2

fi 12.a = a 2 + 27

fi a 2 - 12a + 27 = 0 fi ( x + 2 x x - 1 + x - 2 x + 1)
(a - 3) (a - 9) = 0
= (4 3 - 3 + 4 x - 1 )

fi a = 3, 9
1
fi 2 x ( x - 1) + 2 x ( x - 1 ) = 4 ( x - 1)
fi 3 2 x = 3, 32
1 1 +4 x -1
fi x= ,
2 4
226 Comprehensive Trigonometry with Challenging Problems & Solutions for Jee Main and Advanced

fi 2 x ( )
x -1 + 2 (( x - 1) +
x -1 = 4 x -1) Hence, the solution is x = 4.
3. Given equation is
fi 2 x (( x - 1) + x - 1 ) = 4 (( x - 1) + x -1) 4log3 x = 32 - 4log3 x

fi (2 x - 4) (( x - 1) + x - 1 ) = 0 fi 2.4log3 x = 32
fi 4log3 x = 16 = 42
fi (2 x - 4) = 0 , (( x - 1) + x - 1 ) = 0 fi log 3 x = 2

fi x = 2 , (( x - 1) + x - 1 ) = 0 fi x = 32 = 9
Hence, the solution is x = 3
fi x = 2, ( )
x -1 < 0
4. Given equation is
fi x = 4, x < 1 3 log x 4 + 2 log 4 x 4 + 3 log16 x 4 = 0
fi x = 4,0 < x < 1
3 2 3
fi + + =0
fi x Œ(0,1) » {4} log 4 x log 4 (4 x ) log 4 (16 x )
Hence, the solution set is (0,1) » {4} fi
3
+
2
+
3
=0
log 4 x 1 + log 4 x 2 + log 4 x
INTEGER TYPE QUESTIONS 3 2 3
fi + + = 0 , a = log 4 x
n -1 a 1+ a 2 + a
Ê r + 2 ˆ 99
 log log r (r + 1)
Ë r + 1 ˜¯ ’
1. We have 2Á = 3 3 2
r =0 r =10 fi + =-
a 2+a 1+ a
Ê 2ˆ Ê 3ˆ Ê 4ˆ Ê n + 1ˆ
log 2 Á ˜ + log 2 Á ˜ + log 2 Á ˜ + ... + log 2 Á 6 + 3a + 3a 2
Ë 1¯ Ë 2¯ Ë 3¯ Ë n ˜¯ fi =-
a (2 + a ) 1+ a
= log10 (11) .log11 (12) log12 (13) ....log 99 (100)
6 (1 + a ) 2
=-
= log10 (100) fi a (2 + a ) 1+ a

6 (1 + a ) + 2a (a + 2) = 0
2
Thus, fi
Ê 2ˆ Ê 3ˆ Ê 4ˆ Ê n + 1ˆ fi 6a 2 + 12a + 6 + 2a 2 + 4a = 0
log 2 Á ˜ + log 2 Á ˜ + log 2 Á ˜ + ... + log 2 Á
Ë 1¯ Ë 2¯ Ë 3¯ Ë n ˜¯ fi 8a 2 + 16a + 6 = 0
= log10 (100) fi 4a 2 + 8a + 3 = 0
Ê2 3 4 n + 1ˆ fi 4a 2 + 6a + 2a + 3 = 0
fi log 2 Á . . .....
Ë1 2 3 ˜ = 2 log10 (10) = 2
n ¯ 2a ( 2a + 3) + 1(2a + 3) = 0

Ê n + 1ˆ
fi log 2 Á =2 fi (2a + 3) (2a + 1) = 0
Ë 1 ˜¯
fi n +1 = 4 fi 3 1
a = - ,-
2 2
fi n = 3. 3 1
fi log 4 x = - , -
2. Given equation is 7log2 x = 98 - x log2 7 2 2
fi x log2 7 = 98 - x log2 7 fi x = 4-3/ 2 , 4-1/ 2
fi 2 x log2 7 = 98 1 1
fi a= ,b=
8 2
fi x log2 7 = 49
1Ê 1 1ˆ
fi 7log2 x = 7 2 Hence, the value of +
2 ÁË a b ˜¯
fi log 2 x = 2
1
fi x = 22 = 4
= (8 + 2) = 5
2
Logarithm 227

5. Given equation is fi a2 - a - 2 = 0
( )
x + log10 2 x + 1 = log10 6 + x log10 5 fi (a - 2) (a + 1) = 0
fi (
x + log10 2 x + 1 = log10 6.5 x ) ( ) fi a = 2, - 1

Ê 6.5 x ˆ fi log 2 x = 2, - 1
fi log10 Á x ˜ = x
Ë 2 + 1¯ 1
fi x = 4,
Ê 6.5 x ˆ 2
Hence, the value of (a + 2b + 3) is 8.
x
fi ÁË 2 x + 1˜¯ = 10
8. Given equation is
fi (
10 x 2 x + 1 = 6.5 x ) 6 (log x 2 - log 4 x ) + 7 = 0
fi ( )
2x 2x + 1 = 6
Ê 1 ˆ
fi 6 Á log x 2 - +7=0
fi (2 ) + (2 ) - 6 = 0
x 2 x
Ë log x 4 ˜¯

fi a + a - 6 = 0 , a = (2 )
2 x Ê 1 ˆ
fi 6 Á log x 2 - +7=0
Ë 2 log x 2 ˜¯
fi (a + 3) (a - 2) = 0
a = 2, - 3 Ê 1ˆ
fi fi 6 Á a - ˜ + 7 = 0 , a = log x 2
Ë 2a ¯
fi 2 x = 2, - 3

fi ( )
6 2a 2 - 1 + 14a = 0
2x = 2
fi x=1 fi 3 ( 2a 2
- 1) + 7 a = 0

Hence, the solution is x = 1 fi 6a 2 + 7 a - 3 = 0


6. We have fi 6a 2 + 9a - 2a - 3 = 0

(
log ( x +1) x 2 + x - 6 = 4 )
2
fi 3a (2a + 3) - ( 2a + 3) = 0
fi (3a - 1) (2a + 3) = 0
(x )
2
+ x - 6 = ( x + 1)
2 4

1 3
fi a = ,-
x 4 + 2 x3 - 11x 2 - 12 x + 36 3 2
= x 4 + 4 x3 + 6 x 2 + 4 x + 1 1 3
fi log x 2 = , -
2 x3 + 17 x 2 + 16 x - 35 = 0 3 2
1 1 3
2 x3 - 2 x 2 + 19 x 2 - 19 x + 35 x - 35 = 0 fi = ,-
log 2 x 3 2
( x - 1) (2 x 2
+ 19 x + 35 = 0 ) 2
( x - 1) = 0 fi log 2 x = 3 , -
3
3 -2 / 3
x=1 fi x = 2 ,2
7. Given equation is Thus, the integral solution of x is 8.
( ) 3 Therefore, a = 8
= ( x - 2)
log 2 x - 3 log x 4 3
x-2
Ê 2a - 1ˆ
fi ( )
log 2 x3 - 3 log x 4 = 3
Hence, the value of Á
Ë 5 ˜¯
3 Ê 16 - 1ˆ
fi 3 log 2 x - =3 =Á =3.
log 4 x Ë 5 ˜¯
2
fi log 2 x - =1
log 2 x 9. We have N = 6 log10 2 + log10 31
2 = log10 26 + log10 31
fi a - = 1 , a = log 2 x
a
228 Comprehensive Trigonometry with Challenging Problems & Solutions for Jee Main and Advanced

= log10 (64 ¥ 31) log 2 + 2 log 3 log 2 + 3 log 3


-
=
= log10 (1984) log (3) + 2 log 2 log 3 + 3 log 2

< log10 (1000) = 3 1 + 2 log 2 3 1 + 3 log 2 3


= -
log 2 (3) + 2 log 2 3 + 3
Also, N = log10 (1984) > log10 (10000) = 4 .
1 + 2 x 1 + 3x
Thus, the sum of successive integers = - , x = log 2 3
x+2 x+3
= 3 + 4 = 7.
1 1 1 1/ 3 1 =
(1 - x )2

10. Let S = + 2 + 3 + ... = =


3 3 3 1 - (1 / 3) 2 ( 2 + x ) (3 + x )
We have Hence, the value of 5 ( a - b ) + ab

(0.16)
Ê 1ˆ
log 2.5 Á ˜
Ë 2¯
=
(
5 1 - x2 ) +
(1 + 5x + 6 x )
2

Ê 1ˆ
log 5 Á ˜
( 2 + x ) ( 3 + x ) ( 2 + x ) (3 + x )
Ê 4ˆ Ë 2¯

( ) ( )
2
=Á ˜ 5 1 - x2 + 1 + 5x + 6 x2
Ë 25 ¯
=
Ê 1ˆ
2 log 5 Á ˜
( 2 + x ) (3 + x )
Ê 2ˆ Ë 2¯
2
=Á ˜ x2 + 5x + 6
Ë 5¯ =
Ê 1ˆ
(2 + x ) (3 + x )
-2 log 2 Á ˜
Ê 2ˆ Ë 2¯
5
( x + 2) ( x + 3)
=Á ˜
Ë 5¯ =
( 2 + x ) (3 + x )
-2
Ê 1ˆ
log 2 Á ˜ -2 = 1.
Ê 2ˆ Ë 2¯ Ê 1ˆ
= ÁË ˜¯
5
= ÁË ˜¯ =4
5 2 QUESTIONS ASKED IN PAST IIT-JEE EXAMS
11. We have ab
1. The given equation is
= log12 18 ¥ log 24 54
2 log x a + log a x a + 3 log a2 x a = 0
log 18 log 54
= ¥ 2 log a log a 3 log a
log 12 log 24 + + =0

( ) ¥ log (2.3 )
log 2.32 3
log x log ( ax ) log a 2 x ( )
=
log (3.2 ) log (3.2 )
2 3

2
+
1
+
3
=0
log x log a + log x 2 log a + log x
log 2 + 2 log 3 log 2 + 3 log 3
= ¥ 2 1 3
log (3) + 2 log 2 log 3 + 3 log 2 fi + + = 0,
y b + y 2b + y
1 + 2 log 2 3 1 + 3 log 2 3
= ¥ where log a = b, log x = y
log 2 (3) + 2 log 2 3 + 3
1 + 2 x 1 + 3x fi 6 y 2 + 11by + 4b 2 = 0
= ¥ , x = log 2 3
x+2 x+3 fi 6 y 2 + 3by + 8by + 4b 2 = 0
2
1 + 5x + 6 x
= fi (2 y + b) (3 y + 4b) = 0
x2 + 5x + 6
b 4b
Also, ( a - b ) fi y = - ,-
2 3
= log12 18 - log 24 54
b log a

=
( ) - log (2.3 )
log 2.32 3 when y = -
2
, then log x = -
2
log (3.2 ) log (3.2 )
2 3
Ê 1ˆ
fi log x 2 = - log a = log Á ˜
Ë a¯
Logarithm 229

1 Thus, x Œ( 2, • ) .
fi x2 =
a 6 The given equation is

( ( x + 5 + x )) = 0
-1/ 2
fi x=a
log 7 log 5
4b 4 log a
when y = -
3
, then log x = -
3 fi (log ( x + 5 + x )) = 7
5
0
=1


1
Ê ˆ
3 log x = 4 log Á ˜
Ë a¯
fi ( )
x + 5 + x = 51 = 5

4 fi x+5 =5- x
3 Ê 1ˆ
fi log x = log Á ˜ fi x + 5 = 25 - 10 x + x
Ë a¯
4 fi 10 x = 20
3 Ê 1ˆ
fi x =Á ˜
Ë a¯ fi x=2
fi x=a -4 / 3 . fi x = 4.
Hence, the solution is x = 4
2. We have 2 log10 x - log x (0.01) , x > 1
7. The given equation is
= 2 log10 x - log x (10)
-2

( 2
) 2
(
fi log (2 x + 3) 6 x + 23x + 21 = 4 - log (3 x + 7) 4 x + 12 x + 9 )
= 2 (log10 x + log x (10))
fi log (2 x + 3) ( 2 x + 3) (3x + 7 ) = 4 - log (3 x + 7) (2 x + 3)
2

≥ 2.2 = 4
Thus, the least value is 4. fi 1 + log (2 x + 3) (3x + 7 ) = 4 - 2 log (3 x + 7) ( 2 x + 3)
3. The curve y = 10 x y = log10 x 2
fi log (2 x + 3) (3x + 7 ) = 3 - log
with respect to the line y = x. (2 x + 3) (3 x + 7 )
4. We have log x a + log a x 2
fi y = 3- , where y = log (2 x + 3) (3x + 7 )
Ê y
1 ˆ
= Á log x a + ≥2
Ë log x a ˜¯ fi y2 - 3y + 2 = 0
Thus, the minimum value of the given expression fi ( y - 1) ( y - 2) = 0
fi y = 1, 2
is 2.
5. We have log 0.3 ( x - 1) < log 0.09 ( x - 1)
when y = 1, then log (2 x + 3) (3x + 7 ) = 1
fi log (0.3) ( x - 1) < log (0.3)2 ( x - 1)
fi (3x + 7) = (2 x + 3)
1
fi log (0.3) ( x - 1) < log (0.3) ( x - 1) fi x = –4
2
when y = 2, then log (2 x + 3) (3x + 7 ) = 2
fi 2 log (0.3) ( x - 1) < log (0.3) ( x - 1)
fi (3x + 7) = (2 x + 3)2
log (0.3) ( x - 1) < log (0.3) ( x - 1)
2

fi (3x + 7) = 4 x 2 + 12 x + 9
fi ( x - 1)2 > ( x - 1) fi 4 x2 + 9 x + 2 = 0
fi ( x - 1) - ( x - 1) > 0
2
fi 4 x2 + 8x + x + 2 = 0
fi ( x - 1) ( x - 1 - 1) > 0 fi 4 x ( x + 2 ) + 1( x + 2 ) = 0
fi ( x - 1) ( x - 2) > 0 fi ( x + 2) (4 x + 1) = 0
fi x < 1, x > 2 1
fi x = -2, -
fi x > 2 [x < 1 does not satisfy the given 4
in-equation]
230 Comprehensive Trigonometry with Challenging Problems & Solutions for Jee Main and Advanced

3 1 fix = 3, 2
As x > - , so x = -
2 4 fix = 3, since x = 2 does not satisfy the logarithmic
1 expression.
Hence, the solution is x = -
4 Hence, the solution is x = 3
8. We have 10. Ans. (c)
5
(3 / 4)(log 2 x )2 + log 2 x - p
x 4
= 2 Let log 2 7 = , where
5ˆ q
Ê
fi ÁË (3 / 4) (log 2 x ) + log 2 x - ˜¯ log x = log
2

4
( 2) p, q ΠN and H .C.F ( p, q ) = 1
Ê 5ˆ 1 p
fi ÁË (3 / 4) b + b - ˜¯ b = , b = log 2 x
2

4 2 fi 2q = 7
fi 3b3 + 4b 2 - 5b - 2 = 0 fi 2 p = 7q
fi 3b3 - 3b 2 + 7b 2 - 7b + 2b - 2 = 0 This is not possible for any p, q ΠN and 7 and 2 are
prime.
fi 3b (b - 1) + 7b (b - 1) + 2 (b - 1) = 0
2

Thus, log 2 7 is an irrational number.


(
fi (b - 1) 3b + 7b + 2 = 0
2
) 11. Ans. (b)
fi (b - 1) (3b )
2
+ 6b + b + 2 = 0 The given equation is log 4 ( x - 1) = log 2 ( x - 3)
fi (b - 1) (3b (b + 2) + 1(b + 2)) = 0 1
fi log 2 ( x - 1) = log 2 ( x - 3)
fi (b - 1) (b + 2) (3b + 1) = 0 2
2
1 fi log 2 ( x - 1) = log 2 ( x - 3)
fi b = 1, - 2, -
3 2
fi ( x - 3) = ( x - 1)
1
fi log 2 x = 1, - 2, -
3 fi x2 - 6 x + 9 = x - 1
1
-2
- fi x 2 - 7 x + 10 = 0
fi x = 2, 2 , 2
3

fi ( x - 2 ) ( x - 5) = 0
Thus, the equation has exactly three real solutions of
which exactly one is irrational. fix = 2, 5

7ˆ fix = 5, since x = 2 does not satisfy the equation.


Ê
Ë
(

)
9. Given log 3 2, log 3 2 x - 5 , log 3 Á 2 x - ˜ Œ A.P . Thus, the number of solution is one.
12. Ans. (c)
Ê x 7ˆ
x
( )
fi 2 log 3 2 - 5 = log 3 2 + log 3 ÁË 2 - ˜¯
2
The given equations are

Ê x 7ˆ
(2 x )ln 2 = (3 y )ln 3 , 3ln x = 2ln y
( )
x 2
fi log 3 2 - 5 = log 3 2. ÁË 2 - ˜¯ Now, 3ln x = 2ln y
2
x
(
2 Ê x 7ˆ
fi 2 - 5 = 2. ÁË 2 - ˜¯ ) fi log 3 ( )
ln x
= log 2ln y ( )
2
fi log x log (3) = log y log ( 2)
( )
x 2
fi 2 - 10.2 x + 25 = 2.2 x - 7
log x log ( y )
= = l ( say )
fi (2 ) fi log 2 log (3)
x 2 x
- 12.2 + 32 = 0
ln 2 ln 3
fi ( a ) - 12.a + 32 = 0 Also, ( 2 x ) = (3 y )
2

fi ( a - 8) ( a - 4 ) = 0 (
fi log ( 2 x )
ln 2
) = log ((3 y) )
ln 3

fi a = 8, 4
fi log 2 log (2 x ) = log 3 log (3 y )
fi 2 x = 8, 4 = 23 , 22 fi log 2 (log 2 + log x ) = log 3 (log 3 + log y )
Logarithm 231

fi log 2 (log 2 + l log 2) = log 3 (log 3 + l log 3) 2


fi 3 2 x + 9 x - 8 x - 12 2 = 0

( ) ( 2 x + 3) = 0
2 2
fi (log 2) (1 + l ) = (log 3) (1 + l ) fi3x 2x + 3 - 4 2

( 2
fi (1 + l ) (log 2) - (log 3)
2
)=0 fi ( )(
2x + 3 3 x - 4 2) = 0

(
fi (1 + l ) = 0, ∵ (log 2) - (log 3) π 0
2 2
) fix=-
3 4 2
2 3
,
fi l = -1
4 2
Thus, log x = - log 2, log y = - log 3 fix=
3
Ê 1ˆ Ê 1ˆ
fi log x = log ÁË ˜¯ , log y = log ÁË ˜¯ Thus, 6 + log3/2
2 3
Ê ˆ
1 1 1 1 1 1
fix= , y= Á 4- 4- 4- .......to • ˜
2 3 ÁË 3 2 3 2 3 2 3 2 ˜¯
1
Hence, x0 =
2 Ê 1 4 2ˆ
= 6 + log3/ 2 Á ¥
13 Let Ë3 2 3 ˜¯

1 1 1 Ê 4ˆ
x = 4- 4- 4- .......to • = 6 + log3/ 2 Á ˜
3 2 3 2 3 2 Ë 9¯
2
1 Ê 2ˆ
fix = 4- x = 6 + logÊ 2 ˆ -1 Á ˜
3 2 Á ˜
Ë 3¯
Ë 3¯

2 Ê 1 ˆ Ê 2ˆ
fi x = Á4 - x˜ . = 6 - 2 logÊ 2 ˆ Á ˜
Ë 3 2 ¯ Á ˜ Ë 3¯
Ë 3¯
2
fi 3 2 x + x - 12 2 = 0 = 6-2 = 4
CHAPTER 6
Inverse Trigonometric Function

INVERSE FUNCTION Y

y = f(x)
6.1 INTRODUCTION TO INVERSE
FUNCTION y=
x
y = f -1(x)

X¢ X
A f B
O

1 6
2 7
3 8
4 9

5 g 10
(vi) A function f (x) is said to be involution if for all
Let f : X Æ Y be a bijective function. x for which f (x) and f (f (x
f ( f ( x)) = x .
If we can make another function g from Y to X, then we
shall say that g is the inverse of f. (vii) If f is an invertible function, then f ( )
= f. -1 -1

-1
-1 1 (viii) If f : A Æ B be one one function, then f of = I A
i.e. g = f π
f and f o f -1 = I B , where I A and I B are the iden-
Thus, f -1 ( f ( x )) = x . tity functions of the sets A and B, respectively.
(ix) Let f : A Æ B , g : B Æ C be two invertible
Note:
(i) The inverse of a function exists only when the functions, then ( g o f )
-1
( )
= f -1 o g -1 is also
invertible with f ( x ) .
-1
function f is bijective.
(ii) If the inverse of a function exists, then it is called
an invertible function. RULE TO FIND OUT THE INVERSE
(iii) The inverse of a bijective function is unique. OF A FUNCTION
(iv) Geometrically, f ( x ) is the image of f (x) with
-1 (i) First, we check the given function is bijective or not.
respect to the line y = x. (ii) If the function is bijective, then inverse exists, other-
wise not.
(v) Another way also we can say that f ( x ) is
-1
(iii) Find x in terms of y.
symmetrical with respect to the line y = x.
(iv) And then replace y by x, then we get inverse of f.
i.e. f : R Æ R .
Inverse Trigonometric Function 233

6.2 SOME SOLVED EXAMPLES fi f ( x) = 1 -


1
Ex-1. A function f : R Æ R x +12

2x
f (x) = 3x + 5. Find f ( x ) . f ' ( x) =
-1
fi > 0, " x ΠR +
( x + 1)
2 2
Soln. Given f (x) = 3x + 5
fi f ' ( x) = 3 > 0 fi f is strictly increasing function.
fi f is strictly increasing function. fi f is one one function.
fi f is one one function
x2
Also, R f = R = co-domain Also, let y = 2
fi f is onto function. x +1
Thus, f is a bijective function. fi y.x 2 + y = x 2
Hence, f -1 is exists.
fi x 2 ( y - 1) = - y
Let y = 3 x + 5
y-5 y y
fi x= fi x2 = - =
3 ( y - 1) (1 - y )
x-5
Thus, f -1 ( x ) = . y
3 fi x=
(1 - y )
Ex-2. A function f : (0, • ) Æ ( 2, • )
fi R f = (0, 1) = co-domain
as f ( x ) = x + 2 f -1 ( x ) .
2

fi f is onto function.
Soln. Given f ( x ) = x + 2
2
Thus, f is a bijective function.
fi f ' ( x ) = 2 x > 0 for every x > 0 fi f -1 ( x ) exists.
fi f is strictly increasing function.
x
Hence, f ( x ) =
-1
fi f is one one function. .
1- x
Also, R f = ( 2, • ) = co-domain
fi f is ont function. Ex-4. A function f : [1, • ) Æ [1, • )
Thus, f is a bijective function. f ( x ) = 2 x( x -1) . Find f -1 ( x ) .
Therefore, the inverse of the given function exists. x ( x -1)
Let y = x 2 + 2 Soln. Given f ( x ) = 2 .
Y
fi x2 = y - 2
fi x= y-2

Hence, f ( x ) = x - 2 .
-1 X¢ X

Ex-3. A function f : R Æ [0, 1)


+

x2
f ( x) = f -1 ( x ) . Y¢
x2 + 1
fi f ' ( x ) = 2 x( x -1) ¥ ( 2 x - 1) ¥ log e 2 > 0
x2
Soln. Given f ( x ) = for all x in [1, • )
x2 + 1
fi f is strictly increasing function.
Y
fi f is one one function.
Also, R f = [1, • )
fi R f = [1, • ) = co-domain
X¢ X fi f is onto function.
Thus, f is a bijective function.
So its inverse exists.
Y¢ Let y = 2 x ( x -1)
234 Comprehensive Trigonometry with Challenging Problems & Solutions for Jee Main and Advanced

Y
y = 2x -x
2

fi x 2 - x = log 2 ( y )
fi x 2 - x - log 2 ( y ) = 0 X¢ X

1 ± 1 + 4 log 2 ( y )
fi x=
2
1 + 1 + 4 log 2 ( y ) Y¢
fi x=
2 fi f ' ( x ) = 2 x - 4 ≥ 0 for all x in Df
1 + 1 + 4 log 2 ( x ) fi f is strictly increasing function.
Thus, f -1 ( x ) = . fi f is one one function.
2
Also, R f = [5, • ) = co-domain
Ex-5. If a function f is bijective such that
fi f is onto function.
10 x - 10- x Therefore, f is a bijective function.
f ( x) = f ( x) . -1
10 x + 10- x Hence, its inverse exists.
Soln. Since f is a bijective function, so its inverse exists. Let y = x 2 - 4 x + 5
10 x - 10- x 102 x - 1 fi x 2 - 4 x + (5 - y ) = 0
Let y = = .
10 x + 10- x 102 x + 1
4 ± 16 - 4 (5 - y )
fi y.102 x + y = 102 x - 1 fi x=
2
fi 10 2x
( y - 1) = - y - 1 4 ± 4 y + 16 - 20
fi x=
y +1 y +1 2
fi 10 2x = - =
y -1 1- y 4 ± 4 ( y - 1)
Ê y + 1ˆ fi x= = 2± ( y - 1)
fi 2 x = log10 Á 2
Ë 1 - y ˜¯
fi x = 2+ ( y - 1) , since x≥2
1 Ê1+ yˆ
fi x = log10 Á fi f -1 ( x ) = 2 + ( x - 1)
2 Ë 1 - y ˜¯
Ex-8. Find all the real solutions to the equation
1 Ê1+ xˆ
Thus, f ( x ) =
-1
Ë 1 - x ˜¯ .
log10 Á 1 1
2 x2 - = x +
4 4
Ex-6. A function f : R Æ R Soln. Consider the function
f ( x ) = x + sin x . Find f -1 ( x ) . È 1 ˆ
f : [0, • ) Æ Í- , •˜ , where
Soln. Given f ( x ) = x + sin x Î 4 ¯
1
fi f ' ( x ) = 1 + cos x ≥ 0 for all x in R. f ( x) = x2 -
4
fi f is strictly increasing function Clearly, f is one one and onto function.
fi f is one one function. So its inverse exists.
Also, the range of a function is R
-1 È 1 ˆ
fi f is a onto function Let its inverse is f : Í- , •˜¯ Æ [0, • ) .
Thus, f is a bijective function. Î 4
Hence, f -1 exists. 1
fi f -1 ( x ) = x +
Therefore, f -1 ( x ) = x - sin x . 4
Consequently, we can say that, the two sides of the
Ex-7. A function f : [ 2, • ) Æ [5, • )
given equation are inverse to each other.
as f ( x ) = x 2 - 4 x + 9 . Find its inverse. Thus, the intersection point is the solution of the
given equation.f (x) = x
Soln. Given f ( x ) = x - 4 x + 9
2
1
fi x2 - =x
4
Inverse Trigonometric Function 235

1 From the graph, we can say that, it will be one one and onto
fi x2 - x = only when we considered it in some particlar intervals like
4
2 Ê p p ˆ Ê p 3p ˆ Ê 3p 5p ˆ
Ê 1ˆ 1 ÁË - , ˜¯ , ÁË , ˜¯ , ÁË , ˜¯ and so on
fi ÁË x - ˜¯ = 2 2 2 2 2 2
2 2
If we consider the whole function, then it is not one one
Ê 1ˆ 1 as well as onto.
fi ÁË x - ˜¯ = ±
2 2 Also, when we think the inverse function, then domain
1 1 and range are interchanged.
fi x= ± So the graph of this function is as follows:
2 2
Ï1 1 1 1 ¸ Y
Hence, the solutions are Ì + , - ˝.
Ó2 2 2 2˛ 3π

2
EXERCISE 1 2π

f ( x ) = 3x + 5
2
1. A function f π

where f : R Æ R f -1 ( x ) . π
2
x X¢ X
2. A function f f ( x) = O
π
x -1 -
2
where f : R - {1} Æ R - {1} f -1 ( x ) . -π
1 3π
3. A function f f ( x) = -
2
x2 + 1 - 2π

where f : R » {0} Æ (0 ,1] f -1 ( x ) .


+ 5π
-
2
4. A function f is bijective such that - 3π
2 x - 2- x
f ( x) = x f -1 ( x ) . x = -1 Y¢ x=1
2 + 2- x
È 1 ˆ Ê 1˘ As a whole, inverse of this function does not exist. Its inverse
5. A function f : [ -1,1] Æ Í- , 0˜ » Á 0, ˙ is exists only when, we restrict its range
Î 2 ¯ Ë 2˚
x Ê 3p p ˆ Ê p p ˆ Ê p 3p ˆ
f ( x) = 2 f -1 ( x ) . So in thertvals Á - , - ˜ , Á - , ˜ , Á , ˜ ,
x +1 Ë 2 2¯ Ë 2 2¯ Ë 2 2 ¯
Ê 3p 5p ˆ
ÁË , ˜¯ and so on.
6.3 INVERSE TRIGONOMETRIC 2 2

FUNCTIONS In the conventional of mathematics, we consider it


Ê p pˆ
in Á - , ˜ .
number and the range of sine function is [–1, 1]. Thus, the Ë 2 2¯
graph of f ( x ) = sin ( x ) is as follows:
Graph of f ( x ) = sin x :
È p p˘
Y sin -1 : [ -1,1] Æ Í- , ˙
Î 2 2˚
y=1 È p p˘
Therefore, a function f : [ -1,1] Æ Í- , ˙ .

-2π -π O π 2π
X Î 2 2˚
y=1
f ( x ) = sin -1 x .

So the graph of f ( x ) = sin x is


Y¢ -1
236 Comprehensive Trigonometry with Challenging Problems & Solutions for Jee Main and Advanced

Y Y

y = p/2 y=p

X¢ X y = p/2
O O
y = -p/2 X¢ X

x = -2 Y¢ x=1 Y¢

È p p˘
Thus, D f = [-1,1] and R f = Í- , ˙ CHARACTERISTICS OF ARC
Î 2 2˚
Now, we shall discuss the graphs of other inverse COSINE FUNCTION:
trigonometric functions and their characteristics. 1. D f = [-1, 1]
2. [0, p ]
6.4 GRAPHS OF INVERSE 3. It is not a periodic function.
TRIGONOMETRIC FUNCTIONS 4. It is neither even nor odd function
since, cos -1 ( - x ) = p - cos -1 ( x ) .
(i) sin -1 x :
5. It is strictly decreasing function.
È p p˘
A function f : [ -1, 1] Æ Í- , ˙ 6. It is one one function.
Î 2 2˚ p
7. For 0 < x < ,
f ( x ) = sin -1 x = arc sin x Graph of f ( x ) = sin -1 x. 2
Y cos -1 x < x < cos x
(iii) tan -1 x :
y = p/2
Ê p pˆ
A function f : R Æ ÁË - , ˜¯ is
X¢ X 2 2
O
f ( x ) = tan -1 x .
y = -p/2
Graph of f ( x ) = tan -1 x :
Y
x = -1 Y¢ x=1

y = p/2
CHARACTERISTICS OF ARC SINE FUNCTION
1. D f = [-1, 1]
X¢ X
O

È p p˘ y = -p/2
2. R f = Í- , ˙
Î 2 2˚
3. It is not a periodic function. Y¢
4. It is an odd function.
since, sin -1 ( - x ) = - sin -1 x . CHARACTERISTICS OF ARC
5. It is strictly increasing function TANGENT FUNCTION
6. It is one one function. 1. D f = R
p
7. For 0 < x < , sin x < x < sin -1 x . Ê p pˆ
2 2. R f = Á - , ˜
Ë 2 2¯
(ii) cos -1 x : 3. It is not a periodic function.
A function f : [ -1,1] Æ [0, p ] 4. It is an odd function.
f ( x ) = cos -1 x = arc cos x since, tan -1 ( - x ) = - tan -1 x .
5. It is strictly increasing function.
Graph of f ( x ) = cos x :
-1
6. It is one one function.
Inverse Trigonometric Function 237

7. For 0 < x < p , tan -1 x < x < tan x . È p p˘


2. R f = Í- , ˙ - {0} .
2 Î 2 2˚
(iv) cot -1 x : 3. It is an odd function, since
A function f : R Æ (0, p ) is cosec -1 ( - x ) = -cosec -1 ( x ) .
f ( x ) = cot -1 x . 4. It is non periodic function.
5. It is one one function.
Graph of f ( x ) = cot x
-1
6. It is strictly decreasing function with respect to its
Y domain.
p
7. For 0 < x < , cosec -1 x < x < cosec x .
y=p 2
(v) sec-1 x : A function
y = -p/2
O Ïp ¸
f : ( -•, - 1] » [1, • ) Æ [0, p ] - Ì ˝
X¢ X Ó2˛
f ( x ) = sec-1 x .

Graph of f ( x ) = sec x .
-1

CHARACTERISTICS OF ARC Y
CO-TANGENT FUNCTION:
1. D f = R y=p

2. R f = (0, p )
X¢ y = p/2
3. It is not a periodic function.
4. It is neither even nor odd function X
O
since, cot -1 ( - x ) = p - cot -1 x .
5. It is strictly decreasing function. Y¢
6. It is one one function.
p
7. For 0 < x < , CHARACTERISTICS OF
2 ARC SECANT FUNCTION:
cot x < x < cot -1 x 1. D f = ( -•, - 1] » [1, • ) .
(v) cosec-1 x : Ïp ¸
A function 2. R f = [0,p ] - Ì ˝ .
Ó2˛
È p p˘
f : ( -•, - 1] » [1, • ) Æ Í- , ˙ - {0} 3. It is neither an even function nor an odd odd function,
Î 2 2˚ since sec-1 ( - x ) = p - sec -1 ( x ) .
f ( x ) = cosec-1 x . 4. It is non periodic function.
Graph of f ( x ) = cosec-1 x . 5. It is one one function.
6. It is strictly decreasing function with respect to its domain.
Y
p -1
7. For 0 < x < , sec x < x < sec x .
2
y = p/2
6.4.1 Some Solved Examples
X¢ X
O Ex-1. Find the domain of f ( x ) = sin -1 (3x + 5) .
y = -p/2 Soln. We have -1 £ 3x + 5 £ 1
fi - 6 £ 3 x £ -4
x = -1 Y¢ x=1
4
fi -2£ x £-
CHARACTERISTICS OF ARC 3
CO-SECANT FUNCTION: È 4˘
fi D f = x Œ Í-2, - ˙
1. D f = ( -•, - 1] » [1, • ) Î 3˚
238 Comprehensive Trigonometry with Challenging Problems & Solutions for Jee Main and Advanced

-1 Ê x ˆ -1 Ê x - 1ˆ
Ex-2. Find the domain of f ( x ) = sin Á Ex-4. Find the domain of f ( x ) = sin Á
Ë x + 1˜¯ Ë 2 ˜¯
x
Soln. We have, -1 £ £1 x -1
x +1 Soln. We have, -1 £ £1
2
x
Case I: when £1 fi - 2 £ x -1 £ 2
x +1
x fi -1£ x £ 3
fi -1£ 0
x +1 fi x £ 3 (∵ x ≥ - 1 is rejected )
-1 fi -3 £ x £ 3
fi £0
x +1 Hence, D f = [-3, 3] .
1
fi ≥0
Ex-5. Find the domain of f ( x ) = sin (log 2 x )
-1
x +1
fi x > -1 Soln. We have, -1 £ (log 2 x ) £1
x
Case II: When ≥ -1 fi 2-1 £ x £ 21
x +1
1
x fi £ x £2
fi +1 ≥ 0 2
x +1
2x + 1 È1 ˘
fi ≥0 Hence, D f = Í , 2˙ .
x +1 Î2 ˚
È 1 ˆ
fi x Œ( -•, - 1) » Í- , •˜
-1
(
Ex-6. Find the domain of f ( x ) = sin log 4 x
2
)
Î 2 ¯ Soln. We have, -1 £ log 4 x £ 1 2

È 1 ˆ
Hence, D f = Í- , •˜ . fi 4-1 £ x 2 £ 41
Î 2 ¯
1
fi £ x2 £ 4
-1 Ê x + 1ˆ
2
Ex-3. Find the domain of f ( x ) = sin Á ˜
4
Ë 2x ¯ 1
fi £ x £2
Soln. We have, -1 £ x + 1 £ 1
2 2
2x 1
fi x £2 & x ≥
x +1
2 2
fi £1 fi - 2 £ x £ 2 and
2x
1 1
x2 + 1 x ≥ & x£-
fi £1 2 2
2x È 1 ˘ È1 ˘
fi x Œ Í-2, - ˙ » Í , 2˙
x2 + 1 Î 2˚ Î2 ˚
fi £1 È 1 ˘ È1 ˘
2 x Hence, D f = Í-2, - ˙ » Í , 2˙ .
Î 2˚ Î2 ˚
fi x2 + 1 £ 2 x
Ex-7. Solve for x and y: sin -1 x + sin -1 y = p
fi x - 2 x +1 £ 0
2
Soln. Given sin -1 x + sin -1 y = p
fi ( x - 1) £ 0
2
It is possible only when each term of the given
equation provides the maximum value.
fi ( x - 1)2 = 0 p p
Thus, sin -1 x = & sin -1 y =
fi ( x - 1) = 0 2 2
fi x =1 Êpˆ Êpˆ
fi x = sin Á ˜ = 1 & y = sin Á ˜ = 1
Ë 2¯ Ë 2¯
fi x =±1
Hence, the solutions are x = 1 and y = 1
Hence, D f = {-1, 1}
Inverse Trigonometric Function 239

3p È 5 3˘
Ex-8. If sin -1 x + sin -1 y + sin -1 z = Hence, D f = Í- , - ˙
2 Î 2 2˚
value of p
9 Ex-12. Find the range of f ( x ) = 2 cos -1 (3x + 5) +
x 2013
+y 2013
+z 2013
- 4
x 2014 + y 2014 + z 2014
Soln. We have, 0 £ cos -1 (3x + 5) £ p
3p
Soln. Given sin -1 x + sin -1 y + sin -1 z = fi 0 £ 2 cos -1 (3x + 5) £ 2 p
2
It is possible only when each term will provide us p p p
the maximum value. fi £ 2 cos -1 (3x + 5) + £ 2p +
4 4 4
p p p p
Thus, sin -1 x = , sin -1 y = È
Hence, R f = Í ,
9 ˘
2 2 ˙
Î4 4 ˚
p
& sin -1 z = p
2 ( )
Ex-13. Find the range of f ( x ) = 3 cos -1 - x 2 -
2
.
fi x = 1, y = 1 & z = 1 .
p
Hence, the value of Soln. We have,
2
( )
£ cos -1 - x 2 £ p
9
x 2013 + y 2013 + z 2013 - 2014 3p
x +y 2014
+ z 2014 fi £ 3 cos ( - x ) £ 3 p
-1 2
2
9
=1+1+1– 3p p p p
1+1+1 fi - £ 3 cos ( - x ) - £ 3 p -
-1 2
=3–3 2 2 2 2
= 0. 5p
fi p £ f ( x) £
p
Ex-9. Find the range of f ( x ) = 2 sin -1 (3x + 5) + 2
4 È 5p˘
Hence, R f = Íp ,
p
Soln. We have - £ sin -1 (3x + 5) £
p Î 2 ˙˚
2 2 Ex-14. Solve for x: cos -1 x + cos -1 x 2 = 0 .
fi - p £ 2 sin -1
(3 x + 5 ) £ p Soln. Given cos -1 x + cos -1 x 2 = 0
p p p It is possible only when each term will provide us
fi - p + £ 2 sin -1 (3x + 5) + £ p +
4 4 4 the minimum value.
3p 5p So, cos -1 x = 0 & cos -1 x 2 = 0
fi - £ f ( x) £
4 4 fi x = 1 & x2 = 1
È 3p 5 p ˘ fi x = 1 & x = ±1
Hence, R f = Í- , ˙
Î 4 4 ˚ Hence, the solution is x = 1
Ex-10. Solve the inequality: sin x > sin (3x - 1) Ex-15. Solve for x: ÈÎsin -1 x ˘˚ + ÈÎcos -1 x ˘˚ = 0 , where x is a
-1 -1

Soln. We have, sin -1 x > sin -1 (3x - 1) non negative real number and [,] denotes the greatest
integer function.
fi x > (3x - 1)
Soln. Given ÈÎsin -1 x ˘˚ + ÎÈcos -1 x ˘˚ = 0 and x ≥ 0
fi 2x - 1 < 0
fi È -1 ˘ È -1 ˘
fi x<
1 Îsin x ˚ = 0 & Îcos x ˚ = 0
2 fi x Œ[0, sin 1) & x Œ(cos1, 1]
È 1ˆ fi x Œ(cos1, sin 1)
fi x Œ Í-1, ˜
Î 2¯
Ê x2 ˆ
Ex-11. Find the domain of f ( x ) = cos ( 2 x + 4) .
-1
Ex-16. Find the domain of f ( x ) = cos -1 Á 2 ˜
Ë x + 1¯
Soln. We have, -1 £ 2 x + 4 £ 1
fi - 5 £ 2 x £ -3 x2
Soln. We have, -1 £ £1
x2 + 1
5 3
fi - £ x£-
2 2
240 Comprehensive Trigonometry with Challenging Problems & Solutions for Jee Main and Advanced


x2
£1 fi ( )
f ( x ) = cot -1 1 - ( x - 1)
2

x +12
since (1 - ( x - 1) ) £ 1 & 0 £ cot
2 -1
x £p
x 2
fi £1 -1
x2 + 1 and cot x is strictly decreasing function

x 2 (
so cot -1 (1) £ cot -1 1 - ( x - 1)
2
) £ cot -1
(0 )
fi £1
x2 + 1 p
fi £ f ( x) £ p
fi x +1≥ x
2 2 4
fi 1> 0 Èp ˘
Hence, R f = Í , p ˙
Hence, x Œ R Î4 ˚
Ex-17. Solve for x: cos -1 ( x ) > cos -1 x 2 ( ) Ex-21. Solve for x: ÈÎcot -1 x ˘˚ + ÎÈcos -1 x ˘˚ = 0 ,
where [,] = G.I.F.
Soln. We have, cos -1 ( x ) > cos -1 (x ) 2
Soln. We have, ÈÎcot -1 x ˘˚ + ÎÈcos -1 x ˘˚ = 0
fi x < x2
fi Ècot -1 x ˘ = 0 & Ècos -1 x ˘ = 0
fi x 2- x > 0 Î ˚ Î ˚
-1 -1
fi x ( x - 1) > 0 fi 0 £ cot x < 1 & 0 £ cos x < 1

fi x Œ [-1, 0) fi x Œ(cot 1, • ) & x Œ(cos1, 1]


x Œ(cot 1, 1]
( )

Ex-18. Find the domain of f ( x ) = tan
-1
9 - x2
Ex-22. Find the number of solutions of
Soln. Since tan x -1
x, so sin { x} = cos { x} , " x Π[0, 2p ] .
9 - x2 ≥ 0 Soln. We have, sin { x} = cos { x} , " x Œ [0, 2p ]
fi x2 - 9 £ 0
fi tan { x} = 1
fi ( x + 3) ( x - 3) £ 0 p
fi -3£ x £3 fi {x} = tan -1 (1) =
4
Hence, D f = [-3, 3] Hence, the number of solutions = 6.
Ex-19. Find the range of the function (since {x} is a periodic function with period 1, it
p has one solution between 0 to 1. So, there are six
(
f ( x ) = 2 tan -1 1 - x 2 +
6
) solutions between 0 and 6.28).
p p
Soln. We have, - £ tan -1 1 - x 2 £
2
( 4
)
EXERCISE 2
p
fi - p £ 2 tan -1 1 - x 2 £ ( 2
) Q. Find the domains of
1. f ( x ) = sin ( 2 x - 3)
-1
p p p p
fi - p + £ 2 tan -1 1 - x 2 + £ +
6
(
6 2 6
)
-1 Ê x ˆ
2. f ( x ) = sin Á
fi -
5p
£ f ( x) £
2p Ë x - 1˜¯
6 3
3. f ( x ) = cos (3x + 4)
-1
È 5 p 2p ˘
Hence, R f = Í- , ˙ -1 Ê x - 1ˆ
Î 6 3 ˚ 4. f ( x ) = sin Á
Ë 2 ˜¯
Ex-20. Find the range of f ( x ) = cot -1 2 x - x 2 ( )
-1 Ê 1 - x ˆ
Soln. We have, f ( x ) = cot -1 (2 x - x ) 2 5. f ( x ) = cos Á
Ë 5 ˜¯
fi ( (
f ( x ) = cot -1 1 - x 2 - 2 x + 1 )) -1 Ê x - 2 ˆ
6. f ( x ) = sin Á ˜
Ê1- x ˆ
+ cos -1 Á
Ë 3 ¯ Ë 4 ˜¯
Inverse Trigonometric Function 241

(
7. f ( x ) = sin 2 x - 1
-1 2
) 6.5 CONSTANT PROPERTY
p
(i) sin -1 ( x ) + cos -1 ( x ) = , " x Œ[-1,1]
( )
2
8. f ( x ) = 5p sin -1 x - 6 sin -1 x 2
Ê 3 tan -1 x + p ˆ p
(ii) tan -1 ( x ) + cot -1 ( x ) = , " x ΠR
9. f ( x ) = log 2Á ˜ 2
Ë p - 4 tan -1 x ¯
p
Ê 3 ˆ (iii) cosec -1 ( x ) + sec -1 ( x ) = , " x Œ R - ( -1,1)
10. f ( x ) = cos -1 Á 2
Ë 2 + sin x ˜¯ p p
Proof: (i) Let sin -1 ( x ) = q , - £ q £
Ê x 2 + 1ˆ 2 2
11. f ( x ) = sin -1 Á ˜
Ë 2x ¯ fi x = sinq

-1 Ê x + 1ˆ
2 Êp ˆ
12. f ( )
x = cos Á 2 ˜
fi cos Á - q ˜ = x
Ë2 ¯
Ë x ¯
-1
( (
13. f ( x ) = sin log 2 x + 3x + 4
2
)) fi
Êp ˆ Êp ˆ
cos -1 x = Á - q ˜ , 0 £ Á - q ˜ £ p
Ë2 ¯ Ë2 ¯
Ê Ê x2 ˆ ˆ p p
14. f ( x ) = sin -1
Á 2 Á ˜˜
log Thus, sin -1 x + cos -1 x = q + -q = .
Ë Ë 2 ¯¯ 2 2
15. f ( x ) = sin -1 ÈÎ2 - 3x 2 ˘˚ Similarly, we can easily prove (ii) & (iii).

1 sin -1 x 1 6.5.1 Some solved examples


16. f ( x ) = +3 +
x x-2 Ex-1. Find the range of
(
17. f ( x ) = sin log 2 x
-1 2
) f ( x ) = sin -1 x + cos -1 x + tan -1 x .

-1 Ê x ˆ 1 Soln. We have, f ( x ) = sin -1 x + cos -1 x + tan -1 x


18. f ( x ) = e + sin ÁË - 1˜¯ +
x
2 x -1 £ x £ 1
Now, f (1) = sin -1 (1) + cos -1 (1) + tan -1 (1)
19. f ( x ) = sin (log x 2)
-1

p p 3p
20. f ( x ) = sin (log 2 x ) = +0+ =
-1
2 4 4
and f (-1) = sin ( -1) + cos (-1) + tan ( -1)
-1 -1 -1
Q Find the range of
21. f ( x ) = sin ( 2 x - 3)
-1 p p 3p p
= - +p - =p - =
p 2 4 4 4
22. f ( x ) = 2 sin -1 (2 x - 1) -
4 È p 3p ˘
Thus, R f = Í , ˙
( )
23. f ( x ) = 2 cos - x - p
-1 2 Î4 4 ˚
Ex-2. Solve for x: 4 sin -1 ( x - 2) + cos -1 ( x - 2) = p
p
f ( x ) = tan (1 - x ) -
1 -1
24. 2
2 4 Soln. We have, 4 sin -1 ( x - 2) + cos -1 ( x - 2) = p
25. f ( x ) = cot ( 2 x - x )
-1 2
p
fi 3 sin -1 ( x - 2) + =p
2
26. f ( x ) = sin x + cos x + tan x
-1 -1 -1
p
fi 3 sin -1 ( x - 2) =
27. f ( x ) = sin x + sec x + tan x
-1 -1 -1
2
p p
28. f ( x ) = 3 cot -1 x + 2 tan -1 x + fi sin -1 ( x - 2) =
4 6
p
( x - 2) = sin ÊÁË ˆ˜¯ =
1
29. f ( x ) = cosec ÈÎ1 + sin x ˘˚
-1 2

6 2
( (
30. f ( x ) = sin log 2 x + 3x + 4
-1 2
)) fi x = 2+
1 5
=
2 2
242 Comprehensive Trigonometry with Challenging Problems & Solutions for Jee Main and Advanced

5 Ê ˆ Ê ˆ
Hence, the solution is x = .
2 Á 1 ˜ Á 1 ˜
fi xÁ ˜ = x2 Á 2˜
Ex-3. Solve for x: x
Á1+ ˜ Á1+ x ˜
p
( )
sin -1 x 2 - 2 x + 1 + cos -1 x 2 - x = ( ) 2
.
Ë 2¯ Ë 2¯
Ê 2x ˆ Ê 2x ˆ
2
Soln. As we know that, fi ÁË ˜¯ = Á 2 ˜
p x+2 Ë x + 2¯
if sin -1 ( f ( x )) + cos -1 ( g ( x )) = , then
2 ÏÊ 1 ˆ Ê x ˆ ¸
f ( x) = g ( x) fi x ÌÁ ˜ -Á 2 ˜˝ = 0
ÓË x + 2 ¯ Ë x + 2 ¯ ˛
fi (x 2
) (
- 2 x + 1 = x2 - x ) fi x = 0&Á
Ê 1 ˆ Ê x ˆ
=
fi 2x - x = 1 Ë x + 2 ˜¯ ÁË x 2 + 2 ˜¯
fi x =1 fi x = 0& x = 1
Hence, the solution is x = 1. Ex-6. Solve for x: sin -1 x > cos -1 x .
Ex-4. Find the number of real solutions of
p Soln. We have, sin -1 x > cos -1 x
tan -1
x ( x + 1) + sin -1
x + x +1 = .
2
2 fi 2 sin -1 x > sin -1 x + cos -1 x
Soln. We have, p
p fi 2 sin -1 x >
tan -1
x ( x + 1) + sin -1
x + x +1 =
2 2
2 p
fi sin -1 x >
Ê 1 ˆ p 4
fi cos -1 Á ˜ + sin
-1
x2 + x + 1 =
Ë x + x + 1¯
2 2 Ê pˆ
fi x > sin Á ˜
Ë 4¯
Ê 1 ˆ
˜ = x + x +1
2
fi Á 2 1
Ë x + x + 1¯ fi x>
2
fi x2 + x + 1 = 1 Ê 1 ˘
fi x ŒÁ ,1
fi x2 + x = 0 Ë 2 ˙˚
fi x ( x + 1) = 0
fi x = 0 & -1.
Hence, the number of solutions is 2.
EXERCISE 3
Ê x 2 x3 ˆ Q. Solve for x:
Ex-5. If sin -1 Á x - + - ........˜
(sin x)
-1 2
Ë 2 4 ¯ 1. - 3 sin -1 x + 2 = 0
Ê x 4 x6 ˆ p 2. sin -1 x + sin -1 2 y = p
+ cos -1 Á x 2 - + - .......˜ = , for
Ë 2 4 ¯ 2 3. cos -1 x + cos -1 x 2 = 2p
0< x < 2 x. 4. cos -1 x + cos -1 x 2 = 0
Soln. As we know that, if 5. 4 sin -1 ( x - 1) + cos -1 ( x - 1) = p
p
sin -1 ( f ( x )) + cos -1 ( g ( x )) = , then 6. sin -1 x > cos -1 x
2
p
f ( x) = g ( x)
Ê 1 ˆ
(
7. cot -1 Á 2 ˜ + tan -1 x 2 - 1 =
Ë x - 1¯ 2
)
Ê x 2 x3 ˆ Ê 2 x 4 x6 ˆ
fi x - + - ... =Áx - + - ...˜ Ê x 2 - 1ˆ Ê 2 x ˆ 2p
Á ˜ 8. cot -1 Á + tan -1 Á 2 ˜ =
Ë 2 4 ¯ Ë 2 4 ¯ ˜ Ë x - 1¯
Ë 2x ¯ 3
Ê x x2 ˆ Ê x2 x4 ˆ 3p
fi x Á1 - + - ...˜ = x 2 Á1 - + - ...˜ 9. 4 sin -1 x + cos -1 x =
Ë 2 4 ¯ Ë 2 4 ¯ 4
Inverse Trigonometric Function 243

7p Èp ˆ
10. 5 tan -1 x + 3 cot -1 x = 6. cot -1 x Í 2 , p ˜¯ .
4 Î
11. 5 tan -1 x + 4 cot -1 x = 2p Here, we shall discuss, any inverse trigonometric
p function can be expressed in terms of any other inverse
12. cot -1 x - cot -1 ( x + 1) = trigonometric functions.
2
13. ÈÎsin -1 x ˘˚ + ÎÈcos -1 x ˘˚ = 0 Step I:
Ê 1ˆ
(i) sin -1 ( x ) = cosec-1 Á ˜ , x Œ[-1,1] - {0}
14. ÈÎ tan -1 x ˘˚ + ÎÈcot -1 x ˘˚ = 0 Ë x¯

15. Èsin -1 cos -1 sin -1 tan -1 x ˘ = 0 Ê 1ˆ


Î ˚ (ii) cosec-1 ( x ) = sin -1 Á ˜ , x ≥ 1
Ë x¯
16. ÈÎsin -1 cos -1 sin -1 tan -1 x ˘˚ = 1 Ê 1ˆ
(iii) cos -1 ( x ) = sec-1 Á ˜ , x Œ[-1,1] - {0}
5p 2 Ë x¯
(tan x) + (cot x)
-1 2 -1 2
17. =
8 Ê 1ˆ
(iv) sec -1 x = cos -1 Á ˜ , x ≥ 1
Ë x¯
6.5 CONVERSION OF INVERSE Ï -1 Ê 1 ˆ
:x>0
Ô cot ÁË x ˜¯
TRIGONOMETRIC FUNCTIONS Ô
(v) tan ( x ) = Ì
-1

Case I: When x > 0 Ô-p + cot -1 Ê 1 ˆ : x < 0


ÔÓ ÁË ˜¯
Functions Principal values x
È p˘ Proof: Let tan -1 x = q
1. sin -1 x Í0, 2 ˙
Î ˚
Case I: When x > 0
È p˘
2. cos x -1
Í0, 2 ˙ Then tan (q ) > 0
Î ˚
p
Ê pˆ fi 0<q <
3. tan -1 x ÁË 0, ˜¯ 2
2
1 1
Ê p˘ Now, cot q = =
4. cot -1
x ÁË 0, ˙ tan q x

Ê 1ˆ
Ê p˘ fi q = cot -1 Á ˜
5. cosec x -1
ÁË 0, ˙ Ë x¯

Ê 1ˆ
È pˆ Thus, tan -1 x = cot -1 Á ˜ .
6. sec x -1 Ë x¯
Í0, 2 ˜¯
Î
Case II: When x < 0
Case II: When x < 0 Then tanq < 0
Functions Principal values
È p ˘ p
fi - <q < 0
Í- 2 , 0˙
-1
1. sin x 2
Î ˚
p
È p ˆ fi 0 < -q <
2. cosec-1 x Í- 2 , 0˜¯ 2
Î
p
Ê p ˆ fi - p < -p - q < -p +
3. tan -1 x ÁË - , 0˜¯ 2
2 p
fi - p < -p - q < -
Èp ˘ 2
4. cos -1 x Í2 , p˙ Now, cot ( -p - q ) = cot q
Î ˚
Êp ˘ 1 1
5. sec-1 x ÁË , p ˙ and cot q = =
2 ˚ tan q x
244 Comprehensive Trigonometry with Challenging Problems & Solutions for Jee Main and Advanced

Ê 1ˆ
Thus, ( -p - q ) = cot -1 Á ˜ .
Ë x¯ -1 Ô
(iii) cos x = Ì
(
Ïsin -1 1 - x 2
)
: 0 £ x £1

fi (-p - tan x) = cot


-1 -1 Ê 1 ˆ
ÁË ˜¯
x
Ó ( )
Ô p - sin -1 1 - x 2 : - 1 £ x < 0

Ï Ê 1 ˆ
-1 -1 Ê 1 ˆ Ôcosec-1 Á : 0 < x £1
fi tan x = -p + cot Á ˜ . ˜
Ë x¯ -1 Ô Ë 1 - x2 ¯
(iv) cos x = Ì
Ï -1 Ê 1 ˆ Ô -1
Ê 1 ˆ
Ô tan ÁË x ˜¯ :x>0 Ô p - cosec Á ˜ : -1£ x < 0
Ô Ë 1 - x2 ¯
(vi) cot ( x ) = Ì
-1 Ó
Ôp + tan -1 Ê 1 ˆ : x < 0 Ê
ÔÓ ÁË ˜¯ x ˆ
x (v) sin -1 x = tan -1 Á ˜ : -1 < x < 1
Ë 1 - x2 ¯
Proof: Case I: When x > 0
Let cot ( x ) = q
-1 Ï Ê 1 - x2 ˆ
Ô cot -1 Á ˜ :0 < x £1
fi cotq = x ÔÔ ÁË x ˜¯
p (vi) sin -1 x = Ì
Here, cotq > 0 fi 0<q < Ô Ê 1 - x2 ˆ
2 -1
Ô -p + cot Á ˜ : -1 £ x < 0
1 1 ÔÓ ÁË x ˜¯
Now, tan q = =
cot q x
6.5.1 Some Solved Examples
-1 Ê 1 ˆ
fi q = tan Á ˜ Ê1 Ê 3ˆ ˆ
Ë x¯ Ex-1. Find the value of cos Á cos -1 Á ˜ ˜ .
Ë2 Ë 5¯ ¯
Ê 1ˆ
fi cot -1 x = tan -1 Á ˜ 1 Ê 3ˆ
Ë x¯ Soln. Let cos -1 Á ˜ = q
2 Ë 5¯
Case II: When x < 0
Then cotq < 0 Ê 3ˆ
fi cos -1 Á ˜ = 2q
Ë 5¯
p
fi <q < p 3
2 fi cos (2q ) =
5
1
Now, tan ( -p + q ) = tan q = 3
x fi 2 cos 2 q - 1 =
5
(-p + q ) = tan -1 ÊÁË

fi ˜ 3 8
x¯ fi 2 cos 2 q = 1 + =
5 5
Ê 1ˆ
fi - p + cot -1 x = tan -1 Á ˜ fi cos 2 q =
4
Ë x¯ 5
Ê 1ˆ
fi cot -1 x = p + tan -1 Á ˜ fi cosq =
2
Ë x¯
5
Step II: Êp Ê 1ˆˆ
Ex-2. Find the value of sin Á + sin -1 Á ˜ ˜ .
-1
Ïcos -1 1 - x 2
Ô
(i) sin x = Ì
( : 0 £ x £1 ) Ë4 Ë 2¯ ¯
Êp Ê 1ˆˆ
(
Ô - cos -1 1 - x 2 : - 1 £ x < 0
Ó ) Soln. We have, sin Á + sin -1 Á ˜ ˜
Ë4 Ë 2¯ ¯

Ï -1 Ê 1 ˆ Êp ˆ Ê 1ˆ
= sin Á + q ˜ , q = sin -1 Á ˜
Ôsec Á ˜ : 0 £ x £1 Ë4 ¯ Ë 2¯
Ô Ë 1 - x 2¯
(ii) sin -1 x = Ì Êp ˆ 1
Ô Ê 1 ˆ = sin Á + q ˜ , sin q =
-1 Ë4 ¯
Ô - sec Á ˜ : -1£ x < 0
2
Ó Ë 1 - x 2¯
Inverse Trigonometric Function 245

Êpˆ Êpˆ 2 tan -1 x + p


= sin Á ˜ cos (q ) + cos Á ˜ sin (q ) 2. £0
Ë 4¯ Ë 4¯ 4 tan -1 x - p
=
1
cos (q ) +
1
sin (q ) 3. sin -1 x < sin -1 x 2
2 2 4. cos -1 x > cos -1 x 2
=
1
2
¥
2
3
+
1 1
¥
2 2
(
5. log 2 tan -1 x > 1 )
(cot x) -1 2
3 +1 6. - 5 cot -1 x + 6 > 0
=
2 2 7. sin -1 x < cos -1 x
Ex-4. If m is a root of x + 3x + 1 = 0 2
8. sin x > sin (1 - x )
-1 -1

Ê 1ˆ
of tan -1 (m ) + tan -1 Á ˜ . 9. sin -1 2x > cosec-1 x
Ë m¯
Soln. Let m1 and m2 be the roots of 10. tan -1 3x < cot -1 x
x 2 + 3x + 1 = 0 . 11. cos -1 2x ≥ sin -1 x
Thus, m1 + m2 = - 3 < 0 and m1.m2 = 1 .
12. x - 2 x < sin (sin 2)
2 -1
It is possible only when both are negative.
Ê 1ˆ Ê xˆ
Thus, tan -1 (m ) + tan -1 Á ˜ 13. sin -1 Á ˜ < cos -1 ( x + 1)
Ë m¯ Ë 2¯
= tan -1 ( m ) - p + cot -1 ( m ) 14. tan–1 2x > 2 tan–1 x

= tan -1 ( m ) + cot -1 ( m ) - p Ê Ê 1ˆˆ


( )
15. tan cos -1 x £ sin Á cot -1 Á ˜ ˜
Ë Ë 2¯ ¯
p
= -p
2

= -
p 6.6 COMPOSITION OF TRIGONOMETRIC
2 FUNCTIONS AND ITS INVERSE
( ( (
Ex-5. Prove that cos tan -1 sin cot -1 x ) )) =
x2 + 1
x +2
2
Let y = sin -1 x
fi x = sin y

( ( (
Soln. We have, cos tan -1 sin cot -1 x ) )) fi x = sin sin -1 x( )
(
= cos tan -1 (sin q ) , cot q = x ) fi sin sin( -1
)
x =x

Ê -1 Ê 1 ˆ ˆ ( )
Therefore, sin sin -1 x provide us a real value which lies
= cos Á tan Á ˜˜ in [–1, 1]
Ë Ë 1 + x2 ¯ ¯ Hence,
Ê 1 ˆ
= cos j , tan j = Á
( )
(i) sin sin -1 x = x, x £ 1
˜
Ë 1 + x2 ¯ (ii) cos (cos x ) = x, x £ 1
-1

=
x2 + 1 (iii) tan ( tan x ) = x, x ΠR
-1

x +22

(iv) cot (cot x ) = x, x ΠR


-1

EXERCISE 4 (v) cosec (cosec x ) = x, x ≥ 1 -1

Q Solve for x: (vi) sec (sec x ) = x, x ≥ 1


-1

( )
2
1. 6 sin -1 x - p sin -1 x £ 0
246 Comprehensive Trigonometry with Challenging Problems & Solutions for Jee Main and Advanced

(
(i) Graph of y = sin sin -1 x ) 1. D f = R
Y 2. R f = R

y=1 3. It is a non-periodic function.


(v) Graph of y = cosec cosec -1 x ( )
X¢ X Y
O

y = -1
y=1
x = -1 Y¢ x=1

1. D f = [-1,1] X¢ X
O
2. R f = [-1,1]
y = -1
3. It is a non-periodic function.
(
(ii) Graph of y = cos cos -1 x )
x = -1 Y¢ x=1
Y

1. D f = ( -•, - 1] » [1, • )
y=1
2. R f = ( -•, - 1] » [1, • )
X¢ X
O 3. It is a non-periodic function.
y = -1 (vi) Graph of y = sec sec x
-1
( )
Y
x = -1 Y¢ x=1

1. D f = [-1,1]
y=1
2. R f = [-1,1]
3. It is a non-periodic function. X¢ X
O
(iii) Graph of y = tan (tan -1 x )
y = -1
Y

y=x

x = -1 Y¢ x=1


O
X 1. D f = ( -•, - 1] » [1, • )
2. R f = ( -•, - 1] » [1, • )
3. It is a non-periodic function.

1. D f = R 6.6.1 Some Solved Examples
2. R f = R Ex-1. Let f ( x ) = sin x + cos x ,
-1 -1

3. It is a non-periodic function.
(
(iv) Graph of y = cot cot -1 x ) Ê 1 ˆ
(i) f Á 2 ˜ , m Œ R
Y Ë m + 1¯
y=x Ê m2 ˆ
(ii) f Á 2 ˜ , m Œ R
Ë m + 1¯
m ˆ
(iii) f ÊÁ
X¢ X
O , m ŒR
Ë m 2 + 1˜¯

(
(iv) f m 2 - 2m + 6 , m ΠR)

Inverse Trigonometric Function 247

( )
(v) f m 2 + 1 , m Œ R Ê 1ˆ
Now, tan -1 (m ) + tan -1 Á ˜
Ë m¯
p
Soln. As we know that, sin -1 x + cos -1 x = , for every = tan ( m ) - p + cot ( m )
-1 -1

x in [–1, 1] 2
= -p + tan ( m ) + cot ( m )
-1 -1
1
(i) since 0 < £1 p
m2 + 1 = -p +
2
Ê 1 ˆ p p
so, f Á 2 ˜ =
Ë m + 1¯ 2 = -
2
m2 Ex-4. Solve for x:
(ii) Since, 0 £ < 1,
m2 + 1 Ê Ê 2 x2 + 5ˆ ˆ
sin -1 Á sin Á 2 ˜ ˜ > sin (sin 3)
-1
Ê m2 ˆ p Ë Ë x + 2 ¯¯
so f Á 2 ˜ = .
Ë m + 1¯ 2
2 x2 + 5 1
1 m 1 Soln. Let m = = 2+
(iii) Since, - £ £ x +2 2
x +2
2
2 m +1 2
2

Ê m ˆ p È 5˘
so, f Á 2 ˜ = Thus, m Œ Í2, ˙
Ë m + 1¯ 2 Î 2˚
Ê Ê 2 x2 + 5ˆ ˆ
(iv) Since m 2 - 2m + 6 = (m - 1) + 5
2
now, sin -1 Á sin Á 2 ˜ ˜ > sin (sin 3)
-1

Ë Ë x + 2 ¯¯
Thus, 5 £ (m - 1) + 5 < •
2

Ê Ê 2 x2 + 5ˆ ˆ
Hence, f ((m - 1) + 5)
2
fi sin -1 Á sin Á p - 2
Ë Ë
˜˜
x + 2 ¯¯
(v) Also, 1 £ m 2 + 1 < •
< sin -1 (sin (p - 3))
(
So, f m + 12
) Ê 2 x2 + 5ˆ
2p fi p -Á 2 ˜ >p -3
Ex-2. If cos -1 x + cos -1 y = Ë x +2 ¯
3
Ê 2 x2 + 5ˆ
value of sin -1 x + sin -1 y . fi Á 2 ˜ <3
Ë x +2 ¯
2p
Soln. Given, cos -1 x + cos -1 y = Ê 2 x2 + 5 ˆ
3 - 3˜ < 0
fi Á 2
Now, sin -1 x + sin -1 y Ë x +2 ¯
p p Ê - x2 - 5ˆ
= - cos -1 x + - cos -1 y fi Á 2 ˜ <0
2 2 Ë x +2 ¯
(
= p - cos -1 x + cos -1 y ) Ê x2 + 5 ˆ
2p fi Á 2 ˜ >0
=p- Ë x + 2¯
3 fi x ŒR
p
=
3
6.7 COMPOSITION OF INVERSE
Ex-3. If m is the root of x + 3x + 1 = 0 TRIGONO METRIC FUNCTIONS
2

Ê 1ˆ
of tan -1 (m ) + tan -1 Á ˜ .
Ë m¯ AND TRIGONOMETRIC FUNCTIONS
È p p˘
Soln. Let m1 and m2 are the two roots of the given equation. (i) A function f : R Æ Í- , ˙ is
Now, m1 + m 2 = –3 and m1.m2 = 1 Î 2 2˚
fi m1 and m2 are two –ve roots. f ( x ) = sin -1 (sin x )
248 Comprehensive Trigonometry with Challenging Problems & Solutions for Jee Main and Advanced

Graph of f ( x ) = sin (sin x ) (iii) tan -1 ( tan x ) :


-1

Y p Ê p pˆ
A function f : R - (2n + 1) Æ Á- , ˜
2 Ë 2 2¯
y = p/2
f ( x ) = tan -1 ( tan x )
y = -p/2
X¢ X
Graph of f ( x ) = tan ( tan x ) :
-p O p 2p -1
y = -p/2
Y

Y¢ y = p/2

1. D f = R X¢
O
X

È p p˘ y = -p/2
2. R f = Í- , ˙
Î 2 2˚
3. It is an odd function. Y¢

4. It is a periodic function with period 2p p


1. D f = R - ( 2n + 1) , n ΠI
Ï p p 2
Ô x :-
2
£x£
2 Ê p pˆ
Ô 2. R f = Á - , ˜
p 3p Ë 2 2¯
Ô p-x : £x£
Ô 3. It is an odd function.
5. sin (sin x ) = Ì
-1 2 2
3p 5p 4. It is a periodic function with period p
Ô x - 2p : £x£
Ô 2 2 Ï -p p
Ô 3p p Ô x :
2
<x<
2
Ô-p - x :- £x£- Ô
Ó 2 2 Ô x-p : < x < p
p 3
Ô
5. tan -1 ( tan x ) = Ì
2 2
(ii) cos -1 (cos x ) :
Ô x - 2p : < x < p
p 3
A function f : R Æ [0, p ] Ô 2 2
Ô -3p -p
as f ( x ) = cos (cos x )
-1
Ôx + p : <x<
Ó 2 2
Graph of f ( x ) = cos (cos x ) :
-1
(iv) cot (cot x ) :
-1
Y
A function f : R - ( n p ) Æ (0, p )
y=p

as f ( x ) = cot (cot x )
-1
y = p/2
O
Graph of f ( x ) = cot (cot x ) :
-1

X¢ X Y

Y¢ y =p

1. D f = R y = p/2

2. R f = [0, p ] X¢ X
O
3. It is neither odd nor even function.

4. It is periodic function with period 2p .
Ï x :0 £ x £ p 1. D f = R - n p , n Œ I
Ô 2p - x
Ô : p £ x £ 2p 2. R f = (0, p )
5. cos -1 (cos x ) = Ì
Ô x - 2p : 2p £ x £ 3p 3. It is neither even nor odd function.
ÔÓ - x : -p £ x £ 0 4. It is a periodic function with period p.
Inverse Trigonometric Function 249

Ï x :0< x<p 1. D f = R - ( 2n + 1) p , n Œ I
Ô x - p : p < x < 2p 2
Ô
5. cot -1 (cot x ) = Ì Ïp ¸
Ô x - 2p : 2p < x < 3p 2. R f = [0, p ] - Ì ˝
ÔÓ p + x : - p < x < 0 Ó2˛
3. It is neither even nor odd function.
(v) cosec (cosec x ) : A function
-1
4. It is a periodic function with period 2p
È p p˘
f : R - ( np ) Æ Í- , ˙ - {0} Ï x :0£ x£p
Î 2 2˚ Ô 2p - x : p £ x £ 2p
as f ( x ) = cosec (cosec x )
-1 5. sec-1 (sec x ) = ÔÌ
Ô x - 2p : 2p £ x £ 3p
Graph of f (x) = cosec (cosec x )
-1
ÔÓ - x : -p £ x £ 0
Y
6.8.1 Some Solved Examples
y = p/2 Ex-1. Find the value of
(i) sin -1 (sin 3)
X¢ X
-2p -p O p 2p (ii) sin -1 (sin 5)
y = -p/2
(iii) sin -1 (sin 7 )

(iv) sin (sin 10)


Y¢ -1

1. D f = R - n p , n ΠI
(v) sin (sin 20)
-1

È p p˘
2. R f = Í- , ˙ - {0} Soln. (i) sin (sin 3)
-1
Î 2 2˚
= sin (sin (p - 3))
-1
3. It is an odd function
4. It is a periodic function with period 2p
= (p - 3)
5. cosec (cosec x )
-1
(ii) sin (sin 5)
-1

Ï -p p
= sin (sin (5 - 2p ))
-1
Ô x :
2
£x£
2
Ô
Ô p -x: £x£p 3 p = (5 - 2p )
Ô 2 2 (iii) sin (sin 7 )
-1

3 p 5p
= sin (sin (7 - 2p ))
Ô x - 2p : £x£ -1
Ô 2 2
Ô -3p -p = (7 - 2p )
Ô- x - p : £x£
Ó (iv) sin (sin 10)
2 2 -1

(vi) sec-1 (sec x ) : A function


= sin (sin (3p - 10))
-1

p Ïp ¸
f : R - ( 2n + 1) Æ [0, p ] - Ì ˝ is = (3p - 10)
2 Ó2˛
(v) sin (sin 20)
-1
f ( x ) = sec-1 (sec x )
= sin (sin ( 20 - 6p ))
-1
Graph of f ( x ) = sec (sec x )
-1

Y
= ( 20 - 6p )
Ex-2. Find the value of
(i) cos (cos 2)
y=p -1

(ii) cos (cos 3)


-1
y = p/2

(iii) cos (cos 5)


-1
X¢ X
(iv) cos (cos 7 )
O -1


250 Comprehensive Trigonometry with Challenging Problems & Solutions for Jee Main and Advanced

(v) cos (cos 10)


-1
Soln. We have, cos -1 (sin ( -5))
Soln. (i) cos (cos 2) = 2 = cos ( - sin 5)
-1 -1

(ii) cos (cos 3) = 2 = p - cos (sin 5)


-1 -1

(iii) cos (cos 5)


-1 Ê Êp ˆˆ
= p - cos -1 Á cos Á - 5˜ ˜
Ë Ë ¯¯
= cos (cos ( 2p - 5))
-1 2
Êp ˆ
= ( 2p - 5) = p - Á - 5˜
Ë2 ¯
(iv) cos (cos 7 )
-1
p
= +5
= cos
-1
(cos (7 - 2p )) 2

= (7 - 2p ) Ex-6. Find f ' ( x ) , where f ( x ) = sin (sin x )


-1

(v) cos (cos10)


-1
and -2p £ x £ p .

= cos (cos ( 4p - 10))


-1 Soln. We have, f ( x )

= ( 4p - 10) = sin (sin x )


-1

Ex-3. Find the value of = x + 2p - p - x + x + p - x


-1
( tan 3) = 2p
(i) tan
fi f ' ( x) = 0 .
(ii) tan ( tan 5)
-1

Ex-7. Find f ' ( x ) , where f ( x ) = cos (cos x )


-1

(iii) tan -1 ( tan 7 )


and -p £ x £ 2p .
(iv) tan -1 ( tan 10) Soln. We have f (x)
tan -1 ( tan 15) = cos (cos x )
-1
(v)
Soln. (i) tan -1 ( tan 3) = - x + x + 2p - x
= 2p - x
= tan ( tan (3 - p ))
-1
fi f ' ( x ) = -1
= (3 - p )
Ê Ê 2 x2 + 5ˆ ˆ
(ii) tan -1 ( tan 5) Ex-8. Solve for x: sin -1 Á sin Á 2 ˜˜ < p - 3
Ë Ë x +1 ¯¯
= tan ( tan (5 - 2p ))
-1
Ê Ê 2 x2 + 5ˆ ˆ
= (5 - 2p ) Soln. We have, sin -1 Á sin Á 2 ˜˜ < p - 3
Ë Ë x +1 ¯¯
(iii) tan -1 ( tan 7 )
Ê Ê Ê 2 x2 + 5ˆ ˆ ˆ
sin -1 Á sin Á p - Á 2
= tan ( tan (7 - 2p )) ˜˜˜ > p - 3
-1

Ë Ë Ë x +1 ¯¯¯
= (7 - 2p )
Ê Ê 2 x2 + 5ˆ ˆ
(iv) tan -1 ( tan 10) fi Áp - Á 2 ˜˜ < p - 3
Ë Ë x +1 ¯¯
= tan ( tan (10 - 3p ))
-1
Ê 2 x2 + 5ˆ
-Á 2 ˜ < -3
= (10 - 3p ) fi
Ë x +1 ¯
(v) tan -1 ( tan 15) Ê 2 x2 + 5ˆ
fi Á 2 ˜ >3
= tan ( tan (15 - 5p ))
-1
Ë x +1 ¯
= (15 - 5p ) . Ê 2 x2 + 5 ˆ
fi Á 2 - 3˜ > 0
Ex-4. Find the value of cos -1 (sin ( -5)) . Ë x +1 ¯
Inverse Trigonometric Function 251

9. sin (sin 20)


Ê 2 x 2 + 5 - 3x 2 - 3ˆ -1
fi Á ˜ >0
Ë x2 + 1 ¯ 10. sin (sin 50)
-1

11. sin (sin 80)


fi x2 < 2 -1

fi - 2<x< 2 12. sin (sin 100)


-1

Ex-9. Find the integral values of x satisfying the inequality, Q. Evaluate each of the following:
x 2 - 3x < sin -1 (sin 2) . Ê Ê 7p ˆ ˆ
13. cos -1 Á cos Á ˜ ˜
Ë Ë 6 ¯¯
Soln. We have, x - 3x < sin (sin 2)
2 -1

Ê Ê 31p ˆ ˆ
x 2 - 3x < sin -1 (sin (p - 2)) 14. cos -1 Á cos Á
fi Ë Ë 7 ˜¯ ˜¯
fi x 2 - 3x < (p - 2) Ê Ê 103p ˆ ˆ
15. cos -1 Á cos Á
fi x - 3x + (2 - p ) < 0
2 Ë Ë 7 ˜¯ ˜¯
Ê 3 + 1 + 4p ˆ Ê 3 - 1 + 4p ˆ 16. cos -1 (cos (3))
fi Áx- ˜ Á x- ˜ <0
Ë 2 ¯Ë 2 ¯ 17. cos -1 (cos (5))

fi 3 - 1 + 4p
<x<
3 + 1 + 4p 18. cos -1 (cos (7 ))
2 2
Ex-10. Find the value of 19. cos -1 (cos (10))

sin -1 (sin 50) + cos -1 (cos 50) + tan -1 ( tan 50) 20. cos -1 (cos (12))
Soln. We have 21. cos -1 (cos (15))
= sin (sin 50) + cos (cos 50) + tan ( tan 50)
-1 -1 -1
22. cos -1 (cos ( 40))
= sin
-1
(sin (50 - 16p )) + cos (cos (16p - 50))
-1
23. cos -1 (cos (60))
+ tan -1 ( tan (50 - 16p ))
24. cos -1 (cos (100))
= (50 - 16p ) + (16p - 50) + (50 - 16p )
Q. Find the value of
= (50 - 16p )
25. sin (sin 1) + sin (sin 2) + sin (sin 3)
-1 -1 -1

26. sin (sin 10) + sin (sin 20)


-1 -1

EXERCISE 5 + sin -1 (sin 30) + sin -1 (sin 40)

27. cos (cos1) + cos (cos 2)


-1 -1
Q. Evaluate each of the following:
Ê Ê 7p ˆ ˆ + cos -1 (cos 3) + cos -1 (cos 4)
1. sin -1 Á sin Á ˜ ˜
Ë Ë 3 ¯¯
28. cos (cos10) + cos (cos 20)
-1 -1
Ê Ê 11p ˆ ˆ
2. sin -1 Á sin Á
Ë Ë 3 ˜¯ ˜¯ + cos -1 (cos 30) + cos -1 (cos 40)

29. sin (sin 10) + cos (cos10)


-1 -1
Ê Ê 49p ˆ ˆ
3. sin -1 Á sin Á
Ë Ë 8 ˜¯ ˜¯
30. sin (sin 50) + cos (cos 50)
-1 -1

4. sin (sin 3)
-1
31. sin (sin 100) + cos (cos100)
-1 -1

5. sin (sin 5)
-1
32. cos (sin ( -5)) + sin (cos ( -5))
-1 -1

6. sin
-1
(sin 7) 33. Find the number of ordered pairs of (x, y) satisfying
7. sin (sin 10)
-1 the equations y = sin x and

8. sin -1 (sin 12) y = cos -1 (cos x ) , where x Œ[-2p , 2p ]


252 Comprehensive Trigonometry with Challenging Problems & Solutions for Jee Main and Advanced

34. Let f ( x ) = cos -1 (cos x ) - sin -1 (sin x ) in [0, p ] . Find fi cos A = 1 - x 2 , cos B = 1 - y 2
the area bounded by f (x) and x-axis. Now, sin ( A + B )
Q. Evaluate the following:
Ê Ê 2p ˆ ˆ (i) sin -1 x + sin -1 y
35. tan -1 Á tan Á ˜ ˜
Ë Ë 3 ¯¯ Ï a : x2 + y 2 £ 1
Ê Ê 29p ˆ ˆ Ô
36. tan -1 Á tan Á Ô p - a : x > 0, y > 0, x + y > 1
2 2
Ë Ë 5 ˜¯ ˜¯ = Ì
Ô a : xy < 0, x 2 + y 2 > 1
Ê Ê 121p ˆ ˆ Ô
37. tan -1 Á tan Á Ó-p - a : x < 0, y > 0, x + y > 1
2 2
Ë Ë 12 ˜¯ ˜¯
38. tan ( tan 3)
-1
(
where a = sin -1 x 1 - y 2 + y 1 - x 2 )
39. tan
-1
( tan 5) Proof: Let sin -1 x = A & sin -1 y = B
40. tan -1 ( tan 7 ) fi x = sin A & y = sin B
41. tan -1 ( tan 10) È p p˘
and A, B Œ Í- , ˙
42. tan -1 ( tan 20) Î 2 2˚

tan -1 ( tan 50) fi cos A = 1 - x 2 , cos B = 1 - y 2


43.
Now, sin ( A + B )
44. tan -1 ( tan 1) + tan -1 ( tan 2)
= sin A cos B + cos A.sin B
+ tan -1 ( tan 3) + tan -1 ( tan 4)
= x 1- y + y 1- x
2 2
45. tan -1 ( tan 20) + tan -1 ( tan 40)
+ tan -1 ( tan 60) + tan -1 ( tan 80) Case I: when -1 £ x, y £ 1 & x 2 + y 2 £ 1

sin -1 (sin 15) + cos -1 (cos15) + tan -1 ( tan 15) In this case, x 2 + y 2 £ 1
46.
47. sin -1 (sin 50) + cos -1 (cos 50) - tan -1 ( tan 50) fi 1 - x2 ≥ y 2 & 1 - y 2 ≥ x2

48. 3x 2 + 8 x < 2 sin -1 (sin 4) - cos -1 (cos 4) fi (1 - x ) (1 - y ) ≥ x y


2 2 2 2

Ê Ê 2 x2 + 4 ˆ ˆ
49. sin -1 Á sin Á 2 ˜˜ < p - 3
fi (1 - x ) (1 - y ) - xy ≥ 0
2 2

Ë Ë x +1 ¯¯
fi cos ( A + B ) ≥ 0
fi A+B
6.9 SUM OF ANGLES quadrant.
(i) sin -1 x + sin -1 y Ê p pˆ
fi A + B ŒÁË - , ˜¯
2 2
Ï a : x2 + y 2 £ 1
Ô sin ( A + B ) = x 1 - y 2 + y 1 - x 2
Ô p - a : x > 0, y > 0, x + y > 1
2 2
= Ì
Ô
Ô
a : xy < 0, x 2 + y 2 > 1 fi (
A + B = sin -1 x 1 - y 2 + y 1 - x 2 )
Ó-p - a : x < 0, y > 0, x + y > 1
2 2
-1 -1 -1
fi sin x + sin y = sin x 1 - y + y 1 - x
2
{2
}
where a = sin -1
(x 1 - y 2
+ y 1- x 2
) Case II: when x y < 0 and x 2 + y 2 > 1
Proof: Let sin -1 x = A & sin -1 y = B In this case, we have, xy < 0
fi (x > 0 and y < 0) or (x < 0 and y > 0)
fi x = sin A & y = sin B
Ê p˘ È p ˆ
È p p˘ fi A ŒÁ 0, ˙ and B Œ Í- , 0˜
and A, B Œ Í- , ˙ Ë 2˚ Î 2 ¯
Î 2 2˚
Inverse Trigonometric Function 253

p p p
fi - £ A+ B £ fi £ A + B £ p , since A + B Œ(0, p )
2 2 2
Also, x 2 + y 2 > 1 p
fi -p £ - ( A + B ) £ -
2
fi 1 - x > y and 1 - y > x
2 2 2 2
p
fi 0 £ p - ( A + B) £
fi (1 - x ) (1 - y ) > x y
2 2 2 2
2
Now, sin ( A + B ) = x 1 - y 2 + y 1 - x 2
fi (1 - x ) (1 - y < xy
2 2

fi sin (p - ( A + B )) = x 1 - y 2 + y 1 - x 2
fi - xy < (1 - x ) (1 - y ) < xy
2 2

(1 - x ) (1 - y ) - xy > 0, (∵ xy < 0)
2 2 fi (p - ( A + B )) = sin -1 ( x 1 - y 2 + y 1 - x2 )

fi cos ( A + B ) > 0 fi (
A + B = p - sin -1 x 1 - y 2 + y 1 - x 2 )
fi A+B fi sin -1 x + sin -1 y
the fourth quadrant.


È p p˘
A + B Œ Í- , ˙
(
= p - sin -1 x 1 - y 2 + y 1 - x 2 )
Î 2 2˚
Case IV: when -1 £ x, y < 0 and x 2 + y 2 > 1
fi sin ( A + B ) = x 1 - y 2 + y 1 - x 2
In this case, we have, -1 £ x, y < 0
fi (
A + B = sin -1 x 1 - y 2 + y 1 - x 2 ) fi
È p ˆ È p ˆ
A Œ Í- , 0˜ and B Œ Í- , 0˜
fi sin -1
x + sin -1
y Î 2 ¯ Î 2 ¯

(
= sin -1 x 1 - y 2 + y 1 - x 2 ) fi A + B Œ[-p , 0)
Also, x 2 + y 2 > 1
Case III: when 0 < x, y £ 1 and x 2 + y 2 < 1 fi 1 - x 2 < y 2 and 1 - y 2 < x 2
In this case we have 0 < x, y £ 1
fi (1 - x ) (1 - y ) < x y
2 2 2 2

È p˘ È p˘
fi A Œ Í0, ˙ and B Œ Í0, ˙
Î 2˚ Î 2˚ fi (1 - x ) (1 - y ) < xy (since, xy > 0)
2 2

fi A + B Œ[0, p ]
Also, x + y < 1
2 2
fi (1 - x ) (1 - y ) - xy < 0
2 2

fi cos (A + B) < 0
fi 1 - x 2 < y 2 and 1 - y 2 < x 2 fi A+B
fi (1 - x ) (1 - y ) < x y
2 2 2 2 third quadrant.
p
fi -p £ A + B £
fi (1 - x ) (1 - y ) < xy
2 2 2
p
fi - £ - ( A + B) £ p
fi 1 - x 2 < y 2 and 1 - y 2 < x 2 2
fi (1 - x ) (1 - y ) < x y
2 2 2 2

p
- £ -p - ( A + B ) £ 0
2
fi (1 - x ) (1 - y ) < xy
2 2
fi sin ( A + B ) = x 1 - y 2 + y 1 - x 2

fi (1 - x ) (1 - y ) - xy < 0
2 2
fi - sin {p + ( A + B )} = x 1 - y 2 + y 1 - x 2


cos ( A + B ) < 0
A + B lies either in II quadrant or in III quadrant.
fi (
sin {-p - ( A + B )} = x 1 - y 2 + y 1 - x 2 )
254 Comprehensive Trigonometry with Challenging Problems & Solutions for Jee Main and Advanced

fi (
-p - ( A + B ) = sin -1 x 1 - y 2 + y 1 - x 2 ) (
= cos -1 xy - 1 - x 2 1 - y 2 )
fi (
A + B = -p - sin -1 x 1 - y 2 + y 1 - x 2 ) Case II:
when -1 £ x, y < 0 and x + y < 0
fi sin -1 x + sin -1 y In this case -1 £ x, y < 0
fi A, B Œ[0, p ]
(
= -p - sin -1 x 1 - y 2 + y 1 - x 2 ) fi 0 £ A + B £ 2p and x + y £ 0
fi cos A + cos B £ 0
(ii) sin -1 x - sin -1 y
fi cos A £ cos (p - B)
Ï a : x2 + y 2 £ 1
Ô fi A≥p -B
Ô p - a : x > 0 , y < 0 , x + y >1
2 2
= Ì fi A+ B ≥p
Ô a : x y > 0 , x 2 + y 2 >1
Thus, p £ A + B £ 2p
Ô
Ó-p - a : x < 0 , y > 0 , x + y >1
2 2
fi -2p £ - ( A + B ) £ -p
where a = sin -1
(x 1 - y 2
- y 1- x 2
) fi 0 £ 2p - ( A + B ) £ p

Proof: Do yourself. È p p˘
fi A + B Œ Í- , ˙
(iii) cos -1 x + cos -1 y Î 2 2˚
Ïa :x + y ≥ 0 fi sin ( A + B ) = x 1 - y 2 + y 1 - x 2
= Ì
Ó2p - a : x + y < 0

where a = cos
-1
( xy - 1- x 2
1- y 2
)
fi (
A + B = sin -1 x 1 - y 2 + y 1 - x 2 )
fi sin -1 x + sin -1 y

)
-1 -1
Proof: Let cos x = A and cos y = B
Then x = cos A and y = cos B (
= sin -1 x 1 - y 2 + y 1 - x 2

fi A Œ[0, p ] and B Œ[0, p ] Case III:


Now, when 0 < x, y £ 1 and x 2 + y 2 < 1
sin A = 1 - x 2 and sin B = 1 - y
2
In this case we have 0 < x, y £ 1

cos (A + B) = xy - 1 - x 1 - y
2 2 È p˘ È p˘
fi A Œ Í0, ˙ and B Œ Í0, ˙
Î 2 ˚ Î 2˚
cos (A – B) = xy + 1 - x 1 - y
2 2
fi A + B Œ[0, p ]
Case I:
-1 £ x, y £ 1 and x + y ≥ 0 Also, x 2 + y 2 < 1
In this case, -1 £ x, y £ 1 fi 1 - x 2 < y 2 and 1 - y 2 < x 2


0 £ A + B £ 2p and x + y ≥ 0
0 £ A + B £ 2p and cos A + cos B ≥ 0
fi (1 - x ) (1 - y ) < x y
2 2 2 2

fi cos A ≥ - cos B fi (1 - x ) (1 - y ) < xy


2 2

fi cos A ≥ cos (p - B)
fi A£p -B
fi (1 - x ) (1 - y ) - xy < 0
2 2

fi 0£ A+ B £p fi cos ( A + B ) < 0

cos ( A + B ) = xy - 1 - x 2 1 - y 2 fi A + B lies either in II quadrant or in III quadrant.



p
£ A + B £ p , since A + B Œ(0, p )
fi (
A + B = cos -1 xy - 1 - x 2 1 - y 2 ) fi
2
p
fi cos -1 x + cos -1 y fi -p £ - ( A + B ) £ -
2
Inverse Trigonometric Function 255

fi 0 £ p - ( A + B) £
p
2
(
= -p - sin -1 x 1 - y 2 + y 1 - x 2 )
Now, sin ( A + B ) = x 1 - y 2 + y 1 - x 2 (ii) sin -1 x - sin -1 y
Ï a : x2 + y 2 £ 1
fi sin (p - ( A + B )) = x 1 - y 2 + y 1 - x 2 Ô
Ô p - a : x > 0 , y < 0 , x + y >1
2 2

fi (p - ( A + B )) = sin -1 ( x 1 - y 2 + y 1 - x2 ) = Ì
Ô a : x y > 0 , x 2 + y 2 >1
Ô
( ) Ó-p - a : x < 0 , y > 0 , x + y >1
2 2
fi A + B = p - sin -1 x 1 - y 2 + y 1 - x 2

-1 -1 -1
(
fi sin x + sin y = p - sin x 1 - y + y 1 - x
2 2
) (
where a = sin -1 x 1 - y 2 - y 1 - x 2 )
Proof: Do yourself.
Case IV: (iii) cos -1 x + cos -1 y
when -1 £ x, y < 0 and x 2 + y 2 > 1
Ïa :x + y ≥ 0
In this case, we have, -1 £ x, y < 0 = Ì
Ó2p - a : x + y < 0
È p ˆ È p ˆ
fi A Œ Í- , 0˜ and B Œ Í- , 0˜
Î 2 ¯ Î 2 ¯ (
where a = cos -1 xy - 1 - x 2 1 - y 2 )
fi A + B Œ[-p , 0)
Proof: Let cos -1 x = A and cos -1 y = B
Also, x + y > 1
2 2
Then x = cos A and y = cos B
fi 1 - x 2 < y 2 and 1 - y 2 < x 2 fi A Œ[0, p ] and B Œ[0, p ]

fi (1 - x ) (1 - y ) < x y
2 2 2 2
Now, sin A = 1 - x 2 and sin B = 1 - y
2

fi (1 - x ) (1 - y ) < xy (since, xy > 0)


2 2
cos (A + B) = xy - 1 - x 1 - y
2 2

(1 - x ) (1 - y ) - xy < 0 cos (A – B) = xy + 1 - x 1 - y
2 2
2 2

Case I:
fi cos (A + B) < 0 when -1 £ x, y £ 1 and x + y ≥ 0
fi A+B
third quadrant. In this case, -1 £ x, y £ 1
p fi 0 £ A + B £ 2p and x + y ≥ 0
fi -p £ A + B £
2 fi 0 £ A + B £ 2p and cos A + cos B ≥ 0
p fi cos A ≥ - cos B
fi - £ - ( A + B) £ p
2 fi cos A ≥ cos (p - B )
p
fi - £ -p - ( A + B ) £ 0 fi A£p -B
2 fi 0£ A+ B £p
fi sin ( A + B ) = x 1 - y + y 1 - x cos ( A + B ) = xy - 1 - x 2 1 - y 2
2 2

fi - sin {p + ( A + B )} = x 1 - y + y 1 - x
2 2
fi (
A + B = cos -1 xy - 1 - x 2 1 - y 2 )
(
fi sin {-p - ( A + B )} = x 1 - y + y 1 - x
2 2
) (
fi cos -1 x + cos -1 y = cos -1 xy - 1 - x 2 1 - y 2 )
fi -p - ( A + B ) = sin
-1
(x 1 - y 2
+ y 1- x ) 2 Case II:
when -1 £ x, y < 0 and x + y < 0
-1
fi A + B = -p - sin x 1 - y
2
( + y 1- x )
2
In this case -1 £ x, y < 0
fi A, B Œ[0, p ]
fi sin -1 x + sin -1 y
fi 0 £ A + B £ 2p and x + y £ 0
256 Comprehensive Trigonometry with Challenging Problems & Solutions for Jee Main and Advanced

fi cos A + cos B £ 0 x+ y
<0

fi cos A £ cos (p - B) 1 - xy
fi A≥p -B fi tan (A + B) > 0
fi A+ B ≥p fi A+B
third quadrant.
Thus, p £ A + B £ 2p
fi 0<A+B<p
fi -2p £ - ( A + B ) £ -p x+ y
fi tan ( A + B ) =
fi 0 £ 2p - ( A + B ) £ p 1 - xy

Now, cos (A + B) = xy - 1 - x 1 - y
2 2 Ê x+ yˆ
fi ( A + B ) = tan -1 Á
Ë 1 - xy ˜¯
fi cos (2p - ( A + B )) = xy - 1 - x 1 - y
2 2
Ê x+ yˆ
fi tan -1 x + tan -1 y = tan -1 Á
Ë 1 - xy ˜¯
fi (2p - ( A + B )) = cos-1 ( xy - 1 - x2 1 - y 2 ) Case II: when x < 0, y < 0 and xy < 1

fi (
A + B = 2p - cos -1 xy - 1 - x 2 1 - y 2 ) In this case, x < 0, y < 0 and xy < 1


x+ y
<0
-1 -1 1 - xy
fi cos x + cos y
fi tan (A + B) < 0
(
= 2p - cos -1 xy - 1 - x 2 1 - y 2 ) fi A + B lies either in II quadrant or in IV
quadrant
(iv) cos -1 x - cos -1 y fi A + B lies in the IV quadrant
fi -p < A + B < 0
Ï a : x£ y
x+ y
= Ì Now, tan ( A + B ) =
Ó -a : x> y 1 - xy

(
where a = cos -1 xy + 1 - x 2 1 - y 2 ) fi
Ê x+ yˆ
A + B = tan -1 Á
Ë 1 - xy ˜¯
Proof: Do yourself.
tan -1 x + tan -1 y Ê x+ yˆ
(v) fi tan -1 x + tan -1 y = tan -1 Á
Ë 1 - xy ˜¯
Ï a: xy < 1
Ôp + a : x > 0, y > 0, xy > 1 Case III: when (x > 0, y < 0) or (x < 0 and y > 0)
Ô
Ô -p + a : x < 0, y < 0 , xy > 1 In this case, x >0 and y < 0
Ô Ê pˆ Ê p ˆ
= Ìp fi A ŒÁ 0, ˜ and B ŒÁ - , 0˜
Ô : x > 0, y > 0, xy = 1 Ë 2¯ Ë 2 ¯
Ô2
Ôp Ê p pˆ
: x < 0, y < 0 , xy = 1 fi A + B ŒÁ - , ˜
ÔÓ 2 Ë 2 2¯
x+ y
Ê x+ yˆ
where a = tan -1 Á Now, tan ( A + B ) =
Ë 1 - xy ˜¯ 1 - xy
Ê x+ yˆ
Proof: Let tan -1 x = A and tan -1 y = B fi tan -1 x + tan -1 y = tan -1 Á
Ë 1 - xy ˜¯
Then x = tan A and y = tan B.
Similarly, if x < 0 and y > 0, we have
Ê p pˆ Ê p pˆ
fi A ŒÁ - , ˜ and B ŒÁ - , ˜ Ê x+ yˆ
Ë 2 2¯ Ë 2 2¯ tan -1 x + tan -1 y = tan -1 Á
Ë 1 - xy ˜¯
tan A + tan B x+ y
tan ( A + B ) = = It follows from above three cases that
1 - tan A.tan B 1 - xy
Ê x+ yˆ
Case I: when x > 0, y > 0 and xy < 1 tan -1 x + tan -1 y = tan -1 Á
Ë 1 - xy ˜¯
In this case, x > 0, y > 0 and xy < 1
Inverse Trigonometric Function 257

Case IV: when x > 0, y > 0 and xy > 1 Ê ˆ


In this case, we have x, y > 0 and xy > 1 fi (p + ( A + B )) = tan -1 ÁË 1x-+xyy ˜¯
x+ y
fi <0 Ê x+ yˆ
1 - xy A + B = -p + tan -1 Á
fi Ë 1 - xy ˜¯
fi tan (A + B) < 0
fi A + B either lies in the II quadrant or in the Ê x+ yˆ
IV quadrant. fi tan -1 x + tan -1 y = -p + tan -1 Á
Ë 1 - xy ˜¯
fi A + B lies in the II quadrant.
p (vi) tan -1 - tan -1 y
fi < A+ B <p
2 Ï a : x y > -1
p Ôp + a : xy < -1, x > 0, y < 0
fi - p < ( A + B) - p < 0 Ô
2 Ô -p + a : xy < -1, x < 0 , y > 0
Ô
p = Ìp
fi - < ( A + B) - p < 0 Ô : xy = -1, x > 0, y < 0
2
Ô 2
x+ y Ô p
Now, tan ( A + B ) = ÔÓ - 2 : xy = -1, x < 0, y > 0
1 - xy
x+ y Ê x- y ˆ
fi - tan (p - ( A + B )) = where a = tan -1 Á .
1 - xy Ë 1 + x y ˜¯
Proof: Do yourself.
Ê x+ yˆ
fi A + B - p = tan -1 Á
Ë 1 - xy ˜¯ 6.9.1 Some Solved Examples
Ê x+ yˆ Ê 1ˆ Ê 1ˆ
fi A + B = p + tan -1 Á Ex-1. Find the value of tan -1 Á ˜ + tan -1 Á ˜
Ë 1 - xy ˜¯ Ë 2¯ Ë 3¯

Ê x+ yˆ Ê 1ˆ Ê 1ˆ
tan -1 x + tan -1 y = p + tan -1 Á Soln. We have tan -1 Á ˜ + tan -1 Á ˜
fi Ë 2¯ Ë 3¯
Ë 1 - xy ˜¯
Ê 1 1 ˆ
Case V: When x < 0, y < 0 and xy > 1 +
-1 Á ˜
In this case x, y < 0 and xy > 1 = tan Á 2 3 ˜
1 1
x+ y Á1- ¥ ˜
fi >0 Ë 2 3¯
1 - xy
Ê 5/6 ˆ
fi tan (A + B) > 0 = tan -1 Á
fi A + B lies either in the I quadrant or in the Ë 1 - 1 / 6 ˜¯
III quadrant Ê 5 / 6ˆ
fi A + B lies in the third quadrant = tan -1 Á = tan -1 (1) = p / 4
Ë 5 / 6 ˜¯
p
fi -p < A + B < -
2 Ex-2. Find the value of tan -1 (1) + tan -1 ( 2) + tan -1 (3)
p
fi p - p < p + ( A + B) < p - Soln. We have, tan -1 (1) + tan -1 ( 2) + tan -1 (3)
2
p p Ê 2+3 ˆ
fi 0 < p + ( A + B) < = + p + tan -1 Á
2 4 Ë 1 - 2.3 ˜¯
x+ y p
Now, tan ( A + B ) = = + p + tan -1 ( -1)
1 - xy 4
x+ y p p
fi tan (p + ( A + B )) = = +p -
1 - xy 4 4
=p
258 Comprehensive Trigonometry with Challenging Problems & Solutions for Jee Main and Advanced

Ê 5ˆ Ê 3ˆ Ê 1ˆ
Ex-3. Find the value of tan -1 (9) + tan -1 Á ˜ . = tan -1 Á ˜ + tan -1 Á ˜
Ë 4¯ Ë 4¯ Ë 7¯
Ê 5ˆ Ê 3 1 ˆ
Soln. We have, tan -1 (9) + tan -1 Á ˜ +
Ë 4¯ Á ˜
= tan -1 Á 4 7 ˜
Ê 5 ˆ 3 1
9+ Á1- . ˜
-1 Á 4 ˜
Ë 4 7¯
= p + tan Á
5˜ -1 Ê 25 ˆ
Á 1 - 9. ˜ = tan ÁË ˜¯
Ë 4¯ 25
Ê 41 ˆ p
Á ˜ = .
= p + tan -1 Á 4 ˜ 4
41
Á- ˜
Ë 4¯ Ex-6. If sin -1 x + sin -1 y + sin -1 z = p , prove that
= p + tan -1 ( -1) x 1 - x 2 + y 1 - y 2 + z 1 - z 2 = 2 xyz .
p 3p
=p- = . Soln. Let sin -1 x = A, sin -1 y = B, sin -1 z = C
4 4
Ex-4. Find the value of Then x = sin A, y = sin B, z = sin C
Ê 4ˆ Ê 5ˆ Ê 63 ˆ we have, x 1 - x 2 + y 1 - y 2 + z 1 - z 2
sin -1 Á ˜ + sin -1 Á ˜ - sin -1 Á ˜ .
Ë 5¯ Ë 13 ¯ Ë 65 ¯
= sin A.cos A + sin B.cos B + sin C.cos C
4ˆ -1 Ê Ê 5ˆ Ê 63 ˆ
Soln. We have, sin Á ˜ + sin -1 Á ˜ - sin -1 Á ˜ 1
= (sin 2 A + sin 2 B + sin 2C )
Ë 5¯ Ë 13 ¯ Ë 65 ¯
2
Ê 2 2ˆ 1
-1 4
Á -
Ê 5ˆ
+
5
-
Ê 4ˆ = ( 4sin A.sin B.sin C )
= sin . 1 ÁË ˜¯ . 1 ÁË ˜¯ ˜ 2
ÁË 5 13 13 5 ˜¯
-1 Ê 63 ˆ
= 2sin A.sin B.sin C
– sin ÁË ˜¯ = 2xyz.
65
-1 Ê 4 12 5 3ˆ -1 Ê 63 ˆ Ex-7. If cos -1 x + cos -1 y + cos -1 z = p
= sin ÁË . + . ˜¯ - sin ÁË ˜¯
5 13 13 13 65
prove that x 2 + y 2 + z 2 + 2 xyz = 1
-1 Ê 48 15 ˆ -1 Ê 63 ˆ
= sin ÁË + ˜¯ - sin ÁË ˜¯ Soln. We have, cos -1 x + cos -1 y + cos -1 z = p
65 65 65
fi cos -1 x + cos -1 y = p - cos -1 z
-1 Ê 63 ˆ -1 Ê 63 ˆ
= sin Á ˜ - sin Á ˜
Ë 65 ¯ Ë 65 ¯ fi cos -1 x + cos -1 y = cos -1 (- z )
= 0.
Ê 1ˆ Ê 1ˆ p
Ex-5. Prove that 2 tan -1 Á ˜ + tan -1 Á ˜ = .
fi ( )
cos -1 xy - 1 - x 2 1 - y 2 = cos (- z )
Ë 3¯ Ë 7¯ 4
fi ( xy + z )2 = (1 - x 2 ) (1 - y 2 )
Ê 1ˆ Ê 1ˆ
Soln. We have, 2 tan -1 Á ˜ + tan -1 Á ˜ fi x 2 y 2 + 2 xyz + z 2 = 1 - x 2 - y 2 + x 2 y 2
Ë 3¯ Ë 7¯
fi x 2 + y 2 + z 2 + 2 xyz = 1
Ê 1 ˆ
Á 2. ˜ Ê xˆ Ê yˆ
3 ˜ + tan -1 Ê 1 ˆ Ex-8. If cos -1 Á ˜ + cos -1 Á ˜ = q , prove that,
= tan Á
-1 Ë 2¯ Ë 3¯
ÁË ˜¯
Á Ê 1ˆ 2 ˜ 7
Á1- Á ˜ ˜ 9 x 2 + 12 xy cos q + 4 y 2 = 36 sin 2 q
Ë Ë 3¯ ¯
Ê xˆ Ê yˆ
Ê 2ˆ Soln. Given cos -1 Á ˜ + cos -1 Á ˜ = q
Ë 2¯ Ë 3¯
Á ˜ Ê 1ˆ
= tan -1 Á 3 ˜ + tan -1 Á ˜ Êx y
8 Ë 7¯ x2 y2 ˆ
Á ˜
Ë 9¯ fi cos -1 Á . - 1 - 1- ˜ =q
ÁË 2 3 4 9 ˜¯
Inverse Trigonometric Function 259

Case III: when x > 0, 2x . 3x >1


Ê xy x2 y2 ˆ
Á - 1- 1- ˜ = cosq 1
fi ÁË 6 4 9 ˜¯ fi x>
6
2
Ê x2 ˆ Ê y2 ˆ 3p
Ê xy ˆ Then = tan -1 ( 2 x ) + tan -1 (3x )
fi ÁË - cosq ˜¯ = Á1 - ˜ Á1 - ˜ 4
6 Ë 4 ¯Ë 9¯
Ê 5 x ˆ 3p
fi p + tan -1 Á =
fi x 2 y 2 xy
- cos q + cos 2 q Ë 1 - 6 x 2 ˜¯ 4
36 3
Ê 5 x ˆ 3p p
tan -1 Á = -p = -
= 1- x - y + x y fi
2 2 2 2
Ë 1 - 6x ¯
2 ˜
4 4
4 9 36
Ê 5x ˆ
x 2 y 2 xy fi ÁË ˜ = -1
fi + - cos q = 1 - cos 2 q 1 - 6 x2 ¯
4 9 3
fi 6 x2 - 5x - 1 = 0
x 2 y 2 xy
fi + - cos q = sin 2 q fi x = 1, –1/ 6
4 9 3 Thus, x = 1 is a solution.
fi 9 x 2 + 4 y 2 - 12 xy cos q = 36 sin 2 q p
Ex-11. Solve for x: sin -1 ( x ) + sin -1 ( 2 x ) =
tan -1
(1) + tan (2) + tan (3)
-1 -1 3
Ex-9. Let m = p
cot -1
(1) + cot -1 (2) + cot -1 (3) Soln. We have, sin -1 ( x ) + sin -1 ( 2 x ) =
3
(m - 1)2013 . Ê 3ˆ
fi sin -1 x + sin -1 ( 2 x ) = sin -1 Á ˜
Soln. We have, tan -1 (1) + tan -1 ( 2) + tan -1 (3) = p Ë 2 ¯

and cot (1) + cot (2) + cot (3) Ê 3ˆ


-1 -1 -1
fi sin -1 x - sin -1 Á ˜ = - sin -1 (2 x )
Ë 2 ¯
-1 Ê 1 ˆ -1 Ê 1 ˆ
= tan (1) + tan ÁË ˜¯ + tan ÁË ˜¯
-1
2 3 Êx 3 ˆ
fi sin -1 Á - 1 - x 2 ˜ = sin -1 ( -2 x )
p p Ë2 2 ¯
= +
4 4 Êx 3 2ˆ
=
p fi Á 2 - 2 1 - x ˜ = -2 x
Ë ¯
2
p fi 5x = 3 1 - x2
Hence, m = = 2.
p /2
fi (
25 x 2 = 3 1 - x 2 )
3p
Ex-10. Solve for x: tan ( 2 x ) + tan (3x ) =
-1 -1
. fi 28 x 2 = 3
4
Soln. Case I: When x £ 0 3
fi x=±
Then, tan -1
(2 x ) £ 0, tan (3x ) £ 0,
-1 2 7

fi x£0 3 negative value of x does not


fi x= ,
So, it has no solution. 2 7 satisfy the given equation.

Case II: When x > 0, 2 x. 3 x = 6 x 2 < 1 Êx 3 - 3x 2 ˆ


1 Ex-12. Let f ( x ) = cos -1 ( x ) + cos -1 Á + ˜,
fi x< ÁË 2 2 ˜¯
6 1
for £ x £ 1
3p p
Then = tan -1 ( 2 x ) + tan -1 (3x ) < 2
4 2
f (2013) .
So, it is not possible.
Soln. We have f ( x)
260 Comprehensive Trigonometry with Challenging Problems & Solutions for Jee Main and Advanced

Êx Q. Prove that each of the following:


3 - 3x 2 ˆ
= cos ( x ) + cos Á +
-1 -1 Ê 1ˆ Ê 1ˆ Ê 2ˆ
˜ 13. tan -1 Á ˜ + tan -1 Á ˜ = tan -1 Á ˜
ÁË 2 2 ˜¯ Ë 7¯ Ë 13 ¯ Ë 9¯
-1 Ê 1 ˆ Ê 5 ˆ 3p
= cos ( x ) + cos ÁË ˜¯ - cos ( x ) 14. tan -1 (9) + tan -1 Á ˜ =
-1 -1
2 Ë 4¯ 4
p Ê 3ˆ Ê 3ˆ Ê 8ˆ p
= 15. tan -1 Á ˜ + tan -1 Á ˜ - tan -1 Á ˜ =
3 Ë 4¯ Ë 5¯ Ë 19 ¯ 4
p
Now, f (2013) = Ê 3ˆ Ê 1ˆ Ê 1ˆ Ê 1ˆ p
16. tan -1 Á ˜ + tan -1 Á ˜ + tan -1 Á ˜ + tan -1 Á ˜ =
3 Ë 4¯ Ë 5¯ Ë 7¯ Ë 8¯ 4
Ê 1ˆ Ê 1ˆ p
17. 2 tan -1 Á ˜ + tan -1 Á ˜ =
EXERCISE 6 Ë 3¯ Ë 7¯ 4
Ê 1ˆ Ê 1ˆ Ê 1ˆ p
1. sin -1 x + sin -1 (1 - x ) = cos -1 x 18. 2 tan -1 Á ˜ + tan -1 Á ˜ + 2 tan -1 Á ˜ =
Ë 5¯ Ë 7¯ Ë 8¯ 4
2 -1 3/ 2
( -1
2. x - 4 x > sin sin ÈÎp ˘˚ + cos cos ÈÎp ˘˚
3/ 2
) ( ) Ê 3ˆ Ê 8ˆ Ê 77 ˆ
19. sin -1 Á ˜ + sin -1 Á ˜ = sin -1 Á ˜
Ë 5¯ Ë 17 ¯ Ë 85 ¯
( )
3. cos tan -1 x = x
-1 Ê 4 ˆ -1 Ê 12 ˆ -1 Ê 33 ˆ
4. sin ( tan x ) = cos (cot
-1 -1
( x + 1)) 20. cos ÁË ˜¯ + cos ÁË ˜¯ = cos ÁË ˜¯
5 13 65
Ê xˆ -1 Ê 1 ˆ -1 Ê 1 ˆ -1 Ê 13 ˆ
5. sec-1 Á ˜ - sec -1 x = sec -1 2 21. cos ÁË ˜¯ + cos ÁË - ˜¯ + cos ÁË ˜¯ = p
Ë 2¯ 2 7 14
Ê 1ˆ p
Ê -1 Ê Ê -1 Ê 3ˆ ˆ ˆ ˆ 22. sin -1 Á ˜ + cot -1 (3) = .
6. cos Á tan Á cot Á sin Á x + ˜ ˜ ˜ ˜ Ë 5¯ 4
Ë Ë Ë Ë 2¯ ¯ ¯ ¯
Ê 3ˆ Ê 12 ˆ
(
+ tan sec -1 x = 0 ) 23. 2 tan -1 Á ˜ + tan -1 Á ˜ = p .
Ë 2¯ Ë 5¯
7. Find the smallest +ve integer x so that Q. Write the simplest form of each of the following inverse
Ê Ê xˆ Ê 1 ˆˆ Êpˆ trigonometric functions:
tan Á tan -1 Á ˜ + tan -1 Á ˜ ˜ = tan Á ˜
Ë Ë 10 ¯ Ë x + 1¯ ¯ Ë 4¯ 24. cos
-1
1 - x2
8. Find the least integral value of k for which -1
(
25. cos 1 - 2x
2
)
( k - 2) x 2
+ 8 x + k + 4 > sin -1
(sin 12) + cos (cos12)
-1

holds for all x in R.


26. cos -1 (2 x - 1)
2

Ê cos x ˆ
Ê1+ xˆ 27. tan -1 Á , p p
9. If a = 2 tan -1 Á
Ë 1 - x ˜¯
and Ë 1 + sin x ˜¯ - 2 < x < 2

Ê 1 - x2 ˆ Ê cos x ˆ p p
28. tan -1 Á , - <x<
b = sin -1 Á ˜ for 0 < x < 1, Ë 1 - sin x ˜¯ 2 2
Ë 1 + x2 ¯
Ê cos x - sin x ˆ
then prove that a + b = p. 29. tan -1 Á , 0< x<p
Ë cos x + sin x ˜¯
10. Let f ( x ) = sin (sin x ) , " x Π[-p , 2p ] ,
-1
Ê x ˆ
f ' ( x) . -1
30. tan Á ˜ , –a < x < a
Ë a2 - x2 ¯
11. Let f ( x ) = cos (cos x ) , " x Π[-2p , p ] ,
-1
Ê x ˆ
f ¢ ( x) . 31. sin -1 Á ˜
Ë 1+ x ¯
12. Let f ( x ) = tan ( tan x ) ,
-1
Ê a cos x - b sin x ˆ
32. tan -1 Á
È 3p 5p ˘ f ' ( x) . Ë b cos x + a sin x ˜¯
" x Œ Í- , ˙
Î 2 2˚
Inverse Trigonometric Function 261

Ê 1 + x 2 - 1ˆ ÏÔ 2 cos -1 x : 0 £ x £ 1
33. tan Á-1
ÁË x
˜
˜¯
(ii) cos -1
( )
2x - 1 = Ì
2
-1
ÔÓ2pY - 2 cos x : - 1 £ x < 0

Ê 1 - cos x ˆ
34. tan -1 Á ˜, 0< x<p
y=p
Ë 1 + cos x ¯
Ê 3a 2 x - x3 ˆ
35. tan -1 Á 3 ˜ y = p /2
Ë a - 3ax 2 ¯
Ê 1 + sin x + 1 - sin x ˆ
36. cot -1 Á ˜ X¢ X
Ë 1 + sin x - 1 - sin x ¯ -1 O 1

( )
2 2
37. sin -1 x 1 - x - x 1 - x 2 p = -1 p=1

Ê sin x + cos x ˆ p p Ïa : - 1 < x < 1


38. sin -1 Á ˜ ,- < x <
Ë 2 ¯ 4 4 2x ˆ Ô
-1 Ê
(iii) tan Á = Ì -p + a : x > 1
Ë 1 - x 2 ˜¯ Ô
Ê sin x + cos x ˆ p 5p Ó p + a : x < -1
39. cos -1 Á ˜¯ , 4 < x < 4
Ë 2 where a = 2 tan -1 ( x ) .
Ê 1 + x2 + 1 - x2 ˆ Y
40. tan -1 Á ˜
ÁË 1 + x 2 - 1 - x 2 ˜¯
y = p /2
-1 Ê 3 4 ˆ
41. sin Á cos x + sin x˜ .
Ë5 5 ¯
X¢ X
O
6.10 MULTIPLE ANGLES
(
(i) sin -1 2 x 1 - x 2 ) y = -p /2

x = -1 x=1
Ï -1 1 1 Y¢
Ô 2 sin x : - £x£
Ô 2 2 Ï a : -1£ x £ 1
Ô 1 2x ˆ Ô
-1 Ê
= Ì p - 2 sin -1 x : < x £1 (iv) sin Á = Ì p - a :x >1
2 Ë 1 + x 2 ˜¯ Ô
Ô Ó-p - a : x < -1
Ô -1 1
Ô -p - 2 sin x : - 1 £ x < - where a = 2 tan -1 ( x )
Ó 2
Y
Y
y = p /2
y = p /2
X¢ X
O

y = -p /2
y = -p /2

X¢ x = -1 Y¢ x=1
X
x = -1 1 O 1 x=1
x = -1
2
x=
2 Ê 1 - x 2 ˆ ÏÔ2 tan -1 ( x ) : x ≥ 0
(v) cos -1 Á ˜ =Ì
Y¢ Ë 1 + x 2 ¯ ÔÓ -2 tan -1 ( x ) : x £ 0
262 Comprehensive Trigonometry with Challenging Problems & Solutions for Jee Main and Advanced

Y
1 - tan 2 q
=
1 + tan 2 q
y=p
2
Ê 1ˆ
1- Á ˜
Ë 3¯
= 2
X¢ X Ê 1ˆ
O 1+ Á ˜
Ë 3¯

Y¢ 9 -1
=
9 +1
6.10.1 Some Solved Examples 8 4
= =
Ê Ê 1ˆˆ 10 5
Ex-1. Find the value of sin Á 2 sin -1 Á ˜ ˜
Ë Ë 4¯ ¯ Ê1 Ê 3ˆ ˆ
Ex-4. Find the value of sin Á cot -2 Á ˜ ˜
Ê 1ˆ Ë2 Ë 4¯ ¯
Soln. Let sin -1 Á ˜ = q
Ë 4¯ Ê 3ˆ
Soln. Let cot -1 Á ˜ = q
1 Ë 4¯
fi sinq =
4 Êqˆ
Now, sin ( 2q ) Now, sin Á ˜
Ë 2¯
= 2 sin q . cos q 1 - cosq
=
1 1 2
= 2¥ ¥ 1-
4 16 cos q
1-
15 = sin q . cosec q
=
8 2
Ê Ê 1ˆ ˆ cotq
Ex-2. Find the value of cos Á 2 cos -1 Á ˜ ˜ 1-
Ë Ë 3¯ ¯ = cosecq
2
Ê 1ˆ
Soln. Let cos -1 Á ˜ = q
Ë 3¯ cot q
1-
= 1 + cot 2 q
1
fi cosq = 2
3
Now, cos (2q ) =
1
1-
3/ 4
2 1 + 9/16
= 2 cos 2 q - 1
2 1 3
= -1 = 1-
9 2 5
7 2
= - =
9 5
Ê Ê 1ˆ ˆ Ê 3p Ê 3ˆ ˆ
Ex-3. Find the value of cos Á 2 tan -1 Á ˜ ˜ Ex-5. Find the value of tan -1 Á - 2 tan -1 Á ˜ ˜
Ë Ë 3¯ ¯ Ë 4 Ë 4¯ ¯
Ê 1ˆ Ê 3ˆ
Soln. Let tan -1 Á ˜ = q Soln. Let tan -1 Á ˜ = q
Ë 3¯ Ë 4¯
1 3
fi tanq = fi tan q =
3 4
Now, cos (2q ) Ê 3p ˆ
We have, tan Á - 2q ˜
Ë 4 ¯
Inverse Trigonometric Function 263

Ê 3p ˆ 11. Find the integral values of x satisfying the


tan Á ˜ - tan (2q ) inequation x 2 - 3x < sin -1 (sin 2) .
Ë 4¯
=
Ê 3p ˆ 12. Find the value of x satisfying the inequation
1 + tan Á ˜ .tan (2q )
Ë 4¯ 3x 2 + 8 x < 2 sin -1 (sin 4) - cos -1 (cos 4) .
-1 - tan (2q ) 13. For what value of x,
= ÏÔ x ¸Ô
1 - tan ( 2q ) 1 - 3x 2
f ( x ) = cos -1 x + cos -1 Ì + ˝
2 tan q ÔÓ 2 2 Ô˛
-1 -
= 1 - tan 2 q is a constant function.
2 tan q
1-
1 - tan 2 q 6.11 MORE MULTIPLE ANGLES
tan 2 q - 2 tan q - 1
=
1 - tan 2 q - 2 tan q
(
(i) sin -1 3x - 4 x3 )
Ï -1 1 1
Ô 3 sin x :- £x£
9 6
- -1 2 2
16 4 41 Ô
= = Ô 1
9 6 17 = Ì p - 3 sin -1 x : < x £ 1
1- -
16 4 Ô 2
Ô -1 1
Ô-p - 3 sin x : - 1 £ x < - 2
EXERCISE 7 Ó
Y
Q. Prove that each of the following inverse trigonometric
functions:
y = p/2
Ê 1 ˆ
1. sin Á 2 sin -1 ÊÁ ˆ˜ ˜ =
3
Ë Ë 2¯ ¯ 2 X¢ X
O
Ê -1 Ê 1 ˆ ˆ 23
2. sin Á 3 sin Á ˜ ˜ = y = -p/2
Ë Ë 3 ¯ ¯ 27

Ê1 Ê 1ˆ ˆ 3
3. cos Á cos -1 Á ˜ ˜ = Y¢
Ë2 Ë 8¯ ¯ 4

Ê1 Ê 1 ˆˆ 3 5
4. cos Á cos -1 Á - ˜ ˜ =
(
(ii) cos -1 4 x3 - 3x )
Ë2 Ë 10 ¯ ¯ 10
Ï 1
Ô 3 cos -1 x : £ x £1
Ê1 Ê 1ˆ ˆ 2 2
5. sin Á cos -1 Á ˜ ˜ = Ô
Ë2 Ë 9¯ ¯ 3 Ô 1 1
= Ì 2p - 3 cos -1 x : - £ x <
Ô 2 2
Ê1
6. sin Á tan -1
Ë4 ( 63 )ˆ˜¯ = 2 1 2 Ô -1 1
Ô-2p + 3 cos x : - 1 £ x < - 2
Ó
Ê 1 Ê -1 Ê 24 ˆ ˆ ˆ 3
7. cos Á Á tan Á ˜ ˜ ˜ = Y
Ë4Ë Ë ¯
7 ¯¯ 10
Ê1 Ê 2ˆ ˆ 1
8. tan Á cos -1 Á ˜ ˜ = y=p
Ë2 Ë 3¯ ¯ 5
y = p/2
Ê Ê 1ˆ p ˆ 7
9. tan Á 2 tan -1 Á ˜ - ˜ = -
Ë Ë 5¯ 4 ¯ 17
X¢ X
O
Ê 3p 1 -1 Ê 4 ˆ ˆ 1 - 5
10. tan Á - sin Á - ˜ ˜ =
Ë 4 4 Ë 5¯ ¯ 2
x = -1 x = -1/2 Y¢ x = 1/2 x=1
264 Comprehensive Trigonometry with Challenging Problems & Solutions for Jee Main and Advanced

Ê 3 x - x3 ˆ Êp ˆ Êp ˆ
(iii) tan -1 Á = Á + 2 tan -1 ( x )˜ + Á - 2 tan -1 ( x )˜
˜ Ë2 ¯ Ë2 ¯
Ë 1 - 3x 2 ¯
Ï 1 1 =p
-1
Ô 3 tan x : - <x< Ê 1 ˆ
3 3 Hence, the value of f Á =p .
Ô Ë 2014 ˜¯
Ô 1
= Ìp + 3 tan -1 x : - • < x < -
Ê 6x ˆ Ê xˆ
Ô 3 Ex-3. Let f ( x ) = sin -1 Á 2 + 2 tan -1 Á - ˜
Ô 1 Ë x + 9 ˜¯ Ë 3¯
-1
Ô -p + 3 tan x : <x<•
is independent of x x.
Ó 3
Y Soln. We have, f ( x )
-1 Ê 6 x ˆ -1 Ê x ˆ
= sin ÁË 2 ˜¯ + 2 tan ÁË - ˜¯
y = p/2 x +9 3
Ê Ê xˆ ˆ
X¢ X
O Á 2. ÁË ˜¯ ˜
3 ˜ Ê xˆ
= sin Á
-1
- 2 tan -1 Á ˜
x = -p/2 Á Ê xˆ ˜ 2 Ë 3¯
Á1+ Á ˜ ˜
Ë Ë 3¯ ¯

-1 Y¢ Ê xˆ Ê xˆ
x x
1 = 2 tan -1 Á ˜ - 2 tan -1 Á ˜
3 3 Ë 3¯ Ë 3¯
=0
6.11.1 Some Solved Examples x
It will happen when £1
-1 Ê 2 x ˆ 3
Ex-1. Let f ( x ) = sin Á + 2 tan -1 ( x ) , x > 1,
Ë 1 + x 2 ˜¯ fi x £3
f (2013) . fi -3 £ x £ 3
Soln. We have, f ( x) Ex-4. Find the interval of x for which the function
Ê 1 - x2 ˆ
-1 Ê 2 x ˆ f ( x ) = cos -1 Á ˜ + 2 tan ( x )
-1
= sin ÁË 2˜
+ 2 tan -1 ( x )
1+ x ¯ Ë 1 + x2 ¯
is a constant function.
= p - 2 tan ( x ) + 2 tan ( x )
-1 -1

=p Soln. We have, f ( x )
Hence, the value of
-1 Ê 1 - x ˆ
2
f (2013) = p. ˜ + 2 tan ( x )
-1
= cos Á
Ë 1 + x2 ¯
Ê1+ xˆ Ê 1 - x2 ˆ
Ex-2. Let f ( x ) = 2 tan -1 Á = -2 tan ( x ) + 2 tan ( x ) , x £ 0
-1 -1
˜ + sin -1 Á ˜
Ë1- x¯ Ë 1 + x2 ¯
=0
Ê 1 ˆ It is possible only when x £ 0
for 0 £ x < 1 fÁ
Ë 2014 ˜¯ . x Œ(-•, 0] .

Soln. We have, f ( x )
Ex-5. Find the interval of x for which the function f ( x )
-1 Ê 1 +
xˆ Ê 1 - x2 ˆ
= 2 tan Á ˜
Ë1- x¯
+ sin -1 Á ˜
Ë 1 + x2 ¯
( ) (
= 3 cos -1 2 x 2 - 1 + 2 cos -1 4 x3 - 3x )
is independent of x.
p Ê 1 - x2 ˆ
( -1 -1
2
)
= 2 tan (1) + tan ( x ) + - cos -1 Á ˜
Ë 1 + x2 ¯
Soln. We have f (x)

Ê1- x 2ˆ
( ) ( )
= 3 cos -1 2 x 2 - 1 + 2 cos -1 4 x3 - 3x
Êp ˆ p
= 2 Á + tan -1 ( x )˜ + - cos -1 Á
Ë4 ¯ 2 ˜
Ë 1 + x2 ¯ (
= 3 2 cos -1 x ) + 2 (2p - 3 cos x )
-1
Inverse Trigonometric Function 265

Ê 1 1ˆ
(for 0 £ x < 1 ) Á for - £ x £ ˜
EXERCISE 8
Ë 2 2¯
Q. Prove that
1 Ê 1 + x2 + 1 - x2 ˆ Ê p 1
It is possible only when 0 £ x £ ˆ
1. tan -1 Á ˜= + cos -1 x 2 ˜
ÁË 1 + x 2 - 1 - x 2 ˜¯ ÁË 4 2
È 1˘ 2
fi x Œ Í0, ˙ ¯
Î 2˚
Êp 1 aˆ Êp 1 a ˆ 2b
Also, f (x) 2. tan Á + cos -1 ˜ + tan Á - cos -1 ˜ =
Ë4 2 b¯ Ë4 2 b¯ a
( )
= 3 cos -1 2 x 2 - 1 + 2 cos -1 4 x3 - 3x ( ) Ê p-qˆ Ê q-rˆ Ê r- pˆ
3. tan -1 Á + tan -1 Á + tan -1 Á =p
( -1
)
= 3 2p - 2 cos x + 2 -2p + 3 cos x ( -1
) Ë 1 + pq ˜¯ Ë 1 + qr ˜¯ Ë 1 + pr ˜¯
where p > q > 0 and pr < –1 < qr,
1
(for -1 £ x £ 0 ) (for -1 £ x < - ) Ê ab + 1ˆ Ê bc + 1ˆ Ê ca + 1ˆ
2 4. cot -1 Á + cot -1 Á + cot -1 Á =0
= 2p. Ë a - b ˜¯ Ë b - c ˜¯ Ë c - a ˜¯
È 1ˆ Ê 1 -1 Ê 2 x ˆ 1 2 ˆ
It is possible only when x Œ Í-1, - ˜ -1 Ê 1 - y ˆ
Î 2¯ 5. tan Á sin Á + ˜˜
Ë 1 + x 2 ˜¯ 2
cos Á
Ë2 Ë 1 + y2 ¯ ¯
Hence, the value of x is
È 1ˆ È 1˘ Ê x+ yˆ
=Á , xy < 1
Í-1, - 2 ˜¯ » Í0,
Î Î 2 ˙˚
. Ë 1 - xy ˜¯

Ex-6. If tan -1 y : tan -1 x = 4 : 1 , then express y Ê 1- x ˆ Ê ˆ


-1 Ê 1 - y ˆ y-x
6. tan -1 Á - tan = sin -1
Á ˜
as algebraic function of x. Also, prove that Ë 1 + x ˜¯ ÁË 1 + y ˜¯
ËÁ (1 + x )(1 + y ) ˜¯
2 2

Ê 1∞ ˆ
tan Á 22 ˜ is a root of x 4 - 6 x 2 + 1 = 0 .
Ë 2¯ Ê1 ˆ
(
7. tan -1 Á tan 2 A˜ + tan -1 (cot A) + tan -1 cot 3 A = 0
Ë2 ¯ )
-1 -1
Soln. We have, tan y = 4 tan x
-1 Ê a - b Ê q ˆˆ Ê b + a cos q ˆ
-1 Ê
4 x - 4 x3 ˆ 8. 2 tan Á tan Á ˜ ˜ = cos -1 Á
fi tan -1
y = tan Á ˜ Ë a+b Ë ¯
2 ¯ Ë a + b cos q ˜¯
Ë 1 - 6 x2 + x4 ¯

(
4 x 1 - x2 ) ( ) (
9. tan 2 tan -1 a = 2 tan tan -1 a + tan -1 a3 )
fi y=
1 - 6 x2 + x4 Êx 3 - 3x 2 ˆ p 1
Which is a function of x. 10. cos -1 x + cos -1 Á + ˜ = , < x <1
ÁË 2 2 ˜¯ 3 2
p
Let tan -1 x =
8 11. If sin -1 x + sin -1 y + sin -1 z = p , then prove that
Êpˆ
fi x = tan Á ˜ x 1 - x 2 + y 1 - y 2 + z 1 - z 2 = 2 xyz
Ë 8¯
-1 -1 p 12. If cos -1 x + cos -1 y + cos -1 z = p ,
fi tan y = 4 tan x=
2 then prove that, x 2 + y 2 + z 2 + 2 xyz = 1


(
4 x 1 - x2 ) Æ•
Ê xˆ Ê yˆ
13. If cos -1 Á ˜ + cos -1 Á ˜ = q , then prove that,
1 - 6 x2 + x4 Ë 2¯ Ë 3¯

1 - 6 x2 + x4 = 0 9 x 2 - 12 xy cos q + 4 y 2 = 36 sin 2 q .

Êpˆ 3p
fi x = tan Á ˜ is a root of 1 + x 4 = 6 x 2 14. If sin -1 x + sin -1 y + sin -1 z = , then prove that,
Ë 8¯ 2
x 2 + y 2 + z 2 - 2 xyz = 1 .
266 Comprehensive Trigonometry with Challenging Problems & Solutions for Jee Main and Advanced

3p Ê 1 ˆ Ê 1 ˆ Ê 2ˆ
15. If sin -1 x + sin -1 y + sin -1 z = , then prove that 31. tan -1 Á ˜ + tan -1 Á ˜ = tan -1 Á 2 ˜
2 Ë 1 + 2x ¯ Ë 1 + 4x ¯ Ëx ¯
xy + yz + zx = 3.
3p 32. 2 tan -1 ( 2 x + 1) = cos -1 x
16. If sin–1 x + sin–1 y + sin–1 z =
2
9
33. cos -1 x - sin -1 x = cos -1 x 3 ( )
of x + y 2012 + z 2012 -
2012
x 2013 + y 2013 + z 2013 34. If tan -1 y : tan -1 x = 4 : 1 , express y as an
algebraic function of x. Hence, prove that
17. If cos -1 x + cos -1 y + cos -1 z = 3p , then prove that,
xy + yz + zx = 3. Êpˆ
tan Á ˜ is a root of x 4 + 1 = 6 x 2 .
Ë 8¯
18. If cos -1 x + cos -1 y + cos -1 z = 3p
Ê x 2013 + y 2013 + z 2013 + 6 ˆ PROBLEMS FOR JEE MAIN EXAM
of Á 2014 ˜.
Ë x + y 2014 + z 2014 ¯ Ex-1. Find the principal value of sin -1 (sin 10) .
p Soln. We have sin (sin 10)
-1
19. If tan -1 x + tan -1 y + tan -1 z = , then prove that
2
= sin ( sin (3p - 10))
-1
xy + yz + zx = 1.
p
20. If tan -1 x + tan -1 y = , then prove that = (3p - 10)
4
x + y + xy = 1. Ex-2. Find the principal value of cos -1 (cos 5) .
21. If tan -1 x + tan -1 y + tan -1 z = p , then prove that Soln. We have cos -1 (cos 5)
x + y + z = x y z.
= cos (cos ( 2p - 5))
-1

Ê 1 + x2 - 1 - x2 ˆ
22. If tan -1 Á ˜ = a , then prove that = ( 2p - 5) .
ÁË 1 + x 2 + 1 - x 2 ˜¯
Ex-3. Find the value of tan -1 (1) + tan -1 ( 2) + tan -1 (3) .
x 2 = sin 2a .
Soln. We have tan -1 (1) + tan -1 ( 2) + tan -1 (3)
2 –1 2 –1
23. Let m = tan (sec 2) + cot (cosec
Ê 2+3 ˆ
(
value of m 2 + m + 10 . ) = tan -1 (1) + p + tan -1 Á
Ë 1 - 2.3 ˜¯
1 -1 Ê 3 sin 2q ˆ p p Ê 5ˆ
24. If sin Á = q. = + p + tan -1 Á - ˜
2 Ë 5 + 4 cos 2q ˜¯ 4 4 Ë 5¯

25. Let m =
(tan -1
1 + tan -1 2 + tan -1 3 ) , then prove that =
p
4
+ p + tan -1 ( -1)
(cot -1
1 + cot -1 2 + cot -1 3) p
m +1
= + p - tan -1 (1)
( m + 2) = 64 . 4
Q. Solve for x: p p
= +p -
3p 4 4
26. tan -1 ( 2 x ) + tan -1 (3x ) =
4 = p.
Ê x + 1ˆ Ê x - 1ˆ Ex-4. Find x, if sin -1 x > cos -1 x .
27. tan -1 Á + tan -1 Á = tan -1 (-7 )
Ë x - 1˜¯ Ë x ˜¯
Soln. Given sin -1 x > cos -1 x
p sin -1 x + sin -1 x > sin -1 x + cos -1 x
28. sin -1 ( 2 x ) + sin -1 ( x ) = fi
3
p
p fi 2 sin -1 x >
-1 Ê 1 ˆ
29. sin Á ˜ + cos -1 x = 2
Ë 5¯ 4 p
fi sin -1 x >
p
30. sin -1 ( x ) + sin -1 (3x ) = 4
3
Inverse Trigonometric Function 267


Êpˆ
x > sin Á ˜ =
1 Thus, p £ 3 sin -1 x £ 3p
Ë 4¯ 2 2 2
p p
Also, the domain of sin -1 x is [–1, 1]. fi £ sin -1 x £
6 2
Thus, the solution set is x ŒÊ
1 ˘
,1 Êpˆ Êpˆ
Ë 2 ˙˚ sin Á ˜ £ x £ sin Á ˜
fi Ë 6¯ Ë 2¯
Ex-5. Find x, if sin -1 x < cos -1 x
1
Soln. Given sin -1 x < cos -1 x fi £ x £1.
2
fi sin -1 x + sin -1 x < sin -1 x + cos -1 x È1 ˘
p Hence, the solution set is Í ,1˙
fi 2 sin -1 x < Î2 ˚
2
Ê 2x ˆ
p Ex-8. Find x, if 2 tan -1 x = p + tan -1 Á
Ë 1 - x 2 ˜¯
-1
fi sin x <
4
p
Ê ˆ Ê 2x ˆ
1 Soln. Given 2 tan -1 x = p + tan -1 Á
fi x < sin Á ˜ =
Ë 4¯ 2 Ë 1 - x 2 ˜¯

Also, the domain of sin -1 x is [–1, 1] Ê 2x ˆ


Now, the range of p + tan -1 Á
Ë 1 - x 2 ˜¯
È 1 ˆ
Thus, the solution set is x Œ Í-1, ˜ Ê p 3p ˆ
Î 2¯ is Á , ˜
Ë2 2¯
(
Ex-6. Find x, if 2 sin -1 x = sin -1 2 x 1 - x 2 ) Thus, p £ 2 tan -1 x £ 3p

(
Soln. Given 2 sin -1 x = sin -1 2 x 1 - x 2 ) fi
2
p
£ tan -1 x £
2
3p

(
Now, the range of sin -1 2 x 1 - x 2 ) 4
Êpˆ
4
Êpˆ
fi tan Á ˜ £ x < tan Á ˜
È p p˘ Ë 4¯ Ë 2¯
is Í- , ˙
Î 2 2˚
Êpˆ Ê 3p ˆ
p p and tan Á ˜ < x £ tan Á ˜
Thus, - £ 2 sin -1 x £ Ë 2¯ Ë 4¯
2 2
p p fi 1 £ x < • & - • < x £ -1
fi - £ sin -1 x £ Therefore, the solution set is
4 4
Ê pˆ Êpˆ x Œ(-•, - 1] » [1, • ) .
fi sin Á - ˜ £ x £ sin Á ˜
Ë 4¯ Ë 4¯ Êp Ê 1ˆˆ
Ex-9. Find the value of cos Á + cos -1 Á - ˜ ˜
Êpˆ Êpˆ Ë6 Ë 2¯ ¯
fi - sin Á ˜ £ x £ sin Á ˜
Ë 4¯ Ë 4¯ Êp Ê 1ˆˆ
Soln. We have cos Á + cos -1 Á - ˜ ˜
-
1
£x£
1 Ë6 Ë 2¯ ¯

2 2
Êp -1 Ê 1 ˆ ˆ
È 1 1 ˘ = cos Á + p - cos ÁË ˜¯ ˜
Hence, the solution set is Í- , Ë6 2 ¯
˙
Î 2 2˚
Êp pˆ
(
Ex-7. Find x, if 3 sin -1 x = p + sin -1 3x - 4 x3 ) = cos Á + p - ˜
Ë6 3¯

(
Soln. Given 3 sin -1 x = p + sin -1 3x - 4 x3 ) Ê p 2p ˆ
= cos Á + ˜
Ë6 3¯
(
Now, the range of p + sin -1 3x - 4 x3 ) Ê 5p ˆ
= cos Á ˜
È p 3p ˘ Ë 6¯
is Í , ˙
Î2 2 ˚
268 Comprehensive Trigonometry with Challenging Problems & Solutions for Jee Main and Advanced

-1 Ê ( r + 1) - r ˆ
Ê pˆ •
= cos Á p - ˜ = Â tan Á
Ë 6¯ ˜
r =0 Ë 1 + r ( r + 1) ¯
Êpˆ •
= - cos Á ˜
Ë 6¯ (
= Â tan -1 ( r + 1) - tan -1 ( r )
r =0
)
1 n
= -
2 (
= Â tan -1 ( r + 1) - tan -1 ( r ) , n Æ •
r =0
)
( (
Ex-10. Find the value of cos -1 cos 2 cot -1 ( )) )
2 -1 ( ) (
= tan ( 2) - tan (1) + tan (3) - tan ( 2)
-1 -1 -1 -1
)
( (
Soln. We have cos -1 cos 2 cot -1 ( 2 -1 )) ) + ( tan ( 4) - tan (3)) + ( tan (5) - tan ( 4))
-1 -1 -1 -1

Ê +.......... + ( tan ( n + 1) - tan ( n ))


-1 -1
-1
Ê
-1
Ê 2 -1 ˆˆˆ
= cos Á cos Á 2 cos Á ˜˜˜
ÁË ÁË Ë 4 - 2 2 ¯ ˜¯ ˜¯ = ( tan ( n + 1) - tan (1)) , n Æ •
-1 -1

-1 Ê ( n + 1) - 1 ˆ
Ê
-1 Á
Ê
= cos Á cos ÁÁ cos Á
-1 (
Ê 2 2 -1 2 ˆˆˆ
- 1˜ ˜ ˜˜
) = tan Á ˜, nÆ•
Ë 1 + ( n + 1) .1¯
ÁË ÁË Ë (
Á 4-2 2 ˜˜
¯ ˜¯ ˜¯ ) Ê n ˆ
= tan -1 Á , nÆ•
Ë n + 2 ˜¯
Ê Ê
= cos -1 Á cos Á cos -1 Á
(
Ê 2 3 - 2 2 - 4 + 2 2 ˆˆˆ
˜˜˜
) = tan -1 (1), when n Æ •
Á
Ë
Á
Ë Á
Ë 4 - 2 2 ( ˜¯ ˜¯ ˜
¯ ) p
=
Ê Ê Ê ˆˆˆ 4
2-2 2 ˜
= cos Á cos Á cos Á
-1 -1
˜˜ • Ê 2r -1 ˆ
Á
Ë
Á
Ë Á
Ë 4 - 2(2 ˜¯ ˜¯ ˜
¯ ) Ex-12. Find the value of  tan -1 Á
r =1
˜
Ë 1 + 22 r -1 ¯
Ê Ê
= cos Á cos Á cos Á
-1 -1
Ê 2 1- 2 ( ) ˆˆˆ
˜˜˜
• Ê 2r -1 ˆ
Soln. we have  tan -1 Á ˜
Á
Ë
Á
Ë ÁË 2 2 2 - 1 ( ) ˜¯ ˜¯ ˜
¯
r =1 Ë 1 + 22 r -1 ¯
n
-1 Ê 2r -1 ˆ
-1 Ê
Ê -1 Ê 1 ˆ ˆ ˆ = Â tan Á ˜
= cos Á cos Á cos Á - ˜ ˜ Ë 1 + 2r .2r -1 ¯
2 ¯ ˜¯ ¯
r =1
Ë Ë Ë
r r -1 ˆ
-1 Ê 2 - 2
n
-1 Ê Ê -1 Ê 1 ˆ ˆ ˆ = Â tan Á ˜
= cos Á cos Á p - cos Á Ë 1 + 2r .2r -1 ¯
Ë 2 ˜¯ ˜¯ ˜¯
r =1
Ë Ë
( ( ))
n
Ê Ê p ˆˆ
= cos -1 Á cos Á p - ˜ ˜ r =1
( )
= Â tan -1 2r - tan -1 2r -1
Ë Ë 4¯¯
Ê Ê 3p ˆ ˆ
( ) (
= tan ( 2) - tan (1) + tan -1 22 - tan -1 (2)
-1 -1
( ) )
= cos -1 Á cos Á ˜ ˜
Ë Ë 4 ¯¯
( ( )
+ tan -1 23 - tan -1 22 ( )) +........
Ê 3p ˆ
= ÁË ˜¯
4 (
+ tan -1 (2 ) - tan (2 ))
n -1 n -1


-1 Ê
ˆ 1 = tan 2 - tan (1)
-1 n -1
( )
Ex-11. Find the value of  tan ÁË 2˜
r =0

1+ r + r ¯ = tan -1 (2 ) - p4
n

-1 Ê 1 ˆ
Soln. We have  tan ÁË 2˜ Ê r - r - 1ˆ
r =0 1+ r + r ¯ n
-1
Ex-13. Find the value of  sin Á ˜
ÁË r (r + 1) ˜¯
-1 Ê ˆ
• 1 r =1
= Â tan Á ˜
r =0 Ë 1 + r ( r + 1) ¯
Inverse Trigonometric Function 269

n Ê r - r - 1ˆ p Ê yˆ
-1 = - tan -1 Á ˜
Soln. We have  sin Á ˜ 2 Ë x¯
r =1 ÁË r (r + 1) ˜¯
-1 Ê y ˆ
= cot Á ˜
-1 Ê r - r -1 ˆ
n
= Â tan Á Ë x¯
˜
r =1 Ë 1 + r . r - 1¯
-1 Ê x ˆ
= tan Á ˜
( ( r ) - tan ( r - 1))
n
= Â tan -1 -1 Ë y¯
r =1
p2
= ( tan (1) - tan (0)) + ( tan ( 2) - tan (1)) ( ) ( )
-1 -1 -1 -1 2 2
Ex-15. Find x, if tan -1 x + cot -1 x =
8
+ ( tan (3) - tan ( 2)) + ( tan ( 4) - tan (3))
-1 -1 -1 -1
p2
( ) ( )
2 2
Soln. We have tan -1 x + cot -1 x =
+.......+ ( tan ( n ) - tan (n - 1))
-1 -1 8
p2
= tan ( n ) (tan )
-1 -1 2
fi x + cot -1 x - 2 tan -1 x.cot -1 x =
8
Ex-14. Find the value of 2
Êpˆ p 2
Ê a x - yˆ Ê a -a ˆ fi -1 -1
tan -1 Á 1 + tan -1 Á 2 1 ˜ ÁË ˜¯ - 2 tan x.cot x =
˜
Ë a1 y + x ¯ Ë 1 + a1a2 ¯
2 8

Ê a - an-1 ˆ p2 p2
-1 Ê
a - a2 ˆ fi - 2 tan -1 x.cot -1 x =
+ tan Á 3 + .... + tan -1 Á n
˜ Ë 1 + an an-1 ˜¯
4 8
Ë 1 + a3 a2 ¯
p2
-1 Ê
1ˆ + fi 2 tan -1 x.cot -1 x =
+ tan Á ˜ , where x, y, a1 , a2 ,...., an Œ R 8
a
Ë n¯
Ê a -a ˆ p2
- yˆ -1 Ê a1 x fi tan -1 x.cot -1 x =
Soln. We have tan Á ˜ + tan -1 Á 2 1 ˜ 16
Ë a1 y + x ¯ Ë 1 + a1a2 ¯
Êp ˆ p
2
-1 Ê
a - a2 ˆ Ê a - an-1 ˆ fi a Á - a˜ = , where a = tan -1 x
+ tan Á 3 + .... + tan -1 Á n Ë2 ¯ 16
˜
Ë 1 + a3 a2 ¯ Ë 1 + an an-1 ˜¯
Ê 1ˆ Êp ˆ
fi 16a Á - a˜ = p 2
+ tan -1 Á ˜ Ë2 ¯
Ë an ¯
Ê y ˆ fi 16a 2 - 8ap + p 2 = 0
a1 - Ê a -a ˆ
-1 Á x ˜ ( 4 a - p )2 = 0
= tan Á + tan -1 Á 2 1 ˜ fi
y˜ Ë 1 + a1a2 ¯
Á 1 + a1. ˜
Ë x ¯ fi ( 4a - p ) = 0
Ê a - a2 ˆ Ê a - an -1 ˆ p
+ tan -1 Á 3 + .... + tan -1 Á n fi a=
˜
Ë 1 + a3 a2 ¯ Ë 1 + an an -1 ˜¯ 4
Ê 1ˆ p
+ tan -1 Á ˜ fi tan -1 x =
Ë an ¯ 4
Ê yˆ fi x = 1.
= tan -1 ( a1 ) - tan -1 Á ˜ + tan -1 ( a2 ) - tan -1 ( a1 ) + Hence, the solution is x = 1.
Ë x¯
Ex-16. Find the maximum value of f (x), if
tan -1 ( a3 ) - tan -1 ( a2 ) + ... + tan -1 ( an ) - tan -1 ( an-1 )
( ) + (cosec x)
2 2
f ( x ) = sec-1 x -1

+ cot
-1
(an )
Given f ( x ) = (sec x ) + (cosec x )
-1 2 -1 2
Soln.
-1 Ê y ˆ
= tan ( an ) + cot ( an ) - tan Á ˜
-1 -1
Ë x¯
= (sec x + cosec x ) - 2 sec x . cosec
-1 -1 2 -1 -1
x
270 Comprehensive Trigonometry with Challenging Problems & Solutions for Jee Main and Advanced

2 2
Êpˆ Êp ˆ 3p Ê pˆ p3 p3
= Á ˜ - 2 sec-1 x Á - sec-1 x˜ = ÁË a - ˜¯ + ≥
Ë 2¯ Ë2 ¯ 2 4 32 32
p2 p3
( ) Hence, the minimum value of f (x) is .
2
= - p .sec-1 x + 2 sec-1 x 32
4
Ê p p2ˆ Ex-18. Find x, if ÈÎcot -1 x ˘˚ + ÎÈcos -1 x ˘˚ = 0
( -1 2
)
= 2 Á sec x - .sec x + ˜
Ë 2
-1
8¯ Soln. Given Ècot -1 x ˘ + Ècos -1 x ˘ = 0
Î ˚ Î ˚
Ê p Êpˆ p2 p2ˆ
2
( ) It is possible only when
2
= 2 Á sec-1 x - 2.sec -1 x. +Á ˜ + - ˜
Ë 4 Ë 4¯ 8 16 ¯ fi cot -1 x = 0 & cos -1 x = 0
0 £ cot -1 x < 1 & 0 £ cos -1 x < 1
2
Ê pˆ p2 fi
= 2 Á sec-1 x - ˜ +
Ë 4¯ 8 fi x Œ(cot 1, • ) & x Œ(cos1,1]
p Thus, the solution is x Œ(cot 1,1]
2
Max value of f (x) is at x = 1.
4
Ex-19. Find x, if Èsin -1 x ˘ + Ècos -1 x ˘ = 0
Ex-17. Find the minimum value of f (x), if Î ˚ Î ˚

( ) ( ) Soln. Given ÈÎsin -1 x ˘˚ + ÎÈcos -1 x ˘˚ = 0


3 3
f ( x ) = sin -1 x + cos -1 x
It is possible only when
Given f ( x ) = (sin x ) + (cos x ) -1 3 -1 3
Soln.
fi Èsin -1 x ˘ = 0 & Ècos -1 x ˘ = 0
Î ˚ Î ˚
= (sin x + cos x )
-1 -1
fi 0 £ sin -1 x < 1 & 0 £ cos -1 x < 1

Ë( ) ( )
Ê sin x + cos x - sin x cos xˆ x Œ[0, sin 1) & x Œ(cos1, 1]
-1 2 -1 2 -1 -1 fi
¯
Thus, the solution is x Œ(cos1, sin 1) .
= (sin x + cos x )
-1 -1

Ex-20. Find x, if È tan -1 x ˘ + Ècot -1 x ˘ = 2 .


Ë( )
Ê sin x + cos x - 3 sin x cos xˆ
-1 -1 2 -1 -1 Î ˚ Î ˚
¯
Soln. Given ÈÎ tan -1 x ˘˚ + ÎÈcot -1 x ˘˚ = 2
p ÊÊ p ˆ ˆ
2
-1 Ê p -1 ˆ
= Á ÁË ˜¯ - 3 sin x ÁË - sin x˜¯ ˜ The range of ÈÎ tan -1 x ˘˚ is {–2, –1, 0, 1}
2Ë 2 2 ¯
and Ècot -1 x ˘ is {0, 1, 2, 3 }
p ÊÊ p ˆ ˆˆ Î ˚
2
Êp
= Á Á ˜ - 3 a Á - a˜ ˜
2 ËË 2¯ Ë2 ¯¯ Case I: when ÈÎcot -1 x ˘˚ = 1 & ÈÎ tan -1 x ˘˚ = 1
p Ê p 2 3ap ˆ fi 1 £ cot -1 x < 2 & 1 £ tan -1 x < 2
= Á - + 3a 2 ˜
2Ë 4 2 ¯ fi x Œ(cot 2, cot 1] & x Œ[ tan 1, tan 2)
p Ê Ê 2 ap ˆ p 2ˆ
fi x Œj (∵ cot 1< tan 1)
= 3Á a - ˜+
2 ÁË Ë 2 ¯ 4 ˜¯ Case II: when Ècot -1 x ˘ = 3 & È tan -1 x ˘ = -1
Î ˚ Î ˚
pÊ Ê 2 Êpˆ ˆ p 3p 2 ˆ
2
p 2
3 £ cot -1 x < 4 & - 1 £ tan -1 x < 0
= Á 3 Á a - 2. .a + Á ˜ ˜ + - ˜ fi
2Ë Ë 4 Ë 6¯ ¯ 4 16 ¯
fi x Œ(cot 4, cot 3] & x Œ[- tan 1, 0)
pÊ Ê p 2 3p 2 ˆ
2
pˆ x Œj , (∵cot 3 < - tan 1)
= Á Á
3 a - ˜ + - ˜ fi
2Ë Ë 4¯ 4 16 ¯
Case III: when Ècot -1 x ˘ = 2 & È tan -1 x ˘ = 0
Î ˚ Î ˚
pÊ Ê p2ˆ
2

= Á 3Á a - ˜ + ˜ fi 2 £ cot -1 x < 3 & 0 £ tan -1 x < 1
2Ë Ë 4¯ 16 ¯
fi x Œ(3, cot 2] & x Œ[0, tan 1)
Ê 3p Ê pˆ
2
p3 ˆ
= Á Áa - ˜ + ˜ fi x Œj, (∵ cot 2 < tan 1)
Ë 2 Ë 4¯ 32 ¯
Thus, no such value of x, where the equation is valid.
Inverse Trigonometric Function 271

( (
Ex-21. Find x, if Èsin -1 cos -1 sin -1 tan -1 x
ÎÍ
( )))˚˙˘ = 1 Now, f (1) = sin -1 (1) + sec-1 (1) + tan -1 (1)
p p 3p
( (
Soln. Given Èsin -1 cos -1 sin -1 tan -1 x
ÎÍ
( ))) ˘ =1
˚˙
=
2
+0+ =
4 4
and f (-1) = sin -1 ( -1) + sec-1 ( -1) + tan -1 ( -1)
fi ( (
0 £ sin -1 cos -1 sin -1 tan -1 x ( ))) <1
= -
p
+p -0-
p

fi ( (
0 £ cos -1 sin -1 tan -1 x ))) < sin1
( p
2 4

=
fi cos (sin 1) < (sin ( tan x )) £ 1
-1 -1 4
Ï p 3p ¸
Thus, the range of f (x) is Ì , ˝ .
fi sin (cos (sin 1)) < ( tan x ) £ sin 1
-1 Ó4 4 ˛
p
Ex-25. If tan -1 ( 2 x ) + tan -1 (3x ) =
fi ( )
tan sin (cos (sin 1)) < x £ tan (sin 1) 4
x.

fi ( ( )
x Œ tan sin (cos (sin 1)) , tan (sin 1) ˘˚ Soln Given tan -1 ( 2 x ) + tan -1 (3x ) =
p
4
Ex-22. Find the range of
Ê 2 x + 3x ˆ p
f ( x ) = sin -1 x + tan -1 x + cot -1 x fi tan -1 Á =
Ë 1 - 2 x.3x ˜¯ 4
Soln. Given f ( x ) = sin x + tan x + cot x
-1 -1 -1
Ê 5x ˆ
-1 £ x £ 1 fi ÁË ˜ =1
1 - 6 x2 ¯
Thus, f ( -1) fi 6 x2 + 5x - 1 = 0
= sin -1 ( -1) + tan -1 ( -1) + cot -1 ( -1) fi 6 x2 + 6 x - x - 1 = 0
p p p fi (6 x - 1) ( x + 1) = 0
= - - +p -
2 4 4 1
fi x = -1,
= -p + p = 0 6
and f (1) Ï 1¸
Hence, the solution set is Ì-1, ˝
Ó 6˛
= sin (1) + tan (1) + cot (1)
-1 -1 -1

Ê 4ˆ Ê 1ˆ
p p p Ex-26 If cos -1 x = cot -1 Á ˜ + tan -1 Á ˜ x.
= + + Ë 3¯ Ë 7¯
2 4 4
= p. Ê 4ˆ Ê 1ˆ
Soln. Given cos -1 x = cot -1 Á ˜ + tan -1 Á ˜
Thus, range of f (x) is = ÈÎ f ( -1) , f (1)˘˚ = [0, p ] Ë 3¯ Ë 7¯

Ex-23. Find the range of Ê 3ˆ Ê 1ˆ


fi cos -1 x = tan -1 Á ˜ + tan -1 Á ˜
f ( x ) = sin -1 x + cos -1 x + tan -1 x Ë 4¯ Ë 7¯

Soln. Given f ( x ) = sin -1 x + cos -1 x + tan -1 x Ê 3 1 ˆ


+
˜ -1 Á
p fi
-1
cos x = tan Á 4 7 ˜
= + tan -1 x 3 1
Á1- . ˜
2 Ë 4 7¯
As we know that , the range of
Ê 25 ˆ
p p cos -1 x = tan -1 Á ˜ = tan -1 (1)
tan -1 x is ÊÁ - , ˆ˜ fi Ë 25 ¯
Ë 2 2¯
Thus, the range of f (x) is (0, p) p
fi cos -1 x =
Ex-24. Find the range of 4
f ( x ) = sin -1 x + sec-1 x + tan -1 x 1
fi x=
2
Soln. Given f ( x ) = sin x + sec x + tan x
-1 -1 -1
1
Hence, the solution is x =
The domain of f (x) is {-1, 1} 2
272 Comprehensive Trigonometry with Challenging Problems & Solutions for Jee Main and Advanced

Ê nˆ p Ê 3p p ˆ Ê 3p p ˆ
Ex-27. If cot -1 Á ˜ > , where n Œ N = sin ÁË + ˜ + cos Á +
Ëp¯ 6 4 4¯ Ë 4 4 ˜¯
the maximum value of n.
= sin p + cos p
Ê nˆ p = 0 – 1 = –1
Soln. Given cot -1 Á ˜ >
Ëp¯ 6 Ex-30. Find the number of integral values of k for which the
n Êpˆ equation sin -1 x + tan -1 x = 2k + 1 has a solution.
fi < cot Á ˜
p Ë 6¯
Soln. Given sin -1 x + tan -1 x = 2k + 1
fi n
< 3
Let g ( x ) = sin x + tan x
-1 -1
p
fi n <p 3 Domain of g = [–1, 1]
Now, g ( -1) = sin ( -1) + tan ( -1)
-1 -1
fi n < p 3 = 3.14 ¥ 1.732 = 5.43848
Hence, the max value of n is 5 p p 3p
= - - =-
Ê 5ˆ Ê 12 ˆ p 2 4 4
Ex-28. If sin -1 Á ˜ + sin -1 Á ˜ = x.
Ë x¯ Ë x¯ 2 g (1) = sin -1 (1) + tan -1 (1)
Ê 5ˆ Ê 12 ˆ p p p 3p
Soln. Given sin -1 Á ˜ + sin -1 Á ˜ = = + =
Ë x¯ Ë x¯ 2 2 4 4
Ê 5ˆ p Ê 12 ˆ È 3p 3p ˘
sin -1 Á ˜ = - sin -1 Á ˜ Range of g = Í- ,
Ë x¯ 2 Ë x¯ ˙
Î 4 4˚
Ê 5ˆ Ê 12 ˆ 3p 3p
fi sin -1 Á ˜ = cos -1 Á ˜ Thus, - £ 2k + 1 £
Ë x¯ Ë x¯ 4 4
2
fi 3p 3p
Ê 5ˆ Ê 12 ˆ - -1£ 2k £ -1
fi sin -1 Á ˜ = sin -1 1 - Á ˜ 4 4
Ë x¯ Ë x¯
fi 3 ¥ 3.14 3 ¥ 3.14
2 2 - -1£ 2k £ -1
Ê 5ˆ Ê 12 ˆ 4 4
fi ÁË ˜¯ = 1 - ÁË ˜¯
x x fi -2.35 - 1 £ 2 k £ 2.35 - 1
Ê 169 ˆ fi -3.35 £ 2 k £ 1.35
fi ÁË 2 ˜¯ = 1
x 3.35 1.35
fi - £k £
fi x = 169
2 2 2
fi x = ±13 fi -1.67 £ k £ 0.67
Thus, the integral values of k are –1 and 0.
( ) ( )
Ex-29. If x = sin -1 a 6 + 1 + cos -1 a 4 + 1 + tan -1 a 2 + 1 ( ) Ex-31. If sin -1 x + sin -1 y =
p
Ê pˆ Ê pˆ 2
sin Á x + ˜ + cos Á x + ˜
Ë 4¯ Ë 4¯ Ê 1 + x4 + y 4 ˆ
of Á 2 ˜
Soln. We have Ë x - x2 y 2 + y 2 ¯
( ) ( )
x = sin -1 b6 + 1 + cos -1 b 4 + 1 + tan -1 a 2 + 1 ( ) Soln. Given sin -1 x + sin -1 y =
p
It is possible only when a = 0 2
Thus, x = sin -1 (1) + cos -1 (1) + tan -1 (1) fi sin -1 x =
p
- sin -1 y
2
p p 3p
= +0+ = sin -1 x = cos -1 y
2 4 4 fi
Ê pˆ Ê pˆ
Therefore, sin Á x + ˜ + cos Á x + ˜ fi sin -1 x = sin -1 1 - y 2
Ë 4 ¯ Ë 4¯
Inverse Trigonometric Function 273

x = 1 - y2 3p
fi = + 10
4
fi x2 = 1 - y 2
and f (-1) = sin -1 ( -1) + tan -1 ( -1) + 1 - 4 + 5
fi x + y =1
2 2
3p
Ê 1 + x4 + y 4 ˆ = -+2
Now, 4
Á 2 ˜
Ë x - x2 y 2 + y 2 ¯ Therefore, a + b + 5 = 10 + 2 + 5 = 17


(
Ê 1 + x2 + y 2 2 - 2 x2 y 2 ˆ
˜ ) QUESTIONS WITH SOLUTIONS OF PAST JEE
Á
Ë
2
(
x +y -x y
2 2 2 ˜
¯ ) MAIN EXAMS

Ê
1 + 1 - 2 x2 y 2
ˆ 1. If cot -1 ( )
cos a + tan -1 ( )
cos a = x , then sin x is
=Á ˜
(
ÁË 1 - x 2 y 2 ˜¯ ) (a) 1 (b) cot (a / 2)
2

=
( )
2 1 - x2 y 2 (c) tan a
Êaˆ
(d) cot Á ˜
Ë 2¯
(1 - x y ) 2 2
[JEE Main – 2002]
= 2. p
Soln. Clearly, x =
2
Ex-32. If cos -1 x + cos -1 ( 2 x ) + cos -1 (3x ) = p Thus, sin x = 1
Ans. (a)
and x ax3 + bx 2 + cx = 1
Ê Ê xˆˆ
a 2 + b 2 + c 2 + 10 2. The domain of sin -1 Á log3 Á ˜ ˜ is
Ë Ë 3¯ ¯
Soln. Given cos -1 x + cos -1 ( 2 x ) + cos -1 (3x ) = p (a) [1, 9] (b) [–1, 9]
(c) [–9, 1] (d) [–9, –1].
fi cos -1 2 x + cos -1 3x = p - cos -1 x
[JEE Main - 2002]
fi cos -1 2 x + cos -1 3x = cos -1 (- x )
Soln. As we know that, domian of sin -1 x is [–1, 1]
fi cos -1
(2x.3x - 1 - 4x 2
1 - 9x 2
) = cos -1
(- x ) Ê xˆ
Therefore, -1 £ log3 Á ˜ £ 1
fi (6 x - 2
1 - 4 x2 1 - 9 x2 = - x ) Ë 3¯
Ê xˆ
3-1 £ Á ˜ £ 3

(6 x ) ( 1 - 4x ) Ë 3¯
2 2
fi 2
+x = 2
1 - 9 x2
1 Ê xˆ
fi fi £Á ˜ £3
36 x + 12 x + x = 1 - 4 x - 9 x + 36 x
4 3 2 2 2 4
3 Ë 3¯
fi 12 x3 + 14 x 2 = 1 fi 1£ x £ 9
Also, ax + bx + cx = 1
3 2
Thus the domain of f (x) is [1, 9].
Thus, a = 12, b = 14 and c = 0. Ans. (a)
Hence, the value of a + b + c + 10 2 2 2 3. The trigonometric equation sin -1 x = 2 sin -1 a has a
solution for
= 144 + 196 + 10
1
= 350. (a) all real values (b) a <
2
Ex-33. If f ( x ) = sin -1 x + tan -1 x + x 2 + 4 x + 5 such that 1 1 1
(c) a £ (d) < a < .
R f = [ a, b ] 2 2 2
a + b + 5.
[JEE Main - 2003]
Soln. The domain of sin -1 x + tan -1 x is [–1, 1]
Soln. As we know that, the range of sin -1 x
Now, f (1) = sin -1
(1) + tan (1) + 1 + 4 + 5
-1
È p p˘
is Í- , ˙
Î 2 2˚
274 Comprehensive Trigonometry with Challenging Problems & Solutions for Jee Main and Advanced

Therefore, - p £ 2 sin -1 a £ p Ê yˆ
6. If cos -1 x - cos -1 Á ˜ = a , then
2 2 Ë 2¯
p p 4 x 2 - 4 xy cosa + y 2 is
fi - £ sin -1 a £
4 4
(a) 4 (b) 2 sin 2a
1 1
fi - £a£ (c) -4 sin 2 a (d) 4 sin 2 a
2 2
1 [JEE Main - 2005]
fi a £
2 Ê yˆ
Soln. Given cos -1 x - cos -1 Á ˜ = a
Ans. (c) Ë 2¯
sin -1 ( x - 3) Ê y
4. The domain of the function f ( x ) = is y2 ˆ
9- x 2 fi cos -1 Á x. + 1 - x 2 1 - ˜ =a
ÁË 2 4 ˜¯
(a) [1, 2] (b) [2, 3)
(c) [2, 3] (d) [1, 2) Êxy y2 ˆ
[JEE Main - 2004] fi Á + 1 - x2 1 - ˜ = cosa
ÁË 2 4 ˜¯
Soln. Now, sin -1 ( x - 3) 2
Ê xy ˆ
2 Ê y2 ˆ
-1 £ ( x - 3) £ 1 fi 2 £ x £ 4 - ˜¯ = ÁÁ 1 - x 1 -
2
fi ÁË cosa ˜
2 Ë 4 ˜¯
1
Also, the function Ê Ê xy ˆ ˆ
2
9 - x2 fi Á cos 2
a - x y cos a + ÁË ˜¯ ˜
Ë 2 ¯
fi 9- x >0
2

fi y 2 x2 y 2
fi x2 - 9 < 0 1 - x2 - +
4 4
fi ( x + 3) ( x - 3) < 0
fi y2
fi -3 < x < 3 x 2 - xy cos a + = 1 - cos 2 a
4
Thus, the solution is x Œ[ 2, 3)
fi y2
Hence, the domain is [ 2, 3) x 2 - xy cos a + = sin 2 a
4
Ans. (b) fi 4 x 2 - 4 xy cos a + y 2 = 4 sin 2 a
5. Let f : ( -1,1) Æ B Ans. (d)
Ê 2x ˆ 7. No questions asked in 2006.
f ( x ) = tan -1 Á , then f is both one one
Ë 1 - x 2 ˜¯ Ê xˆ Ê 5ˆ p
8. If sin -1 Á ˜ + cosec-1 Á ˜ = , then x is
and onto, when B lies in Ë 5¯ Ë 4¯ 2
È pˆ Ê pˆ (a) 4 (b) 5 (c) 1 (d) 3
(a) Í0, ˜ (b) Á 0, ˜
Î 2 ¯ Ë 2¯ [JEE Main - 2007]
Ê p pˆ È p p˘ Ê xˆ Ê 5ˆ p
(c) Á - , ˜ (d) Í- , ˙ Soln. Given sin -1 Á ˜ + cosec-1 Á ˜ =
Ë 2 2¯ Î 2 2˚ Ë 5¯ Ë 4¯ 2
[JEE Main - 2005] Ê xˆ Ê 4ˆ p
fi sin -1 Á ˜ + sin -1 Á ˜ =
Soln. Since the f is ont, so the range of f is co-domain. Ë 5¯ Ë 5¯ 2
i.e., Range = B Ê xˆ Ê 3ˆ p
fi sin -1 Á ˜ + cos -1 Á ˜ =
Ê p pˆ Ë 5¯ Ë 5¯ 2
Clearly, range of f is Á - , ˜
Ë 2 2¯ fi x=3
Ê p pˆ Ans. (d)
Thus, B = Á - , ˜ 9. Find the value of
Ë 2 2¯
Ê Ê 5ˆ Ê 2ˆˆ
Ans. (c) cot Á cosec -1 Á ˜ + tan -1 Á ˜ ˜ is
Ë Ë 3 ¯ Ë 3¯¯
Inverse Trigonometric Function 275

5 6 Ex-2 Find the domain of


(a) (b)
( )
2
17 17 f ( x ) = 5p sin -1 x - 6 sin -1 x
3 4
(c) (d) Soln. The function f
17 17
( )
2
fi 5p sin -1 x - 6 sin -1 x ≥0
[JEE Main - 2008]

( )
2
Ê Ê 5ˆ Ê 2ˆ ˆ fi 6 sin -1 x - 5p sin -1 x £ 0
Soln. Given cot Á cosec-1 Á ˜ + tan -1 Á ˜ ˜
Ë Ë 3¯ Ë 3¯ ¯
Ê Ê 3ˆ Ê 2ˆ ˆ
fi (sin x) (6 (sin x) - 5p ) £ 0
-1 -1

= cot Á sin -1 Á ˜ + tan -1 Á ˜ ˜


Ë Ë 5¯ Ë 3¯ ¯ 5p
fi 0 £ sin -1 x £
6
Ê Ê 3ˆ Ê 2ˆ ˆ
= cot Á tan -1 Á ˜ + tan -1 Á ˜ ˜ 1
Ë Ë 4¯ Ë 3¯ ¯ fi 0£ x£
2
Ê Ê 3 2 ˆˆ
Also, sin -1 x
Á -1 Á 4 + 3 ˜ ˜
= cot Á tan Á
3 2 ˜˜ È 1˘
Thus, D f = Í0, ˙ .
Á Á1- . ˜˜
Ë Ë 4 3 ¯¯ Î 2˚
Ex-3 Find the domain of
Ê -1 Ê 17 ˆ ˆ
= cot Á tan Á ˜ ˜
Ë Ë 6 ¯¯ ( (
f ( x ) = sin -1 log 2 x 2 + 3x + 4 ))
Ê -1 Ê 6 ˆ ˆ Soln. f
= cot Á cot Á ˜ ˜
Ë Ë 17 ¯ ¯
(
-1 £ log 2 x 2 + 3x + 4 £ 1 )
6
=
17 fi (
2-1 £ x 2 + 3x + 4 £ 2 )
Ans. (b).
10. No questions asked in 2009 to 2014.
when (x 2
)
+ 3x + 4 £ 2

fi (x 2
+ 3x + 2) £ 0
PROBLEMS FOR JEE ADVANCED EXAM
fi ( x + 1) ( x + 2) £ 0
Ex-1. Find the domain of fi -2 £ x £ -1
Ê x - 2ˆ Ê1- x ˆ 1
f ( x ) = sin -1 Á ˜ + cos -1 Á when x 2 + 3x + 4 ≥
Ë 3 ¯ Ë 4 ˜¯ 2
Ê x - 2ˆ fi 2 x2 + 6 x + 7 ≥ 0
Soln. Let D1: -1 £ Á £1
Ë 3 ˜¯ fi x ŒR
Thus, D f = [-2, - 1] .
fi -3 £ ( x - 2) £ 3
-1 £ x £ 5 Ex-4 x: cos -1 x + cos -1 x 2 = 2p

-5 £ x £ 5 Soln. Given cos -1 x + cos -1 x 2 = 2p

It is possible only when
Ê1- x ˆ
and D2: -1 £ Á £1 cos -1 x = p & cos -1 x 2 = p
Ë 4 ˜¯
x = cosp = -1 and x 2 = -1
fi -4 £ 1 - x £ 4
Thus, no such value of x is exist.
fi -5 £ - x £ 3 Ex-5 x:
p
fi -3 £ x £ 5
-5 £ x £ 5
Ê 1 ˆ
cot -1 Á 2 ˜ + tan -1 x 2 - 1 =
Ë x - 1¯ 2
( )

1
Thus, D f = D1 « D2 = [ -5, 5] Soln. = x2 - 1
x -1
2
276 Comprehensive Trigonometry with Challenging Problems & Solutions for Jee Main and Advanced

( x - 1) = 1 Ê 2 x2 + 4 ˆ
2
fi 2
fi Á p - ˜ <p -3
Ë x2 + 1 ¯
fi ( x - 1) = ±1
2

2 x2 + 4
fi >3
fi x = 1 ± 1 = 2, 0
2
x2 + 1
fi x = 0, ± 2 2 x2 + 4
fi -3>0
Ex-6. x: x2 + 1
Ê x - 1ˆ
2
Ê 2 x ˆ 2p 2 x 2 + 4 - 3x 2 - 3
cot -1 Á ˜ + tan -1 Á 2 ˜ = fi >0
Ë 2x ¯ Ë x - 1¯ 3 x2 + 1
- x2 + 1
-1 Ê
x 2 - 1ˆ -1 Ê 2 x ˆ 2p fi >0
Soln. Given cot Á ˜ + tan ÁË 2 ˜¯ = 3 x2 + 1
Ë 2 x ¯ x -1
x2 - 1
Ê 2x ˆ Ê 2 x ˆ 2p fi <0
fi tan -1 Á + tan -1 Á = x2 + 1
Ë1- x ¯
2 ˜ Ë 1 - x 2 ˜¯ 3

( x - 1) ( x + 1)
Ê 2 x ˆ 2p <0
fi 2 tan -1 Á = x2 + 1
Ë 1 - x 2 ˜¯ 3
fi -1 < x < 1
Ê 2x ˆ p x:
fi tan -1 Á = Ex-8.
Ë 1 - x 2 ˜¯ 3
Ê 2x ˆ Êpˆ
( ) (
x 2 - 4 x > sin -1 sin ÈÎp 3/ 2 ˘˚ + cos -1 cos ÈÎp 3/ 2 ˘˚ )
fi ÁË 2˜
1- x ¯
= tan Á ˜ = 3
Ë 3¯ ( ) (
Soln. we have sin -1 sin ÈÎp 3/ 2 ˘˚ + cos -1 cos ÈÎp 3/ 2 ˘˚ )
2x

3
= 1 - x2 ( )
= sin -1 sin ÈÎp p ˘˚ + cos -1 cos ÈÎp p ˘˚( )
= sin (sin [5.56]) + cos (cos [5.56])
2 -1 -1
fi x2 + x -1 = 0
= sin (sin 5) + cos (cos 5)
3 -1 -1
2
Ê 1 ˆ
= sin (sin (5 - 2p )) + cos (cos ( 2p - 5))
1 4 -1 -1
fi ÁË x + ˜¯ = 1 + 3 = 3
3
= (5 - 2p ) + ( 2p - 5) = 0
Ê 1 ˆ 2
fi ÁË x + ˜¯ = ±
3 3
x2 - 4 x < 0
1 2
fi x=- ± x ( x - 4) < 0
3 3
1 0< x<4
fi x= ,- 3
Ê Ê Ê 3 ˆˆˆ
x: cos Á tan -1 Á cot Á sin -1 ÊÁ x + ˆ˜ ˜ ˜ ˜
3
Ex-9.
Ï 1 ¸ Ë Ë Ë Ë 2¯ ¯ ¯ ¯
Hence, the solution set is Ì- 3, ˝
Ó 3˛
(
+ tan sec -1 x = 0 )
Ê Ê 2x + 4ˆ ˆ 2
Ex-7. x: sin -1 Á sin Á ˜˜ < p - 3 Ê Ê Ê Ê 3ˆ ˆ ˆ ˆ
Ë Ë x +1 ¯¯
2
Soln. Now, cos Á tan -1 Á cot Á sin -1 Á x + ˜ ˜ ˜ ˜
Ë Ë Ë Ë 2¯ ¯ ¯ ¯
Ê Ê 2 x2 + 4 ˆ ˆ Ê Ê Ê 2 ˆˆˆ
Soln. Given sin -1 Á sin Á 2 ˜˜ < p - 3 Á Á Á
Ê
Ê 3ˆ ˆ ˜ ˜
Ë Ë x +1 ¯¯ Á 1- Á x + ˜ ˜˜
Á Á Á Ë 2¯ ˜ ˜ ˜ ˜
= cos Á tan -1 Á cot Á cot -1 ÁÁ
Ê Ê 2 x2 + 4 ˆ ˆ Ê 3ˆ ˜ ˜ ˜ ˜
fi sin -1 Á sin Á p - 2 ˜˜ < p - 3
Á Á Á Á Á x + ˜ ˜˜˜˜
Ë Ë x +1 ¯¯ Á Á ÁË ÁË Ë 2¯ ˜ ˜ ˜ ˜
Ë Ë ¯¯¯¯
Inverse Trigonometric Function 277

Ê 2 ˆ Ê x 1 ˆ
Ê
Ê 3ˆ ˆ +
Á Á 1- Á x + ˜ ˜˜ Á 10 x + 1 ˜
Á -1 Á Ë 2¯ ˜ ˜ fi =1
= cos Á tan Á
Á x 1 ˜
Ê 3ˆ ˜ ˜ Á1- . ˜
Ë 10 x + 1 ¯
Á Á Á x + ˜ ˜˜
ÁË ÁË Ë 2¯ ˜ ˜
¯¯ Ê x 1 ˆ Ê x 1 ˆ
fi ÁË + ˜¯ = ÁË1 - . ˜
Ê Ê 3 ˆˆ 10 x + 1 10 x + 1¯
Á -1 Á x + 2 ˜ ˜ fi x ( x + 1) + 10 = 10 ( x + 1) - x
= cos Á cos Á
Á 1 ˜ ˜˜
Á ˜ fi x 2 + x + 10 = 10 x + 10 - x
Ë Ë ¯¯
Ê 3ˆ fi x2 - 8x = 0
= Áx+ ˜
Ë 2¯ fi x = 0, 8
Hence, the solution set is {0, 8}
Ê1+ xˆ
Ê 3ˆ
(
-1
ÁË x + ˜¯ + tan sec x = 0
2
) Ex-11. If a = 2 tan -1 Á
Ë 1 - x ˜¯
and

Ê Ê x2 - 1 ˆ ˆ Ê 1 - x2 ˆ
Ê 3ˆ -1 b = sin -1 Á ˜ x
ÁË x + ˜ + tan Á tan Á ˜˜ = 0 Ë 1 + x2 ¯
2¯ ÁË ÁË 1 ˜¯ ˜¯
a + b = p.
Ê 3ˆ Ê x2 - 1 ˆ
fi ÁË x + ˜¯ +Á ˜ =0
ÁË 1 ˜¯ Ê1+ xˆ
2 Soln. Given a = 2 tan -1 Á
Ë 1 - x ˜¯

( ) ( )
2
fi Ê 3ˆ 2
= 2 tan -1 1 + tan -1 x
ÁË x + ˜¯ = - x - 1
2
2
Êp ˆ
2 = 2 Á + tan -1 x˜
fi Ê 3ˆ Ë4 ¯
ÁË x + ˜¯ = x - 1
2
2
= p + 2 tan -1 x
fi 9 2
x 2 + 3x + = x 2 - 1
4 Ê 1 - x2 ˆ
9 Also, b = sin -1 Á ˜
fi 3x + = -1 Ë 1 + x2 ¯
4
p Ê 1 - x2 ˆ

9
3x = -1 - = -
13 = - cos -1 Á ˜
4 4 2 Ë 1 + x2 ¯
p
fi x=-
13 = - 2 tan -1 x
12 2
13 Thus, a + b
Hence, the solution is x = -
12 p p
= + 2 tan -1 x + - 2 tan -1 x
Ex-10. x: 2 2
Ê Ê xˆ Ê 1 ˆˆ Êpˆ =p
tan Á tan -1 Á ˜ + tan -1 Á ˜ ˜ = tan Á ˜
Ë Ë 10 ¯ Ë x + 1¯ ¯ Ë 4¯ Ex-12. f ( x ) = 2 sin -1 (2 x - 3)
Soln. We have Soln. As we know that,
Ê Ê x 1 ˆˆ p p
Á -1 Á 10 + x + 1 ˜ ˜ - £ sin -1 (2 x - 3) £
tan Á tan Á =1 2 2
fi x 1 ˜˜
Á Á1- . ˜ ˜ 2p 2p
Ë Ë 10 x + 1 ¯ ¯ fi - £ 2 sin -1 (2 x - 3) £
2 2
278 Comprehensive Trigonometry with Challenging Problems & Solutions for Jee Main and Advanced

-p £ f ( x ) £ p p 1 p p
fi fi - (
< tan -1 1 - x 2 - £ - )
Thus, R f = [-p , p ] 2 2 4 8

p È p p˘
Ex-13. f ( x ) = 2 sin -1 (2 x - 1) - = Í- , - ˙
4 Î 2 8˚

Soln. We have -
p
£ sin -1 (2 x - 1) £
p Ex-16. (
f ( x ) = cot -1 2 x - x 2 )
2 2
Soln. We have 2 x - x 2
fi -p £ 2 sin -1 (2 x - 1) £ p

fi -p -
p p
£ 2 sin -1 (2 x - 1) - £ p -
p (
= - x2 - 2 x + 1 + 1)
4 4 4 = 1 - ( x - 1)2 .
5p 3p
fi - £ f ( x) £ Thus, -• < 1 - ( x - 1) £ 1
2
4 4
È 5p 3p ˘
Thus, R f = Í- , ˙
Î 4 4˚
fi (
cot -1 ( -• ) < cot -1 1 - ( x - 1)
2
) £ cot -1
(1)

Ex-14 ( )
f ( x ) = 2 cos -1 - x 2 - p
fi (
0 < cot -1 1 - ( x - 1)
2
) £ p4
( )
Soln. We have f ( x ) = 2 cos -1 - x 2 - p Ê p˘
= Á 0, ˙ .
Ë 4˚
( ( ))
= 2 p - cos -1 x 2 - p Ex-17.
f ( x ) = sin -1 x + cos -1 x + tan -1 x
= p - 2 cos
-1
(x )
2

Soln. The function f £x£1


As we know that, 0 £ cos -1 x 2 £ p( ) we have f ( x ) = sin -1
x + cos x + tan -1 x
-1

fi ( )
0 £ 2 cos -1 x 2 £ 2 p =
p
2
+ tan -1 x

fi -2p £ -2 cos -1 x 2 £ 0( ) Thus, R f = ÈÎ f ( -1) , f (1)˘˚

fi -2p + p £ p - 2 cos -1 x 2 £ p ( ) È p p p p ˘ È p 3p ˘
= Í - , + ˙= Í , ˙.
Î2 4 2 4˚ Î4 4 ˚
fi -p £ f ( x ) £ p Ex-18.
Thus, R f = [-p , p ] . f ( x ) = sin -1 x + sec-1 x + tan -1 x

p Soln. The function f x = ±1


Ex-15. f ( x) =
1
2
(
tan -1 1 - x 2 - )
4 Now, f (1) = sin -1
(1) + sec (1) + tan -1 (1)
-1

Soln. We have p p 3p
= +0+ =
-• < 1 - x 2 £ 1 2 4 4
Also, f (-1) = sin -1 ( -1) + sec-1 ( -1) + tan -1 ( -1)
fi ( )
tan -1 ( -• ) < tan -1 1 - x 2 £ tan -1 (1)
p p
p = - +p -0-
fi - < tan -1
2
(1 - x ) £ p4
2 2 4
3p
=
p 1 p
fi -
4 2
(
< tan -1 1 - x 2 £
8
) 4
Ï p 3p ¸
Thus, R f = Ì , ˝
p p 1 p p p
fi -
4 4 2
(
- < tan -1 1 - x 2 - £ -
4 8 4
) Ó4 4 ˛
Inverse Trigonometric Function 279

Soln. We have sin (sin 5) + cos (cos10)


Ex-19. -1 -1
p
f ( x ) = 3 cot -1 x + 2 tan -1 x +
4 - tan -1 ( tan 6) + p - cot -1 (cot 10)
Soln. We have f (x)
= sin (sin (5 - 2p )) + cos (cos ( 4p - 10))
-1 -1
p
(
= 2 tan -1
x + cot -1
)
x + cot -1
x+
4 +p - tan -1 ( tan (6 - 2p )) - cot -1 (cot (10 - 4p ))
p p
= 2. + cot -1 x + = (5 - 2p ) + (4p - 10) + p + (6 - 2p ) - (10 - 4p )
2 4
5p = 5p - 9
= cot -1 x +
4 Ex-23. If U = cot -1 ( )
cos 2q - tan -1 ( cos 2q )
Also, 0 < cot -1 x < p
5p 5p 5p sin U = tan 2 q .
fi < cot -1 x + <p +
4 4 4 Soln. Given U = cot -1 ( )
cos 2q - tan -1 ( cos 2q )
5p 9p
fi < f ( x) < Ê 1 ˆ
4 4
fi U = tan -1 Á
Ë cos 2q ˜¯
- tan -1 ( cos 2q )
Ê 5p 9p ˆ
Thus, R f = ÁË , ˜. Ê ˆ
4 4¯ 1
- cos 2q
Á 2q ˜
Ex-20 ( ( (
sin cot -1 tan cos -1 x ))) = x , "x Œ(0,1] fi U = tan -1 Á cos
Á1+ 1 ˜
. cos 2q ˜
Ë cos 2q ¯
( ( (
Soln. We have sin cot -1 tan cos -1 x ))) Ê 1 - cos 2q ˆ
U = tan -1 Á

Ê Ë 2 cos 2q ˜¯
Ê Ê Ê 1 - x2 ˆ ˆ ˆ ˆ
-1 -1
= sin Á cot Á tan Á tan Á ˜˜˜˜ Ê 2 sin 2 q ˆ
Á Á Á
Ë ÁË x ˜¯ ˜¯ ˜ ˜ U = tan -1 Á
Ë Ë ¯¯ fi ˜
Ë 2 cos 2q ¯
Ê Ê 1 - x2 ˆ ˆ Ê sin 2 q ˆ
= sin Á cot -1 Á ˜˜ U = tan -1 Á
ÁË ÁË x ˜¯ ˜¯ fi ˜
Ë cos 2q ¯
Ê Ê xˆˆ Ê sin 2 q ˆ
= sin Á sin -1 Á ˜ ˜ U = sin -1 Á
Ë Ë 1¯¯ fi ˜
Ë sin 4 q + 1 - 2 sin 2 q ¯
=x
Ê Ê sin 2 q ˆˆ
Ex-21. ( (cot (tan
sin cosec -1 -1
x))) = x "x Œ(0,1] fi sin U = sin Á sin -1 Á
Ë
˜˜
Ë sin 4 q + 1 - 2 sin 2 q ¯ ¯

We have sin (cosec (cot ( tan x )))


-1 -1
Soln. Ê sin 2 q ˆ
fi sin U = Á ˜
Ë sin 4 q + 1 - 2 sin 2 q ¯
Ê -1 Ê Ê -1 Ê 1 ˆ ˆ ˆ ˆ
= sin Á cosec Á cot Á cot Á ˜ ˜ ˜ ˜ Ê ˆ
Ë Ë Ë Ë x¯¯¯¯
Á sin 2 q ˜ sin q
2
fi sin U = Á ˜ = = tan 2 q
Ê Ê 1ˆˆ
( ) q
2
= sin Á cosec -1 Á ˜ ˜
2 cos
Ë x¯¯
ÁË sin 2 q - 1 ˜¯
Ë

(
= sin sin -1 ( x ) )
Êp 1 aˆ
=x Ex-24. tan Á + cos -1 ˜
Ë4 2 b¯
Ex-22 Find the value of sin (sin 5) +
-1
Êp 1 a ˆ 2b
+ tan Á - cos -1 ˜ =
cos -1
(cos10) + tan ( tan (-6)) + cot (cot (-10))
-1 -1 Ë4 2 b¯ a
280 Comprehensive Trigonometry with Challenging Problems & Solutions for Jee Main and Advanced

1 Ê aˆ Ê a-b Ê xˆˆ
Soln. Let cos -1 Á ˜ = q Ex-26. 2 tan -1 Á tan Á ˜ ˜
2 Ë b¯ Ë a+b Ë 2¯ ¯
a
cos (2q ) = Ê b + a cos x ˆ
= cos -1 Á
b Ë a + b cos x ˜¯

Êp ˆ Êp ˆ Ê a-b Ê xˆˆ
tan Á + q ˜ + tan Á - q ˜ Soln. We have 2 tan -1 Á tan Á ˜ ˜
Ë4 ¯ Ë4 ¯ Ë a+b Ë 2¯ ¯

= 1 + tan q + 1 - tan q Ê Ê a - bˆ
1- Á
Ê xˆ ˆ
tan 2 Á ˜ ˜
1 - tan q 1 + tan q Á Ë a + b¯˜ Ë 2¯
-1
= cos Á ˜
(1 + tan q )2 + (1 - tan q )2 Á1+ Ê a - b ˆ 2 Ê xˆ ˜
ÁË ˜¯ ˜¯
= ÁË ÁË a + b ˜¯ tan
(1 - tan q )2 2
Ê x ˆ
2 (1 + tan q )
2
Á (a + b) - (a - b) tan 2 ÊÁË ˆ˜¯ ˜
= -1 2
= cos Á ˜
(1 - tan q )
2
Á ( a + b ) + ( a - b ) tan 2 Ê x ˆ ˜
ÁË ÁË ˜¯ ˜¯
2
2
=
cos 2q Ê Ê 2 Ê xˆˆ Ê 2 Ê xˆˆ ˆ
Á a ÁË1 - tan ÁË 2 ˜¯ ˜¯ + b ÁË1 + tan ÁË 2 ˜¯ ˜¯ ˜
2b = cos -1 Á ˜
=
a Á Ê 2 Ê xˆˆ Ê 2 Ê xˆˆ ˜
ÁË a ÁË1 + tan ÁË 2 ˜¯ ˜¯ + b ÁË1 - tan ÁË 2 ˜¯ ˜¯ ˜¯
Ê cos x + cos y ˆ
cos -1 Á
Ë 1 + cos x cos y ˜¯
Ex-25
Ê Ê 2 Ê xˆ ˆ
ˆ
Á Á 1 - tan ÁË ˜¯ ˜ ˜
Ê Ê xˆ Ê yˆˆ Á aÁ 2
= 2 tan -1 Á tan Á ˜ tan Á ˜ ˜ ˜ + b˜
Ë Ë 2¯ Ë 2¯¯ Á ÁÊ 2 Ê xˆˆ ˜ ˜
Á ÁË ÁË1 + tan ÁË 2 ˜¯ ˜¯ ˜¯ ˜
Ê Ê xˆ Ê yˆˆ = cos -1 Á ˜
Soln. R.H.S = 2 tan -1 Á tan Á ˜ tan Á ˜ ˜ Á Ê Ê x ˆ ˆ ˜
Ë Ë 2¯ Ë 2¯¯
Á 1 - tan ÁË 2 ˜¯ ˜ ˜
2
Á
Á a + bÁ ˜˜
Ê Ê xˆ Ê yˆ ˆ Á
1 - tan 2 Á ˜ tan 2 Á ˜ ˜ ÁÊ 2 Ê xˆˆ ˜ ˜
-1
Á Ë 2 ¯ Ë 2¯ Á ÁË ÁË1 + tan ÁË 2 ˜¯ ˜¯ ˜¯ ˜¯
= cos Á ˜ Ë
Á 1 + tan 2 Ê x ˆ tan 2 Ê y ˆ ˜
ÁË ÁË ˜¯ ÁË ˜¯ ˜¯
2 2 -1 Ê a cos x + b ˆ
= cos Á
Ë a + b cos x ˜¯
Ê Ê xˆ Ê yˆ Ê xˆ Ê yˆ ˆ
Á cos 2 Á ˜ cos 2 Á ˜ - sin 2 Á ˜ sin 2 Á ˜ ˜
-1 Ë 2¯ Ë 2¯ Ë 2¯ Ë 2¯ Ex-27 If tan -1 x, tan -1 y, tan -1 z
= cos Á ˜
Á cos 2 Ê x ˆ cos 2 Ê y ˆ + sin 2 Ê x ˆ sin 2 Ê y ˆ ˜
ÁË ÁË ˜¯
2
ÁË ˜¯
2
ÁË ˜¯
2
ÁË ˜¯ ˜¯
2
( x + z ) y 2 + 2 y (1 - xz ) = x + z y Œ(0,1) ,
xz x > 0 and z > 0.
-1 Ê(1 + cos x ) (1 + cos y ) - (1 - cos x ) (1 - cos y ) ˆ
= cos Á (1 + cos x ) (1 + cos y ) + (1 - cos x ) (1 - cos y ) ˜
Ë ¯ Soln. Given tan -1 x, tan -1 y, tan -1 z
Ê ( 2 cos x + 2 cos y ) ˆ fi tan -1 x + tan -1 z = 2 tan -1 y
= cos -1 Á ˜
Ë (2 + 2 cos x cos y ) ¯ Ê x+zˆ Ê 2y ˆ
fi tan -1 Á ˜ = tan -1 Á ˜
Ë 1 - xz ¯ Ë 1 - y2 ¯
-1 Ê (cos x + cos y ) ˆ
= cos Á ˜
Ë (1 + 1cos x cos y ) ¯ Ê x + z ˆ Ê 2y ˆ
ÁË ˜=
fi 1 - xz ¯ ÁË 1 - y 2 ˜¯

fi ( x + z ) (1 - y 2 ) = 2 y (1 - xz )
Inverse Trigonometric Function 281

fi y 2 ( x + z ) + 2 y (1 - xz ) = ( x + z )
p p 2p
- cos -1 x + - cos -1 y =
Ex-28. 2 2 3
p
Ê 33p ˆ
sin -1 Á sin
Ë ˜
7 ¯
Ê
+ cos -1 Á cos
Ë
46p ˆ
7 ¯
˜ fi (
p - cos -1 x + cos -1 y =
2
3
)
2p p
Ê
+ tan -1 Á - tan
13p ˆ -1 Ê Ê 19p ˆ ˆ
˜ + cot ÁË cot ÁË - ˜
fi ( )
cos -1 x + cos -1 y = p - = ......(iii)
Ë 8 ¯ 8 ¯ ˜¯ 3 3
Adding (ii) and (iii), we get,
13p
=
7 2p
2 cos -1 x =
Ê 33p ˆ -1 Ê 46p ˆ 3
Soln. We have sin -1 Á sin ˜¯ + cos ÁË cos ˜
Ë 7 ¯ p
7 fi cos -1 x =
3
Ê 13p ˆ Ê Ê 19p ˆ ˆ
+ tan -1 Á - tan ˜ + cot -1 Á cot Á - p
Ê ˆ 1
Ë 8 ¯ Ë Ë 8 ˜¯ ˜¯ fi x = cos Á ˜ =
Ë 3¯ 2
Ê Ê 5p ˆ ˆ Ê Ê 4p ˆ ˆ
= sin -1 Á sin Á 4p + ˜ ˜ + cos -1 Á cos Á 6p + ˜ ˜ 1
Ë Ë ¯
7 ¯ Ë Ë 7 ¯¯ when x = , y = 0.
2
Ê Ê 3p ˆ ˆ x = 1/2 and y = 0.
+ tan -1 Á - tan Á 2p - ˜
Ë Ë 8 ¯ ˜¯ Ê 1 + x2 - 1 - x2 ˆ
Ex-30. If y = tan -1 Á ˜
Ê Ê 3p ˆ ˆ ÁË 1 + x 2 + 1 - x 2 ˜¯
+ cot -1 Á - cot Á 2p + ˜ ˜
Ë Ë 8 ¯¯
x 2 = sin ( 2 y )
Ê Ê 5p ˆ ˆ Ê Ê 4p ˆ ˆ
= sin -1 Á sin Á ˜ ˜ + cos -1 Á cos Á ˜ ˜ Ê 1 + x2 - 1 - x2 ˆ
Ë Ë 7 ¯¯ Ë Ë 7 ¯¯
Soln. Given y = tan -1 Á ˜
Ê ÁË 1 + x 2 + 1 - x 2 ˜¯
Ê 3p ˆ ˆ Ê Ê 3p ˆ ˆ
+ tan -1 Á tan Á ˜ ˜ + cot -1 Á - cot Á ˜ ˜
Ë Ë ¯
8 ¯ Ë Ë 8 ¯¯
Put x = cos ( 2q )
2

Ê Ê 2p ˆ ˆ Ê Ê 4p ˆ ˆ
= sin -1 Á sin Á p - ˜ ˜ + cos -1 Á cos Á ˜ ˜ Ê 2 cos q - 2 sin q ˆ
Ë Ë 7 ¯¯ Ë Ë 7 ¯¯ Thus, y = tan -1 Á ˜
Ë 2 cos q + 2 sin q ¯
Ê Ê 3p ˆ ˆ Ê Ê 5p ˆ ˆ
+ tan -1 Á tan Á ˜ ˜ + cot -1 Á cot Á ˜ ˜ -1 Ê cos q - sin q ˆ
Ë Ë ¯
8 ¯ Ë Ë 8 ¯¯ = tan Á
Ë cos q + sin q ˜¯
= 2p + 4p + 3p + 5p
7 7 8 8 -1 Ê 1 - tan q ˆ
= tan Á
6p Ë 1 + tan q ˜¯
= +p Ê Êp ˆˆ
7 = tan -1 Á tan Á - q ˜ ˜
Ë Ë4 ¯¯
13p
= Êp ˆ
7 = Á - q˜
Ë4 ¯
Ex-29. x and y:
2p p p
sin -1 x + sin -1 y = , cos -1 x - cos -1 y = fi q= -y
3 3 4
p
Soln. fi
1
2
( )
cos -1 x 2 = - y
4
-1 -1 2p
sin x + sin y = , .......(i) p
3 fi ( )
cos -1 x 2 = - 2 y
2
p
cos -1 x - cos -1 y = ......(ii) p
3 fi ( ) Ê
x 2 = cos Á - 2 y˜
Ë2
ˆ
¯
282 Comprehensive Trigonometry with Challenging Problems & Solutions for Jee Main and Advanced

fi x 2 = sin ( 2 y )
= ( a + b )ÊË b ( a
2 2 2
) (a
+ b2 + a + a 2
+ b2 - b ˆ
¯ )
b 3
Ê1 Ê b ˆˆ
= ( a +b ) a
Ex-31. cosec2 Á tan -1 Á ˜ ˜ 2 2 2
+ b 2 (a + b )
2 Ë 2 Ë a ¯¯

+
a3 Ê1 Ê a ˆˆ
sec2 Á tan -1 Á ˜ ˜ = (a + b ) a 2 + b 2 . ( ) ( )
= a 2 + b 2 (a + b )
2 Ë2 Ë b ¯¯
Soln. We have Ex-32. Find the minimum value of n, if
b3 Ê1 Ê b ˆˆ a
3 Ê1 Ê a ˆˆ Ê n 2 - 10n + 21.6 ˆ p
cosec2 Á tan -1 Á ˜ ˜ + sec2 Á tan -1 Á ˜ ˜ cot -1 Á
2 Ë2 Ë a ¯¯ 2 Ë2 Ë b ¯¯ p ˜ > 6 ,n ŒN
Ë ¯
Ê Ê bˆˆ Ê Êaˆˆ
tan -1 Á ˜ ˜ Á tan -1 Á ˜ ˜ Ê n 2 - 10n + 21.6 ˆ p
b 3 Á Ëa¯ a 3
Ë b¯ Soln. Given cot -1 Á ˜ > 6 ,n ŒN
= cosec2 Á ˜+ sec2 Á ˜ Ë p ¯
2 Á 2 ˜ 2 Á 2 ˜
ÁË ˜¯ Á ˜ Ê n 2 - 10n + 21.6 ˆ Êpˆ
Ë ¯ < cot Á ˜
fi Á ˜ Ë 6¯
Ë p ¯
b3 a3
+
Ê Ê bˆˆ Ê -1 Ê a ˆ ˆ fi (n 2 - 10 n + 21.6 < p 3
Á tan -1 Á ˜ ˜ Á tan ÁË b ˜¯ ˜
Ëa¯
= 2 sin 2 Á ˜ 2 cos 2 Á ˜ (n 2 - 10 n + 21.6 < 5.6
Á 2 ˜ ˜ fi
Á 2
ÁË ˜¯ Á ˜
Ë ¯ fi (n 2 - 10 n + 16 < 0

b3 a3 fi
( n - 2 ) ( n - 8) < 0
= +
Ê Ê b ˆˆ Ê Ê a ˆˆ fi 2<n<8
1 - cos Á tan -1 Á ˜ ˜ 1 + cos Á tan -1 Á ˜ ˜
Ë Ë a ¯¯ Ë Ë b ¯¯ Thus, the minimum value of n is 2.
Ex-33.
b3
= Ï Ï Ê 2- 3ˆ ¸¸
-1 Ê 12 ˆ
Ê Ê
1 - cos Á cos -1 Á
a ˆˆ Ô Ô
sin -1 Ìcot Ìsin -1 Á
Á
˜ + cos Á
˜ Ë
˜ + sec
¯
-1
( 2 )Ô˝Ô˝ = 0
˜˜ ÔÓ ÓÔ Ë 4 ¯ 4 Ô˛Ô˛
ÁË ÁË a 2 + b 2 ˜¯ ˜¯
Soln. We have
a 3 Ï Ï Ê 2- 3ˆ ¸¸
-1 Ê 12 ˆ
+
Ê Ê b ˆˆ
Ô Ô
sin -1 Ìcot Ìsin -1 Á
Á 4
˜ + cos Á
˜ Ë 4 ˜ + sec
¯
-1
( 2 )Ô˝Ô˝
ÔÓ ÔÓ Ë ¯ Ô˛Ô˛
1 + cos Á cos -1 Á ˜˜
ÁË ÁË a 2 + b 2 ˜¯ ˜¯
Ï Ï ¸¸
Ô -1 Ê 4 - 2 3 ˆ -1 Ê 3 ˆ
b 3
a 3 = sin -1 Ô
Ìcot Ìsin Á
Á 8
˜ + cos Á ˜ + sec
˜ Ë 2 ¯
-1
( 2 )Ô˝Ô˝
= + ÔÓ ÔÓ Ë ¯ Ô˛Ô˛
Ê a ˆ Ê b ˆ
1- Á ˜ 1+ Á ˜ Ï Ï Ê 2ˆ ¸¸
ÁË a 2 + b 2 ˜¯ ÁË a 2 + b 2 ˜¯ Ô Ô -1 Á
-1 Ô
= sin Ìcot Ìsin
( )
3 - 1 ˜ p p ÔÔÔ
Á ˜ + 6 + 4 ˝˝
(a )+a ( a ) Ô Ô 8 ÔÔ
b3 2
+ b2 3 2
+ b2 ÁË ˜¯
ÔÓ Ó ˛˛Ô
=
a2 + b2 - a a2 + b2 + b ÏÔ ÏÔ Ê 3 - 1ˆ p p ¸Ô¸Ô
= sin -1 Ìcot Ìsin -1 Á ˜ + + ˝˝
Ê ˆ ÓÔ ÔÓ Ë 2 2 ¯ 6 4 Ô˛˛Ô
= ( a2 + b2 Á
b3
)
ÁË a 2 + b 2 - a
+
a3
˜
a 2 + b 2 + b ˜¯
Ï Ï p p p ¸¸
= sin -1 Ìcot Ì + + ˝˝
Ó Ó12 6 4 ˛˛
( a + b + a ) a ( a + b - b )ˆ
Ê b3 2 2 3 2 2
-1 Ï Êpˆ¸
= ( a +b )
2 2 Á
Á + ˜
˜
= sin Ìcot ÁË ˜¯ ˝
b a 2 2 Ó 2 ˛
ÁË ˜ ¯
Inverse Trigonometric Function 283

= sin -1 (0)
( ( (
and y = sec Ê cot -1 sin tan -1 cosec cos -1 a
Ë ( ))))ˆ¯
= 0.

Ex-34. ( (
x ; Èsin -1 cos -1 sin -1 tan -1 x
ÎÍ
( )))˚˙˘ = 1 , Ê Ê Ê Ê 1 ˆˆˆˆ
= sec Á cot -1 Á sin Á tan -1 Á
ÁË ÁË Ë ˜˜˜˜
Ë a 2 - 1 ¯ ¯ ˜¯ ˜¯

( (
Soln. Given Èsin -1 cos -1 sin -1 tan -1 x
ÍÎ ( )))˘˙˚ = 1 Ê
Ë
Ê 1ˆˆ
= sec Á cot -1 Á ˜ ˜
Ë a¯¯
fi ( (
1 £ sin -1 cos -1 sin -1 tan -1 x ( ))) < 2 = sec Ê sec-1
Ë ( a + 1)ˆ¯
2

fi ( ( ))
sin (1) £ cos -1 sin -1 tan -1 x < sin (2)
= ( a + 1)
2

fi cos (sin ( 2)) < sin ( tan x ) £ cos (sin (1))


-1 -1
Thus, x = y

sin (cos (sin ( 2))) < ( tan x ) £ sin (cos (sin (1)))
-1 Ex-37.

Ê 1ˆ Ê 1ˆ Ê 1ˆ
tan -1 Á ˜ + tan -1 Á ˜ + tan -1 Á ˜ +.......
fi tan (sin (cos (sin ( 2)))) < x £ tan (sin (cos (sin (1)))) Ë 3¯ Ë 7¯ Ë 13 ¯
Ex-35 Soln. We have
Ê 2x ˆ Ê 1ˆ Ê 1ˆ Ê 1ˆ
2 tan -1 x + sin -1 Á is independent of x. tan -1 Á ˜ + tan -1 Á ˜ + tan -1 Á ˜ +...
Ë 1 + x 2 ˜¯ Ë 3¯ Ë 7¯ Ë 13 ¯
Ê 2x ˆ Ê 1 ˆ Ê 1 ˆ Ê 1 ˆ
Soln. Given 2 tan -1 x + sin -1 Á = tan -1 Á + tan -1 Á + tan -1 Á
Ë 1 + x 2 ˜¯ Ë 1 + 2 ˜¯ Ë 1 + 6 ˜¯ Ë 1 + 12 ˜¯
+ ........
= 2 tan -1 x + p - 2 tan -1 x, x > 1
-1 Ê 2 - 1 ˆ -1 Ê 3 - 2 ˆ -1 Ê 4 - 3 ˆ
= p, x > 1 = tan ÁË ˜¯ + tan ÁË ˜¯ + tan ÁË ˜
1 + 2.1 1 + 3.2 1 + 4.3 ¯
Thus, x Œ(1, • )
-1 Ê n + 1 - n ˆ

( ( ( ( ) )) )
+ ...... + tan Á ˜
Ex-36. If x = cosec Ê tan -1 cos cot -1 sec sin -1 a ˆ Ë 1 + (n + 1) .n ¯
Ë ¯
= tan ( 2) - tan (1) + tan (3) - tan ( 2)
-1 -1 -1 -1
Ê -1
and y = sec Ë cot (sin (tan (cosec (cos a))))ˆ¯ ,
-1 -1
+ tan -1 ( 4) - tan -1 (3) + ... + tan -1 ( n + 1) - tan -1 (n )
a Œ[0,1] = tan ( n + 1) - tan (1)
-1 -1
x and y.
-1 Ê n + 1 - 1 ˆ

Ë ( (
Soln. Given x = cosec Ê tan -1 cos cot -1 sec sin -1 a ( ( ))))ˆ¯ = tan Á ˜
Ë 1 + ( n + 1) .1¯
Ê Ê Ê -1 Ê 1 ˆ ˆ ˆ ˆ Ê n ˆ
= tan -1 Á , nÆ•
= cosec Á tan -1
Á cos Á cot Á ˜˜˜˜ Ë n + 2 ˜¯
ÁË ÁË Ë Ë 1 - a 2 ¯ ¯ ˜¯ ˜¯
= tan (1)
-1

Ê Ê Ê Ê 1ˆˆˆˆ
= cosec Á tan -1 Á cos Á cos -1 Á ˜ ˜ ˜ ˜ p
Ë Ë Ë Ë a¯¯¯¯ =
4
Ê -1 Ê 1 ˆ ˆ Ex-38. Find the sum of
= cosec Á tan Á ˜ ˜
Ë Ë a¯¯ Ê 1 ˆ Ê 2 - 1ˆ Ê 3 - 2ˆ
sin -1 Á + sin -1 Á + sin -1 Á
( a + 1)ˆ¯
Ê Ë 2¯˜ ˜ ˜
= cosec Ë cosec
-1 2 Ë 6 ¯ Ë 12 ¯
Ê n - n - 1ˆ
= ( a + 1)
2 +...... + sin -1 Á ˜ +...... to •
Ë n ¥ n + 1¯
284 Comprehensive Trigonometry with Challenging Problems & Solutions for Jee Main and Advanced

-1 Ê n - n - 1 ˆ
Soln. Let tn = sin Á ˜
(
+....... + tan -1 ( 2n + 1) - tan -1 ( 2n - 1) )
Ë n ¥ n + 1¯
(
= tan ( 2n + 1) - tan 1
-1 -1
)
Ê r - r -1 ˆ
-1
= tan Á ˜ Ê -1 ( 2n + 1 - 1) ˆ
ÁË 1 + r ( r - 1) ˜¯ = Á tan
Ë 1 + ( 2n + 1) .1˜¯
= tan
-1
( r ) - tan ( -1
r -1 ) Ê Ê n ˆˆ p
= Á tan -1 Á = , when n Æ • .
Ë Ë n + 1˜¯ ˜¯ 4
-1 Ê n - n - 1 ˆ
n
Now, Â sin Á ˜
r =1 Ë n n +1 ¯ Ê xˆ Ê yˆ
Ex-40. If cos -1 Á ˜ + cos -1 Á ˜ = q , then
Ë 2¯ Ë 3¯
-1 Ê r - r -1 ˆ
n
= Â tan Á ˜
r =1 Ë 1 + r r - 1¯ 9 x 2 - 12 xy cos q + 4 y 2 = 36 sin 2 q
n
= Â tan
r =1
-1
(r - tan -1 r - 1 ) Ê xˆ Ê yˆ
Soln. Given cos -1 Á ˜ + cos -1 Á ˜ = q
Ë 2¯ Ë 3¯
( -1 -1
= tan 1 - tan 0 + tan
-1
) (
2 - tan -1 1 ) Êx y x2 y2 ˆ
+ ( tan -1
3 - tan -1
2 ) + ( tan -1
4 - tan -1
3 ) fi cos -1 Á . - 1 -
ÁË 2 3 4
1- ˜ =q
9 ˜¯
+ .............. + tan
-1
(n - tan -1 n - 1 ) Êx y x2 y2 ˆ
fi cos q = Á . - 1 - 1- ˜
= tan -1 n - tan -1 0 ÁË 2 3 4 9 ˜¯

= tan -1 n Ê x yˆ
2
Ê x2 ˆ Ê x2 ˆ
p fi ÁË cosq - ˜¯ = Á1 - ˜ Á1 - ˜
When n Æ •, the sum is tan -1 (• ) = 6 Ë 4 ¯Ë 9¯
2
Ê 2 xy x2 y 2 ˆ
Ex-39. fi Á cos q - 2. .cos q +
Ë 36 ˜¯
( ) ( ) ( )
6
cot -1 2.12 + cot -1 2.22 + cot -1 2.32 + .......
Soln. We have x2 y 2 x2 y 2
= 1- - +
( ) ( ) ( )
4 9 36
cot -1 2.12 + cot -1 2.22 + cot -1 2.32 + .......
xy x2 y 2
fi cos 2 q - cos q = 1 - -
-1 Ê 1 ˆ -1 Ê 1 ˆ -1 Ê 1 ˆ 3 4 9
= tan Á 2 ˜ + tan Á 2 ˜ + tan Á 2 ˜ + ...
Ë 2.1 ¯ Ë 2.2 ¯ Ë 2.3 ¯
x 2 xy y2
fi - .cos q + = 1 - cos 2 q
-1 Ê 2 ˆ -1 Ê 2 ˆ -1 Ê 2 ˆ 4 3 9
= tan Á ˜ + tan Á ˜ + tan Á ˜ + ...
Ë 4¯ Ë 16 ¯ Ë 36 ¯ fi 9 x 2 - 12 xy cos q + 4 y 2 = 12 sin 2 q
-1 Ê 2 ˆ Ê 2 ˆ Ê 2 ˆ
= tan Á ˜ + tan -1 Á ˜ + tan -1 Á + ...
Ë 1 + 3¯ Ë 1 + 15 ¯ Ë 1 + 35 ˜¯ Note.
Ê1 Ê 5ˆˆ
-1 Ê 3 - 1 ˆ Ê 5-3 ˆ tan Á cos -1 Á ˜ ˜
= tan Á + tan -1 Á Ex-41.
Ë 1 + 3.1˜¯ Ë 1 + 5.3 ˜¯ Ë2 Ë 3 ¯¯

Ê 7-5 ˆ Ê ( 2n + 1) - ( 2n - 1) ˆ 1 Ê 5ˆ
+ tan -1 Á ˜ + ... + tan -1 Á ˜ Soln. Let cos -1 Á ˜ = q
Ë 1 + 7.5 ¯ Ë 1 + (2n + 1) (2n - 1) ¯ 2 Ë 3 ¯

( -1 -1 -1 -1
= tan 3 - tan 1 + tan 5 - tan 3 ) ( ) fi
Ê 5ˆ
cos -1 Á ˜ = 2q
Ë 3 ¯
+ ( tan -1
7 - tan -1 5) + ( tan -1
9 - tan -1 7)
5
fi cos 2q =
3
Inverse Trigonometric Function 285

2
1 - tan 2 q 5 Ê Ê 1 - x2 ˆ 2 ˆ
fi =
1 + tan 2 q fi 1- Á 2Á - 1˜ = 0
ÁË Ë 2 x ˜¯
3
˜¯
1 - tan 2 q + 1 + tan 2 q 5 +3
fi = 2
1 - tan q - 1 - tan q
2 2
5 -3 Ê Ê 1 - x2 ˆ 2 ˆ
fi Á 2Á - 1˜ = 1
2 5 +3 ÁË Ë 2 x ˜¯ ˜¯
fi =
-2 tan 2 q 5 -3
Ê Ê 1 - x2 ˆ 2 ˆ
5 +3 fi Á 2Á - 1˜ = ±1
ÁË Ë 2 x ˜¯
1
fi = ˜¯
tan q2
3- 5

5 (3 - 5 )
2
2
Ê 1 - x2 ˆ
3- fi 2Á ˜ = 1 ± 1 = 2, 0
fi tan 2 q = = Ë 2x ¯
3+ 5 4
2 2

( ) ( ) Ê 1 - x2 ˆ Ê 1 - x2 ˆ
2
3- 5 3- 5 fi 2Á ˜ = 2 & 2 Á 2x ˜ = 0
fi tanq = ± =± Ë 2x ¯ Ë ¯
4 2
2
Note. Ê 1 - x2 ˆ
Á 2x ˜ = 1 & 1 - x = 0
2

Ë ¯
Ex-42.
Î { (
x sin ÈÍ2 cos -1 cot 2 tan -1 x ˙˘ = 0
˚ )} Ê 1 - x2 ˆ
fi Á 2 x ˜ = ±1 & x = ±1
Ë ¯

Î { (
Soln. Given sin ÈÍ2 cos -1 cot 2 tan -1 x ˘˙ = 0
˚ )} fi x 2 + 2 x - 1 = 0, x 2 - 2 x - 1 = 0 & x = ±1

( x + 1)2 = ( ) ( 2)
2 2
2 , ( x - 1) =
2
Let tan -1 x = q fi x = tanq fi &x=±

fi (
sin 2 cos -1 (cot ( 2q )) = 0 ) fi x = -1 ± 2 , 1 ± 2 , ± 1

Ê Ê 1 - tan 2 q ˆ ˆ Ex-43.
sin Á 2 cos -1 Á ˜˜ = 0 Ê

Ë Ë 2 tan q ¯ ¯ y ˆ -1 Ê 3 ˆ
tan -1 x + cos -1 Á ˜ = sin Á
ÁË 1 + y 2 ˜¯ Ë 10 ˜¯
Ê Ê 1 - x2 ˆ ˆ
fi sin Á 2 cos -1 Á ˜˜ = 0
Ë Ë 2 x ¯¯
Ê y ˆ -1 Ê 3 ˆ
Ê Ê Ê 1 - x2 ˆ 2 ˆ ˆ Soln. Given tan -1 x + cos -1 Á ˜ = sin Á
-1
sin Á cos Á 2 Á - 1˜ ˜ = 0 ÁË 1 + y 2 ˜¯ Ë 10 ˜¯
fi ÁË ÁË Ë 2 x ˜¯ ˜¯ ˜¯
Ê 1ˆ
fi tan -1 x + tan -1 Á ˜ = tan -1 3
Ê Ê Ê 1 - x2 ˆ ˆˆ 2 Ë y¯
sin Á cos -1 Á 2 Á - 1˜˜ = 0
fi ÁË ÁË Ë 2 x ˜¯ ˜¯ ˜¯ Ê 1 ˆ
x+
Á y ˜
tan -1 Á ˜ = tan (3)
-1
Ê Ê 2 ˆˆ fi
Ê Ê 1 - x2 ˆ 2 ˆ ˜ 1
Á 1 - x. ˜
Á -1 Á ˜ Ë y¯
fi sin Á sin Á 1 - Á 2 Á ˜ - 1˜ ˜ ˜ = 0
Á ÁË Ë 2 x ¯ ˜¯ ˜
Ë ÁË ˜¯ ¯ Ê xy + 1ˆ
fi tan -1 Á = tan -1 (3)
Ë y - x ˜¯
2
Ê Ê 1 - x2 ˆ 2 ˆ
fi Ê xy + 1ˆ
1- Á 2Á - 1˜ = 0 fi ÁË y - x ˜¯ = 3
ÁË Ë 2 x ˜¯ ˜¯
fi xy y x
286 Comprehensive Trigonometry with Challenging Problems & Solutions for Jee Main and Advanced

fi 3 x + 1 = y (3 - x ) Ê 6 tan q ˆ

3x + 1 Ë 9 + tan 2 q ˜¯
fi y=
3- x Ê 2 ˆ
Á tan q ˜
when x y
=Á 3 ˜
x y Á Ê tan q ˆ 2 ˜
Á1+ Á ˜
Ë Ë 3 ˜¯ ¯
Ex-44. If cos -1 x + cos -1 y + cos -1 z = p
Ê tan q ˆ
of x 2 + y 2 + z 2 + 2 xyz Á 2. ˜
=Á 3 ˜
Soln. We have, cos -1 x + cos -1 y + cos -1 z = p Á Ê tan q ˆ 2 ˜
Á1+ Á ˜
fi cos -1 x + cos -1 y = p - cos -1 z Ë Ë 3 ˜¯ ¯

cos -1 x + cos -1 y = cos -1 (- z ) Ê 3 sin 2q ˆ



( 1
)
q = tan -1 2 tan 2 q - sin -1 Á
Ë 5 + 4 cos 2q ˜¯
( xy - ) = cos (- z)
2
-1
fi cos 1- x 2
1- y 2
Ê tan q ˆ
fi ( 1
)
q = tan -1 2 tan 2 q - .2 tan -1 Á
Ë 3 ˜¯
fi ( xy + z ) 2
(
= 1- x 2
) (1 - y ) 2 2
Ê tan q ˆ
fi x 2 y 2 + 2 xyz + z 2 = 1 - x 2 - y 2 + x 2 y 2 fi ( )
q = tan -1 2 tan 2 q - tan -1 Á
Ë 3 ˜¯
fi x 2 + y 2 + z 2 + 2 xyz = 1 Ê tan q ˆ
2 tan 2 q -
Ex-45. q = tan Á -1 Á 3 ˜
fi q˜
{ ( )}
tan
sin ÈÍ2 cos -1 cot 2 tan -1 x ˘˙ Á 1 + 2 tan 2 q . ˜
Î ˚ Ë 3 ¯
function f (x f (x), Ê 6 tan 2 q - tan q ˆ
x f (x fi tan q = Á ˜
Ë 3 + 2 tan 3 q ¯
x a± b,
fi 3 tan q + 2 tan 4 q - 6 tan 2 q + tan q = 0
fi 2 tan 4 q - 6 tan 2 q + 4 tan q = 0
Soln. fi tan 4 q - 3 tan 2 q + 2 tan q = 0
( )
Note.
fi tan q tan 3 q - 3 tan q + 2 = 0
Ê 3 sin 2q ˆ
Ex-46. If q = tan -1
( 1
2
2
)
2 tan q - sin -1 Á
Ë 5 + 4 cos 2q ˜¯
,
fi tan q ( tan q - 1) ( tan q + 2) = 0
2

q. fi tan q = 0,1, - 2
when tan q = 0 fi q = np , n œ I
Ê 3 sin 2q ˆ p
1
(
Soln. Given q = tan -1 2 tan 2 q - sin -1 Á
2
)
Ë 5 + 4 cos 2q ˜¯
when tan q = 1 fi q = m p + , m ΠI
4
when tan q = -2 fi q = p p + tan -1 ( -2) , p ΠI
Ê 3 sin 2q ˆ
Now, Á
Ë 5 + 4 cos 2q ˜¯ Note.
Ex-47.
Ê ˆ
6 tan q
Á ˜ Ê 1ˆ Ê 1ˆ Ê 142 ˆ
Á 1 + tan 2 q ˜ 3 tan -1 Á ˜ + 2 tan -1 Á ˜ + sin -1 Á
=
Á Ê 1 - tan 2 q ˆ ˜
Ë 2¯ Ë 5¯ Ë 65 5 ˜¯
Á 5 + 4Á ˜˜
ÁË Ë 1 + tan 2 q ¯ ˜¯
Soln. We have
Ê ˆ Ê 1ˆ Ê 1ˆ Ê 142 ˆ
6 tan q 3 tan -1 Á ˜ + 2 tan -1 Á ˜ + sin -1 Á
=Á ˜ Ë 2¯ Ë 5¯ Ë 65 5 ˜¯
( ) (
ÁË 5 1 + tan q + 4 1 - tan 2 q
2
) ˜¯
Inverse Trigonometric Function 287

Ê 1ˆ Ê 132 + 10 ˆ Ê 142 ˆ
Now, 3 tan -1 Á ˜ = tan -1 Á ˜ + tan -1 Á
Ë 2¯ Ë 24 - 55 ¯ Ë 31 ˜¯
Ê 1 Ê 1ˆ 3 ˆ Ê 142 ˆ Ê 142 ˆ
Á 3. - ÁË ˜¯ ˜ = tan -1 Á - + tan -1 Á
2 2 ˜ Ë 31 ˜¯ Ë 31 ˜¯
= tan -1 Á
Á Ê 1ˆ ˜
2
Ê 142 ˆ Ê 142 ˆ
Á 1 - 3 ÁË ˜¯ ˜¯ = - tan -1 Á ˜ + tan -1 Á
Ë 2 Ë 31 ¯ Ë 31 ˜¯
Ê 3 1ˆ
- x
-1 Á 2 8˜ Ex-48.
= tan Á
3˜ ax
Ê ˆ Ê bx ˆ
Á 1- ˜ sin -1 Á ˜ + sin -1 Á ˜ = sin -1 x
Ë 4¯ Ë c¯ Ë c¯

-1 Ê 12 - 1ˆ a 2 + b2 = c2 , c π 0
= tan Á
Ë 2 ˜¯
Ê ax ˆ Ê bx ˆ
-1 Ê 11ˆ
Soln. Given sin -1 Á ˜ + sin -1 Á ˜ = sin -1 x
= tan Á ˜ Ë c¯ Ë c¯
Ë 2¯
Ê ax ˆ Ê bx ˆ
-1 Ê 1 ˆ
fi sin -1 Á ˜ = sin -1 x - sin -1 Á ˜
2 tan Á ˜ Ë c¯ Ë c¯
Ë 5¯
Ê ax ˆ Ê b 2 x 2 bx ˆ
Ê 1 ˆ fi sin -1 Á ˜ = sin -1 Á x 1 - 2 - 1 - x2 ˜
Á 2. ˜ Ë c¯ ÁË c c ˜¯
= tan Á
-1 5 ˜
Á Ê 1ˆ 2 ˜ Ê ax ˆ Ê b 2 x 2 bx ˆ
Á1- Á ˜ ˜ = x - - - x 2
fi ÁË ˜¯ Á Á 1 1 ˜
Ë Ë 5¯ ¯ c Ë c2 c ˜¯
Ê2 ˆ Ê a xˆ Ê x 2 bx ˆ
˜ fi ÁË ˜¯ = ÁË c - b2 x2 - 1 - x2 ˜
-1 Á5 c c c ¯
= tan Á
25 - 1 ˜
Á
Ë 25 ¯
˜
fi xÊ
Ë ( c -b x
2 2 2
)
- b 1 - x 2 - aˆ = 0
¯
Ê 2 25 ˆ
= tan -1 Á ¥ ˜ fi x = 0, c - b x = b 1 - x + a
2 2 2 2
Ë 5 24 ¯
Thus, x
-1 Ê 5 ˆ
= tan ÁË ˜¯ c2 - b2 x2 = b 1 - x2 + a
12

sin Á -1 Ê
142 ˆ fi ( )
c 2 - b 2 x 2 = 2ab 1 - x 2 + b 2 1 - x 2 + a 2
Ë 65 5 ˜¯
fi c 2 - b 2 x 2 = 2ab 1 - x 2 + b 2 - b 2 x 2 + a 2
Ê 142 ˆ
= tan -1 Á
Ë 31 ˜¯ fi c 2 = 2ab 1 - x 2 + b 2 + a 2
fi c 2 = 2ab 1 - x 2 + c 2
Ê 1ˆ Ê 1ˆ Ê 142 ˆ
3 tan -1 Á ˜ + 2 tan -1 Á ˜ + sin -1 Á fi 2ab 1 - x 2 = 0
Ë 2¯ Ë 5¯ Ë 65 5 ˜¯
-1 Ê 11ˆ 5ˆ -1 Ê
Ê 142 ˆ
fi (1 - x ) = 0
2

= tan Á ˜ + tan Á ˜ + tan -1 Á


Ë 2¯ Ë 12 ¯ Ë 31 ˜¯ fi x = ±1

Ê 11 5 ˆ
{-1, 0,1}
+ Ex-49 x
-1 Á 2 12 ˜ -1 Ê 142 ˆ
= tan Á 11 5 ˜ + tan ÁË 31 ˜¯ ( ) (
cos -1 x 6 + cos -1 3 3 x 2 = ) p
Á1- . ˜ 2
Ë 2 12 ¯
288 Comprehensive Trigonometry with Challenging Problems & Solutions for Jee Main and Advanced

-1 Ê ( a + b ) x ˆ Ê (c + d ) x ˆ
p
( )
Soln. Given cos -1 x 6 + cos -1 3 3 x 2 = ( ) fi tan Á 2 ˜ = cot -1 Á 2
2 Ë x - ab ¯ Ë x - cd ˜¯
fi sin -1 ( 1 - 6x ) + cos
2 -1
(3 )
3 x2 =
p
2
Ê ( a + b ) x ˆ Ê x 2 - cd ˆ
fi Á 2 =
Ë x - ab ˜¯ ÁË (c + d ) x ˜¯
1 - 6 x2 = 3 3 x2
fi 1 - 6 x = 27 x
2 4 fi (x 2
)( )
- ab x 2 - cd = ( a + b ) (c + d ) x 2

fi 27 x 4 + 6 x 2 - 1 = 0 fi x 4 - ( a + b + cd ) x 2 + abcd = (a + b ) (c + d ) x 2

fi x - ( a + b + cd + (a + b ) (c + d )) x + abcd = 0
4 2
fi 27 x 4 - 9 x3 + 9 x3 - 3x 2 + 9 x 2 - 1 = 0
fi 9 x3 (3x - 1) + 3x 2 (3x - 1) + (3x - 1) (3x + 1) = 0 since x1 , x2 , x3 , x4
fi ((3x - 1)) (9 x 3
)
+ 3x + (3x + 1) = 0
2 equation, we have
x1 + x2 + x3 + x4 = 0
(3x - 1) (3x (3x + 1) + (3x + 1)) = 0
2
fi  x1 x2 = (ab + cd + (a + b ) (c + d ))
fi (3x - 1) (3x + 1) (3x + 1) = 0
2
 x1 x2 x3 = 0
1 1 i  x1 x2 x3 x4 = abcd
fi x = - , ,±
3 3 3 4

Ï 1 i ¸
(i) Â xi = x1 + x2 + x3 + x4 = 0
̱ , ± ˝. i =1
Ó 3 3˛ 4 Ê 1ˆ
(ii) Â Á ˜
Ex-50. Let x1 , x2 , x3 , x4 i =1 Ë xi ¯

1 1 1 1
Ê aˆ Ê bˆ Ê cˆ Êdˆ p = + + +
tan -1 Á ˜ + tan -1 Á ˜ + tan -1 Á ˜ + tan -1 Á ˜ = x1 x2 x3 x4
Ë x¯ Ë x¯ Ë x¯ Ë x¯ 2
x2 x3 x4 + x1 x3 x4 + x1 x2 x4 + x1 x2 x3
=
4
x1 x2 x3 x4
(i) Â xi = 0 0
i =1 = =0
abcd
4 Ê 1ˆ 4
(ii) Â Á ˜ = 0 (iii) ’ ( xi )
i =1 Ë xi ¯ i =1
4
(iii) ’ ( xi ) = abcd = x1 x2 x3 x4
i =1 = abcd
(iv) ’ ( x1 + x2 + x3 ) = abcd (iv) ’ ( x1 + x2 + x3 )
Soln. Given equation is = ( x1 + x2 + x3 ) ( x1 + x2 + x4 )
Ê aˆ Ê bˆ Ê cˆ Êdˆ p ( x1 + x3 + x4 ) ( x2 + x3 + x4 )
tan -1 Á ˜ + tan -1 Á ˜ + tan -1 Á ˜ + tan -1 Á ˜ =
Ë x¯ Ë x¯ Ë x¯ Ë x¯ 2 = ( - x4 ) ( - x3 ) ( - x2 ) ( - x1 )
Ê a b ˆ Ê c d ˆ = x1 x2 x3 x4
+ +
x x ˜
-1 Á
+ -1 Á x x ˜=p = abcd.
fi tan Á a b˜
tan Á cd˜ 2
Á1- . ˜ Á1- ˜ Ex-51. Let cos -1 ( x ) + cos -1 ( 2 x ) + cos -1 (3x ) = p
Ë x x¯ Ë x x¯ If x
-1 Ê ( a + b ) x ˆ Ê (c + d ) x ˆ p ax3 + bx 2 + cx - 1 = 0
fi tan Á 2 ˜ + tan -1 Á 2 =
Ë x - ab ¯ Ë x - cd ˜¯ 2 of ( a + b + c + 2) .
-1 Ê ( a + b ) x ˆ p Ê (c + d ) x ˆ Soln. We have cos -1 ( x ) + cos -1 ( 2 x ) + cos -1 (3x ) = p
fi tan Á 2 ˜ = - tan -1 Á 2
Ë x - ab ¯ 2 Ë x - cd ˜¯ cos -1 ( 2 x ) + cos -1 (3x ) = p - cos -1 ( x )
Inverse Trigonometric Function 289

cos -1 ( 2 x ) + cos -1 (3x ) = cos -1 ( - x ) x


Ê 1ˆ
tan -1 ( x ) + tan -1 Á ˜ =
Ë x¯
cos -1 Ê2 x.3x -
Ë (1 - 4 x ) (1 - 9 x ) ˆ¯ = cos
2 2 -1
(- x ) p p
(a)
(1 - 4 x ) (1 - 9 x ) = - x
2 2
6 x2 - 2 2

(6 x + x) = (1 - 4 x ) (1 - 9 x )
2 2 2
x + sin y =
2p
, then cos x + cos y is
3
(6 x + x) = (1 - 4 x ) (1 - 9 x )
2 2 2 2
2p p
(a)
3 3
36 x 4 + 12 x3 + x 2 = 1 - 13x 2 + 36 x 4
p
(c) (d) p
12 x3 + 14 x 2 - 1 = 0 6
Thus, a = 12, b = 14, c = 0 p
f (x) = sin x + cos x. Then
( a + b + c + 2) 2
Ê 1ˆ
(a) f Á ˜
Ë 2¯
Ê1 Ê 4ˆ ˆ
( )
Ex-52. If x = sin 2 tan -1 2 , y = sin Á tan -1 Á ˜ ˜
Ë2 Ë 3¯ ¯ f (k k keR
y2 = 1 - x . Ê 1 ˆ
(c) f Á ,keR
Ë 1 + k 2 ˜¯
(
Soln. We have x = sin 2 tan -1 2 ) (d) f
fi x = sin ( 2 q ) , tan 2 = q -1

2 tan q
fi x= , tan q = 2
1 + tan 2 q
x – b cos x = c
4
fi x= a sin x + b cos x is
5 p ab + c(b - a)
Ê1 Ê 4ˆ ˆ a+b
y = sin Á tan -1 Á ˜ ˜
Ë2 Ë 3¯ ¯ p
(c) p ab - c(b - a) (d)
1 Ê 4ˆ a+b 2
fi y = sin (q ) , tan -1 Á ˜ = q
2 Ë 3¯
p
4 sin x x=
fi y = sin (q ) , tan (2q ) = 2
3
1
fi y = sin (q ) , tan (q ) =
2
1
fi y= Ê1- xˆ
5 tan ÁË ˜ £ x £1
1+ x¯
p
Hence, y2 =
1 4
=1- =1- x p
5 5 4
p p p p
(c) – , (d) ,
LEVEL II 4 4 4 2
(MIXED PROBLEMS)
Ê 3ˆ
k x – kx + sin x – cos x = cos Á ˜ has
x is Ë 2 ¯

(c) R (d) None of these.


290 Comprehensive Trigonometry with Challenging Problems & Solutions for Jee Main and Advanced

-p £ x £ 2p , then cos (cos x) is Êp 1 ˆ Êp 1 ˆ


(a) x p-x tan Á + cos -1 x˜ + tan Á - cos -1 x˜ ,
Ë4 2 ¯ Ë4 2 ¯
(c) 2p + x (d) 2p - x xπ
Ê 1ˆ p (a) x x
sin -1 x + cot -1 Á ˜ = , then x
Ë 2¯ 2 2 x
(c) (d)
1 x 2
5 cot -1 (3) + cosec-1 ( 5 ) is
2 3
(c) (d) p p
5 2 (a)
2 3
{sin (cot 3 y, p p
y is (c) (d)
4 6
4 2
(a) y = y= 2n 2n
5 5  sin -1 xi = np , then  xi is
i =1 i =1
2 3
(c) y = – (d) y = (a) n n
5 2
n(n + 1) n(n - 1)
1 (c) (d)
x= x x) is 2 2

( ) ( )
5
u = cot -1 tan a - tan -1 tan a ,
24 24
(a) Ê p uˆ
25 25 then tan Á - ˜
Ë 4 2¯
1 1
(c) (d) –
5 5 (a) tan a cot a
-1 Ê 1 ˆ -1 Ê 1 ˆ (c) tan a (d) cot a
tan Á ˜ + tan Á ˜
Ë 2¯ Ë 3¯ Ê a ˆ Ê b ˆ
tan -1 Á + tan -1 Á ,
p p Ë b + c ˜¯ Ë a + c ˜¯
(a)
4 2 if –C ABC is
p p p
(c) (d) None of these (a)
3 4 3
a + tan b a b ab p
(c) (d) p
Ê a+bˆ a+bˆ
-1 Ê 2
(a) tan -1 Á -p
Ë 1 - ab ˜¯
tan Á
Ë 1 - ab ˜¯
Ê nˆ p
Ê a+bˆ Ê a+bˆ cot -1 Á ˜ > , n e N
(c) p + tan -1 Á (d) p - tan -1 Á Ëp¯ 6
Ë 1 - ab ˜¯ Ë 1 - ab ˜¯
‘n’ is
p
tan -1 (1 + x ) + tan -1 (1 - x ) = is
2 x x
(a) x x Ê 1 ˆ
(c) x x=p x x e Á 0, ˜
Ë 2¯
p and q
3p Ê 1 ˆ
cos -1 ( ) p + cos -1 ( )
1 - p + cos -1 ( )
1- q =
4
(c) Á ,1
Ë 2 ˜¯
(d) x

Ê1 Ê 1ˆ ˆ
(a) p q q p cos Á cos -1 Á ˜ ˜
Ë2 Ë 8¯ ¯
p q
3 3
(a)
4 4
Inverse Trigonometric Function 291

1
(c)
Ê 1ˆ Ê 3 ˆ
16 tan -1 x + cot -1 Á ˜ = sin -1 Á is
x Ë y¯ Ë 10 ˜¯
Ê Ê 1 ˆˆ
( )
tan sec -1 x = sin Á cos -1 Á ˜ ˜ is
Ë Ë 5¯¯
È1 ÏÔ Ê Ê 63 ˆ ˆ ¸Ô˘
(a) ± 5 ±
3 cos Í cos Ìcos Á sin -1 Á ˜ ˜ ˝˙ is
ÍÎ 2 ÓÔ Ë Ë 8 ¯ ¯ Ô˛˙˚
3 5
3 3 3
(c) ± 3 (d) ± (a)
5 5 16 8
3 3
5p 2 (c) (d)
(tan x) + (cot x)
-1 2 -1 2
= x is 4 2
8
tan -1 x + tan -1 y + tan -1 z = p , then the
1 1 1
-1 -1 + + is
cos x + cos 2 x = -p is yz zx xy

1
(c) (d) xyz
a, b, c xyz
Ê a (a + b + c ) ˆ Ê b (a + b + c ) ˆ
-1 -1
˜ + tan Á Ê 1ˆ
q = tan Á ˜ x tan -1 Á ˜ is
Ë bc ¯ Ë ac ¯ Ë x¯
Ê c (a + b + c ) ˆ (a) cot -1 ( x ) cot -1 ( x )
˜,
-1
+ tan Á
Ë ba ¯ (c) -p + cot -1 ( x ) (d) None
tanq is
sin -1 x + cos -1 y + sin -1 z = 2p , is

x
2
tan sin ( -1
)
x > 1 is x 2 + y 2 + z 2 = r 2 , then
È 1 1 ˘ Ê zx ˆ
Í- , ˙ Ê xy ˆ Ê zy ˆ
Î 2 2˚ tan -1 Á ˜ + tan -1 Á ˜ + tan -1 Á ˜
Ë zr ¯ Ë xr ¯ Ë yr ¯
È 1 1 ˘ Ê 1 1 ˆ (a) p p
(c) (-1,1) - Í- , ˙ (d) [-1,1] - ÁË - , ˜
Î 2 2˚ 2 2¯ tan x + tan 2 x + tan -1 3x = p , then
-1 -1

x is
f
( )
ax 2 + sin -1 x 2 - 2 x + 2 + cos -1 x 2 - 2 x + 2 = 0 ( )
( )
1 + x 2 + 2 x sin cos -1 y = 0 is
(a) p p
p p
a
È Ê Ê 2 - 3ˆ ˆ˘ x3 + bx 2 + cx + 1 = 0 (b < c )
-1 Ê 12 ˆ
sin -1 Ícot Á sin -1 Á ˜ + cos Á 4 ˜ + sec -1
2 ˜˙
Í ÁË Ë 4 ¯ Ë ¯ ˜¯ ˙ Ê 1ˆ
Î ˚ of tan -1 a + tan -1 Á ˜
Ëa¯
is
(a) a p
p
(c) p p a, b, g x3 + px 2 + 2 x + p = 0 , the
tan -1 a + tan -1 b + tan -1 g is
292 Comprehensive Trigonometry with Challenging Problems & Solutions for Jee Main and Advanced

(a) np np
(3p )3
(sin x) + (sin y ) + (sin z )
3 3 3
n p p. -1 -1 -1
=
8
( (
Èsin -1 cos -1 sin -1 tan -1 x
ÍÎ ( ))) ˘ =1,
˙˚
(3 x + 4 y - 5 z + 2 ) .
x n
Ê 1ˆ
S = Â cot -1 Á 2r +1 + r ˜ ,
r =1 Ë 2 ¯
lim ( S ) .
nƕ

Ê Ê n Ê 4 ˆˆˆ
Lim Á tan Á Â tan -1 Á 2 .
LEVEL III nÆ• Ë Ë r =1 Ë 4r + 3 ˜¯ ˜¯ ˜¯
(PROBLEMS FOR JEE ADVANCED)

Ê 2x ˆ
( ( ))
sin -1 cos sin -1 x + cos -1 sin cos -1 x ( ( )) 2 sin -1 Á
Ë 1 + x 2 ˜¯
= p x3 .

Ê xˆ Ê yˆ
cos -1 Á ˜ + cos -1 Á ˜ = a
tan -1
{cosec (tan x) - tan (cot x)}
-1 -1 1
= tan -1 x
2
Ë a¯ Ë b¯

where x π 0 . x2 2 xy y2
- cos a + 2 = sin 2 a .
a2 ab b

(
tan tan -1 x + tan -1 y + tan -1 z ) sin -1 x + sin -1 y + sin -1 z = p ,

(
= cot cot -1 x + cot -1 y + cot -1 z . )
x 1 - x 2 + y 1 - y 2 + z 1 - z 2 = 2 xyz .

( ( ( ))) = x "x Œ(0,1] .


sin cot -1 tan cos -1 x
( ) ( )
3 3
f ( x ) = sin -1 x + cos -1 x .
sin (cosec (cot ( tan x ))) = x
-1 -1
p
x sin -1 x + sin -1 2 x = .
"x Œ(0,1] 2
x
Ê 1 ˆ Ê 1 ˆ Ê 2ˆ
tan -1 Á ˜ + tan -1 Á ˜ = tan -1 Á 2 ˜
sin -1 (sin 5) + cos (cos10) + tan ( tan ( -6)) Ë 1 + 2x ¯ Ë 1 + 4x ¯ Ëx ¯
-1 -1

x
+ cot -1 (cot ( -10))
tan ( x - 1) + tan -1 ( x ) + tan -1 ( x + 1)
-1

= tan -1 (3x )
Êx 3 - 3x 2 ˆ Ê1 ˆ
cos -1 x + cos -1 Á + ˜ , " x ŒÁ ,1˜ -1 Ê 1 ˆ -1 p
ÁË 2 2 ˜¯ Ë2 ¯ x sin Á ˜ + cos x = .
Ë 5¯ 4

Ê x 2 - 1ˆ Ê 2 x ˆ 2p
Ê 5-2 6ˆ x cos -1 Á 2 ˜ + tan -1 Á 2 ˜ = .
Ê 1 ˆ Ë x + 1¯ Ë x - 1¯
tan -1 Á - tan -1
Á ˜
3
Ë 2 ˜¯ ÁË 1 + 6 ˜¯
x
Ê 1 - a2 ˆ -1 Ê 1 - b ˆ
( ) ( ) ( )
2
m = sin -1 a 6 + 1 + cos -1 a 4 + 1 - tan -1 a 2 + 1 , 2 tan -1 x = cos -1 Á ˜ - cos Á ˜,
x+y=m Ë 1 + a2 ¯ Ë 1 + b2 ¯
y a b
Inverse Trigonometric Function 293

x
( )
-1 -1
cot -1
x + cot -1 n 2 - x + 1 = cot -1 (n - 1) . f ( x ) = 8sin x
+ 8cos x

x k
Ê x - 1ˆ Ê 2 x - 1ˆ Ê 23 ˆ x - kx + sin (sin 4) > 0
2 -1
x.
tan -1 Á + tan -1 Á = tan -1 Á ˜
Ë x + 1˜¯ Ë 2 x + 1˜¯ Ë 36 ¯
(
A = 2 tan -1 2 2 - 1 and )
x
Ê 1ˆ Ê 3ˆ
Ê xˆ Ê xˆ B = 3 sin -1 Á ˜ + sin -1 Á ˜ ,
sec-1 Á ˜ - sec -1 Á ˜ = sec -1 b - sec -1 a . Ë 3¯ Ë 5¯
Ë a¯ Ë b¯
A B.
• 8n
Ê ˆ Ï Ï Ê 2- 3ˆ ¸¸
 tan -1 ÁË ˜. -1 Ê 12 ˆ
n =1 n - 2n 2 + 5 ¯
4 Ô Ô
sin -1 Ìcot Ìsin -1 Á
Á 4
˜ + cos Á
˜ Ë 4 ˜ + sec
¯
-1
( 2 )Ô˝Ô˝
ÔÓ ÓÔ Ë ¯ Ô˛Ô˛

p
( )
sin -1 e x + cos -1 x 2 = ( ) 2
.
(
f ( x ) = sin -1 cos -1 x + tan -1 x + cot -1 x )
Ê xˆ Ê yˆ 2p
sin -1 Á + sin -1 Á 1 - ˜ + tan -1 y =
sin ( x ) = cos -1
(cos x ) in (0, 2p ) . Ë 2 ˜
¯ Ë 4 ¯ 3

Ê 1ˆ Ê 1ˆ Ê 1ˆ Ê 1ˆ p
tan -1 Á ˜ + tan -1 Á ˜ + tan -1 Á ˜ + tan -1 Á ˜ = (x 2
+ y2 + 1 .)
Ë 3¯ Ë 4¯ Ë 5¯ Ë n¯ 4
n ŒN n. ( x, y )
a x3 + b x 2 + c x + 1 = 0 Ê 1ˆ Ê 1ˆ Ê 1ˆ
tan -1 Á ˜ + tan -1 Á ˜ = tan -1 Á ˜ .
Ë x¯ Ë y¯ Ë 10 ¯
b<c
Ê 1ˆ È Ê 10 ˆ˘ a
tan -1 (a ) + tan -1 Á ˜ .
Ëa¯
Î Ë k =1
-1
Ícot Á Â cot k + k + 1 ˜ ˙ =
2
¯˚ b
( )
x3 + b x 2 + c x + 1 = 0 a and b
a
(a + b + 10) .
p q pr qr
2 tan -1
(coseca ) + tan -1 (2 sin a sec2 a ) . Ê p-qˆ Ê q-rˆ Ê r - pˆ
tan -1 Á + tan -1 Á + tan -1 Á =p .
˜
Ë 1 + pq ¯ ˜
Ë 1 + qr ¯ Ë 1 + rp ˜¯
sin -1 x > cos -1 x
(sin x) + (cos x)
-1 3 -1 3
cos -1 x > sin -1 x = ap 3

(cot x) - 5 (cot x) + 6 > 0


-1 2 -1

Ê x2 ˆ
tan (sin x ) > 1
2 -1
f ( x ) = cot -1 Á 2 ˜
Ë x + 1¯
4 ( tan x ) - 8 ( tan x ) + 3 < 0
2
-1 -1 Êb ˆ
is ( a, b ) ÁË + 2˜¯ .
a
4 cot x - (cot x ) - 3 ≥ 0
-1 -1 2

Ê Ê p ˆˆ
tan -1 y = 4 tan -1 x , Á x < tan Á ˜ ˜ y as an
Ê Ê 2x + 4ˆ ˆ
-1
2 Ë Ë 8 ¯¯
sin Á sin Á 2 ˜˜
<p -2 Êpˆ
Ë Ë 1+ x ¯¯ x tan Á ˜
Ë 8¯
x4 - 6 x2 + 1 = 0 .
{ (sin x )}
2
f ( x ) = sin -1
- sin -1
(sin x )
294 Comprehensive Trigonometry with Challenging Problems & Solutions for Jee Main and Advanced

INTEGER TYPE QUESTIONS


Ê a(a + b + c) ˆ Ê b( a + b + c ) ˆ
tan -1 Á ˜ + tan -1 Á ˜
Ë bc ¯ Ë ac ¯ Ê Ê 2 x2 + 4 ˆ ˆ
sin -1 Á sin Á 2 ˜ ˜ < p - 3 is
Ê c(a + b + c) ˆ Ë Ë x +1 ¯¯
+ tan -1 Á ˜ =p a, b, c
Ë ab ¯ (a, b) , where a, b Œ I (b - a + 5 ) .
Ê 3 sin 2q ˆ . a sin -1 x - b cos -1 x = c
( 1
q = tan -1 2 tan 2 q - sin -1 Á
2
)
Ë 5 + 4 cos 2q ˜¯
a sin -1 x + b cos -1 x
Ê x cos q ˆ Ê cos q ˆ . m p a b + c (a - b)
tan -1 Á - cot -1 Á is , m Œ N , then
˜
Ë 1 - x sin q ¯ Ë x - sin q ˜¯ a+b
(m 2
+m+2 . )
-1 Ê
x - 1ˆ Ê 2x ˆ Ê 2 x ˆ 2p
2
cos Á 2 ˜ + sin -1 Á 2 ˜ + tan -1 Á 2 ˜ = .
Ë x + 1¯ Ë x + 1¯ Ë x - 1¯ 3 m x 2 + 3x + 1 = 0 ,
Ê 1ˆ
tan -1 (m ) + tan -1 Á ˜
Ê xz ˆ Ë m¯
Ê yz ˆ Ê xy ˆ p
tan -1 Á ˜ + tan -1 Á ˜ + tan -1 Á ˜ = . kp
Ë xr ¯ Ë yr ¯ Ë zr ¯ 2 is , k ŒI ( k + 4) .
2
x2 + y 2 + z 2 = r 2 .
p
Ê ˆ -1 Ê m ˆ
( ) ( )
10 3
 tan -1 ÁË ˜¯ = cot ÁË ˜¯ sin -1 x 2 - 2 x + 1 + cos -1 x 2 - x = .
r =1 9r + 3r - 1 2
n 2
m and n
cos -1 ( x ) + cos -1 ( 2 x ) + cos -1 (3x ) = p .
of ( 2m + n + 4) .
If x
10 10 aˆ -1 Ê
  tan ÁË ˜¯ = m p a x3 + b x 2 + c x + d = 0
b =1 a =1 b
(b + c ) - ( a + d ) .
of ( m + 4) .
a, b, g x3 - x 2 - 3x + 4 = 0
( x + 1)
f ( x) =
1
p
(
sin -1 x + cos -1 x + tan -1 x + 2 )
x + 2 x + 10
such that tan -1 a + tan -1 b + tan -1 g = q .
p
f ( x ) is m, then tan (q ) is p and q
q
(104 m - 90) . ( p + q) .
m
sin ( 2 x ) + cos ( 2 x ) + cos x + 1 = 0 in M
cos -1 x + cos -1 ( 2 x ) + p = 0 and N
p and
0< x< x
2
Ê 5ˆ Ê 12 ˆ p
È Ê Ê 7p ˆ ˆ Ê Ê 7p ˆ ˆ ˘ sin -1 Á ˜ + sin -1 Á ˜ =
n = sin Í tan -1 Á tan Á ˜ ˜ + cos -1 Á cos Á ˜ ˜ ˙ Ë x¯ Ë x¯ 2
Î Ë Ë 6 ¯¯ Ë Ë 3 ¯¯ ˚ of M + N
(m 2
+ n2 + m + n + 4 . )
È ˆ˘
Ên
Ê 1ˆ ˆ
f (n ) = Â Á cot -1 Á ˜ - tan -1 (k )˜ Î
Ê1
4 cos Ícos -1 Á
Ë4 ( ˆ
¯ ) Ê1
6 - 2 ˜ - cos -1 Á
Ë4 ( )
6 + 2 ˜˙ .
¯˚
k =- n Ë Ë k¯ ¯
10
such that  ( f (n ) + f (n - 1)) = a p Ê 5 ˆ
n=2 Ë k =1
(
5 cot Á Â cot -1 k 2 + k + 1 ˜
¯
)
(a + 1) .
Inverse Trigonometric Function 295

p sin -1 x + cos -1 x is
3 sin -1 (log 2 x ) + cos -1 (log 2 y ) =
2 p 3p
(a)
11p
and sin -1 (log 2 x ) + 2 cos -1 (log 2 y ) =
2 2
6 5p 7p
(c) (d)
Ê 1 1 ˆ 2 2
Á 2 + 2 + 2˜
Ëx y ¯ È 5p 3p ˘
sin -1 x is Í- , - ˙ such that
a and b x 2 + 5 x - 44 = 0 , Î 2 2˚
5p
(
cot cot -1 a + cot -1 b . ) sin -1 x + cos -1 x =
2
x and y
cos -1 x is
Ê 1ˆ Ê 1ˆ Ê 1ˆ
tan -1 Á ˜ + tan -1 Á ˜ = tan -1 Á ˜ (a) [4p , 5p ] [3p , 4p ]
Ë x¯ Ë y¯ Ë 7¯
x, y). (c) [6p , 7p ] (d) [5p , 6p ]
COMPREHENSIVE LINK PASSAGE PASSAGE II

f AÆB g BÆ y) =
x f (x) = y. The function g B Æ A
f AÆB f . Thus,
PASSAGE 1 we have f (x) = y fi f (y) = x
Function Domain Co-domain
È p 3p ˘
sin x Í2 , 2 ˙
Î ˚
Ê p 3p ˆ
tan x R ÁË , ˜¯
2 2
(sin q) = q q
cos x p p È p p˘
cot x R [p p p Í- 2 , 2 ˙
Î ˚
sin -1 ( - x ) is È p ˘
-1
(c) Í- , 0˙ (d) None of these
(a) - sin x Î 2 ˚
p + sin -1 x (cos q) = q q
Ê p pˆ
(c) 2p - sin -1 x p ÁË - , ˜¯
2 2
(d) 2p - cos -1 1 - x 2 , x > 0 È p p˘
(c) Í- , ˙ (d) None of these
Î 2 2˚
f ( x ) = 3 sin x - 2 cos x , then f (x) is
-1 -1

(a) even function (tan q) = q q


È p p˘
p Í- 2 , 2 ˙
Î ˚
Ê p pˆ
(c) Á - , ˜ (d) None of these
(sin x) - (cos x)
-1 3 -1 3
is Ë 2 2¯
(cosec q) = q q
(a) - 63 p 63 p 3
3

8 8 È p p˘ Ê p pˆ
(a) Í- , ˙ ÁË - , ˜¯
Î 2 2˚
(c) 125 p (d) - 125 p
3 3 2 2
32 32 p p)
296 Comprehensive Trigonometry with Challenging Problems & Solutions for Jee Main and Advanced

(sec q) = q
p
Ïp ¸
Ì ˝
q
Ïp ¸
q) – Ì ˝
(c) cos -1 ( x2 - 1 )
p)
Ó2˛ Ó2˛
(d) None of these
(d) p - cos -1 ( 1- x ) 2

(sinx) = x x x+
1
=2 sin -1 x is
(a) R (B) j x
È p 3p ˘ p p
(a)
Í2 , 2 ˙ 4 2
Î ˚
3p
(c) p (d)
sin -1 (sin 2) + cos -1 (cos 2) is 2
p PASSAGE V
2 ap 2
( )
2
p Let cos -1 x + sin -1 y = ..........(i)
(c) – (d) None of these. 4
2
p2
( )
2
and cos -1 x . sin -1 y = .........(ii)
PASSAGE III 16

{ }
Let f ( x ) = sin cot -1 ( x + 1) - cos tan -1 x and ( ) £ x, y £
a
( ( (
a = cos tan -1 sin cot -1 x ))) & b = cos (2 cos -1
x + sin -1 x )
Ê 4ˆ È 4ˆ
x f (x (a) Á 0, 2 + ˜ Í0, 1 + p ˜¯
Ë p¯ Î
È 4ˆ
(c) R (d) Í0, - 1 + ˜
f (x a Î p¯
a

26 (a) (-•, 2] » [2, • ) (-2, 2)


a2 = , then b
51 È 4˘
(c) Í2, 1 + ˙ (d) R
Î p˚
x and y
a’
PASSAGE IV
ÏÔ Ê p 2 ˆ ¸Ô ÏÔ Ê p 2 ˆ ¸Ô
f AÆB (a) Ìcos Á ˜ , 1˝ Ìcos Á ˜ , - 1˝
g BÆA g(y) = x if and ÔÓ Ë 4 ¯ Ô˛ ÔÓ Ë 4 ¯ Ô˛
f (x) = y. ÏÔ Ê p 2 ˆ ¸Ô
BÆA (c) Ìcos Á ˜ , ± 1˝ (d) {( x, y ) : x Œ R, y Œ R}
f AÆB f ÔÓ Ë 4 ¯ Ô˛

MATCH-MATRIX
{
cos tan -1 ( tan 2) is }
5 5

x x is
(a) cos ( 1- x )
-1 2 Note:

- cos ( 1 - x )
-1 2
Inverse Trigonometric Function 297

Ê 1 ˆ Ê 1 ˆ p
(D) sin -1 Á + cos -1 Á is (S)
Column - I Column - II Ë 2013 ˜¯ Ë 2013 ˜¯ 2

sin -1 (sin 20) (20 - 6p ) Column - I Column - II


3p
sin -1 (sin 10) is (Q) (3p - 10) 4
tan -1 1 + tan -1 2 + tan -1 3 is
p
cos -1 (cos 10) is (R) (4p - 10) 2
Ê 1ˆ Ê 1ˆ
tan -1 1 + tan -1 Á ˜ + tan -1 Á ˜ is
Ë 2¯ Ë 3¯
cos -1 (cos 20) is (S) (5p - 20) p
Ê 5ˆ
Column - I Column - II tan -1 (9) + tan -1 Á ˜ is
Ë 4¯
(0, p ) p
-
f ( x ) = 3 sin -1 -1
x + 2 cos x Ê 2x ˆ
2
2 tan -1 x - tan -1 Á , x > 1 is
is Ë 1 - x 2 ˜¯
È p 3p ˘
f (x) = sin x (Q) Í2 , 2 ˙
Î ˚ Column - I Column - II
+ cos x + tan -1 x is
-1

(sin x) + (cos x) 1
-1 3 -1 3
(A) x=
Ê p 3p ˆ 2
f ( x ) = sin -1 x + p is (R) ÁË - , ˜
2 2¯
(sin x) + (cos x)
-1 2 -1 2
(B)
f ( x ) = 2 tan -1 x + sin -1 x is minimum at (Q) x
Ê 1ˆ
+ sec-1 Á ˜ is
(C) (sin x) - (cos x)
-1 -1

Ë x¯ (S) [0, p ] is minimum at (R) x

(tan x) + (cot x)
-1 2 -1 2
(D)
Column - I Column - II
is minimum at (S) x
(A) sin (sin x) = sin (sin x), if
-1 £ x £ 1 ASSERTION AND REASON
(B) cos (cos x) = cos (cos x), if
(Q) 0 £ x £ 1
(C) tan (tan x) = tan (tan x), if
(R) 0 < x < p
(D) cot (cot x) = cot (cot x), if
(R) - p £ x £ p .
2 2
3p
Column - I Column - II If sin -1 x + sin -1 y + sin -1 z = ,
2

-1 Ê 1 ˆ 1ˆ
(A) sin Á ˜ + cos Á - ˜ -1 Ê 7p (x 2013
+ y 2013 + z 2013 )
Ë 2¯ Ë 2¯ 6 9
-
-1 Ê
1ˆ Ê 1ˆ
(B) sin Á - ˜ + cos -1 Á ˜ is (Q)
5p (x 2014
+y 2014
+ z 2014 )
Ë 2¯ Ë 2¯ p
6 sin -1 x is
2
p
(C) tan -1 ( 3 ) + cot (- 3 ) is
-1
(R)
6 (c) C (d) D
298 Comprehensive Trigonometry with Challenging Problems & Solutions for Jee Main and Advanced

Ê 2x ˆ
2 tan -1 x - tan -1 Á is p ( )
cos -1 4 x3 - 3x = 2p - 3 cos -1 ( x )
Ë 1 - x 2 ˜¯
1 1
x - £x<
2 2
(c) C (d) D
Ê 1ˆ
cot -1 ( x ) = tan -1 Á ˜ , x > 0
Ê 1ˆ p Ë x¯
tan -1 ( p ) + tan -1 Á ˜ is -
Ë p¯ 2 Ê 1ˆ
cot -1 ( x ) = p + tan -1 Á ˜ , x < 0
P x + 2013 x + 2014 = 0 .
2 Ë x¯

2p If a, b x 2 - 3x + 2 = 0 , then sin -1 a
-1 -1
If sin x + sin y =
3 ut not sin -1 b a >b
p
cos -1 x + cos -1 y is sin -1 x is [ -1, 1]
3
p
sin -1 x + cos -1 x = , when x Œ[-1,1]
2
QUESTIONS ASKED IN PAST IIT-JEE EXAMS
a, b, c
cos -1 (cos10) is ( 2p - 5) a (a + b + c )
q = tan -1
bc
cos -1 x is [0, p ] b (a + b + c ) c (a + b + c )
+ tan -1 + tan -1 .
ca ab
Then tan q
If cos -1 x + cos -1 y + cos -1 z = p ,
then x 2 + y 2 + z 2 + 2 xyz = 1 Ï Ê 1ˆ p ¸
tan -1 Ì2 tan -1 Á ˜ - ˝
-1 £ x, y, z £ 1 . Ó Ë 5¯ 4 ˛

(
cos 2 cos -1 x + sin -1 x at x )
p
If tan -1 ( 2 x ) + tan -1 (3x ) = , then p p
4 0 £ cos -1 x £ p and - £ sin -1 x £ .
1 2 2
x is .
6
0 < 2 x, 3 x < 1 È Ê 4ˆ Ê 2ˆ ˘
tan Ícos -1 Á ˜ + tan -1 Á ˜ ˙ is
Î Ë 5¯ Ë 3¯ ˚

Ê 1 - x2 ˆ -1
cos Á ˜ = 2 tan x
Ë 1 + x2 ¯
Ê Ê 2p ˆ ˆ
x≥0 sin -1 Á sin Á ˜ ˜ is
Ë Ë 3 ¯¯
2p 2p p 5p
( )
sin -1 3x - 4 x3 = p - 3 sin -1 x (a) -
3 3
(c)
3
(d)
3
1
< x £1
2
Inverse Trigonometric Function 299

(
A = 2 tan -1 2 2 - 1 ) (B) If a b x, y)

-1 Ê 1 ˆ -1 Ê 3 ˆ
( x - 1) ( y - 1) = 0
2 2

and B = 3 sin Á ˜ + sin Á ˜ is....... (C) If a b x, y)


Ë 3¯ Ë 5¯
y=x
(D) If a b x, y)
(4 x - 1) ( y - 1) = 0
2 2

p
tan -1 x ( x + 1) + sin -1 x 2 + x + 1 = is
2 x

{ )}
1/ 2
È
( ) ( ˘
2
• 1 + x 2 ¥ Í x cos cot -1 x + sin cot -1 x - 1˙
Î ˚

Ê x 2 x3 ˆ
sin -1 Á x - + - .......˜ x
Ë 2 4 ¯ (a) x
1 + x2
Ê x 4 x6 ˆ p
+ cos -1 Á x 2 - + - ....˜ = , (c) x 1 + x 2 (d) 1 + x2
Ë 2 4 ¯ 2

0 < x < 2 , then x is


Ê Ê sin q ˆ ˆ
f (q ) = sin Á tan -1 Á
Ë Ë cos 2q ˜¯ ˜¯
p p
cos tan( (sin (cot x))) =
-1 -1 x2 + 1
x2 + 2
.
-
4
<q <
4
d
d ( tanq )
( f (q ))
p
f ( x ) = sin -1 ( 2 x ) + is
6
Ê 23 Ê n ˆˆ
cot Á Â cot -1 Á1 + Â 2k ˜ ˜ is
Ê 1 1˘ È 1 3ˆ Ë n =1 Ë k =1 ¯ ¯
(a) Á - , Í- 4 , 4 ˜¯
Ë 2 2 ˙˚ Î

È 1 1˘ È 1 1˘
(c) Í- , ˙ (d) Í- , ˙
Î 4 4˚ Î 4 2˚
1/ 2

( ) ( ) Ê Ê
( ) ( )ˆ˜ ˆ
2
sin cot -1 ( x + 1) = cos tan -1 x x is cos tan -1 y + y sin tan -1 y
Á 1 Á +y 4˜

( )
Á y 2 Á cot sin -1 y + tan sin -1 y
ÁË Ë ( ) ˜¯ ˜
˜¯

Let (x, y
( ) (
cot sin -1 1 - x 2 = sin tan -1 x 6 , x π 0 ( ))
x
p
sin (a x ) + cos ( y ) + cos (b x y ) =
-1 -1 -1
2
equation
Ê 1 ˆ Ê 1 ˆ Ê 2ˆ
tan -1 Á + tan -1 Á = tan -1 Á 2 ˜
(A) If a b x, y) Ë 2 x + 1˜¯ Ë 4 x + 1˜¯ Ëx ¯
x2 + y 2 = 1
300 Comprehensive Trigonometry with Challenging Problems & Solutions for Jee Main and Advanced

ANSWERS
EXERCISE 1 [1, 2]
x-5
f -1
( x) = È p p˘
3 Í- 2 , 2 ˙
Î ˚
x
f -1 ( x ) = È 5p 3p ˘
x -1 Í- 4 , 4 ˙
Î ˚
1- x [-p , p ]
f -1 ( x ) =
x
Ê p ˆ
1 Ê1+ xˆ ÁË - , 0˜¯
f ( x ) = log 2 Á
-1 2
2 Ë 1 - x ˜¯
(0, p )
-1 + 1 - 4 x 2
f -1 ( x ) = (0, p )
2x
EXERCISE 2 (0, p )
[1, 2] Ê 5p 9p ˆ
ÁË , ˜
4 4¯
Ê 1˘
ÁË -•,
2 ˙˚
È 5 ˘ {cosec -1
(1)}
Í- 3 , - 1˙
Î ˚
EXERCISE 3
[-3, 3] p
x=
[-6, 6] 2
x y
[-5, 5] x
[-1, 1] x
x
È 1˘
Í0, 2 ˙ Ê 1 ˘
Î ˚ x=Á ,1
Ë 2 ˙˚
È- 3, 1˘
Î ˚
p x=± 2
x = ( 4n + 1) , n ΠI
2 1
x = 3, -
x = {-1, 1} 3
x =j 3 -1
x=
[-2, - 1] 2 2
[-2, - 1] » [1, 2] x = 2 -1
x
È 1 ˘ È 1 ˘
Í-1, - ˙ » Í , 1˙ -1 ± 5
Î 3˚ Î 3 ˚ x=
x =j 2

È 1 ˘ È 1 ˘
(cos1, sin 1)
Í- ,- 2˙ » Í , 2˙
(cos1, tan 1)
Î 2 ˚ Î 2 ˚
(0, 4] ÈÎ tan (sin (sin 1)) , tan (sin 1) )
(0,1) » (1, • )
Inverse Trigonometric Function 301

( ) ( )
È tan sin (cos (sin 1)) , tan sin (cos (sin 1)) ˘ (20 - 6p )
Î ˚
x (50 - 16p )
(25p - 80)
EXERCISE 4
(32p - 100)
È 1˘
Í1, 5p
Î 2 ˙˚
6
(-•, 1] 3p
[-1, 0) 7
[-1, 0) 5p
( tan 2, • ) 7

( tan 3, cot 2)
(2p - 5)
È 1 ˆ
Í-1, - ˜ (7 - 2p )
Î 2¯
Ê1 ˘
(4p - 10)
ÁË , 1˙ (4p - 12)
2 ˚
Ê 1 ˆ Ê 1 ˆ
, 0˜ » Á
(15 - 4p )
ÁË - , 1˜
2 ¯ Ë 2 ¯ (40 - 12p )
Ê 1 ˆ Ê 1 ˆ
ÁË -•, - ˜ » Á 0, ˜¯ (32p - 100)
3¯ Ë 3
(32p - 100)
È 1˘
Í-1, 2 ˙ (2p - 4)
Î ˚
(1 - p - 1, 1 + p - 1 )
(10 - p )
Ê 8 ˆ
ÁË - , 0˜¯ 20 - 4p
5
(-•, - 1) » (1, • ) 7p - 20

5 5
- £x£
6 6 p
EXERCISE 5
p.
p
p
3 -
3
p
- p
3 -
5
p
p
8
12
(p - 3)
(3 - p )
(5 - 2p )
(5 - 2p )
(7 - 2p )
(7 - 2p )
(3p - 10)
(12 - 4p ) (10 - 3p )
302 Comprehensive Trigonometry with Challenging Problems & Solutions for Jee Main and Advanced

42. (20 - 6p ) 34.


x
43. (50 - 16p ) 2
(10 - 3p ) Ê xˆ
44. 35. 3 tan -1 Á ˜
Ë a¯
45. (200 - 63p )
46. (15 - 4p ) 36. - x , x
2 2
46. (16p - 50)
47. 2.
37. sin -1 ( x ) - sin -1 ( x)
Êp ˆ
Ê 3ˆ 38. Á + x˜
48. Á -2, - ˜ Ë4 ¯
Ë 2¯
Ê pˆ
49. (-1, 1) 39. ÁË x - ˜¯
4
p 1
EXERCISE 6 40. + cos -1 x 2
4 2
( )
1. x = 1/2
2. ( -•, 0) » ( 4, • ) 41. ( x + a ) , where a = tan -1 ÊÁË

˜

5 -1
3. x = ±
2 EXERCISE 7
4. No solution Ê 3 - 1 + 4p 3 + 1 + 4p ˆ
5. 11. x ŒÁ , ˜
6. x = ± 2 Ë 2 2 ¯
7. x = –13/12 Ê 2ˆ
12. x ŒÁ -2, - ˜
8. x = 8 Ë 3¯
9. k = 5
È1 ˘
10. 0 13. x Œ Í ,1˙
11. 1 Î2 ˚
12. 4
13. ....... 23 EXERCISE 8
24. sin ( x )
-1
16. 0
18. 1
25. 2 sin -1 ( x )
23. 21
26. 2 cos -1 ( x ) 24. 3
26. x = 1
Ê p xˆ 27. x = 2
27. Á - ˜
Ë 4 2¯
3
Ê p xˆ 28. x = 2
28. Á + ˜ 7
Ë 4 2¯ 3
29.
Êp ˆ 10
29. Á - x˜
Ë4 ¯
1 3
30.
Ê xˆ
30. sin -1 Á ˜ 2 13
Ë a¯ 31. x = 3
31. tan -1 ( x) 32 x = 0
Ê 1ˆ
33. x = 0, ± Á ˜
Ê aˆ Ë 2¯
32. tan -1 Á ˜ - x
Ë b¯ LEVEL II
1 1. (d) 2. (b) 3. (b) 4. (b)
33. tan -1 ( x )
2 5. (d) 6. (b) 7. (b) 8. (d)
Inverse Trigonometric Function 303

9. (a) 10. (c) 11. (c) 12. (c) 39. n = 5


13. (a, c) 14. (a) 15. (d) 16. (b) 40. n =8
17. (b) 18. (c) 19. (c) 20. (b) 41. k = f.
21. (a) 22. (a) 23. (b) 24. (c, d)
25. (a) 26. (b) 27. (b) 28. (a) ( )
44. tan sin (cos (sin 1)) £ x < tan (sin (1)) .
29. (a) 30. (c) 31. (b) 32. (a) 45. [1, • )
33. (a) 34. (c) 35. (b) 36. (c)
È 1 7˘
37. (c) 38. (b) 39. (c) 40. (a) 49. a Œ Í , ˙
41. (b) 42. (a) 43. (a) Î 32 8 ˚
50. x = y = a + 1
2

LEVEL III
53. q Πnp + tan (-2) , n ΠZ
-1
6. (8p - 21) 54. q
p 1
7. 55. x =
3 3
7p 3 p
16. Max value , when x = –1 56.
8 2
57. 36
p3 1
58. 29
and Min value = , when x = .
32 2 59. 4
1 3 60. 6
17 x = 61. 100
2 7
18. x = 3 INTEGER TYPE QUESTIONS
19. x = 0, 1/2, –1/2
3 1. 7
20. x = 2. 4
10 3. 3
21. x = 2 - 3, 3 4. x = 1
5. 3
a-b
22. x = 6. 9
1 + ab 7. 5, where M = 0, N = 1
23. x = n 2 - n + 1, n 8. 2
24. x = 4/3 9. 7
25. x = ab, 10. 8.
26. x = 1/2, y = 1. 11, 9
27. x = 1, y = 2 ; x = 2, y = 7 12. 6
29. –p
COMPREHENSIVE LINK PASSAGES
Ê 1 ˘
32. x ŒÁ ,1 Passage - I : 1. (c) 2. (b) 3. (a) 4. (c) 5. (a)
Ë 2 ˙˚
Passage - II : 1. (b) 2. (a) 3. (c) 4. (a) 5. (a)
Ê 1 ˘ 6. (c) 7. (a)
33. x ŒÁ -1, ˙
Ë 2˚ Passage - III : 1. (a) 2. (c) 3. (b)
Passage - IV : 1. (d) 2. (b) 3. (b)
34. x Œ(-•, cot 3) » (cot 2, • )
Passage - V : 1. (b) 2. (c) 3. (c)
Ê 1 ˆ Ê 1 ˆ
35. x ŒÁ ,1 » -1, - ˜
Ë 2 ˜¯ ÁË 2¯ MATCH MATRIX
1. (A) (P); (B) (Q); (C) (R); (D) (P)
Ê 1ˆ Ê 3ˆ
36 tan Á ˜ < x < tan Á ˜ 2. (A) (S); (B) (P); (C) (Q); (D) (R)
Ë 2¯ Ë 2¯ 3. (A) (P); (B) (Q); (C) (S); (D) (R).
37. cot (3) £ x £ cot (1) 4. (A) (Q); (B) (R); (C) (P);(D) (S).
5. (A) (R); (B) (Q); (C) (P); (D) (R).
38. x Œ(-1,1)
6. (A) (R); (B) (P); (C) (Q); (D) (Q).
304 Comprehensive Trigonometry with Challenging Problems & Solutions for Jee Main and Advanced

ASSERTION AND REASON Ê Ê 1 1 1 1 ˆˆ


1. (a) 2. (a) 3. (a) 4. (a) Á Á x + y + z - xyz ˜ ˜
= cot Á tan Á ˜˜
-1
5. (a) 6. (a) 7. (a) 8. (a)
9. (a) 10. (a) 11. (b) 12. (a) Á Á Ê 1 1 1 ˆ ˜˜
Á Á 1 - ÁË xy + xz + yz ˜¯ ˜ ˜
Ë Ë ¯¯
HINTS AND SOLUTION
Ê -1 Ê xy + yz + zx - 1 ˆ ˆ
LEVEL - III = cot Á tan Á ˜˜
Ë Ë xyz - ( x + y + z ) ¯ ¯
1. We have
Ê -1 Ê xyz - ( x + y + z ) ˆ ˆ
-1 -1 -1 -1 = cot Á cot Á ˜
= sin (cos(sin x)) + cos (sin(cos x)) Ë Ë xy + yz + zx - 1 ˜¯ ¯
-1 -1
= sin ( 1 - x ) + cos ( 1 - x ) Ê -1 Ê ( x + y + z ) - xyz ˆ ˆ
2 2

= cot Á cot Á ˜˜
-1 -1
= cos ( x) + sin ( x) Ë Ë 1 - ( xy + yz + zx ) ¯ ¯

p Ê ( x + y + z ) - xyz ˆ
= = Á ˜
2 Ë 1 - ( xy + yz + zx ) ¯
2. We have
Hence, the result.
-1 -1 -1
= tan {cosec(tan x ) - tan(cot x)} 4. Given expression is
-1 -1
Ê 1 + x 2 - 1ˆ = sin(cot (tan(cos x)))
-1
= tan Á ˜
ÁË x ˜¯ -1 -1
= sin(cot (tan q )), q = cos x

-1 Ê sec q - 1ˆ
-1
= tan Á , x = tan q = sin(cot (tan q )), cos q = x
Ë tan q ˜¯
Ê Ê 1 - x2 ˆ ˆ
-1
-1 Ê 1 - cos q ˆ = sin Á cot Á ˜˜
= tan Á ÁË ÁË x ˜¯ ˜¯
Ë sin q ˜¯

Ê Êq ˆ ˆ 1 - x2
Á 2 sin 2 Á ˜ ˜ = sin j , where cot j =
-1 Ë 2¯ x
= tan Á ˜
Á 2 sin Ê q ˆ cos Ê q ˆ ˜ =x
ÁË ÁË ˜¯ ÁË ˜¯ ˜¯
2 2 5. Given expression is
-1 Ê Êq ˆˆ
= tan Á tan Á ˜ ˜ ( -1
( ( -1
= sin cosec cot tan x )))
Ë Ë 2¯¯
= sin (cosec (cot q )) , where tan q = x
-1
1
= tan -1 x
2 Ê -1 Ê 1 ˆ ˆ
= sin Á cosec Á ˜ ˜
Ë Ë x¯¯
-1 -1 -1
3. We have tan(tan x + tan y + tan z ) 1
= sin j , where cosecj =
x
Ê -1 Ê x + y + z - xyz ˆ ˆ
= tan Á tan Á =x
Ë Ë 1 - xy - yz - zx ˜¯ ˜¯
6. Given expression is
Ê x + y + z - xyz ˆ -1 -1 -1
= sin (sin 5) + cos (cos10) + tan (tan( -6))
= Á
Ë 1 - xy - yz - zx ˜¯
+ cot -1 (cot( -10))
-1
Also, cot(cot x + cot -1 y + cot -1 z ) = (5 - 2p ) + (4p - 10) - (6 - 2p ) + p - (10 - 3p )
Ê -1 Ê 1 ˆ -1 Ê 1 ˆ -1 Ê 1 ˆ ˆ = (5 - 2p ) + (4p - 10) + (2p - 6) + p + (3p - 10)
= cot Á tan Á ˜ + tan Á ˜ + tan Á ˜ ˜
Ë Ë x¯ Ë y¯ Ë z¯¯ = 8p - 21
Inverse Trigonometric Function 305

7. We have 10. Given equation is


Ê 3 - 3x 2 ˆ (3p )3
-1 -1 x Ê1 ˆ (sin -1 x)3 + (sin -1 y )3 + (sin -1 z )3 =
= cos x + cos Á + ˜ , " x Œ Á ,1˜
ÁË 2 2 ˜¯ Ë2 ¯ 8
It is possible only when
-1 -1 Ê 1 1 ˆ p
= cos ( x) + cos Á x ◊ + 1 - 1 - x2 ˜ sin -1 x = = sin -1 y = sin -1 z
Ë 2 4 ¯ 2

-1 -1 Ê 1 ˆ -1
So, x = 1 , y = 1 and z = 1
= cos ( x) + cos Á ˜ - cos ( x)
Ë 2¯ (3x + 4 y - 5 z + 2)
-1 Ê 1 ˆ =3+4–5+2
= cos Á ˜
Ë 2¯ =4
p n
-1 Ê r +1 1ˆ
= 11. We have S = Â cot Á 2 + r ˜
3 r =1 Ë 2 ¯
8. We have 2 r +1
-1 Ê 2 + 1ˆ
n
= Â cot Á ˜
Ê 5-2 6ˆ Ë 2r ¯
-1 Ê 1 ˆ r =1
= tan Á ˜ - tan -1 Á ˜
Ë 2¯ ÁË 1 + 6 ˜¯
n
-1 Ê 2r ˆ
= Â tan Á ˜
Ê ( 3 - 2 )2 ˆ r =1 Ë 1 + 22 r +1 ¯
-1 Ê 1 ˆ -1
= tan Á - tan Á ˜
Ë 2 ˜¯ ÁË 1+ 6 ˜¯ r +1
n
-1 Ê 2 - 2r ˆ
= Â tan Á ˜
Ê ( 3 - 2)ˆ r =1
r +1
Ë 1 + 2 ◊ 2r ¯
-1 Ê 1 ˆ
= tan Á ˜ - tan -1 Á ˜
Ë 2¯ Ë 1+ 3 2 ¯ n
-1 r +1 -1 r
= Â [tan (2 ) - tan (2 )]
r =1
-1 Ê 1 ˆ
= tan Á - ( tan -1 ( 3 ) - tan -1 ( 2 ))
Ë 2 ˜¯ -1 n +1 -1
= [tan (2 ) - tan (2)]
-1 -1 -1
= cot ( 2 ) + tan ( 2 ) - tan ( 3 ) Thus, lim ( S )
nƕ
p p p
= - = -1 n +1 -1
2 3 6 = Lim [tan (2 ) - tan (2)]
nƕ
9. We have
-1 -1
m = sin -1 (a 6 + 1) + cos -1 (a 4 + 1) - tan -1 (a 2 + 1) = [tan (•) - tan (2)]

p
since sin ( ) + cos ( )
-1 -1
= - tan -1 (2)
2
Put a = 0 , then
-1
m = sin -1 (1) + cos -1 (1) - tan -1 (1) = cot (2)

p p p Ê Ê n -1 Ê 4 ˆˆˆ
= - = 12. We have Lim Á tan Á Â tan Á 2
2 4 4 nÆ• Ë Ë r =1 Ë 4r + 3 ˜¯ ˜¯ ˜¯
p
x+ y=
4
Ê Ê Ê ˆˆˆ
p Á Á n -1 Á 1 ˜˜˜
w.r.t to the y-axis is x - y + = 0 = Lim Á tan Á Â tan Á
4 nÆ• 3 ˜˜˜
Á Á r =1 Á r2 + ˜˜˜
Ë Ë Ë 4 ¯¯¯
306 Comprehensive Trigonometry with Challenging Problems & Solutions for Jee Main and Advanced

Ê Ê Ê ˆˆˆ Ê xy x2 y2 ˆ
Á Á n Á ˜˜˜ fi Á - 1 - 2 1 - 2 ˜ = cosa
-1 1 ÁË ab a b ˜¯
= Lim Á tan Á  tan Á ˜˜˜
nÆ• Á Á r =1 Á1 + Ê r2 - 1ˆ ˜˜˜
ÁË ÁË ˜
ÁË ÁË 4 ¯ ˜¯ ˜¯ ˜¯
2
Êxy ˆ
2 Ê x2 y2 ˆ
fi ÁË a b - cosa ˜¯ = ÁÁ 1 - a 2 1 - b 2 ˜˜
Ê Ê ÊÊ 1ˆ Ê 1ˆ ˆˆˆ Ë ¯
Á Á n Á ÁË r + ˜¯ - ÁË r - ˜¯ ˜ ˜ ˜
-1 2 2
= Lim Á tan Á  tan Á ˜˜˜ Ê xy ˆ
2
Ê xy ˆ
Á1 + Ê r + 1ˆ Ê r - 1ˆ ˜˜˜ ÁË ˜¯ - 2 ÁË ˜¯ cos a + cos a
nÆ• Á 2
Á r =1 fi
ÁË ÁË ˜Á ˜ ab ab
ÁË ÁË 2¯ Ë 2 ¯ ˜¯ ˜¯ ˜¯
Ê x 2 y 2 ˆ Ê xy ˆ 2
Ê n ˆ
= Lim Á tan ÊÁ  tan -1 ÊÁ n + 1 ˆ˜ - tan -1 ÊÁ n - 1 ˆ˜ ˆ˜ ˜ =1- Á 2 + 2 ˜ + Á ˜
nÆ• Ë Ë r =1 Ë 2¯ Ë 2¯ ¯ ¯ Ëa b ¯ Ë ab ¯

Ê Ê Ê 1ˆ Ê 1ˆˆˆ x2 Ê xy ˆ y2
Lim Á tan Á tan -1 Á n + ˜ - tan -1 Á ˜ ˜ ˜ fi - 2 ÁË ˜¯ cos a + = 1 - cos 2 a
nÆ• Ë Ë Ë 2¯ Ë 2¯¯¯ a2 ab b2

Ê Ê Ê 1 1 ˆˆˆ x2 Êxy ˆ y2
n+ - fi - 2Á ˜¯ cos a + = sin 2 a
Á Á Á ˜˜
2 2 ˜˜ ˜ ˜ a Ë ab b
2 2
-1
= Lim Á tan Á tan Á
nÆ• Á Á Á1 + 1 Ê n + 1ˆ ˜˜˜ -1 -1 -1
ÁË Á ˜ 15. Put A = sin x , B = sin y , C = sin z
ÁË ÁË 2Ë 2 ¯ ˜¯ ¯˜ ˜¯
fi x = sin A , y = sin B , z = sin C
Ê ˆ
Á n ˜
fi cos A = 1 - x , cos B = 1 - y , cos C = 1 - z
2 2 2
= Lim Á ˜
nÆ• Á 1Ê 1ˆ ˜
ÁË 1 + Á n + ˜ Thus, A + B + C = p
2Ë 2 ¯ ˜¯
Now, L.H.S
Ê ˆ
Á ˜ = x 1- x + y 1- y + z 1- z
2 2 2
1
= Lim Á ˜
nÆ• Á 1 1Ê 1 ˆ˜ = sin A cos A + sin B cos B + sin C cos C
+ +
ÁË n 2 ÁË 2n ˜¯ ˜¯
1
1
= 2. = [sin 2 A + sin 2 B + sin 2C ]
2
13 Given equation is
1
Ê 2x ˆ
2 sin -1 Á = p x3
= (4sin A sin B sin C )
Ë 1 + x 2 ˜¯
2
= 2sin A sin B sin C
-1 Ê2 x ˆ p x3
sin Á = = 2xyz .
Ë 1 + x 2 ˜¯ 2
Hence, the result.
p x3
2 tan -1 x = -1 3 -1 3
16. Given f ( x) = (sin x) + (cos x)
2
-1 -1 -1 -1
= (sin x + cos x){(sin x) + (cos x)
2 2
-1 p x3
tan x=
4 - sin -1 x cos -1 x}
Clearly , there are 3 solutions at x = –1, 0 , 1
p ÏÔÊ p ˆ ¸
2
-1 -1 Ô
-1 Ê x ˆ -1 Ê y ˆ = ÌÁ ˜ - 3 sin x cos x ˝
14. Given cos Á ˜ + cos Á ˜ = a 2 ÓÔË 2 ¯
Ë a¯ Ë b¯ ˛Ô

Ê xy p ÔÏ p 2 -1 Ê p ¸
-1 ˆ Ô
x2 y2 ˆ = Ì - 3 sin x Á - sin x˜ ˝
Ë2 ¯ Ô˛
fi cos -1 Á - 1 - 2 1 - 2 ˜ = a 2 ÔÓ 4
ÁË ab a b ˜¯
Inverse Trigonometric Function 307

p ÏÔ p 2 Êp ˆ ¸Ô 2 + 6x 2
= Ì - 3 a Á - a ˜ ˝ where a = sin -1 x fi =
2 ÓÔ 4 Ë2 ¯ Ô˛ 6 x + 8x 2
x2
1 + 3x 2
p ÔÏ 2 3a p p 2 Ô¸ fi =
= Ì3a - + 3x + 4 x x2
2
˝
2 ÔÓ 2 4 Ô˛
fi 3 x3 + x 2 + 8 x 2 + 6 x = 0
p
= {12a - 6 p a + p }
2 2
8 fi 3 x3 + 9 x 2 + 6 x = 0

12p ÏÔ 2 1 p 2 ¸Ô fi x3 + 3 x 2 + 2 x = 0
= Ì a - p a + ˝
8 ÓÔ 2 12 ˛Ô fi x ( x 2 + 3 x + 2) = 0

ÏÔÊ fi x( x + 1)( x + 2) = 0
p 2 ¸Ô
2
12p pˆ
= ÌÁ a - ˜ + ˝
8 ÔÓË 4¯ 48 Ô˛ fi x = 0, - 1, - 2
19 Given equation is
p3 1
Min value = at x =
32 2 tan -1 ( x - 1) + tan -1 ( x) + tan -1 ( x + 1) = tan -1 (3x)
-1 -1 -1 -1
7p 3 fi tan ( x - 1) + tan ( x + 1) = tan (3x) - tan ( x)
and Max value = at x = -1
8
Ê x - 1 + x + 1ˆ -1 Ê 3 x - x ˆ
17. Given equation is fi tan -1 Á ˜ = tan ÁË 1 + 3x ◊ x ˜¯
Ë 1 - ( x - 1) ¯
2

p
sin -1 x + sin -1 2 x = Ê 2x ˆ Ê 2x ˆ
2 fi ÁË - 2 ˜¯ = ÁË ˜
p x 1 + 3x 2 ¯
fi sin -1 (2 x) = - sin -1 x = cos -1 x
2 fi 2 x (1 + 3x 2 + x 2 ) = 0
fi sin -1 (2 x) = sin -1 ( 1 - x 2 ) 2 x = 0 , (1 + 4 x 2 ) = 0

fi 2 x = 1 - x2 fi x=0
Hence, the solution is x = 0.
fi 4 x2 = 1 - x2
20. Given equation is
fi 5 x2 = 1 Ê 1 ˆ p
sin -1 Á ˜ + cos -1 x =
Ë 5¯ 4
1
fi x2 =
5 Ê 1ˆ p
fi cos -1 Á 1 - ˜ + cos -1 x =
1 Ë 5 ¯ 4
fi x=±
5 Ê 2 ˆ p
fi cos -1 Á ˜ + cos -1 x =
18. Given equation is Ë 5¯ 4
Ê 1 ˆ Ê 1 ˆ Ê 2ˆ
tan -1 Á + tan -1 Á = tan -1 Á 2 ˜ p Ê 2 ˆ
Ë 1 + 2 x ˜¯ Ë 1 + 4 x ˜¯ Ëx ¯ fi cos -1 x = - cos -1 Á ˜
4 Ë 5¯
Ê 1 1 ˆ
+ Êp Ê 2 ˆˆ
-1 Á
1 + 2 x 1 + 4 x ˜ Ê 2ˆ
= tan -1 Á 2 ˜ fi x = cos Á - cos -1 Á ˜ ˜
fi tan Á Ë4 Ë 5 ¯¯
1 1 ˜ Ëx ¯
Á1 - ¥ ˜
Ë 1 + 2x 1 + 4x ¯ Ê
Êp ˆ Ê 2 ˆˆ
fi x = cos Á ˜ cos Á cos -1 Á ˜ ˜
1 + 4x + 1 + 2x 2 Ë 4¯ Ë Ë 5 ¯¯
fi =
1 + 6 x + 8x - 1 2
x2 Êp ˆ Ê Ê 2 ˆˆ
+ sin Á ˜ sin Á cos -1 Á ˜ ˜
Ë 4¯ Ë Ë 5 ¯¯
308 Comprehensive Trigonometry with Challenging Problems & Solutions for Jee Main and Advanced

1 2 1 1 3 fi n 2 x - x 2 + x - n3 + n x - n + n 2 - x + 1
fi x= ¥ + ¥ =
2 5 2 5 10
fi n 2 x - x 2 + x - n3 + n x - n + n 2 - x + 1
21. Given equation is
Ê x 2 - 1ˆ Ê 2 x ˆ 2p = n x - x +1
cos -1 Á 2 ˜ + tan -1 Á 2 ˜ =
Ë x + 1¯ Ë x - 1¯ 3
fi n 2 x - x 2 + x - n3 - n + n 2 = 0
Ê 1 - x2 ˆ Ê 2 x ˆ 2p
fi cos -1 Á - 2 ˜ + tan -1 Á - = fi (n 2 + 1) x - (n 2 + 1)n - x 2 + n 2 = 0
Ë x + 1¯ Ë 1 - x 2 ˜¯ 3
fi (n 2 + 1) ( x - n) - ( x 2 - n 2 ) = 0
Ê1 - x 2ˆ
Ê 2 x ˆ 2p
fi p - cos -1 Á 2 ˜ - tan -1 Á =
Ë x + 1¯ Ë 1 - x 2 ˜¯ 3 fi ( x - n)(n 2 + 1 - x - n) = 0

2p fi ( x - n) = 0 , ( n 2 + 1 - x - n) = 0
fi p - 2 tan -1 ( x) - 2 tan -1 ( x) =
3
fi x = n , n2 - n + 1
2p
-1
fi p - 4 tan ( x) = Hence, the solutions are
3
-1 2p p x = n , n2 - n + 1
fi 4 tan ( x) = p - =
3 3 24. Given equation is
p Ê x - 1ˆ Ê 2 x - 1ˆ Ê 23 ˆ
fi tan -1 ( x) = tan -1 Á ˜ + tan -1 Á ˜ = tan -1 Á ˜
12 Ë x + 1¯ Ë 2 x + 1¯ Ë 36 ¯
Êpˆ Ê x - 1 2x - 1 ˆ
fi x = tan Á ˜ = (2 - 3 )
Ë 12 ¯ Á + ˜ Ê 23 ˆ
fi tan -1 Á x + 1 2 x + 1 ˜ = tan -1 Á ˜
22. Given equation is Á1 - Ê x - 1 2 x - 1 ˆ ˜ Ë 36 ¯
ÁË ÁË x + 1 . 2 x + 1˜¯ ˜¯
Ê 1 - a2 ˆ -1 Ê 1 - b ˆ
2
2 tan -1 x = cos -1 Á ˜ - cos Á ˜
Ë 1 + a2 ¯ Ë 1 + b2 ¯ 2 x2 - x - 1 + 2 x2 + x - 1 23
fi =
fi 2 tan -1 -1
x = 2 tan (a) - 2 tan (b) -1 2 x + 3x + 1 - 2 x + 3x - 1
2 2
26
4 x 2 - 2 23
fi tan -1 -1
x = tan (a) - tan (b) -1 fi =
6x 36
Ê a-b ˆ 2 x 2 - 1 23
fi tan -1 x = tan -1 Á =
Ë 1 + ab ˜¯ fi
x 12
Ê a-b ˆ fi 24 x 2 - 23x - 12 = 0
fi x=Á
Ë 1 + ab ˜¯
fi 24 x 2 - 32 x + 9 x - 12 = 0
23. Given equation is
fi 8 x(3x - 4) + 3(3x - 4) = 0
cot -1 x + cot -1 (n 2 - x + 1) = cot -1 (n - 1)
fi (8 x + 3)(3x - 4) = 0
Ê 1ˆ Ê 1 ˆ Ê 1 ˆ
fi tan -1 Á ˜ + tan -1 Á 2 ˜ = tan -1 Á
Ë x¯ Ë n - x + 1¯ Ë n - 1˜¯ fi
4 3
x = ,-
3 8
Ê 1 ˆ Ê 1 ˆ Ê 1ˆ
fi tan -1 Á 2 ˜ = tan -1 Á ˜ - tan -1 Á ˜ 4 3
Hence, the solutions are x = , -
Ë n - x + 1¯ Ë n - 1¯ Ë x¯ 3 8
Ê 1 1 ˆ 25. Given equation is
Á - ˜
Ê ˆ
= tan -1 Á n - 1 x ˜
1 Ê xˆ Ê xˆ
fi tan -1 Á 2 sec -1 Á ˜ - sec -1 Á ˜ = sec -1 b - sec -1 a
Ë n - x + 1˜¯ Á1 + 1 ˜ Ë a¯ Ë b¯
ÁË x ( n - 1) ˜¯
Inverse Trigonometric Function 309

Ê xˆ Ê xˆ È -1 2 2
fi sec -1 Á ˜ + sec -1 (a) = sec -1 Á ˜ + sec -1 (b) -1 Ê 3 ˆ -1 2
Ë a¯ Ë b¯ = Í tan (1 ) - 0 + tan Á ˜ - tan (1 )
ÍÎ Ë 2¯
Ê aˆ Ê 1ˆ Ê bˆ Ê 1ˆ
fi cos -1 Á ˜ + cos -1 Á ˜ = cos -1 Á ˜ + cos -1 Á ˜ 2
Ê 3ˆ
Ë x¯ Ë a¯ Ë x¯ Ë b¯ + tan -1 (22 ) - tan -1 Á ˜ + ...... +
Ë 2¯
Êa 1 2
1 ˆ
Ê aˆ
fi cos -1 Á ◊ - 1 - Á ˜ 1 - 2 ˜ -1 Ê n + 1ˆ
2
-1 Ê n - 1ˆ

ÁË x a Ë x¯ a ˜¯ = tan ÁË ˜ - tan ÁË ˜ ˙,nÆ•
2 ¯ 2 ¯ ˙˚
Êb 1 2
1 ˆ
Ê bˆ ÏÔ -1 Ê 1ˆ
2 ¸Ô
= cos -1 Á ◊ - 1 - Á ˜ 1 - 2 ˜ = Lim Ì tan Á n + ˜ - 0˝
ÁË x b Ë x¯ b ˜¯ nÆ• Ë 2¯
ÔÓ Ô˛
-1 p
1 ( x 2 - a 2 )(a 2 - 1) 1 ( x 2 - b 2 )(b 2 - 1) = tan (•) =
fi - = - 2
x ax x bx
27. Given equation is
( x 2 - a 2 )(a 2 - 1) ( x 2 - b 2 )(b 2 - 1) p
fi = sin -1 (e x ) + cos -1 ( x 2 ) =
a b 2
It solutions exist only when
( x 2 - a 2 )(a 2 - 1) ( x 2 - b 2 )(b 2 - 1)
fi =
a2 b2 e x = x2
Y
fi ( x 2 - a 2 )(a 2 - 1)b 2 = ( x 2 - b 2 )(b 2 - 1)a 2

fi x 2 ((a 2 - 1)b 2 - a 2 (b 2 - 1))

= a 2b 2 (a 2 - 1) - a 2b 2 (b 2 - 1) X¢ X
O

fi x 2 [a 2 - b 2 ] = a 2b 2 [(a 2 - 1) - (b 2 - 1)]

fi x 2 [(a 2 - b 2 )] = a 2b 2 [(a 2 - b 2 )]

x 2 = a 2b2 28. Given equation is



sin( x) = cos -1 (cos x) in (0, 2p )
fi x = ab
Y
• 8n
-1 Ê ˆ
26. We have  tan Á 4 ˜ y=p
n =1 Ë n - 2n + 5 ¯
2


= Â tan Á
-1 {
Ê 2 (n + 1) 2 - (n - 1) 2 } ˆ˜ y=1

n =1 ÁË 4 + (n - 1)2 ˜¯ X
O p 2p 3p 4p
Ê ÔÏÊ n + 1ˆ 2 Ê n - 1ˆ 2 Ô¸ ˆ
Á ÌÁË ˜ - ÁË ˜ ˝˜

-1 Á ÓÔ 2 ¯ 2 ¯ ˛Ô ˜
= Â tan Á 2 2 ˜ p
n =1
Á 1 + Ê n + 1ˆ ◊ Ê n - 1ˆ ˜ solution is 1 at x =
ÁË ˜ Á ˜ 2
ÁË 2 ¯ Ë 2 ¯ ˜¯
29. Given equation is
• Ï 2¸
-1 ÔÊ n + 1ˆ Ô
Ï 2¸
-1 ÔÊ n + 1ˆ Ô
Ê 1ˆ Ê 1ˆ Ê 1ˆ Ê 1ˆ p
= Â - tan -1 Á ˜ + tan -1 Á ˜ + tan -1 Á ˜ + tan -1 Á ˜ =
tan ÌÁ ˜ ˝ tan ÌÁ ˜ ˝ Ë 3¯ Ë 4¯ Ë 5¯ Ë n¯ 4
n =1 ÔÓË 2 ¯ Ô˛ ÔÓË 2 ¯ Ô˛
310 Comprehensive Trigonometry with Challenging Problems & Solutions for Jee Main and Advanced

Ê 1 1 ˆ Now, f (0) = 1 > 0 , f ( -1) = b - c < 0


+
Á ˜ Ê 1ˆ Ê 1ˆ p
fi tan -1 Á 3 4 ˜ + tan -1 Á ˜ + tan -1 Á ˜ = f (x) has a root in -1 < a < 0
1 1 Ë 5¯ Ë n¯ 4
Á1 - ◊ ˜
Ë 3 4¯ -1 -1
Now, 2 tan (cosec a ) + tan (2 sin a sec a )
2

Ê 7ˆ Ê 1ˆ Ê 1ˆ p
fi tan -1 Á ˜ + tan -1 Á ˜ + tan -1 Á ˜ = -1 Ê 1 ˆ Ê 2 sin a ˆ
Ë 11¯ Ë 5¯ Ë n¯ 4 = 2 tan Á ˜ + tan -1 Á
Ë sin a ¯ Ë cos 2 a ˜¯
Ê 7 1 ˆ Ê 2 sin a ˆ
+ -1 Ê 1 ˆ
-1 Á ˜ Ê 1ˆ p = 2 tan Á + tan -1 Á
fi tan Á 11 5 ˜ + tan -1 Á ˜ = Ë sin a ˜¯ Ë 1 - sin 2 a ˜¯
7 1 Ë n¯ 4
Á1 - . ˜
Ë 11 5 ¯ -1 Ê 1 ˆ
= 2 tan Á + 2 tan -1 (sin a )
Ë sin a ˜¯
Ê 23 ˆ Ê 1ˆ p
fi tan -1 Á ˜ + tan -1 Á ˜ =
Ë 24 ¯ Ë n¯ 4 È -1 Ê 1 ˆ -1 ˘
= 2 Í tan Á
Ë ˜¯ + tan (sin a ) ˙
Î sin a ˚
Ê 1ˆ p Ê 23 ˆ
fi tan -1 Á ˜ = - tan -1 Á ˜
Ë n¯ 4 Ë 24 ¯ Ê pˆ
= 2 Á - ˜ , as sin a < 0 = p
Ë 2¯
Ê 1ˆ Ê 23 ˆ
fi tan -1 Á ˜ = tan -1 (1) - tan -1 Á ˜ -1 -1
32. Given sin x > cos x
Ë n¯ Ë 24 ¯
p
Ê 23 ˆ fi 2 sin -1 x > sin -1 x + cos -1 x =
1- 2
-1 Ê 1 ˆ -1 Á 24 ˜
fi tan Á ˜ = tan Á
Ë n¯ 23 ˜ p
Á1 + ˜ fi sin -1 x >
Ë 24 ¯ 4
Ê 1ˆ Ê 1ˆ Êp ˆ 1
fi tan -1 Á ˜ = tan -1 Á ˜ fi x > sin Á ˜ =
Ë n¯ Ë 47 ¯ Ë 4¯ 2
Ê 1 ˘
fi n = 47 Hence, the solution is x Œ Á ,1
Ë 2 ˙˚
30. Given equation is
-1 -1
33. Given cos x > sin x
x3 + b x 2 + c x + 1 = 0
fi sin -1 x + cos -1 x > 2 sin -1 x
Let f ( x) = x + b x + c x + 1
3 2
p
fi 2 sin -1 x <
Now, f (0) = 1 > 0 , f ( -1) = b - c < 0 2
f (x) has a root in -1 < a < 0 p
fi sin -1 x <
4
-1 -1 Ê 1 ˆ
Now, tan (a ) + tan Á ˜ Êp ˆ 1
Ëa ¯ fi x < sin Á ˜ =
Ë 4¯ 2
-1 -1
= tan a - p + cot a È 1 ˆ
Hence, the solution set is x Œ Í -1, ˜
-1
= -p + (tan a + cot a )
-1 Î 2¯
-1
34. Let a = cot x
p p
= -p + = - The given in-equation reduces to
2 2
31. Given equation is a 2 - 5a + 6 > 0

x3 + b x 2 + c x + 1 = 0 fi (a - 2)(a - 3) > 0

fi a < 2 and a > 3


Let f ( x) = x + b x + c x + 1
3 2

fi cot -1 x < 2 and cot -1 x > 3


Inverse Trigonometric Function 311

fi x > cot(2) and x > cot(3) fi 1£ a £ 3


Hence, the solution set is ( -• , cot( 2)) » (cot 3 , •)
fi 1 £ cot -1 x £ 3
Ê -1 Ê x ˆˆ fi cot(3) £ x £ cot(1)
˜˜ >1
2
35. tan Á tan Á
Ë Ë 1 - x2 ¯ ¯ 38. Given in equation is
2
Ê Ê 2 x2 + 4 ˆ ˆ
ÏÔ Ê Ê x ˆ ˆ ¸Ô sin -1 Á sin Á <p -2
2 ˜˜
-1
fi Ì Á
tan tan Á ˜˜˝ >1 Ë Ë 1+ x ¯¯
ÔÓ Ë Ë 1 - x 2 ¯ ¯ Ô˛
Ê Ê 2( x 2 + 1) + 2 ˆ ˆ
x2 sin -1 Á sin Á
fi >1 fi ˜˜ < p - 2
(1 - x 2 ) Ë Ë x2 + 1 ¯ ¯

x2 Ê Ê 2 ˆˆ
fi -1 > 0 fi sin -1 Á sin Á 2 + 2 ˜ ˜ < p - 2
Ë Ë x + 1¯ ¯
(1 - x 2 )
x2 - 1 + x2 Ê Ê 2 x2 + 4 ˆ ˆ
fi >0 fi sin -1 Á sin Á p - 2 ˜˜ < p - 2
(1 - x 2 ) Ë Ë x +1 ¯¯

2 x2 - 1 Ê 2 x2 + 4 ˆ
fi <0 fi Á p - ˜ <p -2
( x 2 - 1) Ë x2 + 1 ¯

( 2 x + 1)( 2 x - 1) Ê 2 x2 + 4 ˆ
fi <0 fi Á 2 ˜ >2
( x + 1)( x - 1) Ë x +1 ¯
Ê 1 ˆ Ê 1 ˆ Ê x2 + 2 ˆ
fi x Œ Á -1, - ˜ » Á1, ˜ fi ˜ >1
Ë 2¯ Ë 2¯ Á 2
Ë x +1¯
-1 -1
36. Given 4(tan x) - 8(tan x) + 3 < 0
2 which is a true statement.
Hence, x ΠR
fi 4a 2 - 8a + 3 < 0 , a = tan -1 x 39. We have
fi 4a 2 - 6a - 2a + 3 < 0 f ( x) = {sin -1 (sin x)}2 - sin -1 (sin x)

fi 2a(2a - 3) - 1(2a - 3) < 0 Ï -1 1¸ 1


2
= Ìsin (sin x ) - ˝ -
fi (2a - 1)(2a - 3) < 0 Ó 2˛ 4
2
1 3 Ïp 1 ¸ 1
fi <a< = Ì + ˝ -
2 2 Ó 2 2˛ 4
1 3 p
fi < tan -1 x < = (p + 2) , since the maximum value
2 2 4
Ê 1ˆ Ê 3ˆ p
fi tan Á ˜ < x < tan Á ˜ sin -1 (sin x) is -
Ë 2¯ Ë 2¯ 2
40. Clearly, both terms are positive.
-1 -1
37. Given 4 cot x - (cot x) - 3 ≥ 0
2
Applying, A.M ≥ G.M , we get,
-1 -1
x x
fi 4a - a 2 - 3 ≥ 0 , a = cot -1 x 8sin + 8cos 1-
x -1
x
fi ≥ 8sin .8cos
2
fi a 2 - 4a + 3 £ 0
f ( x) -1
≥ 8sin x + cos x
-1

fi (a - 1)(a - 3) £ 0 2
312 Comprehensive Trigonometry with Challenging Problems & Solutions for Jee Main and Advanced

f ( x)
p Ï Ï -1 Ê Ê p ˆ ˆ p p ¸¸Ô
-1 Ô
fi ≥ 82 = sin Ìcot Ìsin Á sin Á ˜ ˜ + + ˝˝
2 ÓÔ Ó Ë Ë 12 ¯ ¯ 6 4 ˛˛Ô

-1 Ï Ï p p p ¸¸
p p 3p 3p
1+
fi f ( x) ≥ 2 8 2 = 2.8 4 = 2.2 4 = 2 4 = sin Ìcot Ì + + ˝˝
Ó Ó12 6 4 ˛˛
3p
-1 Ï Ê 6p ˆ ¸
1+
2 4 = sin Ìcot Á ˜ ˝
41. Given in equation is Ó Ë 12 ¯ ˛
-1
x 2 - kx + sin -1 (sin 4) > 0 = sin (0) = 0
-1 -1 -1 -1
fi x 2 - k x + sin -1 (sin(p - 4)) > 0 44. Given f ( x) = sin (cos x + tan x + cot x)

x 2 - k x + (p - 4) > 0 -1 Ê p -1 ˆ
fi = sin Á + cos x˜
Ë2 ¯
fi For , all x in R, D ≥ 0
Êp -1 ˆ
Thus, -1 £ Á + cos x˜ £ 1
fi k - 4(p - 4) ≥ 0
2 Ë2 ¯
p p
fi k 2 ≥ 4(p - 4) fi -1 - £ cos -1 x £ 1 -
2 2
k
Hence, the solution is k = j cos -1 x is [0, p ]
42. Given So it has no solution
A = 2 tan -1 (2 2 - 1) Df = j
-1 -1
fi A = 2 tan (2.8 - 1) = 2 tan (1.4) 45. Clearly, 0 £ x £ 4
We have
2p
fi A> Ê xˆ Ê xˆ 2p
3 sin -1 Á ˜ + sin -1 Á 1 - ˜ + tan -1 y =
Ë 2 ¯ Ë 4¯ 3
-1 Ê 1 ˆ -1 Ê 3 ˆ
and B = 3 sin Á ˜ + sin Á ˜
Ë 3¯ Ë 5¯ Ê xˆ Ê xˆ 2p
fi sin -1 Á ˜ + cos -1 Á ˜ + tan -1 ( y ) =
-1 Ê 3 4ˆ -1 Ê 3 ˆ
Ë 2 ¯ Ë 2 ¯ 3
fi B = sin Á - ˜ + sin Á ˜
Ë 3 27 ¯ Ë 5¯ p 2p
fi + tan -1 ( y ) =
2 3
-1 Ê 23 ˆ -1 Ê 3 ˆ
fi B = sin Á ˜ + sin Á ˜ 2p p p
Ë 27 ¯ Ë 5¯ tan -1 ( y ) = - =

3 2 6
-1 Ê 3 ˆ -1 Ê 3 ˆ
fi B < sin Á ˜ + sin Á ˜ Êp ˆ 1
Ë 2 ¯ Ë 2 ¯ fi y = tan Á ˜ =
Ë 6¯ 3
p p 2p
fiB< + = ( x 2 + y 2 + 1)
3 3 3
Ê 1 ˆ 52
is = Á16 + + 1˜ =
Hence, A > B Ë 3 ¯ 3
43. We have
-1 Ê 1 ˆ -1 Ê 1 ˆ -1 Ê 1 ˆ
Ï Ï Ê 2- 3ˆ ¸Ô¸Ô 46. Given tan Á ˜ + tan Á ˜ = tan Á ˜
Ô Ô -1 Ê 12 ˆ
Ë x¯ Ë y¯ Ë 10 ¯
sin -1 Ìcot Ìsin -1 Á ˜ + cos Á ˜ + sec -1
( 2 ) ˝˝
ÔÓ ÓÔ ÁË 4 ˜¯ Ë 4 ¯ Ô˛Ô˛ Ê 1 1 ˆ
+
-1
Á x y ˜ -1 Ê 1 ˆ
Ï ÏÔ -1 Ê 3 - 1ˆ ¸Ô¸Ô fi tan Á ˜ = tan Á ˜
-1 Ô -1 Ê 3 ˆ Ë 10 ¯
= sin Ìcot Ìsin Á ˜ + cos Á 2 ˜ + cos ( 2 ) ˝˝
-1
Á1 - 1 . 1 ˜
ÓÔ ÓÔ Ë 2 2 ¯ Ë ¯ ˛ÔÔ˛ Ë x y¯
Inverse Trigonometric Function 313

Ê x+ y ˆ 49. Given equation is


Ê1ˆ
fi tan -1 Á ˜ = tan -1 Á ˜
Ë x y - 1¯ Ë 10 ¯ (sin -1 x)3 + (cos -1 x)3 = p a3

Ê x+ y ˆ Ê 1 ˆ p
fi [(sin -1 x)2 + (cos -1 x )2 - 2 sin -1 x cos -1 x] = p a3
fi ÁË x y - 1˜¯ = ÁË 10 ˜¯ 2

fi 10( x + y ) = xy - 1 Èp 2 ˘
fi Í - 3 sin -1 x cos -1 x ˙ = 2ap 2
y (10 - x) = -1 - 10 x Î 4 ˚

È 2 Êp ˆ˘
Íp - 12 b ÁË 2 - b˜¯ ˙ = 8ap
2
Ê 1 + 10 x ˆ fi
fi y=Á Î ˚
Ë 10 - x ˜¯
fi [p 2 - 6bp + 12b 2 ] = 8a p 2

(11,111) , (111,11) , (9, - 91) , ( -91, 9) Ê 2 bp p 2 ˆ 8a p 2 2ap 2


fi Á b - 2 + 12 ˜ = 12 = 3
Ë ¯
È Ê 10 -1 2 ˆ˘ a
47. Given Í cot Á Â cot (k + k + 1)˜ ˙ =
2
Ê pˆ p2 p2
Î Ë k =1 ¯˚ b fi ÁË b - ˜¯ = (8a - 1) +
4 12 16
a È Ê 10 ˆ˘
= Í cot Á Â cot -1 (k 2 + k + 1)˜ ˙
2
fi Ê pˆ p2
b Î Ë k =1 ¯˚ fi ÁË b - ˜¯ = (32a - 1)
4 48
È Ê 10 -1 Ê 1 ˆˆ˘
= Í cot Á Â tan Á
2
Ê -1 pˆ p2
Î Ë k =1 Ë 1 + k + k ˜¯ ˜¯ ˙˚
2 fi ÁË sin x - ˜ = (32a - 1)
4¯ 48
È Ê 10 -1 Ê ( k + 1) - k ˆ ˆ
˘ As we know that ,
= Í cot Á Â tan Á ˜ ˜ ˙
ÍÎ Ë k =1 Ë 1 + (k + 1)k ¯ ¯ ˙˚ p p
- £ sin -1 x £
È Ê 10 ˆ˘ 2 2
-1 -1
= Í cot Á Â [tan (k + 1) - tan (k )]˜ ˙ p p p p p
Î Ë k =1 ¯˚ fi - - £ sin -1 x - £ -
2 4 4 2 4
(
= ÈÎ cot tan (11) - tan (1) ˘˚
-1 -1
) 3p Ê -1 pˆ p
fi - £ Á sin x - ˜ £
4 Ë 4¯ 4
È Ê -1 Ê 11 - 1 ˆ ˆ ˘
= Í cot Á tan Á
Î Ë Ë 1 + 11. ˜¯ ˜¯ ˙˚ Ê pˆ
2
9p 2
fi 0 £ Á sin -1 x - ˜ £
Ë 4¯ 16
È Ê -1 Ê 5 ˆ ˆ ˘
= Í cot Á tan Á ˜ ˜ ˙
Î Ë Ë 6¯¯ ˚ p2 9p 2
fi 0£ (32a - 1) £
48 16
È Ê -1 Ê 6 ˆ ˆ ˘
= Í cot Á cot Á ˜ ˜ ˙ fi 0 £ (32a - 1) £ 27
Î Ë Ë 5¯¯ ˚
6 fi 1 £ 32a £ 28
=
5
1 7
(a + b + 10) is 21. fi £ a£
32 8
48. We have
x2 1
p-qˆ
-1 Ê Ê q-r ˆ Ê r - pˆ 50. Let g ( x) = =1-
tan Á + tan -1 Á + tan -1 Á x +1
2
1 + x2
˜
Ë 1 + pq ¯ ˜
Ë 1 + qr ¯ Ë 1 + rp ˜¯
Clearly, Rg = [0 ,1)
-1 -1 -1 -1
= (tan p - tan q ) + (tan q - tan r )
+p + (tan -1
r - tan -1
p)
-1
(
Now, R f = ( f (1), f (0)] = cot (1), cot (0) ˘˚
-1

=p
314 Comprehensive Trigonometry with Challenging Problems & Solutions for Jee Main and Advanced

Êp p ˘ 53. We have
= Á , ˙
Ë 4 2˚ 1 Ê 3 sin 2q ˆ
q = tan -1 (2 tan 2 q ) - sin -1 Á
Êb ˆ 2 Ë 5 + 4 cos 2q ˜¯
ÁË + 2˜¯
a 1 Ê 6 tan q ˆ
fi q = tan -1 (2 tan 2 q ) - sin -1 Á
Êp ˆ 2 Ë 9 + tan 2 q ˜¯
Á2 ˜
= Á + 2˜ = 2 + 2 = 4 Ê Ê tan q ˆ ˆ
p
Á ˜ 1 Á 2 ÁË 3 ˜¯ ˜
Ë4 ¯ fi
-1 -1
q = tan (2 tan q ) - sin Á
2
˜
2 Á 1 + 1 tan 2 q ˜
-1 -1 ÁË ˜¯
51. Given tan y = 4 tan x 3

Ê 4 x - 4 x3 ˆ 1 Ê tan q ˆ
-1
y = tan -1 Á fi q = tan -1 (2 tan 2 q ) - ¥ 2 tan -1 Á
fi tan ˜
Ë 1 - 6 x2 + x4 ¯ 2 Ë 3 ˜¯

Ê 4 x - 4 x3 ˆ Ê tan q ˆ
fi q = tan -1 (2 tan 2 q ) - tan -1 Á
fiy=Á ˜ Ë 3 ˜¯
Ë 1 - 6 x2 + x4 ¯
Ê tan q ˆ
1 Ê 1 - 6 x2 + x4 ˆ 2 tan 2 q -
fi =Á Á
q = tan -1 Á 3 ˜
y Ë 4 x - 4 x3 ˜¯ fi
tan q ˜
Á 1 + 2 tan 2 q ◊ ˜
1 Ë 3 ¯
fi = cot( 4q ), where , x = tan q
y
6 tan 2 q - tan q
fi tan(q ) =
1 p 3 + 2 tan 3 q
fiClearly, = 0 is zero only when q =
y 8
fi 3 tan(q ) + 2 tan 4 q - 6 tan 2 q + 2 tan q = 0
Hnece, x - 6 x + 1 = 0
4 2
fi 2 tan 4 q - 6 tan 2 q + 5 tan q = 0
a (a + b + c ) b (a + b + c )
52. Let x = ,y = fi tan q (2 tan 3 q - 6 tan q + 5) = 0
bc ac

c(a + b + c) fi tan q = 0 , (2 tan 3 q - 6 tan q + 5) = 0


and z =
ab fi q = np , n ΠI
Now, x + y + z - xyz 54. We have
Ê x cos q ˆ Ê cos q ˆ
Ê a c ˆ ( a + b + c )3 / 2 tan -1 Á - cot -1 Á
= (a + b + c) Á +
b
+ -
˜
Ë 1 - x sin q ¯ Ë x - sin q ˜¯
Ë bc ca ab ˜¯ abc
-1 Ê x cos q ˆ Ê x - sin q ˆ
= tan Á - tan -1 Á
Ê a + b + c ˆ (a + b + c) 3/ 2 ˜
Ë 1 - x sin q ¯ Ë cos q ˜¯
= (a + b + c) Á -
Ë abc ˜¯ abc
Ê x cos q x - sin q ˆ
-
(a + b + c) 3/ 2
(a + b + c) 3/ 2 -1 Á
1 - x sin q cos q ˜
= tan Á
= - x cos q x - sin q ˜
abc abc Á1 + ¥ ˜
Ë 1 - x sin q cos q ¯
=0
-1 Ê x cos q - x + x sin q + sin q - x sin q ˆ
2 2 2

Now, tan
-1
x + tan -1
y + tan -1
z = tan Á ˜
Ë cos q (1 - x sin q ) + x cos q ( x - sin q ) ¯
-1 Ê x + y + z - xyz ˆ
-1 Ê ( x + 1)sin q - 2 x sin q ˆ
2 2
= tan Á ˜ = tan -1 (0) = p
Ë 1 - xy - yz - zx ¯ = tan Á 2 ˜
Ë ( x + 1) cos q - x sin 2q ¯
Hence, the result.
Inverse Trigonometric Function 315

-1 Ê ( x - 2 x sin q + 1)sin q ˆ
2
= tan Á 2 -1 Ê (3r + 2) - (3r - 1) ˆ
10
˜ = Â tan Á
Ë ( x - 2 x sin q + 1) cos q ¯
r =1 Ë 1 + (3r + 2)(3r - 1) ˜¯
-1
= tan (tan q ) = q 10
-1 -1
55. Given equation is = Â [tan (3r + 2) - tan (3r - 1)]
r =1
Ê x 2 - 1ˆ Ê 2x ˆ Ê 2 x ˆ 2p
cos -1 Á 2 ˜ + sin -1 Á 2 ˜ + tan -1 Á 2 ˜ = -1 -1
= tan (32) - tan (2)
Ë x + 1¯ Ë x + 1¯ Ë x - 1¯ 3
2p -1 Ê 32 - 2 ˆ
p - 2 tan -1 x + 2 tan -1 x - 2 tan -1 x = = tan Á

3 Ë 1 + 32.2 ˜¯
2p -1 Ê 30 ˆ -1 Ê 6 ˆ
fi p - 2 tan -1 x = = tan Á ˜ = tan Á ˜
3 Ë 65 ¯ Ë 13 ¯
p
fi 2 tan -1 x = -1 Ê 13 ˆ
3 = cot Á ˜
Ë 6¯
p
fi tan -1 x = ( 2 m + n + 4)
6
Êp ˆ 1 = 26 + 6 + 4
fi x = tan Á ˜ =
Ë 6¯ 3 = 36.
Êp ˆ 1 58. We have S
Hence, the solution is x = tan Á ˜ =
Ë 6¯ 3 10 10
-1 Ê a ˆ
= Â Â tan Á ˜
56. We have b =1 a =1 Ë b¯
Ê y zˆ Ê z xˆ Ê x yˆ 10 È
tan -1 Á ˜ + tan -1 Á ˜ + tan -1 Á ˜ -1 Ê 1 ˆ -1 Ê 2 ˆ -1 Ê 3 ˆ -1 Ê 10 ˆ ˘
Ë xr ¯ Ë yr¯ Ë zr ¯ = Â Í tan Á ˜ + tan Á ˜ + tan Á ˜ + .... + tan Á ˜ ˙
b =1 Î Ë b ¯ Ë b ¯ Ë b ¯ Ë b ¯˚

Ê yz zx xy xyzˆ -1 Ê 1ˆ -1 Ê 1 ˆ -1 Ê 1 ˆ -1 Ê 1 ˆ
Á + + - 3 ˜ = tan Á ˜ + tan Á ˜ + tan Á ˜ + .... + tan Á ˜
Ë 1¯ Ë 2¯ Ë 3¯ Ë 10 ¯
-1 Á x r yr zr r ˜
= tan
Á Ê z 2 x2 y 2 ˆ ˜ Ê 2ˆ Ê 2ˆ Ê 2ˆ Ê 2ˆ
Á 1- Á 2 + 2 + 2 ˜ ˜
ÁË + tan -1 Á ˜ + tan -1 Á ˜ + tan -1 Á ˜ + .... + tan -1 Á ˜
Ër r r ¯ ˜¯ Ë 1¯ Ë 2¯ Ë 3¯ Ë 10 ¯

Ê yz zx xy xyzˆ Ê 3ˆ Ê 3ˆ Ê 3ˆ Ê 3ˆ
+ + - 3 ˜ + tan -1 Á ˜ + tan -1 Á ˜ + tan -1 Á ˜ + .... + tan -1 Á ˜
Á Ë 1¯ Ë 2¯ Ë 3¯ Ë 10 ¯
-1 x r yr zr r ˜
= tan Á
Á Ê x2 + y 2 + z 2 ˆ ˜ ...... .......... .........
Á 1- Á
ÁË r2 ˜ ˜˜
Ë ¯ ¯ ...... .......... .........
Ê yz zx xy xyzˆ ...... .......... .........
+ + - 3
Á
-1 x r yr zr r ˜
= tan Á ˜ Ê 10 ˆ Ê 10 ˆ Ê 10 ˆ Ê 10 ˆ
Á 1-1 ˜ + tan -1 Á ˜ + tan -1 Á ˜ + tan -1 Á ˜ + .... + tan -1 Á ˜
ÁË ˜¯ Ë1¯ Ë 2¯ Ë 3¯ Ë 10 ¯

p p p
-1
= tan (•) = = 10 ¥ + 45 ¥
2 4 2
10
-1 Ê 3 ˆ = 25p
57. Given  tan Á 2
r =1 Ë 9r + 3r - 1˜¯ (m + 4) is 29 .
10
-1 Ê 3 ˆ
= Â tan Á
r =1 Ë 1 + (3r + 2)(3r - 1) ˜¯
316 Comprehensive Trigonometry with Challenging Problems & Solutions for Jee Main and Advanced

59. We have 10
1 ( x + 1) Now, Â ( f (n) + f (n - 1))
f ( x) = (sin -1 x + cos -1 x + tan -1 x) + 2 n=2
p x + 2 x + 10 10
It will provide us the max value at x = 1 = Â (n p + (n - 1)p )
n=2
1 Êp ˆ 2
f (1) = Á + tan -1 (1)˜ + 10
pË2 ¯ 13 = Â ((2n - 1)p )
n=2
1 3p 2
= ¥ + = (3 + 5 + 7 + 9 + ... + 19)p
p 4 13
3 2 39 + 8 47 = (1 + 3 + 5 + 7 + 9 + ... + 19)p - p
= = + = =
= (10 )p - p
4 13 52 52 2

(104 m - 90) is 4.
= 99p
60. We have (a + 1) is 100.
sin(2 x) + cos(2 x) + cos x + 1 = 0
sin 2 x + (1 + cos 2 x) + cos x = 0 INTEGER TYPE QUESTIONS

-1
Ê Ê 2 x2 + 4 ˆ ˆ
Ê pˆ
1. Given sin Á sin Á 2 ˜˜ < p - 3
in Á 0, ˜ Ë Ë x +1 ¯¯
Ë 2¯
2 x2 + 4
So it has no solution fi >3
Thus, m = 0 x2 + 1
2 x2 + 4
È -1 Ê Ê 7p ˆ ˆ -1 Ê Ê 7p ˆ ˆ ˘ fi -3>0
Also, n = sin Í tan Á tan Á ˜ ˜ + cos Á cos Á ˜ ˜ ˙ x2 + 1
Î Ë Ë ¯
6 ¯ Ë Ë 3 ¯¯˚
2 x 2 + 4 - 3x 2 - 3
Êp p ˆ Êp ˆ fi >0
= sin Á + ˜ = sin Á ˜ = 1 x2 + 1
Ë 6 3¯ Ë 2¯
1 - x2
fi >0
x2 + 1
(m 2 + n 2 + m + n + 4) x2 - 1
fi <0
x2 + 1
= 0 +1 + 0 + 1 + 4
fi x Π( -1,1)
= 6.
n Ê (b - a + 5) = 1 + 1 + 5 = 7.
-1 Ê 1 ˆ -1 ˆ
61. We have f (n) = Â Á cot Á ˜ - tan (k )˜
k =- n Ë Ë k ¯ ¯ -1 -1
2. Given a sin x - b cos x = c
-1 Ê ˆ
-1 Ê 1 ˆ -1 Êp ˆ
= Â Á cot Á ˜ - tan (k )˜ fi a sin -1 x - b Á - sin -1 x˜ = c
k =- n Ë Ë k ¯ ¯ Ë2 ¯
n Ê ˆ
Ê 1ˆ bp
+ Â Á cot -1 Á ˜ - tan -1 (k )˜ fi (a + b)sin -1 x = c +
k =1 Ë Ë k¯ ¯ 2
-1 2c + bp
= Â (tan -1 (k ) + p - tan -1 (k )) fi sin -1 x =
k =- n 2(a + b)
n
+ Â (tan -1 (k ) - tan -1 (k )) -1 p 2c + bp
Now, cos x = -
k =1 2 2(a + b)
-1
(a + b)p - 2c - bp
= Â (p ) + 0 fi cos -1 x =
k =- n 2(a + b)
= np
Inverse Trigonometric Function 317

a p - 2c fi 36 x 4 + 12 x3 + x 2 = 1 - 13x 2 + 36 x 4
fi cos -1 x =
2( a + b )
fi 12 x3 + 14 x 2 - 1 = 0
-1 -1
Now, a sin x + b cos x
Thus, a = 12, b = 14, c = 0, d = –1
Ê 2c + bp ˆ Ê a p - 2c ˆ (b + c) - (a + d )
= aÁ + bÁ
˜
Ë 2( a + b ) ¯ Ë 2(a + b) ˜¯ = 14 – 11 = 3.
Ê 2ac + abp ˆ Ê ab p - 2bc ˆ 6. Given equation is
= Á +
Ë 2(a + b) ˜¯ ÁË 2(a + b) ˜¯ x3 - x 2 - 3 x + 4 = 0

Ê 2ac + abp + ab p - 2bc ˆ fi a + b + g = 1, ab + bg + ga = -3 , abg = -4


= Á ˜¯
Ë 2(a + b) It is given that ,
Ê ab p + c(a - b) ˆ tan -1 a + tan -1 b + tan -1 g = q
= Á ˜¯
Ë ( a + b)
Ê a + b + g - abg ˆ
fi tan -1 Á =q
Clearly, m = 1 Ë 1 - (ab + bg + ga ) ˜¯
(m 2 + m + 2) is 4
Ê1 + 4ˆ
fi tan -1 Á =q
Ë 1 + 3 ˜¯
So , both roots are negative 5
Thus m is a negative root fi tanq =
4
-1 -1 Ê 1 ˆ
Now, tan (m) + tan Á ˜ ( p + q) is 9
Ë m¯
7. Given equation is
-1 -1
= tan (m) - p + cot (m) cos-1 x + cos -1 (2 x) + p = 0
-1 -1
= -p + (tan (m) + cot (m)) fi cos -1 (2x) + cos -1 ( x) = -p
p p
= -p + =- fi cos -1 (2 x ◊ x - 1 - x 2 1 - 4 x 2 ) = -p
2 2
Clearly, k = –1 fi (2 x 2 - 1 - x 2 1 - 4 x 2 ) = cos( -p ) = -1
(k + 4) is 3.
4. Given equation is fi (2 x 2 + 1)2 = (1 - x 2 )(1 - 4 x 2 )
p
sin -1 ( x 2 - 2 x + 1) + cos -1 ( x 2 - x) = fi 4 x4 + 4 x2 + 1 = 1 - 5x2 + 4 x4
2
It is true only when fi 9 x2 = 0
( x 2 - 2 x + 1) = x 2 - x fi x=0
fi x =1 But x = 0
So it has no solution
M=0
-1 -1 -1
5. Given cos ( x) + cos (2 x) + cos (3x) = p
-1 Ê 5 ˆ -1 Ê 12 ˆ p
-1 -1 -1 Again, sin Á ˜ + sin Á ˜ = ,
fi cos (3x) + cos (2 x) = p - cos ( x) Ë x¯ Ë x¯ 2

fi cos -1 (3x ◊ 2 x - 1 - 9 x 2 1 - 4 x 2 ) = cos -1 ( - x) Ê 5ˆ Ê 144 ˆ p


fi sin -1 Á ˜ + cos -1 Á 1 - 2 ˜ =
Ë x¯ Ë x ¯ 2
fi (3x ◊ 2 x - 10 - 9 x 2 1 - 4 x 2 ) = ( - x) It is true only when,

fi (6 x 2 + x ) 2 = ( - 1 - 9 x 2 1 - 4 x 2 ) 2 Ê 5ˆ 144
ÁË ˜¯ = 1 - 2
x x
318 Comprehensive Trigonometry with Challenging Problems & Solutions for Jee Main and Advanced

25 144 Ï p
fi =1-
x 2
x 2 ÔÔ 3a + b = 2
Ì
Ôa + 2b = 11p
169
fi =1
x2 ÔÓ 6
fi x 2 = 169 On solving, we get,
fi x = ±13 p
a=- and b = p
6
Claerly, x = 13
p
Thus, N =1 fi sin -1 (log 2 x) = - and cos -1 (log 2 y ) = p
M + N + 4 = 5. 6
8. We have 1
fi (log 2 x) = - and (log 2 y ) = -1
2
È ˆ˘
Î
Ê1
4 cos Í cos -1 Á
Ë4 ( ˆ
) Ê1
6 - 2 ˜ - cos -1 Á
¯ Ë4 ( )
6 + 2 ˜˙
¯˚ -
1
fi x=2 2 and y = 2-1
È -1 Ê 3 - 1ˆ -1 Ê 3 + 1ˆ
˘
1 1
= 4 cos Í cos Á ˜ - cos Á ˜˙ fi = 2 and = 2
ÍÎ Ë 2 2 ¯ Ë 2 2 ¯ ˙˚ x y
-1 -1 1 1
= 4 cos[cos (cos(75∞)) - cos (cos(15∞))] fi + +2=2+4+2=8
x 2
y2
= 4 cos(75∞ - 15∞)
11. Here, both roots are negative
= 4 cos(60∞)
-1 -1
Now, cot(cot a + cot b )
= 2.
9. We have Ê -1 Ê 1 ˆ -1 Ê 1 ˆ ˆ
= cot Á p + tan Á ˜ + p + tan Á ˜ ˜
Ê 5 ˆ Ë Ëa ¯ Ë b ¯¯
5 cot Á Â cot -1 (k 2 + k + 1)˜
Ë k =1 ¯ Ê -1 Ê 1 ˆ -1 Ê 1 ˆ ˆ
= cot Á 2p + tan Á ˜ + tan Á ˜ ˜
Ê 5 ˆˆ Ë Ëa ¯ Ë b ¯¯
-1 Ê 1
= 5 cot Á  tan Á ˜˜
Ë k =1 Ë 1+ k + k ¯¯
2
Ê -1 Ê 1 ˆ -1 Ê 1 ˆ ˆ
= cot Á tan Á ˜ + tan Á ˜ ˜
Ê 5 Ë Ëa ¯ Ë b ¯¯
-1 Ê ( k + 1) - k ˆ ˆ
= 5 cot Á Â tan Á
Ë k =1 Ë 1 + (k + 1)k ˜¯ ˜¯ Ê -1 Ê a + b ˆ ˆ
= cot Á tan Á
Ê 5
-1 -1 ˆ Ë Ë ab - 1˜¯ ˜¯
= 5 cot Á Â [tan (k + 1) - tan k ]˜
Ë k =1 ¯ Ê -1 Ê ab - 1ˆ ˆ
= cot Á cot Á
-1
= 5 cot(tan (6) - tan (1))
-1
Ë Ë a + b ˜¯ ˜¯

Ê -1 Ê 6 - 1 ˆ ˆ Ê -1 Ê ab - 1ˆ ˆ
= 5 cot Á tan Á = cot Á cot Á
Ë Ë 1 + 6.1˜¯ ˜¯ Ë Ë a + b ˜¯ ˜¯

Ê -1 Ê 5 ˆ ˆ Ê 44 + 1ˆ
= Á =9
= 5 cot Á tan Á ˜ ˜
Ë Ë 7¯¯ Ë 5 ˜¯

Ê -1 Ê 7 ˆ ˆ
12. Given equation is
= 5cot Á cot Á ˜ ˜ Ê 1ˆ
Ë Ë 5¯¯ Ê 1ˆ Ê 1ˆ
tan -1 Á ˜ + tan -1 Á ˜ = tan -1 Á ˜
Ë x¯ Ë y¯ Ë 7¯
= 7.
-1 -1
10. a = sin (log 2 x) and b = cos (log 2 x) Ê x+ y ˆ Ê 1ˆ
fi tan -1 Á = tan -1 Á ˜
The given equations reduces to Ë x y - 1˜¯ Ë 7¯
Inverse Trigonometric Function 319

Ê x + y ˆ Ê 1ˆ Ï -1 Ê 1 ˆ p¸
fi ÁË x y - 1˜¯ = ÁË 7 ˜¯ 2. Given Ì2 tan Á ˜ - ˝
Ë 5¯ 4 ˛
Ó
fi 7x + 7 y = x y - 1 Ê Ê ˆ
2 ˆ
Á Á ˜ p˜
Ê 7 x + 1ˆ = tan Á tan Á
-1 5 ˜- ˜
fi y=Á
Ë x - 7 ˜¯ Á Á Ê 1ˆ 2 ˜ 4 ˜
Á Á1- Á ˜ ˜ ˜
Hence, the possible ordered pairs are Ë Ë Ë 5¯ ¯ ¯
(8, 57), (9, 32), (12,17), (17,12), (32, 9), (57, 8) Ê -1 Ê 5 ˆ -1 ˆ
= tan Á tan Á ˜ - tan (1)˜
Ë Ë 12 ¯ ¯
Ê Ê 5 ˆˆ
Á -1 Á 12 - 1 ˜ ˜
QUESTION ASKED IN PART IIT-JEE EXAMS
1. Now, = tan Á tan Á
5 ˜˜
Á Á 1 + .1˜ ˜
Ê a(a + b + c ˆ Ê b( a + b + c ) ˆ Ë Ë 12 ¯ ¯
tan -1 Á ˜ + tan -1 Á ˜
Ë bc ¯ Ë ca ¯ Ê -1 Ê 7 ˆ ˆ
= tan Á tan Á - ˜ ˜
Ë Ë 17 ¯ ¯
Ê a(a + b + c b( a + b + c ) ˆ
Á + ˜
-1 bc ca Ê 7ˆ
= tan Á ˜ = Á- ˜
Á 1 - a ( a + b + c ) b( a + b + c ) ˜ Ë 17 ¯
ÁË ˜¯
bc ca 3. Now, cos(2cos–1x + sin–1x)
Ê Ê (a + b + c) ˆ Ê a bˆ ˆ = cos(cos–1x + sin–1x + cos–1x)
Á ÁË ˜¯ + Á + ˜
c Ë b a ˜¯ ˜ Êp -1 ˆ
= cos Á + cos x˜
= tan Á
-1
Á Ë2 ¯
(a + b + c) (a + b + c) ˜
Á1- ˜
ÁË c c ¯˜ = –sin(cos–1x)

Ê Ê (a + b + c) ˆ Ê a + b ˆ ˆ
ÁÁ ˜¯ + ÁË ˜
= - sin sin(-1
1 - x2 )
-1 Ë c ab ¯ ˜
= tan Á ˜
Á Ê c - a - b - cˆ ˜ = - 1- x
2

ÁË ÁË ˜¯ ˜¯
c When x
Ê 1 ˆ 2 4
Ê Ê (a + b + c) ˆ Ê a + b ˆ ˆ is - Á 1 - ˜ = - .
ÁÁ ˜¯ + ÁË ˜ Ë 25 ¯
ab ¯ ˜
5
-1 Ë c
= tan Á ˜
Á Ê a + bˆ ˜ È -1 Ê 4 ˆ -1 Ê 2 ˆ ˘
ÁË ÁË - ˜¯ ˜¯ 4. Given tan Ícos Á ˜ + tan Á ˜ ˙
c Î Ë 5 ¯ Ë 3¯ ˚

-1 Ê c(a + b + c) ˆ Ê -1 Ê 4 ˆ -1 Ê 2 ˆ ˆ
= tan Á - ˜ = tan Á tan Á ˜ + tan Á ˜ ˜
Ë ab ¯ Ë Ë 5¯ Ë 3¯ ¯

-1 Ê c ( a + b + c ) ˆ
Ê Ê 3 2 ˆˆ
= - tan Á ˜ Á -1 Á 4 + 3 ˜ ˜
Ë ab ¯ = tan Á tan Á
3 2 ˜˜
Á Á1- ◊ ˜˜
q Ë Ë 4 3 ¯¯

-1 Ê c ( a + b + c ) ˆ Ê c(a + b + c) ˆ Ê 9+8 ˆ
= - tan Á + tan -1 Á =Á
Ë ab ˜
¯ Ë ab ˜
¯ Ë 12 - 6 ˜¯
=0 17
= -
Thus, tan q = 0 6
320 Comprehensive Trigonometry with Challenging Problems & Solutions for Jee Main and Advanced

5. No questions asked in between 1984 to 1985 11. No questions asked in 2000.


12. We have
Ê Ê 2p ˆ ˆ
6. Given sin 1Á sin Á ˜ ˜ Ê x 2 x3 ˆ Ê 2 x 4 x6 ˆ
Ë Ë 3 ¯¯ x - + -
Á ..˜ = Áx - + - ..˜
Ë 2 4 ¯ Ë 2 4 ¯
Ê Ê p ˆˆ
= sin 1Á sin Á p - ˜ ˜
Ë Ë 3¯¯ Ê x x2 ˆ Ê x2 x4 ˆ
fi x Á1 - + - ..˜ = x 2 Á1 - + - ..˜
Ê Ê p ˆˆ Ë 2 4 ¯ Ë 2 4 ¯
= sin 1Á sin Á ˜ ˜
Ë Ë 3¯¯
Ê ˆ Ê ˆ
Êpˆ Á ˜ Á ˜
=Á ˜ fi xÁ
1
˜ = x
2Á 1 ˜
Ë 3¯ Á 2ˆ˜
Ê
Á1- - ˜x ˆ Ê x
7. No questions asked in between 1987 to 1988 ÁË ÁË 2 ˜¯ ˜¯ Á1- Á - ˜ ˜
ÁË Ë 2 ¯ ˜¯
-1 -1 2p
8. We have A = 2 tan (2 2 - 1) = > 2 tan ( 3 ) = Ê ˆ
3 Ê ˆ
Á 1 ˜ Á ˜
2Á 1 ˜
-1 Ê 1 ˆ -1 Ê 3 ˆ fi xÁ ˜ = x
and B = 3 sin Á ˜ + sin Á ˜ Á1+ Ê xˆ ˜ Á Ê x2 ˆ ˜
Ë 3¯ Ë 5¯ ÁË ÁË 2 ˜¯ ˜¯ Á1+ Á ˜ ˜
ÁË Ë 2 ¯ ˜¯
-1
Ê Ê 1ˆ Ê 1ˆ ˆ
3
-1 Ê 3 ˆ
= sin Á 3 Á ˜ - 4 Á ˜ ˜ + sin Á ˜ Ê x2 ˆ xˆ
Ë
Ë 3 ¯ Ë ¯
3 ¯ Ë 5¯ Ê
fi x Á1 + ˜ = x 2 Á1 + ˜
Ë 2¯ Ë 2¯
-1 Ê 23 ˆ -1 Ê 3 ˆ
= sin Á ˜ + sin Á ˜ fi x(2 + x2) = x2(2 + x)
Ë 27 ¯ Ë 5¯
fi (2x + x3) = (2x2 + x3)
-1 Ê3ˆ Ê 3ˆ
< sin Á ˜ + sin -1 Á ˜ fi x2 = x
Ë 2 ¯ Ë 2 ¯
fi x(x – 1) = 0
2p
= fi x = 0, 1
3
Hence, the solution set is {0, 1}
Thus, A > B 13. We have cos(tan–1(sin(cot–1x)))
10. Ans. (c)
Given Ê Ê Ê Ê 1 ˆˆˆˆ
-1 -1
= cos Á tan Á sin Á sin Á ˜˜˜˜
p ÁË ÁË Ë Ë x 2 + 1 ¯ ¯ ˜¯ ˜¯
tan -1 x( x + 1) + sin -1 x 2 + x + 1 =
2
Ê -1 Ê 1 ˆ ˆ

Ê
-1
cos Á
1 ˆ
˜ + sin
Ë x + x + 1¯
2
-1
( )
x + x +1 =
2 p
2
= cos Á tan Á
Ë
˜˜
Ë x2 + 1 ¯ ¯

Ê Ê x2 + 1 ˆ ˆ
Ê
Thus, Á
1 ˆ
˜ =
Ë x2 + x + 1 ¯
( x2 + x + 1 ) ÁË
-1
= cos Á cos Á 2 ˜˜
ÁË x + 2 ˜¯ ˜¯

( ) Ê x2 + 1 ˆ
2
fi x2 + x + 1 =1 =Á 2 ˜
ÁË x + 2 ˜¯
fi x2 + x + 1 = 1 14. Ans. (d)
fi x2 + x = 0 p
Given f (x) = sin -1 (2 x) +
fi x(x + 1) = 0 6
fi x = 0, –1 p
sin -1 (2 x) + ≥0
6
Inverse Trigonometric Function 321

-1 p fi x2 + y2 = 1
sin (2 x) ≥ -
6 Ans. (P)
Ê pˆ 1 (B) When a = 1, b = 1, then
(2 x) ≥ sin Á - ˜ = -
Ë 6¯ 2 p
sin–1x + cos–1y + cos–1(xy) =
1 2
x≥-
4 p p
fi _ cos–1x + cos–1y + cos–1(xy) =
Also, –1 £ 2x £ 1 2 2
fi cos–1x = cos–1y + cos–1(xy)
1 1
- £x£
2 2 fi cos–1x – cos–1y = cos–1(xy)

È 1 1˘
fi (
cos -1 xy - 1 - x 2 1 - y 2 = cos–1(xy) )
= Í- , ˙
Î 4 2˚
15. Given sin(cot–1(x + 1)) = cos(tan–1x)
fi ( xy - )
1 - x 2 1 - y 2 = (xy)

fi 1 - x2 1 - y 2 = 0
Ê Ê 1 ˆˆ
fi sin Á sin -1 Á ˜˜
Ë Ë x2 + 2 x + 2 ¯ ¯ fi (1 – x2)(1 – y2) = 0

Ê -1 Ê 1 ˆ ˆ Ans. (Q)
= cos Á cos Á ˜˜ (C) When a = 1, b = 2, then
Ë Ë x2 + 1 ¯ ¯
p
1 1 fi sin–1x + cos–1y + cos–1(2xy) =
fi = 2
x2 + 2 x + 2 x2 + 1 p p
fi _ cos–1x + cos–1y + cos–1(2xy) =
2 2
fi x2 + 2 x + 2 = x2 + 1

fi x2 + 2x + 2 = x2 + 1
fi (
cos -1 xy - 1 - x 2 1 - y 2 = cos–1(2xy) )
fi 2x + 2 = 1
1
fi ( xy - 1 - x 1 - y ) = (2xy)
2 2

fi x= -
( 1 - x 1 - y ) = (–xy)
2
2 fi
2 2 2

16. No questions asked in between 2005 to 2006.


fi (1 – x2)(1 – y2) = x2y2
p
17. Given sin–1(ax) + cos–1(y) + cos–1(bxy) = fi 1 – x2 – y2 + x2y2 = x2y2
2
(A) when a = 1, b = 0, then fi x2 + y2 = 1
p Ans. (P)
sin–1(x) + cos–1(y) + cos–1(0) =
2 (D) When a = 2, b = 2, then
–1 –1 p p
fi sin (x) + cos (y) + = p
2 2 fi sin–1(2x) + cos–1y + cos–1(2xy) =
2
fi sin–1(x) + cos–1(y) = 0
p p
fi sin–1(x) = –cos–1(y) = - sin
-1
( 1 - y2 ) fi
2
– cos–1(2x) + cos–1y + cos–1(2xy) =
2

fi x = - 1- y
2 fi cos–1(2x) = cos–1y + cos–1(2xy)

2 2
fi cos–1(2x) – cos–1y = cos–1(2xy)
fi x =1–y
fi (
cos1 2 x. y - 1 - 4 x 2 1 - y 2 = cos–1(2xy) )
322 Comprehensive Trigonometry with Challenging Problems & Solutions for Jee Main and Advanced

fi (2x.y - 1 - 4x 1 - y ) = (2xy) 2 2
Now,
d
d (tan q )
(tan q ) = 1

fi ( 1 - 4x 1 - y ) = 0
2 2 21. No questions asked in 2010
n
2 2 22. Now, Â (2k )
fi (1 – 4x )(1 – y ) = 0 k -1

Ans. (S) = 2(1 + 2 + 3 + … + n)


18. We have Ê n(n + 1) ˆ
= 2Á
1+ x 2 –1 –1 2 1/2 Ë 2 ˜¯
¥ x cos (cot x) + sin (cot x)
= (n2 + n)
ÈÏ Ê -1 Ê x ˆˆ
2 Ô 23
= 1 + x ÍÌ x cos Á cos Á ˜˜  cot -1 (1 + n + n 2 )
ÍÔ Ë Ë 1 + x2 ¯ ¯
ÎÓ n -1

-1 Ê ˆ
1/ 2 23 1
Ê -1 Ê 1 ˆ ˆ ¸Ô
2 ˘ = Â tan Á
˙ Ë 1 + n + n ˜¯
2
= + sin Á sin Á ˜ ˜ ˝ - 1˙ n -1
Ë Ë x 2 + 1 ¯ ¯ Ô˛
˙˚ 23
-1 Ê 1 ˆ
= Â tan Á
1/ 2 n -1 Ë 1 + (n + 1)n ˜¯
ÈÊ ˆ
2 ˘
x2 1
= 1 + x ÍÁ + - ˙ -1 Ê ( n + 1) - n ˆ
2 23
˜ 1
ÍË x 2 + 1 x2 + 1 ¯ ˙ = Â tan Á
Î ˚ n -1 Ë 1 + (n + 1)n ˜¯
1/ 2 23
ÈÊ 2 ˆ
2 ˘ -1 -1
= Â (tan (n + 1) - tan (n))
2 Í x +1 ˙
= 1+ x Á - 1
ÍË x 2 + 1 ˜¯
n -1
˙
Î ˚ = (tan–1(2) – tan–1(1)) + (tan–1(3) – tan–1(2))
+ … + (tan–1(24) – tan–1(23))
( )
1/ 2
È 2 ˘
= 1+ x Í x +1 - 1˙
2 2

Î ˚ = tan–1(24) – tan–1(1)

-1 Ê 24 - 1ˆ -1 Ê 23 ˆ
( )
1/ 2
= 1 + x ÈÎ x + 1 - 1˘˚
2 2 = tan Á ˜ = tan Á ˜
Ë 1 + 24 ¯ Ë 25 ¯
Thus, the given expression reduces to
= x 1+ x
2

Ê -1 Ê 23 ˆ ˆ
Ans. (c) = cot Á tan Á ˜ ˜
Ë Ë 25 ¯ ¯
19. No questions asked in between 2009 to 2010.
Ê -1 Ê 25 ˆ ˆ
Ê -1 Ê sin q ˆ ˆ = cot Á cot Á ˜ ˜
20. Given f (q) = sin Á tan Á Ë Ë 23 ¯ ¯
Ë Ë cos 2q ˜¯ ˜¯
25
Ê -1 Ê ˆˆ =
sin q 23
= sin Á sin Á ˜˜
Ë Ë sin 2 q + cos 2q ¯ ¯ Ê cos(tan -1 y ) + y sin(tan -1 y ) ˆ
23. We have Á ˜
Ê sin q ˆ Ë cot(sin -1 y ) + tan(sin -1 y ) ¯
=Á ˜
Ë sin 2 q + cos 2 q - sin 2 q ¯ Ê Ê Ê 1 ˆˆ Ê ˆ
Ê y ˆˆ
Á cos Á cos -1 Á -1
˜ ˜ + y sin Á sin Á ˜˜ ˜
Ê sin q ˆ Á ÁË ÁË y 2 + 1 ˜¯ ˜¯ ÁË ÁË 1 + y 2 ˜¯ ¯˜ ˜
=Á ˜ =Á ˜
Ë cos 2 q ¯ Á Ê Ê 1 - y2 ˆ ˆ Ê Ê ˆ ˆ ˜
y
Á cot Á cot -1 Á -1
˜ ˜ + tan Á tan Á ˜˜ ˜
Ê sin q ˆ ÁË ÁË ÁË y ˜¯ ˜¯ ÁË ÁË 1 - y 2 ˜¯ ˜¯ ˜
=Á = tan q ¯
Ë cos q ˜¯
Inverse Trigonometric Function 323

QUESTIONS WITH SOLUTIONS OF PAST


ÊÊ 1 ˆ Ê y ˆˆ IIT-JEE EXAMS FROM 1981 TO 2015
ÁÁ ˜ + yÁ ˜˜
Á ÁË y 2 + 1 ˜¯ ÁË 1 + y 2 ˜¯ ˜
1. Let a, b, c positive real numbers such that
=Á ˜
Á Ê 1 - y2 ˆ Ê y ˆ ˜ a (a + b + c )
Á Á ˜ +Á ˜ ˜ q = tan -1
ÁË ÁË y ˜¯ ÁË 1 - y 2 ˜¯ ˜¯ bc
b (a + b + c ) c (a + b + c )
Ê Ê 1 + y2 ˆ ˆ + tan -1 + tan -1 .
Á Á ˜ ˜ ca ab
Á ÁË y 2 + 1 ˜¯ ˜ Then tan q
=Á ˜
Á Ê 1 - y2 + y2 ˆ ˜ Soln. Now,
ÁÁ ˜˜
ÁË ÁË y 1 - y 2 ˜¯ ˜¯ a (a + b + c ) b (a + b + c )
Let x = , y=
bc ca
= y ( 1- y ) 4

and z =
c (a + b + c )
ab
Thus, the given expression reduces to
The given equation reduces to

( )
1/ 2
Ê 1 2 ˆ q = tan -1 x + tan -1 y + tan -1 z
= Á 2 y 1- y + y4 ˜
4
Ëy ¯
Ê x + y + z - xyz ˆ
q = tan -1 Á
Ë 1 - xy - yz - zx ˜¯
1/ 2
Ê y (1 - y )
2 ˆ 4
=Á + y4 ˜
Ë y 2
¯ Ê x + y + z - xyz ˆ
tanq = Á
4
= ((1 – y ) + y ) 4 1/2 Ë 1 - xy - yz - zx ˜¯

=1 Now, x + y + z

24. Given cot sin


-1
(
1 - x 2 = sin tan -1 ( x 6 ) ) ( ) =
a (a + b + c )
bc
+
b (a + b + c )
ca
+
c (a + b + c )
ab
Ê Ê x ˆˆ
fi cot Á cot -1 Á ˜˜ a 2 (a + b + c ) b2 (a + b + c ) c 2 (a + b + c )
Ë Ë 1 - x2 ¯ ¯ = + +
abc abc abc
Ê -1 Ê x 6 ˆ ˆ
= sin Á sin Á a+b+c
Ë
˜˜
Ë 6 x2 + 1 ¯ ¯
= (a + b + c )
abc
Ê x ˆ Ê x 6 ˆ Also, x y z
fi Á ˜ =Á ˜
Ë 1 - x2 ¯ Ë 6 x2 + 1 ¯ a (a + b + c ) b (a + b + c ) c (a + b + c )
= . .
bc ca ab
Ê 1 ˆ Ê 6 ˆ
fi Á ˜ =Á ˜
Ë 1 - x ¯ Ë 6x + 1¯
2 2 (a + b + c ) (a + b + c )
= ¥ abc
fi 6(1 – x2) = (6x2 + 1) (abc )2
fi 6 – 6x2 = 6x2 + 1 (a + b + c ) (a + b + c )
=
fi 12x = 5 2
(abc )
5 Thus, x + y + z - xyz = 0
fi x2 =
12 tanq = 0 .
12
fi x= ±
5
324 Comprehensive Trigonometry with Challenging Problems & Solutions for Jee Main and Advanced

Ê -1 Ê 3 ˆ -1 Ê 2 ˆ ˆ
Ï Ê 1ˆ p ¸ = tan Á tan ÁË ˜¯ + tan ÁË ˜¯ ˜
tan -1 Ì2 tan -1 Á ˜ - ˝ Ë 4 3 ¯
Ó Ë 5¯ 4 ˛
Ê Ê 3 2 ˆˆ
Ï Ê 1ˆ p ¸
Soln. Given tan Ì2 tan -1 Á ˜ - ˝ Á -1 Á 4 + 3 ˜ ˜
= tan Á tan Á
Ó Ë 5¯ 4 ˛ 3 2 ˜˜
Á Á1- . ˜˜
Ê Ê ˆ Ë Ë 4 3 ¯¯
2 ˆ
Á Á ˜ p˜
5 ˜- ˜ Ê 9+8 ˆ
= tan Á tan Á
-1

Á Á Ê 1ˆ 2 ˜ 4 ˜ Ë 12 - 6 ˜¯
Á Á1- Á ˜ ˜ ˜
Ë Ë Ë 5¯ ¯ ¯ 17
= -
Ê -1 Ê 5 ˆ ˆ 6
= tan Á tan ÁË ˜¯ - tan (1)˜
-1
Ë 12 ¯ Ê Ê 2p ˆ ˆ
sin -1 Á sin Á ˜ ˜ is
Ê Ê 5 ˆˆ Ë Ë 3 ¯¯
Á -1 Á 12 - 1 ˜ ˜
= tan Á tan Á 2p 2p p 5p
5 ˜˜ (a) - (b) (c) (d)
Á Á 1 + .1˜ ˜ 3 3 3 3
Ë Ë 12 ¯ ¯
Ê Ê 7 ˆˆ
= tan Á tan -1 Á - ˜ ˜
Ë Ë 17 ¯ ¯ Ê Ê 2p ˆ ˆ
Soln. Given sin -1 Á sin Á ˜ ˜
Ê 7ˆ Ë Ë 3 ¯¯
= ÁË - ˜¯
17
Ê Ê p ˆˆ
= sin -1 Á sin Á p - ˜ ˜
( -1
cos 2 cos x + sin -1
)
x at x = 1/5, Ë Ë 3¯¯
p p Ê Ê p ˆˆ
where 0 £ cos -1 x £ p and - £ sin -1 x £ . = sin -1 Á sin Á ˜ ˜
2 2 Ë Ë 3¯¯

(
Soln. Now, cos 2 cos -1 x + sin -1 x ) Êpˆ
=Á ˜
Ë 3¯
( -1 -1 -1
= cos cos x + sin x + cos x )
Êp -1 ˆ
= cos ÁË + cos x˜¯
2 (
A = 2 tan -1 2 2 - 1 and )
(-1
= - sin cos x ) Ê 1ˆ Ê 3ˆ
B = 3 sin -1 Á ˜ + sin -1 Á ˜
Ë 3¯ Ë 5¯
(
= - sin sin -1 1 - x 2 ) Soln. We have A = 2 tan -1 2 2 - 1 ( )
= - 1- x 2

When x > 2 tan -1 ( 3 ) = 23p


Ê 1 ˆ 2 4
expression is - Á 1 - ˜ = - . Ê 1ˆ Ê 3ˆ
Ë 25 ¯ 5 and B = 3 sin -1 Á ˜ + sin -1 Á ˜
Ë 3¯ Ë 5¯
È Ê 4ˆ Ê 2ˆ ˘
tan Ícos -1 Á ˜ + tan -1 Á ˜ ˙ is
Î Ë 5¯ Ë 3¯ ˚ -1
Ê Ê 1ˆ Ê 1ˆ ˆ
3
-1 Ê 3 ˆ
(a) 6/17 (b) 17/6 (c) –17/6 (d) –6/17 = sin Á 3 ÁË 3 ˜¯ - 4 ÁË 3 ˜¯ ˜ + sin ÁË 5 ˜¯
Ë ¯

È Ê 4ˆ Ê 2ˆ ˘ -1 Ê 23 ˆ -1 Ê 3 ˆ
Soln. Given tan Ícos -1 Á ˜ + tan -1 Á ˜ ˙ = sin ÁË ˜¯ + sin ÁË ˜¯
Î Ë 5¯ Ë 3¯ ˚ 27 5
Inverse Trigonometric Function 325

-1 Ê 3 ˆ -1 Ê 3 ˆ Ê ˆ Ê ˆ
< sin Á ˜ + sin Á ˜ Á ˜
Ë 2 ¯ Ë 2 ¯ Á 1 ˜ 1
xÁ ˜=x
2Á ˜
fi Á Ê x2 ˆ ˜
2p Á1- Ê - xˆ ˜
= ÁË ÁË 2 ˜¯ ˜¯ Á1- Á - ˜ ˜
3 ÁË Ë 2 ¯ ˜¯
Thus, A > B. Ê ˆ
Ê ˆ
Á 1 ˜ Á ˜
p 2Á 1 ˜
tan -1 x ( x + 1) + sin -1 x 2 + x + 1 = is fi xÁ ˜=x
2 Á1+ Ê xˆ ˜ Á Ê x2 ˆ ˜
ÁË ÁË 2 ˜¯ ˜¯ Á1+ Á ˜ ˜
ÁË Ë 2 ¯ ˜¯
(a) 0 (b) 1
(c) 2 (d) •
Ê x2 ˆ Ê xˆ
Soln. Given fi x Á1 + ˜ = x 2 Á1 + ˜
Ë 2 ¯ Ë 2 ¯
p
tan -1 x ( x + 1) + sin -1 x 2 + x + 1 =
2 fi ( )
x 2 + x 2 = x 2 (2 + x )

-1
Ê
fi cos Á 2
1 ˆ
˜ + sin
-1
( )
x2 + x + 1 =
p fi (2 x + x ) = (2 x
3 2
+ x3 )
Ë x + x + 1¯ 2
fi x =x2

fi x ( x - 1) = 0
Thus,
Ê
Á 2
1 ˆ
˜=
Ë x + x + 1¯
( x2 + x + 1 ) fi x = 0, 1
Hence, the solution set is {0, 1}.

( ) =1 ( ( ( ) )) =
2
x2 + 1
fi x2 + x + 1 9. Prove that cos tan -1 sin cot -1 x .
x2 + 2
fi x2 + x + 1 = 1
fi x2 + x = 0 ( ( (
Soln. We have cos tan -1 sin cot -1 x ) ))
fi x ( x + 1) = 0 Ê Ê Ê Ê 1 ˆˆˆˆ
-1 -1
= cos Á tan Á sin Á sin Á ˜˜˜˜
fi x = 0, –1 ÁË ÁË Ë Ë x 2 + 1 ¯ ¯ ˜¯ ˜¯
Ê x 2 x3 ˆ
sin -1 Á x - + - .......˜ Ê -1 Ê 1 ˆ ˆ
Ë 2 4 ¯ = cos Á tan Á ˜˜
Ë Ë x2 + 1 ¯ ¯
Ê x 4 x6 ˆ p
+ cos -1 Á x 2 - + - ....˜ = , Ê Ê x2 + 1 ˆ ˆ
Ë 2 4 ¯ 2 -1
= cos Á cos Á 2 ˜˜
ÁË ÁË x + 2 ˜¯ ˜¯
0 < x < 2 , then x is
(a) 1/2 (b) 1 Ê x2 + 1 ˆ
(c) –1/2 (d) –1 =Á 2 ˜
ÁË x + 2 ˜¯
Soln. We have
Ê x 2 x3 ˆ Ê 2 x 4 x6 ˆ p
f ( x ) = sin -1 ( 2 x ) + is
Á x - 2 + 4 - ..˜ = Á x - 2 + 4 - ....˜ 6
Ë ¯ Ë ¯
Ê ˆ Ê ˆ Ê 1 1˘ È 1 3ˆ
x x2 x2 x4 (a) ÁË - , (b) Í- , ˜¯
fi x Á1 - + - ..˜ = x 2 Á1 - + - ....˜ 2 2 ˙˚ Î 4 4
Ë 2 4 ¯ Ë 2 4 ¯
È 1 1˘ È 1 1˘
(c) Í- , (d) Í- ,
Î 4 4 ˙˚ Î 4 2 ˙˚
326 Comprehensive Trigonometry with Challenging Problems & Solutions for Jee Main and Advanced

a = 1 and b = 1, then (x, y)


p
Soln. Given f ( x ) = sin ( 2 x ) + ( )( )
-1
(Q) lies on x 2 - 1 y 2 - 1 = 0
6
p a = 1 and b = 2, then (x, y)
sin -1 ( 2 x ) + ≥0 (R) lies on the line y = x
6
p a = 2 and b = 2, then (x, y)
sin -1 ( 2 x ) ≥ -
6 (S) ( )(
lies on 4 x 2 - 1 y 2 - 1 = 0 )

(2 x ) ≥ sin ÊÁË -
1
˜ =-
6¯ 2 p
Soln. Given sin -1 ( a x ) + cos -1 ( y ) + cos -1 (b x y ) = .
1 2
x≥- (A) When a = 1, b = 0, then
4
p
Also, -1 £ 2 x £ 1 sin -1 ( x ) + cos -1 ( y ) + cos -1 (0) =
2
1 1 p p
- £x£ fi sin -1 ( x ) + cos -1 ( y ) + =
2 2 2 2
fi sin -1 ( x ) + cos -1 ( y ) = 0

( 1- y )
È 1 1˘
= Í- , ˙ sin -1 ( x ) = - cos -1 ( y ) = - sin -1 2
Î 4 2˚ fi

( )
sin cot -1( x + 1) = cos tan -1 x ( ) x is fi x = - 1 - y2

(a) –1/2 (b) 1/2 (c) 0 (d) 9/4 fi x2 = 1 - y 2


fi x2 + y 2 = 1
Soln. Given sin cot ( -1
( x + 1)) = cos ( tan -1
x ) Ans. (P).
(B) When a = 1, b = 1, then
Ê Ê 1 ˆˆ p
fi sin Á sin -1 Á ˜˜ sin -1 x + cos -1 y + cos -1 ( xy ) =
Ë Ë x2 + 2 x + 2 ¯ ¯ 2
p p
Ê Ê 1 ˆˆ fi - cos -1 x + cos -1 y + cos -1 ( xy ) =
= cos Á cos -1 Á ˜˜
2 2
Ë Ë x2 + 1 ¯ ¯ cos -1 x = cos -1 y + cos -1 ( xy )

1
=
1 fi cos -1 x - cos -1 y = cos -1 ( xy )


x2 + 2 x + 2 x2 + 1
fi ( )
cos -1 xy - 1 - x 2 1 - y 2 = cos -1 ( xy )

( xy - )
x2 + 2 x + 2 = x2 + 1
fi 1 - x 2 1 - y 2 = ( xy )
fi x + 2x + 2 = x + 1
2 2

fi 2x + 2 = 1 fi 1 - x2 1 - y 2 = 0

fi x=-
1 fi (1 - x ) (1 - y ) = 0
2 2

2
Ans. (Q).
(C) When a = 1, b = 2, then
Let (x, y) be such that p
fi sin -1 x + cos -1 y + cos -1 ( 2 xy ) =
p 2
sin -1 ( a x ) + cos -1 ( y ) + cos -1 (b x y ) =
2 p p
fi - cos -1 x + cos -1 y + cos -1 ( 2 xy ) =
Column - I Column - II 2 2
a = 1 and b = 0, then (x, y) fi cos -1 x = cos -1 y + cos -1 ( 2 xy )
(P) lies on the circle x 2 + y 2 = 1
Inverse Trigonometric Function 327

fi cos -1 x - cos -1 y = cos -1 ( 2 xy ) ÈÊ ˆ


2 1/ 2
˘
x2 1
= 1 + x 2 ÍÁ ˙
fi (
cos -1 xy - 1 - x 2 1 - y 2 = cos -1 ( 2 xy ) ) ÍË x 2 + 1
Î
+ ˜
x + 1¯
2
- 1
˙
˚

fi ( xy - 1 - x 1 - y ) = (2xy)
2 2 ÈÊ 2
2 Í x +1
= 1+ x Á
ˆ
˜
2 ˘
- 1˙
1/ 2

ÍË x 2 + 1 ¯ ˙
( 1 - x 1 - y ) = (- xy)
2

2 2 2 Î ˚

( )
1/ 2
È 2 ˘
fi (1 - x ) (1 - y ) = x y
2 2 2 2 = 1 + x2 Í
Î
x2 + 1 - 1˙
˚
fi 1 - x2 - y 2 + x2 y 2 = x2 y 2
( )
1/ 2
= 1 + x 2 È x 2 + 1 - 1˘
Î ˚
fi x + y =1
2 2

Ans. (P). = x 1+ x
2

(D) When a = 2, b = 2, then Ans. (c).


p 14. No questions asked between 2009 and 2010.
fi sin -1 ( 2 x ) + cos -1 y + cos -1 (2 xy ) =
2 Ê Ê sin q ˆ ˆ
p p 15. Let f (q ) = sin Á tan -1 Á
fi - cos -1 (2 x ) + cos -1 y + cos -1 (2 xy ) = Ë Ë cos 2q ˜¯ ˜¯
2 2
fi cos -1 ( 2 x ) = cos -1 y + cos -1 ( 2 xy ) where - p < q < p
4 4
fi cos -1 ( 2 x ) - cos -1 y = cos -1 ( 2 xy ) d
( f (q ))
fi (
cos -1 2 x. y - 1 - 4 x 2 1 - y 2 = cos -1 ( 2 xy ) ) d ( tanq )

fi (2x.y - 1 - 4x 1 - y ) = (2xy)
2 2 Ê
Ë
Ê sin q ˆ ˆ
Soln. Given f (q ) = sin Á tan -1 Á
Ë cos 2q ˜¯ ˜¯

( 1 - 4x 1 - y ) = 0
2 2
Ê -1 Ê
= sin Á sin Á
sin q ˆˆ
˜˜
fi (1 - 4 x ) (1 - y ) = 0
2 2 Ë Ë sin 2 q + cos 2q ¯ ¯

Ans. (S). Ê sin q ˆ


=Á ˜
x < 1, then Ë sin 2 q + cos 2 q - sin 2 q ¯

{ ( )}
1/ 2
È
) ( ˘
2
1 + x 2 ¥ Í x cos cot -1 x + sin cot -1 x - 1˙ Ê sin q ˆ
Î ˚ =Á ˜
Ë cos 2 q ¯
equals
x Ê sin q ˆ
=Á = tan q
(a) (b) x Ë cos q ˜¯
1 + x2
d
(c) x 1 + x 2 (d) 1 + x2 Now, ( tan q ) = 1
d ( tan q )

Soln. We have 16. No questions asked in 2012.


Ê 23 Ê n ˆˆ
cot Á Â cot -1 Á1 + Â 2k ˜ ˜ is
{ ( )}
1/ 2
È
) ( ˘
2
1 + x 2 ¥ Í x cos cot -1 x + sin cot -1 x - 1˙ Ë n =1 Ë k =1 ¯ ¯
Î ˚
(a) 23/25 (b) 25/23
ÈÏ Ê -1 Ê x ˆˆ (c) 23/24 (d) 24/23
2 Ô
= 1 + x ÍÌ x cos Á cos Á ˜˜
ÍÔ Ë Ë 1 + x2 ¯ ¯
ÎÓ n
1/ 2 Soln. Now, Â ( 2 k )
Ê -1 Ê 1 ˆ ˆ ¸Ô
2 ˘ k =1
+ sin Á sin Á - ˙
˜˜ ˝
Ë x + 1 ¯ ¯ Ô˛
1
˙ = 2 (1 + 2 + 3 + ..... + n )
Ë 2
˙˚
328 Comprehensive Trigonometry with Challenging Problems & Solutions for Jee Main and Advanced

Ê n (n + 1) ˆ ÊÊ 1 ˆ Ê y ˆˆ
= 2Á ÁÁ ˜ + yÁ ˜˜
Ë 2 ˜¯
Á ÁË y 2 + 1 ˜¯ ÁË 1 + y 2 ˜¯ ˜

(
= n2 + n ) Á Ê 1 - y2 ˆ Ê y ˆ ˜
˜
Á Á ˜ +Á ˜ ˜
ÁË ÁË y ˜¯ ÁË 1 - y 2 ˜¯ ˜¯
 cot -1 (1 + n + n 2 )
23

n =1 Ê Ê 1 + y2 ˆ ˆ
23
Ê 1 ˆ Á Á ˜ ˜
= Â tan -1 Á 2˜
Á ÁË y 2 + 1 ˜¯ ˜
n =1 Ë 1+ n + n ¯ =Á ˜
Á Ê 1 - y2 + y2 ˆ ˜
-1 Ê ˆ ÁÁ ˜˜
23 1
=  tan Á ˜ ÁË ÁË y 1 - y 2 ˜¯ ˜¯
n =1 Ë 1 + (n + 1) n ¯
23
-1 Ê ( n + 1) - n ˆ
= Â tan Á ˜ =
y ( 1- y ) 4

n =1 Ë 1 + ( n + 1) n ¯ Thus, the given expression reduces to

( )
23

( ) + y ˜¯
1/ 2
= Â tan -1 ( n + 1) - tan -1 ( n ) Ê 1 2

= Á 2 y 1- y
4
n =1
Ëy
( ) (
= tan -1 ( 2) - tan -1 (1) + tan -1 (3) - tan -1 ( 2) )
( )
1/ 2
Ê y2 1 - y4 ˆ
(
+........ + tan -1 ( 24) - tan -1 (23) ) = Á
ÁË y2
+y ˜
4
˜¯
= tan -1 ( 24) - tan -1 (1)
(( ) )
1/ 2
24 - 1ˆ = 1 - y4 + y4
-1 Ê Ê 23 ˆ
= tan Á = tan -1 Á ˜
Ë 1 + 24 ˜¯ Ë 25 ¯ =1
Thus, the given expression reduces to
Ê Ê 23 ˆ ˆ
= cot Á tan -1 Á ˜ ˜
Ë Ë 25 ¯ ¯
( ) (
cot sin -1 1 - x 2 = sin tan -1 x 6 , x π 0 ( ))
x
Ê Ê 25 ˆ ˆ
= cot Á cot -1 Á ˜ ˜
Ë Ë 23 ¯ ¯ (
Soln. Given cot sin -1 1 - x 2 = sin tan -1 x 6 ) ( ( ))
= 25 Ê -1 Ê x ˆˆ Ê -1 Ê x 6 ˆ ˆ
23 fi cot Á cot Á ˜ ˜ = sin Á sin Á ˜˜
Ë Ë 1 - x2 ¯ ¯ Ë Ë 6 x2 + 1 ¯ ¯
1/ 2 Ê x ˆ Ê x 6 ˆ
Ê Ê
( ) ( )ˆ˜ ˆ
2
cos tan -1 y + y sin tan -1 y fi Á ˜ =Á ˜
Á 1 Á +y 4˜
Ë 1 - x2 ¯ Ë 6 x2 + 1 ¯
(
Á y 2 Á cot sin -1 y + tan sin -1 y
ÁË Ë ) ( ) ˜¯ ˜
˜¯
Ê 1 ˆ Ê 6 ˆ
fi Á ˜ =Á ˜
Ë 1 - x2 ¯ Ë 6 x2 + 1 ¯
(
Ê cos tan -1 y + y sin tan -1 y ) ( )ˆ˜
Soln. We have Á
(
ÁË cot sin -1 y + tan sin -1 y ) ( ) ˜¯ (
fi 6 1 - x = 6x + 1
2 2
) ( )
fi 6 - 6 x2 = 6 x2 + 1
Ê Ê Ê 1 ˆˆ Ê ˆ
Ê y ˆˆ
Á cos Á cos -1 Á -1
˜ ˜ + y sin Á sin Á ˜˜ ˜ fi 12 x 2 = 5
Á ÁË ÁË y 2 + 1 ˜¯ ˜¯ ÁË ÁË 1 + y 2 ˜¯ ¯˜ ˜
5
=Á ˜ fi x2 =
Á Ê Ê 1 - y2 ˆ ˆ Ê Ê y ˆˆ ˜ 12
Á cot Á cot -1 Á -1
˜ ˜ + tan Á tan Á ˜˜ ˜
ÁË ÁË ÁË y ˜¯ ˜¯ ÁË ÁË 1 - y 2 ˜¯ ˜¯ ˜ 12
¯ fi x=±
5
Inverse Trigonometric Function 329

Ê ˆ
equation 6x + 2 Ê 2ˆ
fi Á ˜ =Á 2˜
-1 Ê
tan Á
1 ˆ Ê 1 ˆ
+ tan -1 Á
Ê 2ˆ
= tan -1 Á 2 ˜ is....
(
ÁË 8 x 2 + 6 x + 1 - 1 ) ˜¯ Ë x ¯
˜
Ë 2 x + 1¯ ˜
Ë 4 x + 1¯ Ëx ¯
Ê ˆ
3x + 1 Ê 1ˆ
fi Á ˜ =Á 2˜
Soln. We have (
ÁË 8 x + 6 x + 1 - 1
2
) ˜¯ Ë x ¯
Ê 1 ˆ Ê 1 ˆ Ê 2ˆ
tan -1 Á + tan -1 Á = tan -1 Á 2 ˜ fi 3 x3 + x 2 = 8 x 2 + 6 x
Ë 2 x + 1˜¯ Ë 4 x + 1˜¯ Ëx ¯
fi 3 x3 - 7 x 2 - 6 x = 0
Ê 1 1 ˆ
Á + ˜

Ê 1 1 ˆ
Ê 2ˆ
tan -1 Á 2 x + 1 4 x + 1 ˜ = tan -1 Á 2 ˜
Ëx ¯
fi (
x 3x 2 - 7 x - 6 = 0 )
Á1- ˜
ÁË ÁË 2 x + 1 ¥ 4 x + 1˜¯ ˜¯ x (3 x

2
- 9x + 2x - 6 = 0)
Ê 4x + 1 + 2x + 1 ˆ Ê 2ˆ fi x ( x - 3) (3x + 2) = 0
fi tan -1 Á ˜ = tan -1 Á 2 ˜
Ë (2 x + 1) (4 x + 1) - 1¯ Ëx ¯ 2
fi x = 0 , 3, -
3

21. No questions asked in 2015.


CHAPTER 7
Properties of Triangles

7.1 INTRODUCTION fi CD = b sin A ......................(i)


In any D, the three sides and the three angles are generally CD CD
From DBCD, sin B = =
called the elements of the triangle. BC a
A triangle which does not contain a right angle is called fi CD = a sin B .......................(ii)
an oblique triangle.
From (i) and (ii), we get,
In any DABC, the measures of the angles –BAC, –CBA
b sin A = a sin B
and –ACB are denoted by the letters.
A, B and C respectively, and the sides BC, CA and AB sin A sin B
fi = ...............(iii)
opposite to the angles A, B and C are respectively denoted a b
by a, b and c. These six elements of a triangle are not Similarly, we can prove that,
independent and are connected by the relations. sin A sin C
(i) A + B + C = p = ..............(iv)
a c
(ii) a + b > c; b + c > a; c + a > b
From (iii) and (iv), we get,
7.1.1 Sine Rule Statement sin A sin B sin C
= =
The sides of a triangle are proportion to the sines of the a b c
a b a b c
angles opposite to them, i.e. in a DABC, = = fi = =
sin A sin B sin A sin B sin C
c
. The above rule may also be expressed as
sin C 7.1.2 Some Solved Examples
sin A sin B sin C a 2 - b2 sin( A - B)
= = . Ex-1. If in a DABC, = ,
a b c a +b2 2 sin( A + B)
A then the triangle is right angled or isosceles.
a 2 - b2 sin( A - B)
Soln. We have =
c b a +b
2 2
sin( A + B)

a 2 - b2 sin ( A - B ) sin ( A + B )
fi =
B a D C a +b
2 2
sin 2 ( A + B )
Proof: Let ABC be a triangle such that a 2 - b2 sin 2 A - sin 2 B
fi =
AB = c, BC = a, CA = b a 2 + b2 sin 2 (p - C )
Draw CD is perpendicular on AB.
a 2 - b2 sin 2 A - sin 2 B
CD CD fi =
From DACD, sin A = = a 2 + b2 sin 2 C
AC b
Properties of Triangles 331


a 2 - b2
=
(
k 2 a 2 - b2 ) fi b2 - a 2 = c2 - b2
a 2 + b2 k 2c2 fi sin 2 B - sin 2 A = sin 2 C - sin 2 B


a 2 - b2
=
(a 2
- b2 ) fi sin ( B + A) sin ( B - A)
a +b2 2
c 2
= sin (C + B ) sin (C - B )
fi ( 2 Ê 1
a - b2 Á 2 ) -

Ë a + b 2 c 2 ˜¯
=0
fi sin C (sin B cos A - cos B sin A)

a 2 = b2 ,
1
=
1 = sin A (sin C cos B - cos C sin B )

a +b2 2
c 2
Dividing both the sides by sin A sin B sin C ,
fi a = b, a + b = c
2 2 2
we get,
Thus, the triangle is isosceles or right angled. fi cot A - cot B = cot B - cot C
Ex-2. In a DABC such that –A = 45∞ and –B = 75∞ fi cot A , cot B , cot C are in A.P.
a+c 2.
Soln. We have –C = 180∞ - ( A + B ) A b+c
Ex-4. If cot = , then prove that
2 a
= 180∞ - (45∞ + 75∞)
DABC is right angled.
= 180∞ - 120∞ Soln. We have
= 60∞ Ê Aˆ b + c
cot Á ˜ =
From the sine rule, we can write Ë 2¯ a
a b c
= = Ê A ˆ sin B + sin C
sin A sin B sin C fi cot Á ˜ =
Ë 2¯ sin A
a b c
fi = = Ê B + Cˆ Ê B - Cˆ
sin(45∞) sin(75∞) sin(60∞)
Ë 2 ˜¯ Ë 2 ˜¯
2 sin Á cos Á
fi Ê Aˆ
cot Á ˜ =
a b c Ë 2¯ sin A
fi = = = k ( say )
1 3 +1 3
Ê p Aˆ Ê B - Cˆ
2 sin Á - ˜ cos Á
Ë 2 ˜¯
2 2 2 2
Ê Aˆ Ë 2 2¯
fi cot Á ˜ =
Now, a + c 2 Ë 2¯ Ê Aˆ Ê Aˆ
2 sin Á ˜ cos Á ˜
Ë 2¯ Ë 2¯
k Ê k 3ˆ
= +Á ˜ 2
2 Ë 2 ¯ Ê Aˆ Ê B - Cˆ
2 cos Á ˜ cos Á
Ê Aˆ Ë 2 ¯ Ë 2 ˜¯
Ê k 3ˆ fi cot Á ˜ =
k Ë 2¯ Ê Aˆ Ê Aˆ
= +Á ˜ 2 sin Á ˜ cos Á ˜
2 Ë 2¯ Ë 2¯ Ë 2¯
Ê 3 + 1ˆ Ê B - Cˆ
˜k Ë 2 ˜¯
=Á cos Á
Ë 2 ¯ fi Ê Aˆ
cot Á ˜ =
Ë 2¯ Ê Aˆ
Ê 3 + 1ˆ sin Á ˜
Ë 2¯
= 2Á ˜k
Ë 2 2 ¯
Ê Aˆ Ê B - Cˆ
cos Á ˜ cos Á
= 2b Ë 2¯ Ë 2 ˜¯
fi =
Ex-3. In a triangle DABC, if a 2 , b 2 , c 2 are in A.P. Ê Aˆ Ê Aˆ
sin Á ˜ sin Á ˜
Ë 2¯ Ë 2¯
then prove that cot A, cot B, cot C are in A.P.
Ê Aˆ Ê B - Cˆ
Soln. Given a 2 , b 2 , c 2 are in A.P. fi cos Á ˜ = cos Á
Ë 2¯ Ë 2 ˜¯
332 Comprehensive Trigonometry with Challenging Problems & Solutions for Jee Main and Advanced

A B-C Ê B - Cˆ
fi =
Ë 2 ˜¯
cos Á
2 2
=
fi A+C = B Ê Aˆ
sin Á ˜
fi 2 B = A + B + C = 180∞ Ë 2¯

fi B = 90∞ . ÊA ˆ
sin Á + B˜
Thus, the triangle is right angled. Ë2 ¯
=
Ex-5. In any triangle ABC, prove that Ê Aˆ
sin Á ˜
Ê sin 2 A + sin A + 1ˆ Ë 2¯
’Á ˜ > 27 . Ex-7. In a triangle ABC, prove that,
Ë sin A ¯
a sin ( B - C ) b sin (C - A) c sin ( A - B )
= = .
Ê sin 2 A + sin A + 1ˆ b2 - c2 c2 - a2 a 2 - b2
Soln. We have ’ Á ˜
Ë sin A ¯ a b c
Soln. Let = = =k
sin A sin B sin C
Ê 1 ˆÊ 1 ˆ
= ÁË sin A + + 1˜ Á sin B + + 1˜
sin A ¯ Ë sin B ¯ a sin ( B - C )
We have,
Ê 1 ˆ b2 - c2
ÁË sin C + + 1˜
k sin A.sin (C - A)
sin C ¯
=
> ( 2 + 1) ( 2 + 1) ( 2 + 1) = 27 (
k 2 sin 2 C - sin 2 A )
(applying A.M ≥ G.M ) sin ( B + C ) .sin ( B - C )
=
Ex-6: In a triangle ABC, prove that, (
k sin 2 B - sin 2 C )
ÊA ˆ Ê Aˆ
a sin Á + B˜ = (b + c ) sin Á ˜ .
Ë2 ¯ Ë 2¯
=
(sin B - sin C ) = 1
2 2

a b c k (sin B - sin C )
2 k 2

Soln. Let = = =k
sin A sin B sin C b sin (C - A)
b+c c2 - a2
We have,
a k sin (C + A) .sin (C - A)
k (sin B + sin C ) =
=
k sin A
(
k 2 sin 2 C - sin 2 A )
sin 2C - sin 2 A 1
(sin B + sin C ) = =
=
sin A (
k sin C - sin A
2 2
) k

Ê B + Cˆ Ê B - Cˆ c sin ( A - B )
Ë 2 ˜¯ Ë 2 ˜¯
2 sin Á cos Á Also,
= a 2 - b2
Ê Aˆ Ê Aˆ k sin C sin ( A - B )
2 sin Á ˜ cos Á ˜
Ë 2¯ Ë 2¯ =
(
k 2 sin 2 A - sin 2 B )
Ê p Aˆ Ê B - Cˆ
2 sin Á - ˜ cos Á sin ( A + B ) sin ( A - B )
Ë 2 2¯ Ë 2 ˜¯ =
=
Ê Aˆ Ê Aˆ
2 sin Á ˜ cos Á ˜
(
k sin 2 A - sin 2 B )
Ë 2¯ Ë 2¯
sin 2 A - sin 2 B 1
= =
Ê Aˆ Ê B - Cˆ
2 cos Á ˜ cos Á
Ë 2¯ Ë 2 ˜¯
(
k sin A - sin B
2 2
) k
= Hence, the result.
Ê Aˆ Ê Aˆ
2 sin Á ˜ cos Á ˜
Ë 2¯ Ë 2¯
Properties of Triangles 333

Ex-8. In a DABC, if cos A + 2 cos B + cos C = 2 1 - cos A cos A 1 - cos 2 A sin 2 A


then prove that the sides of a triangle are in A.P. = = = =1
sin A sin A sin 2 A sin 2 A
Soln. Given cos A + 2 cos B + cos C = 2
fi C = 90∞
fi cos A + cos C = 2 (1 - cos B )
Hence, A = 45∞ = B , C = 90∞
fi Ê A+ Cˆ Ê A- Cˆ Ê Bˆ
2 cos Á = 4 sin 2 Á ˜ a b c
Ë 2 ˜¯
cos Á
Ë 2 ˜¯ Ë 2¯ Now, = =
sin A sin B sin C
Ê p Bˆ Ê A- Cˆ Ê Bˆ a b c
fi cos Á - ˜ cos Á ˜ = 2 sin 2 Á ˜ fi = =
Ë 2 2¯ Ë 2 ¯ Ë 2¯ sin 45∞ sin 45∞ sin 90∞
Ê Bˆ Ê A- Cˆ Ê Bˆ a b c
fi sin Á ˜ cos Á = 2 sin 2 Á ˜ fi = =
Ë 2¯ Ë 2 ˜¯ Ë 2¯ 1 1 1
Ê A- Cˆ Ê Bˆ 2 2
fi cos Á ˜ = 2 sin Á ˜
Ë 2 ¯ Ë 2¯ a b c
fi = =
1 1 2
Ê Bˆ
Multiplying both sides by 2 cos Á ˜ , we get, fi
Ë 2¯ a : b : c = 1 :1 : 2
Hence, the result.
Ê Bˆ Ê A- Cˆ Ê Ê Bˆ Ê Bˆˆ
fi 2 cos Á ˜ cos Á ˜ = 2 Á 2 cos Á ˜ sin Á ˜ ˜
Ë 2¯ Ë 2 ¯ Ë Ë 2¯ Ë 2 ¯¯
EXERCISE 1
Ê A+ Cˆ Ê A- Cˆ
fi 2 sin Á = 2 sin B Q. In a DABC,
Ë 2 ˜¯
cos Á
Ë 2 ˜¯
sin( B - C ) b2 - c2
fi sin A + sin C = 2 sin B 1. Prove that
sin( B + C )
= .
a2
fi a + c = 2b 2. Prove that a sin (B – C) + b sin (C – A)
fi a, b, c are in A.P. + c sin (A – B) = 0.
Ex-9. In a DABC, if cos A cos B + sin A sin B sin C = 1. a 2 sin( B - C ) b 2 sin(C - A)
3. Prove that +
sin B + sin C sin C + sin A
then prove that a : b : c = 1 : 1 : 2
c 2 sin( A - B)
Soln. We have cos A cos B + sin A sin B sin C = 1 + = 0.
sin A + sin B
1 - cos A cos B b2 - c2 c2 - a2
fi = sin C 4. Prove that +
sin A sin B cos B + cos C cos C + cos A
1 - cos A cos B a 2 - b2
fi = sin C £ 1 + = 0.
sin A sin B cos A + cos B
fi 1 - cos A cos B £ sin A sin B 1 + cos( A - B ) cos C a 2 + b2
5. Prove that = 2 .
1 + cos( A - C ) cos B a + c2
fi 1 £ sin A sin B + cos A cos B
6. Prove that a2, b2, c2 are in A.P., if
fi cos ( A - B ) ≥ 1
sin A sin( A - B)
= .
fi cos ( A - B ) = 1 sin C sin( B - C)

fi cos ( A - B ) = cos (0)


7.2 COSINE RULE
fi A- B = 0
Statement
fi A=B In any triangle ABC
1 - cos A cos B
Therefore, sin C = b2 + c2 - a 2
sin A sin B (i) cos A =
2bc
334 Comprehensive Trigonometry with Challenging Problems & Solutions for Jee Main and Advanced

a 2 + c2 - b2
(ii) cos B =
2ac
=
1 2
2
(
a + b2 - c2 - a 2 - c2 + b2 )
(iii) cos C =
a 2 + b2 - c2 1 2
(
= b - c2 - c2 + b2
2
)
2ab
Proof:
C
=
1
2
(
2b 2 - 2c 2 )
a b
(
= b2 - c2 )
Hence, the result.
Ex-2. In a triangle ABC, if

B c D A
(a + b + c ) (a - b + c ) = 3ac –B.
Soln. We have
CD CD
(i) From DBCD, sin A = = (a + b + c ) (a - b + c ) = 3ac
AC b
fi CD = b sin A fi (a + c )2 - b2 = 3ac
AD AD fi a 2 + c 2 - b 2 = 3ac - 2ac = ac
From DACD, cos A = =
AC b
a 2 + c2 - b2 ac 1
fi AD = b cos A fi = =
2ac 2ac 2
fi AB - BD = b cos A
fi 1
fi c - BD = b cos A cos B =
2
fi BD = c - b cos A p
fi B=
From DBCD, 3
BC 2 = BD 2 + CD 2 Hence, the angle B is 60∞
a
a 2 = (c - b cos A) + (b sin A)
2 2
fi Ex-3. In any DABC, if 2 cos B = prove that the triangle
is isoceles. c
fi a 2 = c 2 - 2 bc cos A + b 2 cos 2 A + b 2 sin 2 A
a
= c 2 - 2 bc cos A + b 2 Soln. We have 2 cos B =
c
fi 2 bc cos A = b 2 + c 2 - a 2 Ê a 2 + c2 - b2 ˆ a
fi 2Á ˜=c
b2 + c2 - a 2 Ë 2ac ¯
fi cos A =
2b c Ê a 2 + c2 - b2 ˆ
Similarly, we can easily proved that, (ii) and (iii). fi Á ˜ =a
Ë a ¯
7.2.1 Some Solved Examples fi (a 2
)
+ c2 - b2 = a 2
Ex-1. In a triangle ABC, prove that
a (b cos C - c cos B ) = b - c .2 2
fi (c2
)
- b2 = 0

fi c2 = b2
Soln. We have a (b cos C - c cos B )
fi c=b
= ( ab cos C - ac cos B ) Thus, the triangle is isosecles.
Ê a 2 + b2 - c2 ˆ Ê a 2 + c2 - b2 ˆ Ex-4. In a triangle ABC, if a cos A = b cos B , then
= ab Á ˜ - ac Á ˜ prove that D is right angled isoceles.
Ë 2ab ¯ Ë 2ac ¯
Soln. We have a cos A = b cos B
Ê a 2 + b2 - c2 ˆ Ê a 2 + c2 - b2 ˆ
=Á ˜ -Á ˜ Ê b2 + c2 - a 2 ˆ Ê a 2 + c2 - b2 ˆ
Ë 2 ¯ Ë 2 ¯ aÁ ˜ = b Á ˜
Ë 2 bc ¯ Ë 2 ac ¯
Properties of Triangles 335

Ê b2 + c2 - a 2 ˆ Ê a 2 + c2 - b2 ˆ
fi aÁ ˜ = b Á ˜
Ë b ¯ Ë a ¯
Ê Bˆ Ê A- Cˆ
fi ( ) ( )
a 2 b2 + c2 - a 2 = b2 a 2 + c2 - b2
=
cos Á ˜ cos Á
Ë 2¯ Ë 2 ˜¯
Ê Bˆ Ê Bˆ
fi a (c - a ) = b (c - b )
2 2 2 2 2 2 sin Á ˜ cos Á ˜
Ë 2¯ Ë 2¯

fi c (a - b ) = (a - b )
2 2 2 4 4
Ê A- Cˆ
Ë 2 ˜¯
cos Á
c (a - b ) = (a - b ) (a + b )
=
fi 2 2 2 2 2 2 2
Ê Bˆ
sin Á ˜
Ë 2¯
fi ( a - b ) (( a + b ) - c ) = 0
2 2 2 2 2

Ê A- Cˆ
(a - b ) = 0 , (a + b ) = c Ë 2 ˜¯
cos Á
fi 2 2 2 2 2
=
Ê 60∞ ˆ
a = b , (a + b ) = c Ë 2 ˜¯
sin Á
fi 2 2 2

Thus, the triangle is right angled isosceles. = 2 cos ÊÁ A - C ˆ˜ .


Ex-5. In a triangle ABC, the angles are in A.P, then prove Ë 2 ¯

Ê A- Cˆ a+c Ex-6. In a triangle ABC, prove that


that, 2 cos Á =
Ë 2 ˜¯ a - ac + c
2 2
Ê b2 - c2 ˆ Ê c2 - a2 ˆ Ê a 2 - b2 ˆ
Á ˜ sin 2 A + Á ˜ sin 2 B + Á ˜
Soln. Since the angles are in A.P, so A + C = 2B Ë a2 ¯ Ë b2 ¯ Ë c2 ¯
fi A + B + C = 3B
sin 2C = 0
fi 3B = 180∞
fi B = 60∞ Ê b2 - c2 ˆ
Soln. We have, Á ˜ sin 2 A
Ë a2 ¯
1
fi cos B = cos(60∞) =
2 Ê b2 - c2 ˆ
=Á ˜ ( 2 sin A.cos A)
a 2 + c2 - b2 1 Ë a2 ¯
fi =
2ac 2 Ê b2 - c2 ˆ Ê Ê b2 + c2 - a 2 ˆ ˆ
=Á ˜Á 2 k a Á ˜˜
fi a 2 + c 2 - b 2 = ac Ë a2 ¯ Ë Ë 2bc ¯¯

fi a 2 + c 2 - ac = b 2
( )(
Ê b2 - c2 b2 + c2 - a 2

)ˆ˜
Now, R.H.S ÁË k abc ˜¯
a+c
=
a 2 - ac + c 2 =
1
k abc
{( ) (
¥ b4 - c4 - a 2 b2 - c2 )}
a+c
=
b Ê c2 - a2 ˆ
Similarly, Á ˜ ¥ sin 2 B
k (sin A + sin C ) Ë b2 ¯
=
k sin B
Ê A+ Cˆ Ê A- Cˆ
=
1
k abc
{( ) (
¥ c4 - a 4 - b2 c2 - a 2 )}
Ë 2 ˜¯
2 sin Á ˜ cos Á
Ë 2 ¯ Ê a 2 - b2 ˆ
=
Ê Bˆ Ê Bˆ Á ˜ sin 2C and
2 sin Á ˜ cos Á ˜ Ë c2 ¯
Ë 2¯ Ë 2¯
336 Comprehensive Trigonometry with Challenging Problems & Solutions for Jee Main and Advanced

Ê bc + 2bc ˆ
=
1
k abc
{(
¥ a 4 - b4 - c2 a 2 - b2) ( )} =Á
Ë bc ˜¯
, from (i)

Ê b2 - c2 ˆ Ê c2 - a2 ˆ Ê 3bc ˆ

Thus, Á ˜ sin 2 A + Á ˜ sin 2 B Ë bc ˜¯
Ë a2 ¯ Ë b2 ¯
=3
Ê a 2 - b2 ˆ
+Á ˜ sin 2C
Ë c2 ¯ Ex-8 If
1
+
1
=
3
–C.
a+c b+c a+b+c
=
1
k abc
{(
¥ b4 - c4 - a 2 b2 - c2 ) ( )} Soln. Given
1
+
1
=
3
a+c b+c a+b+c
+
1
k abc
{(
¥ c4 - a 4 - b2 c2 - a 2) ( )} fi
a+b+c a+b+c
+ =3
a+c b+c

+
1
k abc
{(
¥ a 4 - b4 - c2 a 2 - b2) ( )} fi 1+
b
a+c
+1+
a
b+c
=3

b a
=
1
(
¥ È b4 - c4 + c4 - a 4 + a 4 - b4
k abc Î
) fi +
a+c b+c
=1

fi b ( b + c ) + a ( a + c ) = ( a + c ) (b + c )
( ) ( )
+a 2 c2 - b2 + b2 a 2 - c2 + c2 b2 - a 2 ˘
˚ ( )
fi b 2 + bc + a 2 + ac = ab + ac + bc + c 2
=0
fi a 2 + b 2 - c 2 = ab
Ex-7. In a triangle ABC, if – A = 60∞
Ê a bˆ Ê c aˆ Ê a 2 + b2 - c2 ˆ
the value of ÁË1 + + ˜¯ ÁË1 + - ˜¯ . Now, cos (C ) = Á ˜
c c b b Ë 2ab ¯

Soln. Given –A = 60∞ ab 1


= =
2ab 2
fi cos A = cos(60∞)
p
1 fiC =
fi cos A = 3
2
Ex-9. If in a triangle ABC,
b2 + c2 - a 2 1 2 cos A 2 cos B 2 cos C 1 b
fi = + + = + ,
2bc 2 a b c bc ca
fi (b 2
)
+ c 2 - a 2 = bc .....(i) A in degrees.
Soln. We have
Ê a bˆ Ê c aˆ 2 cos A 2 cos B 2 cos C 1 b
Now, Á1 + + ˜ Á1 + - ˜ + + = +
Ë c c¯ Ë b b¯ a b c bc ca
Ê c + a + bˆ Ê b + c - aˆ 2bc cos A ac cos B 2bc cos C
=Á ˜¯ ÁË ˜¯ fi + +
Ë c b abc abc abc
Ê b + c + aˆ Ê b + c - aˆ a2 b2
=Á ˜¯ ÁË ˜¯ = +
Ë c b abc abc
Ê ( b + c )2 - a 2 ˆ fi 2bc cos A + ac cos B + 2bc cos C = a2 + b2
=Á ˜
Ë bc ¯
fi (b 2
) 12 (a
+ c2 - a2 + 2
+ c2 - b2 )
Ê b 2 + c 2 - a 2 + 2bc ˆ

Ë bc ˜
¯ ( )
+ a 2 + b2 - c2 = a 2 + b2
Properties of Triangles 337

fi 2b 2 - 2a 2 + c 2 + a 2 - b 2 = 0 7.3 PROJECTION FORMULAE


fi a 2 = b2 + c2 Statement:
In any triangle ABC
DABC is a right angled triangle at A
(i) a = b cos C + c cos B
Thus, –A = 90∞ (ii) b = c cos A + a cos C
(iii) c = a cos B + b cos A
Proof:
EXERCISE 2
C
Q. In any DABC,
a
1. Prove that b (c cos A - a cos C ) = c 2 - a 2 ( ) b

2. Prove that
B D A
cos A cos B cos C a 2 + b 2 + c 2 c
+ + =
a b c 2abc
From DABD, we have,
3 Prove that
a = BC = BD + DC
2 Ê Cˆ 2 Ê Cˆ
(a - b) 2
cos Á ˜ + (a + b ) sin Á ˜ = c
Ë 2¯
2
Ë 2¯
2
= c cos B + b cos C

4. Prove that fi a = b cos C + c cos B

(b 2
) ( ) ( )
- c 2 cot A + c 2 - a 2 cot B + a 2 - b 2 cot C = 0
Similarly, we can eaisly proved that (ii) and (iii).

7.3.1 Some Solved Examples


5. In a DABC, a = 4, b = 3 , – A = 60∞
Ex-1. In a triangle ABC, prove that,
Then prove that c is a root of the equation sin B c - a cos B
=
c 2 - 3c - 7 = 0 sin C b - a cos C
c - a cos B
6. In a DABC, if ( a + b + c ) (b + c - a ) = l bc Soln. We have,
b - a cos C
l.
a cos B + b cos A - a cos B
=
7. If the angles A, B, C of a triangle are in A.P. c cos A + a cos C - a cos C
and its sides a, b, c are in G.P., prove that
b cos A
a 2 , b 2 , c 2 are in A.P. =
c cos A

8. If the line segment joining the points P ( a1 , b1 ) b


=
c
and Q ( a2 , b2 ) subtends an angle q at the origin,
k sin B
Ê ˆ =
a1a2 + b1b2 k sin C
prove that cosq = Á ˜
ÁË a 2 + b 2 a 2 + b 2 ˜¯ sin B
1 1 2 2
=
sin C
9. In a triangle ABC, if cot A, cot B, cot C are
Ex-2. In any triangle ABC, prove that,
in A.P., prove that a 2 , b 2 , c 2 are in A.P.
Ê 2 C Aˆ
2 Á a sin + c sin 2 ˜ = a + c – b
Ë 2 2¯
10. If the sides of a triangle are a, b and a 2 + ab + b 2
Ê 2 C Aˆ
Soln. We have, 2 Á a sin + c sin 2 ˜
Ë 2 2¯
338 Comprehensive Trigonometry with Challenging Problems & Solutions for Jee Main and Advanced

Ê Q. In any DABC, prove that


2 Ê Cˆˆ Ê 2 Ê Aˆ ˆ
= a Á 2 sin Á ˜ ˜ + c Á 2 sin Á ˜ ˜ cos A cos B
Ë Ë 2 ¯¯ Ë Ë 2 ¯¯ 5. +
b cos C + c cos B c cos A + a cos C
= a (1 - cos C ) + c (1 - cos A)
cos C a 2 + b2 + c2
= a + c - (a cos C + c cos A) + =
a cos B + b cos A 2abc
= a+c-b
2
(
6. 2 (bc cos A + ca cos B + ab cos C ) = a + b + c
2 2
)
Ex-3. In any triangle ABC, prove that,
cos A cos B
+
b cos C + c cos B c cos A + a cos C 7.4 NAPIER’S ANALOGY
+
cos C
=
a +b +c2 2 2
(LAW OF TANGENTS)
a cos B + b cos A 2abc
Statement:
Soln. We have, In any DABC, prove that
cos A cos B
+ Ê B - Cˆ b-c Ê Aˆ
(i) tan Á = cot Á ˜
b cos C + c cos B c cos A + a cos C Ë 2 ˜¯ b+c Ë 2¯
cos C
+ Ê C - Aˆ Ê c - a ˆ B
a cos B + b cos A (ii) tan Á ˜ =Á ˜ cot
Ë 2 ¯ Ë c + a¯ 2
cos A cos B cos C
= + + Ê A - Bˆ Ê a - bˆ C
a b c (iii) tan Á ˜ =Á ˜ cot
Ë 2 ¯ Ë a + b¯ 2
Ê b2 + c2 - a 2 ˆ Ê c2 + a 2 - b2 ˆ
=Á +
Ë 2 abc ˜¯ ÁË 2 abc ˜¯ a
=
b
=
c
=k
Proof: (i) Let
sin A sin B sin C
Ê a 2 + b2 - c2 ˆ
+Á Ê b - cˆ Ê Aˆ
Ë 2 abc ˜¯ We have, Á
Ë b + c ˜¯
cot Á ˜
Ë 2¯

=
1
2 abc
{( ) ( ) (
b2 + c2 - a 2 + c2 + a 2 - b2 + a 2 + b2 - c2 )} Ê sin B - sin C ˆ

Ê Aˆ
¥ cot Á ˜
Ë sin B + sin C ˜¯ Ë 2¯

=
(a 2
+ b2 + c2 ) Ê Ê B + Cˆ Ê B - Cˆ ˆ
2 abc Á 2 cos ÁË 2 ˜¯ sin ÁË 2 ˜¯ ˜ Ê Aˆ
=Á ˜ ¥ cot Á ˜
Á 2 sin Ê B + C ˆ cos Ê B - C ˆ ˜ Ë 2¯
EXERCISE 3 ÁË ÁË ˜ ÁË ˜
2 ¯ 2 ¯ ˜¯
Q. In any DABC, prove that Ê Ê B + Cˆ Ê B - Cˆ ˆ
Á 2 cos ÁË 2 ˜¯ sin ÁË 2 ˜¯ ˜ Ê Aˆ
Ê 2 C Aˆ =Á ˜ ¥ cot Á ˜
1. 2 Á a sin + c sin 2 ˜ = a + c – b Ë 2¯
Ë 2 2¯ Á 2 sin Ê B + C ˆ cos Ê B - C ˆ ˜
ÁË ÁË ˜ ÁË ˜
2 ¯ 2 ¯ ˜¯
Ê 2 C Bˆ
2. 2 Á b cos + c cos 2 ˜ = a + b + c Ê Ê Aˆ Ê B - Cˆ ˆ
Ë 2 2¯
Á sin ÁË 2 ˜¯ sin ÁË 2 ˜¯ ˜ Ê Aˆ
Q. In any DABC, =Á ˜ ¥ cot Á ˜
Á cos Ê A ˆ cos Ê B - C ˆ ˜ Ë 2¯
sin B c - a cos B ÁË ÁË ˜¯ ÁË ˜¯ ¯˜
3. Prove that = 2 2
sin C b - a cos C
Ê Ê Aˆ Ê B - Cˆ ˆ Ê Aˆ
4. Prove that (b + c ) cos A + (c + a ) cos B + ( a + b ) cos C Á sin ÁË 2 ˜¯ sin ÁË 2 ˜¯ ˜ cos ÁË 2 ˜¯
=Á ˜¥
Á cos Ê A ˆ cos Ê B - C ˆ ˜ sin Ê A ˆ
= (a + b + c ) ÁË ÁË ˜¯
2
ÁË ˜
2 ¯ ¯˜
ÁË ˜¯
2
Properties of Triangles 339

Ê B - Cˆ Ê A - Bˆ 4
= tan Á fi 2 cos 2 Á -1 =
Ë 2 ˜¯ Ë 2 ˜¯ 5

Similarly, we can easily proved that (ii) and (iii) Ê A - Bˆ 4 9


fi 2 cos 2 Á =1+ =
Ë 2 ˜¯ 5 5
7.4.1 Some Solved Examples
Ê A - Bˆ 9
fi cos 2 Á =
Ex-1. In any DABC, b = 3 + 1, c = 3 - 1 Ë 2 ˜¯ 10
and –A = 60∞
Ê A - Bˆ 3
Ê B - Cˆ fi cos Á ˜ =
tan Á . Ë 2 ¯
Ë 2 ˜¯
10

Soln. As we know that, Ê A - Bˆ 1


fi tan Á =
Ë 2 ˜¯ 3
Ê B - C ˆ Ê b - cˆ Ê Aˆ
tan Á ˜ =Á ˜ cot Á ˜
Ë 2 ¯ Ë b + c¯ Ë 2¯ Ê a - bˆ Ê Cˆ 1
fi ÁË ˜¯ cot ÁË ˜¯ =
a+b
Ê

( ) (
3 +1 - )
3 -1 ˆ Ê 60∞ ˆ
˜ cot Á
2 3

Ë 2 ˜¯ Ê 6 - 3ˆ Ê Cˆ 1
ÁË ( 3 + 1) + ( )
3 - 1 ˜¯ fi ÁË ˜ cot ÁË ˜¯ =
6 + 3¯ 2 3
1
= cot (30∞) 1 Ê Cˆ 1
3 fi cot Á ˜ =
3 Ë 2¯ 3
1
= ¥ 3 Ê Cˆ
3 fi cot Á ˜ = 1
Ë 2¯
=1
C p
fi =
Ex-2. In any DABC, b = 3, c = 1, B - C = 90∞ 2 4
–A. p
Soln. As we know that, fi C=
2
Ê B - C ˆ Ê b - cˆ Ê Aˆ
tan Á =
Ë 2 ˜¯ ÁË b + c ˜¯
cot Á ˜ p
Ë 2¯ C is .
2
Ê 90∞ ˆ Ê 3 - 1ˆ Ê Aˆ
fi tan Á = cot Á ˜
Ë 2 ˜¯ ÁË 3 + 1˜¯ Ë 2¯ EXERCISE 4
Ê Aˆ
fi ( )
tan(45∞) = 2 - 3 cot Á ˜
Ë 2¯
ABC, a = 4, b = 2 and cos (A – B) = 4/5,
then prove that angle C is 90∞.
Ê Aˆ 2. In a triangle ABC
fi cot Á ˜ =
1
= 2+ 3 ( )
Ë 2¯ (
2- 3 ) Ê B - Cˆ
x = tan Á
Ë 2 ˜¯
Ê Aˆ Ê C - Aˆ
tan Á ˜ , y = tan Á
Ë 2¯ Ë 2 ˜¯
Ê Bˆ
tan Á ˜
Ë 2¯
Ê Aˆ and
fi cot Á ˜ = cot(15∞)
Ë 2¯ Ê A - Bˆ Ê Cˆ
z = tan Á tan Á ˜ , then prove that
A = 30∞ Ë 2 ˜¯ Ë 2¯

Hence, the angle A is 30∞. x + y + z + xyz = 0
Ex-3. ABC, a = 6, b = 3 and
cos ( A - B ) = 4/ 5 C. 3. In a triangle ABC a = 5, b = 4 and
31
4 cos ( A - B ) = , then prove that c = 6.
Soln. Given cos ( A - B ) = 32
5
340 Comprehensive Trigonometry with Challenging Problems & Solutions for Jee Main and Advanced

7.5 HALF-ANGLED FORMULAE Ê ( 2s - 2c ) ¥ ( 2s - 2b ) ˆ


=Á ˜¯
Ë 2bc

Ê 2 (s - c) ¥ (s - b)ˆ
DABC will be denoted =Á ˜¯
Ë bc
by 2s
i.e. a + b+ c = 2s and its area is denoted by D. Ê Aˆ Ê 2 (s - c) ¥ (s - b)ˆ
fi 2 sin 2 Á ˜ = Á ˜¯
Ë 2¯ Ë bc
i.e. D= s ( s - a)( s - b)( s - c)
Ê Aˆ Ê (s - c) ¥ (s - b)ˆ
Ê Aˆ Ê Bˆ Ê Cˆ fi sin 2 Á ˜ = Á ˜¯
sin Á ˜ , sin Á ˜ , sin Á ˜ : Ë 2¯ Ë bc
Ë 2¯ Ë 2¯ Ë 2¯
In any DABC, Ê Aˆ (s - c) ¥ (s - b)
fi sin Á ˜ =
A ( s - b)( s - c) Ë 2¯ bc
(i) sin =
2 bc
Similarly, we can easily prove that (ii) and (iii)
B ( s - c)( s - a ) Ê Aˆ Ê Bˆ Ê Cˆ
(ii) sin = cos Á ˜ , cos Á ˜ , cos Á ˜ :
2 ca Ë 2¯ Ë 2¯ Ë 2¯
C ( s - a)( s - b) In any DABC,
(iii) sin =
2 ab A s( s - a)
(i) cos =
2 bc
Proof: (i) We have,
B s ( s - b)
2Ê Aˆ (ii) cos =
2 sin Á ˜ 2 ca
Ë 2¯
C s ( s - c)
= 1 - cos A (iii) cos =
2 ab
2 Ê Aˆ
= 2 sin Á ˜ Proof: (i) We have,
Ë 2¯
Ê Aˆ
Ê b2 + c2 - a 2 ˆ 2 cos 2 Á ˜
=1- Á Ë 2¯
2bc ˜
Ë ¯
= 1 + cos A
Ê 2bc - b 2 - c 2 + a 2 ˆ
=Á ˜ Ê b2 + c2 - a 2 ˆ
Ë 2bc ¯ =1+ Á ˜
Ë 2bc ¯
(
Ê a 2 - b 2 + c 2 - 2bc

)ˆ˜ Ê 2bc + b 2 + c 2 - a 2 ˆ
ÁË 2bc ˜¯ =Á ˜
Ë 2bc ¯
Ê a 2 - ( b - c )2 ˆ Ê ( b + c )2 - a 2 ˆ
=Á ˜ =Á ˜
Ë 2bc ¯ Ë 2b c ¯
Ê {a + (b - c )} ¥ {a - (b - c )} ˆ Ê ( a + b + c ) (b + c - a ) ˆ
=Á ˜ =Á ˜¯
Ë 2bc ¯ Ë 2b c

Ê (a + b - c ) ¥ (a + c - b) ˆ Ê ( a + b + c ) (b + c + a - 2 a ) ˆ
=Á =Á ˜¯
Ë 2bc ˜¯ Ë 2b c

Ê ( a + b + c - 2c ) ¥ (a + b + c - 2b ) ˆ Ê 2 s ( 2 s - 2a ) ˆ
=Á =Á ˜¯
Ë 2bc ˜¯ Ë 2b c
Properties of Triangles 341

Ê 2s ( s - a ) ˆ B
=Á (ii) cos
Ë b c ˜¯ 2
(iii) cos A
2ÊA ˆ Ê 2s ( s - a ) ˆ
fi 2 cos Á ˜ = Á Soln. We have 2s = a + b + c = 13 + 14 + 15
Ë 2¯ Ë b c ˜¯
13 + 14 + 15 42
s= = = 21
Ê Aˆ Ê s (s - a)ˆ 2 2
fi cos 2 Á ˜ = Á
Ë 2 ¯ Ë b c ˜¯ A ( s - b)( s - c)
(i) sin =
2 bc
Ê Aˆ Ê s (s - a)ˆ
fi cos Á ˜ = Á
Ë 2¯ Ë b c ˜¯ (21 - 14) (21 - 15)
=
14.15
Similarly, we can easily proved that (ii) and (iii).
7.6 1
Ê Aˆ Ê Bˆ Ê Cˆ = =
tan Á ˜ , tan Á ˜ , tan Á ˜ : 14.15
Ë 2¯ Ë 2¯ Ë 2¯ 5

In any DABC, A s( s - a)
(ii) cos =
A ( s - b)( s - c) 2 bc
(i) tan =
2 s( s - a) 21( 21 - 13)
=
14.15
B ( s - c)( s - a )
(ii) tan = 21.8
2 s ( s - b) =
14.15

C ( s - a)( s - b) 4 2
(iii) tan = = =
2 s ( s - c) 5 5
(iii) cos A
Proof: (i) We have, 2 Ê Aˆ
= 2 cos Á ˜ - 1
Ê Aˆ Ë 2¯
tan Á ˜
Ë 2¯ Ê 4ˆ
= 2Á ˜ -1
Ê Aˆ Ë 5¯
sin Á ˜
Ë 2¯ 8-5 3
= . = =
Ê Aˆ 5 5
cos Á ˜
Ë 2¯
Ê Aˆ b+c
(s - b) (s - c) Ex-2. In a DABC cos Á ˜ = ,
Ë 2¯ 2c
bc
= prove that DABC is right angled at C.
s (s - a)
bc Ê Aˆ b+c
Soln. We have cos Á ˜ =
Ë 2¯ 2c
Ê Aˆ (s - b) (s - c)
fi tan Á ˜ = . s (s - a)
Ë 2¯ s (s - a) =
b+c

bc 2c
Similarly, we can easily proved that (ii) and (iii)
s (s - a) b + c
fi =
7.5.1 Some Solved Examples bc 2c
Ex-1. In a DABC a = 13, b = 14 and c fi 2s ( 2s - 2a ) = 2b (b + c )

A fi (a + b + c ) (b + c - a ) = 2b (b + c )
(i) sin
2
342 Comprehensive Trigonometry with Challenging Problems & Solutions for Jee Main and Advanced

fi ((b + c ) 2
)
- a 2 = 2b (b + c ) Ê p - B - Bˆ
= 2ac sin Á
Ë 2
˜¯
fi b 2 + c 2 - a 2 = 2b 2
Êp ˆ
a 2 + b2 = c2 = 2ac sin Á - B˜
fi Ë2 ¯
ABC is right angled at C.
= 2ac cos B
Ex-3. In a DABC, prove that,
Ê a 2 + c2 - b2 ˆ
2 Ê Cˆ Bˆ 2Ê = 2 ac
b cos Á ˜ + c cos Á ˜ = s Á 2ac ˜
Ë 2¯ Ë 2¯ Ë ¯

2 Ê Cˆ 2 Ê Bˆ
Soln. We have, b cos Á ˜ + c cos Á ˜ (
= a +c -b
2 2 2
)
Ë 2¯ Ë 2¯
Ex-6 . In a DABC, 3a = b + c
Ê s (s - c)ˆ Ê s (s - b)ˆ
= bÁ ˜ + cÁ Ê Bˆ Ê Cˆ
Ë ab ¯ Ë a c ˜¯ cot Á ˜ cot Á ˜ .
Ë 2¯ Ë 2¯
s
= (s - c + s - b) Ê Bˆ Ê Cˆ
a Soln. We have, cot Á ˜ cot Á ˜
Ë 2¯ Ë 2¯
s
= (2 s - c - b)
a s (s - b) s (s - c)
= ¥
s
= (a + b + c - c - b )
(s - a) (s - c) (s - a ) (s - b)
a
s (s - b) s (s - c)
= ¥
=s ( ) ( ) ( a ) (s - b)
s - a s - c s -
Ex-4. In a DABC, prove that
s2
Ê Aˆ Ê Bˆ Ê Cˆ =
bc cos 2 Á ˜ + ca cos 2 Á ˜ + ab cos 2 Á ˜ = s 2
Ë 2¯ Ë 2¯ Ë 2¯
( s - a )2
s
Soln. We have =
s-a
Ê Aˆ Ê Bˆ Ê Cˆ
bc cos 2 Á ˜ + ca cos 2 Á ˜ + ab cos 2 Á ˜ 2s
Ë 2¯ Ë 2¯ Ë 2¯ =
2s - 2a
Ê s (s - a)ˆ Ê s (s - b)ˆ Ê s (s - c)ˆ
= bc Á ˜ + ca Á ˜ + ab Á a+b+c
Ë bc ¯ Ë ca ¯ Ë ab ˜¯ =
a + b + c - 2a
= s (s - a ) + s (s - b) + s (s - c)
4a
=
= s (3s - ( a + b + c )) 4a - 2 a
= s (3s - 2s ) = 2.
= s¥s Ex-7. In a DABC, prove that
= s2 Ê Aˆ Ê Bˆ 2c
1 - tan Á ˜ tan Á ˜ =
Ê A- B + Cˆ Ë 2¯ Ë 2 ¯ (a + b + c )
Ex.-5 In a DABC, prove that 2ac sin Á ˜¯
Ë 2 Ê Aˆ Ê Bˆ
Soln. We have 1 - tan Á ˜ tan Á ˜
Ë 2¯ Ë 2¯
2
(
= a +c -b .
2 2
)
(s - b) (s - c) (s - a ) (s - c)
Ê A- B + Cˆ = 1-
Soln. We have 2ac sin Á ˜¯ s (s - a) s (s - b)
Ë 2
Ê A + C - Bˆ
= 2ac sin Á ˜¯
( s - c )2
Ë 2 = 1-
s2
Properties of Triangles 343

(s - c) 2s s (s - a)
= 1- = ¥
s ( 2 s - 2a ) (s - b) (s - c)
c
= (a + b + c ) Ê Aˆ
s = ¥ cot Á ˜
(b + c - a ) Ë 2¯
2c
= Ê Aˆ Ê Bˆ Ê Cˆ
2s Ex-9 In a DABC cot Á ˜ , cot Á ˜ , cot Á ˜ are in A.P.,
Ë 2¯ Ë 2¯ Ë 2¯
2c
= then prove that a, b, c are in A.P.
(a + b + c ) A B C
Soln. Given cot , cot , cot are in A.P.
2 2 2
Ex-8. In a DABC, prove that: Ê Bˆ Ê Aˆ Ê Cˆ
Ê Aˆ Ê Bˆ Ê C ˆ Ê a + b + cˆ Ê Aˆ fi 2 cot Á ˜ = cot Á ˜ + cot Á ˜
cot Á ˜ + cot Á ˜ + cot Á ˜ = Á Ë 2¯ Ë 2¯ Ë 2¯
Ë 2 ¯ Ë b + c - a ˜¯
cot Á ˜
Ë 2¯ Ë 2¯ Ë 2¯
s (s - b)
A B C fi 2
Soln. We have, cot
2
+ cot
2
+ cot
2
(s - a) (s - c)
s (s - a) s (s - b) s (s - a) s (s - c)
= + = +
(s - b) (s - c) (s - a) (s - c) (s - b) (s - c) (s - a ) (s - b)
s (s - c) (s - b)
+ fi 2
(s - a ) (s - b) (s - a) (s - c)
s2 (s - a)
2
s 2 (s - b)
2 (s - a) (s - c)
= + = +
s (s - a ) (s - b) (s - c) s (s - b) (s - a ) (s - c) (s - b) (s - c) (s - a ) (s - b)

s2 (s - c)
2
( s - b )2
+ fi 2
s (s - c) (s - a ) (s - b) (s - b) (s - a ) (s - c)
s2 (s - a)
2
s 2 ( s - b)
2
s2 (s - c)
2
( s - a )2 ( s - c )2
= + + = +
D2 D2 D2 (s - a ) (s - b) (s - c) (s - a ) (s - b) (s - c)
s (s - a ) s (s - b) s (s - c) fi 2 (s - b) = (s - a ) + (s - c)
= + +
D D D
fi 2 ( s - b) = (2 s - a - c )
s
= (s - a + s - b + s - c) fi 2b = a + c
D
s fi a, b, c ΠA.P
=
D
(3s - (a + b + c))
B C
s Ex-10 In a DABC, c(a + b) cos = b(a + c) cos , then
= (3s - 2s ) 2 2
D
prove that the triangle is isosceles.
s2
= B C
D Soln. We have, c(a + b) cos = b(a + c) cos
2 2
s2
= s (s - b) s (s - c)
s ( s - a ) (s - b) ( s - c) fi c (a + b) ¥ = b (a + c ) ¥
ac ab
s s (s - a) (s - b) (s - c)
= ¥ fi c (a + b) ¥ = b (a + c ) ¥
( a)
s - ( b) (s - c)
s - c b
344 Comprehensive Trigonometry with Challenging Problems & Solutions for Jee Main and Advanced

(s - b) (s - c) Ê Aˆ Ê Bˆ Ê s - cˆ
c 2 (a + b) ¥ = b2 (a + c ) ¥
2 2
fi 5. In a DABC, tan Á ˜ tan Á ˜ = Á
c b Ë 2¯ Ë 2 ¯ Ë s ˜¯

fi c (a + b) ¥ ( s - b) = b (a + c ) ¥ ( s - c )
2 2 6. In a DABC, prove that
Ê b - cˆ 2 Ê Aˆ Ê c - a ˆ 2 Ê Bˆ
fi c (a + b) ¥ ( s - b) = b (a + c ) ¥ ( s - c )
2 2 ÁË ˜¯ cos ÁË ˜¯ + ÁË ˜¯ cos ÁË ˜¯
a 2 b 2
c (a + b) ¥ (a + c - b) = b (a + c ) ¥ (a + b - c )
2 2
fi Ê a - bˆ Ê Cˆ
+Á cos 2 Á ˜ = 0
Ë c ˜¯ Ë 2¯
(a + c - b) (a + b - c )
fi =
b (a + c ) c (a + b)
2 2
7. In a DABC, prove that
1 1 1 1 Ê Aˆ Ê Bˆ Ê Cˆ
fi - = - cot Á ˜ + cot Á ˜ + cot Á ˜
b ( a + c ) ( a + c )2 c ( a + b ) ( a + b )2 Ë 2¯ Ë 2¯ Ë 2 ¯ (a + b + c )
2
= 2
1 1 1 1 cot A + cot B + cot C a + b2 + c2
fi - = -
b (a + c ) c (a + b) (a + c ) 2
( a + b )2 8. In a DABC, prove that
A B C
a c + c 2 - ab - b 2 ( a + b ) - ( a + c )
2 2 (b - c ) cot ÊÁË ˆ˜¯ + (c - a ) cot ÊÁË ˆ˜¯ + (a - b) cot ÊÁË ˆ˜¯ = 0
fi = 2 2 2
bc ( a + b ) ( a + c ) ( a + c ) 2 ( a + b )2 9. In a DABC


a (c - b ) + c - b( 2 2
) = 2 a (b - c ) + ( b 2
-c 2
) Ê Aˆ 5 Ê B ˆ 20
tan Á ˜ = , tan Á ˜ =
Ë 2¯ 6 Ë 2 ¯ 37
, then prove
bc (a + c ) (a + b)
that a, b, c are in A.P.
( c - b ) ( a + b + c ) (b - c ) ( 2 a + b + c )
fi = Ê Bˆ Ê Cˆ
bc (a + c ) (a + b) 10. In a DABC c ( a + b ) cos Á ˜ = b (a + c ) cos Á ˜
Ë 2¯ Ë 2¯
Ê ( a + b + c ) ( 2a + b + c ) ˆ
fi (c - b ) Á + =0
Ë bc (a + c ) (a + b) ˜¯ then prove that the triangle ABC is isosceles.

fi (c - b ) = 0 7.6 AREA OF TRIANGLE


Ê Ê ( a + b + c ) ( 2a + b + c ) ˆ ˆ Statement:
Á∵ Á + ˜ π 0˜
Ë Ë bc (a + c ) (a + b) ¯ ¯ DABC is given by
1
fi b=c (i) D = bc sin A
2
fi D is isoceles.
1
(ii) D = ca sin B
2
EXERCISE 5 1
(iii) D = ab sin C
1. In a DABC, 3a = b + c, then prove that 2

Ê Bˆ Ê Cˆ 1
cot Á ˜ cot Á ˜ = 2 Proof: (i) We have, D = ¥ AB ¥ CD
Ë 2¯ Ë 2¯ 2
1
Bˆ 2Ê
fi D= ¥ c ¥ b sin A
2. In a DABC cos A + cos C = 4 sin Á ˜ , then prove 2
Ë 2¯
1
that a, b, c are in A.P. fi D = bc sin A
2
3. In a DABC s – a) (s – b) = s(s – c)
then prove that angle C is 90∞. Similarly, we can easily proved that (ii) and (iii).
A B C
4. In a DABC , cot , cot anr in A.P., then In any DABC: D = s ( s - a)( s - b)( s - c)
2 2 2
prove that a, b, c are in A.P.
Properties of Triangles 345

1
Proof: As, we know that D = bc sin A
2 fi
( 3 +1 ) =
b
=
c
sin (105∞) sin 45∞ sin 30∞
1 Ê Aˆ Ê Aˆ
=
2
¥ bc ¥ 2 sin Á ˜ cos Á ˜
Ë 2 ¯ Ë 2¯

( 3 +1 ) =
b
=
c
cos (105∞) sin 45∞ sin 30∞
(s - b) (s - c) s (s - a)
= bc ¥ ¥
bc bc

( )=
3 +1 b
=
c
= s (s - a ) (s - b) ( s - c) 3 +1 1 1
2 2 2 2
7.6.1 Some Solved Examples fi 2 2 = b 2 = 2c
Ex-1. In any DABC a = 2 , b = 3 and c = 5 ,
fi b=2 & c= 2
DABC. 1
b c sin A .
1 3 2
Soln. DABC = ¥ 2¥ 3 =
2 2 1
= ¥ 2 ¥ 2 ¥ sin(105∞)
Ex-2. In any DABC, prove that 2

a 2 - b2 sin A sin B 1 3 +1
D= ¥ . = ¥2¥ 2 ¥
2 sin( A - B) 2 2 2

Soln. We have,
a 2 - b2
¥
sin A sin B
=
( 3 +1 )
2 sin( A - B) 2

=
(
k 2 sin 2 A - sin 2 B ) ¥ sin A.sin B Ex-4. In a DABC, prove that:
Ê Aˆ Ê Bˆ Ê Cˆ
2 sin ( A - B ) cot Á ˜ + cot Á ˜ + cot Á ˜
Ë 2¯ Ë 2¯ Ë 2¯ (a + b + c)2
= 2
k 2 ¥ sin ( A + B ) ¥ sin ( A - B ) sin A.sin B cot A + cot B + cot C a + b2 + c2
= ¥
2 sin ( A - B ) Soln. We have, cot A + cot B + cot C
k ¥ sin ( A + B ) ¥ sin A.sin B
2
cos A cos B cos C
= = + +
2 sin A sin B sin C
k 2 ¥ sin (p - C ) ¥ sin A.sin B b2 + c2 - a 2 c2 + a 2 - b2 a 2 + b2 - c2
=
2
= 2bc + 2ac + 2ab
a b ak bk ck
k 2 ¥ sin C ¥ ¥
= k k
b2 + c2 - a 2 c2 + a 2 - b2 a 2 + b2 - c2
2 = + +
2abc k 2abc k 2abc k
1
= ¥ a b sin C
2 a 2 + b2 + c2
=
= D. 2abc k
Ex-3.
=
(a 2
+ b2 + c2 )
3 + 1)cm., then prove that the Ê1 ˆ
4 ¥ Á ab˜ ¥ sin C
1 Ë2 ¯
( 3 + 1) cm2.
(a ) ..............(i)
2 2
+ b2 + c2
a b c =
Soln. As we know that, = = 4¥D
sin A sin B sin C
Ê Aˆ Ê Bˆ Ê Cˆ
Also, cot Á ˜ + cot Á ˜ + cot Á ˜
Ë 2¯ Ë 2¯ Ë 2¯
346 Comprehensive Trigonometry with Challenging Problems & Solutions for Jee Main and Advanced

(a + b + c ) A
¥ cot ÊÁ ˆ˜
1 1 1
= Now, + +
(b + c - a ) Ë 2¯ a 2
b 2
g2
s2 a2 b2 c2
= ....................(ii) = + +
D 4D 2 4D 2 4D 2
Dividing (ii) by (i), we get,
Ê Aˆ Ê Bˆ Ê Cˆ =
(a 2
+ b2 + c2 )
cot Á ˜ + cot Á ˜ + cot Á ˜ 4D 2
Ë 2¯ Ë 2¯ Ë 2¯ s2
=
cot ( A) + cot ( B ) + cot (C ) (
4 a 2 + b2 + c2 ) 1
= ¥
(
a 2 + b2 + c2 )
D 4D
Ê Aˆ Ê Bˆ Ê Cˆ
cot Á ˜ + cot Á ˜ + cot Á ˜
Ë 2¯ Ë 2¯ Ë 2¯ ( a + b + c )2 1
= ¥ (cot A + cot B + cot C )
fi = 2 D
cot ( A) + cot ( B ) + cot (C ) (
a + b2 + c2 ) (cot A + cot B + cot C )
=
s2
D
Ex.-5. DABC, prove that D < .
4
Soln. Let a, b, and c s be the
Ex.-7. p1 , p2 , p3
semi perimeter.
vertices A, B, C and D
s, (s – a), (s – b) and (s – c)
ABC, prove that
Applying, A.M. ≥ G.M., we get,
1 1 1 2ab Ê Cˆ
s + (s - a ) + (s - b) + (s - c) + - = ¥ cos 2 Á ˜ .
fi p1 p2 p3 ( a + b + c ) ¥ D Ë 2¯
4
Soln. Let AD = p1, BE = p2 and CF = p3
≥ 4 s (s - a ) (s - b) ( s - c)
1 1 1
D=
¥ a ¥ p1 = ¥ b ¥ p2 = ¥ c ¥ p3
4 s - (a + b + c ) 4 2 2 2 2
fi ≥ D
4 2D 2D 2D
fi p1 = , p2 = , p3 =
4 s - 2s 1 a b c
fi ≥ (D )2
4 1 1 1
Now, + -
s 1 p1 p2 p3
fi ≥ (D )2
2 a b c
= + -
s2 2D 2D 2D
fi D<
4 (a + b - c )
=
2D
Ex-6. a, b, g
ABC, then prove that (a + b + c - 2c )
=
1 1 1 1 2D
+ 2 + 2 = (cot A + cot B + cot C )
a 2
b g D (2s - 2c )
=
Soln. Let AD = a, BE = b and CF = g 2D
(s - c)
=
1 1 1 D
D= ¥ a ¥ AD = ¥ b ¥ BE = ¥ c ¥ CF
2 2 2 2ab ¥ s ( s - c ) 1
= ¥
2D 2D 2D D ¥s 2ab
fi AD = , BE = , CF =
a b c
2ab s ( s - c )
2D 2D 2D = ¥
fi a= , b= , g = D ¥s 2ab
a b c
Properties of Triangles 347

2 ab Ê Cˆ Soln. Here,
= ¥ cos 2 Á ˜ 1 1 1
(a + b + c ) D Ë 2¯ D = ¥ p ¥ p1 = ¥ q ¥ p2 = ¥ r ¥ p3
2 2 2
Ex-8. a, b, c and d
2D 2D 2D
Ê a 2 + b2 + c2 ˆ fi p= ,q = ,r=
Á ˜ p1 p2 p3
Ë d2 ¯

Soln. We have, ( a - b ) + (b - c ) + (c - d ) ≥ 0 sin P sin Q sin R


2 2 2
= =
p q r
fi ( )
2 a 2 + b 2 + c 2 ≥ 2 ( ab + bc + ca )
Given sin P, sin Q, sin R are in A.P.
p, q, r ΠA.P
fi ( ) (
3 a 2 + b2 + c2 ≥ a 2 + b2 + c2 ) fi
2D 2D 2D
+2 (ab + bc + ca ) fi , , ΠA.P
p1 p2 p3
fi ( )
3 a 2 + b2 + c 2 > (a + b + c ) > d 2
2

7.6.2 m-n Theorem


3 (a 2
+ b2 +c )
2 Statement: D is a point on the side BC
fi >1 BD : DC = m : n a n d – ADC = q, – BAD = a and
d2 – DAC = b , then


(a 2
+ b2 + c2 )>1 (i) (m + n) cot q = m cot a - n cot b
d2 3 (ii) (m + n) cot q = n cot B - m cot C
(a 2
+ b2 + c2 ) is 1 . Soln. Given
BD m
= and –ADC = q
d2 3 DC n

3 A
Ex-9. In a triangle ABC A + cos B + cos C = ,
2 a b

3
Soln. We have, cos A + cos B + cos C = q
2
B m D n C
Ê Aˆ Ê Bˆ Ê Cˆ 3
fi 1 + 4 sin Á ˜ sin Á ˜ sin Á ˜ = So, –ABD = 180∞ - (a + 180∞ - q ) = (q - a )
Ë 2¯ Ë 2¯ Ë 2¯ 2

Ê Aˆ Ê Bˆ Ê Cˆ 1 and –ACD = 180∞ - (q + b )


fi sin Á ˜ sin Á ˜ sin Á ˜ =
Ë 2¯ Ë 2¯ Ë 2¯ 8
BD AD
From, DABD, = ......(i)
It is possible only when sin a sin (q - a )
Ê Aˆ 1 Ê Bˆ 1 Ê Cˆ 1
sin Á ˜ = , sin Á ˜ = , sin Á ˜ = DC AD
Ë 2¯ 2 Ë 2¯ 2 Ë 2¯ 2 From DADC, =
sin b sin (180∞ - (q + b ))
A p B p C p
fi = , = , =
2 6 2 6 2 6 DC AD
fi = .......(ii)
sin b sin (q + b )
p p p
fi A= , B= , C=
3 3 3 Dividing (i) and (ii), we get,

D BD sin b sin (q + b )
=
Ex-10. In a DPQR P, sin Q, sin R are in A.P. DC sin a sin (q - a )
348 Comprehensive Trigonometry with Challenging Problems & Solutions for Jee Main and Advanced

m sin b sin q cos b + cos q sin b


fi =
n sin a sin q cos a - cos q sin a

m (sin q cos b + cos q sin b ) sin a 7.6.3 Some Solved Examples


fi =
n (sin q cos a - cos q sin a ) sin b Ex-1. AD ABC
to AB. Prove that tan A + 2 tan B = 0
Dividing the R.H.S by sin a sin b sin q , we get,
Soln. Since AD is the median, so BD : DC = 1 : 1
m cot b + cot q A
=
n cot a - cot q
90° A - 90°

fi n (cot b + cot q ) = m (cot a - cot q )


90°+ B

fi (m + n) cot q = m cot a - n cot b B D C

BD m Clearly, –ADC = 90∞ + B .


(ii) Given = and – ADC = q
DC n
Now, applying m : n
o
– ADB = 180 - q
(1 + 1) cot(90∞ + B) = 1.cot(90∞) - 1.cot( A - 90∞)
Here, – ABD = B and – ACD = C
fi -2 tan B = 0 - ( - tan A)
So, – BAD = 180∞ - (180∞ - q + B ) = q - B
fi -2 tan B = tan A
and – DAC = (180∞ - (q + C )) fi tan A + 2 tan B = 0
BD AD
DABD, = ......(i)
sin (q - B ) sin B fi
1 1 1
, , ΠA.P
p1 p2 p3
DADC,
DC AD p1 , p2 , p3 ΠH .P
= fi
sin (180∞ - (q + C )) sin C

DC AD D BC
fi = ........(ii) Ex-2.
sin (q + C ) sin C ABC and D be its area, then prove that
Dividing (i) by (ii), we get, b2 - c2
cot q = , where –ADB = q
BD sin (q + C ) sin C
4D
. =
DC sin (q - B ) sin B Soln.
A

m sin (q + C ) sin C
fi . =
n sin (q - B ) sin B
q
m sin C.sin (q - B ) C D B
fi =
n sin B.sin (q + C ) By m : n
m sin C (sin q cos B - cos q sin B ) (1 + 1) cot q = 1.cot C - 1.cot B
fi =
n sin B (sin q cos C + cos q sin C ) 2 cot q = cot C - cot B

a 2 + b2 - c2 a 2 + c2 - b2
hand side by sin B sin C sin q , we get, fi 2 cot q = -
2ab sin C 2ab sin B
m cot B - cot q
=
n cot C + cot q a 2 + b2 - c2 a 2 + c2 - b2
fi 2 cotq = -
fi (m + n) cot q = n cot B - m cot C 4D 4D
Properties of Triangles 349

fi 2 cotq =
(
2 b2 - c2 ) R.
4D
D

fi cotq =
(b 2
- c2 )
4D 7.7.2 Circum-Radius:
R DABC is given by
a b c
(i) R = = =
EXERCISE 6 2 sin A 2 sin B 2 sin C
abc
1. In a DABC –A = 60∞, b = 4 cm. and c = 3 cm. (ii) R =
DABC is 3 cm2. 4D
Proof:
cos A cos B cos C
2. In a DABC = = and the side A
a b c
a ABC is
3 F E

O
ABC, a = 6, b = 3 and cos (A – B) = 4/5, B C
then prove that ar ( D ABC ) = 9 sq.u .
D

A + cos B + cos C = 3/2, then prove that the


triangle ABC Let ABC be a D
BC, CA and AB intersect at O O
( )
5. In a triangle ABC, D = 6 + 2 3 sq.u and OA = OB = OC = R
(i) We have –BOC = 2 –BAC = 2A
– B = 45∞ , a = 2 ( 3 + 1), then prove that the side b is 4.
fi –BOD = –COD = A
30∞ and 45∞ and the BD a a
BOD, sin A = = =
( 3 + 1) , then prove that OB 2
R
2R

ar ( D ABC ) = ( 3 + 1) sq.u.
1
a
2 fi R=
2sin A
Similarly, it can be shown that
b c
4 3 , then prove that the remaining R= and R =
2sin B 2sin C
two sides are 2 and 3, respectively.
a b c
Hence, R = = =
2 sin A 2 sin B 2 sin C
7.6 RADII OF CIRCLE CONNECTED (ii) As we know that,
WITH A TRIANGLE 1
D= bc sinA.
2
7.7 Introduction
2D
fi sin A =
bc
a 2D
fi =
7.7.1 Circumcircle of a triangle and its radius: 2R bc
D is called fi 4DR = abc
350 Comprehensive Trigonometry with Challenging Problems & Solutions for Jee Main and Advanced

abc fi 1 – cos2A + 1 – cos2B + sin2C = 2


fi R= Ex-5. In any DABC, prove that
4D
A cos a + b cos b + c cos c = 4R sin A sin B sin C
Soln. We have, a cos a + b cos b + c cos C
6.7.3 Some solved examples
= 2R (sin A.cos A + sin B.cos B + sin C.cos C )
Ex-1. In a DABC a = 18 cm., b = 24 cm. and c = 30 cm.,
2R
= [2 sin A.cos A + 2 sin B.cos B + 2sin C cos C ]
Soln. Clearly , the triangle is right angled. 2
(∵ 182 + 242 = 302)
= R (sin 2 A + sin 2 B + sin 2C )

1 = R ( 4 sin A. sin B.sin C )


= ¥ 24 ¥ 18 = 12 ¥ 18
2
= 4R.sin A.sin B.sin C
=R Ex-6. In any DABC, prove that
D = 2 R 2 sin A sin B sin C .
abc
=
4D 1
Soln. We have, D = ¥ a ¥ b ¥ sin C
2
18 ¥ 24 ¥ 30
= = 15.
4 ¥ 12 ¥ 18 D=
1
¥ 2 R sin A ¥ 2 R sin B ¥ sin C

Ex-2. 2 3 2

fi D = 2 R 2 ¥ sin A.sin B.sin C


Soln. As we know that,
Ex.-7 In any triangle ABC, prove that,
a
= 2R sin A sin B sin C 3
sin A + + =
a b c 2R
2 3
fi 2R = sin A sin B sin C
sin(60∞) Soln. We have, + +
a b c
3
fiR= =2 sin A sin B sin C
3 = + +
2 R sin A 2 R sin B 2 R sin C
2
1 1 1 3
= + + =
Ex-3. 2R 2R 2R 2R
R.
Ex-8. In any triangle ABC, a, b, c are in A.P. and p1, p2 and
Soln. Let a = 3, b = 4 and c = 5. Clearly, it is a right angled p3
triangle 1 1 1 3R
1 that, + + £
¥4¥3¥ p1 p2 p3 D
2
R Soln. Let AD = p1, BE = p2 and CF = p3 .
abc
= 1 1 1
4D D= ¥ a ¥ p1 = ¥ b ¥ p2 = ¥ c ¥ p3
2 2 2
3¥ 4¥5
= = 10. 2D 2D 2D
6 fi p1 =, p2 = , p3 =
Ex-4. R2 = a2 + b2 + c2, then prove that the triangle is a b c
right angled 1
+
1
+
1
Now,
Soln. We have 8R2 = a2 + b2 + c2 p1 p2 p3
8R2 = (2RsinA)2 + (2RsinB)2 + (2RsinC)2 a+b+c
=
fi sin2A + sin2B + sin2C = 2 2D
Properties of Triangles 351

2 R (sin A + sin B + sin C ) 1 Ê1 ˆ


= = ÁË ab sin C ˜¯
2D DR 2
3R
£ (∵ sin A £ 1, sin B £ 1, sin C £ 1) =
1
¥D
D DR
Ex-9. If p1, p2 and p3 are the altitudes of a triangle ABC 1
from the vertices a, b and c, respectively, then =
R
1 1 1 1
prove that + + =
p1 p2 p3 r
EXERCISE 7
1 1 1
Soln. Here, D = ap1 = bp2 = cp3 1 In a triangle ABC, the sides are 6, 8 and 10, respec-
2 2 2
2D 2D 2D
fi p1 = , p2 = , p3 = 2 If twice the square on the diameter of the circle is equal
a b c
to sum of the squares on the sides of the inscribed
1 1 1 triangle ABC, then prove that
Now, + +
p1 p2 p3 sin 2 A + sin 2 B + sin 2 C = 2
a+b+c 3 In an acute angled triangle ABC, prove that
=
2D cos A 1
=
2s 4R2 - a2 R
=
2D 4 In an acute angled triangle ABC,
s cos B 1
= prove that =
D 4R - b
2 2 R
1 5 In an acute angled triangle ABC,
=
r cos C 1
prove that =
Ex-10. If p1, p2 and p3 are the altitudes of a triangle ABC 4R - c
2 2 R
from the vertices a, b and c, respectively, then 6 If p1, p2 and p3 are the altitudes of a triangle ABC from
cos A cos B cos C 1 the vertices a, b and c, respectively, then
prove that + + =
p1 p2 p3 R a 2b2 c 2
prove that p1 p2 p3 =
Soln. We have 8R3
cos A cos B cos C 7. If p1, p2 and p3 are the altitudes of a triangle ABC from
+ + the vertices a, b and c, respectively, then prove that
p1 p2 p3
bp1 cp2 ap3 a 2 + b 2 + c 2
1 + + =
= (a cos A + b cos B + c cos C ) c a b 2R
2D
8 O is the circum centre of R1 , R2 and R3 and
1
=
2D
[2 R sin A cos A + 2 R sin B cos B D OBC , D OCA are respectively the radii of the cir-
cum-centre of the triangles and DOAB and DOAB,
+2R sin C cos C ] prove that
R a b c abc
= (sin 2 A + sin 2 B + sin 2C ) + + =
2D R1 R2 R3 R3
R 9. In an acute angled triangle ABC, prove that
= (4 sin A sin B sin C )
2D a sec A + b sec B + c sec C
=2
2R tan A tan B tan C
= (sin A sin B sin C ) 10. In any triangle ABC, prove that
D
2R Ê a ˆ Ê b ˆ (a cos A + b cos B + c cos C ) = 4R sin A sin B sin C
= ¥ Á ˜ ¥ Á ˜ ¥ (sin C )
D Ë 2R ¯ Ë 2R ¯
352 Comprehensive Trigonometry with Challenging Problems & Solutions for Jee Main and Advanced

7.8 INSCRIBED CIRCLE AND then ID, IE and IF are perpendicular to these sides and ID
= IE = IF = r.
ITS RADIUS (i) We have, ar(DABC)
On this topic, we shall discuss various circles connected = ar ( D IBC ) + ar ( D ICA) + ar ( D IAB )
with a triangle and the formula for the circle which can be
1 1 1
inscribed within a D and touch each of the sides is called its fi D= ar + br + cr
inscribed circle or incircle. 2 2 2
The centre of this circle is the point of intersection of 1
fi D= r (a + b + c)
the bisecter of the angle of the triangle. The radius of this 2
circle is always denoted by ‘r’ and is equal to the length 1
= r ¥ 2s
of the perpendicular from its centre to any of the sides of 2
the D.
= r sD
7.8.1 In-radius fi r=
s
Statement:
(ii) We know that, the length of the tangents to a circle
Prove that the in-radius r of the inscribed circle of a triangle
from a given point is equal.
ABC is given by
fi AE = AF, bd = BF and CD = CE
D Now, 2s = a + b+ c
(i) r = .
s = BC + CA + AB
A B C = (BD + DC) + (CE + EA)+ (AF + FB)
(ii) r = ( s - a ) , r = (s - b) , r = (s - c) ; = 2 (BD + AE + CD)
2 2 2
= 2 (BC + AE)
Ê Bˆ Ê Cˆ = 2 (a + AE)
a sin Á ˜ sin Á ˜
Ë 2¯ Ë 2¯ fi s = a + AE.
(iii) r = ;
Ê Aˆ fi AE = s – a
cos Á ˜ Ê Aˆ r
Ë 2¯ Now, in DIAE, tan Á ˜ =
Ë 2 ¯ AE
Ê Aˆ Ê Cˆ Ê Aˆ Ê Aˆ
b sin Á ˜ sin Á ˜ fi r = AE tan Á ˜ = (s – a) tan Á ˜ .
Ë 2¯ Ë 2¯ Ë 2¯ Ë 2¯
r= ;
Ê Bˆ Ê Bˆ
cos Á ˜ Similarly r = (s – b) tan Á ˜ ,
Ë 2¯ Ë 2¯
Ê Cˆ
Ê Bˆ Ê Aˆ r = (s – c) tan Á ˜ .
c sin Á ˜ sin Á ˜ Ë 2¯
Ë 2¯ Ë 2¯
r=
Ê Cˆ Hence, the result.
cos Á ˜
Ë 2¯ (iii) In a D IBD and DICD , we have

Ê Aˆ Ê Bˆ Ê Cˆ Ê Bˆ r Ê Cˆ r
(iv) r = 4 R sin Á ˜ sin Á ˜ sin Á ˜ tan Á ˜ = and tan Á ˜ =
Ë 2¯ Ë 2¯ Ë 2¯ Ë 2¯ BD Ë 2¯ CD

Proof: r r
A fi BD = , CD =
Ê Bˆ Ê Cˆ
tan Á ˜ tan Á ˜
Ë 2¯ Ë 2¯
A/2
Now, a = bd + CD
r r r r
= +
Ê Bˆ Ê Cˆ
tan Á ˜ tan Á ˜
Ë 2¯ Ë 2¯
r
B/2 B/2 È Ê Bˆ Ê Cˆ ˘
B C Í cos ÁË 2 ˜¯ cos ÁË 2 ˜¯ ˙
Let ABC be a triangle such that the internal bisectors of the = rÍ + ˙
angles of the triangle intersect at I. Suppose the incircle Í sin Ê B ˆ sin Ê C ˆ ˙
ÍÎ ÁË 2 ˜¯ ÁË ˜¯ ˙
touches the sides BC, CA and AB at D, E and F respectively 2 ˚
Properties of Triangles 353

È Ê Cˆ Ê Bˆ Ê Cˆ Ê Bˆ ˘ 7.8.2 Some Solved Examples


Í sin ÁË 2 ˜¯ cos ÁË 2 ˜¯ + cos ÁË 2 ˜¯ sin ÁË 2 ˜¯ ˙ Ex-1. In any DABC, prove that
= rÍ ˙ a cos a + b cos B + c cos C = 4 R sin a sin B sin C.
Í Ê Bˆ Ê Cˆ ˙
ÍÎ sin Á ˜ sin Á ˜ ˙˚ Soln. We have, a cos a + b cos B + c cos C
Ë 2¯ Ë 2¯
= 2R (sin A cos A + sin B.cos B + sin C.cos C )
Ê C Bˆ
sin Á + ˜
Ë 2 2¯ = R (2 sin A cos A + 2 sin B.cos B + 2 sin C.cos C )
=r
Ê Bˆ Ê Cˆ = R (sin 2 A + sin 2 B + sin 2C )
sin Á ˜ sin Á ˜
Ë 2¯ Ë 2¯
= R ( 4sin A.sin B.sin C )
Ê p Aˆ
sin Á - ˜
Ë 2 2¯ = 4 ( R sin A.sin B.sin C )
=r
Ê Bˆ Ê Cˆ Ex-2. In a DABC, prove that
sin Á ˜ sin Á ˜
Ë 2¯ Ë 2¯ 1 1 1 1
+ + =
Ê Aˆ ab bc ca 2rR
cos Á ˜
Ë 2¯
=r Soln. We have,
1 1
+ +
1
Ê Bˆ Ê Cˆ ab bc ca
sin Á ˜ sin Á ˜
Ë 2¯ Ë 2¯
a+b+c
=
Ê Bˆ Ê Cˆ abc
sin Á ˜ sin Á ˜
Ë 2¯ Ë 2¯
fir = a 2s
A
Ê ˆ =
cos Á ˜ 4D R
Ë 2¯
D
Ê Cˆ Ê Aˆ
b sin Á ˜ sin Á ˜
Ë 2¯ Ë 2¯ = r
Similarly, r = , 2D R
Ê Bˆ
cos Á ˜ 1
Ë 2¯ =
2r R
Ê Aˆ Ê Bˆ
c sin Á ˜ sin Á ˜
Ë 2¯ Ë 2¯ Ex-3. In a DABC, prove that
r= rˆ
Ê Cˆ Ê
cos Á ˜ cos a + cos b + cos c = Á1 + ˜
Ë 2¯ Ë R¯
Ê Bˆ Ê Cˆ Soln. We have, cos a + cos B + cos C
a sin Á ˜ sin Á ˜
Ë 2¯ Ë 2¯ Ê A + Bˆ Ê A - Bˆ
(iv) We have, r = = 2 cos Á + cos C
Ë 2 ˜¯
cos Á
Ê Aˆ Ë 2 ˜¯
cos Á ˜
Ë 2¯
Êp Cˆ Ê A - Bˆ
= 2 cos Á - ˜ cos Á + cos C
Ê Bˆ Ê Cˆ Ë 2 2¯ Ë 2 ˜¯
sin Á ˜ sin Á ˜
Ë 2¯ Ë 2¯
=2 R sin a ¥ Ê Cˆ Ê A - Bˆ
Ê Aˆ = 2 sin Á ˜ cos Á + cos C
cos Á ˜ Ë 2¯ Ë 2 ˜¯
Ë 2¯
Ê Cˆ Ê A - Bˆ Ê Cˆ
Ê Aˆ Ê Aˆ Ê Bˆ Ê Cˆ = 2 sin Á ˜ cos Á + 1 - 2 sin 2 Á ˜
4 R sin Á ˜ cos Á ˜ sin Á ˜ sin Á ˜ Ë 2¯ Ë 2 ˜¯ Ë 2¯
Ë 2¯ Ë 2¯ Ë 2¯ Ë 2¯
= Ê Cˆ Ê Ê A - Bˆ Ê Cˆˆ
Ê Aˆ = 1 + 2 sin Á ˜ Á cos Á - sin Á ˜ ˜
cos Á ˜ Ë 2¯Ë Ë 2 ¯ ˜ Ë 2 ¯¯
Ë 2¯

Ê Aˆ Ê Bˆ Ê Cˆ Ê Cˆ Ê Ê A - Bˆ Ê A + Bˆˆ
= 1 + 2 sin Á ˜ Á cos Á - cos Á
= 4R sin Á ˜ sin Á ˜ sin Á ˜
Ë 2¯ Ë 2¯ Ë 2¯ Ë 2¯Ë Ë 2 ¯ ˜ Ë 2 ˜¯ ˜¯

Hence, the result.


354 Comprehensive Trigonometry with Challenging Problems & Solutions for Jee Main and Advanced

Ê Cˆ Ê Ê Aˆ Ê Bˆˆ =
1
(1 + cos ( A) + 1 + cos B + 1 + cos C )
= 1 + 2 sin Á ˜ Á 2 sin Á ˜ sin Á ˜ ˜
Ë 2¯Ë Ë 2¯ Ë 2 ¯¯ 2
Ê Aˆ Ê Bˆ Ê Cˆ =
1
(3 + cos ( A) + cos B + cos C )
= 1 + 4 sin Á ˜ sin Á ˜ sin Á ˜
Ë 2¯ Ë 2¯ Ë 2¯ 2
Ê rˆ 1Ê Ê r ˆˆ
= Á1 + ˜ = Á 3 + Á1 + ˜ ˜
Ë R¯ 2Ë Ë R¯¯
Ex-4. In a DABC, prove that 1Ê rˆ
= Á 4 + ˜¯
s D 2Ë R
sin A + sin B + sin C = =
R Rr
Ê r ˆ
Soln. We have, sin A + sin B + sin C = Á2 +
Ë 2 R ˜¯
a b c
= + + Ex-7. If the distances of the sides of a triangle ABC from
2R 2R 2R
a circum-center be x, y and z, respectively,
a+b+c a b c abc
= then prove that + + = .
2R x y z 4 xyz
2s
= Soln. Let O is the circum-centre and OD = x,
2R OE = y, OF = z, respectively.
s Also, OA = R = OB = OC
=
R We have, x = OD = R cos A
D a a
= .cos A =
r 2sin A 2 tan A
=
R
D a
= fi tan A =
rR 2x
b c
Ex-5. In any DABC, prove that Similarly, tan B = & tan C =
a cot a + b cot B + c cot C = 2(r + R) 2y 2z
Soln. We have, a cot a + b cot B + c cot C As we know that, in a triangle ABC,
È cos A cos B tan A + tan B + tan C = tan A.tan B.tan C
= Í2 R sin A ¥ + 2 R sin B ¥
Î sin A sin B a b c a b c
fi + + = . .
cos C ˘ 2x 2 y 2z 2x 2 y 2z
+2R sin C ¥
sin C ˙˚ a b c a.b.c
fi + + = .
x y z 4.x. y.z
= 2R (cos A + cos B + cos C )
Ex-8. If in a triangle DABC, O is the circum-centre and R
Ê rˆ is the circum - radius and R1, R2, R3 are the circum
= 2 R Á1 + ˜
Ë R¯ radii of the traingles D OBC , D OCA and
= 2 (R + r ) DOAB, respectively, then prove that
a b c abc
Ex-6. In any DABC, prove that + + = .
R1 R2 R3 R3
Ê Aˆ Ê Bˆ Ê Cˆ r
cos 2 Á ˜ + cos 2 Á ˜ + cos 2 Á ˜ = 2 + Soln. We have,
Ë 2¯ Ë 2¯ Ë 2¯ 2R
OB.OC.BC R.R.a R 2 .a
A B C R1 = = =
Soln. We have, cos2 + cos2 + cos2 4 D OBC 4 D1 4 D1
2 2 2
a 4 D1
1Ê 2 Ê Aˆ 2 Ê Bˆ 2 Ê Cˆˆ fi = 2
= Á 2 cos Á ˜ + 2 cos Á ˜ + 2 cos Á ˜ ˜ R1 R
2Ë Ë 2¯ Ë 2¯ Ë 2 ¯¯
Properties of Triangles 355

b 4 D2 c 4 D3 BD AB c
Similarly, = 2 and = 2 fi = =
R2 R R3 R DC AC b
a b c DC b
+ + fi +1 = +1
Thus, BD c
R1 R2 R3
BC b + c
4D1 4D 2 4D 3 fi =
= + + BD c
R 2
R 2
R2 ac
4 ( D1 + D 2 + D 3 ) fi BD =
b+c
=
R2 BD AD
In, DABD, =
4D A
Ê ˆ sin B
= sin Á ˜
R2 Ë 2¯

4D BD sin B
= fi AD =
R2 Ê Aˆ
sin Á ˜
Ë 2¯
Ex-9. If p1, p2, p3 are respectively the perpendiculars from ac sin B 2D
the vertices of a D to the opposite sides, = =
Ê Aˆ (b + c)sin ( A / 2)
(abc )2 (b + c)sin Á ˜
then prove that p1 ◊ p2 ◊ p3 = . Ë 2¯
8R3 2D
Soln. Let AD = p1, BE = p2 and CF = p3 Similarly, BE =

(c + a ) sin ÊÁË ˜
1 1 1 2¯
Then, D = a p1 = b p2 = c p3
2 2 2 2D
& CF =

fi p1 =
2D
, p2 =
2D
, p3 =
2D (a + b) sin ÊÁË ˜
a b c 2¯
2D 2D 2D
We have, p1. p2 . p3 = . . EXERCISE 8
a b c
8 D3 1. In a DABC, if a = 4 cm., b = 6 cm. and c = 8 cm., then
=
abc
3
Ê abc ˆ its in-radius.
Ë 4 R ˜¯

3. If the sides of a triangle are 3 : 7 : 8, then
=
abc R:r.
4. Two sides of a triangle are 2 and 3 and the
Ex-10 Find the bisectors of the angles of a DABC included angle is 30∞, then prove that its in-radius
A
is
1
2
(3 -1 . )
A/2
5. In an equilateral triangle, prove that R = 4 r.
r r 6. If a, b, c are sides of a triangle ABC, then
I 1 1 1 1
prove that + + = .
ab bc ca 2 r R
r
B/2 C/2
7. In a triangle ABC, prove that
B C
Since IA is the internal angle bisctor of a sec A + b sec B + c sec C
= R.
2 tan A.tan B.tan C
–A, so we can write
AB BD 8. In a DABC, prove that
= A B C
AC DC (b + c ) tan ÊÁË ˆ˜¯ + (c + a ) tan ÊÁË ˆ˜¯ + (a + b) tan ÊÁË ˆ˜¯
2 2 2
356 Comprehensive Trigonometry with Challenging Problems & Solutions for Jee Main and Advanced

= 4 (r + R )
A

9. In a DABC, if C = 90∞ , prove that


A/2 A/2
1
(a + b) = R + r B D C
2
F E

7.9 ESCRIBED CIRCLE OF A


TRIANGLE AND THEIR RADII
7.9.1 In circle which touches the side BC and two sides AB
and AC produced of a triangle ABC is called the escribed
opposite to the angle A. Let I1 be the point of intrersection of external bisectors of
Its radius is denoted by r1. Similarly, r2 and r3 denote angles B and C of DABC. Suppose the circle touches the
the radii of escribed circle opposite to the angle b and c, side BC at D and sides AB and AC produced at F and E,
respectively. respectively.
The centres of the escribed circle are called the ex- Clearly I1D = I1E = I1F = r1.
centres. The centre of the escribed circle opposite to the (i) ar(DABC)
angle a is the point of intersection of the external bisectors = ar ( D I1 AC ) + ar ( D I1 AB ) - ar ( D I1 BC )
of angles B and C.
The internal bisector of angle a also passes through the 1 1 1
fi D= r1b + r1c - r1a
same point. 2 2 2
The centre is generally denoted by I1. 1
= r (b + c – a)
2 1
7.9.2 Radii of Ex-circles r1
Statement: fi D= ( a + b + c - 2a )
2
In any DABC, prove that the ex-radii are given by
r1
(i) r1 =
D
; r2 =
D
; r3 =
D = ( 2 s - 2a )
2
s-a s-b s-c
A B C = r1 ( s - a )
(ii) r1 = s tan ; r2 = s tan ; r3 = s tan
2 2 2 D
fi r1 =
B C s-a
a cos cos
2 2 ; Similarly, it can be shown that
(iii) r1 =
A D D
cos r2 = and r3 =
2 s-b s-c
C A
b cos cos (ii) We know that the lengths of the tangents to a circle
r2 = 2 2 ; from an external point are equal.
B
cos fi AE = AF, BD = BF and CD = CE
2 Now, AE + AF= (AC + CE) + (AB + BF)
A B = (AC + CD) + (AB + BD)
c cos cos
2 2 = AC + AB + (BD + CD)
and r3 =
C = AC + AB + BC
cos
2 =b+c+a
A B C fi 2AF = 2s
(iv) r1 = 4R sin cos cos fi AF = s
2 2 2
Ê Aˆ I F r r
B A C Now, D I1 AF , tan Á ˜ = 1 = 1 = 1
r2 = 4R sin cos cos Ë 2 ¯ AF AF s
2 2 2
C A B A
r3 = 4R sin cos cos fi r1= s tan
2 2 2 2
Properties of Triangles 357

Similarly, it can be shown that a


B C and R=
r2= s tan and r3 = s tan 2sin A
2 2 B C
2 R sin A cos cos
Hence, the result. 2 2
fi r1=
A
(iii) In D I1 BD , we have cos
2
Ê p - B ˆ I1 D r
tan Á = = 1 A A B C
Ë 2 ˜¯ BD BD sin cos cos cos
=R 2 2 2 2
Ê p Bˆ r A
fi tan Á - ˜ = 1 cos
Ë 2 2¯ BD 2
B r A B C
fi cot = 1 = 4R sin cos cos
2 BD 2 2 2
B A C
B Similarly, r2 = 4R sin cos cos
fi BD = r1 tan 2 2 2
2
C C A B
Similarly, in D I1CD , we have, CD = r1 tan . and r3 = 4R sin cos cos .
2 2 2 2
Now, a = BC Hence, the result
= BD + DC This completes the proof of the statement.
B C
= r1 tan + r1 tan 7.9.3 Some Solved Examples
2 2
Ex-1. In a DABC, if a = 18 cm., b = 24 cm.,
Ê B Cˆ and c r1, r2 and r3.
= r1 Á tan + tan ˜
Ë 2 2¯ Soln. Now 2s = a + b + c = 18 + 24 + 30 = 72
Ê B + Cˆ fi 2 s = 72
Ë 2 ˜¯
sin Á
= r1 fi s = 36
B C
cos cos
2 2 We have, D = s ( s - a ) ( s - b ) ( s - c )
A 36 (36 - 18) (36 - 24) (36 - 30)
cos =
= r1 2
B C = 36 ¥ 18 ¥ 12 ¥ 6
cos .cos
2 2
= 36 ¥ 9 ¥ 12 ¥ 12
B C
a cos cos = 6 ¥ 3 ¥ 12
fi r1 = 2 2
A = 216
cos
2 D 216 216
Thus, r1 = = = = 12
A C s - a 36 - 18 18
b cos cos
Similarly, r2 = 2 2 D 216 216
B r2 = = = = 18
cos s - b 36 - 24 12
2
A B D 216 216
c cos cos and r3 = = = = 36
2 2 s - c 36 - 30 6
and r3 =
C s-c b-c
cos =
2 Ex-2. In a triangle ABC if ,
s-a a-b
B C
a cos cos then prove that a, b, c are in A.P.
(iv) We have, r1 = 2 2
A s-c b-c
cos Soln. Given =
2 s-a a-b
358 Comprehensive Trigonometry with Challenging Problems & Solutions for Jee Main and Advanced

2s - 2c b - c Soln. Since a, b, c are in A.P. as well as in G.P., so


fi =
2 s - 2a a - b a=b=c
a+b-c b-c D
fi = Now, r1 = = r2 = r3
b+c-a a-b s-a
a+b-c b+c-a Êr r ˆ
fi = Thus, Á 1 - 2 + 10˜
b-c a-b Ë r2 r3 ¯
a c = 1 – 1 + 10 = 10
fi +1 = -1
b-c a-b
Ex-5. If r1 < r2 < r3 and the ex-radii of a right angled
c a triangle and r1 = 1, r2 = 2 , then prove that
fi + =2
a-b b-c
c (b - c ) + a ( a - b ) 3 + 17
r3 = .
fi =2
(a - b) (b - c ) 2
D D
bc - c 2 + a 2 - ab Soln. We have, r1 = = 1 , r2 = =2
fi =2 s-a s-b
( a - b ) (b - c ) D
and r3 =
fi (
bc - c + a - ab = 2 ab - ac - b + bc
2 2 2
) s-c
D
fi 2b 2 - bc - c 2 + a 2 - 3ab + 2ac = 0 fi s-a = D , s-b=
2
Ex-3. If r1 , r2 , r3 are in H.P., then prove that D
a, b, c are in A,P and s - c =
r3
Soln. Given r1 , r2 , r3 are in H.P.
Ê 1ˆ Ê1 1ˆ
2r r fi c = D Á1 + ˜ , a = D Á + ˜ ,
fi r2 = 1 3 Ë 2¯ Ë 2 r3 ¯
r1 + r3
Ê 1ˆ
D D b = D Á1 + ˜
D
2. . Ë r3 ¯
fi = s - a s -c
s-b D D Since triangle is right angled, so
+
s-a s-c
a 2 + b2 = c2
1 1
D 2 (r3 + 2)
2. . 2 2 2
1 s-a s-c Ê 3ˆ Ê r + 1ˆ
fi = fi D2 Á ˜ = + D2 Á 3 ˜
s-b 1 1 Ë 2¯ 4 (r3 )
2
Ë r3 ¯
+
s-a s-c
1 Ê 1 1 ˆ fi
Ê 3ˆ
2
(r3 + 2) + Ê r3 + 1ˆ
2 2

fi +
1 1 ÁË ˜¯ = ÁË r ˜¯
ÁË ˜¯ = 2. . 2 4 ( r3 )
2
s-b s-a s-c s-a s-c 3

9r32 = ( r3 + 2) + 4 (r3 + 1)
2 2
1 Ê s-c+s-a ˆ 2 fi
fi Á ˜ =
s - b Ë (s - a) (s - c)¯ (s - a) (s - c) fi 4 r32 - 12r3 - 8 = 0
fi (2s - a - c ) = 2 ( s - b) fi r32 - 3r3 - 2 = 0
fi a + c = 2b
3 ± 17 3 + 17
fi a, b, c ΠA.P fi r3 = = .
2 2
Ex-4. In a triangle ABC, if a, b, c are in A.P. as well as r3 is positive.
as in G.P. then prove that the value of Ex-6. Two sides of a triangle are the roots of p
Ê r1 r2 ˆ x 2 - 5 x + 3 = 0 . If the angle between the sides is .
3
ÁË r - r + 10˜¯ is 10. then prove that the value of r.R is 2/3.
2 3
Properties of Triangles 359

Soln. Let a, b be the sides of a triangle.


Then a + b = 5 and ab = 3 = (
3¥ 7+4 3 )
Now, cos C =
a 2 + b2 - c2 ( )
= 12 + 7 3 sq. u.
2ab
Ex-9. If in a triangle r = r1 – r2 – r3, then prove that the
Ê p ˆ 19 - c
2
fi cos Á ˜ = triangle is right angled.
Ë 3¯ 6
Soln Given r = r1 - r2 - r3
1 19 - c 2 fi r1 - r = r2 + r3
fi =
2 6
D D D D
fi - = +
fi 19 - c 2 = 3 s-a s s-b s-c
fi c=4 1 1 1 1
fi - = +
D abc s-a s s-b s-c
Thus, r.R = ¥
s 4D (s - a - s ) (s - c + s - b)
fi =
abc abc 3.4 12 2 (s - a ) (s - b) (s - c)
= = = = =
4s 2 (a + b + c ) 2 (5 + 4) 18 3 a a
fi =
Ex-7. In an isosceles triangle of which one angle is 120∞, (s - a ) (s - b) (s - c)
circle of radius 3 is inscribed, then prove that the (s - b) (s - c)
fi =1
(
area of the triangle is 12 + 7 3 sq. u. ) (s - a)
Ê Aˆ
a b fi tan 2 Á ˜ = 1
Soln. By sine rule, = Ë 2¯
sin(120∞) sin(30∞)
Ê Aˆ
a b fi tan Á ˜ = 1
fi = Ë 2¯
3/2 1/ 2
Ê Aˆ Êpˆ
fi a=b 3 fi tan Á ˜ = tan Á ˜
Ë 2¯ Ë 4¯
r= 3
p
fi A=
3 2
fi = tan(15∞)
a/2 Thus, the triangle is right angled.
Ex-10. In a DABC, prove that r.r1.r2.r3 = D2.

2 3
a
= 2- 3 ( ) Soln. We have, r.r1.r2 .r3

2 3 D D D D
fi a= = . . .
s (s - a ) (s - b) (s - c)
(2 - 3 )
D4
a 2 3 1 2 =
Now, b = = ¥ = s (s - a ) (s - b) ( s - c)
3 (2 - 3 ) 3 (2 - 3 ) D4
Thus, the required area =
D2
1
= ¥ ab ¥ sin(30∞) = D2
2
1 2 3 2 1 (r1 + r2 ) =
(r2 + r3 ) = (r3 + r1 )
= ¥ ¥ ¥ Ex-14. Prove that .
(
2 2- 3 2- 3 2) ( ) 1 + cos C 1 + cos A 1 + cos B

( 3)
r1 + r2
2
= 3¥ 2+ Soln. We have,
1 + cos C
360 Comprehensive Trigonometry with Challenging Problems & Solutions for Jee Main and Advanced

D D Ê 1 1 ˆ Ê 1 1ˆ Ê 1 1ˆ
+ Soln. We have, Á + ˜ Á + ˜ Á + ˜
= s-a s-b Ë r1 r2 ¯ Ë r2 r3 ¯ Ë r3 r1 ¯
Ê Cˆ
2 cos 2 Á ˜
Ë 2¯ Ê s - a s - bˆ Ê s - b s - cˆ Ê s - c s - aˆ
=Á + ˜Á + ˜Á + ˜
Ë D D ¯Ë D D ¯Ë D D ¯
D (s - a + s - b)
(s - a ) (s - b) =
1
¥ abc
= D3
Ê Cˆ
2 cos 2 Á ˜
Ë 2¯ 1
= 3
¥ abc
D (2s - a - b) Ê abc ˆ
= ÁË ˜
s (s - c) 4R ¯
(s - a ) (s - b) ¥ 2 ¥
ab 64 R3
=
D ¥ abc ( a b c )2
=
s ( s - a ) (s - b) ( s - c)
D ¥ abc abc EXERCISE 9
= =
D 2
D
1 1 1 1
Similarly, we can easily proved that, 1. In a triangle DABC, prove that + + = , where
r1 r2 r3 r
(r2 + r3 ) = abc & (r3 + r1 ) =
abc r is inradius and r1 , r2 , r3 are exradii.
1 + cos A D 1 + cos B D
(r2 + r3 ) = abc & (r3 + r1 ) abc 2. In a triangle DABC, prove that
Thus, = r1 + r2 + r3 - r = 4 R
1 + cos A D 1 + cos B D
3. In a triangle DABC, prove that
Ê1 1ˆ Ê1 1 ˆ Ê1 1ˆ
Ex-15 Prove that Á - ˜ Á - ˜ Á - ˜ r1r2 + r2 r3 + r3 r1 = s 2
Ër r ¯Ër r ¯Ër r ¯ 1 2 3
4. In a triangle DABC, prove that
16 R r1 + r2 - r3 + r = 4 R cos C
=
r 2 (a + b + c )
2
Ê r ˆÊ r ˆ
5. In a triangle if Á1 - 1 ˜ Á1 - 2 ˜ = 2 ,
Ê1 1ˆ Ê1 1 ˆ Ê1 1ˆ Ë r2 ¯ Ë r3 ¯
Soln. We have, Á - ˜ Á - ˜ Á - ˜
Ë r r1 ¯ Ë r r2 ¯ Ë r r3 ¯ prove that the triangle is right angled.
6. In a triangle DABC, prove that
Ê s (s - a )ˆ Ê s (s - b)ˆ Ê s (s - c)ˆ
=Á - - - 1 1 1 1 a 2 + b2 + c2
ËD D ˜¯ ÁË D D ˜¯ ÁË D D ˜¯ + + + =
r12 r12 r12 r12 D2
1
= (s - s + a ) (s - s + b) (s - s + c) 7. In a triangle DABC, prove that
D3
(r1 - r ) (r2 - r ) (r3 - r ) = 4r 2 R
1
= ¥ abc 8. In a triangle DABC, prove that
D3
(
r 2 + r12 + r2 2 + r32 = 16 R 2 - a 2 + b 2 + c 2 )
abc s2
= 16 ¥ ¥ . 9. In a triangle DABC, prove that
4 D D 2 ( 2 s )2
b-c c-a a-b
+ + =0
16 R r1 r2 r3
=
r 2
( a + b + c )2 10. In a triangle DABC, prove that
(r1 + r2 ) (r2 + r3 ) (r3 + r1 ) = 4 R
Ê 1 1 ˆ Ê 1 1ˆ Ê 1 1ˆ 64 R3
Ex-16. Prove that Á + ˜ Á + ˜ Á + ˜ = (r1r2 + r2 r3 + r3r1 )
Ë r1 r2 ¯ Ë r2 r3 ¯ Ë r3 r1 ¯ ( a b c )2
Properties of Triangles 361

7.10 REGULAR POLYGON \ A2M =


1
a = A3M.
If a polygon has all its sides equal in length and also all its 2
angles equal then the polygon is called a regular polygon. We know that, the whole angle subtended at the
The circle inscribed in the regular polygon and touching centre ‘O’ by the regular polygon is 2p
all the sides of the regular polygon is called inscribed circle. radian 2p
The circle which passes through all the vertices of the \ –A2 O A3 =
n
regular polygon is called its circumscribed circle. p
If the polygon has n-sides, then the sum of the interrior fi –A2 O M = –A3 OM = .
n
(n - 2) ¥ p
angles is (n – 2) ¥ p and each angle is . From DA2OM, we have,
n
1
7.10.1 Statement A2 M a
Êpˆ 2 a
sin Á ˜ = = =
In a regular polygon of A1A2A3....An of n-sides of each length Ë n¯ OA2 R 2R
a is given by
a a Êpˆ
a Ê aˆ fiR = = cosec Á ˜
(i) R = cosec Á ˜ Êpˆ 2 Ë n¯
2 Ë n¯ 2sin Á ˜
Ë n¯
where R = circum-radius.
Êpˆ AM a
a Ê aˆ again, tan Á ˜ = 2 =
(ii) r = cot Á ˜ , where r = in-radius. Ë n¯ OM 2r
2 Ë n¯
a Ê aˆ
a2n Êpˆ fir= cot Á ˜
(iii) D = cot Á ˜ 2 Ë n¯
4 Ë n¯
ii) Let the area of the regular polygon be denoted by D.
where D = area of the regular polygon Join O to the n-vertices A1, A2,....., An of the regular
n R2 Ê 2p ˆ polygon.
(iv) D = sin Á ˜ The regular polygon is divided into n-isosceles triangle
2 Ë n¯
of equal area.
Ê 2p ˆ Therefore, the area D of regular polygon is given by
(v) D = n r tan Á ˜
2
Ë n¯ D = n-times the area of isosceles DA2 OA3
1
Proof: fiD = n ¥ ¥ A2 A3 ¥ OM
2
1 1
fiD = n ¥ ¥a¥ OM
2 2
O 1 a Êpˆ
fiD = n ¥ ¥a¥ cot Á ˜
A1 A4 2 2 Ë n¯
R p p R
n n na 2 Êpˆ
fiD = cot Á ˜
4 Ë n¯
A2 A3
(iii) In D OA2 M , we have,
(i) Let A1A2, A2A3 and A3A4 be three consecu
Êpˆ OM
tive sides of one regular polygon. cos Á ˜ =
Ë n¯ OA2
A1A2 = A2A3 = A3A4 = a.
Let the bisectors OA2 and OA3 of the angles Êpˆ
–A1A2A3 and –A2A3A4 respectively meet at O. fi OM = R cos Á ˜
Ë n¯
Clearly, O is the centre of both circumscribed 1
A2 M a
circle and inscribed circle. Êpˆ
Also, sin Á ˜ = = 2
Let R and r be the radii of circumcribed Ë n¯ OA2 R
and inscribed circle, then clearly Êpˆ
OA2 = OA3 = R and fi a = 2R sin Á ˜
Ë n¯
OM = r, where OM is perpendicular from O on A2A3.
362 Comprehensive Trigonometry with Challenging Problems & Solutions for Jee Main and Advanced

na 1 2 - A0 A12
fi D= ¥ OM fi =
2 2 2
n Êpˆ Êpˆ
= ¥ 2R sin Á ˜ ¥ R cos Á ˜ fi A0 A12 = 1
2 Ë n¯ Ë n¯
fi A0 A1 = 1
nR 2 Ê 2p ˆ
= sin Á ˜
2 Ë n¯ Ê 2p ˆ OA0 + OA1 - A0 A2
2 2 2
Also, cos Á ˜ =
Ë 3¯ 2.OA0 .OA1
(iv) In D OA2 M , we have OM = r
Êpˆ AM 1 1 + 1 - A0 A22
\ tan Á ˜ = 2 fi - =
Ë n¯ OM 2 2.1.1
a 1 2 - A0 A22
Êpˆ a fi - =
fi tan Á ˜ = 2 = 2 2
Ë n¯ r 2r
fi A0 A2 = 3
Êpˆ
fi a = 2r tan Á ˜
Ê 4p ˆ OA0 + OA4 - A0 A4
2 2 2
Ë n¯
Again, cos Á ˜ =
Ë 3¯ 2.OA0 .OA4
na
fi D= ¥ OM
2 1 + 1 - A0 A42
=
n Ê nˆ 2.1.1
= ¥ 2r tan Á ˜ ¥ r
2 Ë 2¯ 2 - A0 A42
=
Ê nˆ 2
= nr2 tan Á ˜
Ë 2¯ 1 2 - A0 A42
fi - =
Hence, the result. 2 2
This completes the proof of the statement.
fi A0 A4 = 3
7.10.2 Some Solved Examples Hence, the value of
Ex-1. If A0, A1, ......A5 be the consecutive vertices of a
A0 A1. A0 A2 . A0 A4 = 1. 3. 3 = 3 .
the product of length of A0A1, A0A2 and A0A4. Ex-2. If the Area of circle is A1 and area of regular pentagon
Soln. inscribed in the circle is A2.
A2 Find the ratio of area of two.
Soln. Let R be the radius of the circle .
A4 A3
Then, A1 = p R
2

O
nR 2 Ê 2p ˆ
D= sin Á ˜
A1 2 Ë n¯
A5
5. R 2 Ê 360∞ ˆ
A0 and A2 = sin Á
2 Ë 5 ˜¯
Here, OA0 = O A1 = O A2 = ............= OA5 = 1.
5 2 5R 2
and = R sin(72∞) = ¥ cos(18∞)
2p 2 2
–A0OA1 = = –A1OA2 = ............ = –A4OA5
6 A1
Now,
Êpˆ OA02 + OA12 - A1 A2 A2
Now, cos Á ˜ =
Ë 3¯ 2OA0 .OA1 p R2 2p
= = ¥ sec(18∞)
1 1 + 1 - A1 A2 5 2
R cos(18∞)
5
fi =
2 2.1.1 2
Properties of Triangles 363

Ex-3. Let A1, A2, A3, A4 and A5 be the vertices of a


regular pentagon inscribed in a unit circle taken in = 6 2 ( )
3 +1

order. Show that A1A2 ¥ A1A3 = 5 . Ex-5. A regular pentagon and a regular decagon have the
Soln. same perimiter. Find the ratio of its area.
A4 Soln. Let the perimeter of the pentagon and the decagon
A3 be 10 x.
Then each side of the pentagon is 2x and the decagon
O is x.
A5 Let A1 = the area of the pentagon
2p
A2
5 Êpˆ
= 5 x cot Á ˜
2
Ë 5¯
A1
and A2 = the area of the decagon
Here, OA1 = OA2 = OA3 = OA4 = OA5 = 1
5 2 Êpˆ
and = x cot Á ˜
2 Ë 10 ¯
2p
–A1OA2 = = –A2OA3 = .......... = –A4OA5 Êpˆ Êpˆ
5 5 x 2 cot Á ˜ 2 cot Á ˜
A Ë 5¯ Ë 5¯
Ê 2p ˆ OA + OA2 - A1 A2
2 2 2
Now, 1 = =
Now, cos Á ˜ = 1 A2 5 2 Êpˆ Êpˆ
Ë 5¯ 2.OA1.OA2 x cot Á ˜ cot Á ˜
2 Ë 10 ¯ Ë 10 ¯
1 + 1 - A1 A22
fi sin(18∞) = 2 cot(36∞) 2 cos(36∞)sin(18∞)
2.1.1 = =
cot(18∞) sin(36∞) cos(18∞)
5 - 1 2 - A1 A22
fi = 2 cos(36∞)sin(18∞)
4 2 =
2 sin(18∞) cos 2 (18∞)
5 -1 5 - 5
fi A1 A22 = 2 - = 2 cos(36∞)
2 2 =
(1 + cos(36∞))
5- 5
fi A1 A2 = Ê 5 + 1ˆ
2 2Á ˜
Ë 4 ¯
5+ 5 =
Similarly, A1 A3 = . Ê 5 + 1ˆ
2 Á1 + 4 ˜
Thus, A1A2 ¥ A1A3 Ë ¯

Ê 5 - 5ˆ Ê 5 + 5ˆ 25 - 5 20
=
2 ( )
5 +1
= Á 2 ˜ ¥Á 2 ˜ = = = 5
Ë ¯ Ë ¯ 4 4 ( 5 + 5)
Ex-4. The sides of a regular do-decagon is 2 ft . 2 ( 5 + 1)
Find the radius of the circumscribed circle. =
a Êpˆ 5 ( 5 + 1)
Soln. R = cosec Á ˜ , As we know that, the circum-
2 Ë n¯
2
radius of n sided regular polygon =
5
a Êpˆ
= cosec Á ˜ , where a = side and n = number of Ex-6. If 2a be the sides of a regular polygon of n-sides. R
2 Ë n¯
sides and r be the circum radius and inradius, then prove
Êpˆ Êp ˆ
= 6 ¥ cosec Á ˜ that r + R = a cot Á ˜ .
Ë 12 ¯ Ë 2n ¯

= 6 ¥ cosec (15∞) 2a Êpˆ Êpˆ


Soln. We have, R = cosec Á ˜ = a cosec Á ˜
6¥2 2 2 Ë n¯ Ë n¯
=
3 -1
364 Comprehensive Trigonometry with Challenging Problems & Solutions for Jee Main and Advanced

2a Êpˆ Êpˆ Soln. Let the perimeter of the two polygons are
and r = cot Á ˜ = a cot Á ˜ n x and 2 n x, respectively .
2 Ë n¯ Ë n¯
Then each side of the polygons are 2x and x.
Now, r + R Let A1 = the area of the polygon of n sides
Êpˆ Êpˆ Êpˆ
= a cot Á ˜ + a cosec Á ˜ = n x cot Á ˜
2
Ë n¯ Ë n¯ Ë n¯

Ê Êpˆ Ê p ˆˆ and A2 = the area of the decagon


= a Á cot Á ˜ + cosec Á ˜ ˜
Ë Ë n ¯ Ë n ¯¯ 5 2 Ê p ˆ
= n cot Á ˜
Ê
2 Ë 2 n¯
Êpˆˆ
Á 1 + cos ÁË n ˜¯ ˜ Êpˆ Êpˆ
= aÁ ˜ 5 n 2 cot Á ˜ 2 cot Á ˜
Á sin Ê p ˆ ˜ A Ë n¯ Ë n¯
ÁË ÁË ˜¯ ˜¯ Thus, 1 = =
n A2 5 p
Ê ˆ Êpˆ
n 2 cot Á ˜ cot Á ˜
2 Ë 2n ¯ Ë 2n ¯
Ê Ê p ˆ ˆ
Á 2 cos 2 Á ˜ ˜
Ë 2 n¯ Êpˆ Êpˆ
= aÁ ˜ 2 cos Á ˜ sin Á ˜
Ë n¯ Ë 2n ¯
Á Ê p ˆ Ê p ˆ˜ =
Á 2 sin ÁË 2 n ˜¯ cos ÁË 2n ˜¯ ˜ Êpˆ Êpˆ
Ë ¯ 2 sin Á ˜ cos 2 Á ˜
Ë 2n ¯ Ë 2n ¯
Ê p ˆ
= a cot Á ˜ Êpˆ
Ë 2 n¯ 2 cos Á ˜
Ë n¯
=
Ex-7. A regular pentagon and a regular decagon have the Êpˆ
1 + cos Á ˜
Ë n¯
Soln. Let A1 be the area of the regular pentagon A2 be the
area of the regular decagon. Ex-9. Let A1, A2, A3, ..........., An be the vertices of
Therefore, A1 = A2 an n-sided regular polygon such that
1 1 1
5a 2 Ê p ˆ 6b
2
Êpˆ = + n.
fi cot Á ˜ = cot Á ˜ A1 A2 A1 A3 A1 A4
4 Ë 5 ¯ 4 Ë 6¯
Soln. Let O be the centre and A1A2.......An be the regular
Êpˆ Êpˆ polygon of n-sides.
fi 5a 2 cot Á ˜ = 6b 2 cot Á ˜
Ë 5¯ Ë 6¯

fi 5a 2 cot(36∞) = 6 b 2 cot(30∞) A3

fi 5a cot(36∞) = 6 3 b
2 2

O
a2 b2 2p A2
fi = =l n
6 3 5 cot(36∞)
A1
Hence, the ratio of their perimeters An
5a
=
6b Let OA1 = OA2 = ...................= OAn = r

5 l6 3 5 3 and –A1OA2 = –A2OA3


= ¥ = tan(36∞)
6 5l cot(36∞) 6 2p
= ......... = –An OA1 =
n
Ex-8. If the number of sides of two regular polygon having
From the triangle OA1A2,
the same perimeter be n and 2n, respectively, prove
that their areas are in the ratio Ê 2p ˆ OA + OA2 - A1 A2
2 2 2
cos Á ˜ = 1
2 cos (p /n ) : (1 + cos (p /n )) . Ë n¯ 2.OA1.OA2
Properties of Triangles 365

r 2 + r 2 - A1 A22 Ê 4p ˆ Ê 3p ˆ
= fi ÁË ˜¯ = ÁË p - ˜¯
2.r.r n n
Ê 2p ˆ Ê 7p ˆ
fi A1 A22 = 2r 2 - 2r 2 cos Á ˜ fi ÁË ˜=p
Ë n¯ n ¯

Ê Ê 2p ˆ ˆ fi n= 7
fi A1 A22 = 2r 2 Á1 - cos Á ˜ ˜
Ë Ë n ¯¯
Ê 2p ˆ Ex-10. If A, A1, A2, A3 are the areas of incircle and the ex-
fi A1 A22 = 2r 2 .2 sin 2Á ˜ circles of a triangle, then prove that
Ë n¯
1 1 1 1
Ê 2p ˆ + + = .
= 4r 2 .sin 2 Á ˜ A1 A2 A3 A
Ë n¯
Soln. Let r be the radius of the in-circle and r1, r2 and r3
Ê 2p ˆ are the ex-radii of the given triangle
fi A1 A2 = 2r.sin Á ˜ .
Ë n¯ 1 1 1
Then + +
Ê 4p ˆ A1 A2 A3
Similarly, A1 A3 = 2r.sin Á ˜
Ë n¯
1 1 1
= + +
Ê 6p ˆ p r12 p r12 p r12
& A1 A4 = 2r.sin Á ˜
Ë n¯
1 Ê 1 1 1ˆ
1 1 1 = + +
Given, = + p ÁË r1 r2 r3 ˜¯
A1 A2 A1 A3 A1 A4
1 1
1
=
1
+
1 = ¥
fi p r
Ê 2p ˆ Ê 4p ˆ Ê 6p ˆ
2r.sin Á ˜ 2r.sin Á ˜ 2r.sin Á ˜
Ë n¯ Ë n¯ Ë n¯ 1
=
1 1 1 p r2
fi = +
Êpˆ Ê 2p ˆ Ê 3p ˆ 1
sin Á ˜ sin Á ˜ sin Á ˜
Ë n¯ Ë n¯ Ë n¯ =
A
1
-
1
=
1 Hence, the result.

Êpˆ Ê 3p ˆ Ê 2p ˆ
sin Á ˜ sin Á ˜ sin Á ˜
Ë n¯ Ë n¯ Ë n¯

Ê 3p ˆ Êpˆ
EXERCISE 12
sin Á ˜ - sin Á ˜
Ë n¯ Ë n¯ 1
fi = 1. If R is the radius of the circum-scribing circle of a
Êpˆ Ê 3p ˆ Ê 2p ˆ regular polygon of n-sides with side length ‘a’, then
sin Á ˜ sin Á ˜ sin Á ˜
Ë n¯ Ë n¯ Ë n¯ R is
Ê 2p ˆ Êpˆ a Êpˆ a Ê p ˆ
(a) sin Á ˜ (b) sin
2 cos Á ˜ sin Á ˜
Ë n¯ Ë n¯ 1 2 Ë n ¯ 2 ÁË 2 n ˜¯
fi =
Êpˆ Ê 3p ˆ Ê 2p ˆ a Ê p ˆ a Êpˆ
sin Á ˜ sin Á ˜ sin Á ˜ (c) (d)
Ë n¯ Ë n¯ Ë n¯ cosec Á ˜ cosec Á ˜
2 Ë 2 n¯ 2 Ë n¯
Ê 2p ˆ Ê 2p ˆ Ê 3p ˆ
fi 2 cos Á ˜ sin Á ˜ = sin Á ˜ 2 If a is the area and s be the semi perimeter of a triangle,
Ë n¯ Ë n¯ Ë n¯
then
Ê 4p ˆ Ê 3p ˆ s2 s2
fi sin Á ˜ = sin Á ˜ (a) A £ (b) A £
Ë n ¯ Ë n¯ 3 3 2
Ê 4p ˆ Ê 3p ˆ s2 s2
fi sin Á ˜ = sin Á p - ˜ (c) A > (d) A >
Ë n ¯ Ë n¯ 3 4
366 Comprehensive Trigonometry with Challenging Problems & Solutions for Jee Main and Advanced

3 If A1, A2, A3 are the areas of an inscribed polygon of We have, PL = LB tan (PBL)
2n sides and n-sides and circum-scribed polygon of = LB tan (90∞ - C )
n-sides, then A2, A1, A3 are in
(a) A.P. (b) G.P. = AB cos B cot C
(c) H.P. (d) A.G.P. c
= ¥ cos B cos C
4 If the perimeter of the circle and the perimeter of the sin C
polygon of n-sides are same, then the ratio of the area
= 2RcosB cosC.
of the circle and the area of the polygon of n-sides is
Êpˆ p Êpˆ p Similarly, PM = 2R cos A cos C,
(a) tan Á ˜ : (b) cos Á ˜ :
Ë n¯ n Ë n¯ n PN = 2R cos A cos B

Êpˆ p Êpˆ p Again, AP = AM sec(LAC) = c cosA cosec C


(c) sin Á ˜ : (d) cot Á ˜ : c
Ë n¯ n Ë n¯ n
= ¥ cos A = 2R cos A.
5. The sum of the radii of the circle, which are respec- sin C
tively inscribed in and circum-scribed about a regular Similarly, BP = 2R cos B and CP = 2Rcos C.
polygon of n-sides, is
a Êpˆ a Ê p ˆ 7.11.2 The sides and the angles of the pedal triangle
(a) cot Á ˜ (b) cot Á ˜
2 Ë n¯ 2 Ë 2 n¯ A

a Ê p ˆ a Êpˆ N
(c) tan Á ˜ (d) tan Á ˜ M
2 Ë 2 n¯ 2 Ë n¯
P

7.11 ORTHOCENTRE AND PEDAL B L C


TRIANGLE OF ANY TRIANGLE Since the angles PLC, PMC are right angles, so the points
A P, L, C and M lie on a circle.
N
Thus, –PLM = –PCM = 90∞ - A
M Similarly, P, L, B and M lie on a circle and therefore
P –PLM = –PBN = 90∞ - A
Hence, –NLM = 180∞ - 2 A
B
= the supplement of 2A.
L C
So, –LMN = 180∞ - 2 B and
Let ABC be any triangle and let AL, BM and CN be the
–MNL = 180∞ - 2C
perpendiculars from a, b and c upon the opposite sides of
the triangle. They meet at a point P. This point P is called Hence, its angles are the supplements of twice the angles
the orthocentre of the triangle. of the triangle.
The triangle LMN, which is formed by joining the feet Again, from the triangle AMN, we have,
of the perpendiculars, is called the pedal triangle. MN AM AB cos A
= =
sin A sin ( ANM ) cos ( PNM )
7.11.1 Distances of the Orthocentre from the angular points
c cos A c cos A
of the pedal triangle = =
A
cos ( PAM ) sin C
c
N fi MN = sin A cos A = a cosA
M sin C
P
So, NL = b cos B and LM = c cos C. Hence, the sides of the
pedal triangles are
a cosA, b cosB and c cosC, respectively.
B L C
Properties of Triangles 367

7.11.3 Area of a pedal triangle LMN of a triangle ABC is In-radius


2D cosA cosB cosC ar ( DLMN )
=
A semi - perimeter ( DLMN )

N 1 2
R .sin 2 A.sin 2 B.sin 2C
M = 2
P 2 R.sin A.sin B.sin C
= 2R.cos A.cos B.cos C

B L C

Here, PL = 2R cos B cos C, 7.12 DISTANCE BETWEEN THE


OM = 2R cosC . cosA, CIRCUMCENTRE AND
ON = 2R cos A. cos B
ar ( DLMN )
ORTHOCENTRE
A
1
= ¥ ( R sin 2 A) ¥ ( R sin 2 B ) ¥ (sin 2C )
2
1
= ¥ R 2 ¥ (sin 2 A.sin 2 B.sin 2C ) F O
L
2
P
1
= ¥ R 2 ¥ (8 sin A.sin B.sin C ) ¥ (cos A.cos B.cos C )
2 B D K C
1 2 Ê a b c ˆ
= ¥ R ¥ Á 8. . . ˜ ¥ (cos A.cos B.cos C ) If OF perpendicular to AB, we have,
2 Ë 2R 2R 2R ¯
–OAF = 90∞ - –AOF = 90∞ - C
1 abc
= ¥ ¥ (cos A.cos B.cos C ) Also, –PAL = 90∞ - C
2 R
Thus, –OAP
= 2D ¥ (cos A.cos B.cos C )
= A - –OAF - –PAL
7.11.4 The circum-radius of a pedal triangle LMN of a
R = A - 2 (90∞ - C )
triangle ABC is
2 = A + 2C - 180∞
Circum-radius = A + 2C – (A + B + C) = C – B
MN R sin 2 A R sin 2 A R Also, OA = R and PA = 2R cosA
= = = =
2 sin ( MLN ) 2 sin(180∞ - 2 A) 2 sin 2 A 2
Thus, OP = OA + PA - 2.OA.PA.cos (OAP )
2 2 2

7.11.5 The in-radius of a pedal triangle LMN of a triangle


= R + 4 R cos A - 4 R cos A cos (C - B )
2 2 2 2
ABC is 2R cos A cos B cos C
A = R + 4 R cos A ÈÎcos A - cos (C - B )˘˚
2 2

N
= R - 4 R cos A ÈÎcos ( B + C ) - cos ( B - C )˘˚
2 2
M
P
= R - 8 R cos A cos B cos C
2 2

= R (1 - 8 cos A cos B cos C )


2

B L C
fi OP = R (1 - 8cos A cos B cos C )
368 Comprehensive Trigonometry with Challenging Problems & Solutions for Jee Main and Advanced

7.13 DISTANCE BETWEEN THE 7.14 DISTANCE BETWEEN THE


CIRCUMCENTRE AND THE CIRCUMCENTRE AND CENTROID
INCENTRE i.e. OG = R -
2 2
9
(
1 2
a + b2 + c2 . )
F
Soln:
A B

I H
O E
G
O
C
C A
B
As we know that, centroid divides the orthocentre and
D circumcentre in the ratio 2:1
Let O be the circumcentre and OF be perpendicular to AB. 1
Thus, OG = .OH
Let I be the incentre and IE perpendicular to AC. 3
Then –OAF = 90∞ - C 1
fi OG = .OH
2 2

–OAI = –IAF - –OAF 9

=
A
2
- (90∞ - C ) =
9
(
1 2
R - 8 R 2 cos A.cos B.cos C )
R2
A
= +C -
2
A+ B+C
2
fi OG =
2
9
(
¥ 1 - 4 {cos ( A + B ) + cos ( A - B )}.cos C )
R2
=
C-B
2
fi OG =
2
9
(
¥ 1 + 4 cos 2 C + 4 cos ( A - B ) .cos ( A + B ) )
R2
Also, AI =
IE
Ê Aˆ
=
r
Ê Aˆ
fi OG =
2
9
(
1 + 4 cos 2 C + 4 cos 2 A - 4 sin 2 B )
sin Á ˜ sin Á ˜
Ë 2¯ Ë 2¯ R2
Ê Bˆ Ê Cˆ
fi OG =
2
9
(
1 + 4 - 4 sin 2 C + 4 - 4 sin 2 A - 4 sin 2 B )
= 4 R sin Á ˜ sin Á ˜
Ë 2¯ Ë 2¯

Ê Bˆ Ê Cˆ Ê Ê B Cˆˆ
fi OG =
2 R2
9
( (
9 - 4 sin 2 A + sin 2 B + sin 2 C ))
= 1 - 8 sin Á ˜ sin Á ˜ Á cos Á + ˜ ˜
Ë 2¯ Ë 2¯Ë Ë 2 2 ¯¯ 1
fi OG = R -
2 2
¥
9
Ê Bˆ Ê Cˆ Ê Aˆ
= 1 - 8 sin Á ˜ sin Á ˜ sin Á ˜
Ë 2¯ Ë 2¯ Ë 2¯ {(2R sin A) 2
+ ( 2 R sin A) + ( 2 R sin A)
2 2
}

Ê Bˆ Ê Cˆ Ê Aˆ
OI = R 1 - 8 sin Á ˜ sin Á ˜ sin Á ˜
Ë 2¯ Ë 2¯ Ë 2¯
fi OG = R -
2 2 1 2
9
(
a + b2 + c2 )
Also, OI2 7.15 DISTANCE BETWEEN THE
Ê Aˆ Ê Bˆ Ê Cˆ
= R - 2 R ¥ 4 R sin Á ˜ sin Á ˜ sin Á ˜
2
Ë 2¯ Ë 2¯ Ë 2¯
INCENTRE AND ORTHOCENTRE
i.e., OH = 2r - 4 R cos A cos B cos C
2 2 2
= R - 2 Rr
2
Soln. Let ABC be a triangle, H is the orthocentre and I is
fi OI = R 2 - 2 Rr the incentre.
Hence, the result.
Properties of Triangles 369

A
7.16 EXCENTRAL TRIANGLE

I E
F IA
C C
H
D IB
I
B B
A

Join AH, AI and IH.


IC
In triangle AIH,
IH 2 = AH 2 + AI 2 - 2. AH . AI .cos (–IAH )
A
–IAH = - –HAC
2 The triangle formed by joining the three excentres I A , I B , I C
of a DABC is called an excentral of excentric triangle
A 1
= - (90∞ - C ) = (C - B ) (i) DABC is the pedal triangle of D I A I B I C
2 2
Ê p Aˆ Ê p B ˆ Ê p C ˆ
È 2 2 Ê Bˆ Ê Cˆ (ii) Its angles are Á - ˜ , Á - ˜ , Á - ˜
Thus, 4 R Ícos A + 4 sin Á ˜ sin Á ˜
2 Ë 2 2¯ Ë 2 2¯ Ë 2 2¯
Î Ë 2¯ Ë 2¯
Ê Aˆ
Ê Bˆ Ê Cˆ Ê Cˆ Ê Cˆ Ê Bˆ ˘ (iii) Its sides are I B I C = 4 R cos Á ˜ ,
-4 cos A sin Á ˜ sin Á ˜ cos Á ˜ cos Á ˜ cos Á ˜ ˙ Ë 2¯
Ë 2¯ Ë 2¯ Ë 2¯ Ë 2¯ Ë 2¯˚
È Ê Bˆ
2 Ê Bˆ 2 Ê Cˆ˘ I A I C = 4 R cos Á ˜ ,
– Í-16 R cos A.sin Á ˜ .sin Á ˜ ˙
2
Ë ¯ Ë 2¯˚ Ë 2¯
Î 2
2È 2 Ê Bˆ 2 Ê Cˆ
Ê Cˆ
= 4 R Ícos A + 4 sin Á ˜ sin Á ˜ (1 - cos A)
2 and I A I B = 4 R cos Á ˜
Ë 2¯ Ë 2¯ Ë 2¯
Î
- cos A.cos B.sin C ] Ê Aˆ Ê Bˆ
(iv) II A = 4 R sin Á ˜ ; II B = 4 R sin Á ˜
Ë 2¯ Ë 2¯
2È 2 Ê Aˆ 2 Ê Bˆ 2 Ê Cˆ
= 4 R Ícos A + 8 sin Á ˜ sin Á ˜ sin Á ˜
2
Ê Cˆ
Î Ë 2¯ Ë 2¯ Ë 2¯ II C = 4 R sin Á ˜
Ë 2¯
- cos A.sin B.sin C ] (v) Incentre I of DABC is the orthocentre of the excentric
triangle D I A I B I C .
2 Ê Aˆ 2 Ê Bˆ 2 Ê Cˆ
= 32 R sin Á ˜ sin Á ˜ sin Á ˜
2
Ë 2¯ Ë 2¯ Ë 2¯ (vi) ar ( I A I B I C )

+4 R 2 cos A. (cos A - sin B.sin C ) 1 Ê Ê Bˆˆ Ê Ê Bˆˆ Ê Aˆ


= ¥ Á 4 R cos Á ˜ ˜ ¥ Á 4 R cos Á ˜ ˜ ¥ sin Á 90∞ - ˜
2 Ë Ë ¯
2 ¯ Ë Ë ¯
2 ¯ Ë 2¯
È 2 Ê Aˆ 2 Ê Bˆ 2 Ê Cˆ˘
= 2 Í16 R sin Á ˜ sin Á ˜ sin Á ˜ ˙
2

Î Ë 2¯ Ë 2¯ Ë 2¯˚ Ê Aˆ Ê Bˆ Ê Cˆ
= 8.R .cos Á ˜ .cos Á ˜ .cos Á ˜
2
Ë 2¯ Ë 2¯ Ë 2¯
-4 R 2 cos A. ÈÎcos ( B + C ) + sin B.sin C ˘˚
I 2 I3
(vii) Circum-radius =
= 2r - 4 R cos A.cos B.cos C. 2sin (–I 2 I1 I 3 )
2 2

Hence, the result. I 2 I3


=
Ê Aˆ
2 sin Á 90∞ - ˜
Ë 2¯
370 Comprehensive Trigonometry with Challenging Problems & Solutions for Jee Main and Advanced

Ê Aˆ Soln.
4 R cos Á ˜ A
Ë 2¯
= = 2R
Ê Aˆ
2 cos Á ˜ E
Ë 2¯
Soln: We know that, F
I
Ê Aˆ Ê Bˆ Ê Cˆ
II A = a sec Á ˜ , II B = a sec Á ˜ , II C = a sec Á ˜
Ë 2¯ Ë 2¯ Ë 2¯
B D C
Also,
Ê Aˆ Ê Bˆ Ê A ˆ IF
I A I B = c cosec Á ˜ , I B I C = c cosec Á ˜ , We have, sin Á ˜ =
Ë 2¯ Ë 2¯ Ë 2 ¯ IA
Ê Aˆ
ÊCˆ fi r = IA . sin Á ˜
I C I A = c cosec Á ˜ Ë 2¯
Ë 2¯
Thus, II A . II B . II C Ê Aˆ Ê Bˆ Ê Cˆ Ê Aˆ
fi 4 R sin Á ˜ sin Á ˜ sin Á ˜ = IA . sin Á ˜
Ë 2¯ Ë 2¯ Ë 2¯ Ë 2¯
Ê Aˆ Ê Bˆ Ê Cˆ
= abc ¥ sec Á ˜ sec Á ˜ sec Á ˜ ..........(i) Ê Bˆ Ê Cˆ
Ë 2¯ Ë 2¯ Ë 2¯ fi IA = 4 R sin Á ˜ sin Á ˜ ,
Ë 2¯ Ë 2¯
Also, a = 2R sinA, b = 2R sin b and c = 2R sin C
From (i), we get, II A . II B . II C Ê Aˆ Ê Cˆ
Similarly, IB = 4 R sin Á ˜ sin Á ˜
Ë 2¯ Ë 2¯
= ( 2 R sin A)( 2 R sin B )( 2 R sin C )
Ê Aˆ Ê Bˆ
Ê Aˆ Ê Bˆ Ê Cˆ and IC = 4 R sin Á ˜ sin Á ˜
¥ sec Á ˜ sec Á ˜ sec Á ˜ Ë 2¯ Ë 2¯
Ë 2¯ Ë 2¯ Ë 2¯
7.18 If I be the incentre of a DABC, then
Ê A Aˆ Ê B Bˆ Ê C Cˆ Ê Aˆ Ê Bˆ ÊCˆ
ÁË 2 sin cos ˜¯ ÁË 2 sin cos ˜¯ ÁË 2 sin cos ˜¯ IA. IB . IC = abc ¥ tan Á ˜ tan Á ˜ tan Á ˜
= 8R ¥
3 2 2 2 2 2 2 Ë 2¯ Ë 2¯ Ë 2¯
A B C
cos cos cos Soln.
2 2 2 A
A B C
= 64 R sin sin sin
3
2 2 2 E

A B C F
= 16 R ¥ 4 R sin
2
sin sin I
2 2 2
= 16 R r
2
B D C
Hence, the result.
(vi) If I is the incentre and I A , I B , I C are the excentres From the above diagram,
of the triangle DABC, then prove that r r r
IA = , IB = , IC =
II A .II B .II C = 16 r R 2 Ê Aˆ Ê Bˆ Ê Cˆ
sin Á ˜ sin Á ˜ sin Á ˜
Ë 2¯ Ë 2¯ Ë 2¯
7.17 The distance between the incentre and the angular Thus,
poins of A DABC
IA.IB.IC
Ê Bˆ Ê Cˆ
i.e., IA = 4 R sin Á ˜ sin Á ˜ , r3
Ë 2¯ Ë 2¯ =
Ê Aˆ Ê Bˆ ÊCˆ
Ê Aˆ Ê Cˆ sin Á ˜ sin Á ˜ sin Á ˜
IB = 4 R sin Á ˜ sin Á ˜ , Ë 2¯ Ë 2¯ Ë 2¯
Ë 2¯ Ë 2¯
r 3 ¥ 4R
Ê Aˆ Ê Bˆ =
and IC = 4 R sin Á ˜ sin Á ˜ Ê Aˆ Ê Bˆ Ê Cˆ
Ë 2¯ Ë 2¯ 4 R sin Á ˜ sin Á ˜ sin Á ˜
Ë 2¯ Ë 2¯ Ë 2¯
Properties of Triangles 371

r 3 .4 R D 2 abc D a b c
= = 4 r 2 .R = 4 . 2 . = abc. 2 Similarly, + +
r s 4 D s R1 R2 R3

s ( s - a ) (s - b) ( s - c) Thus,
4
( D1 + D 2 + D3 )
= abc ¥ R2
s 2

4 4D abc
s ( s - a ) (s - b) ( s - c) = ¥D= =
= abc ¥ R R R3
2 2
s 4

(s - a ) (s - b) (s - c) Hence, the result.


= abc ¥ . .
s s s 7.20 The distance between the centre of the nine - point
circle from the Angle a is
(s - a ) (s - b) (s - b) (s - c) R
= abc ¥ .◊ . (1 + 8 cos A.cos B.cos C )
s (s - c) s (s - a) =
2
(s - c) (s - a) Soln.
◊ .
s (s - b) A

Ê Cˆ Ê Aˆ Ê Bˆ
= abc ¥ tan Á ˜ tan Á ˜ tan Á ˜
Ë 2¯ Ë 2¯ Ë 2¯
P
Ê Aˆ Ê Bˆ Ê Cˆ z
= abc ¥ tan Á ˜ tan Á ˜ tan Á ˜
Ë 2¯ Ë 2¯ Ë 2¯
N
Q
Hence, the result.
O
7.19. O is the circumcentre of a triangle DABC and D
R1 , R2 & R3 are respectively the radii of the R y
H
circumference of the D’s OBC, OCA and OAB,
C
a b c abc x M
respectively, then + + = 3 . B L
Soln. R1 R2 R3 R
Let ABC be a triangle, H = orthocentre,
O = circumcentre, D = nine-point centre
A
As we know that, nine point centre is the mid-point
of the orthocentre and circumcentre of a triangle.
Thus, AOH be a triangle, where, AD is the median.
O
In DAOH, 2(AD2 + DO2) = AH2 + AO2
B C 1
fi 2 AD 2 = AH 2 + AO 2 - OH 2 .......(i)
2
abc Now from the triangle ABC, we can write,
As we know that, R = AH = 2R cosA, OA = R,
4D

Let DOBC = D1 , DOCA = D 2 , DOAB = D 3 OH = R (1 - 8 cos A.cos B.cos C ) .

0 B.0C.BC R.R.a R 2 .a From (i), we get, 2AD2


In R1 = = = ,
R2
4D1 4D1 4D1
(
= R 4 cos A + 1 –
2 2
) 2
(1 - 8 cos A.cos B.cos C )
a 4D1
fi =
R1 R 2 R2
= È1 + 8 cos A.{cos (180∞ - B + C ) + cos B.cos C }˘˚
2 Î
b 4D 2 c 4D 3
Similarly, = 2 & = 2 R2
R2 R R3 R fi AD2 = ¥ È1 + 8cos A {- cos B.cos C
4 Î
372 Comprehensive Trigonometry with Challenging Problems & Solutions for Jee Main and Advanced

+ sin B.sin C + cos B.cos C }˘˚ 1


¥
=
4 ( ab + cd )
2
R2
fi AD 2
= (1 + 8cos A.sin B.sin C )
{ )}
4
R È
(
2
ÍÎ 2 ( ab + cd ) + a + b - c - d
2 2 2 2
fi AD = (1 + 8cos A.sin B.sin C )
2
Hence, the result.
{( (
¥ 2 ab + cd ) + a 2 + b 2 - c 2 - d 2 ) }˘˙˚
2

7.17 QUADRILATERAL 1
= ¥
4 ( ab + cd )
2
7.17.1 Area of a quadrilateral, which is inscribed in a circle.
D
b {( ) (
È a 2 + b 2 + 2ab - c 2 - 2cd + d 2
ÎÍ )}
C

a ¥ {(c + 2cd + d ) - ( a + b - 2ab )}˘˙


2 2 2 2
˚
d1
{(a + b) - (c - d ) } ¥ {(c + d ) - (a - b) }
d2 c
2 2 2 2

A =
d 4 (ab + cd )
2
B

¥ ÈÎ{(a + b + c - d ) (a + b - c + d )}
1
Let ABCD be a cyclic quadrilateral such that =
4 ( ab + cd )
2
AB = a, BC = b, CD = c and AD = d.
ar ( ABCD ) = ar ( DABC ) + ar ( DADC ) ¥ (c + d + b - a ) (c + d + a - b )˘˚

1 1 Let a + b + c + d = 2 s
= ab sin B + cd sin D Thus, ( a + b + c - d )
2 2
1 1 = ( a + b + c + d - 2d ) = 2 ( s - d )
= ab sin B + cd sin (p - B )
2 2 Similarly, ( a + b + d - c ) = 2 ( s - c ) ,
1 1
= ab sin B + cd sin B (a + c + d - b) = 2 ( s - b) ,
2 2
1
(b + c + d - a ) = 2 ( s - a ) .
= (ab + cd ) sin B
2 fi sin 2 B

From D’s, BAC and BCD, we have, 2 (s - d ) ¥ 2 (s - c) ¥ 2 (s - b) ¥ 2 (s - a )


=
4 ( ab + cd )
2
a 2 + b 2 - 2 ab cos B = c 2 + d 2 - 2 cd cos D
fi a 2 + b 2 - 2 ab cos B = c 2 + d 2 + 2 cd cos B fi sin 2 B
Ê a 2 + b2 - c2 - d 2 ˆ 16 ¥ ( s - a ) ¥ ( s - b ) ¥ ( s - c ) ¥ ( s - d )
fi cos B = Á =
2 (ab + cd ) ˜¯ 4 ( ab + cd )
2
Ë
fi (ab + cd ) sin B
Now, sin B = 1 - cos B
2 2

(a 2
+ b2 - c2 - d 2 )
2
= 2¥ (s - a ) (s - b) (s - c) (s - d )
= 1-
Hence, the area of the quadrilateral
(2 (ab + cd ))2
1

=
È
{ (
ÍÎ 2 ( ab + cd ) - a + b - c - d
2 2 2 2 2
)}
2 ˘
˙˚
=
2
(ab + cd ) sin B

4 ( ab + cd )
2 = (s - a ) (s - b) (s - c) (s - d )
Properties of Triangles 373

7.17.2 The radius of the circle circumscribing the


2ab cos B - 2cd cos D = a 2 + b 2 - c 2 - d 2 ...(ii)
quadrilateral ABCD.
D Squaring (i) and (ii) and adding, we get,
b
( )
2
16 D 2 + a 2 + b 2 - c 2 - d 2
C

a = 4a b + 4c d
2 2 2 2
d1 d2 c
-8 abcd (cos B cos D - sin B sin D )
A
= 4 (a b + c d ) - 8 abcd cos ( B + D )
d 2 2 2 2
B

= 4 (a b + c d ) - 8 abcd cos 2a
2 2 2 2
a 2 + b2 - c2 - d 2
We have, cos B =
2 (ab + cd ) = 4 (a b + c d ) - 8 abcd 2 cos a - 1
2 2 2 2 2
( )
and AC = a + b - 2 ab cos B
2 2 2
= 4 ( ab + cd ) - 16 abcd cos a
2 2

Ê a 2 + b2 - c2 - d 2 ˆ
= a + b - 2 ab ¥ Á
2 2
fi 16 D
2
ab + cd ˜
Ë ¯
= 4 ( ab + cd )
2

=
(a 2
) (
+ b 2 cd + c 2 + d 2 ab )
( )
2
(ab + cd ) - a 2 + b2 - c2 - d 2 - 16 abcd cos 2 a

(ac + bd ) (ad + bc ) ...................(iii)


= Thus, 16 D
2
(ab + cd )
= 2 ( s - a ) .2 ( s - b ) . 2 ( s - c ) . 2 ( s - d )
In DABC
1 AC -16 abcd cos 2 a
In ¥ ,R
2 sin B
fi D 2 = ( s - a ) ( s - b ) ( s - c ) ( s - d ) - abcd cos 2 a
Ê ( ac + bd ) ( ad + bc ) ˆ
= Á ( s - a ).( s - b).( s - c ).( s - d ) - abcd
Ë (ab + cd ) ˜¯ fi D= cos 2 a
7.17.4. Area of a quadilateral which can have a circle
(s - a ) (s - b) (s - c) (s - d )
= ∏4 inscribed in it.
(ab + cd )2 D R C

1 AC 1 Ê (ab + cd ) (ac + bd ) (ad + bc ) ˆ


¥ =
2 sin B 4 ÁË ( s - a ) ( s - b ) ( s - c ) ( s - d ) ˜¯ S Q

= ar ( ABCD )
7.17.3 Area of a quadrilateral ABCD, when it is not A P B
inscribed.
ar (ABCD) We have, AP = AS, BP = BQ, CQ = CR
= ar ( DABC ) + ar ( DACD ) and DR = RS
AP + BP + CR + DR = AS + BQ + CQ + DS
1 1
= ab sin B + cd sin D fi AB + CD = BC + DA
2 2
fi a+c=b+d
fi 4 D = 2 ab sin B + 2 cd sin D ...............(i)
a+b+c+d
Also, Hence, s = =a+c=b+d
2
a 2 + b 2 - 2ab cos B = c 2 + d 2 - 2cd cos D
Thus, s – a = c, s – b = d, s – c = a, s – d = b
374 Comprehensive Trigonometry with Challenging Problems & Solutions for Jee Main and Advanced

As we know that,
d 2 = OA 2 + OD 2 - 2.OA.OD.cosq .
D 2 = abcd - abcd cos 2 a = abcd sin 2 a
We have, a - b + c - d
2 2 2 2

fi D = abcd sin a
= 2 cos q (OA.OB + OB.OC + OC.OD + OA.OD )
7.17.5. The area of a quadrilateral, which can be both = 2.cos q . AC. BD
inscribed in a circle and circumscribed about
another circle and the radius of the later circle is From (i), we get, area of the quadrilateral ABCD

2 abcd =
2
(
1 2
a - b2 + c2 - d 2 ) tan q .
a+b+c+d
7.17.7. a, b, c and d are the sides of a quadrilateral and
p and q be its diagonals, then its area is
1
4
¥ ( 4 p q - (a + c - b - d ) )
2 2 2 2 2 2 2

A
d
D
a
q
In a quadrilateral ABCD, p
C q c
–B + –D = 180∞
b
fi 2a = 180∞
B
fi a = 90∞
Hence, the area of the quadrilateral, which is As we know that,
inscribed in a circle and circumscribed anothe circle a 2 - b 2 + c 2 - d 2 = 2 p q cos q
is a b c d .
D 2D 2 abcd Area of a quadrilateral ABCD
We have, r = = =
s 2s a + b + c + d 1
p q sinq
=
7.17.6 a, b, c and d are the sides of a quadrilateral 2
taken in order, and q is the angle between the 1 2 2 2
diagonals opposite to b or d, then the area of the = p q sin q
4
1 2 2
( 2
quadrilateral is a + c - b - d tanq
2
)
4 =
1 2 2
4
(
p q 1 - cos 2 q )
A
d

( )
D 1
4 p 2 q 2 - ( 2 pq cosq )
2
a =
q 4
p
( )
q 1 Ê 2 2
4 p q - a 2 - b2 + c2 - d 2 ˆ
C c 2
=
b 2 Ë ¯

B
Hence, the result.
7.17.8. If a quadrilateral can be inscribed in a circle, then
Area of a quadrilateral ABCD the angle between its diagonals is
1
= ¥ AC ¥ BD ¥ sinq .......................(i) Ê2 ( s - a ) ( s - b) ( s - c ) ( s - d ) ˆ
2 sin -1 Á ˜
ÁË (ac + bd ) ˜¯
Now, a = OA + OB - 2.OA.OB.cos (180∞ - q )
2 2 2

b 2 = OC 2 + OB 2 - 2.OC.OB.cosq , Soln. Let ABCD be a quadrilateral, whose sides are


a, b, c and d respectively and its diagonals are p and q.
c 2 = OC 2 + OD 2 - 2.OC.OD.cos (180∞ - q ) Let the angle between the diagonals be q.
Properties of Triangles 375

Then, pq = AC + bd . = x ( x - 1) > 0 as x > 1


1
Area of a quadrilateral ABCD = ¥ p q sinq and a – c
2
2 (s - a ) (s - b) (s - c) (s - d )
2
(
= x + x +1 - x -1
2
) ( )
sinq =
(ac + bd ) = x + 2 > 0, as x > 1
Therefore, a is the greast side
Ê2 (s - a ) (s - b) (s - c) (s - d ) ˆ fi A is the greatest angle
q = sin -1 Á ˜
ÁË (ac + bd ) ˜¯
b2 + c2 - a 2
Now, cos A =
Hence, the result. 2bc

(2 x + 1)2 + ( x 2 - 1) - ( x 2 + x + 1)
7.17.9 If a quadrilateral can be inscribed in a circle as 2 2
well as circumscribed about another circle, then
=
the angle between its diagonal is 2 (2 x + 1) ( x 2 - 1)
Ê ac - bd ˆ
cos -1 Á
Ë ac + bd ˜¯
. (2 x + 1)2 + (2 x 2 + x ) (- x - 2)
=
Since the quadrilateral be circumscribed, then we 2 (2 x + 1) ( x 2 - 1)
1
can write, p q sin q = abcd (2 x + 1)2 + x (2 x + 1) (- x - 2)
2 =
2 (2 x + 1) ( x 2 - 1)
Ê 2 abcd ˆ
fi sin q = Á
Ë pq ¯
˜
(2 x + 1) (2 x + 1 - x 2 - 2 x )
=
2 (2 x + 1) ( x 2 - 1)
Therefore, cos q = 1 - sin q
2

Ê 4abcd ˆ Ê ac - bd ˆ =
(1 - x ) = - 1
2

= cosq = 1 - Á 2˜
=Á ˜ 2 ( x - 1)
2 2
Ë (ac + bd ) ¯ Ë ac + bd ¯
1
Ê ac - bd ˆ fi cos A = -
-1 2
fi q = cos ÁË ˜
ac + bd ¯
fi A = 120∞
Hence, the result.
a b
Ex-2. In any triangle ABC, cos q = , cos j = ,
PROBLEMS FOR JEE ADVANCED EXAM b+c a+c
c
cosy = where j, q and y lie between 0 and
Ex-1. The sides of a triangle are x + x + 1, 2 x + 1
2
a+b
2 Êqˆ 2 Êjˆ 2 Êy ˆ
and x 2 - 1 , prove that the greatest angle is 120°. p, prove that tan Á ˜ + tan Á ˜ + tan Á ˜ = 1
Ë 2¯ Ë 2¯ Ë 2¯
Soln. Let a = x + x + 1, b = 2 x + 1and c = x - 1
2 2
a
Soln. Given cosq =
First we have to shown that, which one is greatest b+c
amongst the sides a, b and c. a a+b+c
1 + cosq = 1 + =
Clearly, a > 0, b > 0 and c > 0 b+c b+c

fi x >1 Êqˆ a + b + c
fi 2 cos 2 Á ˜ =
Ë 2¯ b+c
Now, a – b
2 (b + c )
(
2
)
= x + x + 1 - ( 2 x + 1) fi
Êqˆ
sec2 Á ˜ =
Ë 2 ¯ (a + b + c )
= x -x
2
Êqˆ 2 (b + c )
fi 1 + tan 2 Á ˜ =
Ë 2 ¯ (a + b + c )
376 Comprehensive Trigonometry with Challenging Problems & Solutions for Jee Main and Advanced

Similarly, we can easily proved that Soln. We have


Êjˆ 2 (c + a ) sin 3 q = sin ( A - q ) sin ( B - q ) sin (C - q )
1 + tan 2 Á ˜ =
Ë 2 ¯ (a + b + c )
fi 2 sin 3 q = sin ( A - q ) {2 sin ( B - q ) sin (C - q )}
Êy ˆ 2 (a + b) = sin ( A - q ) {cos ( B - C ) - cos ( B + C - 2q )}
1 + tan 2 Á ˜ =
Ë 2 ¯ (a + b + c )
fi 4 sin 3 q
Adding, we get,
= 2 sin ( A - q ) {cos ( B - C ) - cos ( B + C - 2q )}
2 Êqˆ 2 Êjˆ ˆ 2 Êy
3 + tan Á ˜ + tan Á ˜ + tan Á ˜ = 2sin ( A - q ) cos ( B - C )
Ë 2¯ Ë 2¯ Ë 2¯
-2 sin ( A - q ) cos ( B + C - 2q )
4 (a + b + c )
=
(a + b + c ) = (sin ( A + B - q - C ) - sin ( A + C - q - B ))
- (sin ( A + B + C - 3q ) - sin ( B + C - A - q ))
Êqˆ Êjˆ Êy ˆ
fi 3 + tan 2 Á ˜ + tan 2 Á ˜ + tan 2 Á ˜ = 4
Ë 2¯ Ë 2¯ Ë 2¯ = sin (p - ( 2C + q )) + sin (q - ( 2 B + q ))

Êqˆ Êjˆ Êy ˆ - sin 3q + sin (p - ( 2 A + q ))


fi tan 2 Á ˜ + tan 2 Á ˜ + tan 2 Á ˜ = 1
Ë 2¯ Ë 2¯ Ë 2¯ fi sin 3q + 4 sin 3 q
Hence, the result. = sin ( 2 A + q ) + sin (2 B + q ) + sin (2C + q )
Ex-3. Given the product p of sines of the angles of a triangle
fi 3sinq = (sin 2 A + sin 2 B + sin 2C ) cos q
and the product q
p and + (cos 2 A + cos 2 B + cos 2C ) sin q
q and whose roots are the tangents of the angles of
the triangle. fi (3 - cos 2 A - cos 2 B - cos 2C ) sin q
Soln. Given sin A sin B sin C = p = (sin 2 A + sin 2 B + sin 2C ) cos q
cos A cos B cos C = q fi {(1 - cos 2 A) + (1 - cos 2B ) + (1 - cos 2C )} sin q
p
Thus, tan A tan B tan C = = 4sin A sin B sin C cos q
q
Also, A+ B+C =p
fi (2 sin 2
A + 2 sin 2 B + 2 sin 2 C sin q )
tan A + tan B + tan C = tan A tan B tan C = 4sin A sin B sin C cos q

tan A + tan B + tan C =


p fi Î(
2 È sin 2 A + sin 2 B - sin 2 C )
q
Also, tan A tan B + tan B tan C + tan C tan A
(
+ sin 2 B + sin 2 C - sin 2 A )
1+ q + (sin 2
)
C + sin 2 A - sin 2 B ˘ sin q
˚
=
q = 4sin A sin B sin C cos q
Hence, the required equation is fi 4 [sin A sin B cos C + sin B sin C cos A
Ê pˆ Ê1+ qˆ Ê pˆ + sin C sin A cos B ] sin q
x3 - Á ˜ x 2 + Á ˜ x-Á ˜ =0
Ë q¯ Ë q ¯ Ë q¯
= 4sin A sin B sin C cos q
q x - p x + (1 + q ) x - p = 0 .
3 2
cot q = cot A + cot B + cot C .

Ex-4. In a triangle ABC, if Ex-5. The base of a triangle is divided into three parts.
sin 3 q = sin ( A - q ) sin ( B - q ) sin (C - q ) If t1 , t2 , t3 be the tangents of the angles subtended
by these parts at the opposite vertex, prove that
prove that cot q = cot A + cot B + cot C
Properties of Triangles 377

Ê 1 1ˆÊ 1 1ˆ Ê 1ˆ Ê 1ˆ
+
ÁË t t ˜¯ ÁË t +
t3 ˜¯
= 4 Á 1 + ˜ fi (t1 + t2 ) (t3 + t2 ) = 4t1t3t22 Á1 + ˜
1 2 2 Ë t22 ¯ Ë t22 ¯

Soln. Ê t1 + t2 ˆ Ê t3 + t2 ˆ Ê 1ˆ
fi ÁË t t ˜¯ ÁË t t ˜¯ = 4 Á1 + t 2 ˜
A 12 2 3 Ë 2¯

a b g
Ê 1 1ˆÊ 1 1ˆ Ê 1ˆ
fi +
ÁË t t ˜¯ ÁË t + = 4 Á 1 + ˜
1 2 2 t3 ˜¯ Ë t22 ¯

Ex-6. In a DABC, prove that


Ê rˆ
cos A + cos B + cos C = Á1 + ˜
Ë R¯
q
B x P x Q x C Soln. We have, cos A + cos B + cos C
Let BP = PQ = QC = x and also Ê A + Bˆ Ê A - Bˆ
= 2 cos Á cos Á + cos C
Ë 2 ¯ ˜ Ë 2 ˜¯
let –BAP = a , –PAQ = b , –QAC = g
It is given that tan a = t1 , tan b = t2 Êp Cˆ Ê A - Bˆ
= 2 cos Á - ˜ cos Á + cos C
Ë 2 2¯ Ë 2 ˜¯
and tan g = t3
Ê Cˆ Ê A - Bˆ
Applying m : n rule in triangle ABC, we get, = 2 sin Á ˜ cos Á + cos C
Ë 2¯ Ë 2 ˜¯
(2 x + x ) cot q = 2 x cot (a + b ) - x cot g .....(i)
Ê Cˆ Ê A - Bˆ Ê Cˆ
= 2 sin Á ˜ cos Á + 1 - 2 sin 2 Á ˜
From DAPC, we get, Ë 2¯ Ë 2 ˜¯ Ë 2¯
( x + x ) cot q = x cot b - x cot g .......(ii) Ê Cˆ Ê Ê A - Bˆ Ê Cˆˆ
= 1 + 2 sin Á ˜ Á cos Á ˜ - sin Á ˜ ˜
Dividing (i) by (ii), we get, Ë 2¯Ë Ë 2 ¯ Ë 2 ¯¯
2 cot (a + b ) - cot g 3 Ê Cˆ Ê Ê A - Bˆ Ê A + Bˆˆ
= = 1 + 2 sin Á ˜ Á cos Á - cos Á
cot b - cot g 2 Ë 2¯Ë Ë 2 ˜¯ Ë 2 ˜¯ ˜¯
fi 4 cot (a + b ) - 2 cot g = 3 (cot b - cot g ) Ê Cˆ Ê Ê Aˆ Ê Bˆˆ
= 1 + 2 sin Á ˜ Á 2 sin Á ˜ sin Á ˜ ˜
Ë 2¯Ë Ë 2¯ Ë 2 ¯¯
fi 4 cot (a + b ) = 3 cot b - cot g
Ê Aˆ Ê Bˆ Ê Cˆ

1
=
1 = 1 + 4 sin Á ˜ sin Á ˜ sin Á ˜
Ë 2¯ Ë 2¯ Ë 2¯
4 cot (a + b ) 3 cot b - cot g
Ê rˆ
tan (a + b ) 1 = Á1 + ˜
fi = Ë R¯
4 3 1
- Ex-7. In a DABC, prove that
tan b tan g
s D
tan a + tan b tan b tan g sin A + sin B + sin C = =
fi = R Rr
4 (1 - tan a tan b ) 3 tan g - tan b Soln. We have, sin A + sin B + sin C
t1 + t2 tt a b c
fi = 23 = + +
4 (1 - t1t2 ) 3t3 - t2 2R 2R 2R
a+b+c
fi ( )
4 t2t3 - t1t22t3 = 3t1t3 - t1t2 + 3t2t3 - t22 =
2R
fi (t t
2 3 )
+ t1t2 + t1t3 + t22 = 4t1t3 + 4t1t22t3 =
2s
2R
fi (t3 (t2 + t1 ) + t2 (t1 + t2 )) = 4t1t3 (1 + t22 ) s
=
R
378 Comprehensive Trigonometry with Challenging Problems & Solutions for Jee Main and Advanced

D 1
= (a cos A + b cos B + c cos C )
= r 2D
R
1
D =
2D
[2 R sin A cos A + 2 R sin B cos B
=
rR
+2R sin C cos C ]
Ex-8. In any DABC, prove that
R
a cot A + b cot B + c cot C = 2(r + R) = (sin 2 A + sin 2 B + sin 2C )
Soln. We have, a cot A + b cot B + c cot C 2D
È cos A cos B R
= Í2 R sin A ¥ + 2 R sin B ¥ = (4 sin A sin B sin C )
Î sin A sin B 2D
2R
+2R sin C ¥
cos C ˘ = (sin A sin B sin C )
sin C ˙˚ D
2R Ê a ˆ Ê b ˆ
= 2R (cos A + cos B + cos C ) = ¥ Á ˜ ¥ Á ˜ ¥ (sin C )
D Ë 2R ¯ Ë 2R ¯
Ê rˆ
= 2 R Á1 + ˜ 1 Ê1 ˆ
Ë R¯ = Á ab sin C ˜¯
DRË2
= 2 (R + r ). 1
= ¥D
Ex-9. In any DABC, prove that DR
Ê Aˆ Ê Bˆ Ê Cˆ r 1
cos 2 Á ˜ + cos 2 Á ˜ + cos 2 Á ˜ = 2 + =
Ë 2¯ Ë 2¯ Ë 2¯ 2R R
A B C Ex-11. If the distances of the sides of a triangle ABC from
Soln. We have, cos2 + cos2 + cos2 a circum-center be x, y and z, respectively,
2 2 2
1Ê Ê Aˆ Ê Bˆ Ê Cˆˆ a b c abc
= 2 cos 2 Á ˜ + 2 cos 2 Á ˜ + 2 cos 2 Á ˜ ˜ then prove that + + = .
Á
2Ë Ë 2¯ Ë 2¯ Ë 2 ¯¯ x y z 4 xyz
Soln. Let O is the circum-centre and OD = x,
=
1
2
(1 + cos ( A) + 1 + cos B + 1 + cos C ) OE = y, OF = z, respectively.
Also, OA = R = OB = OC
=
1
2
(3 + cos ( A) + cos B + cos C ) We have, x = OD = R cos A
a a
1Ê Ê r ˆˆ = .cos A =
= Á 3 + Á1 + ˜ ˜ 2sin A 2 tan A
2Ë Ë R¯¯
a
1Ê rˆ fi tan A =
= Á4 + ˜ 2x
2Ë R¯
b c
Ê r ˆ Similarly, tan B = & tan C =
= Á2 + 2y 2z
Ë 2 R ˜¯
As we know that, in a triangle ABC,
Ex-10. If p1, p2 and p3 are the altitudes of a triangle ABC tan A + tan B + tan C = tan A.tan B.tan C
from the vertices A, B and C, respectively, then a b c a b c
fi + + = . .
cos A cos B cos C 1 2x 2 y 2z 2x 2 y 2z
prove that + + =
p1 p2 p3 R a b c a.b.c
fi + + = .
Soln. We have x y z 4.x. y.z
cos A cos B cos C
+ + Ex-12. If in a triangle DABC, O is the circum-center and
p1 p2 p3 R is the circum-radius and R1, R2, R3 are the circum
radii of the traingles D OBC , D OCA and
Properties of Triangles 379

DOAB, respectively, then prove that Ê Bˆ


sin 2 Á ˜
a b c abc r Ë 2¯
+ + = . Similarly, 2 =
R1 R2 R3 R3 ca r
Soln. We have, Ê Cˆ
sin 2 Á ˜
OB.OC.BC R.R.a R 2 .a r Ë 2¯
R1 = = = and 3 =
4 D OBC 4 D1 4 D1 ab r
a 4 D1 r1 r2 r3
fi = 2 Now, + +
R1 R bc ca ab
b 4 D2 c 4 D3 1 Ê 2 Ê Aˆ Ê Bˆ Ê Cˆˆ
Similarly, = 2 and = 2 = sin Á ˜ + sin 2 Á ˜ + sin 2 Á ˜ ˜
R2 R R3 R r ÁË Ë 2¯ Ë 2¯ Ë 2 ¯¯
a b c 1 Ê Ê Aˆ Ê Bˆ Ê Cˆˆ
Thus, + + = Á 2 sin 2 Á ˜ + 2 sin 2 Á ˜ + 2 sin 2 Á ˜ ˜
R1 R2 R3 2r Ë Ë 2 ¯ Ë 2 ¯ Ë 2 ¯¯
4D1 4D 2 4D 3
= + + =
1
2r
(1 - cos ( A) + 1 - cos ( B ) + 1 - cos (C ))
R 2
R 2
R 2

4 ( D1 + D 2 + D 3 )
=
R 2
=
1
2r
(
3 - (cos ( A) + cos ( B ) + cos (C )) )
4D 1 Ê Ê r ˆˆ
= = 3 - Á1 + ˜ ˜
R 2 2r ÁË Ë R¯¯
4D 1 Ê rˆ
= = ÁË 2 - ˜¯
R 2 2r R
abc Ê1 1 ˆ
= =Á -
4R 3 Ë r 2 R ˜¯
bc ca ab
Hence, the result. Ex-14. In any DABC prove that + +
Ex-13. In any DABC, prove that r1 r2 r3
r1 r r 1 1 ÈÊ a b ˆ Ê b c ˆ Ê c a ˆ ˘
+ 2 + 3 = – = 2R ÍÁ + ˜ + Á + ˜ + Á + ˜ - 3˙
bc ca ab r 2R Ë
Î b a ¯ Ë c b ¯ Ë a c ¯ ˚
r1 bc ca ab
Soln. We have Soln. We have, + +
bc r1 r2 r3
Ê Aˆ Ê Bˆ Ê Cˆ Ê 1 1 1 ˆ
4 R sin Á ˜ cos Á ˜ cos Á ˜ = abc Á + +
=
Ë 2¯ Ë 2¯ Ë 2¯ Ë a r1 b r2 c r3 ˜¯
(2 R sin B )(2 R sin C ) Ê s - a s - b s - cˆ
= abc Á + +
Ê Aˆ Ë aD bD c D ˜¯
sin Á ˜
Ë 2¯ abc Ê s - a s - b s - c ˆ
= = + +
Ê Bˆ Ê Cˆ D ÁË a b c ˜¯
4 R sin Á ˜ sin Á ˜
Ë 2¯ Ë 2¯
2 Ê Aˆ abc Ê 2 s - 2a 2s - 2b 2s - 2c ˆ
sin Á ˜ = + +
Ë 2¯ 2 D ÁË a b c ˜¯
=
Ê Aˆ Ê Bˆ Ê Cˆ
4 R sin Á ˜ sin Á ˜ sin Á ˜ abc Ê b + c c+a a+b ˆ
Ë 2¯ Ë 2¯ Ë 2¯ = -1+ -1+ - 1˜
Á
2D Ë a b c ¯
Ê Aˆ
sin 2 Á ˜ abc Ê Ê a b ˆ Ê b c ˆ Ê a c ˆ ˆ
Ë 2¯
= = Á + ˜ +Á + ˜ +Á + ˜ -3
r 2 D ÁË Ë b a ¯ Ë c b ¯ Ë c a ¯ ˜¯
380 Comprehensive Trigonometry with Challenging Problems & Solutions for Jee Main and Advanced

ÊÊ a bˆ Ê b cˆ Ê a cˆ ˆ It will maximum, when AB = AC


= 2 R Á Á + ˜ + Á + ˜ + Á + ˜ - 3˜ Thus, AB2 + AC2 = BC 2 = 4 R2
Ë Ë b a¯ Ë c b¯ Ë c a¯ ¯
fi 2 AB2 = 4R2
Ê B - Cˆ
Ex-15. In a DABC, prove that (r + r1) tan Á fi AB2 = 2R2
Ë 2 ˜¯
Now, 2s = AB + BC + CA = 2AB + BC
Ê C - Aˆ Ê A - Bˆ
+ (r + r2) tan Á
Ë 2 ˜¯
+ (r + r3) tan Á
Ë 2 ˜¯
= 0. (
= 2R 1 + 2 )
Ê B - Cˆ D R2 R
and r = = =
Soln. We have, (r + r1) tan Á
Ë 2 ˜¯ s (
R 2 +1 ) ( 2 +1 )
ÊD
=Á +
D ˆ Ê b - cˆ
˜ ¥Á ˜
Ë s s - a¯ Ë b + c¯
Ê Aˆ
cot Á ˜
Ë 2¯ Therefore,
1 1 1 1
+ + = =
( 2 +1 )
r1 r2 r3 r R
Ê s - a + sˆ Ê b - cˆ s (s - a)
= DÁ ˜ ¥Á ˜¥ Ex-17 Let A1, A2, A3, ..........., An be the vertices of an n-sided
Ë s (s - a) ¯ Ë b + c ¯ (s - b) (s - c) regular polygon such that
Ê b + c ˆ Ê b - cˆ 1 1 1 1
= DÁ ˜¥ ¥ = + ,
ÁË s ( s - a ) ˜¯ ÁË b + c ˜¯ ( s - b) (s - c) A1 A2 A1 A3 A1 A4
n.
Ê b - cˆ 1
= D ¥ (b + c ) ¥ Á ˜ ¥ Soln. Let O be the centre and A1A2.......An be the regular
Ë b + c¯ s (s - a ) (s - b) (s - c) polygon of n-sides.
1
= D ¥ (b - c ) ¥
D
A3
=b–c
Ê C - Aˆ
Similarly, (r + r2) tan Á =c–a O
Ë 2 ˜¯ 2p A2
n
Ê A - Bˆ
and (r + r3) tan Á =a–b
Ë 2 ˜¯
A1
An

Ê B - Cˆ Ê C - Aˆ
Now, (r + r1) tan Á + (r + r2) tan Á Let OA1 = OA2 = ...................= OAn = r
Ë 2 ˜¯ Ë 2 ˜¯
and –A1OA2 = –A2OA3
Ê A - Bˆ
+ (r + r3) tan Á
Ë 2 ˜¯ = ......... = –An OA1 =
2p
n
=b–c+c–a+a–b
From the triangle OA1A2,
=0
Ê 2p ˆ OA + OA2 - A1 A2
2 2 2
Ex-16. If a triangle of maximum area is inscribed within a cos Á ˜ = 1
circle of radius R, then prove that Ë n¯ 2.OA1.OA2
1 1 1 2 +1 r 2 + r 2 - A1 A22
+ + = =
r1 r2 r3 R 2.r.r
Soln. Let ABC be a right angled triangle in a Ê 2p ˆ
circle of radius R fi A1 A22 = 2r 2 - 2r 2 cos Á ˜
Ë n¯
Therefore, BC = 2R = diameter.
Ê Ê 2p ˆ ˆ
1 fi A1 A22 = 2r 2 Á1 - cos Á ˜ ˜
Now, D = ¥ AB ¥ AC ¥ sin(90∞) Ë Ë n ¯¯
2
Ê 2p ˆ
1 fi A1 A22 = 2r 2 .2 sin 2Á ˜
= ¥ AB ¥ AC Ë n¯
2
Properties of Triangles 381

Ê 2p ˆ 1
+
1
+
1
=
1
= 4r 2 .sin 2 Á ˜ .
Ë n¯ A1 A2 A3 A
Ê 2p ˆ Soln. Let r be the radius of the in-circle and r1, r2 and r3
fi A1 A2 = 2r.sin Á ˜ .
Ë n¯ are the ex-radii of the given triangle
Ê 4p ˆ 1 1 1
Similarly, A1 A3 = 2r.sin Á ˜ Then + +
Ë n¯ A1 A2 A3
Ê 6p ˆ 1
+
1
+
1
& A1 A4 = 2r.sin Á ˜ =
Ë n¯ p r12 p r12 p r12
1 1 1
Given, = + 1 Ê 1 1 1ˆ
A1 A2 A1 A3 A1 A4 = + +
p ÁË r1 r2 r3 ˜¯
1 1 1
fi = + 1 1
Ê 2p ˆ Ê 4p ˆ Ê 6p ˆ = ¥
2r.sin Á ˜ 2r.sin Á ˜ 2r.sin Á ˜ p r
Ë n¯ Ë n¯ Ë n¯
1 1 1 1
fi = + =
Êpˆ Ê 2p ˆ Ê 3p ˆ p r2
sin Á ˜ sin Á ˜ sin Á ˜
Ë n¯ Ë n¯ Ë n¯
1
1 1 1 =
fi - = A
Êpˆ Ê 3p ˆ Ê 2p ˆ
sin Á ˜ sin Á ˜ sin Á ˜ Hence, the result.
Ë n¯ Ë n¯ Ë n¯
Ex-19. The sides of a triangle are in A.P. and the greatest
Ê 3p ˆ Êpˆ and the least angles are q and j, then prove that
sin Á ˜ - sin Á ˜
Ë n¯ Ë n¯
4 (1 + cos q ) (1 - cos j ) = cos q + cos j
1
fi =
Êpˆ Ê 3p ˆ Ê 2p ˆ
sin Á ˜ sin Á ˜ sin Á ˜ Soln. Let a, b, c be the sides of a triangle such that a and
Ë n¯ Ë n¯ Ë n¯
c are the least and the greatest side of DABC
Ê 2p ˆ Êpˆ It is given that a, b, c are in A.P.
2 cos Á ˜ sin Á ˜
Ë n¯ Ë n¯ 1 fi 2b=a+c
fi =
Êpˆ Ê 3p ˆ Ê 2p ˆ
sin Á ˜ sin Á ˜ sin Á ˜ a 2 + b2 - c2
Ë n¯ Ë n¯ Ë n¯ Now, cosq =
2 ab
Ê 2p ˆ Ê 2p ˆ Ê 3p ˆ
fi 2 cos Á ˜ sin Á ˜ = sin Á ˜ a 2 - c2 b2
Ë n¯ Ë n¯ Ë n¯ = +
2 ab 2ab
Ê 4p ˆ Ê 3p ˆ
fi sin Á ˜ = sin Á ˜
Ë n ¯ Ë n¯ (a + c ) (a - c ) b
= +
Ê 4p ˆ Ê 3p ˆ a (a + c ) 2a
fi sin Á ˜ = sin Á p - ˜
Ë n ¯ Ë n¯ (a - c ) b
= +
Ê 4p ˆ Ê 3p ˆ a 2a
fi ÁË ˜ = Á p - ˜¯
n ¯ Ë n 2a - 2c + b
=
Ê 7p ˆ 2a
fi ÁË ˜=p
n ¯ 4a - 4c + a + c
=
n= 7 4a

5a - 3c
Ex-18. If A, A1, A2, A3 are the areas of incircle and the ex- =
4a
circles of a triangle, then prove that
382 Comprehensive Trigonometry with Challenging Problems & Solutions for Jee Main and Advanced

b2 + c2 - a 2 Ex-20. If a, b, g are the lengths of the altitudes of a triangle


and cosj = ABC, then prove that
2bc
1 1 1 1
b2 c2 - a 2 + + = (cot A + cot B + cot C )
= + a 2
b 2
g 2
D
2bc 2bc
Soln. Let AD = a, BE = b and CF = g
b (c - a ) (c + a )
= + Then,
2c (c + a ) c
1 1 1
b (c - a ) D = ¥ a ¥ AD = ¥ b ¥ BE = ¥ c ¥ CF
= + 2 2 2
2c c
2D 2D 2D
b + 2c - 2a fi AD = , BE = , CF =
= a b c
2c
2b + 4c - 4a 2D 2D 2D
= fi a= , b= , g =
4c a b c
a + c + 4c - 4a 1 1 1
= Now, + +
4c a 2
b 2
g2
5c - 3a
= a2 b2 c2
4c = + +
4D 2 4D 2 4D 2
Now, 4 (1 - cos q ) (1 - cos j )
=
(a 2
+ b2 + c2 )
Ê 5a - 3c ˆ Ê 5c - 3a ˆ 4D 2
= 4 Á1 - ˜ Á1 - ˜
Ë 4a ¯ Ë 4c ¯
1
= ¥
(
a 2 + b2 + c2 )
Ê 4a - 5a + 3c ˆ Ê 4c - 5c + 3a ˆ D 4D
= 4Á ˜¯ ÁË ˜¯
Ë 4a 4c 1
= ¥ (cot A + cot B + cot C )
Ê 3c - a ˆ Ê 3a - c ˆ D
= 4Á
Ë 4a ˜¯ ÁË 4c ˜¯
(cot A + cot B + cot C )
=
1 Ê 3c - a ˆ Ê 3a - c ˆ D
= Á ˜Á ˜
4Ë a ¯Ë c ¯ Hence, the result.
1 Ê (3c - a ) (3a - c ) ˆ Ex.-21. If p1, p2, p3 are the altitudes of a triangle from the
= vertices a, b, c and D be the area of the triangle ABC,
4 ÁË ac ˜¯
prove that
9ac - 3c 2 + ac - 3a 2 1 1 1 2ab Ê Cˆ
= + - = ¥ cos 2 Á ˜ .
4ac p1 p2 p3 ( a + b + c ) ¥ D Ë 2¯
10ac - 3c 2 - 3a 2 Soln. Let AD = p1, BE = p2 and CF = p3
=
4ac 1 1 1
Then, D = ¥ a ¥ p1 = ¥ b ¥ p2 = ¥ c ¥ p3
5ac - 3c 2 + 5ac - 3a 2 2 2 2
=
4ac 2D 2D 2D
fi p1 = , p2 = , p3 =
c (5a - 3c ) + a (5c - 3a ) a b c
= 1 1 1
4ac Now, + -
p1 p2 p3
Ê (5c - 3a ) (5a - 3c ) ˆ
=Á + a b c
Ë 4a 4c ˜¯ = + -
2D 2D 2D
= cos q + cos j (a + b - c )
=
Hence, the result. 2D
Properties of Triangles 383

(a + b + c - 2c ) (a + b + c ) abc
= =
2D (a + b + c )
(2s - 2c )
= abc
2D =
(a + b + c )
(s - c)
= Hence, the result.
D
Ex-23. Two circles of radii a and b cut each other at an angle
2ab ¥ s ( s - c ) 1 q, then prove that the length of the common chord
= ¥
D ¥s 2ab 2ab sin q
is .
=
2ab s ( s - c )
¥ (a 2
+ b 2 + 2ab cos q )
D ¥s 2ab
Soln.
2 ab Ê Cˆ
= ¥ cos 2 Á ˜
(a + b + c ) D Ë 2¯ P

Ex-22. Three circles whose radii are a, b, c touch one another q


externally and the tangents at their points of contact
meet in a point, prove that the distance of this point C1 M C2
from either of their points of contact is
1/ 2 T1 T2
Ê abc ˆ
ÁË ˜
a + b + c¯
Soln.
Let – C1 PC2 = q
a 2 + b 2 - (C1C2 )
2

A
Thus, cos (180∞ - q ) =
2ab

fi (C1C2 )2 = a 2 + b2 + 2ab cosq


F O E
fi (C1C2 ) = a 2 + b 2 + 2ab cosq
B D C
1
Area of DC1PC2 = ab sinq
2

Let a, b, c be the centres of three circles whose radii 1


So, area of D C1 PC2 = .C1C2 .PM
are a, b and c, respectively. 2
Clearly, OD = OF = OE 1 1
So, O will be its in-centre. fi C1C2 .PM = ab sin q
2 2
Let OD = OF = OE = r
Let s be the semi-parameter of DABC ab sinq
fi PM =
a+b+b+c+c+a C1C2
Thus, s = = a+b+c
2 ab sin q
fi PM =
Area of a triangle ABC a 2 + b 2 + 2ab cos q
= s (s - a - b) (s - b - c) (s - c - a ) Hence, the length of the common chord
= PQ = 2 PM
= (a + b + c ) abc
2ab sin q
D =
Now, OD = r = a + b 2 + 2ab cos q
2
s
384 Comprehensive Trigonometry with Challenging Problems & Solutions for Jee Main and Advanced

Ex-24. If the sides of a triangle are in A.P. and if the greatest a 2a


angle exceeds the least angle a, then show that Also, =
sin ( 2q + a ) sin q + sin (q + a )
the sides are in the ratio (1 - x ) : x : (1 + x )
sin q + sin (q + a )
1 - cos a fi =2
where x = sin ( 2q + a )
7 - cos a
Ê aˆ Êaˆ
2 sin Á q + ˜ cos Á ˜
Soln. Let the sides be a - d , a , a + d Ë 2¯ Ë 2¯
fi =2
A Ê aˆ Ê aˆ
2 sin Á q + ˜ cos Á q + ˜
Ë 2¯ Ë 2¯
d q
a
+ Êaˆ
a

cos Á ˜
Ë 2¯
q+a fi =2
Ê aˆ
B a-d C cos Á q + ˜
Ë 2¯
Consider d > 0.
Thus, the greatest side is a + d and the smallest Êaˆ
cos Á ˜
side is a – d. Ê aˆ Ë 2¯
fi cos Á q + ˜ =
Ë 2¯ 2
Let – A = q , –C = q + a and
– B = 180∞ - ( 2q + a ) Êaˆ
4 - cos 2 Á ˜
Ê aˆ Ë 2¯
Applying sine rule, we get, fi tan Á q + ˜ = ....(ii)
Ë 2¯ Êaˆ
a-d a a+d cos Á ˜
= = Ë 2¯
sin q sin (180∞ - ( 2q + a )) sin (q + a )
From (i) and (ii), we get,
a-d a a+d
fi = = Êaˆ Êaˆ
sin q sin (2q + a ) sin (q + a ) d
tan Á ˜
Ë 2¯
sin Á ˜
Ë 2¯
= =
2a a 2 Êaˆ Êaˆ
= 4 - cos Á ˜ 4 - cos 2 Á ˜
sin q + sin (q + a ) Ë 2¯ Ë 2¯
a-d a+d Êaˆ
Now, = cos Á ˜
sin q sin (q + a ) Ë 2¯
a-d sin q Êaˆ
fi = sin 2 Á ˜
a + d sin (q + a ) Ë 2¯
=
Êaˆ
2a sin q + sin (q + a ) 4 - cos 2 Á ˜
fi = Ë 2¯
2d sin q - sin (q + a )
1 - cos a
Ê aˆ Êaˆ
2 sin Á q + ˜ cos Á ˜ 2
2a Ë 2¯ Ë 2¯ =
fi = 1 + cos a
2d Ê aˆ Êaˆ 4-
2 cos Á q + ˜ sin Á ˜ 2
Ë 2¯ Ë 2¯
1 - cos a
Ê aˆ = =x
tan Á q + ˜ 7 - cos a
a Ë 2¯
fi = Hence, the required ratio is
d Êaˆ
tan Á ˜
Ë 2¯ = a - d :a:a + d
d d
Êaˆ = 1 - :1:1 +
tan Á ˜ a a
d Ë 2¯
fi = .......(i) = 1 - x :1:1 + x .
a Ê aˆ
tan Á q + ˜
Ë 2¯ Hence, the result.
Properties of Triangles 385

Ex-25. The sides a, b, c of a triangle ABC are the roots A


of x - px + qx - r = 0 , then prove that its area
3 2
w
is
1
4
(
p 4 pq - p3 - 8r . )
O
Soln. Given equation is x - px + qx - r = 0
3 2
w
w
Thus, a + b + c = p, ab + bc + ca = q, abc = r B C
Now, D2 c sin w
fi OB = .......(i)
= s (s - a ) (s - b) ( s - c) sin B

pÊ p ˆÊ p ˆÊ p ˆ Now, from DOBC, we get,


= ÁË - a˜¯ ÁË - b˜¯ ÁË - c˜¯ OB BC
2 2 2 2 =
sin (C - w ) sin (180∞ - C )
p ÈÊ p ˆ
2 2
Ê pˆ
= ÍÁ ˜ - (a + b + c ) Á ˜ OB a
2 ÍÎË 2 ¯ Ë 2¯ fi =
sin (C - w ) sin C
Ê pˆ ˘ a sin (C - w )
+ ( ab + bc + ca ) Á ˜ - abc ˙ OB =
Ë 2¯ ˚ fi .......(ii)
sin C
p ÈÊ p ˆ Ê p ˆ
3 2
Ê pˆ ˘ From (i) and (ii), we get,
= ÍÁ ˜ - Á ˜ p + Á ˜ q - r ˙
2 ÍÎË 2 ¯ Ë 2 ¯ Ë 2¯ ˙˚ c sin w a sin (C - w )
=
sin B sin C
p Ê p3 - 2 p3 + 4 pq - 8r ˆ
= k sin C sin w k sin A sin (C - w )
2 ÁË 8 ˜
¯ =

sin B sin C
p
=
16
(4 pq - p3 - 8r ) sin C sin w sin A sin (C - w )
fi =
sin B sin ( A + B )
D=
1
4
(
p 4 pq - p3 - 8r ) sin C sin w sin (C - w )
fi =
Thus, area of a triangle = D sin A sin B sin A cos B + cos A sin B

=
1
4
(
p 4 pq - p3 - 8r ) fi
sin C sin w
=
sin C cos w - cos C sin w
sin A sin B sin A cos B + cos A sin B
Hence, the result.
fi sin C sin A cos B sin w + sin C cos A sin B sin w
Ex-26. Let O be a point inside a triangle ABC such that
–OAB = –OBC = –OCA = w then prove that = sin A sin B sin C cos w - sin A sin B cos C sin w
(i) cot w = cot A + cot B + cot C Dividing both the sides by sin A sin B sin C sin w
(ii) cosec w = cosec A + cosec B +cosec C
2 2 2 2 we get
Soln. Let – OCB = C - w cot B + cot A = cot w - cot C

and – BOC = 180∞ - w - (C - w ) = 180∞ - C fi cot A + cot B + cot C = cot w

Similarly, – AOB = 180∞ - B (ii) We have


Now from DOAB, we have cot A + cot B + cot C = cot w
OB
=
AB fi (cot A + cot B + + cot C )2 = cot 2 w
sin w sin (180∞ - B )
OB c
fi (cot 2
)
A + cot 2 B + cot 2 C + 2 = cot 2 w
fi = fi cosec 2 A - 1 + cosec 2 B - 1
sin w sin B
386 Comprehensive Trigonometry with Challenging Problems & Solutions for Jee Main and Advanced

+cosec 2C - 1 + 2 = cosec 2w - 1 = k [sin C cos A cos q + sin C sin A sin q

fi cosec 2 A + cosec 2 B + cosec 2C = cosec 2w + sin A cos C cos q - sin A sin C sin q ]

fi cosec 2w = cosec 2 A + cosec 2 B + cosec 2C = k ÈÎcos q (sin A cos C + cos A sin C )˘˚
Hence, the result.
= k cos q sin ( A + C )
Ex-27. ABCD is a trapezium such that AB is parallel to CD
and CB is perpendicular to them. = k cos q sin (p - B )
If – ADB = q , BC = p and CD = q, then = k cos q sin B

prove that AB =
(p 2
)
+ q 2 sin q
.
= ( k sin B ) cos q
p cos q + q sin q
= b cosq
Soln.
Ex-29. If the median of a DABC through a is perpen
D q C
a dicular to AB, prove that tan A + 2 tan B = 0 .
q Soln. Since AD is the median, so BD : DC = 1 : 1
p
A

p - (q + a) a 90° A - 90°
A B

90°+ B
From DBCD, we get, BD = p2 + q2

Let – ABD = – BDC = a B D C

then – DAB = p - (q + a ) . Clearly, –ADC = 90∞ + B .


Now, applying m : n rule, we get,
Now, from DABD, we have
AB BD (1 + 1) cot(90∞ + B) = 1.cot(90∞) - 1.cot( A - 90∞)
=
sin q sin (p - (q + a )) fi -2 tan B = 0 - ( - tan A)
AB BD -2 tan B = tan A
fi = fi
sin q sin (q + a )
fi tan A + 2 tan B = 0
BD sin q
fi AB = Hence, the result.
sin (q + a )
BD 2 sin q Ê Aˆ Ê Bˆ Ê Cˆ 1
AB = Ex-30. In a DABC, prove that sin Á ˜ sin Á ˜ sin Á ˜ £
fi Ë 2¯ Ë 2¯ Ë 2¯ 8
BD sin (q + a )
BD 2 sin q Ê Aˆ Ê Bˆ Ê Cˆ
= Soln. Let u = sin Á ˜ sin Á ˜ sin Á ˜
BD sin q cos a + BD cos q sin a Ë 2¯ Ë 2¯ Ë 2¯

fi AB =
(
p 2 + q 2 sin q ) fi
Ê Ê Aˆ Ê Bˆˆ Ê Cˆ
2u = Á 2 sin Á ˜ sin Á ˜ ˜ sin Á ˜
q sin q + p cos q Ë Ë 2¯ Ë 2 ¯¯ Ë 2¯

Hence, the result. Ê Ê A - Bˆ Ê A + Bˆˆ Ê Cˆ


fi 2u = Á cos Á - cos Á sin Á ˜
Ex-28. In an triangle ABC, if q any angle, then prove Ë Ë 2 ˜¯ Ë 2 ˜¯ ˜¯ Ë 2¯
that b cos q = c cos ( A - q ) + a cos (C + q )
Ê Ê A - Bˆ Ê Cˆˆ Ê Cˆ
= Á cos Á - sin Á ˜ ˜ sin Á ˜
Soln. We have Ë Ë 2 ˜¯ Ë 2 ¯¯ Ë 2¯
c cos ( A - q ) + a cos (C + q )
Ê Cˆ Ê A - Bˆ Ê Cˆ
fi sin 2 Á ˜ - cos Á ˜ .sin Á ˜ + 2u = 0
= k sin C cos ( A - q ) + k sin A cos (C + q ) Ë 2¯ Ë 2 ¯ Ë 2¯

= k ÈÎsin C cos ( A - q ) + sin A cos (C + q )˘˚


Properties of Triangles 387

Ê Cˆ Therefore, from (i), we get,


since sin Á ˜ is real, so D ≥ 0
Ë 2¯ x2 + y 2 + z 2 ≥ 1
Ê A - Bˆ Ê Aˆ Ê Bˆ Ê Cˆ
fi cos 2 Á - 8u ≥ 0 tan 2 Á ˜ + tan 2 Á ˜ + tan 2 Á ˜ ≥ 1
Ë 2 ˜¯ fi
Ë 2¯ Ë 2¯ Ë 2¯
Ê A - Bˆ Ex-32. If the distances of the vertices of a triangle from the
fi cos 2 Á ≥ 8u
Ë 2 ˜¯ points of contact of the incircle with the sides be a,
b, g, then prove that
1 Ê A - Bˆ
fi u £ cos 2 Á
8 Ë 2 ˜¯ r2 =
a bg
where r is in-radius.
a +b +g
1
fi u£ Soln. Let the in-circle touches the side AB at p
8
where AP = a
Ê Aˆ Ê Bˆ ÊCˆ 1
Hence, sin Á ˜ sin Á ˜ sin Á ˜ £ . A
Ë 2¯ Ë 2¯ Ë 2¯ 8
a
Ex-31. In a DABC, prove that
Ê Aˆ Ê Bˆ Ê Cˆ P
tan 2 Á ˜ + tan 2 Á ˜ + tan 2 Á ˜ ≥ 1 r
Ë 2¯ Ë 2¯ Ë 2¯ I

Ê Aˆ Ê Bˆ Ê Cˆ
Soln. Let x = tan Á ˜ , y = tan Á ˜ , z = tan Á ˜ B C
Ë 2¯ Ë 2¯ Ë 2¯
Let I be its in-centre and AI bisects –BAC
To prove, x + y + z ≥ 1
2 2 2
Now, from DIPA,
Now, x + y + z - xy - yz - zx
2 2 2
Ê Aˆ r
tan Á ˜ =
1È Ë 2¯ a
=
Î
( x - y )2 + ( y - z )2 + ( z - x )2 ˘˚ ≥ 0
2 Ê Aˆ
fi a = r cot Á ˜
Ë 2¯
Thus, x + y + z ≥ xy + yz + zx ......(i)
2 2 2

Since in DABC, Ê Bˆ Ê Cˆ
Similarly, b = r cot Á ˜ , g = r cot Á ˜
Ë 2¯ Ë 2¯
A+ B+C =p
In a triangle ABC, we have
A B p C
fi + = - Ê Aˆ Ê Bˆ Ê Cˆ Ê Aˆ Ê Bˆ Ê Cˆ
2 2 2 2 cot Á ˜ + cot Á ˜ + cot Á ˜ = cot Á ˜ cot Á ˜ cot Á ˜
Ë 2¯ Ë 2¯ Ë 2¯ Ë 2¯ Ë 2¯ Ë 2¯
Ê A Bˆ Êp Cˆ
fi tan Á + ˜ = tan Á - ˜ a b g a b g
Ë 2 2¯ Ë 2 2¯ fi + + = . .
r r r r r r
Ê A Bˆ Ê Cˆ
fi tan Á + ˜ = cot Á ˜ a + b + g abg
Ë 2 2¯ Ë 2¯ fi = 3
r r
Ê Aˆ Ê Bˆ abg
tan Á ˜ + tan Á ˜
Ë 2¯ Ë 2¯ 1 fi r2 =
fi = a +b +g
Ê Aˆ Ê Bˆ Ê Cˆ
1 - tan Á ˜ tan Á ˜ tan Á ˜ Hence, the result.
Ë 2¯ Ë 2¯ Ë 2¯
Ê Aˆ Ê Cˆ Ê Bˆ Ê Cˆ Ê Aˆ Ê Bˆ Ex-33. If t1 , t2 and t3 are the lengths of the tangents drawn
fi tan Á ˜ tan Á ˜ + tan Á ˜ tan Á ˜ + tan Á ˜ tan Á ˜ = 1 from the centre of the ex-circle to the circum-circle
Ë 2¯ Ë 2¯ Ë 2¯ Ë 2¯ Ë 2¯ Ë 2¯
of DABC, prove that
Ê Aˆ Ê Bˆ Ê Bˆ Ê Cˆ Ê Cˆ Ê Aˆ abc
fi tan Á ˜ tan Á ˜ + tan Á ˜ tan Á ˜ + tan Á ˜ tan Á ˜ = 1 1 1 1
Ë 2¯ Ë 2¯ Ë 2¯ Ë 2¯ Ë 2¯ Ë 2¯ + + =
t12 t22 t32 a+b+c
fi xy + yz + zx = 1
388 Comprehensive Trigonometry with Challenging Problems & Solutions for Jee Main and Advanced

Soln. Ex-34. In DABC


A Ê Aˆ Ê Bˆ
 cot 2 ÁË ˜¯ cot 2 ÁË ˜¯
2 2
P A
Ê ˆ
O
’ cot 2 ÁË ˜¯
2
R
B C
Ê Aˆ Ê Bˆ
 cot 2 ÁË ˜¯ cot 2 ÁË ˜¯
2 2
Soln. We have
Ê Aˆ
I1
’ cot 2 ÁË ˜¯
2
Ê 2 Ê Aˆ 2 Ê Bˆ 2 Ê Bˆ 2 ÊCˆ
ÁË cot ÁË 2 ˜¯ cot ÁË 2 ˜¯ + cot ÁË 2 ˜¯ cot ÁË 2 ˜¯

Let O and I1 be respectively the centres of the circum- ÊCˆ Ê Aˆ ˆ


+ cot 2 Á ˜ cot 2 Á ˜ ˜
circle and the ex-circle touching the line BC Ë 2¯ Ë 2 ¯¯
=
Ê Aˆ Ê Bˆ ÊCˆ
Clearly, OI1 = R + 2 Rr1
2
cot 2 Á ˜ cot 2 Á ˜ cot 2 Á ˜
Ë 2¯ Ë 2¯ Ë 2¯
fi R 2 + t12 = R 2 + 2 Rr1 1 1 1
= + +
Ê Aˆ Ê Bˆ Ê Cˆ
fi (R 2
) (
+ t12 = R 2 + 2 Rr1 ) cot 2 Á ˜ cot 2 Á ˜ cot 2 Á ˜
Ë 2¯ Ë 2¯ Ë 2¯

fi t12 = 2 Rr1 2 Ê Aˆ 2 Ê Bˆ 2 Ê Cˆ
= tan Á ˜ + tan Á ˜ + tan Á ˜
Ë 2¯ Ë 2¯ Ë 2¯
1 1
fi =
t12 2 Rr1 ≥1

1 1 1 1 Hence, the minimum value is 1.


Similarly, = , 2 =
t22 2 Rr2 t3 2 Rr3 Ex-35. A triangle has base 10 cm long and the base angles
are 50∞ and 70∞ . If the perimeter of the triangle
1 1 1
Now, + + is x + y cos ( z ∞) where z Œ (0, 90∞) , then
t12 t22 t32
( x + y + z ).
=
1
+
1
+
1 Soln. Let BC = a = 10
2 Rr1 2 Rr2 2 Rr3 A

1 Ê 1 1 1ˆ
= + +
2 R ÁË r1 r2 r3 ˜¯ c b

1 1 50° 70°
= .
2R r B a = 10 C

1 s From sine rule, we have


= .
2R D a b c
= =
1 (a + b + c ) sin A sin B sin C
= .
abc 2D a b c
2. fi = =
4D sin(60∞) sin (50∞) sin (70∞)
(a + b + c ) =
a+b+c
=
abc sin(60∞) + sin (50∞) + sin (70∞)
Hence, the result. a+b+c
=
3
+ 2 sin (60∞) cos(10∞)
2
Properties of Triangles 389

a+b+c Ê Aˆ Ê Bˆ Ê Cˆ
= = 4 sin Á ˜ .sin Á ˜ .sin Á ˜
3 Ë 2¯ Ë 2¯ Ë 2¯
+ 3 cos(10∞)
2 1 Ê Aˆ Ê Bˆ Ê Cˆ
a a+b+c = ¥ 4 R sin Á ˜ .sin Á ˜ .sin Á ˜
fi = R Ë 2¯ Ë 2¯ Ë 2¯
sin(60∞) 3
+ 3 cos(10∞) r
2 =
10 a+b+c R
fi =
3 3 Ex-37. In a right angled triangle ABC, the hypotenuse BC
+ 3 cos(10∞)
2 2 of length ‘a’ is divided into n equal parts (n an odd
10 a+b+c positive integers). Let a be the acute angle subtending
fi = from a by that segment which contains the mid-point
1 1 + 2 cos(10∞)
of the hypotenuse of the triangle, prove that
fi a + b + c = 10 + 20 cos(10∞) 4n h
tan a = 2 [Roorkee – 1983]
fi a + b + c = x + y cos ( z ∞) (
n -1 a )
fi x = 10, y = 20, z = 10
Soln.
Hence, the value of ( x + y + z )
A
= 10 + 20 + 10 = 40 .
Ex-36 In a DABC a
q
a cos A + b cos B + c cos C h
a+b+c
a cos A + b cos B + c cos C
Soln. We have .
a+b+c
B a/n a/n D E M F a/n a/n
1 Ê 2 sin A cos A + 2 sin B cos B + 2 sin C cos C ˆ x
=
2 ÁË sin A + sin B + sin C ˜¯
a
1 Ê sin 2 A + sin 2 B + sin 2C ˆ Each part of the base BC be
= Á n
2 Ë sin A + sin B + sin C ˜¯
Let AD = h is the altitude.
1 Ê 4sin A sin B sin C ˆ The nth part EF which contains the middle point
= Á
2 Ë sin A + sin B + sin C ˜¯ M subtends an angle a at A.
Let – DAE = q and – EAF = a
Ê 2sin A sin B sin C ˆ
=Á \ – DAF = q + a
Ë sin A + sin B + sin C ˜¯
Also, let DM = x
Ê ˆ
a a
Á 2 sin A sin B sin C ˜ Then DE = x - and DF = x +
=Á ˜ 2n 2n
Á 4 cos Ê A ˆ cos Ê B ˆ cos Ê C ˆ ˜
ÁË ÁË ˜¯ ÁË ˜¯ ÁË ˜¯ ˜¯ In DADF,
2 2 2
Ê Ê Aˆ Ê Aˆ Ê Bˆ Ê Bˆ ˆ a
x-
Á 16 sin ÁË 2 ˜¯ cos ÁË 2 ˜¯ .sin ÁË 2 ˜¯ cos ÁË 2 ˜¯ ˜ tanq =
DE
= 2n = 2nx - a
Á ˜ AD h 2nh
Á Ê Cˆ Ê Cˆˆ ˜
Á sin Á ˜ cos Á ˜ ˜ ˜
Ë 2¯ Ë 2 ¯¯
˜ 2nh tanq = (2nx - a ) .........(i)

Á Ê Aˆ Ê Bˆ Ê Cˆ ˜ Also, in DADF
Á 4 cos Á ˜ cos Á ˜ cos Á ˜ ˜
Ë 2¯ Ë 2¯ Ë 2¯ a
Á ˜ x+
Á ˜
tan (a + q ) = 2n = 2nx + a ....(ii)
Á ˜ h 2nh
Ë ¯
390 Comprehensive Trigonometry with Challenging Problems & Solutions for Jee Main and Advanced

Elimination q from (i) and (ii), we get, s


=
2nh tan a + 2nx - a s-a
2s
Ê (2nx - a ) tan a ˆ =
= 2nx + a - ( 2nx + a ) Á ˜¯ 2 s - 2a
Ë 2nx
(a + b + c )
=
fi 2nh tan a = 2a -
( 4n x
2 2
-a 2
) tan a ( a + b + c ) - 2a
2nh (a + b + c )
=


Ê
Á 2nh +
(
4n 2 x 2 - a 2 )ˆ˜ tan a = 2a (b + c - a )
ÁË 2nh ˜¯ (a + 3a ) = 4a = 2.
=
(3a - a ) 2a
fi Á
(
Ê 4n 2 h 2 + 4n 2 x 2 - a 2 )ˆ˜ tan a = 2a a 2 - b2 sin ( A - B )
ÁË 2nh ˜¯ Ex-39. If in a triangle, = .
a +b2 2
sin ( A + B )
4anh Prove that it is either a right angled triangle or an
fi tan a =
(
4n 2 h 2 + 4n 2 x 2 - a 2 ) isosceles triangle. [Roorkee – 1987]
a 2 - b2 sin ( A - B )
4anh Soln. We have = .
fi tan a = sin ( A + B )
( ) a +b2 2
4n 2 h 2 + x 2 - a 2
4anh sin 2 A - sin 2 B sin ( A - B )
fi tan a = fi =
Ê a2 ˆ sin A + sin B
2 2
sin ( A + B )
4n 2 Á ˜ - a 2
Ë 4¯ sin ( A + B ) sin ( A - B ) sin ( A - B )
fi =
Ê aˆ sin A + sin B
2 2
sin ( A + B )
ÁË∵ AM = BM = ˜¯
2
fi sin ( A - B ) = 0,
4a nh
fi tan a =
(n - 1) a
2 2 Ê sin ( A + B )
and Á 2 -
1 ˆ
˜ =0
4nh Ë sin A + sin B sin ( A + B ) ¯
2

fi tan a =
(n - 1) a
2
Now, sin ( A - B ) = 0
Note. No questions asked in 1984, 1985. fi A=B
Ex-38. If a, b, c are the sides of a triangle ABC and fi D is isosceles
Ê Bˆ ÊCˆ
3a = b + c , then prove that cot Á ˜ cot Á ˜ = 2. Ê sin ( A + B ) 1 ˆ
Ë 2¯ Ë 2¯ -
and Á 2 ˜ =0
Ë sin A + sin 2 B sin ( A + B ) ¯
[Roorkee – 1986]

Ê Bˆ Ê Cˆ fi sin 2 A + sin 2 B = sin 2 ( A + B )


Soln. We have cot Á ˜ cot Á ˜
Ë 2¯ Ë 2¯ sin 2 A + sin 2 B = sin 2 A cos 2 B

s (s - b) s (s - c) + cos 2 A sin 2 B + 2 sin A sin B cos A cos B
= ¥
(s - a) (s - c) (s - a ) (s - b)
fi -2 sin 2 A sin 2 B
s (s - b) s (s - c) +2 sin A sin B cos A cos B = 0
=
( ) (s - c) (s - a ) (s - b)
s - a
fi sin A sin B = cos A cos B
s 2
fi tan A tan B = 1
=
( s - a )2
Properties of Triangles 391

fi tan A = cot B Ex-41. If x, y, z are the perpendicular distances of the vertices


of a triangle ABC from the opposite sides and D be
Êp ˆ the area of the triangle, then prove that
fi tan A = tan Á - B˜
Ë2 ¯
1 1 1 1
+ + = (cot A + cot B + cot C ) .
p x 2
y 2
z 2
D
fi A= -B
2 [Roorkee Main – 1990]
p Soln. From DABC,
fi A+ B =
2 1 1 1
D = ax= ay = az
p p 2 2 2
fi C = p - ( A + B) = p - = 1 a 1 b 1 c
2 2 fi = , = , =
x 2D y 2D z 2D
Thus, the triangle DABC is right angled.
Ex-40. In any triangle ABC, show that Now, L.H.S
1 1 1
Ê Ê Aˆ Ê Bˆˆ Ê 2 Ê Bˆ 2 Ê Aˆ ˆ = 2+ 2+ 2
ÁË cot ÁË 2 ˜¯ + cot ÁË 2 ˜¯ ˜¯ ÁË a sin ÁË 2 ˜¯ + b sin ÁË 2 ˜¯ ˜¯ x y z

a 2 + b2 + c2
Ê Cˆ =
= c cot Á ˜ . [Roorkee – 1988] 4 D2
Ë 2¯
Soln. We have and R.H.S
Ê Ê Aˆ Ê Bˆˆ Ê 2 Ê Bˆ 2 Ê Aˆ ˆ =
1
(cot A + cot B + cot C )
ÁË cot ÁË 2 ˜¯ + cot ÁË 2 ˜¯ ˜¯ ÁË a sin ÁË 2 ˜¯ + b sin ÁË 2 ˜¯ ˜¯ D
1 Ê b2 + c2 - a 2 a 2 + c2 - b2 a 2 + b2 - c2 ˆ
Ê s (s - a) s (s - b) ˆ = + +
=Á + Dk ÁË 2abc 2bac 2abc ˜
¯
Ë (s - b) (s - c) ( s - a ) ( s - c ) ˜¯
1 Ê a 2 + b2 + c2 ˆ
Ê (s - a) (s - c) (s - b) (s - c)ˆ =
¥ Á a. + b. ˜¯ D k ÁË 2abc ˜
¯
Ë ac bc
1 Ê a 2 + b2 + c2 ˆ
s È
Í
( s - a ) ( s - b) ˘˙ = .
D ÁË 2.2 D ˜
¯
= +
( s - c ) ÍÎ ( s - b) ( s - a ) ˙˚
Ê 1 1 ˆ
Ê (s - a ) (s - c) (s - b) (s - c)ˆ ÁË∵ D = ab sin C = ( abc k )˜¯
2 2
¥Á + ˜¯
Ë c c
Ê a 2 + b2 + c2 ˆ
=Á ˜
s Ê s-a+s-b ˆ Ë 4 D2 ¯
= Á ˜
( s - c ) ÁË ( s - a ) ( s - b) ˜¯ Hence, the result.
s-c Ex-42. The two adjacent sides of a cyclic quadrilateral are
¥
c
((s - a ) + (s - b)) 2 and 5 and the angle between them is 60°. If the
area of the quadrilateral is 4 3
Ê c s ˆ s-c two sides. [Roorkee Main – 1991]
=Á ˜ ¥ ( 2 s - ( a + b )) Soln. Suppose AC = 2, AB = 5, BC = x, CD = y
ÁË ( s - c ) ( s - a ) ( s - b) ˜¯ c
and – BAD = 60∞
Ê c s ˆ
=Á ˜ ¥ (s - c) 1
ÁË ( s - c ) ( s - a ) ( s - b) ˜¯ Area of D ABC = .5.2.sin(60∞)
2
s (s - c) Ê Cˆ 5 3
= c = c cot Á ˜ . =
(s - a ) (s - b) Ë 2¯ 2
Note. No questions asked in 1989.
392 Comprehensive Trigonometry with Challenging Problems & Solutions for Jee Main and Advanced

From (i), we get,


x C
D
x 2 + y 2 = 13
120° fi x = 3, y = 2
y
2 Note. No questions asked in 1992.
60° Ex-43. In a triangle ABC, R is the circum-radius and
B 8R 2 = a 2 + b 2 + c 2 . The the triangle ABC is
5
A
(a) Acute angled
Also, from DABC, (b) Right angled
(c) Obtuse angled
25 + 4 - BD 2 (d) None of these.
cos(60∞) =
2.5.2 Soln. As we know that
29 - BD 2 a b c
= = = = 2R
20 sin A sin B sin C
Now we have,
29 - BD 2 1
fi = 8R 2 = a 2 + b2 + c 2
20 2
fi BD 2 = 19 fi (
8R 2 = 4 R 2 sin 2 A + sin 2 B + sin 2 C )
fi BD = 19
fi (sin 2
)
A + sin 2 B + sin 2 C = 2
Since A, B, C, D are concyclic, so
– BCD = 180∞ - 60∞ = 120∞
fi (cos 2
)
A - sin 2 C + cos 2 B = 0

Then from DBCD, fi cos ( A + C ) cos ( A - C ) + cos 2 B = 0

( 19 )
2
x2 + y 2 - fi cos (p - B ) cos ( A - C ) + cos 2 B = 0
cos(120∞) =
2 xy
fi - cos B cos ( A - C ) + cos 2 B = 0
( 19 )
2
x2 + y 2 -
fi =-
1 fi cos B (cos ( A - C ) - cos B ) = 0
2 xy 2
fi cos B .2 cos A cos C = 0
( 19 )
2
x +y -
2 2

fi = -1 fi cos A = 0, cos B = 0, cos C = 0


xy
p
fi A= =B=C
fi x 2 + y 2 + xy = 19 .......(i) 2
Again, area of DBCD Thus, the triangle is right angled.
Ex-44. If the sides a, b, c of a triangle are in A.P., then
1 3 3 xy
= .x. y. = . Ê Aˆ Ê Cˆ
2 2 4 tan Á ˜ + tan Á ˜
Ë 2¯ Ë 2¯
Thus, area of quad. ABCD = 4 3
Ê Bˆ
in terms of cot Á ˜ . [ Roorkee Main – 1993]
5 3 3 xy Ë 2¯
fi + =4 3
2 4
Ê Aˆ Ê Cˆ
Soln. We have tan Á ˜ + tan Á ˜
5 xy Ë 2¯ Ë 2¯
fi + =4
2 4
(s - b) (s - c) (s - a ) (s - b)
xy 5 3 = +
fi = 4- = s (s - a) s (s - c)
4 2 2
fi xy = 6 (s - b) Ê (s - c) (s - a ) ˆ
= Á +
s Ë (s - a) ( s - c ) ˜¯
Properties of Triangles 393

A
(s - b) Ê s-c+s-a ˆ
= Á ˜
s ÁË ( s - a ) ( s - c ) ˜¯

(s - b) Ê 2s - c - a ˆ
= Á ˜
s ÁË ( s - a ) ( s - c ) ˜¯
P Q
(s - b)
= ¥ (a + b + c - c - a ) R
s (s - a) (s - c) 30°
B D E C
(s - b)
= b¥ Let – BOD = q
s (s - a) (s - c)

2s (s - b) OB 2 + OD 2 - BD 2
= ¥ Then cos q =
3 s (s - a) (s - c) 2. OB . OD
1+1- 3 1
2 s (s - b) fi cos q = =-
= ¥ 2.1.1 2
3 (s - a) (s - c)
fi q = 120∞
2 Ê Bˆ
= ¥ cot Á ˜ Therefore, –AOD = –AOB + –BOD
3 Ë 2¯
Hence, the result. = 60∞ + 120∞ = 180∞
Note. No questions asked in 1994. fi AOD is a straight line
3 3
Ex-45. A cyclic quadrilateral ABCD of area is fi AD = diameter = 2
4 2p
If – BOC = j , then – COD = -j
inscribed in a unit circle. If one of its sides 3
AB = 1 and the diagonal BD = 3 . Find the length Area of the cyclic parallelogram ABCD

of the other sides. [Roorkee Main – 1995] = ar of ( DAOB + DBOC + DBOD )


Soln.
3 3 1
F fi = .1.1.sin(60∞)
4 2
A
1 1 Ê 2p ˆ
+ .1.1.sin j + .1.1.sin Á - j˜
2 2 Ë 3 ¯

I 1Ê 3 Ê 2p ˆˆ 3 3
O E fi Á + sin j + sin Á - j˜ ˜ =
2Ë 2 Ë 3 ¯¯ 4
1Ê Ê 2p ˆˆ 3 3 3
B
C fi Á sin j + sin Á - j˜ ˜ = -
2Ë Ë 3 ¯¯ 4 4
D 1Ê Ê 2p ˆˆ 3
fi Á sin j + sin Á - j˜ ˜ =
2Ë Ë 3 ¯¯ 2
In DAOB,
Ê Ê 2p ˆˆ
OA = OB = 1 fi ÁË sin j + sin ÁË 3 - j ˜¯ ˜¯ = 3
and AB = 1 (given)
Êpˆ Ê pˆ
Thus, –OAB = –OBA = –AOB = 60∞ fi 2 sin Á ˜ cos Á j - ˜ = 3
Ë 3¯ Ë 3¯
In DBOD, OA = 1, OB = 1, OB = 1 3 Ê pˆ
fi 2¥ ¥ cos Á j - ˜ = 3
and BD = 3 2 Ë 3¯
394 Comprehensive Trigonometry with Challenging Problems & Solutions for Jee Main and Advanced

Ê pˆ 3
fi cos Á j - ˜ = 1 sin q
Ë 3¯ 2
fi = 3
Ê pˆ Ê 3 + 1ˆ
fi ÁË j - ˜¯ = 0 Á 2 2 ˜ sin(45∞ - q )
3 Ë ¯

fi j=
p fi 2 sin q = ( )
3 + 1 sin ( 45∞ - q )
3
2 sin q = ( 3 + 1)
1
Thus – BOC = 60∞ fi (cos q - sin q )
2
and – COD = 120∞ - 60∞ = 60∞
BC OB
fi 2 sin q = ( ) 3 + 1 (cos q - sin q )
In =
sin 60∞ sin 60∞ fi (3 + 3 ) sin q = ( 3 + 1) cosq
sin q ( 3 + 1)
BC = OB = 1
1
fi – C = 60∞ and – A = 75∞ fi = =
cos q (3 + 3 ) 3
Similarly, we can prove that CD = 1
Hence, AB = 1, BC = 1, CD = 1 and AD = 2. 1
fi tanq =
Ex-46. In a triangle ABC, –C = 60° and –A = 75°. If D is a 3
point on AC such that the area of the triangle BAD p
fi q=
is 3 times the area of the triangle BCD 6
angle –ABD. [Roorkee Main – 1996] p
Soln. Hence, – ABD = .
6
B
4
Ex-47 If in a triangle ABC, a = 6, b = 3 and cos ( A - B ) = ,
5
q
a c
4
Soln. Given cos ( A - B ) =
60° 75° 5
C D A
Ê A - Bˆ 4
fi 2 cos 2 Á -1 =
Ë 2 ˜¯ 5
Here, – B = 180∞ - (60∞ + 75∞) = 45∞
Ê A - Bˆ 4
Let – ABD = q fi 2 cos 2 Á ˜ =1+
Ë 2 ¯ 5
ar ( D BAD )
It is given that, = 3 Ê A - Bˆ 9
ar ( D BCD ) fi cos 2 Á =
Ë 2 ˜¯ 10
1
cx sin q Ê A - Bˆ 3
fi cos Á =
fi 2 = 3 Ë 2 ˜¯ 10
1
ax sin(45∞ - q ) Ê A - Bˆ 1
2 fi tan Á =
Ë 2 ˜¯ 3
c sin q
fi = 3 Ê a - bˆ Ê Cˆ 1
a sin (45∞ - q ) fi ÁË ˜¯ cot ÁË ˜¯ =
a+b 2 3
sin C sin q
fi = 3 Ê 6 - 3ˆ Ê Cˆ 1
sin A sin (45∞ - q ) fi ÁË ˜¯ cot ÁË ˜¯ =
6+3 2 3
sin 60∞ sin q
fi = 3 1 Ê Cˆ 1
sin 75∞ sin ( 45∞ - q ) fi cot Á ˜ =
3 Ë 2¯ 3
Properties of Triangles 395

Ê Cˆ 3 16 + a 2 - 6
fi cot Á ˜ = 1 fi =
Ë 2¯ 2 2 .4.a
Ê Cˆ Êpˆ a 2 + 10 3
fi cot Á ˜ = cot Á ˜ fi =
Ë 2¯ Ë 4¯ 8a 2
p fi a 2 - 4 3 a + 10 = 0
fi C=
(a - 2 3 ) = 12 - 10 = 2
2 2
Thus, ar(DABC) fi
1
= ab sin C fi (a - 2 3 ) = ± 2
2
fi a=2 3± 2
1
= ¥ 6 ¥ 3 ¥ sin(90∞) A
2
= 9 sq units.
b
Ex-48. Two sides of a triangle are of lengths 6 and 4 and c
the angle opposite to smaller side is 30°. How many
such triangles are possible? 30°
Find the length of their third side and area.
B 2 3- 2 C1 D C2
[Roorkee Main – 1998]
Soln. 2 3+ 2
A
Now, ar ( D ABC1 )

c b =
1
2
( )
¥ 4 ¥ 2 3 - 2 ¥ sin(30∞)

B
30°
a C
(
= 2 3 - 2 sq units)
We have and ar ( D ABC2 )
b c
=
sin B sin C
=
1
2
( )
¥ 4 ¥ 2 3 + 2 ¥ sin(30∞)


6
=
4
sin(30∞) sin C
( )
= 2 3 + 2 sq. units.
Ex-49. The radii r1, r2, r3 of escribed circles of a triangle
41 2 ABC are in the harmonic progression. If its area is
fi sin C = ¥ = <1
6 2 6
C may be acute or obtuse lengths of its sides. [Roorkee Main – 1999]
Soln. Given 2 s = 24
Also, we observe that b < c
fi s = 12
fiB<C
i.e., if C is obtuse, then B should be acute Also, r1 , r2 , r3 ΠH .P
fi It is possible. D D D
fi , , ΠH .P
s-a s-b s-c
Thus, two triangles are possible in this case.
D D D
Now, applying cosine rule, fi , , ΠH .P
s-a s-b s-c
c2 + a 2 - b2 1 1 1
cos B = fi , , ΠH .P
2 ac s-a s-b s-c
16 + a 2 - 6
fi cos(30∞) = fi ( s - a ) , ( s - b) , ( s - c ) Œ A.P
2 .4.a
fi a, b, c ΠA.P
396 Comprehensive Trigonometry with Challenging Problems & Solutions for Jee Main and Advanced

fi 2b=a+c and abc = 40


fi 3b = a + b + c = 24 m cos A cos B cos C
Now, = + +
n a b c
fi b=8
b2 + c2 - a 2 c2 + a 2 - b2 a 2 + b2 - c2
= + +
Again, r1 , r2 , r3 ΠH .P 2abc 2abc 2abc
1 1 1
, , ΠA.P a 2 + b2 + c2
fi =
r1 r2 r3 2abc


2 1 1
= + (a + b + c )2 - 2 (ab + bc + ca )
r2 r1 r3 =
2abc
2r1r3
fi r2 = (11) 2
- 2.38
r1 + r3 =
2.40
fi r1r2 + r2 r3 = 2r1r3 121 - 76
=
fi r1r2 + r1r3 + r2 r3 = 2r1r3 + r1r3 = 3r1r3 80

3r1r3 = s 2 = 144 45 9
fi = =
80 16
144
fi r1r3 = = 48 Thus, m = 9 and n = 16
3
Hence, the value of m + n
D D
fi ¥ = 48 = 9 + 16 = 25.
(s - a) (s - c)
24 24 LEVEL I
fi ¥ = 48
(12 - a ) (12 - c ) (QUESTIONS BASED ON FUNDAMENTALS)
1. In any triangle ABC, prove that
fi (12 - a ) (12 - c ) = 12
fi 144 - 12 ( a + c ) + ac = 12
(b 2
) ( ) ( )
- c 2 cot A + c 2 - a 2 cot B + a 2 - b 2 cot C = 0

fi 144 - 12.16 + c (16 - c ) = 12 2. In a tringle DABC, prove that


a sin ( B - C ) + b sin (C - A) +c sin ( A - B ) = 0
fi 16c - c - 64 = 0
2

3. In a tringle DABC, prove that


fi c 2 - 16c + 64 = 0
a 2 sin ( B - C ) b 2 sin (C - A) c 2 sin (C - A)
fi ( c - 8) 2
=0
sin A
+
sin B
+
sin C
=0

fi c=8 4. In a triangle ABC, prove that for any angle q,


So, the lengths of the sides are 8 cm, 8 cm, 8 cm. b cos ( A - q ) + c cos ( B + q ) = c cos q
Note. No questions asked in 2000, 2001.
Ex-50. In a triangle ABC, let the sides a, b, c are the roots 2 Ê Cˆ
5. If cos A + cos B = 4 sin Á ˜ , prove that the sides, a,
Ë 2¯
of x - 11x + 38 x - 40 = 0 . If the value of
3 2

cos A cos B cos C m c, b of the triangle ABC are in A.P.


+ + = , where m and
a b c n sin A sin ( A - B )
6. If in a DABC = , then prove that,
n (m + n). sin C sin ( B - C )

Soln. Given equation is x - 11x + 38 x - 40 = 0 a 2 , b 2 , c 2 are in A.P.


3 2

It is also given that a, b and c are its roots 7. In a triangle ABC, prove that,
Thus, a + b + c = 11, 2 (bc cos A + ca cos B + ab cos C ) = a 2 + b 2 + c 2
ab + bc + ca = 38
Properties of Triangles 397

8. In a tringle DABC, prove that 19. In any triangle ABC, prove that,
Cˆ 2 Ê Cˆ 4 D (cot A + cot B + cot C ) = a 2 + b 2 + c 2
(a - b)2 cos2 ÊÁË ˜ + (a + b ) sin ÁË ˜¯ = c
2 2
2¯ 2
20. In any triangle ABC, prove that,
9. With usual notation, if in a triangle ABC,
Ê 2abc ˆ Ê Aˆ Ê Bˆ Ê Cˆ
b+c c+a a+b ÁË ˜¯ .cos ÁË ˜¯ cos ÁË ˜¯ cos ÁË ˜¯ = D
= = , then prove that, a+b+c 2 2 2
11 12 13
21. If in a triangle ABC, a = 6, b = 3 and
cos A cos B cos C 4
= = cos ( A - B ) =
7 19 25 5
10. Let a, b and c be the sides of a DABC. If a , b & c 22. If in a triangle ABC, –A = 30∞ and the area of the
2 2 2

are the roots of the equation x - Px + Qx - R = 0 ,


3 2
a2 3
triangle is , then prove that either
where p, q and R 4
cos A cos B cos C B = 4 C or C = 4 B
+ + , in terms of P, Q and R.
a b c 23. In any triangle DABC, prove that,
11. In a triangle ABC, prove that, R r (sin A + sin B + sin C ) = D .
b 2 sin 2C + c 2 sin 2 B = 2bc sin A 24. In any triangle DABC, prove that,
a cos B cos C + b cos C cos A
sin B c - a cos B
12. In a tringle DABC, prove that = D
sin C b - a cos C + c cos A cos B =
R
13. In a DABC, prove that,
25. In any triangle DABC, prove that,
È Ê Cˆ Ê Aˆ ˘
2 Ía sin 2 Á ˜ + c sin 2 Á ˜ ˙ = c + a - b 1 1 1 1
Î Ë 2¯ Ë 2¯˚ + + =
bc ca ab 2 R r
14. In a tringle DABC, prove that
26. In any triangle DABC, prove that,
2ÊAˆ Ê Bˆ Ê Cˆ
cos Á ˜ cos 2 Á ˜ cos 2 Á ˜ Ê Aˆ Ê Bˆ Ê Cˆ r
Ë 2¯ Ë 2¯ Ë 2¯ s2 cos 2 Á ˜ + cos 2 Á ˜ + cos 2 Á ˜ = 2 +
+ + = Ë 2¯ Ë 2¯ Ë 2¯ 2R
a b c abc
27. In any triangle DABC, prove that,
15. In a DABC, prove that
a cot A + b cot B + c cot C = 2 ( R + r )
Ê 2 Ê Aˆ 2 Ê Bˆ 2 Ê Cˆˆ
ÁË bc cos ÁË 2 ˜¯ + ca cos ÁË 2 ˜¯ + ab cos ÁË 2 ˜¯ ˜¯ 28. In any triangle DABC, prove that,
r1 + r2 - r3 + r = 4 R cos C
1
= ( a + b + c )2 29. In any triangle DABC, prove that,
4
16. In a DABC, prove that, 1 1 1 1 a 2 + b2 + c2
+ + + =
r12 r2 2 r32 r2 D2
A B
(b - c ) cot ÊÁË ˆ˜¯ + (c - a ) cot ÊÁË ˆ˜¯ 30. In any triangle DABC, prove that, prove that,
2 2
Ê Cˆ
+ ( a - b ) cot Á ˜ = 0
2
Ê1 1 1 1ˆ 4Ê 1 1 1ˆ
Ë 2¯ ÁË r + r + r + r ˜¯ = r ÁË r + r + r ˜¯
1 2 3 1 2 3
17. If the sides a, b, c
31. In any triangle DABC, prove that,
Ê Aˆ Ê Cˆ Ê Bˆ
the value of tan Á ˜ + tan Á ˜ in terms of cot Á ˜ bc - r2 r3 ca - r1r3 ab - r1r2
Ë 2¯ Ë 2¯ Ë 2¯ = =
r1 r2 r3
18. If D is mid-point of CA in triangle ABC and D is the
area of triangle, then prove that 32. Prove that the radii of the three escribed circles of a
4D triangle are the roots of
tan (– ADB ) = 2 x3 - ( r + 4 R ) x 2 + s 2 x - r s 2 = 0
a - c2
398 Comprehensive Trigonometry with Challenging Problems & Solutions for Jee Main and Advanced

33. If a , b & g are the respective altitudes of a triangle 45. In usual notation, if r1 = r2 + r3 + r , then prove that the
ABC, prove that triangle is right angled.
1 1 1 cot A + cot B + cot C 46. If A, B, C are the angles of a triangle, then prove that
+ + =
a 2
b 2
g 2
D r
cos A + cos B + cos C = 1 + , where r = in-radius and
R
34. If a , b & g are the respective altitudes of a triangle
R = circum - radius.
ABC, prove that 47. In triangle ABC, if 8R = a + b + c , prove that the
2 2 2 2

1 1 1 Ê 2ab Ê Cˆˆ triangle is right angled.


+ - =Á ¥ cos 2 Á ˜ ˜
a b g Ë (a + b + c ) D Ë 2 ¯¯ 48. Let A1 , A2 , A3 ,........, An be the vertices of an n-sided
1 1 1
35. If in a triangle DABC, a + b + c = 8R , then prove
2 2 2 2 regular polygon such that = + , then
A1 A2 A1 A3 A1 A4
that the triangle ABC is right angled triangle.
prove that the value of n is 7.
36. The sides of a triangle are x + x + 1 , 2x + 1 and
2
2
49. If A, A1 , A2 & A3 are the area of the inscribed and
x – 1, prove that the greatest angle is 120°. escribed circles of a triangle, then prove that
37. The sides of a triangle are three consecutive natural 1 1 1 1
numbers and its largest angle is twice the smallest one. = + + .
Determine the side of the triangle. A A1 A2 A3
38 ABC is a triangle. Its area is 12 sq. cm and base is 6 50. The sides of a quadrilateral are 3, 4, 5 and 6 cms. The
cm. The difference of the angles is 60°. Prove that the sum of a pair of opposite angles is 120°. prove that
angle A opposite to the base is given by 8 sin A – 6 the area of the quadrilateral is 3 30 sq. cm.
cosA = 3. 51. In triangle ABC, prove that,
a
39. In any triangle ABC, if cosq = , r 2 + r12 + r22 + r32 = 16 R 2 - a 2 - b 2 - c 2
b+c
52. In triangle ABC, prove that,
b c
cosj = , cosy = , where Ê Aˆ Ê Bˆ Ê Cˆ
a+c a+b IA.IB.IC = abc tan Á ˜ .tan Á ˜ .tan Á ˜
Ë 2¯ Ë 2¯ Ë 2¯
q , j &y lie between 0 and p, prove that,
Ê r ˆÊ rˆ
2 Êqˆ Êjˆ Êy ˆ 53. In a triangle ABC, if Á1 - 1 ˜ Á1 - 1 ˜ = 2 ,
tan Á ˜ + tan 2 Á ˜ + tan 2 Á ˜ = 1 Ë r2 ¯ Ë r3 ¯
Ë 2¯ Ë 2¯ Ë 2¯
40. The sides of a triangle are ABC are in A.P. If the an- prove that the triangle is right angled.
gles a and c are the greatest and the smallest angles
54. In triangle ABC, prove that, cot A + cot B + cot C ≥ 1
2 2 2
respectively, then prove that,
55. If cosA=tan B, cosB = tan C and cosC = tan A, then
4 (1 - cos A) (1 - cos C ) = cos A + cos C
prove that, sin A = sin B = sin C = 2 sin 18.
41. In triangle ABC, if a, b, c are in H.P., prove that
Ê Aˆ Ê Bˆ Ê Cˆ LEVEL II
sin 2 Á ˜ , sin 2 Á ˜ , sin 2 Á ˜ are also in H.P. MIXED PROBLEMS
Ë 2¯ Ë 2¯ Ë 2¯
42. In a triangle ABC, if the sides a, b, c be in A.P., prove that a 3 + b3 + c 3
1. In DABC, a > b > c, if = 8,
Ê Aˆ Ê Bˆ Ê Cˆ sin 3 A + sin 3 B + sin 3 C
cos A. cot Á ˜ , cos B. cot Á ˜ , cos C. cot Á ˜
Ë 2¯ Ë 2¯ Ë 2¯ then the maximum value of a is
1
are also in A.P. (a) (b) 2
43. The sides of a triangle are in A.P. and its area is 3/5 th 2
of an equilateral triangle of the same perimeter. Prove (c) 8 (d) 64
that the sides are in the ratio 3 : 5 : 7. 2. Sides of a triangle ABC are in A.P. If a < min {b, c},
44. The ex-radii r1 , r2 , r3 of a triangle ABC are in H.P., then cos A may be equal to
prove that the sides a, b, c are in A.P. 3c - 4b 3c - 4b
(a) (b)
2a 2c
Properties of Triangles 399

4c - 3b 4c - 3b Ê Bˆ Ê Cˆ
(c) (d) 11. If b + c = 3a, then the value of cot Á ˜ cot Á ˜ is
2b 2c Ë 2¯ Ë 2¯
3. If a DABC, a4 + b4 + c4 = 2a2b2 + b2c2 + 2c2a2, (a) 1 (b) 2
then sin A is
(c) 3 (d) 2
1 1
(a) (b) 12. Let A0A1A2A3A4A5 be a regular hexagon inscribed in a
2 2
circle of unit radius. The product of length of the line
3 3 +1 segment A0A1, A0A2, A0A4 is
(c) (d)
2 2 2 3
2 2 2 (a) (b) 3 3
4. In a triangle ABC, 2a + 4b + c = 4ab + 2ac, then the 4
numerical value of cos B is 3 3
3 (c) 3 (d)
(a) 0 (b) 2
8
5 7 a cos A + b cos B + c cos C
(c) (d) 13. In a DABC, the value of is
8 8 a+b+c

5. If a, b, c be the sides of DABC and if roots of the R R


(a) (b)
equation a(b – c) x2 + b (c – a) x + c (a – b) = 0 are r 2r
Ê Aˆ Ê Bˆ Ê Cˆ r 2r
equal then sin2 Á ˜ . sin2 Á ˜ . sin2 Á ˜ are in (c) (d)
Ë 2¯ Ë 2¯ Ë 2¯ R R
(a) A.P. (b) G.P. 14. In a DABC, the sides a, b, c are the roots of the equa-
(c) H.P. (d) A.G.P. cos A cos B
6. In a triangle ABC, (a + b + c) (b + c – a) = k bc if tion x3 – 11x2 + 38x – 40 = 0. Then + +
a b
(a) k < 0 (b) k > 6 cos C
(c) 0 < k < 4 (d) k > 4 is
c
7. a3 cos (B – C) + b3 cos (C – A) + c3 cos (A – B) is equal
(a) 3 abc (b) (a + b + c) 3
(a) 1 (b)
(c) abc (a + b + c) (d) 0 4
cos A cos B cos C 9
8. If = = and a = 2, then the area (c) (d) None
a b c 16
of the triangle is 15. The ex-radii of a D r1, r2, r3 are in A.P., then the sides
(a) 1 (b) 2 a, b, c are in
(a) A.P. (b) G.P.
3
(c) (d) 3 (c) H.P. (d) A.G.P.
2
sin 2 A + sin A + 1
9. If in a DABC, cos A + 2 cos B + cos C = 2, then a, b, 16. In any DABC, Â is always greater
c are in sin A
than
(a) A.P. (b) G.P.
(a) 9 (b) 3
(c) H.P. (d) None
(c) 27 (d) 36
10. If in a DABC, sin A + sin B + sin3C = 3 sinA sinB sinC,
3 3

then the value of Ê rˆ Ê rˆ


17. In a triangle Á1 - 1 ˜ Á1 - 1 ˜ = 2, then the triangle
a b c Ë r2 ¯ Ë r3 ¯
b c a is
(a) right angled (b) isosceles
c a b
(c) equilateral (d) None
(a) 0 p
(b) (a + b + c)2 18. In a DABC, a = 2b and |a – b| = , then –C is
3
(c) (a + b + c) (ab + bc + ca) p p
(a) (b)
(d) None 4 3
400 Comprehensive Trigonometry with Challenging Problems & Solutions for Jee Main and Advanced

p 28. In a DABC, tan A tan B tan C = 9. For such triangles,


(c) (d) None if tan2A + tan2B + tan2C = l, 7 then
6
3
(a) 9. 3 < l < 27 (b) l £/ 27
19. If the median of DABC, through A is perpendicular to
AB, then (c) l < 9. 3
3 (d) l > 27
(a) tanA + tanB = 0
29. In a DABC, a cos A = b + c2, then
2 2 2
(b) 2 tanA + tanB = 0
(c) tanA + 2 tanB = 0 p p p
(a) A < (b) <A<
(d) None 4 4 2
3 p p
20. In a DABC, cos A + cos B + cos C = , then the D (c) A > (d) A =
2 2 2
(a) Isosceles (b) right angled 30. In a DABC, A : B : C = 3 : 5 : 4, then a + b + c 2 is
(c) equilateral (d) None (a) 2b (b) 2c
21. If A1........An be a regular polygon of n-sides and (c) 3b (d) 3a
1 1 1 31. If A, B, C are angles of a triangle such that the angle
= + , then A is obtuse, then tan B tan C <
A1 A2 A1 A3 A1 A4
(a) 0 (b) 1
(a) n = 5 (b) n = 6 (c) 2 (d) 3
(c) n = 7 (c) None
32. In a triangle, if r1 > r2 > r3, then
Ê Aˆ 5 Ê Cˆ 2 (a) a > b > c (b) a < b < c
22. In a DABC, tan Á ˜ = and tan Á ˜ = s , then
Ë 2¯ 6 Ë 2¯ 5 (c) a > b and b < c (d) a < b and b > c
(a) a, c, b ΠA.P. (b) a, b, c ΠA.P. A B C
33. 4rR cos cos cos is
(c) b, a, c ΠA.P. (d) a, b, c ΠG.P. 2 2 2
23. If the angles of a triangle are in the ratio 1 : 2 : 3, then (a) s (b) s2
the corresponding sides are in the ratio (c) D2 (d) D
(a) 2 : 3 : 1 (b) 3 :2:1 34. If (a – b) (s – c) = (b – c) (s – a),
then r1, r2, r3 are in
(c) 2 : 3 :1 (d) 1 : 3 :2 (a) H.P. (b) G.P.
24. In a DABC, a cot A + b cot B + c cot C is (c) A.P. (d) A.G.P.
(a) r + R (b) r – R 35. If c2 = a2 + b2, 2s = a + b + c, then
(c) 2(r + R) (d) 2(r – R) 4s (s – a) (s – b) (s – c) is
25. If A, A1, A2, A3 are the areas of in circle and the ex (a) s4 (b) b2c2
2 2
circles of a triangle, then (c) c a (d) a2b2
1 1 1 1 1 1 1
+ + is 36. + + + is
A1 A2 A3 r12 r22 r32 r2

(a)
2
(b)
1 a 2 + b2 + c2 Sa 2
(a) (b)
A A s2 D2
1 3 (c) 4R (d) 4r
(c) (d)
2 A A 37. (r1 – r) (r2 – r) (r3 – r) is
Ê sin A + sin A + 1ˆ
2 R
26. In any DABC, P Á ˜ is (a) (b) 4R2r
Ë sin A ¯ r

always greater than (c) 4Rr2 (d) 4R


(a) 9 (b) 3 38. If the sides be 13, 14, 15, then
(c) 27 (d) 36 b-c c-a a-b
27. In an equilateral triangle, R : r : r2 is + + is
r1 r2 r3
(a) 1 : 1 : 1 (b) 1 : 2 : 3
(c) 2 : 1 : 3 (d) 3 : 2 : 4
Properties of Triangles 401

(a) 5 (b) 4 (a) 3 (b) 2


(c) 0 (d) 1 (c) 4 (d) 1.
r1 r r 50. In a triangle of ABC, if cos A + cos B = 4 sin2 (C/2),
39. + 2 + 3 is then a, b and c are in
bc ca ab
(a) A.P. (b) G.P.
1 1 (c) H.P. (d) None.
(a) – (b) 2R – r
2R r
1 1 LEVEL III
(c) r – 2R (d) –
r 2R (FOR JEE ADVANCED EXAM ONLY)
40. r1 + r2 = cos A + 2 cos C sin B
1 If in a triangle ABC, = , prove
Ê Cˆ Ê Cˆ cos A + 2 c cos B sin C
(a) c tan Á ˜ (b) c cot Á ˜
Ë 2¯ Ë 2¯ that the triangle ABC is either isoceles or right angled.
2. In a triangle ABC, if
Ê Cˆ Ê Cˆ
(c) c sin Á ˜ (d) c cos Á ˜ Ê A + Bˆ
Ë 2¯ Ë 2¯ a tan A + b tan B = (a + b ) tan Á ,
Ë 2 ˜¯
41. 16R2 r r1 r2 r3 is
(a) abc (b) a3b3c3 prove that the triangle is isosceles.
(c) a2b2c2 (d) a2b3c4 3. In any triangle ABC, prove that,

42. If
r r
= 2 , then
(r2 + r1 ) (r3 + r2 ) sin C = 2 r3 r2 r3 + r1 r3 + r1 r2
r1 r3 4. In any triangle ABC, prove that,
(a) A = 90°
(c) C = 90°
(b) B = 90°
(d) None
(
r 2 + r12 + r22 + r32 = 16 R 2 - a 2 + b 2 + c 2 )
43. In a DABC, the value of r r1r2r3 is 5. In any triangle ABC, prove that,
(a) D (b) D2 Ê Ê B - Cˆˆ Ê Ê C - Aˆ ˆ
(c) D3 (d) D4 ÁË ( r + r1 ) tan ÁË 2 ˜¯ ˜¯ + ÁË ( r + r2 ) tan ÁË 2 ˜¯ ˜¯

44. If r1 = r2 + r3 + r, then the D is Ê Ê C - Aˆ ˆ


+ Á (r + r3 ) tan Á =0
(a) Equilateral (b) Isosceles Ë Ë 2 ˜¯ ˜¯
(c) Right angled (d) None
6. In any triangle ABC, prove that,
45. (r1 + r2) (r2 + r3) (r3 + r1) is
(a) Rs2 (b) 2Rs2 Ê Aˆ Ê Bˆ Ê Cˆ
tan Á ˜ tan Á ˜ tan Á ˜
(c) 3Rs 2
(d) 4Rs2 Ë 2¯ Ë 2¯ Ë 2¯ 1
+ + =
46. The diameter of the circum-circle of a triangle with ( a - b ) ( a - c ) ( b - a ) (b - c ) ( c - a ) ( c - b ) D
sides 5 cm, 6 cm., 7 cm is Ê r ˆÊ rˆ
7. If Á1 - 1 ˜ Á1 - 1 ˜ = 2 , prove that the
(a)
3 6
cm. (b) 2 6 cm Ë r2 ¯ Ë r3 ¯
2
triangle is right angled triangle.
35 35
(c) cm (d) 8. In triangle ABC, prove that,
48 2 6
area of thein-circle
47. In a DABC, the sides are in the ratio 4 : 5 : 6. The ratio
of the circum-radius and the in-radius is area of triangle ABC
(a) 8 : 7 (b) 3 : 2
p
(c) 7: 3 (d) 16 : 7 =
A
Ê ˆ Ê Bˆ Ê Cˆ
48. If in a triangle, R and r are the circum radius and in cot Á ˜ .cot Á ˜ .cot Á ˜
Ë 2¯ Ë 2¯ Ë 2¯
radius, respectively, then the H.M. of the ex-radii of
the triangle is
(a) 3r (b) 2R Ê Aˆ
9. If a, b, c are in A.P., prove that, cos A.cot Á ˜ ,
(c) R + r (d) None Ë 2¯
49. If a, b and c are the sides of a triangle ABC and 3a =
Ê Bˆ Ê Cˆ
b + c, the value of Cot (B/2) cot (C/2) is cos B.cot Á ˜ , cos C.cot Á ˜ are in A.P.
Ë 2¯ Ë 2¯
402 Comprehensive Trigonometry with Challenging Problems & Solutions for Jee Main and Advanced

10. If the circumference of the DABC lies on its incircle, 24. Prove that the circum-radius of an ex-central triangle
then prove that, cos A + cos B + cos C = 2 I 2 I3
is = 2R.
2sin( I1 I 2 I 3 )
11. ABCD is a trapezium such that AB, DC are parallel 25. Prove that the distance between the in-center and the
and BC is perpendicular to them. If angle –ADB = q , ex-centers are

BC = p and CD = q, prove that, AB =


(p 2
)
+ q 2 sin q Ê Aˆ Ê Bˆ
II1 = 4 R sin Á ˜ , II 2 = 4 R sin Á ˜ ,
p cos q + q sin q Ë 2¯ Ë 2¯
12. Let O be the circumcenter and H be the orthocenter of Ê Cˆ
DABC. If Q is the mid-point of OH, then show that II 3 = 4 R sin Á ˜ .
Ë 2¯
R
AQ = 1 + 8cos A cos B cos C .
26. If a , b , c are in A.P., then prove that
2 2 2
2
13. If I1 , I 2 & I 3 are the centres of escribed circles of cot a . cot b, cot c are in A.P.
abc 27. In any triangle ABC, prove that,
DABC, prove that the area of D I1 I 2 I 3 = .
2r a3 cos ( B - C ) + b3 cos (C - A)
14. In DABC, prove that, +c3 cos ( A - B ) = 3abc
a 2 (s - a ) + b2 (s - b) + c2 (s - c)
28. The sides of a triangle are in A.P. and the greatest and
Ê Ê Aˆ Ê Bˆ Ê Cˆˆ least angles are q and f, respectively, then prove that
= 4 R D Á1 - 4 sin Á ˜ sin Á ˜ sin Á ˜ ˜
Ë Ë 2¯ Ë 2¯ Ë 2 ¯¯ 4 (1 - cos q ) (1 - cos j ) = cos q + cos j .
15. Let O be a point inside a triangle ABC such that 28. If in a triangle, the bisector of the side c be perpendic-
–OAB = –OBC = –OCA = w , then prove that ular to the side d, prove that 2 tanA + tan c = 0.
(i) cot A + cot B + cot C = cot w 29. In any triangle, if q be any angle, then prove that,
b cos q = c cos ( A - q ) + a cos (C + q ) .
(ii) cosec A + cosec B + cosec C = cosec w
2 2 2 2

30. If AD, BE and CF are the internal bisectors of the


16. Find the distance between the circum-center and the angles of a triangle ABC, prove that
mid-points of the sides of a triangle. 1 Ê Aˆ 1 Ê Bˆ 1 Ê Cˆ
17. Find the distance between the in-center and the angular cos Á ˜ + cos Á ˜ + cos Á ˜
AD Ë 2 ¯ BE Ë 2 ¯ CF Ë 2¯
points of a triangle.
1 1 1
18. Prove that the distance between the circum-centre (O) = + +
and the in-center (I) is a b c
Ê Ê Aˆ Ê Bˆ Ê Cˆˆ 31. Prove that the triangle having sides 3x + 4y, 4x + 3y
OI = R ¥ Á1 - 8 sin Á ˜ sin Á ˜ sin Á ˜ ˜ and 5x + 5y units, respectively, where x, y > 0, is obtuse
Ë Ë 2 ¯ Ë 2 ¯ Ë 2 ¯¯
angled.
19. Prove that the ratio of circum-radius and in-radius of 32. If D be the area and ‘s’ be the semi perimeter of a
an equilateral triangle is 1/2. s2
triangle, then prove that, D £
20. Prove that the ratio of the area of the in-circle to the 3 3
Ê Aˆ Ê Bˆ Ê Cˆ
area of a triangle is p : tan Á ˜ ◊ tan Á ˜ ◊ tan Á ˜ . 33. Let ABC be a triangle having altitudes h1 , h2 & h3
Ë 2¯ Ë 2¯ Ë 2¯
from the vertices A, B, C, respectively, and r be the
21. Prove that the distance of the orthocenter from the h + r h2 + r h3 + r
sides and angular points of a triangle is in-radius, prove that, 1 + + ≥ 6.
h1 - r h2 - r h3 - r
2 R cos A, 2 R cos B and 2 R cos C.
34. Two circles of radii a and b cut each other at an angle
22. Prove that the distance between the circum-center and
q. Prove that the length of the common chord is
the orthocenter of a triangle is OH = 2ab sin q
R 1 - 8 cos A.cos B.cos C .
a 2 + b 2 + 2ab cos q
23. Prove that the area of an ex-central triangle
Ê Aˆ Ê Bˆ Ê Cˆ
is 8R 2 cos Á ˜ cos Á ˜ cos Á ˜ .
Ë 2¯ Ë 2¯ Ë 2¯
Properties of Triangles 403

35. If a, b, g are the distances of the vertices of a triangle 45. In a triangle ABC, the vertices A, B, C are at distances
from the corresponding points of contact with the of p, q, r from the orthocentre, respectively. Prove that
a bg Ê a b c ˆ abc
in-circle, prove that r =
2
a +b +g ÁË p + q + r ˜¯ = pqr
36. Tangents are drawn to the in-circle of a triangle ABC 46. The internal bisectors of the angles of a triangle ABC
which are parallel to its sides . If x, y, z be the lengths meet the sides in D, E and F. Prove that the area of the
of the tangents and a, b, c be the sides of a triangle,
x y z 2D abc
then prove that, + + = 1 . triangle DEF is .
a b c ( a + b ) (b + c ) ( c + a )
47. In a triangle ABC, the measures of the angles A, B, C
37. If t1 , t2 & t3 be the lengths of the tangents from the
are 3a , 3b and 3 g , respectively. P, Q, and R are the
e-centers of escribed circles to the circum-circles,
points within the triangle such that
1 1 1 2s
prove that 2 + 2 + 2 = . –BAR = –RAQ = –QAC = a ,
t1 t1 t1 abc
–CBP = –PBR = –RBA = b and
38. if x, y, z be the lengths of the perpendiculars from the
circumcentre on the sides BC, CA, AB of a triangle –ACQ = –QCP = –PCB = g , then prove that
ABC, prove that, AR = 8R sin b sin g cos (30∞ - g )
a b c abc
+ + = 48. If in a triangle ABC, the median AD and the perpen-
x y z 4 xyz dicular AE from the vertex A to the side BC divides
the angle A into three equal parts, show that
39. If p1 , p2 , p3 are the altitudes of the triangle ABC from
A A 3 a2
the vertices a, b and c, respectively, prove that cos ÊÁ ˆ˜ .sin 2 ÊÁ ˆ˜ = .
Ë3¯ Ë 3 ¯ 32 b c
cos A cos B cos C 1
+ + = 49. If the sides of a triangle are in A.P. and if its greatest
p1 p2 p3 R
angle exceeds the least angle by a, show that the sides
40. The product of the sines of the angles of a triangle is 1 - cos a
p and the product of their cosines is q. Prove that the are in the ratio (1 - x ) :1: (1 + x ) , where x = .
7 - cos a
tangents of the angles are the roots of
p
q x3 - p x 2 + (1 + q ) x - p = 0 . 50. Let A, B, C be three angles such that A = and tan
4
41. In a triangle ABC, if cos A . cos B + sin A sin B sin C B tanC = p. Find all possible values of p such that A,
=1, prove that the sides are in the ratio 1 : 1 : 2 . B, C are the angles of a triangle.
42. The base of a triangle is divided into three equal parts. Ans. p < 0 & p ≥ 3 + 2 2 .
If t1 , t2 & t3 be the tangents of the angles subtended by
INTEGER TYPE QUESTIONS
these parts at the opposite vertices, prove that,
Ê 1 1ˆÊ 1 1ˆ Ê 1ˆ
ÁË t + t ˜¯ ÁË t + t ˜¯ = 4 ÁË1 + t 2 ˜¯ . a 2 + b2 + c2
1 2 2 3 2 of .
R2
43. The three medians of a triangle ABC make angle
a , b , g with each other, prove that, cot a + cot b + cot g cos A cos B cos C
2. In any triangle ABC, = =
a b c
+ cot A + cot B + cot C = 0
Ê r1 + r2 + r3 ˆ
44. Perpendiculars are drawn from the angles A, B, C of ÁË r ˜¯ .
an acute angled triangle on the opposite sides and 3. In any triangle ABC
produced to meet the circumscribing circle. If these
parts be a, b, g , respectively, then prove that Êr +r +r ˆ
value of Á 1 2 3 ˜ .
Ë r ¯
a b c
+ + = 2 ( tan A + tan B + tan C )
a b g
404 Comprehensive Trigonometry with Challenging Problems & Solutions for Jee Main and Advanced

4. In any triangle ABC 3. If in a triangle ABC, the sides a, b, c are in A.P., then
Ê sin A + sin A + 1ˆ
2 prove that
value of Á ˜. Ê Aˆ Ê Cˆ Ê Bˆ
Ë sin A ¯ cot Á ˜ + cot Á ˜ = 2 cot Á ˜ .
Ë 2¯ Ë 2¯ Ë 2¯
5. In a triangle ABC
4. If in a triangle of ABC, A = 3 B, then prove that
1
8R 2
( ( ))
r 2 + r12 + r22 + r32 + a 2 + b 2 + c 2 .
sin b =
1 3b - a
.
2 b
where r = in-radius, R = circum - radius and
r1 , r2 , r3 are ex-radii. 5. Let D1 , D 2 denote the areas of a triangle and that of
1 the incircle . prove that
6. In a triangle ABC, the median AD = .
11 - 6 3 D1 Ê A B Cˆ
= Á cot .cot .cot ˜ : p
D2 Ë 2 2 2¯
and it divides the Angle a into angles 30∞ and 45∞
Find the length of the side BC. 6. If the ex-radii r1, r2, r3 of a triangle ABC are in H.P,
7. In a triangle ABC then prove that the sides a, b, c are in A.P.
7. The lenghts of the sides of a triangle are three consec-
(r1 + r2 ) (r2 + r3 ) (r1 + r3 ) utive natural numbers and its largest angle is twice the
, where
R s2 smallest one. Determine the sides of the triangle.
8. Let O be a point in the triangle ABC such that
R = circum - radius, r1 , r2 , r3 are ex-radii.
–OAC = –OCB = –OBA = a ,
and s is the semi perimeter. then prove that cot a = cot A + cot B + cot C
8 If in a triangle ABC, a = 6, b = 3 and 9. If in a triangle ABC, two sides are a = 6, b = 3 and
4
4 cos (A – B) = , then prove that the area of the
cos ( A - B ) = 5
5
triangle is 9.
9 A triangle has base 10 cm long and the base angles
10. In a triangle ABC, the angles A, B, C are in A.P., then
are 50∞ and 70∞ . If the perimeter of the triangle prove that
is x + y cos ( z ∞) where z Œ(0, 90∞) , then Ê A- Cˆ a+c
2 cos Á =
Ë 2 ˜¯ a 2 - ac + c 2
Ê x + y + zˆ
ÁË y ˜¯ .
(QUESTIONS ASKED IN IIT-JEE
10 In any D ABC
EXAMS WITH THEIR SOLUTIONS)
a cot A + b cot B + c cot C
. 1. If p1 , p2 , p3 are the altitudes of a triangle from the
(r + R ) vertices A, B, C, respectively and D be the area of a
triangle, prove that
SELF ASSESMENT I 2ab
1 1 1 Ê Cˆ
+ + = cos 2 Á ˜ .
CH: PROPERTIES OF TRIANGLES p1 p1 p1 ( a + b + c ) D Ë 2¯

Time : 3 Hrs. Max. Marks : 100. [IIT-JEE-1978]


2. A quadrilateral ABCD is inscribed in a circle S and A,
Give answer of the following questions. B, C, D are the points of contacts with S of an other
1. If in a triangle ABC, tan A = 2, tan B = 3/2 and quadrilateral which is circumscribed about S. If this
c = 2 65 quadrilateral is also cyclic, prove that

2. If in a triangle ABC, cos 3 A + cos 3B + cos 3C = 1 AB 2 + CD 2 = BC 2 + AD 2 . [IIT-JEE-1978]


then prove that the measure of one of the angles must 3. If two sides of a triangle and the included angle are
2p
be
3
. ( )
given by a = 3 + 1 cm, b = 2 cm and C = 60∞
the other two angles and the third side.
[IIT-JEE-1979]
Properties of Triangles 405

4. If a circle is inscribed in a right angled triangle ABC 16. There exists a triangle ABC satisfying the conditions
with right angled at B, show that the diameter of the p
circle is equal to AB + BC – AC. (i) b sin A = a, A <
2
[IIT-JEE-1979]
5. ABC is a triangle, d is the middle point of BC. If AD p
(ii) b sin A > a, A >
is perpendicular to AC, prove that 2

cos A cos C =
(
2 c2 - a2 ). [IIT-JEE-1980]
(iii) b sin A > a, A >
p
2
3ac
p
6. Let the angles A, B, C of a triangle ABC be in A.P. and (iv) b sin A < a, A < ,b > a
2
let b : c = 3 : 2 . Find the angle A. [IIT-JEE-1981] [IIT-JEE-1986]
7. No questions asked in 1982. 17. In a triangle, the lengths of the two larger sides are 10
and 9, respectively. If the angles are in A.P., then the
8. The ex-radii r1 , r2 , r3 of DABC are in H.P. length of the third side can be
Show that the sides a, b, c are in A.P.
(a) 5 - 6 (b) 3 3
[IIT-JEE-1983]
9. For a triangle ABC, it is given that
(c) 5 (d) 5 + 6
3
cos A + cos B + cos C = , prove that thetriangle is [IIT-JEE-1987]
2
18. If the angles of a triangle are 30∞ and 45∞ and the
equilateral. [IIT-JEE-1984]
10. With usual notation, if in a triangle ABC included side is ( )
3 + 1 , then the area of the
b+c c+a a+b triangle is ...... [IIT-JEE-1988]
= = , prove that
11 12 13 19. ABC is an isosceles triangle inscribed in a circle of
cos A cos B cos C radius r. If AB = AC and h is the altitude from A to
= = [IIT-JEE-1984] BC, then the triangle ABC has perimeter
7 19 25
P = ...... and area D = ...... and
11. In a triangle ABC, the median to the side BC is at length
Ê Dˆ
1 also lim Á 3 ˜ = ...... [IIT-JEE - 1989]
and it divides the angle a into angles of xÆ0 Ë p ¯
11 - 6 3
20. In a triangle ABC, a is greater than angle B. If the
30∞ and 45∞ . Find the length of the side BC.
measures of angles A and B satisfy the equation
[IIT-JEE 1985] 3 sin x - 4 sin 3 x - k = 0 , then the measure of
12. In a triangle ABC, if cot A, cot B, cot C are angle C is
in A.P., then prove that a , b , c are in A.P.
2 2 2 p p
(a) (b)
3 2
[IIT-JEE-1985]
13. The set of all real numbers a such that 2p 5p
(c) (d)
a + 2a , 2a + 3, a + 3a + 8 are the sides of
2 2 3 6
[IIT-JEE-1990]
A triangle. is ........... [IIT-JEE-1985]
21. ABC is a triangle such that
14. The sides of a triangle inscribed in a given circle
1
subtends angle a , b & g at the centre. The minimum sin ( 2 A + B ) = sin (C - A) = - sin ( B + C ) = .
value of the arithmetic mean of 2
Ê pˆ Ê pˆ Ê pˆ If A, B and C are in A.P., determine the values of
cos Á a + ˜ , cos Á b + ˜ , cos Á g + ˜ is ... A, B and C. [IIT-JEE-1990]
Ë 2 ¯ Ë 2 ¯ Ë 2¯
22. The sides of a triangle are three consecutive natural
[IIT-JEE-1986] numbers and its largest angle is twice the smallest one.
15 In a triangle ABC, Determine the sides of a triangle .
cos A cos B + sin A sin B sin C = 1 , [IIT-JEE-1991]
show that a : b : c = 1 : 1 : 2 [IIT-JEE-1986]
406 Comprehensive Trigonometry with Challenging Problems & Solutions for Jee Main and Advanced

23. In a triangle of base a, the ratio of the other two sides


1 2
is r (< 1). Show that the altitude of the triangle is less (c) (d)
3 3[IIT-JEE-1995]
ar
than or equal to . [IIT-JEE-1991] 31. In a triangle ABC, a:b:c = 4:5:6. The ratio of the radius
1 - r2
of the circum-circle to that of in-circle is.....
24. Three circles touch one another externally. [IIT-JEE-1996]
The tangents at their points of contact meet a point
whose distance from a point of a contact is 4. Find the p
ratio of the product of the radii to the sum of the radii 32. Let ABC be three angles such that A = and
4
of the circles. [IIT-JEE-1992] tan ( B ) tan (C ) = p . Find all possible values of p such
25. If in a triangle ABC, that a, b, c are the angles of atriangle.
2 cos A 2 cos B 2 cos C 1 b [IIT-JEE-1997].
+ + = + ,
a b c bc ca 33. Prove that a triangle ABC is equilateral if and only if
A in degrees. [IIT-JEE-1993] tan ( A) + tan ( B ) + tan (C ) = 3 3 . [IIT-JEE-1998]
26. Let A1 , A2 ,...., An be the vertices of n sided regular 34. If in a triangle PQR, sinP, sinQ, sinR are in A.P.
polgon such that then
1 1 1 (a) the altitude are in A.P.
+ = n. [IIT-JEE-1994].
A1 A2 A1 A3 A1 A2 (b) the altitude are in H.P.
(c) the medians are in G.P.
27. Consider the following statement concerning a (d) the medians are in A.P. [IIT-JEE-1998]
triangle ABC 35. Let ABC be atriangle having O and I as its circum-cen-
(i) The sides a, b, c and area of D are rational tre and in-centre, respectively. If R and r are the
Ê Bˆ Ê Cˆ circum-radius and in-radius, respectively, then prove
(ii) a, tan Á ˜ , tan Á ˜ are rational
Ë 2¯ Ë 2¯ that ( IO ) = R - 2 Rr . Further show that the triangle
2 2

BIO is a right angled triangle if and only if b is the


(iii) a,sin A,sin B,sin C are rational
arithmetic mean of a and c. [IIT-JEE-1999]
Then prove that (i ) fi (ii ) fi (iii ) fi (i ) 36. In any triangle ABC, prove that
[IIT-JEE-1994].
Ê Aˆ Ê Bˆ ÊCˆ Ê Aˆ Ê Bˆ ÊCˆ
28. In a DABC , AD is an altitude from A, Given cot Á ˜ + cot Á ˜ + cot Á ˜ = cot Á ˜ cot Á ˜ cot Á ˜ .
Ë 2¯ Ë 2¯ Ë 2¯ Ë 2¯ Ë 2¯ Ë 2¯
abc [IIT-JEE-2000]
b > c, –C = 23∞ and AD = 2 –B . 37. Let ABC be a triangle with incentre I and in-radius r
a - c2
Let D, E, F be the feet of the perpendicular from I to
[IIT-JEE-1994].
the sides BC, CA and AB, respectively, If r1 , r2 , r3 are
29. If the length of the sides of a triangle are 3, 5, 7, then the radii of the circles inscribed in the quadrilaterals
the largest angle of the triangle is AFIE, BDIF and CEIF, respectively,
p 5p prove that,
(a) (b)
2 6 r1 r r r1r2 r3
+ 2 + 3 =
2p 3p r - r1 r - r2 r - r3 ( r1 - r2 ) ( r2 - r3 ) ( r3 - r1 )
(c) (d)
3 4 [IIT-JEE-2000]
[IIT-JEE-1994]
Ê A- B + Cˆ
p p 38. In a triangle ABC, 2ac sin Á ˜¯ =
30. In a triangle ABC, –B = & –C = . Let D divide Ë 2
3 4
(a) a + b - c (b) c + a - b
2 2 2 2 2 2
BC internally in the ratio 1:3, then
sin (–BAD ) (c) b - c - a
2 2 2
(d) c - a - b
2 2 2
is equal to
sin (–CAD ) [IIT-JEE-2000]
p
(a)
1
(b)
1 39. In a triangle ABC, let –C = . If r is the in-radius
3 2
6
and R is the circum-radius of the triangle, then
Properties of Triangles 407

2 ( R + r ) is equal to Aˆ Ê B - Cˆ
(a) a + b (b) b + c
(b) (b - c ) cos ÊÁË ˜¯ = a sin ÁË ˜
2 2 ¯
(c) c + a (d) a + b + c
B + Cˆ Ê Aˆ
[IIT-JEE-2000] (c) (b + c ) sin ÊÁË ˜ = a cos ÁË ˜¯
40. If D is the area of a triangle with side lengths a, b and 2 ¯ 2
1
c, then show that D £ (a + b + c ) abc . (d) (b - c ) cos ÊÁË
Aˆ Ê B + Cˆ
˜¯ = 2a sin ÁË ˜
4 2 2 ¯
Also show that the equality occurs in the above in-
[IIT-JEE-2005]
equality if and only if a = b = c [IIT-JEE-2001]
41. Which of the following pieces of data does not unique- 47. One angle of an isosceles triangle is 120∞ and radius
ly determine an acute angled triangle ABC of its incircle is 3 . Then the area of the triangle in
(R being the radius of the circum-circle)? sq. units is
(a) a, sinA, sinB (b) a, b, c
(c) a, sinB, R (d) a, sinA, R (a) (7 + 12 3 ) (
(b) 12 - 7 3 )
[IIT-JEE-2002]
42. If the angles of a triangle are in the ratio 4:1:1, then (c) (12 + 7 3 ) (d) 4p
ratio of the longest side to the perimeter is
[IIT-JEE-2006]
(a)
3
(b)
1 48. In a triangle ABC, internal angle bisector of –A meets
(2 + 3 ) 6 side BC in D, DE ^ AD meets AC in E and AB in F.
Then
1 2 (a) AE is H.M. of b and c
(c) (d)
(2 + 3 ) 3
[IIT-JEE-2003]
Ê 2bc ˆ
(b) AD = Á
Ê Aˆ
Ë b + c ˜¯
cos Á ˜
Ë 2¯
43 If I n is the area of n-sided regular polygon inscribed
in a circle of unit radius and On be the area of the Ê 4bc ˆ Ê Aˆ
(c) EF = Á
Ë b + c ˜¯
polygon circumscribing the given circle, prove that sin Á ˜
Ë 2¯
(d) DAEF is isosceles.
O Ê

Ê 2I ˆ [IIT-JEE-2006]
I n = n Á1 + 1 - Á n ˜ ˜ . [IIT-JEE-2003] 49. Let ABCD be a quadrilateral with area 18, with side
2 ÁË Ë n ¯ ˜
¯ AB parallel to the side CD and AB = 2 CD. Let AD
44. The side of a triangle are in the ratio 1 : 3 : 2 , then be perpendicular to AB and CD . If a circle is drawn
the angles of the triangle are in the ratio is inside the quadrilateral ABCD touching all the sides,
(a) 1:3:5 (b) 2:3:4 then its area is
(c) 3:2:1 (d) 1:2::3 (a) 3 (b) 2
[IIT-JEE-2004] (c) 3/2 (d) 1
45. In an equilateral triangle, three coins of radii 1 unit [IIT-JEE-2007]
each are kept so that they touch each other and also 50 A straight line through the vertex P of a triangle PQR
the sides of the triangle. Area of the triangle is intersects the side QR at the point S and the cirum-cir-
cle of the triangle PQR at the point T. If S is not the
(a) 4 + 2 3 (b) 6 + 4 3 centre of the circum-circle, then
1 1 2
Ê 7 3ˆ Ê 7 3ˆ (a) + <
(c) Á12 + (d) Á 3 + PS ST QS ¥ SR
Ë 4 ˜¯ Ë 4 ˜¯
[IIT-JEE-2005] 1 1 2
(b) + >
46. In a triangle ABC, a, b, c are the lengths of its sides and PS ST QS ¥ SR
A, B, C are the angles of a triangle ABC. The correct
1 1 4
relation is given by (c) + <
PS ST QR
B - Cˆ Ê Aˆ
(a) (b - c ) sin ÊÁË ˜¯ = a cos ÁË ˜¯ 1 1 4
2 2 (d) + > [IIT-JEE-2008]
PS ST QR
408 Comprehensive Trigonometry with Challenging Problems & Solutions for Jee Main and Advanced

51. In a triangle ABC BC A 3 45


Ê Aˆ (a)
cos B + cos C = 4 sin 2 Á ˜ 4D 4D
Ë 2¯ 2 2
Ê 3 ˆ Ê 45 ˆ
If a, b c ÁË ˜¯ ÁË ˜¯
A, B C 4D 4D
then [IIT-JEE-2012]
(a) b + c = 4a b + c = 2a
A x
A pair of y.
If x - c = y , where c
2 2

[IIT-JEE-2009]

( 3 +1 ) (a)
3y 3y
p 2p 2 x ( x + c) 2c ( x + c )
k
k k 3y 3y
k]., [, ] = G.I.F [IIT-JEE-2010] 4 x ( x + c) 4 c ( x + c)
ABC a, b, c
A, B C, [IIT-JEE-2014]
a = 6, b
15 3 . If –ACB r
COMPREHENSIVE LINK PASSAGE
(FOR JEE ADVANCED EXAM ONLY)
r2 . [IIT-JEE-2010]
p
54. Let ABC –ACB =
6
let a, b c
to A, B C x PASSAGE 1
a = x 2 + x + 1, b = x 2 - 1 c = 2x + 1 If p1, p2, p3 ABC, from the
a, b, c D

(
(a) - 2 + 3 ) (1 + 3 )
(2 + 3 ) 4 3
1 1 1 1
[IIT-JEE-2010] 1. If + + = p1 p2 p3
p p1 p2 p3 2
53. Let ABC –ACB =
6
let a, b c
to A, B C x
a = x + x + 1, b = x - 1
2 2
c = 2x + 1 cos A cos B cos C
+ +
p1 p2 p3
(
(a) - 2 + 3 ) (1 + 3 ) 1 1
(2 + 3 )
(a)
4 3 r R
[IIT-JEE-2010]
a 2 + b2 + c2 1
55. Let PQR D with a = 2, b 2R D
c = 5/2, where a, b, c b 2 p1 c 2 p1 a 2 p1
P, Q R + +
c a b
2 sin P - sin 2 P
(a) D D
2 sin P + sin 2 P D D
Properties of Triangles 409

1 1 1 PASSAGE III
+ +
p13 p23 p33 G ABC
A, B, C BC, CA, AB at D, E, F
( Â a )2 (’ a )3 P, Q, R G
(a)
4D 2 8D 3
BC, CA, AB L, M, N
Âa 2
Pa 2
BC, CA, AB
4D 2
8D 2
5. In the triangle ABC PG
(a) a, b, c
a, b, c 1 1
(a) b sin C c sin C
a, b, c 2 2
A, B, C 2
b sin C
1
c sin B
3 3
2. ar ( D GPL ) : ar ( D ALD )
PASSAGE II
ABCD
R.

Then cos B =
(a 2
+ b2 - c2 - d 2 ) DPQR
2 (ab + cd )
1
(a)
1 2
9
( )
a + b 2 + c 2 sin A.sin B.sin C
(ab + cd ) sin B
2 1 2
18
( )
a + b 2 + c 2 sin A.sin B.sin C
= (s - a ) (s - b) (s - c) (s - d ) ,
where, s =
a+b+c+d
.
2 2
9
( )
a + b 2 + c 2 sin A.sin B.sin C
2
AC2 . BD2 = (ac + bd)2
1 2
3
( )
a + b 2 + c 2 sin A.sin B.sin C
i.e., AC. BD = AB . CD + BC . AD
PASSAGE IV
AC
R= In a triangle ABC, R
2sin B abc
R= r1 , r2 & r3
4D
D D D
r1 = , r2 = & r3 = r
s-a s-a s-a
D
that r = .
s

1. If r1 = r + r2 + r3

a, b, c, d
A B C
r Ê rˆ
(a) abcd 2 (abcd ) (a) 1 + 2 Á1 + ˜
Ë
R R¯
2¥ (abcd ) 1Ê rˆ 1Ê rˆ
ÁË1 + ˜¯ ÁË1 + ˜¯
2 R 4 R
410 Comprehensive Trigonometry with Challenging Problems & Solutions for Jee Main and Advanced

Ê r ˆÊ rˆ a 2 + b2 + c2 a 2 + b2 + c2
3. If Á1 - 1 ˜ Á1 - 1 ˜ = 2 (a)
Ë r2 ¯ Ë r3 ¯ 2abc abc
a 2 - b2 + c2 a 2 + b2 - c2
abc abc
r1 + r2 + r3 - 4 R 2
(
2 2 4 2 2 4
)
4. If c - 2 a + b c + a + a b + b = 0 ,
4

(a) 2 r r
r r. then the angle c

1 1 1
+ +
r1 r2 r3 cos 2 A cos 2 B
-
1 2 a 2
b2
(a)
r r 1 1 1 1
(a) + -
1 3 a2
b2
a 2
b2
2r 2r 2 2 2 2
- +
r1 r2 r3 a2 b2 a2 b2
+ +
bc ca ab
1 1 1 1 MATCH MATRIX
(a) - -
r R 2r R (FOR JEE ADVANCED EXAM ONLY)
1 1 1 1
- - . ABC, then
r 2R r 3R
Column-I Column-II
b C+c B c
PASSAGE V (B) c A+a C (Q) b
b2 + c2 - a 2
ABC, cos A = , (C) a B+b A (R) a
2bc
(D) (b + c A + (c + a B a+b+c
a +c -b
2 2 2
a +b -c
2 2 2
+ (a + b C
cos B = cos C = ,
2ac 2ab
ABC
where a = BC, b = CA c = AB Column-I Column-II
a B – C) + b C – A)
+c A–B P) 3abc
A, B, C
a+c
2 2
(
(B) a cos B - cos C
2
)
a 2 + c 2 - ac (
+b 2 cos 2 C - cos 2 A )
Ê A- Cˆ
(a) 2 cos Á
Ë 2 ˜¯
Ê A- Cˆ
2 sin Á
Ë 2 ˜¯
+c 2 (cos 2
A - cos 2 B) abc

Ê A- Cˆ Ê A- Cˆ Ê b2 - c2 ˆ
sin Á (C) Á ˜ sin 2 A
Ë 2 ˜¯
cos Á
Ë 2 ˜¯ Ë a2 ¯
1 1 1 Ê c2 - a2 ˆ
2. If + = , then +Á ˜ sin 2 B
a+c b+c a+b+c Ë b2 ¯
(a) – C = 75∞ – A = 75∞ Ê a 2 - b2 ˆ
+Á ˜ sin 2C
– A = 60∞ – C = 60∞ Ë c2 ¯
cos A cos B cos C (D) a cos ( B - C ) + b cos (C - A)
3 3
+ +
a b c
+c3 cos ( A - B ) abc.
Properties of Triangles 411

sin B
ABC (C) 2 cos A =
sin C
Column- I Column - II then D
B C (
P) a + b + c
2 2 2
) (D) a tan A + b tan B
Ê A + Bˆ
A+ ca B
(a 2
+ b2 -c )
2
= (a + b ) tan Á
Ë 2 ˜¯
,

(a +c )
(B) 2 (bc (Q)
+ ab C
2
+ b2 2
then D

cot A + cot B + cot C a 2 + b2 + c2 ABC, if


(C) (R) Column - I Column - II
Ê Aˆ Ê Bˆ Ê Cˆ
cot Á ˜ + cot Á ˜ + cot Á ˜ (a + b + c )2 sin A + sin B
Ë 2¯ Ë 2¯ Ë 2¯
3 3
+ sin C = ,
Ê Bˆ Ê Cˆ 2
cot Á ˜ + cot Á ˜ then D
Ë 2¯ Ë 2¯ Ê 2a ˆ
(D) ÁË ˜ (B) tan A + tan B \
Ê Aˆ b + c - a¯
cot Á ˜ + tan C = 3 3,
Ë 2¯
then D
ABC, if Ê Aˆ Ê Bˆ
(C) 8 sin Á ˜ sin Á ˜
Column- I Column - II Ë 2¯ Ë 2¯
A B C a, b, c Ê Cˆ
sin Á ˜ = 1,
Ë 2¯
Ê Aˆ then D
(B) cos A.cot Á ˜ , (Q) a2, b2, c2
Ë 2¯ (D) a + b + c = 8R
2 2 2 2

then D
Ê Bˆ Ê Cˆ
cos B.cot Á ˜ , cos C.cot Á ˜
Ë 2¯ Ë 2¯
ABC, if r
r1, r2 r3
2 Ê Aˆ 1 1 1 DABC
(C) sin Á ˜ , (R) , ,
Ë 2¯ a b c Column - I Column - II
1 1 1
Ê Bˆ Ê Cˆ + + r
sin 2 Á ˜ , sin 2 Á ˜ r1 r2 r3
Ë 2¯ Ë 2¯
1 1 1 1 D2
(B) + + +
Ê Aˆ r12 r22 r32 r2 a 2 + b2 + c2
(D) tan Á ˜ , a, b, c
Ë 2¯ 1
(C) r1 + r2 + r3 - 4 R
Ê Bˆ Ê Cˆ r
tan Á ˜ , tan Á ˜
Ë 2¯ Ë 2¯ r1 r2 r3 1 a 2 + b2 + c2
(D) + + +
bc ac ba 2 R D2

ABC, if ABC, if r R
Column- I Column - II
cot A + cot B + cot C = 3 , DABC
then D Column - I Column - II
(B) (a + b ) sin ( A - B)
2 2
A B C P)
D
r. R
= ( a - b ) sin ( A + B ) ,
2 2
Ê rˆ
(B) a A+b B+c C Q) Á1 + ˜
then D Ë R¯
412 Comprehensive Trigonometry with Challenging Problems & Solutions for Jee Main and Advanced

A B C R) 2 ( r + R ) A D s
s

2Ê 2 Ê Bˆ 2Ê Cˆ
D2 £
(D) cos Á ˜ + cos Á ˜ + cos Á ˜ 4
Ë 2¯ Ë 2¯ Ë 2¯
R A.M ≥ G.M
Ê r ˆ (a) A B
(S) Á 2 +
Ë 2 R ˜¯ C D
A ABC A B+
C = 2, then a, b, c
ABC, if r r1, r2 R ABC, cos A + cos B + cos C
r3 D ABC
of Ê Aˆ Ê Bˆ Ê Cˆ
= 1 + 4 sin Á ˜ .sin Á ˜ .sin Á ˜
Column - I Column - II Ë 2¯ Ë 2¯ Ë 2¯
r - r1 + r2 + r3 R C (a) A B
(B) r - r2 + r1 + r3 R B C D
(C) r - r3 + r1 + r2 R A A
a 2 + b 2 + c 2 = 8 R 2 , where R
(D) r1 + r2 + r3 - r
R a 2 = b2 + c2
ABC (a) A B
Column - I Column - II C D
cot 2 A + cot 2 B + cot 2 C A A, B, C D
(B) tan A + tan B + tan C
2 2 2
A B C D=0
Ê Aˆ Ê Bˆ R A, B, C D
(C) cosec Á ˜ + cosec Á ˜ R) 6
Ë 2¯ Ë 2¯
A B C D=0
Ê Cˆ (a) A B
+ cosec Á ˜
Ë 2¯ C D
A B C A a A+b B+c
C £s
ABC
R
Column - I Column - II
A B C P) 1 Ê Aˆ Ê Bˆ Ê Cˆ 1
sin Á ˜ sin Á ˜ sin Á ˜ £
A B C Q) 3/2 Ë 2¯ Ë 2¯ Ë 2¯ 8
2 Ê Aˆ 2 Ê Bˆ (a) A B
(C) tan Á ˜ + tan Á ˜ (R) 1/4
Ë 2¯ Ë 2¯ C D
Ê Cˆ A ABC
+ tan 2 Á ˜
Ë 2¯ r1 + r2 + r3
Ê Aˆ Ê Bˆ Ê Cˆ r
(D) sin Á ˜ .sin Á ˜ .sin Á ˜ S) 1/8.
Ë 2¯ Ë 2¯ Ë 2¯ R ABC, if
cos A cos B cos C r +r +r
= = , then 1 2 3 = 9
ASSERTION AND REASON a b c r
Codes: (a) A B
A R R C D
A. A ABC
(B) Both A R R
A. R ABC, r1 + r2 + r3 = 4 R
(C) A R (a) A B
(D) A R C D
Properties of Triangles 413

A A, A1, A2, A3 x + y + z = 2 (R + r)
R ABC,
1 1 1 1 Ê Aˆ Ê Bˆ Ê Cˆ
+ + = r = 4 R sin Á ˜ .sin Á ˜ .sin Á ˜
A1 A2 A3 A Ë 2¯ Ë 2¯ Ë 2¯

1 1 1 1 (a) A B
R + + = C D
r1 r2 r3 r
A ABC, r1 + r2 + r3 - r = 4 R
(a) A B
C D abc
R ABC, R =
A x, y z 4D
ABC (a) A B
a b c abc C D
+ + =
x y z xyz A ABC,
R ABC, Ê Ê Cˆ Ê Aˆ ˆ
tan A + tan B + tan C = tan A . tan B . tan C 2 Á a sin 2 Á ˜ + c sin 2 Á ˜ ˜ = a + c - b
Ë Ë 2¯ Ë 2 ¯¯
(a) A B
C D R ABC,
b=c A+a C.
A x, y z (a) A B
ABC C D

ANSWERS
EXERCISE 2 INTEGER TYPE QUESTIONS
6. 0 £ l £ 4 7. 120∞. 1. 8
2 1
EXERCISE 6 3 9
4 3
5. 2
6. 5
LEVEL I 7. 1
8. 9
9. 2
LEVEL II 10. 2

COMPREHENSIVE LINK PASSAGES:

MATCH MATRIX
Æ (R); (B) Æ(Q); (C) Æ Æ
Æ (R); (B) Æ (R); (C) Æ (R); (D) Æ
Æ (Q); (B) Æ Æ (R); (D) Æ
Æ (Q); (B) Æ Æ (R); (D) Æ
Æ (R); (B) Æ Æ Æ
Æ (R); (B) Æ (R); (C) Æ (R); (D) Æ ( Q)
414 Comprehensive Trigonometry with Challenging Problems & Solutions for Jee Main and Advanced

7. (A) Æ (R); (B) Æ (S); (C) Æ (P); (D) Æ ( R) (b – c)[a – 2(b + c)] = 0
8. (A) Æ (Q); (B) Æ (R); (C) Æ (P); (D) Æ ( S) .
9. (A) Æ (R); (B) Æ (Q); (C) Æ (P); (D) Æ ( S) (b – c) = 0
10. (A) Æ (Q); (B) Æ (P); (C) Æ (R); (D) Æ ( Q). b=c
11. (A) Æ (Q); (B) Æ (S); (C) Æ (P); (D) Æ ( S).
D is isosceles.
ASSERTION AND REASON 2. We have
1. (a) 2. (b) 3. (a) 4. (d)
Ê A + Bˆ
5. (a) 6. (b) 7. (c) 8. (a) a tanA + b tanB = (a + b) tan Á
Ë 2 ˜¯
9. (a) 10. (a) 11. (a) 12. (a)
ÊCˆ
a tanA + b tanB = (a + b) cot Á ˜
SELF ASSESMENT TEST Ë 2¯
1.
7. ( 4, 5, 6.) Ê Ê C ˆˆ Ê ÊCˆ ˆ
a Á tan A - cot Á ˜ ˜ = b Á cot Á ˜ - tan B ˜
Ë Ë 2 ¯¯ Ë Ë 2¯ ¯
LEVEL III
Ê ÊCˆˆ Ê ÊCˆ ˆ
(PROBLEMS FOR JEE ADVANCED)
Á sin A cos ÁË 2 ˜¯ ˜ Á cos ÁË 2 ˜¯ sin B ˜
aÁ - ˜ = bÁ - ˜
cos A + 2 cos C sin B Á cos A sin Ê C ˆ ˜ Á sin Ê C ˆ cos B ˜
1. Given = ÁË Á ˜
Ë 2 ¯ ˜¯ ÁË Á
Ë 2¯ ˜ ˜¯
cos A + 2 cos B sin C
cosA (sinB – sinC) + (sin2B – sin2C) = 0 Ê ÊCˆ ÊCˆˆ
Á sin A sin ÁË 2 ˜¯ - cos A cos ÁË 2 ˜¯ ˜
cosA (sinB – sinC) + 2cos(B + C) sin(B – C) = 0 aÁ ˜
Á ÊCˆ ˜
ÁË sin Á ˜ cos A ˜¯
cosA (sinB – sinC) + 2cos(p + C)sin(B – C) = 0 Ë 2¯
cosA (sinB – sinC) – 2cosAsin(B – C) = 0 Ê ÊCˆ ÊCˆˆ
Á cos B cos ÁË 2 ˜¯ - sin B sin ÁË 2 ˜¯ ˜
cosA [(sinB – sinC) – 2sin(B – C)] = 0 = bÁ ˜
Á ÊCˆ ˜
ÁË sin Á ˜ cos B
cosA = 0, [(sinB – sinC) – 2sin(B – C)] = 0 Ë 2¯ ¯˜
when cos A = 0 Ê Cˆ Ê Cˆ
a cos Á A + ˜ b cos Á B + ˜
– A = 90° Ë 2¯ Ë 2¯
- =
ÊCˆ ÊCˆ
D is right angled. cos A sin Á ˜ cos B sin Á ˜
Ë 2¯ Ë 2¯
When [(sinB – sinC) – 2sin(B – C)] = 0 Ê Cˆ Ê Cˆ
a cos Á A + ˜ b cos Á B + ˜
[(sinB – sinC) – 2(sinBcosC – cosBsinC)] = 0 Ë 2¯ Ë 2¯
- =
cos A cos B
È Ê a 2 + b2 - c2 a 2 + c2 - b2 ˆ ˘
Í (b - c) - 2 Á b -c ˜˙ = 0 Ê Cˆ Ê Cˆ
ÍÎ Ë 2ab 2ac ¯ ˙˚ sin A cos Á A + ˜ sin B cos Á B + ˜
Ë 2¯ Ë 2¯
- =
È Ê a 2 + b2 - c2 a 2 + c2 - b2 ˆ ˘ cos A cos B
Í (b - c) - Á - ˜˙ = 0
ÍÎ Ë a a ¯ ˙˚ Ê Cˆ Ê Cˆ
– 2sinA cosB cos Á A + ˜ = 2sinB cosA cos ÁË B + ˜¯
Ë 2¯ 2
È 1 2 2˘
Í (b - c) - a (a + b - c - a - c + b ˙ = 0
2 2 2 2
Î ˚ Ê Cˆ
– (sin(A + B) + sin(A – B)) cos Á A + ˜
Ë 2¯
È 2 2 2 ˘
Í (b - c) - a (b - c ) ˙ = 0 Ê Cˆ
Î ˚ = (sin(A + B) – sin(A – B)) cos Á B + ˜
Ë 2¯
[a(b – c) – 2(b2 – c2)] = 0
Properties of Triangles 415

Ï Ê Cˆ Ê Cˆ¸ Ê 1 1 ˆ
sin(A + B) Ìcos Á B + ˜ + cos Á A + ˜ ˝ = bD 2 Á +
Ó Ë 2¯ Ë 2 ¯˛ Ë s( s - b) ( s - a )(s - c) ˜¯
Ï Ê Cˆ Ê Cˆ¸ Ê ( s - a )( s - c) + s ( s - b) ˆ
= sin(A – B) Ìcos Á A + ˜ - cos Á B + ˜ ˝ = bD 2 Á
Ó Ë 2¯ Ë 2 ¯˛ Ë s( s - a)(s - c)(s - c) ˜¯
Ï Ê A+ B +Cˆ Ê B - Aˆ ¸ Ê 2 s 2 - 2 s ◊ s + ac ˆ
sin(A + B) Ì2 cos Á cos Á ˝
Ó Ë 2
˜
¯ Ë 2 ˜¯ ˛ = bD 2 Á ˜
Ë s( s - a)(s - c)(s - c) ¯
Ï Ê A+ B +Cˆ Ê B - Aˆ ¸
= sin(A – B) Ì2 sin Á sin Á ˝ Ê 2 s 2 - 2 s ◊ s + ac ˆ
Ó Ë 2
˜
¯ Ë 2 ˜¯ ˛ = bD 2 Á ˜
Ë s( s - a)(s - c)(s - c) ¯
Ï Ê A+ B +Cˆ Ê B - Aˆ ¸
sin(A – B) Ì2 sin Á sin Á ˝ =0
Ó Ë 2
˜
¯ Ë 2 ˜¯ ˛ Ê 2s 2 - 2s 2 + ac ˆ
= bD 2 Á ˜
Ë D2 ¯
Ï Ê A+ B +Cˆ Ê B - Aˆ ¸
sin(A – B) = 0, Ì2 sin Á ˜¯ sin ÁË ˜˝ = 0
Ó Ë 2 2 ¯˛ Ê ac ˆ
= bD 2 Á 2 ˜
ËD ¯
sin(A – B) = 0
= abc
(A – B) = 0
Similarly, c(rr3 + r1r2) = abc
A=B
Hence, the result.
Thus, the triangle is isosceles. Ê B -Cˆ
4. Now, (r + r1) tan Á
3. We have a(rr1 + r2r3) Ë 2 ˜¯
ÊD D D D ˆ ÊD D ˆ Ê b - cˆ
= aÁ ◊ + ◊ ÊCˆ
Ë s ( s - a ) ( s - b) ( s - c) ˜¯ =Á + ˜ ÁË
Ë s (s - a) ¯ b + c
˜¯ cot ÁË ˜¯
2
Ê 1 1 ˆ Ê1 1 ˆ Ê b - cˆ s( s - a)
= aD 2 Á +
Ë s( s - a ) ( s - b)(s - c) ˜¯ = DÁ + ˜ ÁË ˜¯
Ë s ( s - a ) ¯ b + c ( s - b)( s - c)
Ê ( s - b)( s - c) + s ( s - a ) ˆ Ê s - a + sˆ Ê b - cˆ s( s - a)
= aD 2 Á
Ë s ( s - a )( s - b)( s - c) ˜¯ = DÁ ˜ ÁË ˜¯
Ë s( s - a ) ¯ b + c ( s - b)( s - c)
Ê 2s 2 - (a + b + c) s + bc ˆ
= aD 2 Á Ê b - cˆ 1
˜ = D(2s - a) Á
Ë s ( s - a )( s - b)( s - c) ¯ Ë b + c ˜¯ s ( s - a)( s - b)( s - c)
Ê 2s 2 - 2s ◊ s + bc ˆ Ê b - cˆ 1
= aD 2 Á = D(a + b + c - a) Á ¥
˜
Ë s ( s - a)( s - b)( s - c) ¯ Ë b + c ˜¯ D
Ê 2 s 2 - 2 s 2 + bc ˆ Ê b - cˆ
= aD 2 Á = (b + c) Á
Ë D2
˜
¯ Ë b + c ˜¯

= abc = (b – c)
Thus, L.H.S.
Also, b(rr2 + r1r3)
= (b – c) + (c – a) + (a – b)
ÊD D D D ˆ
= bÁ ◊ + ◊ = 0.
Ë s ( s - b) ( s - a ) ( s - c) ˜¯
5. We have
Ê1 1 1 1 ˆ Ê Aˆ Ê Bˆ ÊCˆ
= bD Á ◊ + ◊
Ë s ( s - b) ( s - a ) ( s - c) ˜¯
tan Á ˜ tan Á ˜ tan Á ˜
Ë 2¯ Ë 2¯ Ë 2¯
+ +
(a - b)(a - c) (b - a)(b - c) (c - a )(c - b)
416 Comprehensive Trigonometry with Challenging Problems & Solutions for Jee Main and Advanced

Ê Aˆ ( s - b)( s - c) Ê Aˆ Ê Bˆ ÊCˆ
tan Á ˜ Now, cot Á ˜ ◊ cot Á ˜ ◊ cot Á ˜
Ë 2¯ s( s - a) Ë 2¯ Ë 2¯ Ë 2¯
Now, =
(a - b)(a - c) (a - b)(a - c) 1// 2
È s( s - a) s ( s - b) s ( s - c) ˘
D =Í ◊ ◊ ˙
= Î ( s - b)( s - c) ( s - a)( s - c) ( s - a)( s - b) ˚
s ( s - a )(a - b)(a - c)
1/ 2
È s3 ˘
So, LHS =Í ˙
Î ( s - a)( s - b)(s - c) ˚
DÈ 1 1
= Í - È ˘
1/ 2
s Î ( s - a )(a - b)(a - c) ( s - b)(a - b)(b - c) s4
=Í ˙
1 ˘ Î s( s - a )( s - b)(s - c) ˚
-
( s - c)(c - a)(b - c) ˚˙ È s4 ˘
1/ 2

=Í 2˙
D È (b - c) (c - a) (a - b) ˘ ÎD ˚
= + +
s(a - b)(b - c)(c - a) Î ( s - a) ( s - b) ( s - b) ˙˚
Í
s2
=
D È Â (b - c){s 2 - (b + c) s + bc} ˘ D
= Í ˙
s(a - b)(b - c)(c - a ) ÍÎ ( s - a )( s - b)( s - c) ˙˚ Hence, the area
D p
= =
s(a - b)(b - c)(c - a) A
Ê ˆ Ê Bˆ ÊCˆ
cot Á ˜ cot Á ˜ cot Á ˜
Ë 2¯ Ë 2¯ Ë 2¯
È s 2  (b - c) - s (b 2 - c 2 ) +  bc(b - c) ˘
¥Í ˙
ÍÎ ( s - a)( s - b)( s - c) ˙˚ Ê Aˆ
7. We have cos A ◊ cot Á ˜
Ë 2¯
=
D È
¥Í
 bc(b - c) ˘
s(a - b)(b - c)(c - a) Î ( s - a)( s - b)( s - c) ˙˚ Ê Aˆ
cos Á ˜
Ê Ê Aˆ ˆ Ë 2¯
È D (a - b)(b - c)(c - a) ˘ = Á1 - 2 sin 2 Á ˜ ˜
=Í ˙ Ë Ë 2 ¯¯ Ê Aˆ
Î s(a - b)(b - c)(c - a) ( s - a)( s - b)( s - c) ˚ sin Á ˜
Ë 2¯
D
= Ê Aˆ
s(a - b)(b - c)(c - a) = cos Á ˜ - sin A
Ë 2¯
ÈD˘ Now, a, b, c are in A.P.
=Í 2˙
ÎD ˚
sinA, sinB, sinC are also in A.P.
1
= Ê Aˆ Ê Bˆ ÊCˆ
D and cot Á ˜ cot Á ˜ cot Á ˜ Œ A.P.
Ë 2¯ Ë 2¯ Ë 2¯
Hence, the result.
Thus, their differences are also in A.P.
area of the incircle
6. We have 8. Let BC = a = c, CA = b = 10c and AB = c
area of triangle ABC
Clearly, a2 + b2 = 101c2
p r2 Applying, sine rule, we get,
D
sin A sin B sin C
= =
pr 2 c 10c c
D sin A sin B sin C
= = = k (say)
pr p D
2
pD 2 c 10c c
= = ¥ 2 = 2
D D s s
Properties of Triangles 417

cot C 8 Ê abc ˆ
2
Now, = ¥Á
cot A + cot B ˜ ¥ Ds
abc Ë 4D ¯
cos C 8abc s
= ¥
= sin C 16 D
sin( A + B)
sin A sin B abc
=
cos C 2r
11. Clearly, r1 + r2 + r3 = r + 4R
= sin C
sin C and r1r2 + r2r3 + r3r1 = s2
sin A sin B
D2
=
cos C
¥ sin B and r1r2r3 = = Ds = s2r
( s - a)( s - b)( s - c)
sin C
Hence, the required equation is
sin B
= cos C ¥ x3 – (r + 4R)x2 + s2x – rs2 = 0
sin C
12. Let r be the in-radius and R be the circum radius of
Ê a 2 + b 2 - c 2 ˆ sin B
=Á and equilateral triangle
2ab ˜ ¥ sin C
Ë ¯
D 3a 2 2 a
Now, r = = = ¥ =
Ê c 2 + 100c 2 - c 2 ˆ 10ck s 4 3a 2 3
= k2 Á ˜ ¥ ck
Ë 2 ◊ ck ◊ 10ck ¯
abc a2 a
and R = = = .
= 5 ¥ 10 = 50 4D 3a 2
3

9. Do yourself. 4
10. We have
a
A
I3 I2 r 2 3 1
Thus, = =
I R a 2
B C 3
Hence, the result.
I1 13. We have
A
Area of the triangle DI1I2I3 I3 I2
I
1
= ¥ (product of two sides) B C
2 ¥ (sine of included angles)
I1
1 Ê Ê Bˆˆ Ê Ê C ˆˆ Ê Aˆ
= ¥ Á 4 R cos Á ˜ ˜ ¥ Á 4 R cos Á ˜ ˜ ¥ sin Á 90∞ - ˜
2 Ë Ë ¯
2 ¯ Ë Ë ¯
2 ¯ Ë 2¯ Hence, the area

Ê Aˆ Ê Bˆ ÊCˆ =
1
¥ (product of two sides)
= 8R 2 ¥ cos Á ˜ cos Á ˜ cos Á ˜
Ë 2¯ Ë 2¯ Ë 2¯ 2 ¥ (sine of including angles)
s(s - a) s ( s - b) s ( s - c) 1 Ê Ê Bˆˆ Ê Ê C ˆˆ Ê Aˆ
= 8R 2 ¥ ¥ ¥ = ¥ 4 R cos Á ˜ ˜ ¥ Á 4 R cos Á ˜ ˜ ¥ sin Á 90∞ - ˜
bc ca ab 2 ÁË Ë 2 ¯¯ Ë Ë 2 ¯¯ Ë 2¯
8R 2 1 Ê Ê Bˆˆ Ê Ê C ˆˆ Ê Aˆ
= ¥ s s ( s - a)( s - b)( s - c) = ¥ Á 4 R cos Á ˜ ˜ ¥ Á 4 R cos Á ˜ ˜ ¥ cos Á ˜
abc 2 Ë Ë 2 ¯¯ Ë Ë 2 ¯¯ Ë 2¯

8R 2 s Ê Aˆ Ê Bˆ ÊCˆ
= ¥D = 8R 2 ¥ cos Á ˜ cos Á ˜ cos Á ˜
abc Ë 2¯ Ë 2¯ Ë 2¯
418 Comprehensive Trigonometry with Challenging Problems & Solutions for Jee Main and Advanced

14. Hence, the circum-radius 16. We have a3 cos(B – C)


I 2 I3 = a2 k sin A cos(B – C)
=
2sin( I 2 I1 I 3 ) = a2k sin(B + C) cos(B – C)

Ê Aˆ a2k
4 R cos Á ˜ = [2sin (B + C) cos (B – C)]
Ë 2¯ 2
=
Ê Aˆ a2k
2 sin Á 90∞ - ˜
Ë 2¯ = [sin2B + sin2C]
2
Ê Aˆ k3
4 R cos Á ˜
Ë 2¯ = [sin2A sin2B + sin2A sin2C]
= 2
Ê Aˆ
2 cos Á ˜
Ë 2¯ = k3 [sin2A sinB cosB + sin2A sinC cosC]

= 2R Similarly, b3cos(C – A)

= k3[sin2B sinC cosC + sin2B sinA cosA]


A
I3 I2
and c3 cos(A – B)
I

B C = k3[sin2C sinA cosA + sin2C sinB cosB]

Adding all we get,


I1
k3[sinA sinB(sinA cosB + cosA sinB)
–IBI2, –ICI1 are right angles
+ sinB sinC (sinB cosC + cosB sinC)
Here, II1 is the diameter of the circum-circle of the
triangle D BCI1 + sinC sinA (sinC cosA + cosC sinA)]

BC a = k3 [sinA sinB sin(A + B) + sinB sinC


Thus, II1 = =
sin(–BI1C ) Ê Aˆ sin(B + C) + sinC sinA sin(C + A)]
sin Á 90∞ - ˜
Ë 2¯
= k3[sinA sinB sinC + sinA sinB sinC + sinA sinB sinC]
a
=
Ê Aˆ = k3[3sinA sinB sinC]
cos Á ˜
Ë 2¯
= 3(k sin A)(k sin B)(k sin C)
2 R sin A
= = 3abc
Ê Aˆ
cos Á ˜ 17. Do yourself
Ë 2¯
18. We have c cos(A + q) + a cos (C + q)
Ê Aˆ Ê Aˆ = c(cos A cos q + sin A sin q)
2 R + 2 sin Á ˜ cos Á ˜
Ë 2¯ Ë 2¯
= + a(cos C cos q – sinC sin q)
Ê Aˆ
cos Á ˜
Ë 2¯ = cos q(c cos A + a cos C)
Ê Aˆ + sin q(c sin A – a sin C)
= 4 R sin Á ˜
Ë 2¯
= b cos q + sin q(k sin C sin A – k sin C sin A)
Ê Bˆ
Similarly, II2 = 4 R sin Á ˜ , = b cos q + sin q ¥ 0
Ë 2¯
= b cos q
ÊCˆ
II3 = 4 R sin Á ˜
Ë 2¯
Properties of Triangles 419

19. Let BC = a, AC = b, AB = c 3s - 2 s 3
A
≥ ( s - a)( s - b)( s - c)
3
a/2 a/2 s 3
c b ≥ ( s - a)( s - b)( s - c)
3
a
3
Ê sˆ
B D C ÁË ˜¯ ≥ ( s - a)( s - b)( s - c)
a 3

Clearly, (D ABC) = ar(D ABD) + ar(D ACD) s4 ≥ 27 ¥ s(s – a)(s – b)(s – c)


s4 ≥ 27 ¥ D2
1 1 Ê Aˆ 1 Ê Aˆ
bc sin A = ca sin Á ˜ + ba sin Á ˜
2 2 Ë 2¯ 2 Ë 2¯ s2 ≥ 3 3 ¥ D
Ê Aˆ Ê Aˆ s2
bc sin A = ca sin Á ˜ + ba sin Á ˜
Ë 2¯ Ë 2¯ D£
3 3
1 Ê Aˆ 1 Ê 1 1ˆ 22. Do yourself.
cos Á ˜ = Á + ˜
a Ë 2 ¯ 2 Ë b c¯ 23. We have 2a + 2b + 2g = a + b + c = 2s
1 Ê Bˆ 1 Ê 1 1ˆ a+b+g=s
Similarly, cos Á ˜ = Á + ˜
b Ë 2 ¯ 2 Ë c a¯ and a = s – a, b = s – b, g = s – c
A
1 Ê C ˆ 1 Ê 1 1ˆ
And cos Á ˜ = Á + ˜
g Ë 2 ¯ 2 Ë a b¯
a a
A/2
Adding, we get,
F E
1 Ê Aˆ 1 Ê Bˆ 1 ÊCˆ
cos Á ˜ + cos Á ˜ + cos Á ˜ r I r
a Ë 2¯ b Ë 2¯ g Ë 2¯ g
b
1 Ê 1 1ˆ 1 Ê 1 1ˆ 1 Ê 1 1 ˆ r
= Á + ˜+ Á + ˜+ Á + ˜ B/2 C/2
2 Ë a b¯ 2 Ë b c¯ 2 Ë c a¯ B b D g C

1 Ê 2 2 2ˆ
= Á + + ˜ D2 s ( s - a)( s - b)( s - c)
2 Ë a b c¯ Now, r = 2
=
s 2
s2
Ê 1 1 1ˆ
=Á + + ˜ ( s - a)( s - b)( s - c)
Ë a b c¯ =
s
1 Ê Aˆ 1 Ê Bˆ 1 ÊCˆ
Hence, cos Á ˜ + cos Á ˜ + cos Á ˜ a ◊ b ◊g
AD Ë 2 ¯ BE Ë 2 ¯ CF Ë 2¯ =
a +b +g
1 1 1
= + + 24. In triangle ALM, we have
a b c A
20. Do yourself
21. Clearly, 2s = a + b + c B C
L M
As we know that, A.M ≥ G.M r

( s - a ) + ( s - b) + ( s - c ) 3 I
≥ ( s - a)( s - b)( s - c)
3
B C
3s - (a + b + c) 3
≥ ( s - a)( s - b)( s - c)
3 x AM AL
AL = x ◊ = =
sin A sin B sin C
420 Comprehensive Trigonometry with Challenging Problems & Solutions for Jee Main and Advanced

sin B bx a
AM = x ◊ = = 2 tan A
sin A a x
b c
Similarly, =2 tan B, = 2 tan C
r = ex – radius of DALM y z
As we know that,
Ê x + AL + AM ˆ Ê Aˆ
r= Á ˜ tan Á ˜ Tan A + tan B + tan C = tan A tan B tan C
Ë 2 ¯ Ë 2¯
a b c a b c
Ê cx bx ˆ + + = ◊ ◊
x+ + 2x 2 y 2z 2x 2 y 2z
Á
r =Á a 2 ˜ tan Ê A ˆ
˜ ÁË ˜¯ a b c abc
2 2
Á ˜ + + =
Ë ¯ x y z 4 xyz
Ê a + b + cˆ Ê A ˆ sx Ê Aˆ Hence, the result.
r =Á x tan Á ˜ = tan Á ˜
Ë 2a ˜¯ Ë 2¯ a Ë 2¯ 26. Given sinA sinB sinC = p
Ê A ˆ sx Ê Aˆ And cosA cosB cosC = q
( s - a) tan Á ˜ = tan Á ˜
Ë 2¯ a Ë 2¯ Here, A + B + C = p
sx tan (A + B + C) = tan(p) = 0
(s – a) =
a p
tan A + tan B + tan C = tan A ◊ tan B ◊ tan C =
sx sx q
Similarly, (s – b) = , (s – c) = Also, (1 – tan A tan B – tan B tan C – tan C tan A)
b c
Adding, we get cos( A + B + C ) 1
= =-
sx sx sx cos A cos B cos C q
(s – a) + (s – b) + (s – c) = + +
a b c Thus, tanA tanB + tanB tanC + tanC tanA
sx sx sx
3s – (a + b + c) = + + 1 Ê q + 1ˆ
a b c =1+ =
q ÁË q ˜¯
sx sx sx
3s – 2s = + + Hence, the required equation is
a b c
sx sx sx Ê pˆ Ê q + 1ˆ Ê pˆ
+ + =s x3 - Á ˜ x 2 + Á x-Á ˜ =0
a b c Ë q¯ Ë q ˜¯ Ë q¯
x x x qx3 – px2 + (q + 1)x – p = 0
+ + =1
a b c 27. Let the medians AD, BE and CF meet at O such that
Hence, the result. –BOC = a, –AOC = b, –AOB = g
25. It is given that, x = OD = R cos A A

A
b E
F
O
g
O a

B C B D C
D
Let AD = p1, BE = p2, CF = p3
a
= ◊ cos A 2 2 2
2sin A Clearly, OA = p , OB = p , OC = p
3 1 3 2 3 3
a From DAOC, we get,
=
2 tan A
Properties of Triangles 421

OA2 + OC 2 - AC 2 b 2 + c 2 - 5a 2
cos b = Similarly, cot a =
2OA ◊ OC 12D
4 2 4 2 b 2 + a 2 - 5c 2
p1 + p3 - b 2 and cot g =
fi cos b = 9 9 12D
2 2
2 ◊ p1 ◊ p3 Now, cota + cotb + cotg
3 3

4 p12 + 4 p32 - 9b 2 3(a 2 + b 2 + c 2 )


= -
fi cos b = …(1) 12D
8 p1 p3

1 (a 2 + b 2 + c 2 )
Also, ar(DAOC) = OA ◊ OC ◊ sin b = -
2 4D
1 1 (a 2 + b 2 + c 2 )
fi D = OA ◊ OC ◊ sin b Also, cot A + cot B + cot C =
3 2 4D
1 12 2 Hence,
fi D= p1 ◊ p3 ◊ sin b
3 23 3 Cot A + cot B + cot C + cota + cotb + cotg = 0
3D 28. Let AO be the perpendicular from A on BC. When AO
fi sin b = …(ii)
2 p1 p3 is produced, it meets the circumscribing circle at D
such that OD = a
Dividing (i) by (ii), we get,
A
4 p12 + 4 p32 - 9b 2
cot b = …(iii)
12 D
Again AD is the median of DABC
O
So, AB2 + AC2 + 2BD2 + 2AD2 B C
C B
2
Ê aˆ D
fi b2 + c2 = 2 ◊ Á ˜ + 2 p1
2
Ë 2¯
Since angles in the same segment are equal.
a2
fi b2 + c2 = = + 2 p12 Thus, –ADB = –ACB = –C
2
2b 2 + 2c 2 - a 2 and –ADC = –ABC = –B
fi p1 =
2
4 OB
From DBOD, tan(C) = ….(i)
2 a + 2c - b
2 2 2 OD
similarly, p2 =
2
4 OC
From DCOD, tan(B) = ….(ii)
2a 2 + 2b 2 - c 2 OD
and p3 =
2
4 Adding (i) and (ii), we get
Now, from (iii), we get, OB + OC BC a
tan B + tan C = = = ….(iii)
4 p12 + 4 p32 - 9b 2 OD OD a
cot b =
12D b
similarly, tan C + tan A = ….(iv)
b
(2b 2 + 2c 2 - a 2 ) + (2a 2 + 2c 2 - b 2 ) - 9b 2
=
12D c
and tan A + tan B = ….(v)
g
a 2 + c 2 - 5b 2
=
12D
422 Comprehensive Trigonometry with Challenging Problems & Solutions for Jee Main and Advanced

Adding (iii), (iv) and (v), we get, 30. Here, AD is the internal bisector of the angle A
A
a b c
+ + = 2(tan A + tan B + tan C)
a b g
Hence, the result. F
I
E

29. Let H be the othocentre of the triangle ABC such that


HA = p, HB = q, HC = r
A A D C

BD c
p Clearly, =
F E DC b

q r DC b
H fi =
BD c
A D C
DC b b+c
fi +1 = +1 =
BD c c

–HBD = –EBC = 90° – C DC + BD b + c


fi =
BD c
–HCD = –FCB = 90° – B
a b+c
–BHC = 180° – (–HBD + –HCD) fi =
BD c
= 180° – (90° – C + 90° – B) BD a
fi =
= (B + C) = 180° – A c b+c

Similarly, –AHC = 180° – B BF c


Similarly, =
a a+b
and –AHB = 180° – C
1
◊ BD ◊ BF ◊ sin B
Now, ar ( DBDF ) 2
Now, =
ar ( DABC ) 1
ar(DBHC) + ar(DCHA) + ar(DAHB) = ar(DABC) a c sin B
2
1 1
fi qr sin (–BHC) + pr sin (–AHC) BD ◊ BF BD BF ac
2 2 = = ◊ =
ac a c (a + b)(b + c)
1
+ pq sin(–AHB) = D ar ( DCDE ) ac
2 Similarly, =
ar ( DABC ) (a + b)(b + c)
fi qr sin (180° – A) + pr sin (180° – B)
ar ( DAEF ) bc
+ pq sin (180° – C) = 2D and =
ar ( D ABC) (b + c)(a + c)
fi qr sin A + rp sin B + pq sin C = D ar ( DDEF )
Thus,
Ê a ˆ Ê b ˆ Ê c ˆ abc ar ( DABC )
fi qr Á ˜ + rp Á ˜ + pq Á ˜ =
Ë 2R ¯ Ë 2R ¯ Ë 2R ¯ 4R DABC - ( DBDF + DCDE + DAEF )
=
fi aqr + brp + cpq = abc D ABC

aqr brp cpq abc DBDF DCDE DAEF


fi + + = = 1- - -
pqr pqr pqr pqr DABC DABC DABC

a b c abc ac ab bc
fi + + = = 1- - -
p q r pqr (a + b)(b + c) (a + c)(b + c) (a + b)(a + c)
Properties of Triangles 423

(a + b)(b + c)(c + a) - {ac(a + c) + ab(a + b) + bc(b + c)} 4 R sin b sin g (3 - 4 sin 2 g ) cos(30∞ - g )
= =
(a + b)(b + c)(c + a) 2 sin(60∞ - g ) cos(30∞ - g )
2abc
= 4 R sin b sin g (3 - 4 sin 2 g ) cos(30∞ - g )
(a + b)(b + c)(c + a) =
sin(90∞ - 2g ) + sin(30∞)
ar ( DDEF ) 2abc
= 4 R sin b sin g (3 - 4 sin 2 g ) cos(30∞ - g )
ar ( DABC ) (a + b)(b + c)(c + a) =
1
2Dabc cos(2g ) +
ar (DDEF) = 2
(a + b)(b + c)(c + a)
8R sin b sin g (3 - 4 sin 2 g ) cos(30∞ - g )
Hence, the result. =
2 cos(2g ) + 1
31. Since A + B + C = 180°
A 8R sin b sin g (3 - 4 sin 2 g ) cos(30∞ - g )
=
2(1 - 2 sin 2 g ) + 1
a a a
8R sin b sin g (3 - 4 sin 2 g ) cos(30∞ - g )
=
R Q (3 - 4 sin 2 g )
= 8R sin b sing in g (30° – g)
b g
b P g
Hence, the result.
b g
a
B C 32. Since AD is the median, BD = DC =
2
3a + 3b + 3g = 180° A
Also, –DAE = –CAE =
a + b + g = 60° 2
A
Clearly, –ARB = 180° – (a + b)
Applying sine rule in triangle ARB
AR c c b
=
sin b sin(180∞ - (a + b ))
AR c a/4 a/4
=
sin b sin(a + b ) B a/2 D E C
a/2
c
sin(a + b ) Applying cosine rule in triangle ABD, we get,
2R sin C sin b
Ê A ˆ AB + AD - BD
2 2 2
=
sin(a + b ) cos Á ˜ =
Ë 3¯ 2 AB ◊ AD
2 R sin(3g )sin b
= Ê aˆ
2
sin(a + b ) c2 + b2 - Á ˜
Ê Aˆ Ë 2¯ 4b 2 + 4c 2 - a 2
2 R sin(3g )sin b cos Á ˜ = = ….(i)
= Ë 3¯ 2bc 8bc
sin(60∞ - g )
Applying cosine rule in triangle ABC, we get,
2 R sin b sin g (3 - 4 sin 2 g )
= b2 + c2 - a 2
sin(60∞ - g ) cos A =
2bc
2 R sin b sin g (3 - 4 sin 2 g ) cos(30∞ - g )
= ◊ Ê Aˆ Ê Aˆ b + c - a
2 2 2
sin(60∞ - g ) cos(30∞ - g ) 4 cos3 Á ˜ - 3 cos Á ˜ = …. (ii)
Ë 3¯ Ë 3¯ 2bc
424 Comprehensive Trigonometry with Challenging Problems & Solutions for Jee Main and Advanced

Subtracting (ii) from (i), we get, Ê r1 + r2 + r3 ˆ 3


ÁË ˜ ≥
Ê Aˆ Ê Aˆ 3 ¯ 1 1 1
4 cos Á ˜ - 4 cos3 Á ˜ + +
Ë 3¯ Ë 3¯ r1 r2 r3

4b 2 + 4c 2 - a 2 b 2 + c 2 - a 2 Ê r1 + r2 + r3 ˆ 3
= - ÁË 3 ˜¯ ≥ 1 = 3r
8bc 2bc
r
4b 2 + 4c 2 - a 2 - 4b 2 - 4c 2 + 4a 2 Ê r1 + r2 + r3 ˆ
= ÁË ˜¯ ≥ 9
8bc r
3a 2
= Ê sin 2 A + sin A + 1ˆ
8bc 4. We have Á
sin A ˜
Ë ¯
Ê Aˆ Ê Ê A ˆ ˆ 3a
2
4 cos Á ˜ Á1 - cos 2 Á ˜ ˜ =
Ë 3¯Ë Ë 3 ¯ ¯ 8bc Ê 1 ˆ
= Á sin A + + 1˜
Ë sin A ¯
Ê Aˆ Ê A ˆ 3a
2
4 cos Á ˜ sin 2 Á ˜ =
Ë 3¯ Ë 3 ¯ 8bc Ê 1 ˆ
= Á sin A + ˜ +1
Ë sin A ¯
Ê Aˆ Ê A ˆ 3a
2
cos Á ˜ sin 2 Á ˜ =
Ë 3¯ Ë 3 ¯ 32bc ≥2+1=3
Hence, the minimum value is 3.
INTEGER TYPE QUESTIONS 5. We have
1. As we know that, in a right angled triangle 1
2 2 2 2 {(r 2 + r12 + r22 + r32 ) + (a 2 + b 2 + c 2 )}
a + b + c = 8R 8R 2
Ê a 2 + b2 + c2 ˆ 1
= ¥ 16 R 2
Á ˜ =8 =8 8R 2
Ë R2 ¯
= 2.
Êr +r +r ˆ
2. We have Á 1 2 3 ˜ 3 -1
Ë r ¯ 6. By m-n theorem cot q =
2
Ê Aˆ Ê Aˆ Ê Aˆ
4 R cos Á ˜ cos Á ˜ cos Á ˜
Ë 2¯ Ë 2¯ Ë 2¯
45°
Ê Ê Aˆ Ê Bˆ Ê Cˆˆ 30°
ÁË tan ÁË 2 ˜¯ + tan ÁË 2 ˜¯ + tan ÁË 2 ˜¯ ˜¯
=
Ê Aˆ Ê Bˆ Ê Cˆ
4 R sin Á ˜ sin Á ˜ sin Á ˜ L
Ë 2¯ Ë 2¯ Ë 2¯
B a/2 D a/2 C
Ê Ê Aˆ Ê Bˆ Ê C ˆˆ
ÁË tan ÁË 2 ˜¯ + tan ÁË 2 ˜¯ + tan ÁË 2 ˜¯ ˜¯ 2
= So, tanq =
Ê Aˆ Ê Bˆ ÊCˆ ( 3 - 1)
tan Á ˜ tan Á ˜ tan Á ˜
Ë 2¯ Ë 2¯ Ë 2¯ 2 ( 3 - 1)
sinq = and cosq =
=1 (8 - 2 3 ) (8 - 2 3 )
3. As we know that From DADC,
1 1 1 1
+ + = a /c AD AD
r1 r2 r3 r = =
sin(45∞) sin(p - (q + 45∞) 1
Using, A.M ≥ G.M (sin q + cos q )
2
Properties of Triangles 425

a /c AD ÊCˆ
= cot Á ˜ = 1
sin(45∞) 1 Ë 2¯
(sin q + cos q )
2
C = 90°
a AD Hence, the area of the triangle
=
2 (sin q + cos q )
1
= a b sin (90°)
a (8 - 2 3 ) 1 2
= ¥
2 ( 3 + 1) 11 - 6 3 1
= ¥6¥3 = 9
2
a (8 - 2 3 ) 1
= ¥ 9. We have
2 (4 - 2 3 ) 11 - 6 3 10 b c
= =
a (8 - 2 3 ) 3 sin(50∞) sin(70∞)
= =1
2 (8 - 2 3 ) 20 b c
= =
3 sin(50∞) sin(70∞)
a=2
Now, perimeter
Therefore, the length of the side BC is 2.
= 10 + b + c
7. We have
(r1 + r2)(r1 + r2)(r1 + r2) = 4 R s2 = 10 +
20
sin(50∞) +
20
sin(70∞)
3 3
(r1 + r2 )(r1 + r2 )(r1 + r2 )
Thus, =4 =4 20
R s2 = 10 + [sin(50°) + sin(70°)]
3
4
8. We have cos (A – B) = 20
5 = 10 + [cos(40°) + cos(20°)]
3
Ê A - Bˆ 20
1 - tan 2 Á
Ë 2 ˜¯ 4 = 10 + 10 + ¥ 2 cos (30°) cos (10°)
= 3
Ê A - Bˆ 5
1 + tan 2 Á
Ë 2 ˜¯ 20 3
= 10 + 10 + ¥2¥ ¥ cos (10°)
3 2
Ê Ê A - Bˆˆ Ê Ê A - Bˆˆ
4 Á1 + tan 2 Á = 5 Á1 - tan 2 Á
Ë Ë 2 ˜¯ ˜¯ Ë Ë 2 ˜¯ ˜¯ = 10 + 20 cos (10°)
Thus, x = 10, y = 20, z = 10
Ê A - Bˆ
9 tan2 Á =1 = 1
Ë 2 ˜¯ Ê x + y + zˆ
Hence, the value of Á
Ë y ˜¯ is 2
Ê A - Bˆ 1
tan2 Á = 10. We have
Ë 2 ˜¯ 9
a cot A + b cot B + c cot C
Ê A - Bˆ 1 (r + R)
tan Á =
Ë 2 ˜¯ 3
cot A + cot B + cot C
=
Ê a - bˆ ÊCˆ 1 (r + R)
ÁË ˜¯ cot ÁË ˜¯ =
a+b 2 3 2(r + R)
=
1 ÊCˆ 1 (r + R)
cot Á ˜ =
3 Ë 2¯ 3 =2
426 Comprehensive Trigonometry with Challenging Problems & Solutions for Jee Main and Advanced

HINTS & SOLUTIONS OF PAST 3. By the law of cosines,


IIT-JEEQUESTIONS
a 2 + b2 - c2
cos C =
1 1 1 2ab
1. We have, D = ap1 = bp2 = cp3
2 2 2 a 2 + b2 - c2
1 a 1 b 1 c fi cos(60∞) =
fi = , = , = 2ab
p1 2D p2 2D p3 2D
1 a 2 + b2 - c2
fi =
1
+
1
-
1 2 2ab
Now,
p1 p2 p3
fi ab = a 2 + b 2 - c 2
1
= (a + b - c ) fi c 2 = a 2 + b 2 - ab
2D
( ) ( )
2
fi c2 = 1 + 3 + 22 - 2 1 + 3
( a + b )2 - c 2
=
2D ( a + b + c ) fi c2 = 1 + 3 + 2 3 + 4 - 2 - 2 3 = 6
a 2 + b 2 - c 2 + 2ab fi c= 6
=
2D (a + b + c ) sin A sin B sin C
From sine law, = =
2ab cos C + 2ab a b c
=
2 (a + b + c ) D a
fi sin A = .sin C
ab (cos C + 1) c
=
(a + b + c ) D
fi sin A =
( 3 +1 )¥ 3
=
3 +1
= sin(105∞)
2ab Ê Cˆ 6 2 2 2
= ¥ cos 2 Á ˜
(a + b + c ) D Ë 2¯
fi A = 105∞, B = 180 - ( A + C ) = 15∞, c = 6
p D ac ac
2. Let –APD = q , as –PAO = –PDO = 4. We have, r = = =
2 s 2s a + b + c
–AOD = p - q
ac ( a + c - b )
By the law of cosines, =
( a + c )2 - b 2
OA + OD - AD = 2 (OA)(OD ) cos (p - q )
2 2 2
ac ( a + c - b )
=
fi AD 2 = 2r 2 + 2r 2 cosq a 2 + c 2 + 2ac - b 2
Êqˆ
AD 2 = 2r 2 (1 + cos q ) = 4r 2 cos 2 Á ˜
Ë 2¯ =
1
2
(
(a + c - b) ∵ a 2 + c 2 = b2 )
Since ABCD is a cyclic quadrilateral, so 1
fi r= (a + c - b)
–QRS = p - q 2
2Êp qˆ 2 Êqˆ fi 2r = ( a + c - b ) = AB + BC - AC .
Thus, BC = 4r cos Á - ˜ = 4r sin Á ˜
2 2 2
Ë 2 2¯ Ë 2¯
Hence, the result.
Therefore, b2 + c2 - a 2
Ê 5. We have, cos A = ........(i)
Êqˆ Ê q ˆˆ 2bc
AD 2 + BC 2 = 4r 2 Á cos 2 Á ˜ + sin 2 Á ˜ ˜ = 4r 2
Ë Ë 2¯ Ë 2¯¯
a 2 + b2 - c2
fi cos C = ........(ii)
Similarly, we can easily prove that, 2ab
AB 2 + CD 2 = 4r 2 AC b 2b
From DADC , cos C = = = ......(iii)
DC a / 2 a
Hence, AB + CD = AD + BC
2 2 2 2
Properties of Triangles 427

From (ii) and (iii), we get,


Ê Cˆ Ê Ê A - Bˆ Ê Cˆˆ 1
fi 2 sin Á ˜ Á cos Á ˜ - sin Á ˜ ˜ =
a +b -c
2 2
2b 2 Ë 2¯Ë Ë 2 ¯ Ë 2 ¯¯ 2
=
2ab a Ê Cˆ Ê Ê A - Bˆ Ê A + Bˆˆ 1
fi 2 sin Á ˜ Á cos Á - cos Á =
fi a 2 - c 2 = 3b 2 Ë 2¯Ë Ë 2 ˜¯ Ë 2 ˜¯ ˜¯ 2
From (i), we get, Ê Cˆ Ê Aˆ Ê Bˆ 1
fi 2 sin Á ˜ ¥ 2 sin Á ˜ sin Á ˜ =
b - 3b
2
2b
2
b 2 Ë 2¯ Ë 2¯ Ë 2¯ 2
cos A = =- =-
2bc 2bc c Ê Aˆ Ê Bˆ Ê Cˆ 1
fi sin Á ˜ sin Á ˜ sin Á ˜ =
Ê 2b ˆ Ê b ˆ 2 c - a
Thus, cos A.cos C = Á ˜ Á - ˜ =
2 2
( ) Ë 2¯ Ë 2¯ Ë 2¯ 8

Ë a ¯ Ë c¯ 3ac It is possible only when,


Ê Aˆ 1 Ê Bˆ 1 Ê Cˆ 1
sin Á ˜ = ,sin Á ˜ = ,sin Á ˜ =
6. Let –A = –B - a and –C = –B + a Ë 2¯ 2 Ë 2¯ 2 Ë 2¯ 2
We have –A + –B + –C = 180∞ Ê Aˆ p Ê B ˆ p Ê C ˆ p
fi Á ˜ = ,Á ˜ = ,Á ˜ =
fi –B = 60∞ Ë 2¯ 6 Ë 2¯ 6 Ë 2¯ 6
b c p p p
From sine laws, = fi A= , B = ,C =
sin B sin C 3 3 3
c Ê 2 ˆ Ê 3ˆ 1 Thus, D is equilateral.
fi sin C = .sin B = Á .˜ Á ˜ =
b Ë 3 ¯Ë ¯2 2 b+c c+a a+b 2 (a + b + c )
10. We have, = = =
11 12 13 36
fi C = 45∞
(a + b + c )
= =l
Thus, –A = 180∞ - ( B + C ) 18
= 180∞ - (60∞ + 45∞) = 75∞ . Thus, a = 7l , b = 6l & c = 5l
7. No questions asked in 1982. b2 + c2 - a 2
fi cos A =
D D D 2bc
8. We have, r1 = , r2 = , r3 =
s-a s-b s-c 36l 2 + 25l 2 - 49l 2 1
= =
Given r1 , r2 , r3 ΠH .P 2 (6l )(5l ) 5
D D D
fi , , ΠH .P 19 5
s-a s-b s-c Similarly, cos B = , cos C =
35 7
1 1 1
fi , , ΠH .P 1 19 5
s-a s-b s-c fi cos A : cos B : cos C = : :
5 35 7
fi s - a, s - b, s - c ΠA.P
fi cos A : cos B : cos C = 7 : 19 : 25
fi - a, - b, - c ΠA.P cos A cos B cos C
Thus, = = .
fi a, b, c ΠA.P 7 19 25
Hence, the result. 11.
3 12. Given cot A , cot B , cot C ΠA.P
9. We have, cos A + cos B + cos C =
2 cos A cos B cos C
fi , , ΠA.P
Ê A + Bˆ Ê A - Bˆ 3 sin A sin B sin C
fi 2 cos Á ˜ cos Á ˜ + cos C =
Ë 2 ¯ Ë 2 ¯ 2

(b 2
+ c2 - a2 ) , (c 2
+ a 2 - b2 ) , (a 2
+ b2 - c2 ) ΠA.P
Êp Cˆ Ê A - Bˆ Ê Cˆ 3
fi 2 cos Á - ˜ cos Á + 1 - 2 sin 2 Á ˜ = 2abc 2abc 2abc
Ë 2 2¯ Ë 2 ˜¯ Ë 2¯ 2
Ê Cˆ Ê A - Bˆ Ê Cˆ 3
- 2 sin 2 Á ˜ = - 1 =
1

(b 2
+ c 2 + a 2 - 2a 2 ) , (c 2
+ a 2 + b 2 - 2b 2 ),
fi 2 sin Á ˜ cos Á
Ë 2¯ Ë 2 ˜¯ Ë 2¯ 2 2 2abc 2abc
428 Comprehensive Trigonometry with Challenging Problems & Solutions for Jee Main and Advanced

(a 2
+ b 2 + c 2 - 2c 2 ) Œ A.P Êaˆ Ê bˆ Êg ˆ
= 4 sin Á ˜ sin Á ˜ sin Á ˜
Ë 2¯ Ë 2¯ Ë 2¯
2abc
It will be greatest, when
fi (b + c + a - 2 a ) , ( c + a
2 2 2 2 2 2
)
+ b 2 - 2b 2 ,
a b g
= = fia = b =g
(a + b + c - 2c ) ΠA.P
2 2 2 2 2 2 2
2p
fi (-2a ) , (-2b ) , (-2c ) ΠA.P
2 2 2
Thus, a =
3
=b =g

fi a 2 , b 2 , c 2 ΠA.P Therefore, the least value of M is

Hence, the result. 1Ê Ê 2p ˆ ˆ 3


= - Á 3 sin Á ˜ ˜ = -
3Ë Ë 3 ¯¯ 2
13. Let x = a + 2a, y = 2a + 3, z = a + 3a + 8
2 2

Here, x > 0, y > 0 and z > 0 15. We have cos A cos B + sin A sin B sin C = 1
1 - cos A cos B
since z = a + 3a + 8 > 0 for every a in R
2
fi = sin C
sin A sin B
Ê 3ˆ
fi a > 0 Á∵ a < -2, a > 0 and a > - ˜ 1 - cos A cos B
Ë 2¯ fi = sin C £ 1
sin A sin B
Also, z - x = a + 8 > 0, z - y = a 2 + a + 5 > 0
fi 1 - cos A cos B £ sin A sin B
Thus, x+ y> z
fi 1 £ sin A sin B + cos A cos B
fi (
a 2 + 3a + 8 < a 2 + 2a + ( 2a + 3) ) fi cos ( A - B ) ≥ 1
fi a>5
fi cos ( A - B ) = 1
fi a Œ (5, •)
fi cos ( A - B ) = cos (0)
1Ê Ê pˆ Ê pˆ Ê p ˆˆ
14. Let M = Á cos Á a + ˜ + cos Á b + ˜ + cos Á g + ˜ ˜ fi A- B = 0
3Ë Ë 2¯ Ë 2¯ Ë 2 ¯¯
fi A=B
1
= - (sin a + sin b + sin g ) 1 - cos A cos B
3 Therefore, sin C =
sin A sin B
M will be least, when (sin a + sin b + sin g ) is provid- 1 - cos A cos A 1 - cos 2 A sin 2 A
ing us the greatest value. = = = =1
sin A sin A sin 2 A sin 2 A
Let z = sin a + sin b + sin g fi C = 90∞
= sin a + sin b + sin ( 2p - (a + b )) Hence, A = 45∞ = B , C = 90∞
= sin a + sin b - sin (a + b ) a b c
Now, = =
Êa + bˆ Êa - bˆ Êa + bˆ Êa + bˆ sin A sin B sin C
= 2 sin Á ˜ cos Á ˜ - 2 sin Á ˜ cos Á
Ë 2 ¯ Ë 2 ¯ Ë 2 ¯ Ë 2 ˜¯ a b c
fi = =
sin 45∞ sin 45∞ sin 90∞
Êa + bˆ Ê Êa - bˆ Ê a + b ˆˆ
= 2 sin Á - cos Á
Ë 2 ˜¯ ˜¯
˜ Á cos Á ˜ a b c
Ë 2 ¯Ë Ë 2 ¯ fi = =
1 1 1
Êa + bˆ Êaˆ Ê bˆ
= 2 sin Á ¥ 2 sin Á ˜ sin Á ˜
Ë 2 ˜¯
2 2
Ë 2¯ Ë 2¯
a b c
Ê gˆ Êaˆ Ê bˆ fi = =
= 4 sin Á p - ˜ sin Á ˜ sin Á ˜ 1 1 2
Ë 2 ¯ Ë 2 ¯ Ë 2¯
fi a : b : c = 1 :1 : 2
Êg ˆ Êaˆ Ê bˆ
= 4 sin Á ˜ sin Á ˜ sin Á ˜ Hence, the result.
Ë 2¯ Ë 2¯ Ë 2¯
Properties of Triangles 429

16. Ans. (a, d)


From sine rule,
a
=
b

( 3 +1 ) =b= c
sin A sin B Ê 3 + 1ˆ 1 1
b sin A = a sin B Á 2 2 ˜ 2 2
Ë ¯
b sin A = a sin B £ a , since sin B £ 1 fi 2 2 = 2b = 2c \
p p
In this case, B = , A < fi b = 2, c = 2
2 2
p Thus, the area of the given triangle
Also b sin A < a, if 0 < B < p , B π
2 1
= ¥ bc ¥ sin A
p p 2
If A < , B > A, B π ,
2 2
=
1
¥ 2 ¥2¥
3 +1 ( )
p 2 2 2
then b sin A < a , A < ,b>a
2
17. Ans. (a, d) =
( 3 +1) sq cm.
2
Let –A = a - q , –B = q , –C = p + q
19. We have, BC = 2BD, AD = h, OD = h – r, so
since –A + –B + –C = p , q = 60∞
that BC = 2 r - (h - r ) = 2 2rh - h
2 2 2
Thus, the largest angle of DABC is a and the smallest
angle is C Thus, P = 2 AB + BC
Let x be the smallest side of the triangle .
x 2 + 102 - 92 fi 2AB = P - BC
Therefore, cos (60∞) =

fi x + 19 = 10 x
2
2.x.10
fi 2 AB = 2 ( 2hr - h 2
+ 2hr - 2 2hr - h 2 )
fi AB = 2hr
fi x 2 - 10 x + 19 = 0
The area of the DABC = A
fi ( x - 5)2
=6 = BD ¥ AD

( x - 5) = ± = h 2hr - h
2
fi 6
A h 2hr - h 2
fi x =5± 6 Now, =
P3
( )
3
8 2hr - h 2 + 2hr
18. Let –B = 30∞, –C = 45∞ and a = ( )
3 +1
2r - h
Then –A = (p - (–B + –C )) =
( )
3
8 2r - h + 2r
= (p - (30∞ + 45∞)) = 105∞
Ê Aˆ 2r 1
From sine rule, Thus, lim Á 3 ˜ = =
hÆ 0 Ë P ¯
( ) 128 r
3
8 ¥ 2 2r
a b c
= =
sin A sin B sin C 20. Ans. (c)
The given equation is k = 3 sin x - 4 sin x = sin 3x
3
a b c
fi = =
sin(105∞) sin(30∞) sin(45∞) Thus, k = sin 3 A, k = sin 3B


( 3 +1 )= b
=
c fi sin 3 A = sin 3B = sin (p - 3B )
sin(105∞) sin(30∞) sin(45∞) fi 3 A = (p - 3B )
430 Comprehensive Trigonometry with Challenging Problems & Solutions for Jee Main and Advanced

fi 3( A + B) = p a 2 + 4a a+4
p fi cosq = =
fi ( A + B) = 2a (1 + a ) 2 (a + 1)
3
a +1 a+4
p 2p Therefore, =
Therefore, –C = p - ( A + B ) = p - = 2 ( a - 1) 2 ( a + 1)
3 3

21. Given A, B and C are in A.P. fi (a + 1)2 = (a + 4) (a - 1)


fi 2B = A + C fi a 2 + 2a + 1 = a 2 + 3a - 4
fi 3B = A + B + C = p fi A=5
p
fi B= Hence, the sides of the triangle are 4, 5, 6.
3
23. Let a = BC, b = CA, c = AB and p = AD.
1 Ê 5p ˆ
Now, sin ( 2 A + B ) = = sin Á ˜ 1 1
DABC = ap = bc sin A
2 Ë 6¯ 2 2
5p ˆ
fi (2 A + B ) = ÊÁË ˜ fi p=
bc
sin A
6¯ a
Ê 5p ˆ 5p p 3p p abc
fi 2A = Á - B˜ = - = = fi p= sin A
Ë 6 ¯ 6 3 6 2 a2

fi A=
p
4 fi p=
(
abc sin 2 B - sin 2 C ) sin A
1 Êpˆ
a 2
(sin 2
B - sin 2
C)
Also, sin (C - A) = = sin Á ˜
2 Ë 6¯ abc sin ( B + C ) sin ( B - C )
fi p= sin A

p
(C - A) = ÊÁË ˆ˜¯ (b 2
)
- c 2 sin 2 A

abc sin ( B - C )
6
fi p=

Êp ˆ Ê p p ˆ 5p
C = Á + A˜ = Á + ˜ =
Ë6 ¯ Ë 6 4 ¯ 12
= 75∞ (b 2
- c2 )
ab 2 r sin ( B - C )
fi p= , where c = br
Thus, A = 45∞, B = 60∞, C = 75∞ .
(b 2
- b2 r 2 )
22. Let the sides of a triangle are a – 1, a, a + 1, Ê ar ˆ
fi p=Á sin ( B - C )
where a ΠI - {1}
+ Ë 1 - r 2 ˜¯

Ê ar ˆ
Let q is the smallest angle and 2 q is the greatest fi p£Á , since sin ( B - C ) £ 1
Ë 1 - r 2 ˜¯
angle of the triangle.
By the sine rule, 24.
sin q sin ( 2q )
=
a -1 a +1
C
a + 1 sin (2q ) r3
fi = = 2 cos q r3
a -1 sin q P
r1 R
a +1 O
r2
fi cosq = A
2 ( a - 1) r1
Q
r1 B
Again by the cosine rule,
(a + 1)2 + a 2 - (a - 1)2
cos q =
2.a. ( a + 1)
Properties of Triangles 431

Consider three circles with centres at A, B and C with 1 1 1


radii r1 , r2 , r3 , respectively, which touch each other fi = +
Êpˆ p
Ê ˆ
2 Ê 3p ˆ
externally at P, Q, R. 2r sin Á ˜ 2r sin Á ˜ 2r sin Á ˜
Ë n¯ Ë n¯ Ë n¯
Let the common tangents at P, Q, R meet each other
at O. 1
=
1
+
1

Then OP = OQ = QR = 4 Êpˆ Ê 2p ˆ Ê 3p ˆ
sin Á ˜ sin Á ˜ sin Á ˜
Also, OP ^ AB, OQ ^ AC , OR ^ BC Ë n¯ Ë n¯ Ë n¯
Here, O is the in-centre of the triangle ABC
1 1 1 Êpˆ
For DABC , fi = + , Á ˜ =q
sin q sin 2q sin 3q Ë n ¯
s=
(r1 + r2 ) + (r3 + r2 ) + (r1 + r3 ) = r + r2 + r3 1 1 1
2
1 fi - =
sin q sin 3q sin 2q
and D = (r1 + r2 + r3 ) r1r2 r3 sin 3q - sin q 1
fi =
D sin q sin 3q sin 2q
Now, from the relation r = , we get,
s 2 cos 2q sin q 1
(r1 + r2 + r3 ) r1r2 r3 fi =
=4 sin q sin 3q sin 2q
r1 + r2 + r3
fi 2 sin 2q cos 2q = sin 3q
r1r2 r3
fi =4 fi sin ( 4q ) = sin (3q )
r1 + r2 + r3
fi sin ( 4q ) = sin (p - 3q )
r1r2 r3 16
fi = 16 = fi (4q ) = (p - 3q )
r1 + r2 + r3 1
p
fi (r1r2 r3 ) : (r1 + r2 + r3 ) = 16 : 1 fi q=
7
25. We have p p
2 cos A 2 cos B 2 cos C 1 b fi =
+ + = + n 7
a b c bc ca
fi n=7
2bc cos A ac cos B 2bc cos C a 2 b2
fi + + = + 27. (i ) fi (ii )
abc abc abc abc abc
fi 2bc cos A + ac cos B + 2bc cos C = a 2 + b 2 Suppose a, b, c and area D are rational.
a+b+c
fi ( ) (
1
b2 + c2 - a 2 + a 2 + c2 - b2
2
) Thus, s =
2
= rational

(
+ a 2 + b2 -c )=a
2 2
+ b2 Ê Bˆ
Since tan Á ˜ =
D
Ë 2 ¯ s (s - b)
fi 2b 2 - 2a 2 + c 2 + a 2 - b 2 = 0
Ê Cˆ D
fi a 2 = b2 + c2 and tan Á ˜ =
Ë 2 ¯ s (s - c)
DABC is a right angled triangle at A
and D, a, s, s - a, s - b all are rational.
Thus, –A = 90∞ . Ê Bˆ Ê Cˆ
Therefore a, tan Á ˜ , tan Á ˜ are rational.
26. Let O be the centre r be the radius of the circle Ë 2¯ Ë 2¯
passing through Ai , where i = 1, 2, 3, ....., n
(ii ) fi (iii)
2p
Here, –Ai OAi +1 = , where i = 1, 2, 3, ...., n Ê Bˆ Ê Cˆ
n Consider a, tan Á ˜ , tan Á ˜ are rational.
Ë 2¯ Ë 2¯
1 1 1
Now, + = 2 tan ( B / 2)
A1 A2 A1 A3 A1 A2 sin B = = rational
1 + tan 2 ( B / 2)
432 Comprehensive Trigonometry with Challenging Problems & Solutions for Jee Main and Advanced

2 tan (C / 2) fi ( B - C ) = (90∞)
and sin C = = rational
1 + tan (C / 2)
2
fi ( B - 23∞) = (90∞)
Ê Aˆ Ê p Ê B + Cˆˆ fi B = 113∞
Also, tan Á ˜ = tan Á - Á
Ë 2¯ Ë 2 Ë 2 ˜¯ ˜¯
29. As we know that, the largest angle is the opposite to
Ê B + Cˆ the largest side.
= cot Á Let a = BC = 3, b = CA = 5, c = AB = 7
Ë 2 ˜¯
32 + 52 - 7 2 15 1
Ê Bˆ Ê Cˆ Thus, cos C = =- =-
1 - tan Á ˜ tan Á ˜ 2.3.5 30 2
Ë 2¯ Ë 2¯
= = rational 1 Ê 2p ˆ
Ê Bˆ Ê Cˆ fi cos C = - = cos Á ˜
tan Á ˜ + tan Á ˜ Ë 3¯
Ë 2¯ Ë 2¯ 2
Thus, sin A = rational. 2p
fi C=
Hence, a, sin A, sin B, sin C are rational. 3
(iii) fi (i ) p p
30. Let –B = , –C = , –BAD = q , –CAD = j
Suppose a, sin A, sin B, sin C are rational 3 4
By the sine rules, By the sine rule,
a
=
b
=
c sin q sin (p /3)
sin A sin B sin C =
BD AD
sin B sin C
fi b = a. , c = a. 3 BD
sin A sin A fi sin q = .
2 AD
b, c = rational
sin j sin (p /4)
1 and =
Also, D = bc sin A = rational DC AD
2
1 DC
This completes the proof. fi sin j = .
28. Figure 2 AD
AD = b sin (23∞) sin –BAD sin q
Now, =
sin –CAD sin j
abc
fi = b sin (23∞) 3 BD AD
b - c2
2
= . . 2.
2 AD DC
a sin (23∞)
fi = BD 3
b -c
2 2
c = .
DC 2
a sin A
fi = 1 3 1
b -c
2 2
a = ¥ =
3 2 6
a2
fi sin A =
b2 - c2 31 Let a = 4k , b = 5k , c = 6k
sin A
2
1 15
fi sin A =
sin B - sin C
2 2 s= (a + b + c ) = k
2 2
fi sin A = sin 2 B - sin 2 C 15 7
s-a= k - 4k = k
fi sin A = sin ( B + C ) sin ( B - C ) 2 2
fi sin A = sin (p - A) sin ( B - C ) s-b=
15 5
k - 5k = k
2 2
fi sin A = sin ( A) sin ( B - C )
15 3
fi sin ( B - C ) = 1 = sin (90∞) s-c= k - 6k = k
2 2
Properties of Triangles 433

R abc s 33. Let ABC be an equilateral triangle


Now, = ¥ p
r 4D D Then –A = = –B = –C
3
abcs
= Therefore, tan A + tan B + tan C
4D 2
Êpˆ Êpˆ Êpˆ
abcs = tan Á ˜ + tan Á ˜ + tan Á ˜
= Ë 3¯ Ë 3¯ Ë 3¯
4s ( s - a ) ( s - b) ( s - c )
Êpˆ
abc = 3 tan Á ˜
= Ë 3¯
4 (s - a ) (s - b) (s - c)
=3 3
(4k )(5k )(6k )
= Conversely, let tan A + tan B + tan C = 3 3
Ê7 ˆÊ5 ˆÊ3 ˆ
4Á k˜ Á k˜ Á k˜
Ë2 ¯Ë2 ¯Ë2 ¯ Here, A + B + C = p

=
16 fi A+ B =p -C
7
fi tan ( A + B ) = tan (p - C ) = - tan C
p 3p
32. Here, B + C = p - = tan A + tan B
4 4 fi = - tan C
1 - tan A.tan B
Ê 3p ˆ
fi tan ( B + C ) = tan Á ˜ = -1
Ë 4¯ fi tan A + tan B = - tan C + tan A.tan B.tan C

tan B + tan C fi tan A + tan B + tan C = tan A.tan B.tan C


fi = -1
1 - tan B tan C Thus,

fi tan B + tan C = -1 + tan B tan C = -1 + p fi tan A + tan B + tan C = tan A.tan B.tan C = 3 3

Let tan B and tan C are the roots, then p


It is possible only when, A = B = C = .
3
x 2 - ( tan B + tan C ) x + tan B.tan C = 0
Thus, the triangle is equilateral.
fi x 2 - ( p - 1) x + p = 0 34. Ans. (b)
It has real roots. Here, D =
1 1 1
¥ p ¥ p1 = ¥ q ¥ p2 = ¥ r ¥ p3
2 2 2
So, D ≥ 0 2D 2D 2D
fi p= ,q = ,r=
fi ( p - 1)2 - 4 p ≥ 0 p1 p2 p3

fi p2 - 2 p + 1 - 4 p ≥ 0 From sine rule of a triangle,


sin P sin Q sin R
= =
fi p2 - 6 p + 1 ≥ 0 p q r

fi ( p - 3)2 - 8 ≥ 0 Given sin P,sin Q, sin R are in A.P.

( p - 3)2 - (2 ) fi p, q, r ΠA.P
2
fi 2 ≥0
2D 2D 2D
ΠA.P
fi ( p - 3 + 2 2 )( p - 3 - 2 2 ) ≥ 0 fi , ,
p1 p2 p3

p £ (3 - 2 2 ) , p ≥ (3 + 2 2 )
1 1 1
fi fi , , ΠA.P
p1 p2 p3
fi p Œ(-•, (3 - 2 2 )) » ((3 + 2 2 ) , • ) fi p1 , p2 , p3 Œ H .P
Thus, the altitudes are in H.P.
434 Comprehensive Trigonometry with Challenging Problems & Solutions for Jee Main and Advanced

35. A B C
F
fi + + =p
2 2 2
A A B C
fi + =p -
2 2 2
Ê A Bˆ Ê Cˆ
I fi tan Á + ˜ = tan Á p - ˜
O E Ë 2 2¯ Ë 2¯
Ê Aˆ Ê Bˆ
C tan Á ˜ + tan Á ˜
B Ë 2¯ Ë 2¯ Ê Cˆ
fi = cot Á ˜
Ê Aˆ Ê Bˆ Ë 2¯
D 1 - tan Á ˜ .tan Á ˜
Ë 2¯ Ë 2¯
Let O be the circumcentre and OF be Ê Aˆ Ê Bˆ
tan Á ˜ + tan Á ˜
perpendicular to AB. Ë 2¯ Ë 2¯ 1
fi =
Let I be the in-centre and IE perpendicular to AC. A
Ê ˆ B
Ê ˆ Ê Cˆ
1 - tan Á ˜ .tan Á ˜ tan Á ˜
Ë 2¯ Ë 2¯ Ë 2¯
Then –OAF = 90∞ - C
–OAI = –IAF - –OAF Ê Aˆ ÊCˆ Ê Bˆ ÊCˆ
fi tan Á ˜ tan Á ˜ + tan Á ˜ tan Á ˜
Ë 2¯ Ë 2¯ Ë 2¯ Ë 2¯
A
= - (90∞ - C ) Ê Aˆ Ê Bˆ
2 = 1 - tan Á ˜ . tan Á ˜
Ë 2¯ Ë 2¯
A A+ B+C
= +C - Ê Aˆ ÊCˆ Ê Bˆ ÊCˆ
2 2 fi tan Á ˜ tan Á ˜ + tan Á ˜ tan Á ˜
Ë 2¯ Ë 2¯ Ë 2¯ Ë 2¯
C-B
= Ê Aˆ Ê Bˆ
2 + tan Á ˜ . tan Á ˜ = 1
Ë 2¯ Ë 2¯
IE r
Also, AI = = Ê Aˆ Ê Bˆ Ê Cˆ
Ê Aˆ Ê Aˆ Dividing both sides by tan Á ˜ .tan Á ˜ .tan Á ˜ ,
sin Á ˜ sin Á ˜ Ë 2¯ Ë 2¯ Ë 2¯
Ë 2¯ Ë 2¯
Ê Bˆ Ê Cˆ we get,
= 4 R sin Á ˜ sin Á ˜ 1 1 1
Ë 2¯ Ë 2¯ fi + +
ÊCˆ ÊCˆ ÊCˆ
Ê Bˆ Ê Cˆ Ê Ê B Cˆˆ tan Á ˜ tan Á ˜ tan Á ˜
Ë 2¯ Ë 2¯ Ë 2¯
= 1 - 8 sin Á ˜ sin Á ˜ Á cos Á + ˜ ˜
Ë 2¯ Ë 2¯Ë Ë 2 2 ¯¯
1 1 1
Ê Bˆ Ê Cˆ Ê Aˆ = . .
= 1 - 8 sin Á ˜ sin Á ˜ sin Á ˜ ÊCˆ ÊCˆ ÊCˆ
Ë 2¯ Ë 2¯ Ë 2¯ tan Á ˜ tan Á ˜ tan Á ˜
Ë 2¯ Ë 2¯ Ë 2¯
Ê Bˆ Ê Cˆ Ê Aˆ Ê Aˆ Ê Bˆ ÊCˆ
fi OI = R 1 - 8 sin Á ˜ sin Á ˜ sin Á ˜ cot Á ˜ + cot Á ˜ + cot Á ˜
Ë 2¯ Ë 2¯ Ë 2¯ fi
Ë 2¯ Ë 2¯ Ë 2¯

Also, OI Ê Aˆ Ê Bˆ ÊCˆ
2
= cot Á ˜ cot Á ˜ cot Á ˜
Ê Aˆ Ê Bˆ Ê Cˆ Ë 2¯ Ë 2¯ Ë 2¯
= R 2 - 2 R ¥ 4 R sin Á ˜ sin Á ˜ sin Á ˜
Ë 2¯ Ë 2¯ Ë 2¯
37. Figure
= R - 2 Rr
2 Here, HE = JK = r1
But IE = r
fi OI = R 2 - 2 Rr So, IH = r - r1
Hence, the result. Ê p Aˆ
In a right triangle IHJ, –JIH = Á - ˜
36. We have, A + B + C = p Ë 2 2¯
Properties of Triangles 435

Ê p Aˆ r = a+b
fi tan Á - ˜ = 1
Ë 2 2 ¯ r - r1 40.
Ê Aˆ r 41. By the sine rule,
fi cot Á ˜ = 1 a b c
Ë 2 ¯ r - r1 = =
sin A sin B sin C
Ê Bˆ r Ê Cˆ r
Similarly, cot Á ˜ = 2 , cot Á ˜ = 3 . a b c
Ë 2 ¯ r - r2 Ë 2 ¯ r - r3 = =
sin A sin B sin (p - ( A + B ))
In a triangle ABC, we have a b c
= =
Ê Aˆ Ê Bˆ Ê Cˆ Ê Aˆ Ê Bˆ Ê Cˆ sin A sin B sin ( A + B )
cot Á ˜ + cot Á ˜ + cot Á ˜ = cot Á ˜ cot Á ˜ cot Á ˜
Ë 2¯ Ë 2¯ Ë 2¯ Ë 2¯ Ë 2¯ Ë 2¯
b. c and C,
r1 r r r r r if we know a, sinA, sin B.
fi + 2 + 3 = 1 . 2 . 3
r - r1 r - r2 r - r3 r - r1 r - r2 r - r3 A, B and C by using
the half angle formulae, if we know the values of a, b
r1 r r r1r2 r3 and c.
fi + 2 + 3 =
r - r1 r - r2 r - r3 ( r - r1 ) ( r - r2 ) ( r - r3 ) a b c
By using, = = = 2R
sin A sin B sin C
[IIT-JEE-2000]
Ê A- B + Cˆ –B, –C and the sides a, b and c,
38. We have, 2ac sin Á ˜¯
Ë 2 if we know a, sinB and R
Ê A + C Bˆ –B, –C and the sides b and c,
= 2ac sin Á - ˜
Ë 2 2¯ if we just know a,sin A, R , since
Ê p B Bˆ a b c
= 2ac sin Á - - ˜ = = = 2 R gives
Ë 2 2 2¯ sin A sin B sin C

Êp ˆ b c
= 2ac sin Á - B˜ & from which we cannot obtain
Ë2 ¯ sin B sin C

= 2ac cos B b, c, –B & –C .


Ê a 2 + c2 - b2 ˆ 42. Let the angles of the triangle ABC are 4q , q and q
= 2ac Á ˜
Ë 2ac ¯
Also 4q + q + q = 180∞
(
= a +c -b
2 2 2
) fi 6q = 180∞
39. Ans. (a) 180∞
As ABC be a right angled triangle, circum radius of fi q= = 30∞
6
this triangle is half of its hypotemuse.
1 2 By sine rules,
Thus, R = a + b2 a b c
2 = =
sin A sin B sin C
Ê Cˆ Êpˆ
Also, r = ( s - c ) tan Á ˜ = ( s - c ) tan Á ˜ = ( s - c ) a b c
Ë 2¯ Ë 4¯ fi = =
sin(120∞) sin(30∞) sin(30∞)
Now, 2 ( R + r )
a b c
fi = =
sin (60∞) sin (30∞) sin (30∞)
= a 2 + b 2 + 2s - 2c
a b c
= c + 2s - 2c fi = =
3 1 1
= 2s - c
2 2 2
= a+b+c-c
436 Comprehensive Trigonometry with Challenging Problems & Solutions for Jee Main and Advanced

a b c
( 3l )
2
+ ( 2l ) - ( l )
2 2
fi = = 6l 2 3
3 1 1 cos A = = =
Hence, the required ratio is
2. ( 3l ).(2l ) 4 3l 2
2

a 3 p
= = fi A=
a+b+c 2+ 3 6
p p
Similarly, B = ,C =
1 Ê 2p ˆ 3 2
I n = n. . (OA1 ) . (OA1 ) sin Á ˜
2 Ë n¯ A : B : C = 30∞ : 60∞ : 90∞ = 1 : 2 : 3

p 45.
Ê 2p ˆ
= sin Á ˜ A
2 Ë n¯

Êpˆ
B1 B2 = 2 ( B1 L ) = 2 (OL ) tan Á ˜
Ë n¯

Êpˆ
= 2.1.tan Á ˜
Ë n¯ P Q
R
Êpˆ 30°
= 2 tan Á ˜
Ë n¯ B D E C

Ê1 ˆ Êpˆ BD
Thus, On = n Á ( B1 B2 ) (OL )˜ = n tan Á ˜ We have, = cot (30∞) = 3
Ë2 ¯ Ë n¯ PD
In BD = 3 PD
Now,
On
(n / 2) sin (2q ) p DE = PQ = PR + RQ = 2
= , where q =
n tan q n
BC = BD + DE + EC = 3 + 2 + 3 = 2 ( 3 +1 )
2 tan q1
= . Area of DABC
(
1 + tan q 2 tan q
2
) 3
¥ ( BC )
2
=
= cos q
2
4

=
1
2
(
2 cos 2 q ) =
4
3
¥4 ( 3 +1 )
2

1
(1 + cos 2q ) ( 3 + 1)
2
= = 3
2

1Ê Ê 2I ˆ

= 3 (3 + 1 + 2 3 )
= Á1 + 1 - Á n ˜ ˜
2 ÁË Ë n ¯ ˜¯
= 3 (4 + 2 3 )

On Ê = (6 + 4 3 )

Ê 2I ˆ
In = Á1 + 1 - Á n ˜ ˜
2 ÁË Ë n ¯ ˜
¯ b-c
46. We have,
44. Ans. (d) a
Let the sides of the triangle be l , 3l , 2l sin B - sin C
=
By the cosine rule, sin A
Properties of Triangles 437

Ê B + Cˆ Ê B - Cˆ 3
Ë 2 ˜¯
2 cos Á
Ë 2 ˜¯
sin Á = ¥ l2
= 4
Ê Aˆ Ê Aˆ
2 sin Á ˜ cos Á ˜ 3
( )
2
Ë 2¯ Ë 2¯ = ¥4 2+ 3
4
Ê p Aˆ Ê B - Cˆ
=
cos Á - ˜ sin Á
Ë 2 2¯ Ë 2 ˜¯ = (
3 7+4 3 )
Ê Aˆ Ê Aˆ
sin Á ˜ cos Á ˜
Ë 2¯ Ë 2¯
(
= 12 + 7 3 )
48. Ans. (a, b, c)
Ê Aˆ Ê B - Cˆ Ê B - Cˆ
sin Á ˜ sin Á
Ë 2 ˜¯
sin Á
Ë 2¯ Ë 2 ˜¯ A
= =
Ê Aˆ Ê Aˆ Ê Aˆ
sin Á ˜ cos Á ˜ cos Á ˜
Ë 2¯ Ë 2¯ Ë 2¯
E
Ê B - Cˆ B
Ë 2 ˜¯
sin Á D C
b-c
Thus, =
a Ê Aˆ
cos Á ˜
Ë 2¯ F

Aˆ Ê B - Cˆ
(b - c ) cos ÊÁË ˜ = a sin ÁË ˜
Let AD = p
2¯ 2 ¯
ar ( DABC ) = ar ( DABD ) + ( DADC )
47. We have, from sine rule,
1 1 Ê Aˆ 1 Ê Aˆ
a b c bc sin A = bp sin Á ˜ + cp sin Á ˜
= = 2 2 Ë 2¯ 2 Ë 2¯
sin A sin B sin C
Ê Aˆ
a b c fi bc sin A = p (b + c ) sin Á ˜
fi = = Ë 2¯
sin(120∞) sin(30∞) sin(30∞)
Ê Aˆ Ê Aˆ Ê Aˆ
a b c fi bc 2 sin Á ˜ cos Á ˜ = p (b + c ) sin Á ˜
fi = = Ë 2¯ Ë 2¯ Ë 2¯
3 1 1
2 2 2 2 bc Ê Aˆ
fi p= cos Á ˜
a b c (b + c ) Ë 2 ¯
fi = = = l ( say )
3 1 1 2 bc Ê Aˆ
AD = cos Á ˜
D bc sin A bc sin A (b + c ) Ë 2 ¯
Now, r = = =
s 2s (a + b + c ) Also,
AD Ê Aˆ
= cos Á ˜
AE Ë 2¯
Ê 3ˆ 2
Á 2 ˜l Ê A ˆ 2bc 2
Ë ¯ AE = AD sec Á ˜ = =
fi 3= Ë 2¯ b + c 1 1
( 3 +1+1 l ) +
b c
Ê 3ˆ Thus, AE is the H.M. of b and c
fi (
3 2+ 3 =Á ˜l
Ë 2 ¯
) Again,
DE Ê A ˆ FD
= sin Á ˜ ,
Ê Aˆ
= sin Á ˜
AD Ë ¯
2 AD Ë 2¯
fi l = 2 2+ 3 ( ) Ê Aˆ
EF = DE + FD = 2 AD sin Á ˜
Thus, the area of the DABC Ë 2¯
1 4bc Ê Aˆ Ê Aˆ
= .bc.sin A = cos Á ˜ sin Á ˜
2 b+c Ë 2¯ Ë 2¯
1
= .l.l.sin(120∞)
2
438 Comprehensive Trigonometry with Challenging Problems & Solutions for Jee Main and Advanced

2bc QS + SR
= ¥ sin A Also, > QS ¥ SR
b+c 2

49. Ans. (b) 1 2


fi >
Given AB CD , CD = 2 AB . QS ¥ SR QR
Let AB = a, CD = 2a 1 1 4
fi + >
and the radius of the circle be r. PS ST QR
Let the circle touches at P, BC at Q, AD at R 2 Ê Aˆ
and CD at S. 51. We have, cos B + cos C = 4 sin Á ˜
Ë 2¯
Then AR = AP = r, BP = BQ = a – r,
DR = DS = r and CQ = CS = 2a – r Ê B + Cˆ Ê B - Cˆ Ê Aˆ
fi 2 cos Á = 4 sin 2 Á ˜
Ë 2 ˜¯
cos Á
In triangle BEC, BC = BE + EC
2 2 2 Ë 2 ˜¯ Ë 2¯

fi ( a - r + 2 a - r ) 2 = ( 2 r )2 + a 2 fi
Ê p Aˆ Ê B - Cˆ
cos Á - ˜ cos Á
Ê Aˆ
= 2 sin 2 Á ˜
Ë 2 2¯ Ë ˜
¯ Ë 2¯
2
fi (3a - 2r )2 = (2r )2 + a 2
Ê Aˆ Ê B - Cˆ Ê Aˆ
fi 9a 2 + 4r 2 - 12ar = 4r 2 + a 2 fi sin Á ˜ cos Á ˜ = 2 sin 2 Á ˜
Ë 2¯ Ë 2 ¯ Ë 2¯
3
fi a= r Ê B - Cˆ Ê Aˆ
2 fi cos Á = 2 sin Á ˜
Ë 2 ˜¯ Ë 2¯
Also, ar (Quad. ABCD ) = 18
Ê B - Cˆ Êp B + Cˆ
ar (Quad. ABED ) + ar ( DBCE ) = 18 fi cos Á = 2 sin Á -
fi Ë 2 ˜¯ Ë2 2 ¯
˜
1
fi a.2r + .a.2r = 18 Ê B - Cˆ Ê B + Cˆ
2 fi cos Á = 2 cos Á
Ë 2 ˜¯ Ë 2 ˜¯
fi 3ar = 18
3 Ê B - Cˆ
Ë 2 ˜¯ 2
cos Á
fi 3¥ ¥ r 2 = 18
2 fi =
Ê B + Cˆ 1
Ë 2 ˜¯
fi r =42 cos Á

fi r=2
Ê B - Cˆ Ê B + Cˆ
+ cos Á
Ë 2 ˜¯ Ë 2 ˜¯ 2 + 1
cos Á
Thus, the radius is r = 2. =

50. Ans. (b, d) Ê B - Cˆ Ê B + Cˆ 2 -1
- ÁË ˜
Ë 2 ˜¯
cos Á cos
P 2 ¯
Ê Bˆ Ê Cˆ
2 cos Á ˜ cos Á ˜
Ë 2¯ Ë 2¯
R
fi =3
O Ê Bˆ Ê Cˆ
2 sin Á ˜ sin Á ˜
Q S Ë 2¯ Ë 2¯

T Ê Bˆ Ê Cˆ
fi cot Á ˜ cot Á ˜ = 3
Ë 2¯ Ë 2¯
Here, PS ¥ ST = QS ¥ SR
s (s - b) s (s - c)
Now, A.M > G.M fi . =3
1 1
(s - a) (s - c) (s - a ) (s - b)
+
PS ST > 1 1 s
fi . fi =3
2 PS ST s-a
1 1 2 fi 3s - 3a = s
fi + >
PS ST QS ¥ SR fi 2s = 3a
fi a + b + c = 3a
Properties of Triangles 439

fi b + c = 2a = 100 + 36 + 60
Hence, the result. = 196.
Êpˆ Êpˆ fi C = 14
52. We have 2 cos Á ˜ + 2 cos Á ˜ = 3 + 1
Ë 2k ¯ Ë k¯
Now, 2s = a + b + c = 6 + 10 + 14 = 30
Êpˆ Êpˆ 3 +1
fi cos Á ˜ + cos Á ˜ = fi s = 15
Ë 2k ¯ Ë k¯ 2
D 15 3
Êqˆ 3 +1 p Therefore, r = = = 3
fi cos Á ˜ + cos (q ) = , where =q s 15
Ë 2¯ 2 k
Êqˆ Êqˆ 3 +1 fi r2 = 3
fi 2 cos 2 Á ˜ - 1 + cos Á ˜ =
Ë 2¯ Ë 2¯ 2 53. Ans. (b)
We have a + b - c
2 2 2
Êqˆ Êqˆ 3 +1 3+3
fi 2 cos 2 Á ˜ cos Á ˜ = +1 =
Ë 2¯ Ë 2¯
( ) ( )
2 2 2 2
= x + x + 1 + x - 1 - ( 2 x + 1)
2 2 2

3+3 Êqˆ
fi 2t 2 + t - = 0 , where t = cos Á ˜ = x + x + 1 + 2x + 2x + 2 + x - 2x + 1
4 2 3 2 4 2
2 Ë 2¯

fi t=
-1 ± 1 + 4 3 + 3( ) = -1 ± (2 )
3 +1
-4 x 2 - 4 x - 1

= 2 x + 2 x - 3x - 2 x + 1
4 3 2
4 4

fi t=
-2 - 2 3 3
,
(
= ( x - 1) ( x + 1) 2 x + 2 x - 1
2
)
4 2
Êpˆ a +b -c
2 2 2

3 Now, cos Á ˜ =
fi t= Ë 6¯ 2ab
2
Êqˆ 3 Êpˆ fi
3
=
( )(
x2 - 1 2 x2 + 2 x - 1 )
fi cos Á ˜ =
Ë 2¯ 2
= cos Á ˜
Ë 6¯ 2 ( )(
2 x2 + x + 1 x2 - 1 )

Êqˆ Êp ˆ
ÁË ˜¯ = ÁË ˜¯ fi 3 ( x + x + 1) = 2 (2 x + 2 x - 1)
2 2
2 6
Êpˆ
q =Á ˜
fi (2 - 3 ) x + (2 - 3 ) x - (1 + 3 ) = 0
2


x = - ( 2 + 3 ) , ( 3 + 1)
Ë 3¯

p Êpˆ
fi =Á ˜ fi k = 3
k Ë 3¯ Thus, x = ( 3 + 1) .

1 54. Ans. (c)


53. We have, ar ( DABC ) = ab sin C 2 sin P - sin 2 P
2 We have
2 sin P + sin 2 P
1
fi 15 3 = .6.10.sin C 2 sin P - 2 sin P cos P
2 =
2 sin P + 2 sin P cos P
3 Ê 2p ˆ
fi sin C = = sin Á ˜ , 1 - cos P
2 Ë 3¯ =
1 + cos P
since C is obtuse.
Ê 2p ˆ 2 sin 2 ( P / 2)
fi C =Á ˜ =
Ë 3¯ 2 cos 2 ( P / 2)

2 Ê Pˆ
Also, c = a + b - 2ab cos C
2 2 2
= tan Á ˜
Ë 2¯
Ê 2p ˆ
= 10 + 6 - 2.10.6.cos Á ˜
2 2
Ë 3¯
440 Comprehensive Trigonometry with Challenging Problems & Solutions for Jee Main and Advanced

(s - b) (s - c) fi a 2 + b 2 - c 2 = - ab
=
s (s - a) a 2 + b2 - c2 ab 1
fi =- =-
2ab 2ab 2
(s - b) (s - c)
2 2
1
= fi cos C = -
s (s - a ) (s - b) ( s - c) 2

=
((s - b) (s - c))2 fi C = 120∞
D2 abc D
Now, R= ,r =
4D s
2
ÊÊ 7ˆ Ê 5ˆ ˆ
ÁË ÁË 4 - 2 ˜¯ ÁË 4 - 2 ˜¯ ˜¯ Thus,
r
=
4D 2
= , where s = 4 R s ( abc )
D2
2
Ê 1 Ê 3ˆ ˆ
2 Ê1 ˆ
4 Á ab sin 120∞˜
ÁË 2 ÁË 2 ˜¯ ˜¯ Ë2 ¯
=
= x+c
D2 . y.c
2
2
Ê 3 ˆ Ê1
2
=Á 3ˆ
Ë 4 D ˜¯ 4 Á . y. ˜
Ë2 2 ¯
=
57. Ans. (b) x+c
. y.c
Given a + b = x , ab = y 2

x2 - c2 = y 3y
Also, =
2 ( x + c) c
fi (a + b)2 - c 2 = ab
CHAPTER 8
The Heights and Distances

8.1 INTRODUCTION 8.3 BEARING OF A LINE


In this chapter we shall study how to measure the height The bearing of a horizontal line, i.e. a line in a horizontal
of the object and distance the points with the help of plane is the positive acute angle made by this line with the
trigonometric relations. north - south line in the same horizontal plane.
If a line is said to bear 20∞ west of north, we mean that
8.2 ANGLE OF ELEVATION, ANGLE OF it is inclined to the north direction at angle of 20∞, this angle
is measured from the north towards the west.
DEPRESSION AND THE LINE OF N A
SIGHT D
15°
40°
P

ht
f sig
W E
eo
Lin O

Angle of elevation 70°


O A C 35°

B
Let the point O is the observer and the point P is the object S
under observation.
The line OP is called the line of sight of the point P. Here, the bearings of the lines OA, OB, OC and OD are,
Let OA be the horizontal line in the same vertical plane respectively, N 40∞ E , S 35∞, S 75∞W and N15∞W
with OP. N
The acute angle –AOP, between the line of sight and P (NE)
the horizontal line known as the angle of elevation.
O A 45° Q (ENE)
45°
Angle of depression
22°
Lin W E
eo O
fs
igh
t

The acute angle –AOP, between the line of sight and S


the horizontal line known as the angle of depression, where North East means equally inclined to north and east.
the object P below the horizontal line OA. South - East means equally inclined to south and east. E-N-E
means equally inclined to east and north east.
442 Comprehensive Trigonometry with Challenging Problems & Solutions for Jee Main and Advanced

8.4 SOME SOLVED EXAMPLES Let the angle of elevation of the sun be q.
Then it shadow on the ground OA = BQ
Ex-1. A chimney of 20 m high standing vertically on the
Given OA = 2 3 = BQ , PQ = 6
-1 Ê 1 ˆ
top of a building, subtends an angle of tan Á ˜
Ë 6¯ 2 3 1
So, tanq = =
at a distance of 70 m from the foot of the building. 6 3
Find the height of the building. Êpˆ
Soln. Let QR be the chimney, PQ be the vertical tower fi tan q = tan Á ˜
Ë 6¯
and O be the point of observation.
1 p
We have OP = 70, QR = 20 and tanq = fi q=
6 6
R Hence, the angle of elevation is 30∞.
Ex-3. The angle of elevation of the top of a tower at any
p
Q point on the ground is and after moving 20 m
6
q p
towards the tower it becomes .
a 3
O P
Find the height of the tower.
Let –POQ = a and –QOR = q and PQ = h Soln. Let PQ be the vertical tower of height h and A and
h + 20 B are the point of observations.
Then tan (q + a ) = We have AB = 20 m
70
P
tan q + tan a h + 20
fi =
1 - tan q tan a 70
1 h h
+
fi 6 70 = h + 20
1 h p /6 p /3
70
1- .
6 70 A B Q

70 + 6h h + 20 Let BQ = x m
fi =
420 - h 70
Êpˆ h
2 In DPBQ, tan Á ˜ =
fi 4900 + 420h = 420h - h + 8400 - 20h Ë 3¯ x
fi h 2 + 20h = 3500 fi h=x 3 ......(i)
fi (h + 10)2 = 3500 + 100 = 3600 Êpˆ h
In DPAQ, tan Á ˜ =
fi h + 10 = 60 Ë 6 ¯ 20 + x
fi h = 50 1 h
fi =
Hence, the height of the building is 50 m. 3 20 + x
Ex-2. If a flag stuff on 6 m high, placed on the top of a 20 + x = h 3

tower costs a shadow of 2 3 m along the ground,
then find the angle of elevation of the sun. fi 20 + x = x 3( ) 3
Soln. Let PQ be the flag staff placed on the top fi 20 + x = 3x
of a vertcal tower QR.
R fi 2 x = 20
fi x = 10
B Q Hence, the height of the vertical tower = 10 3 .
Ex-4. When the suns altitude increases from 30∞ to 60∞ ,
q q the length of the shadow of a tower decrease by 5 m.
O A
Find the height of the tower.
R
The Heights and Distances 443

Soln. Let PR be the height of the tower when the angle of p


elevation is (x – 5), the length of the shadow is QR We have QR =10 and –MRQ =
4
= x (say) Ê p ˆ QM
In DMRQ, tan Á ˜ =
Ë 4¯ 10
QM
P fi 1=
10
fi QM = 10

h Ê p ˆ 10
Also, cos Á ˜ =
Ë 4 ¯ RM
30° 60°
1 10
Q 5 S x- 5 R fi =
2 RM
When the angle of elevation is 60°, then the length
of the shadow is (x – 5). fi RM = 10 2
h fi PM = 10 2
In DPRQ, tan (30∞) =
x Thus, PQ = QM + PM
1 h
fi =
3 x
= 10 + 10 2 = 10 ( 2 +1 )
Ex-6. A person standing on the bank of a river observes that
fi x=h 3 ......(i)
the angle subtended by a tree on the opposite bank
h p
In DPRS, tan (60∞) = is , when he retreats back 40 m from the bank he
x-5 3
p
h find that the angle to be . Then find the breadth
fi 3= of the river. 6
x-5
h Soln. Let PQ be the tree of height h and M, N be the point
fi 3= of observations.
h 3-5
We have MN = 40 m
fi 3h - 5 3 = h P
fi 2h = 5 3
5 3
fi h=
2
5 3
Hence, the height of the tower is .
2 p /6 p /3
Ex-5. A tree is broken by wind, its upper part touches the
ground at a point 10 m from the foot of the tree and M 40 m N Q
p
makes an angle with the ground. Find the whole Let the breadth of the river be QN = x
4
length of the tree. Êpˆ PQ h
Soln. Let PQ be the whole length of the tree and PM its In DPQM, tan Á ˜ = =
Ë 6 ¯ x + 40 x + 40
broken part, which touches the ground at R
P 1 h
fi =
3 x + 40
x + 40
M fi h= .......(i)
3
Êpˆ h
In DPQN, tan Á ˜ =
Ë 3¯ x
p /4 h
fi 3=
R 10 Q x
444 Comprehensive Trigonometry with Challenging Problems & Solutions for Jee Main and Advanced

h=x 3 Hence, the height of the hill


fi .......(ii)
= ( x + h)
From (i) and (ii), we get,
x + 40 h cot b
x 3= = +h
3 cot a - cot b

fi 3x = x + 40 h cot a
=
fi 2 x = 40 (cot a - cot b )
fi x = 20 Ex-8. A ladder rests against a vertical wall an angle a to
the horizontal. Its foot is pulled away from the wall
Hence, the breadth of the river is 20 m. through a distance ‘a’ so that it slides a distance ‘b’
Ex-7. Let the angle of elevation of the top of a hill observed down the wall making an angle b with the horizontal.
from the top and bottom of a building of height h Êa + bˆ
Prove that a = b tan Á .
are a and b, respectively. Then find the height of the Ë 2 ˜¯
hill.
Soln. Let AB be the hill, B being its top and CD be the Soln. Let AC represents the ladder of length l
building of height h. After being pulled through a distance ‘a’ away from
Thus, CD = h and consider BM = x the wall, the new position of the ladder is A’C ‘.
B A

a A¢
D M

h
h
b a
C b
A
C C¢ B
x
In DBDM, tan (a ) = Clearly, AC = A ' C ' = l and CC ' = a , AA ' = b
DM
x – ACB = a and –A ' C ' B = b
fi DM = = x cot (a ) .......(i)
tan (a ) Thus, a = BC '- BC
x+h = l cos b - l cos a = l (cos b - cos a )
In DBCA, tan (b ) =
AC and b = AB - A ' B '
x+h = l sin a - l sin b = l (sin a - sin b )
fi AC =
tan (b )
a l (cos b - cos a )
x+h Therefore, =
fi DM = .......(ii) b l (sin a - sin b )
tan (b )
a (cos b - cos a )
From (i) and (ii), we get, fi =
b (sin a - sin b )
x x+h
= Ê b +aˆ Êa - bˆ
tan (a ) tan (b ) 2 sin Á sin Á
a Ë 2 ˜¯ Ë 2 ˜¯
x x h fi =
fi - = b Êa + bˆ Êa - bˆ
tan (a ) tan b tan ( b ) 2 cos Á sin Á
Ë 2 ˜¯ Ë 2 ˜¯
Ê 1 1 ˆ h a Êa + bˆ
fi xÁ - ˜ = fi = tan Á
Ë tan (a ) tan b ¯ tan ( b ) b Ë 2 ˜¯

fi x (cot a - cot b ) = h cot b fi


Êa + bˆ
a = b tan Á
Ë 2 ˜¯
h cot b
fi x=
(cot a - cot b ) Hence, the result.
The Heights and Distances 445

Ex-9. At the foot of a mountain the elevation of its summit b cot a - a cot b
is 45°, after ascending 1 km towards the mountain cot (q ) =
b-a
upon an incline of 30°, the elevation changes to 60°.
Find the height of the mountain. Soln. Let A and B be two stations due south of the
Soln. Let C be the foot and A be the top of the mountain. tower OP which leans towards north.
and AB = h P
A

h
60°
D E
1 mk
b a q
30° N
45° S B b A a C x Q
C F B
Let AC = a, BC = b and CQ = x
3 3
CF = CD cos (30∞) = 1. = km Clearly, tan a =
h
2 2 a+x
1 1
DF = CD sin (30∞) = 1. = km cot a =
a+x
2 2 fi .......(i)
h
AB h
In DABC, tan (45∞) = = and tan (b ) =
h
BC BC b+x
fi BC = h b+x
fi cot (b ) = .......(ii)
3 h
Now, DE = BF = BC – CF = h - .
2 From (i) and (ii), we get,
1 b cot (a ) - a cot ( b )
and AE = AB – BE = AB – DF = h -
2
ab + bx ab + ax (b - a ) x
= - =
AE h h h
From DAED, tan (60∞) =
DE x b cot a - a cot b
1 fi =
h- h (b - a )
fi 3= 2
3 b cot a - a cot b
h- fi cot (q ) =
2 (b - a )
3 1 Hence, the result.
fi 3h- = h- Ex-11. From an aeroplane vertically over a straight road the
2 2
angles of depression of two consecutive kilometer
3 1
fi ( )
3 -1 h = - = 1
2 2
stones on the same side are 45° and 60°. Find the
height of the aeroplane.

fi h=
1
=
( 3 +1 ) Soln. Let B and C be two consecutive kilometer stones.
Then BC = 1 km and A be the position of the plane
( 3 -1 ) 2 at a certain time.
Ê 3 +1 ˆ
Hence, the height of the mountain is Á ˜ km.
( ) X
45°
A

60°
ÁË 2 ˜
¯
Ex-10. Two stations due south of a tower, which leans h
towards north are at a distances a and b from its
foot. If a and b are the elevations of the top of the 45° 60°
tower from these stations, prove that its inclination
B C D
q to the horizontal is given by
446 Comprehensive Trigonometry with Challenging Problems & Solutions for Jee Main and Advanced

Let AD = h and CD = x
h =
20 ( 3 -1) m / sec
In DABD, tan(45∞) = 1
1+ x
fi x +1 = h = 20 ( )
3 - 1 m / sec
fi x = h -1 ......(i)
LEVEL I
h (PROBLEMS BASED ON FUNDAMENTALS)
In DACD, tan(60∞) =
x
1. A tower subtends an angle q at a point A in the plane
h of its base. The angle of depression of the foot of the
fi = 3
x tower at a point ‘h’ m just above ‘A’ is a. Find the
h height of the tower.
fi = 3
h -1 2. The angle of elevation of the top of an incomplete
vertical pillar at a horizontal distance of 100 m from
fi ( )
3 -1 h = 3 p
its base is . If the angle of elevation of the top of
4
fi h=
3
=
3 ( 3 +1) the complete pillar at the same point is to be
p
such
3
( 3 -1 ) 2
that the height of the pillar is increased by h m, then
Ê 3 + 3ˆ find h.
Hence, the height of the plane is Á ˜ km. 3. A person walking along a straight road observes that
Ë 2 ¯ at two points 1000 m apart, then angle of elevation of
Ex-12. A bird is perched on the top of a tree 20 m high and p 5p
a vertical tower in front of him are and . Find
its elevation from a point on the ground is 45°. It flies 6 12
the height of the tower.
off horizontally straight away from the observer and
4. A man in a boat rowing uniformly away from a cliff
in one second the elevation of the bird is reduced to
150 m high takes 2 minutes to change the angle of
30°. Find its speed.
p p
Soln. Let the bird alight at B, the top of the tree BD and O elevation of the top of the hill from to . Find the
be the observer, where BD = 20 m speed of the boat. 3 4
B M 5. A flagstaff of 5 m high stands on a building of 25 m
high. The flagstaff and the building subtends equal
angles at a point P, 30 m high above the ground. Find
the distance of P from the top of flagstaff.
20 m
6. AB is a vertical tower ‘A’ being its foot standing on a
horizontal ground. ‘C’ is the mid-point of AB. Portion
45° CB subtends an angle q at the point P on the ground.
30°
O D N If AP = 2AB, then find tan(q).
Thus, BD = MN = 20 m 7. At the foot of a mountain the elevation of its peak is
p
20 found to be , after ascending 10 m toward the
In DBOD, tan(45∞) = 4
OD p
mountain up a slope of inclination, the elevation
fi OD = 20 p 6
is found to be . Find the height of the mountain.
MN 20 3
In DMON, tan(30∞) = =
ON 20 + DN 8. A man finds that at a point due south of a vertical
20 1 p
fi = tower the angle of elevation of the tower is . He
20 + DN 3 3
then walks due west 10 6 m on the horizontal plane
fi 20 + DN = 20 3 p
and find the angle of elevation of the tower to be .
fi DN = 20 ( 3 -1 ) 6
Find the original distance of the man from the tower.
distance
Thus, speed = 9. The angle of elevation of the top of vertical tower
time from a point A on the horizontal ground is found to
The Heights and Distances 447

p 3 +1 3 -1
be . From ‘A’ a man walks 10 m up a path sloping (c) (d)
4 3 -1 3 +1
at an angle p/6. After this the slope becomes steeper
and after walking up another 10 m, the man reaches
the top of the tower. Find the distance of ‘A’ from the LEVEL III
foot of the tower. (PROBLEMS FOR JEE MAIN)
10. A vertical tower erected at the focus of the parabola 1. A man on a cliff observes a ship at an angle of de-
p pression 30° approaching the shore just beneath him.
y 2 = 40 x subtends an angle at the vertex of the
3 Three minutes later the angle of depression of the ship
p
parabola. If the tower subtends an angle at a point is 60°. How soon will it reach the shore?
P lying on the parabola. 6 2. A vertical tower subtends an angle of 60° at a point on
Then find the possible co-ordinates of point P. the same level as the foot of the tower. On moving 100
m further from the first point in line with the tower, it
LEVEL II subtends an angle of 30° at the point. Find the height
(MIXED PROBLEMS) of the tower.
3. Find the height of a tower when it is found that on
1. On level ground the angle of elevation of the top of
walking 80 m towards it along a horizontal line through
the tower is 30°. On moving 20 meters near then the
its base, the angular elevation of its top changes from
angle of elevation is 60°. The height of the tower is
30° and 60°.
(a) 20 3 m (b) 10 3 m 4. A vertical pole on one side of a street subtends a right
(c) 10 ( )
3 -1 m (d) None
angle at a window exactly on the opposite side. If the
angle of elevation of the window from the base of the
pole be 60° and the width of the street be 30 m, find
2. From the top of a light house 60 meters high with its
the heights of the window and top of the pole.
base at sea level, the angle of depression is 15°. The
5. An object is observed from three points A, B, C lying
distance of the boat from the foot of the light house is
in a horizontal straight line which passes directly un-
Ê 3 - 1ˆ Ê 3 + 1ˆ derneath the object. The angular elevation at B is twice
(a) 60 ¥ Á ˜m (b) 60 ¥ Á ˜m
Ë 3 + 1¯ Ë 3 - 1¯ that at A and at C three times at A. If AB = a, BC = b,
find the height of the object.
Ê 3 + 1ˆ 6. A man notices two objects in a striaght line due west of
(c) Á ˜m (d) None
Ë 3 - 1¯ him. After walking a distance c due north he observes
that the objects subtend an angle a at his eye and after
3. Three poles whose feet A, B, C lie on a circle subtend
walking a further distance c due north, an angle b. Find
angles a, b, g and respectively, at the centre of the
the distance between the objects.
circle. If the height of the poles are in A.P, then
7. The angle of elevation of an aeroplane from a point 200
cot (a ) , cot (b ) , cot (g ) are in
meters above a lake is 45° and the angle of depression
(a) A.P (b) G.P of its replection is 75°. Find the height of the aeroplane
(c) H.P (d) None above the surface of the lake.
4. A ladder 20 ft long reaches a point 20 ft below the top 8. From the bottom of a pole of height h, the angle of
of a flag. The angle of elevation of the top of the flag elevation of the top of a tower is a. The pole subtends
at the foot of the ladder is 60°. Then the height of the an angle b at the top of the tower. Find the height of
flag is the tower.
(a) 25 ft (b) 30 ft 9. The angle of elevation of a cloud from a point x feet
(c) 35 ft (d) 40 ft above a lake is q and the angle of depression of its
5. From an aeroplane vertically over a straight horizontal reflection in the lake is j. Find its height.
road, the angles of depression of two consecutive mile- 10. A train is moving at a constant speed at an angle q East
stones on opposite sides of the aeroplane are observed of North. Observations of the train are made from a fixed
to 45° and 60°. Then the height in miles of aeroplane point. It is due north at some instant. Ten minutes earlier
above the road is its bearing was a West of North, where as 10 minutes
3 3 afterwards its bearing is b East of North. Find tan(q).
(a) (b)
3 +1 3 -1
448 Comprehensive Trigonometry with Challenging Problems & Solutions for Jee Main and Advanced

ANSWERS
LEVEL I LEVEL III
1. h tan q cot a 3
1. min
2. h = 100 ( 3 -1 m ) 2
2. 50 3 m
3. 250 ( 3 +1 )
3. 40 3 m
(
4. 25 3 - 3 m/ min ) 4. 30 3 m , 40 3 m
3
5. 5 ¥ a (a + b) (3b - a )
2 5.
2b
6. 2/9
3c
7. 5 ( 3 +1 ) 6.
2 cot b - cot a

8. 5 3 m 7. 200 3 m
8. h sin a cos (a - b ) cos ec b
9. 5 ( 3 +1 )
x sin (j + q )
10. ( 20 , 20 2 ) 9.
sin (j - q )

LEVEL II 2sin a sin b


10.
1. (b) 2. (b) 3. (c) 4. (b) 5. (a) sin (a - b )

You might also like